{ "cells": [ { "cell_type": "markdown", "metadata": {}, "source": [ "# 3. Linear Regression – Applied\n", "\n", "Excercises from **Chapter 3** of [An Introduction to Statistical Learning](http://www-bcf.usc.edu/~gareth/ISL/) by Gareth James, Daniela Witten, Trevor Hastie and Robert Tibshirani.\n", "\n", "I've elected to use Python instead of R." ] }, { "cell_type": "code", "execution_count": 1, "metadata": {}, "outputs": [], "source": [ "import numpy as np\n", "import pandas as pd\n", "import seaborn as sns; sns.set()\n", "import matplotlib.pyplot as plt\n", "from mpl_toolkits.mplot3d import Axes3D\n", "import statsmodels.api as sm\n", "import statsmodels.formula.api as smf\n", "import patsy\n", "from scipy import stats\n", "from sklearn import datasets\n", "from IPython.display import display, HTML" ] }, { "cell_type": "markdown", "metadata": {}, "source": [ "### Q8. This question involves the use of simple linear regression on the Auto data set." ] }, { "cell_type": "code", "execution_count": 2, "metadata": {}, "outputs": [ { "data": { "text/html": [ "
\n", "\n", "\n", " \n", " \n", " \n", " \n", " \n", " \n", " \n", " \n", " \n", " \n", " \n", " \n", " \n", " \n", " \n", " \n", " \n", " \n", " \n", " \n", " \n", " \n", " \n", " \n", " \n", " \n", " \n", " \n", " \n", " \n", " \n", " \n", " \n", " \n", " \n", " \n", " \n", " \n", " \n", " \n", " \n", " \n", " \n", " \n", " \n", " \n", " \n", " \n", " \n", " \n", " \n", " \n", " \n", " \n", " \n", " \n", " \n", " \n", " \n", " \n", " \n", " \n", " \n", " \n", " \n", " \n", " \n", " \n", " \n", " \n", " \n", " \n", " \n", " \n", " \n", " \n", " \n", " \n", " \n", " \n", " \n", " \n", "
indexmpgcylindersdisplacementhorsepowerweightaccelerationyearoriginname
0018.08307.0130350412.0701chevrolet chevelle malibu
1115.08350.0165369311.5701buick skylark 320
2218.08318.0150343611.0701plymouth satellite
3316.08304.0150343312.0701amc rebel sst
4417.08302.0140344910.5701ford torino
\n", "
" ], "text/plain": [ " index mpg cylinders displacement horsepower weight acceleration \\\n", "0 0 18.0 8 307.0 130 3504 12.0 \n", "1 1 15.0 8 350.0 165 3693 11.5 \n", "2 2 18.0 8 318.0 150 3436 11.0 \n", "3 3 16.0 8 304.0 150 3433 12.0 \n", "4 4 17.0 8 302.0 140 3449 10.5 \n", "\n", " year origin name \n", "0 70 1 chevrolet chevelle malibu \n", "1 70 1 buick skylark 320 \n", "2 70 1 plymouth satellite \n", "3 70 1 amc rebel sst \n", "4 70 1 ford torino " ] }, "execution_count": 2, "metadata": {}, "output_type": "execute_result" } ], "source": [ "auto_df = pd.read_csv('./data/auto.csv')\n", "\n", "# Remove missing values\n", "auto_df = auto_df.drop(auto_df[auto_df.values == '?'].index)\n", "auto_df = auto_df.reset_index()\n", "auto_df.head()" ] }, { "cell_type": "code", "execution_count": 3, "metadata": {}, "outputs": [], "source": [ "# Convert quantitive datatypes to numerics\n", "datatypes = {'quant': ['mpg', 'cylinders', 'displacement', 'horsepower', 'weight', 'acceleration', 'year'],\n", " 'qual': ['origin', 'name']}\n", "\n", "quants = auto_df[datatypes['quant']].astype(np.float_)\n", "\n", "auto_df = pd.concat([quants, auto_df[datatypes['qual']]], axis=1) " ] }, { "cell_type": "markdown", "metadata": {}, "source": [ "#### **(a)** Use the lm() function to perform a simple linear regression with mpg as the response and horsepower as the predictor. Use the summary() function to print the results. Comment on the output. For example:\n", "\n", "- i. Is there a relationship between the predictor and the response?\n", "- ii. How strong is the relationship between the predictor and the response?\n", "- iii. Is the relationship between the predictor and the response positive or negative?\n", "- iv. What is the predicted mpg associated with a horsepower of 98? What are the associated 95 % confidence and prediction intervals?" ] }, { "cell_type": "code", "execution_count": 4, "metadata": {}, "outputs": [], "source": [ "# Let's start by implementing linear regression from scratch\n", "# using numpy linear algebra\n", "\n", "intercept_const = pd.DataFrame({'intercept': np.ones(auto_df.shape[0])})\n", "\n", "X = pd.concat([intercept_const, auto_df['horsepower']], axis=1)\n", "y = auto_df['mpg']\n", "\n", "def linear_model(X, y):\n", " \"\"\"Estimation of parameteres for linear regressions model\n", " by Ordinary Least Squares (OLS)\"\"\"\n", " XTX_inv = np.linalg.inv(X.T @ X)\n", " XTy = X.T @ y\n", " beta = XTX_inv @ XTy\n", " return beta\n", "\n", "def predict(beta, X):\n", " \"\"\"Perdict y given beta parameters and X variables\"\"\"\n", " return X @ beta\n", "\n", "beta = linear_model(X, y)\n", "y_pred = predict(beta, X)" ] }, { "cell_type": "code", "execution_count": 5, "metadata": {}, "outputs": [ { "data": { "text/html": [ "
\n", "\n", "\n", " \n", " \n", " \n", " \n", " \n", " \n", " \n", " \n", " \n", " \n", " \n", " \n", " \n", " \n", " \n", " \n", " \n", " \n", " \n", " \n", " \n", " \n", " \n", " \n", " \n", " \n", " \n", " \n", " \n", " \n", " \n", " \n", "
coefficientsstandard_errort-statisticP>|t|
feature
intercept39.9358610.71566655.8023760.0
horsepower-0.1578450.006429-24.5518480.0
\n", "
" ], "text/plain": [ " coefficients standard_error t-statistic P>|t|\n", "feature \n", "intercept 39.935861 0.715666 55.802376 0.0\n", "horsepower -0.157845 0.006429 -24.551848 0.0" ] }, "execution_count": 5, "metadata": {}, "output_type": "execute_result" } ], "source": [ "# Our handrolled linear model doesn't provide the useful statistics \n", "# that summary(lm()) would provide in R. \n", "# For each estimated coefficient we would like to get the follow stats\n", "# - coefficient \n", "# - standard error\n", "# - t-value\n", "# - p-value\n", "\n", "# Add constant for bias variable\n", "intercept_const = pd.DataFrame({'intercept': np.ones(auto_df.shape[0])})\n", "#Load data\n", "X = pd.concat([intercept_const, auto_df['horsepower']], axis=1)\n", "y = auto_df['mpg']\n", "\n", "# Predict coefficients and responses\n", "coefficients = linear_model(X, y)\n", "y_pred = predict(beta, X)\n", "\n", "\n", "# Calculate Mean Squared Error\n", "MSE = np.sum(np.square(y_pred - y)) / y.size\n", "\n", "# Variance of each variable in X\n", "variance = MSE * (np.linalg.inv(X.T @ X).diagonal()) # To discuss: how is variance derived from MSE?\n", "\n", "# Standard error of each variable in X\n", "# given by Var(X) = SE(X)^2\n", "standard_error = np.sqrt(variance)\n", "\n", "# t-statistic given by t = β - 0 / SE(β)\n", "t_statistic = coefficients / standard_error\n", "\n", "# p-values\n", "p_values = 2*(1 - stats.t.cdf(X.shape[0], np.abs(t_statistic)))\n", "\n", "\n", "# Present results\n", "results = pd.DataFrame({'feature': X.columns,\n", " 'coefficients': coefficients,\n", " 'standard_error': standard_error,\n", " 't-statistic': t_statistic,\n", " 'P>|t|': p_values})\n", "\n", "results.set_index('feature')" ] }, { "cell_type": "code", "execution_count": 6, "metadata": {}, "outputs": [ { "name": "stdout", "output_type": "stream", "text": [ " OLS Regression Results \n", "==============================================================================\n", "Dep. Variable: mpg R-squared: 0.606\n", "Model: OLS Adj. R-squared: 0.605\n", "Method: Least Squares F-statistic: 599.7\n", "Date: Wed, 26 Sep 2018 Prob (F-statistic): 7.03e-81\n", "Time: 17:22:54 Log-Likelihood: -1178.7\n", "No. Observations: 392 AIC: 2361.\n", "Df Residuals: 390 BIC: 2369.\n", "Df Model: 1 \n", "Covariance Type: nonrobust \n", "==============================================================================\n", " coef std err t P>|t| [0.025 0.975]\n", "------------------------------------------------------------------------------\n", "const 39.9359 0.717 55.660 0.000 38.525 41.347\n", "horsepower -0.1578 0.006 -24.489 0.000 -0.171 -0.145\n", "==============================================================================\n", "Omnibus: 16.432 Durbin-Watson: 0.920\n", "Prob(Omnibus): 0.000 Jarque-Bera (JB): 17.305\n", "Skew: 0.492 Prob(JB): 0.000175\n", "Kurtosis: 3.299 Cond. No. 322.\n", "==============================================================================\n", "\n", "Warnings:\n", "[1] Standard Errors assume that the covariance matrix of the errors is correctly specified.\n" ] } ], "source": [ "# The statsmodels library provides a convenient means to get the\n", "# same statistics\n", "\n", "X = auto_df['horsepower']\n", "X = sm.add_constant(X) # add bias constant\n", "y = auto_df['mpg']\n", "\n", "results = sm.OLS(y, X).fit()\n", "print(results.summary())" ] }, { "cell_type": "markdown", "metadata": {}, "source": [ "- i. Is there a relationship between the predictor and the response?\n", "\n", "> Yes, the low P-value associated with the t-statistic for horsepower suggests so.\n", "\n", "- ii. How strong is the relationship between the predictor and the response?\n", "\n", "> For a unit increase in horsepower, our model predicts mpg will decrease by -0.1578. So for example, increasing horsepower by 10 is expected to decrease efficiency by -1.578 mpg. \n", "\n", "- iii. Is the relationship between the predictor and the response positive or negative?\n", "\n", "> Negative\n", "\n", "- iv. What is the predicted mpg associated with a horsepower of 98? What are the associated 95 % confidence and prediction intervals?" ] }, { "cell_type": "code", "execution_count": 7, "metadata": {}, "outputs": [ { "name": "stdout", "output_type": "stream", "text": [ "24.467 mpg\n" ] } ], "source": [ "# predicted mpg associated with a horsepower of 98\n", "\n", "def predict(model, X):\n", " return model.T @ X\n", "\n", "X_ex = np.array([1, 98])\n", "\n", "y_ex = predict(coefficients, X_ex)\n", "print(str(np.round(y_ex, 3)) + ' mpg')" ] }, { "cell_type": "code", "execution_count": 8, "metadata": {}, "outputs": [ { "name": "stdout", "output_type": "stream", "text": [ "[21.814544959722646, 27.119609345302205]\n" ] } ], "source": [ "# the associated 95% confidence and prediction intervals\n", "\n", "model_min = results.conf_int(alpha=0.05)[0]\n", "model_max = results.conf_int(alpha=0.05)[1]\n", "\n", "confidence_interval = [predict(model_min, X_ex), predict(model_max, X_ex)]\n", "print(confidence_interval)" ] }, { "cell_type": "markdown", "metadata": {}, "source": [ "#### **(b)** Plot the response and the predictor. Use the abline() function to display the least squares regression line." ] }, { "cell_type": "code", "execution_count": 9, "metadata": {}, "outputs": [ { "data": { "image/png": "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\n", "text/plain": [ "
" ] }, "metadata": {}, "output_type": "display_data" } ], "source": [ "# Let's plot our predicted regression\n", "\n", "df = pd.concat([auto_df['horsepower'], auto_df['mpg']], axis=1)\n", "ax = sns.scatterplot(x='horsepower', y='mpg', data=df)\n", "ax.plot(auto_df['horsepower'], y_pred);" ] }, { "cell_type": "markdown", "metadata": {}, "source": [ "#### **(c)** Use the plot() function to produce diagnostic plots of the least squares regression fit. Comment on any problems you see with the fit." ] }, { "cell_type": "code", "execution_count": 10, "metadata": {}, "outputs": [], "source": [ "# Functions to emulate R's lm().plot() functionality\n", "# Providing powerful residual plots for simple AND multivariate\n", "# linear regresssion\n", "# - bring your own predictions\n", "# - underlying stats available as pandas dataframe\n", "# - visualise linearity and outliers in multiple dimensions\n", "\n", "\n", "def lm_stats(X, y, y_pred):\n", " \"\"\" LEVERAGE & STUDENTISED RESIDUALS\n", " - https://en.wikipedia.org/wiki/Studentized_residual#How_to_studentize\n", " \"\"\"\n", " # Responses as np array vector\n", " try: \n", " y.shape[1] == 1\n", " # take first dimension as vector\n", " y = y.iloc[:,0]\n", " except:\n", " pass\n", " y = np.array(y)\n", " \n", " # Residuals\n", " residuals = np.array(y - y_pred)\n", " \n", " # Hat matrix\n", " H = np.array(X @ np.linalg.inv(X.T @ X)) @ X.T\n", " \n", " # Leverage\n", " h_ii = H.diagonal()\n", " \n", " ## Externally studentised residual\n", " # In this case external studentisation is most appropriate \n", " # because we are looking for outliers.\n", " \n", " # Estimate variance (externalised)\n", " σi_est = []\n", " for i in range(X.shape[0]):\n", " # exclude ith observation from estimation of variance\n", " external_residuals = np.delete(residuals, i)\n", " σi_est += [np.sqrt((1 / (X.shape[0] - X.shape[1] - 1)) * np.sum(np.square(external_residuals)))]\n", " σi_est = np.array(σi_est)\n", " \n", " # Externally studentised residuals\n", " t = residuals / σi_est * np.sqrt(1 - h_ii)\n", " \n", "\n", " # Return dataframe\n", " return pd.DataFrame({'residual': residuals,\n", " 'leverage': h_ii, \n", " 'studentised_residual': t,\n", " 'y_pred': y_pred})\n", "\n", "\n", "def lm_plot(lm_stats_df): \n", " \"\"\"Provides R style residual plots based on results from lm_stat()\"\"\"\n", " # Parse stats\n", " t = lm_stats_df['studentised_residual']\n", " h_ii = lm_stats_df['leverage']\n", " y_pred = lm_stats_df['y_pred']\n", " \n", " # setup axis for grid\n", " plt.figure(1, figsize=(16, 18))\n", " \n", " # Studentised residual plot\n", " plt.subplot(321)\n", " ax = sns.regplot(x=y_pred, y=t, lowess=True)\n", " plt.xlabel('Fitted values')\n", " plt.ylabel('Studentised residuals')\n", " plt.title('Externally studentised residual plot', fontweight='bold')\n", " # Draw Hastie and Tibshirani's bounds for possible outliers\n", " ax.axhline(y=3, color='r', linestyle='dashed')\n", " ax.axhline(y=-3, color='r', linestyle='dashed');\n", " \n", " # Normal Q-Q plot\n", " plt.subplot(322)\n", " ax = stats.probplot(t, dist='norm', plot=plt)\n", " plt.ylabel('Studentised residuals')\n", " plt.title('Normal Q-Q', fontweight='bold')\n", " \n", " # Standardised residuals\n", " plt.subplot(323)\n", " ax = sns.regplot(x=y_pred, y=np.sqrt(np.abs(t)), lowess=True)\n", " plt.xlabel('Fitted values')\n", " plt.ylabel('√Standardized residuals')\n", " plt.title('Scale-Location', fontweight='bold')\n", " \n", " # Residuals vs Leverage plot\n", " plt.subplot(324)\n", " ax = sns.scatterplot(x=h_ii, y=t)\n", " plt.xlabel('Leverage')\n", " plt.ylabel('Studentised residuals')\n", " plt.title('Externally studentised residual vs Leverage', fontweight='bold');\n" ] }, { "cell_type": "code", "execution_count": 11, "metadata": {}, "outputs": [ { "data": { "image/png": "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\n", "text/plain": [ "
" ] }, "metadata": {}, "output_type": "display_data" } ], "source": [ "X = pd.concat([auto_df['horsepower']], axis=1)\n", "# Create the Design Matrix by adding constant bias variable\n", "intercept_const = pd.DataFrame({'intercept': np.ones(X.shape[0])})\n", "X = np.array(pd.concat([intercept_const, X], axis=1))\n", "\n", "y = auto_df['mpg']\n", "\n", "lm_plot(lm_stats(X, y, y_pred))" ] }, { "cell_type": "markdown", "metadata": {}, "source": [ "The above residual plot grid shows the relationship between the horsepower predictor and the mpg response. There are several things to note:\n", "\n", "- **Non-linearity of the data:** The top-left residual plot exhibits a discernable pattern, in this case u-shaped, that suggests our linear model is not providing a optimal fit to our data - the relationship is non-linear. A discernable pattern in this plot suggests that our model is failing to account for some of the reducible variance in the responses. There is still a discernable pattern in the bottom-left plot suggesting that a quadratic transform only improves the fit of our model slightly.\n", "- **Heteroscedasticity – Non-constant variance of error terms** The top-left residual plot exhibits a conical shape. This suggests that there is some heteroscedasticity in our predictor. The standardised plot (bottom-left) also exhibits this characteristic suggesting that standardisation doesn't alleviate the issue – to address this we might consider fitting our model by *weighted least squares*.\n", "- **Outliers and leverage:** the bottom-right residual vs leverage plot suggests that there are several potential outliers (points in top-right of axis) that could be having a strong effect (leverage) on our model. We should add more predictors to our model to clarify outliers.\n", "- The top-right plot shows that our studentised residuals have a slightly non-normal distribution (TODO: ellaborate)\n", "\n" ] }, { "cell_type": "code", "execution_count": 12, "metadata": {}, "outputs": [ { "data": { "image/png": "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\n", "text/plain": [ "
" ] }, "metadata": {}, "output_type": "display_data" } ], "source": [ "def lm_residual_corr_plot(lm_stats_df):\n", " r = lm_stats_df['residual']\n", " # Residuals correlation\n", " plt.figure(1, figsize=(16, 5))\n", " ax = sns.lineplot(x=list(range(r.shape[0])), y=r)\n", " plt.xlabel('Observation')\n", " plt.ylabel('Residual')\n", " plt.title('Correlation of error terms', fontweight='bold'); \n", "\n", "lm_residual_corr_plot(lm_stats(X, y, y_pred))" ] }, { "cell_type": "markdown", "metadata": {}, "source": [ "- **Correlation of error terms:** The Correlation of Error Terms plot shows errors against ordered observations in our dataset. We see a slight increase in error above the 300th observation suggesting some correlation effect. This could mean that our estimated standard errors underestimate the true standard errors. Our confidence and prediction intervals may be narrower than they should be." ] }, { "cell_type": "markdown", "metadata": {}, "source": [ "### 9. This question involves the use of multiple linear regression on the Auto data set.\n", "\n", "#### **(a)** Produce a scatterplot matrix which includes all of the variables in the data set." ] }, { "cell_type": "code", "execution_count": 13, "metadata": {}, "outputs": [ { "data": { "image/png": "iVBORw0KGgoAAAANSUhEUgAABaAAAAWYCAYAAABAiWEIAAAABHNCSVQICAgIfAhkiAAAAAlwSFlzAAALEgAACxIB0t1+/AAAADl0RVh0U29mdHdhcmUAbWF0cGxvdGxpYiB2ZXJzaW9uIDIuMi4yLCBodHRwOi8vbWF0cGxvdGxpYi5vcmcvhp/UCwAAIABJREFUeJzsvX18VOWZ//855z7nzEySAQZIIm4FH6oDRmiDJVEStQvKd+sTWH67hFoh1kAlbaHWbWvAb4WuEPpg+9Pdxa5ii9ituL+lLVLtd1HZtoKVUGW/KoWRRRS3SsLDQCYPM3Oefn9MzsnMnHPCTDIzmYfr/Xr11TCembnnnOt+uu7r+lycrusgCIIgCIIgCIIgCIIgCIIgiEzDj3YDCIIgCIIgCIIgCIIgCIIgiOKEHNAEQRAEQRAEQRAEQRAEQRBEViAHNEEQBEEQBEEQBEEQBEEQBJEVyAFNEARBEARBEARBEARBEARBZAVyQBMEQRAEQRAEQRAEQRAEQRBZgRzQBEEQBEEQBEEQBEEQBEEQRFYgBzRBEARBEARBEARBEARBEASRFcgBTRAEQRAEQRAEQRAEQRAEQWQFckATBEEQBEEQBEEQBEEQBEEQWYEc0ARBEARBEARBEARBEARBEERWKGYHtADg4oH/J4hCgmyXKFTIdolChWyXKETIbolChWyXKFTIdolChOyWyAuK2QA/AeDY6dM90DR9tNsyLHy+MgSDfaPdjLxitO9JZaWXy8HX5J3tjvZ9zyTF9FuA1H9Pqdpupig2uxkuub4PObJboMBtl+zTntG8L8Uy5pJtnZ9iu0fFYrtAfj+bfG1bIbcr32w3X+/lSCjG3wTQeiGeYn3G6VDq9yCb/oVijoAueASBjXYT8g66J6NDMd33YvotQPH9nnyF7nMMug/5CT0Xe+i+jBy6h+eH7lH+ks/PJl/bRu3KHIXY5vNRjL8JKN7fNRzoXtA9yObvJwc0QRAEQRAEQRAEQRAEQRAEkRXIAU0QBEEQBEEQBEEQBEEQBEFkBXJAEwRBEARBEARBEARBEARBEFmBHNAEQRAEQRAEQRAEQRAEQRBEViAHNEEQBEEQBEEQBEEQBEEQBJEVyAFNEIQjjPHQGY+uYB90xoMxGjKI7GDYmsJxZGtE0UB2TRQLZMsEkd9QHyWI9KG9LkEMkov+IGT8E0sE7xgP3K70bl84oiDU3Z+lFhFEZmGMR7BPxoYtHegK9qPK58Hq5jr4ykSoqjbazSOKCLI1ohghuyaKBbJlgshvqI8SRPpQvyGIQXLVH8gBPUzcLgG33b8jrffsfGQ+QllqD0FkGgUwByAA6Ar2Y8OWDrS3NoAb3aYRRQbZGlGMkF0TxQLZMkHkN9RHCSJ9qN8QxCC56g+UY0AQhC2qppsDkEFXsB+qpo9Si4hihWyNKEbIroligWyZIPIb6qMEkT7UbwhikFz1B3JAEwRhC+M5VPk8Ca9V+TxgPJ0JE5mFbI0oRsiuiWKBbJkg8hvqowSRPtRvCGKQXPUHckATBGGLAGB1c505EBk6QKTbQ2QasjWiGCG7JooFsmWCyG+ojxJE+lC/IYhBctUfqH8RBGGLqmrwlYlob20AOA7QdQgDrxNEJom3NVXTwXiObI0oeMiuiWKBbJkg8hvqowSRPrTXJYhBctUfyAFNEARBjDqqqkFgPMBzMa0pnoPAeFoEEgUN2TVRCDDGQwGGdFypqgYOAxsHVYea+2YSRF6TSj/KJtRHCYIgiHyHHNAEQdjCGI+zfTLWD1RDrfJ5sKa5DuPKRHKeEBmH7I0oBNJ1MDDGI9gnm1WljXQ2H9k1kSekYqOj7VgjiHynVMd6GhuIQob2HkQyxpjWFewDGF9SY1qu5jHSgCYIwhYVHH6x6zBa5k9He2sDWuZPxy92HYYKKsxAZB4VnLkA9E/2oWX+dIRlFQo4MEZTFTH6GAuztk17sbz9FTzx67cR0XSoHAed8bZ2qgDmQg6IVZPesKUDSo7bThBOnM9GNU037f6Rf30TxztDCKs6wBiNzQQxQCmO9clzYtumvQj2yRkZFxjjoTMeyhDzK0GMlPi9BxDrt+u3dGRkr0s2XHgwxqNXVnG8M4SuM3043hlCr6yWzLPL1TxGEdA5JCqrqKz0pvUe7xgPQt39WWoRQTijczpuu+4yPPbcAfMUbOWiWugcyAVNZBxF00zn8103T0uwu1KIIiLyn/iFmX+yD7dddxke/MlrQ9qpqunmQs6gK9gPVdNpAUbkBeez0XO9EWzY0gGf101jM0E4UIpjvZOzor21YUT7BKcovHHj9Iy0myAMjL1HPF3Bfigj7LelmhFR6OiMQ/BMBI9vf8t8bquaalFWWY5S0DTK1TxWrHNiXiKJDLfdvyOt9+x8ZD5CWWoPQQyJzpkbTSA2AD323AG0tzaOcsOIYoTnOVT5PFg453KL3WViQ0MQIyV+YZaqnbIBu45f0FX5PGA8B6i0mSZGn/PZqKzENugt86fT2EwQDpTiWJ8tZ4WTY/uHq64fwacShBXeod/yPEbkcMzW4QyRXRRVx6PbEtc5j247gA2tDSUhG5GreawU7iVBEMNA0+0XlppenAtpYnQRGIdvfOFqjK2QHDc0BDGaMJ5DfU01VjfXYXJ1RUp2KgBY3VyHKp8HAMwoGDr9J/KF89moKPCo8nngLROHtHlKNyZKmVIc6w1nRTymsyKV9zuMGU6ObVmhyFEisxh7j/h++40vXA2BjcxNPNThDJG/aA7PTSuRoSdX81gxz4sEQYyAIU/BSmQgJnIHA+AtF8Gh9KKIiMJAYhya5k1F+5YOtMyfnpKdqqoGX5mI9tYGKtJE5CVONgoAOuOhaTpWN9chGAo72jwDR+nGRElTamN9zHnMYXVznaXfCzh/8OhQEgU6YDvWiAIPRSmBPHjCkUwXvTT2HisWzoBbEhCOKvCWi2AY2Va3FDMiigGB8bbPTWAcilrQf4D4eQwcB+h6VuYxCk8gCMIWxnNY1VSbcAq2qqk25cgGgkgHWQXWPfk6Ht12ACsX1ZZUFBFRGMgq0D6wWd6++0jKdqqqGjhVg6Dr4FStaB0SROGSbKMAzOJizd/dhWd3HcbkC7yOkTGlWICNIJIplbHecB5/6x/34PHtb2HFwhn4lwfmYmNrY8qHTkONGU5ReGPLXdn7UUTek42il1FVx7onX8e6zfvQtmkv1m3eh3VPvo7oCJ3EpZgRUQyQ72NwHqvylWVtHqN+QBCELRFZxdYXDqFl/nR4y0SE+mRsfeEQ7r/zaho4iIxjFALpCvbjmRcH7a7SVwYRetFu5IjCIb5YTeB40LTTiyeNAeNR1NFuRGmR7Bzad7ATxz7qxve/1mgb4alyXMkVYCOIUiV+fOgK9mPd5n2o8nliY0OKc+BQEgWcrttGk/Ml5AQirGRDVzlbOuallhFRLJDvIzfQvSQIwhae5xAMhbFhS4f5WiYKMxCEHfGFQALHg9iwpQNVPg82pLGhIYhsklysJnA8iM073saG1oZYlMAot48gMoXTpjwqx6I7BQBQddPmKd2YIEqHTDjtzjdmqKoGDrCMNUTpkg1ncTbnLrLhwoN8H7mBJDgIgrBFYPZpKCMtzEAQdpC9EfkO2ShRKqRbXIzSjQmidBhp8UGAxgwifTJhd8mQHRLx0Do/N1D/IgjCFk7VUenzYO2ya8FzgKYDgsCBo2gmIgtwqg6f12UWAtF1oHqCB7KsgWc8pa4Ro06yjYajCnxe17DHxEwX0yGITGFsypMLhEmMQxS8VYKD0o0JYkjsxvtCxWl8SKX4oAGNGUS6ZMLuklFVDRO8EtpbG6HqOhjHQWKAHKVw11KEfB+5oZDnP4Igskx/v4L1cRP9muY6uMrE0W4WUYSoqoZykWFytRfggFCvjLZ/3mupjk6bE2K0iLfRkW6YjWI6yRspsnEiH7CrhC4xDqdDUUebpXRjgrDHabwfN64wnRqZch7TmEGkQzYOLRjjh5zXiNKDfB/ZhyQ4CIKwRQHMARiI6Wytp6r2RBYxKu9Cd66OThCjiWGjgq6PqDq0UzEdsnEiX0iuhB5VdbJZghgGTuP9ud7I6DZsBGRqLiSIdMi03dFajIiHfB+5gSKgCYKwRdV0+LzuhEqw23cfoar2RMZJTk0Fh6xUpSaIXOMks5GtyusEkTU4WNYDgeNBslmCOA9O472sxCKAcw3JPxFEDNrrEvHQ2nxwfugK9gFZksAslXtJEESaSCKPJbdMw6PbDphpKKuaaiGJPDTSxiIyhFNqan1NNfYd7DSvy1RVaoLIFUPJbOhA1iqvE0SmYYxHsFfG5h1vm7a8clEtdr56lGyWIM6DUTwtebwXBR6Kktv1NMk/EcQgtNcl4nEaq0tlnZOr+SEvJTj8fv8P/X7/loG/P+33+//k9/vf9fv9m/1+PznNCSIHaCrMCRmInQA+uu0ANJqPiQzilP52z+1XUVVqoqAZKrWTKq8ThYSdLT/23AHcc/tVZLMEcR6cxvux5a6ct4UkBwhiENrrEvGU+to8V/ND3t1Pv98/F8BSAC8MvPRzAC2BQOB1v9//FIBlAB4frfYRRKmgaJptGoqiafk3cBAFi1O6EweOqqMTBc1QqXycrme8mA5BZIuhxmmyWYIYGqfiaTyfewEOSjEniEFor0vEY1eAuZTW5rmaH/IqAtrv948HsB7AhoF/TwHgCQQCrw9csgXA345O6wiitDDSUOIx01AIIkM42RnPgwrcEAXN+cZQKuJEFApDjdMEQZyffBnvaW1PEINQfyCSSS7AXEpr81z1h3xbOv4LgDUAggP/vhDAx3H//WMAn8h1owiiFBEArElKQ1lTQmkoRG4gOyOKFadUPnCAzngwlm9LMIKwR2JcSaelEkShwhgPnfFQOA4646kvEwVNsj2PdB1V6pILBBFPrvbknK7nh6C23+9vAXBlIBD4ht/vbwbwWQBPAtgYCASuG7jmcgA7A4HA1BQ+8mIAx7LT2hi33b8jret3PjJ/WO8h8opcHIlejCzbbipomo7ToX6oig5d18FxHJjAYYLXMyppg8SIyTvb1TQdZ3siCEcVcAOpTqqqQxR5TBjjgSCQg47Iid0CGRh3NU3Hud4IZEWDKPDweiR090URlVXwPAeOA/7ll29h38FOVPk8ePBL9ZhywRgaT4uXvBtzDZJt1dCiTX6N5zlomo4PTnTjX//PIcydNQVjKySMH+PGxLE0RhcxeWu7pYTdnBLqj1r66FDv/+BENx7+6T6zoNSDX6rHRVXetD6nwCDbLVKc7Hmk6yi7+dDp8xRFQzAUhqLqEBgHn9edqXkwL+w2nXtBFB+apqPrbB9kWQPPAZoOiCKPqnFlQ9lB2gaSTwc8iwBM8vv9/wVgPIAKADqASXHXXADgo3Q+9PTpHmha5p3slZXejH+mEydPhnL2XflOZaV3VO9HLp97tmw3VQS3gLPdEWx8er850T+wdBYEnoMSLtxyJaNtQ5km1d+Tb7YbX2nX53VbqlDnW1X2YrOb4ZLr+5BLuwWGP+4mV46ur6lG07ypaI+rJL2qqRZnQ1EAMU21h3+6D+2tDeAyYONkn/aM5n3JtzHXwK7K+dpl10CWNay3qXzOiczc8O872AkgFhWTKdstBoqt/+Wr7Q6HfH42Q7XNrp+2Nddh267D5iHm+dZJOuPNvgtY5x0OgKKoOB2WU27XaJJKu/LNdvP1Xo6E0fpN57PnkVI18LuS+4OBKDGcCkUT1nVtzXWY6JUgR0dWtTAf7NZuzMm3vVguKca+ez54iaHrTF/CfnxVUy1ExkFzsPHh2G7ehC4EAoGbAoHAVYFA4NMAvgPg+UAgcDeAsN/vbxi47C4Avx21RhJECSHLuul8BmIT/can90OW8yNrgihs4ivtLpxzuaUKNVVlJwqJ5MrRc2dNMTcpwGBl9YVzLjffYxT2IIhcYlflvPNMn+l8Nl4zxmBZsS/SRLZLENnDrp+2b+nA3FlTzH+fb500VEEpgig0Rtueoyos67r2LR0Yoe85b7Abc2gvVlooqm7Zjz+67QAUNbN9LJ8ioJ24E8CTfr9/DIA3ATw2yu0hiJJAdagMTJWyiUwQv5D0lom2tqZpABuNxhFEmiRvjJxs2lsmmv82C3tkeGFHEENht4l3S4LzfM841NdUm9HPANkuQWQbJ2db/BxyvjW5UVCqK9gP/2QfFs65HGMrJDCeB4NeklGNROESb88GmZiLGOOhAOgK9gGMhwDY9g3nfbFWFPvioRz8xfD7iPOjOdiApmU2ajkv7SkQCGwBsGXg7/8LoG4020MQpQhjPJbNr0FdzSRTA7rj4MdgjEMhHoemusAgckP8QjLUJ6PK58GMT07EHZ+9HIw3dKc4MDBEVR2qpoPxXME/N8MOi+X3EDGSN0aaruOhlnq4JQGSxKPMJYJD7PnfOOsivPXfp8xCN3bBM2QnRLaw28SHo0rCa/7JPnzp9ivBcRwiURUt86fjwonl+POxIJrmXYELJ1aAAwfG+JzaJfULolCJt11J5KGpQ69HJZE355BQn4ztu48gGApD03Wsbq6Dt0xEOKrEPsshBNMosPbsrsO47brL8Nhz+StzRhDnw7DnZIkIp3VUKqQjO8F43sEBzgOqtQUuj4hwVIOqaWA8D7fEI9JvL++RDzCewx03XIob6y4G4wFVA17ueJ8Om0sIgfG2NiBk2PeTlw5ogiBGH4/E48pLJ2LN43sTtK48Eo+wUlj5RqRrlX/ELyS37z6C/91SD1nWsW7zHwftbWkdyssEPPj4a0Xx3MgOi5d4e/Z53RAFHo9uO4AZn5yImxsuwUNP/DFhHF1yyzTosmb73MlOiGxit4mvHl+GNc11WD9gv8s/fxXCETVh/l9zdx1uqp+CtU++Pip2Sf2CKFTSrXnBGI+zoSge3/5Wgg5npc+DM+ci2Lzj7YT3jvNK4FRrRLOqavCViVi+YAbaNu2xpNa3tzbkrMovQYwUw57bWxsydgjpJDth1zckBjy07Bp0nemDWxJiB7fjyyAxQE7aFrs8IrrOhi160VXj3HnrhHaJPK6feVHiPqy5Di6RR5Tm2JJAcrABSeQRzaDvJ280oAmCyC/6o5qt1lV/tPAmIdK1yj/iF5L33zkTZS4RG59OsrenO6AoetE8N7LD4kVVNXjLRbTMn47775yJH/78DXQF+3HHZy+3aOm3b+mALDunP5OdENkkfux9om0u2lsbUC4yjBt47Zt3XY3unqhFB3D9zzrQeaZv1OyS+gVRqKRb80IBLJrsj247AMbz+NEv3rC899hH3Qj2yWDMuq1XVW1IST2CKCRUVQOnahB0HZxqf4gPxA5xdMZD4TjojLftG0AsI8HndWN1cx3aWxtih0Fet23f0FQdiqLh8e1voW3TXjy+/S0oigbNJjo47LCHDufxHjoi67ZtjpRY7SfDdrqCfUPaTjESke3tNiJn1m5L544SBJEWquqwYC3ANJzRLlxB2GMsJD0i72hvfFIIgqFFlcrCMt8gOyxydGDzjrehqIPPmfFw1AwcakPkpMFWiHZP5B92m3jjNU3THTWh3ZJgeU1z2Jek6gBIuc00fhIFipZCzYt4O3a0dYd1klsSEpzYyX1PEmPSAfHU11SD8TSfEMWHkXHQtmkvlre/grZNex0PaCSRx/LPXwVRiG02RIHD8s9fBUm0XqsAWP+zROfc+p/ZH4IOpRedrxRimzNNvO3c8/BLQ9pOMZIr309p3E2CINKGMeuCtcrniWlAFxiG5mU8ZuEKYlRhjMfpUBS8wzNK9i1U+TzQoae0sMw3yA6LG0PaIP45qxpsnznPc9AdxlInOylUuycKC4HxpiZ0PFU+D8JRxfIabzN+peMASBUaP4lChecH19NGzYt4qnweCPxg33CydUXVbV8P9cmmE9uu750LRbF22TXme+trqtE0byraNu2h+YQoOtLJluHAIRxRE6KawxEVnI04zVAF2pJhvMMems/fPlaIbc40pZ5plSvfT+lYFEEQaeES+Zhe1cBAFK8FlUkyHSVlh+EYiv8tRuEKYnQxJnuO4/DA0lkJz+iBpXUQBM7y3H772jG0zJ+O9tYGtMyfjmd3HS6IxQHZYXFjSBu4JIZVTbWo8nnwq98dsbHrWXhhz3tQHCIKnOzkqeffKdlFMZE7eB4YUyGZNgzEbPD+O6/GmAop4bWVi2oBzmrHmd7EMcaDMY7GT6Iw4XSsXBTrT9t3H7H0rVVNteDZ4OV2c8DKRbX45X8esX19++4j5mGMXd9bv6UDblEwZXeWL5hhSbOO9U8OCschGAqTM5ooWNLJlpFVDc//4WjCnuL5PxyFbCPtwTs4aO0OYd0u+z2025W//UpisG2zxM7zxiIiHUmWYkQUeNs9iyhk1m5p3UYQhC19YQV/+vMJrF/RAF3XwXEcfvfGh7jh6osyNnDkqqhQvOYlOA7Q9REXriAyg7FQPNMdxn/88X081HKtWXn3V787gi/8zVS0tzZC03TwPMAYh6unXZBQzT3mBBntX3J+slFAhchHdPi8LqxYOANuScD4MS6sW34tOAza9cv7P8TnZl9qGwVgZyfggH0HOxOuMzZUtJAjhosoMUTVWOot43lIDOiXNTzxy3fwzbuuxne/PBuapuP0uX789PmDAIANrQ0IhiIIdkew89WjWL5guuVzh3IApGuvyQXcViycgQsnVkBkPBictdQJIh9gjIeic9j5aszJNbm6Aqe7w/ja330aAuMR6pOx9YVDuP/OmWbfUFUN47ySOYeE+mQ88+IhBENhfPFzU7GxtRGyquGjUz3m68ZhTMSh7ylaTHJHAKA4pNqfOtePb//THirwSeQtdnOWHE0sjmZkEMTbuJktk3Twz3HAbdddZtlTcJzNpmLgIMm6/7A6J3v7Fbz3P0G0tzaabf2vdztR7q7M2zWbHFVROdY10ObY2tMlcoiGSyfUQRJ5S5HYVU21kEQeWjRzRfjylXBEwYt7j1n24n93kz+jdpuvfYAYICqrqKz0pvWecERBqLv//BcSxBAwxuPCieXgOQ6qroPnOFw4sTw2gWdoPZpO9eGRoqoaOACVlV6cPBlC8U8j+Q9jPHTEForB7gje+u9TeHn/hwAA/2QfmuZdgaisgec5MJ6DqungOOCNQyfQMn86vGUiQn0ydr56FCsWfgoqQ947dg07FABA1ckOiwhRYjgdipqOsvsWz8S53gh6+1Ws2/xH+LxuLJxzOebOmoyGT10It4shKsfSOgV+0JnGGA8FMVsWeB48A6Kyhoda6rFt17sIHA8CcN5QEUQqiBLDqVDUjIQ0op3GlUsY55UQiWpmJXSD+ppqcBzgLZMwrsKF5QtmQNN18Iw3x1xjXP/eVxtxrieK7buPIHA8OGx7jV8ndAX7sW7zPlT5PLEDmjwc44niJ36MHmq9YRyeHHzvFJYtmA5FifWPP/35BH71+/fgn+zDwjmX40u314DxiQcqDMA4r9vSPxkATVVRJjFMrh6D+++cmeCIM1Kok51vPM9BR6yt+sBrydec64kCyO5anCCSSbU/Oc1ZE71SghPayCBIDm4SAOuaW+dMhzIQs/3HnjuA9tZGa0N1mAdJ8fsPu0NYxnOWmgluScjrNRtjPPqiKhRFB88BigqoOgcX40tmrtVUmBHxxjN+/g9H8eUFM0a7aTmBMR7eMjHhNW+ZmFHfD0AO6LxHEhluu39HWu/Z+ch8hLLUHqJ0KJd4XFjlRdumPQkTfbnEo0/JjNssk1FSRGFhbMye3XUYKxfVYuerR83IAp/XbZ5Ax/9t2OEDS+vw3EuHse9gJ6p8HnxryWdwtieCDT/LbiQ9QTjBGI9+WUtwlP342Tdx78IZ2PriQXxryWcQiaqmHdfXVGPRTVOx8elBm11zdx3GVkgIDjix46Mvtr4Qi3KL/9txQ0UQKRBVYUnD37brML7yt59G07yp2PriwYRor/qaaiy55UqcON2H5/9w1BI1trq5DhO8knkIEx8htvPVo1g8b+qw7JXWCUQ+kU7mngLg4HuncOWlE7F6095Ep9k4D6ZMGmvpQ8bnRFUd23YdTnCEbNt1GMsXTIcwUDvDrg2A1fm2qqkWP3jmDXPemOCVLNesXFSLZ148ZLad+hiRC9LpT3ZzVvuWDrS3NiYclKiqhgleKSH62C5SGgA0h2wATdORrDwhAFg8b2pKju0yF7PdQ5e5GPr78nNfwok8es5GEtalDyytg3ucKxYKWwpwum1EfKmcxHkkHtfPvMgMPjDs1iPxCGfI9wOQBjRBEA70RjXbib43mrlJiIoKlS5GVNu+g5145sVDuPu2q1DuFvBQy7W4/86ZpqNu4ZzLzb+BmB1ufLoDc2dNMf8d6o2azmfjNdLHJXKJAiAYiiRsZALHg+gLy/ibay/GmDIpwY7nzppiLvKBwWrqvf2qJSvk0W0HsHDO5ebf37zrarS3NtABCzEikive+yf7cNt1l+Ho/5xFe9zY3DJ/Or731UYsXzADXWf68Oi2A5g7a4olamzDlg5EVWtW02PPHcDyBTOGba+0TiDyibT0zTmg/qpJtmvpa6660LYPGZ+jajr2HezEhi0daNu011wvqZo+ZBviJZyeaLsRKxbOwNYXDiFwPBjXT/W4a+aivbURO189ambXANTHiNyQTn9KnrOM69WkKoBGcfPBIpt7cDoUtdU1dyqAbld3L7FvzR1yHdYXsd9D90Xyd80mK7plXbrx6Q7ISn5GbGcD3SEiXi+RW9DvYLf9GbZbOtgkCMIWVXWY6NXMRUSklSZFFA2M8aZOoZGCqmk6zvZEsG3X21jVVGvanrdMtLXD+BQhtyRQhBwxqqiajnM9UdTXVGPurClmxNqf/nwCjZ/+hMU57WTXjMeQ9m5E5gg6ybcQqWGX3gzAIpOxcM7leOy5A7hv8UzTBgPHg9iwpQMA8ETbXHOsdbJfZweBDhU6WJxUR6rQOoHIJ9KKyNc5KIqadl9xnUfGZqg2uOL6OwCs27zP9jpO1005MAYdi+dNxbGPuqmP5ZhU5SeKlXT6E+Pt5WUYzwPqoKWmI+8oAFi77Bp0numDWxIQjiqoHl82Ytsfylmer/uSXOz78x3H56aXxj0Yal4iDWiCILKOk44cYxwyFVpKRdkMZLW7AAAgAElEQVRKDyPdLhgKo76m2pLqtKqpFgIbLCAS6pNt7TDUJ5v/DkeVlAuOEEQ2YDyHQ8dOYdFNfmx8ej98Xjea5l2B/3XtJQhHFZS7XQk26mTXqmavzWnYe5XPAyFps0UQTtilN69ddg1kWcP6pPT7igGnspNtMp43x9qhrrF7/S8nQ6Z2c7rySLROIPKJdAqcaZrmOKY79RWB5xHsG1rGBg5tSH7vQy31KbU1uY+5XQJ0WaU+lmVyVYg9n0mnP0kMaGuus2hASwyQ45ZE6co2ybKGx7e/ZX7mmuY6QEwW4Ig9r15ZtTiry0VmeV6pOsvzCec2cyVzEiU4+D6EDGsg5yu58P0AJMFBEIQDZRKPtuY6MzXJ1K+SMjtsqKoGTo1V5+ZUrWQWXaWKEZmwbde7aL71Kkuq06PbDkASGb7xhatR5fNg++4jWNVUa7HDV/Z/YP7bWy6Z1xuvGdE7BJELBACfm32J6Xy+6+ZpeHz7W/jxL95EsDsMQeCwOm48fWX/B3hg6awEm71v8Uz86ndHsHJRor2vaqrF9t1HzL95676IIGyxiwTrPNNnOp+N1x577gC8Hskcc5NtcHVzHRgDqseXYVVTLV7Z/4HlmrbmOrjERDs3bHbbrnfN7xqOPBKtE4h8wYjIT2W9wfMcXu543zLWP7B0Fnb/6bhlbbO6uQ48g1nIdnVzHe5bPBOyomLFwk+ZjkmnNhjvNfr2tl3v2n6HXVvj+5jP66Y+lgPSknMpUtLpT3JUxcQBbWdDOia5ACEQc2rX11RjdXMd2lsbsLq5DvU11baSMgpgmQ/XOzwDnXHojyb+l/6oAp1ZP9fjst9De1z563pzOez7XRne9+czjOcsY+aqptqSkSMSBd52vhKFzNoA7c8JgrAlrGgYl1TEgWdAuIS0oIjMY0QmdAX70dMftWiQLpxzOVRNw5hyESsWzkBFmYSxFS60tzYiqqgIdkcwYawLc2dNwYIbPolQn4wnf/UOAGBDa0OscAhFyBE5RlU1cByHrmA/WuZPN4tp3nXztIQibg/fOxuqpqO3X8b4MTG7VvWYzYb6oqirmYQ3Dp3Aw/fOBgB0nukDADTfeiVCfTK2vnAI9985kxZvRErYRYI5Shbpuil18cyLh/D1ppmY6PMAuo6nnn8H+w52or6mGsvvmI5lC6ZD14F1y69FT7+MYHfELJAWH0nJ8xx+8MwbCdqyJI9EFDLx0cKaFnMyg9Oh6LHoufh1hwDghpkX4fdvfoiHWq6FKHBQVB2//M8jeHn/h/BP9mHFwhmoHl8Ol8ggMSCq6VjVVAuPS0woBhZfZNApKyAiJ6ZPB44HsfWFQ7Q2ylOowGr6GS5yVDWlY6CqCZHPBhLjbIsFSoyzXJ/OM9ABhCNqQrT0qqZa2O2Kw30yqn3uhD20x8UjHJe9mW+oqg5B4LB22bXgOUDTAVXXoJZQJmlEVrH1hUMJxV9Lad0djih441An1q9ogK7r4DgOv3vjQ9ww8xMkwUEQRPbRVOA3e47ixrqLwXhAVlS83PE+bm28rFSKwRJZID7dLtgdMf/2T/aZzrqW+dOxecfb5qLQP9mHpbdciQnj3Bg/1g1Vi0WQ7jvYaX5ulc8TW5TqOqCSPi6Re4xiNoY+ruGINuz4bCiKv5zswQXjy1HukbDj90djhyihqGWj5BF5RFUdv9nznqkpDQDjvBJJyxApY5fe7ChZxAG+MhEbWxuhQ8e53ij+0hUyN9sAsO9gJ4591I0NrQ1YtuFl89DQWyZi7qwp4DgOqqJCYDwwoFXbNO8KbNv1rumEJnkkotBRVQ0C4xEMDy2fYDjXbm28dECTmUPr918xPydwPIh1m/fhe19thDjGjVBYRTiqIiqrluLLG7Z04Ptfa4QOHqqmQ0dsE8/FrXkY43HHDZea63ZVA17ueJ/WRnlKOvITxYyqanFO5ZHbqKzZR5Zv/Eqj5VphCCmcZKkMTYWlXz667QDaWxtt98Wqog3573xDVnX8/MVDCXVMXtn/AZYtmFEykgkCzyMYCpu1L4DSkr5ziQwzp1Xhf7pCpszMzGlVcIkMajRzuRnkgCYIwhbGOFxXexHWbf6jubh+YGkdmMBBK/4xmMgS8QWlDHmNR7cdMAtgJRe48k/24Uu31+CRf30jIeJgyS1XAog5RKhgDpEPGLYdDIUTHNEAEg5YBsfTWdARi7homT/dLDRlFMuRGIemeVNt9A6tUTwEYYddAb/q8WVY01yXoAFtjp+qBp3xON4ZczzHFyQ06Ar2Q5Y13HHDpWj89Cfwva37Ez5nglfC6aRDlVVNtdj6wiEEQ2Eaq4miIJ1CZwaGNEDy4bnP68KZ7n70hRXHfufzunHW5rAy3uHtEnlcPzNx3R6TxuERpajnvIMKrGYHRbWPalZsiukJIocHls7Cxqf3J6zNBJGDYhMt7fO6E6Jjt+8+YhstLUoMp0JRy/rNTjIkX+A43VKXZ+WiWnAlFHXGM5j70vj1C89QMr4PWdYTovy/8YWrM/4d5IAuQqKyispKb1rvCUcUhLr7z38hUTKomm6m/wGxyXvj0x1o/4r9SS9BpEJyup0k8mhvbYQcVyk+vsDVFz831XQ+A4MRBysWzsDyBTNwz+2aY8peqVcXJ3KLYdvjvFKCI7or2J9wwAIY4+l+rFg4wyzOtnJRLZ558RACx4NQNR1RcObmxXhP+5YObGy1RvEQhB1O6c0QmWPKs6bppkyHU7HBnrCMz82+BA/+5DUbB1yjxTH36LYD2NDaAB4ceBaTCmCMpzGZKFhScUbZFZl7YOksAIOH5w8snYVQfxSahiH7XdO8K87r8I7Iuu2c4RShSYwuVGA1higxRFWYUhUSw4ictDwP1NdUWyJ5eR4Wz35U1vDcS4GEfvzcSwEsWzDdEvUriTyW3DLN4pyURB5aNPGZRVUUYF/kLOvUx547gHabyPFiJSprJS3BoWg6fvSLxD33j37xRsZ9P6VwL0sOSWS47f4dab1n5yPzEcpSe4jCRFUTteSAAV0smxNkgkiH+HQ7TtWhajo+Pt1rbriMIliPPXcAE8d5bO3QLQlQNc0xrZSqixPZxnHTpGLAEe0yo5vio6ENDDs2/o6Xn3n/4xDGVkj20aeqBneS1ihBOOGU3swB8AzYcETTwBiDS+TQF1FNmY74sTg+IkrTdARDEQftTPu1g64B3eGhIzgJolAY2hmlDhyAcxan8can92Ptsmux4IZPIhxVoOk6nvzVO2iadwUAOPa7SRMrHPobAI4Dz3PQHPqeqmm0bs9TMi0/kU2MoI6uYB9wngPEVANAshEpLIkMi27yW6KaJZFBSWqDpunYd7AzISsBAO653eqA1jTdVoLDTtrDaR7M576oOehha5qOUql9zXgO47xSwmulJH2nOmQPZNr3k699gCCIUYY56GLFBuFRbNgwSWfhROQGY4PWF5ExcZzHTHsKHA9i56tHsX4gssfODsNRBTzPQYf9wnao9FiBIqOJESJKDN1hBV1n+kydtKrxZRjjFiBHVaiqBgbAWy7i600zUenz2NpxKK4gTVewH2MrJDMSeuGcy23f89GpHkyu9oJDdsc1yiAobuw2/g/fOxtPPf8O/vbGK8zx+JkXD2HFwhmYNLEC0IH/eP0Y5l1zMT7s7HFYI9ivHXh+0BlnaEdHZBUKJEvxtnjIDolsMxwbc9KD3djaaB6AR2Q1oR8Ydi8wDpU+D06f68eTv3oHwVAY1ePLEJE1S7+7cGIFRIHHidN9tv3qLydDZhbNhtYGxz6ZrF9K/YpIh3SCOtK5Nt1I4VTsVlE00/lsfObGp/c7akDbRUsLNnvdobIekh20znvo/NUS5h00yXmeA0pkaCh16buhdekz9z2loilOEESauCQebc11qPJ5AGBQS04qvGHDWAy1bdqLex5+CW2b9iLYJ4OxwvstxcLgM9mDr//o9/j5bw+hanwZ1q9owL88MBd/c+3FOHGqF6fP9ePvv3h1gh2uaqrFmAoJP3jmDcdn6VTZGhxMW1je/grZAjEsVHA4G4rg8e1voW3TXjy+/S2cDUWgxm2ZVFWDxHMocwt4Yc97+PaSWQl2/I0vXI1yj4D21gasbq5DfU01KjySKcOxffcRPLA0cQxeuagW23a9G9vwZHFci/9s6ifFid3GPxiKYN/BTvyf195H1fgy/MOXZ2NVUy1cooCe/ij+ffe7mHfNJThxuhev7P8AKxfVJtjn6rvrIIo8VietHVY31wGcbjqf77p5GjbveBvf/qc9aNu0x9G2yA6JbDNcG1McIhwVTTMPwM/1RM1+EG/3y9tfwepNe+GWBHxrydVob22AS2L4yfa3AADfHeh3AuMhCBwUVcPuPx23zCGrm+uw9/9+ZH73C3ves123S0neMepXRLo4BXXYlSVL59qhIoWTSdVuFYcM3uToZwAQGbDklishCrG1myhwsX/bhPyKAo+7b6vB5h1vo23TXmze8Tbuvq0GomDtNxJDSn0xv9Atc/rKRbUAij/y1yCq2ssYRUsg+hmIZfYk7zseWFoHSczs3EAR0ARB2BKVNfzhzQ/xUMu1CdW0b7vustFuWtoMp1gMkV2Sn8m+g5049lE3WuZPR9V4NyZNqADPA//28rtY8NnL8PWmmRjndUESeJzu7scTv3wHgeNBYOBzkp+l0ykudGtKLNkCkS6KqtlGv8V0bgeRoyomeCXc2ngZdOhob22EqmkIhiIo9wj47uZ9CVEWL3e8b9p14HgQsqJgxcIZcEsCQn0ynnkxVsiN8VxWxzUaM4sfu42/4TCrq5mENZv2mv/dP9mHpnlXYOGcy8F4YNuud3HXzdOw89WjaJk/HWMrJIwf40ZPXxR//+gf4PO6B6M3GQ8GHYoeG4Pt9NCdbIvskMg2w7WxoSLFjAPweCkNO7tf/7PY98QEcXQ0zbsCW184ZM4BVT5PrCCtyOOvP3MR/u3lgNnfxpS7sOv1Y/js1Rfhw84eAMC0SybCWybG5hldB+M4Wz1d6ldEujgFddgV4Evn2nQihVO126GjOBMdiSpgBhPES+mUu60uMs1BH9cuslqOqpjolcw1Xya0rbOOzuGNQycs+/5bGwtv3z9c0rHdYkRRNTz30uEkTfTDuPfzMzL6PaVwLwmCGAaKqqPMLUISeei6DsY4lLlFKAU4CJf6hJJvMMYjYvNMZnxyIi77q7FQVA0fneqBJDI0fOpC00kHAO2tDWjbtDfhfUZks8742DPleQiMwz98eTY+OtWDbbveRTAUxurmuiE0EskWiNTRNDho5QFiXIqoS2QIRWR0nh6U6rio2ou+sGwprtm+pQPrVzTgo1O9mDtriunU85apWPvk6+bmaO2ya8CBg6JpaJk/Hdt3HzEdFpmyZRozix+7jf+hY6fw8L2zwfMcHmq5Fn0RGeGIAo9bwPe3/gldwX481FKPYChsysR4y0QAOjgOiCqDNmnIArS3NkBVNQgsFhkdL0tgSBJ4y0TogEWKg+yQyDaGjcXbYqhPxvk8sQJgavzHywxIjEMUHL731Uac64nid298iJb50zG52l7DuSvYjx8/+ya+teQzcIkCvr64FqfO9uM3e97D4nlTByQGBuU+DK3aKp8HLfOn47HnDmBVUy10HQma0YbkgRxXHC1evmA4/YpkO4qTVIoApuPUZTxnK2thd60RKWyVPIBF8iBVuxUArGmuw/q4z1zTXBez16TfrjpI6dhJgCgO+riKgz4ul/R7OT6/j3dcIocb66eg60yvuV69sX4KXCKHfPabZ5J07LwYkRUd3jIJf1VZAcYDY8pd8JZJkEkDmiCIXOBxM3zmyguw5vG9CYuCMhdDtL+wFpylPqHkE0YKXTAUNp+Jf7IPLQuugsh4tG3aY9rbfYtn4hPV3oTnFuqTLQvbQ8dOIdSbqDe3qqkWW184ZDqex3klcKoOBfaa0mQLRDoIzH5MERmHYO9gobUfff169EeUhOiab3zhalx84RiLjmDgeBA8z2HxvKkJtrymuQ7f/1ojorIGSeRxLhTF2icH+4mhGR04HsyYLdOYWfwkb/zra6px/cyL8NTz7+C26y5LcGataqqFz+tGV7Af23a9a+rUbtjSgfqaaiy6yY/Vm/ba2qThHFBVDb4yEQok1NdU4/brL4PHJWLj0846oWSHRLYxnGXJNn++ApmGPbe3NpgOWYlxOBOKJji+Vi6qxc5Xj+Ke269yrAPg87oRiaoJBQ1XN9dhwkAhNoXjbB1fRnHbieM8+N8/eS3BifbsrsNYvmAGVI4z23Y6FJubVjXV4qGWejOzZvvuI2ZmjVO/osLOxUmqRQCdDlzsnLrp6OimEymcznzg8QhYu+xa8Byg6YAg2Dt/04vWHsqxnnQP3AJOnotY7kHlWBeiYTsxktFH0fSUo8GLFYlxDgcipaEB7XIx3NxwKdZt/qP5+x9YWgeXxKCGMzfOk9gTQRC2RGXNVgcpIhfeQtNYOCVr55XOlJo/GCl023a9i5WLalFfU427bp6Gnr6oJbXux8++CUXRzOcGAB0HP8aim/wJGmyfm32J5b2PboulvBopeqqqx6LwQLZAjBym61hzd6Idrbm7Dhw/mCLqn+zDmHLJEl3zo1+8AVnWEmz4rpunob6mGtCtKabrB+xX0HWoqm46N4z/bqR3Z9KWqZ8UP3JUxYSxLqxddi1+8u05aJk/He1bOjB31hSLVIAxngIxaZitLxzCd78823xfcsGneJtkA1FfRvQkY0DTvKmIyqrpfDbel6wTSnZIZBsBwD23X2UrC3M+N5GqauBUDYKug1M1RB3G56W31MDtYhZbXrmoFtt3H8HCOZdb5okNcbqjhuMtHsN5XeXzgEOig9o/2YfbrrsMbZv2mFq5p0NR/P7ND+HzusFznFm/YPOOt7HklmlYu+yaIftVOrq+ROHgVAQw2f8bf+Dy1IM3ob21wfHwIV0dXU3VMagzrA/824oAYO2ya/BQSz3aWxvwUEu9rd3qjMPJYD/WPvlH3Pu93Vj75B9xMtgPnVmd0IzZ9y1mc60gcpb9x6Kb/BBE67UR2f4eROT8PThVVN02GlwpocPeqKpj266YBEV7awNa5k/Htl2HS0YDOlbAM9FuNz7dAUXJrO+H1nAEQdiiqpp9td8Mp2HkgviFEzgO0HVKHRwljGgDn9cNj5th+YIZaNu0B/ctnmkbhdDTL2NVUy2e/8NRzJ01BRdVVaC7L4Kv/d2nITAeoT4ZfRHFMTrI+Ds5Ci8+aolsgUgXVdUwvsIatdMvD0q8LJxzOWTZXvIl1CdbnBTfXT4bHA/byGhNAxico3UuvnAM2lsbMmbL1E+KC7vUeQAIR1S0fn83gJi8UXxUZTzx4ykABENhHD8RgihwGFPusr1+bIU06CweiJ58dtdhLLm5BlFZRfX48vNKcZAdEtlGVTVwDhHG6Uq9OI3P53oiEBiHCV4JD987G6qq48SZXjNLwKnPqRrA8TwYx2HtsmsSpJiMyOoHls5CT380IaK53CPYOrTXLZ+NmVOr8eNn30xY34ejCsrdItSoszuZ5HCyw2jLmqiaw15P06zazqoGDkBlpRcnT4Yskc+Dn5lGVHGakfWyrCVE6K5prkNyxUAnR2pyjQ4AEHgO3/jC1aa2s5GlJvCcVa5D0S2HrRuf3m+rAZ3Ofc0XNE23bbOmlU7Eqqbp2Hew05Q6Mmi5fTryun5khlAdCnhm2veTr32AyDFRWUVlpTet94QjCkLd/ee/kChIJJFhyS3TElICVzXVQhJ5aFHa/BHDQ+B51NdU429vvALhiIqopA5oqGm2qXWyrKHS5zFlCXxeN5bcMg3/+G//Zdrlt5fMQn1NdcKCwYgOMv6OT9EzFtECAKi64yKaIJxgjDfTmeM3TWPKJNOOvWUiTp3rt7XrMreIG2ddhLqaSeZC3+Xica4nis073rY4GXToYIyHDnsJGbfEoITljNoy9ZPiwG6Dv6a5DqLIo/NMn2lPRjSl01is64N/G3a59JYrITD7IlITx3ogIJZ5ojMez+46jNuuu8xM7XyopR5VPg98Xjfuunmao/xBKnY42k4corDhMyT14iQREI4qYDyPiKrjwZ+8Ztp8MBQGAISjiu37/nIyZGqp37d4Jh5e0YBQbxRuSUBUVrDk5hq83PE+6momJTjl2pbWmZI5Bl3BfnT3RjBhrHOfM6TK0vltJIczfPJB1sTlsNdziWzIA4mhSMdW0imIqQCWDIP1NtdqDg5wJ0eqKHJmsedwVIFoE9EMDB2YleycdN5DM2jDvK/ZxtkWeKglsu/nB/aoyTIrvI3MSjHCHNZzjHHIZLpLqRxoEOdBEhluu39HWv9zu+j8opjRdfsTZF0vvIWmschr27QX9zz8Eto27UWwTwZjNATmGp4Bd996FUK9UTy67YDpoON5DquaahPSU1c11eLCyvIEWQK7VNXvbd1v6ivGv3f77iMJ+l0EkSmcNk08i0kG1NdUY0y5C7/Z8x4eWDrLkna95TfvYOGcKxJSOWVZw4afWdO3m2+9Ck89/w4UOEsSjC13jcJdIAoBO1tdv6UDnWf6TCmkKp8H23cfMQ+Z7cbiieM8eGTV9djQ2oBJleX48h2xqui/fe0Yvr1klsUmDeczEHMMJ0t7GFrSTfOuGJb8gUH8/G7IDdD8TqRDpqRejOJnyX2nanwZnvj1WzgZjBUdDBwP4pkXD6Fl/nT8YOV1uOTCsVh9t/V923a9C2BQkqzzdC/A6egK9iIcVcFzwLRLJuLHz76ZmOr/dAea5l2R0LYqnwfneqJgPOfY54591I1gnwxNs67zSQ4n8+SDrImq2e/1VBsbSJV0bEXVdMz45ET88zfn4CffnoN//uYczPjkRNvvH8qxnPD9AwV246nyeSDYFAGMyCqe+OU7kJXY98mKjid++Q4iNoK/oshjyS3TEtZtS26ZZuuwLsQ9dDZsodBgDLYyK3aSLMWIKPCWPcsDS2dBFDK7nqJ5gyAIW2TFPg1DVgov3S6dE3Yiu0RlDaH+WARPV7AfP//tYdy3eCZ4nsNPnz+YEFmw9YVD+HpTLc72RMxn55SqynHcQJo2cOpsP3ToaL71SoT6ZGzbdRjLF0ynZ01kDKcU06isYYJXwuJ5U7H1xYO47brLwPNcgl0badef/+vLE8akYChi+5k9/VHsO9iJe26/Cpyu20oS8HleXZ0YPZxs1S0JCY4wb5mICyaWIRxR8f8+e8AyFn/p9hqIAo/+qAJN1XGmO2xGXf75WBAt86djbIWUEPlswHgOYyukhHYYWtL332kvv5Rqaj/N78RIyZTUi6pqGFcmYuNXGqGoGnQd4DgOTz3/NvYd7MTcWVPM6LLA8SA2bOlAlc8T09P1iNjQ2oCTwX74xrjx41+8icDxoPnZXcF+uF0CotFBCYKHWuot/cq49sKJFeZ3JRZDnI5JEyscx4QNWzrww1XXZ+0eEYPkg6yJ4pByr6jDl4pIx1bcLoabGy5JKno2C24Xg5JUrI/n7aMzk6NTGXSsaa5LKAa6prkODNYMGsZzCIbC2LClI+Ez7aK1NRW2DtqNrVYJDuc9dP5KcDjbQuHt+4eLqsFWZqX9K40lsZ6IRFW8uPcYHmq5FoyP3Y9f/e5IzAmfwe/JK3vy+/3fBfD/IKZE/1QgEPiR3++/EcCPAHgAPBcIBB4czTYSRKmQqzSMXJAPizwiBuM59PRFAcTsKXA8iC2/+TO+vrjWdhHIs5jGaHKauF1qXyyyQMfqx/davrdU9LuI3DBUimlU1U2HmLdMQtM8Pzbs6LBce64nmvCZ53qitp8Z7I4kbIhIGoNIh6FkAQAkOcIaceJ0r+1YPGGsG9t2BXBT/WT0hRVMmjCo4Wx8BgBs+c5NUFQOKseZ2uiaqsPndVnaEQyFwRycCqmm9g81vzPGk5OMSIlUx9Xzyb0YkjMP/uS1WDRya4MpD7Z99xGsXFRrkb4wPoNjPH787JtomT/dlOcwiMnVuPDkr99OyCK4b/FM2/4jCjwevnc2gqEIzvVEsfPVo7j9+svwyL++gS9+bqrte4zaBLKimc6W+N+rD9wfTtcT7hFJ4AyPfJA1cR5/eUAd/uoi1f4ky/a6yu2tNg4/Trf0n5WLagEu8V4ZB0GpOMCNwoadZ/pMCY7q8WWx65OuVTQHB62NrnO27ms2GXouHsWG5RDnKHurzEoxwngOb/33Kby8/0PztSqfB1/4X1OBDA7peZOf5vf7bwAwB8AMAJ8B8DW/3/8pAD8FMB/ANACz/H7/50avlQRROpRJPNqSUqjamutQJuXNsJEyThXEGUUN5hwBQPX4Mozzus0075gT+qAlZW9VUy1+9K9vYuuLB82UICNNPP66NXfXIRxV0LZpL97/OGT7rA0NXYLIBEOlmKoDhVzWr5iNWxovxZO/ftuUOTCvvbsOr+z/IOEzX9n/geUzVy6qxSv7P8Cau+sADtAZT3ZMpIWdra5prkP1+DLztfqaajx872xouo7JF3jxrbs+Y0nB7IvEHNaiIODx7W/heGePZay944ZLEQxF0bZpz4Acxh6cCkXBMw4ekbftMy6RG1Fqv9P8/v7HIZLjIDJKqnIv8YcixqE5ADPjYMXCGXj8W3Ow8auNGFsuIaLp0BgPgfFYu+wavLL/A9OxDAzOBU/++m0suOGT8E/2mZ/377vfte0/TNfh4jlMHOvGxZO8+PIdMzDO60IwFMbPf3vYso5auWhQtsxIt07l95IEzvBxifb7LJeYu3vnEjmHNuRmf6Q6OHVVG6kKTuew89WjaJk/He2tDWiZPx07Xz0KTre2VVU1cKoGQdfBDdQScCT57Q4/3ah3EE+VzwPBxtZH+74OB7fL3h7drtLpy7zDeqJUsgzdkv06zZ1h3w+XDS0av9//naSXdAB9AN4JBAL/McT7xEAgIPv9/ikA9gD4IoDvBAKBuQP//S4Afx0IBL6UQjMuBnDs9OkeWy2rkVJZ6cVt9/GoMcMAACAASURBVO9I6z07H5lfdO85eTKU1ntGilF5d7SorPTmYgS6GFm03VTRBYbX3/4IdTWToOs6OI5Dx8GPcc30C8EphXUUmg+FPrJFqn0in2yXMR4646FqsRRVTQMYD4T6ZQS7w6geX47OM73YtutdMwW1vqYaS2+pAccBPf0yZFnDxHEeMD62KPzmP76KrmA//JN9luI6Rurp8gXTwQ3zeY/22JMv5Po+5MhugWGMu05RX7zE8JeTvYjKqpkq7Z/sw8I5l2NshQRvmQSOB8IRFe1xY9IDS2dBEHicPtuPijIJZS4RUVnB2Z4IytwCvv1Pex3HLrJPe0bzvuTbmJtsq0AsmYkxDmeTCmquXXYNzoai4DiYhZaCoTAevrcBfzkZglsSoKgaPG4B39/6J/N9G1obsHrTXksEVXtrIzhVtbRDYhxOh6J4dtdhzJ01BWMrJPi8LnhEHnI0tXWG3fy+clGtKXVjSBwMd+xPhWLrf/lkuyNlJM8m2V45cHhg0x4b+060L53xaBvoB05rkt+98SFurJuCH/3ijYSCXz6vC2VuATqA/rCKUF8UUVkFz3PgOQ7hqAJJZFjz+Gvm93//a41QVf280Z6ixBBVY04/SWQ42xOBS2Q4cTq23gqGwljdXIdLLhyL06d7En6H0+9N5ZpMkcqzzDfbHarNOmN44tdvWQqeLV8wA1wOo2Qlt4CIrEPVNDCeh0vkEA07p7qm8hxSjYrXGUObbZ9qtNwDXmI43R1Gd0/UjFYeUyFhwhg3tBTnC0s7JQH/c7LHUnjvE5UVliKMvCTgLzbX/lVlhW1hwXTuaz7YrS4w/ObVo7ix7mJTfuHljvdx63WXFdy+f7gIbhGnzvabUfnG2nziOA+UsDzazcs6vMQQDEVxNhQ2+9g4rxs+r+TYx4Zju9nKPp8OYDaAf0csaP8OAO8D+Du/318XCAT+we5NA87ndQD+HsD/B+BCAB/HXfIxgE+k05AJEyrSbjyROpWV3pL4ztFgtG3341M9+MOBj1DpKzcXRn848BFmXTkJkwrwGYwbp+OHq66HrGgQBR5jy11Fc6KZb31iKNvVNB3neiOIRFS8/3E3tu8+gns/Px2SKJgF2NpbG7Bu876E9+072Il75k/HIz9/w3RKP/XgTajyleHE6d6EytSKqmFVUy0mjHHjeGeP6YhYfseMEd2rfLvPo0Ux34fzjbuG/cqKBibysZQ0NXFMOdMdxqPbDuC+xYPatvESBe2tDRjndaG8TEB7ayNUTYOi6vjlfx7BTfVTsG3Xu1g453IoZZrp/Gu+9UoAg/q2P1x1PSrHlye0rZify0gohfsy3PVCLN20z6KhvPbJ19Eyf3qCDId/sg99Ydk8VKnyeXDf4plY1VSLynEeuCQBkahqG80GHhBE0Zx/Jw70FUPqoyvYb8oUVPk8MfuuLEv5dxjzeziq4v2Pus0x3/x+jsu6HZSCnWWDXKx1h/NsNE3HBye68fBP95n2/g9fnm1v30n2pSga1txdh/U/60DgeBA7Xz2Kh++dDV0HFFXHr353BHU1k0zns/E5j247gPvvnAmXJMQOM3kOL+37AJ+9+qIEp1fb0jr4J/sQDIXx4Jfq4fOmH6HXFezD4//+FpbcMg2TJlZgVVMtTp/rxzhvrG9WVnrRFew77+91ukbVAbdbzPhaO5/6Waq269Tmj0/1Yt/BTnPsM7hnvj7ifVb8WuV8ex5N06H1RgAlVoTMW+4Cb+NTUhQNwVAYH5/qhSAw+LxuCDbFyez6zoNfqseUC8ZY2nDybJ+trAbHA5XjE+/B6XP9CRroxhwEDH987zrTZ6vr3N7aiKqkz/z4VC+2vnDIUh/h/i9ebfu8Ur2vucbJbj8+1YM/Hwti2iUTzd/352NBfG72yO2xUOg604fnXgokPOPnXgpg+YIZFnsoRk4G+8zAGIMqnwcbv9KY0bE3Ww7oagBXBwKBEwDg9/vXI+ZQvg7AGwBsHdAAEAgEHvL7/d8DsBPAFYhFTxtwSFOBJJsR0ARKMQI6Z9812hHQkiRgyS3TLCe9ksgXZKSPcRoPjoMsqzh7tq/go5+BtCKgc9CaGE62axep9q0ln4EOIBxVLCmryRMgB5hOhSqfB9B1nDwZgiAxW1s93T2oYxp/fXx7UtUtHO2xJ18YhQjonH0XMPS4G2+/Pq/bYnNGZHJkIP3ayY7DUQUnTiuo8pXjqRffwtJbrkTnmT7MnTUZvjHWz13VVAstLlutK9iPcETByfDgc8hkRFIxMcoR0Dn7ruGuF3TG40x32NaBNLZCSnitad4VFkf1j599EysWzgAABLvD8JZJFpuvr6lGqCdqyUDylotQVXu9xWT7ThXGeGze8balzyWP/Zmm2OaHQrDdVDnfs3EaF3XGmw40IGaXH53qsR3T4+2LMR69sgpF07Bi4QxUlEnweiQ89fw72Hew03SwuSRmsX2f1w1REND2z3sSou+eeymQ0I72pzuwYaDIpoDYPUwX5rBuMrbdJ0+GAIdaMAn9yeGa9z/qxuYdb2c02zDFCOgRf0+qjDQCmjFmr7nLcSMaT9LJ+kz1WlFiOBWKJmRttTXXYaJXsmarCMzSdx7+6T5s/EojkBxJKzBTVsNw+O189Si+fMcMyz3QGcOPn33TMge1tzZark11vaNynGMNAetnMvuChTbPK93M23yw22Lb9w8HleMdD4VK4R4oHG+vc646//7h2G62RF0mGM5nAAgEAqcHXosCsI1f9/v9U/1+/6cHru8D8EsAnwUwKe6yCwB8lKU2EwQRh6bptqfCo+kUHy7xGnX3PPwSadSNEgpgcWCEeqPY+PR+uCXB1JwyCvUk6xMaRbOSNUJ1zb4yNTcg5GZcL7HByAPSLSTSJd5+F8653GJzG7Z0xCQNBjTk7Ox4VVMtvOUStu16F30RGXNnTcHaJ1/Hus370LZpLzpP95533B2Ofj3ZO2GHqulm8ct4qnwes/ir8e8L4ooOGnQF+zFpYgVC/TEHs2ije3nP7VdZxv0NWzpw9H+68ZeTVh3pkdRnGEqbnSCSGWpctCtuuW2Xvd5yvH0pADrP9OF7T/8J6zbvQ7A7gu888Zrp0OgK9uOx5w6g3C1abL9p3hXY+HRiX9n49H7MnTUl4TqjKNZ5tW2HQFPt101anH8wlf5kd42hJx0/LxJWJAZbzV1phNXO7NbaTs8h1WujKhIiI7uC/Wjf0gG7rHzF4WBRsSmsyHgOt19/GTbveBttm/Zi8463cfv1l9nOAZqDXnTyvpQxHmeT+vVZh/WOs66z9fvTeV7pPIN8Qdft9/3ZkOvNVxgr7ZpRuaqZla012Xt+v78dwBOIRS3fA+Co3++vh3MdzUsBrPP7/Y2IHb/OB/AvAH7g9/s/CeAYgC8gVpSQIIgsM2Rl+VFq03BxWgi0D0SQELnBzqbckoCuYD+ismKm4Rkpq+uWz0ZPfxTB7siAhvMMPNE21xLNoKr2i9KJ4zz43lcbca4npjG6eN5UM/qAbIJIl3j79ZaJjuOji48VVduwpcMsODVpYgV4Djh5th9P/uodBENhBLsjGFshJXyOwOyjDyQxNuoObng4yGlI8pG9E3YwnsMr+z+wTYHevvtdMyqt0ufBx6d6baP1zvVEzMrx4YiKiV7JlJZhPA9Nt19LeMtEbPnNn7GqqTYh4mqNUcxzGL9HVTX4ykS0tzaUVKQ/MTyGGheNjXi87QZDYYzzSkPal6rp5roGcJ4rXBLDg1+qT5ApcDrkmTDWnfCa6RCwceil/NsdnHmKFvdbUuhPyde8/3HIIoGjajodAtkgR63jpcSQsv69E0Pt35KfQ6rXOhYL1DTLZ/I8bOcKnodlYI/Iqr2sxZ0zbT7X2iftPlcFh/VJ/Xr9lg5sbG1EMiIPc70WH6ks8taIyXSeVzrPIF+QFfs2y2r+tjnz6Lhv8Uwz0n5Q5qU04Bksa7JVTbXgGaBnUAY8W/Z0N4B/BHAAsfn9NwBaACxCTN/ZQiAQeNHv99cNvEcFsD0QCGzz+/0nAWwH4AbwImK60gRBZBme51BfU20pjsHzXJpCOKNPIS4EihG7DV04qqDK58HJs2G8sv8DrFt+LfrCCsrcIrb8ZjBlde2yawYqU3MQGA9F06FyHASeh8jsF7uapkNRNVSOc2PJzTUIRxUoA++PkE0QaRJvv07yGoznLBtygecRlhV858nXExx8bxw6gVsaL0V9TTVubbwUE8Z6IIk86muqE9L/qnweTBjrxuY1N0LTgN++9h5ubbwMAuNt02nt0k5pDCTsEAAsnjcVz+46jJb503FRVQVOnOk1HUgv7//w/2fv/eOjqM798feZX/sj2SQLJCgKWCkEDEkNmCAkagvK/Vblh82rJfiDRA0osQWtbSHQTwErBGtbr9aLLWJF6FXovVQRofei0FpBBRS8IOJCEQGFEgKbsPmxO7sz8/1jMpOdnZnNbrIJu8m8Xy9eL7J79szMmec85znPeZ73gxy3A0uqxuNPf/1ctzFZMKsIuz75CqO+MUCtFB8UAAYSiCSBhoQQ0ev9cXkDke124NGZhaApgqd+VIpWvwA/H4IrjUNAEEHTVNzO475IM2Oh84imF8MPEsMdU0SQI48ZABAk3UEJTRHVrqnztkKUJCypGgc7x2gKekICMtI4rHykFCFRAkUAymCu5LgdyEq3qYfpO/adxMzJIwEiU+h0JONmc0JZz9wuO8omDofLybYVOKQ0vwu0/c7WtrYZ+SAEQQSBOQVOV53lFmSEF5GM5vw0srXN3gNtsteLbEtTxlQrNEUBEcUCGZrCgoobdMUCGZqCGGmzhNUC6OhelWj7yDkZeWBpfriit3eCvIDsLLvGqWznKARajQvOiZp7kjQZA5HPFcu4JhNM9/2k74QpEBDsPXwWS6rGawoxTrlp2OW+tR4BAeC0s5hbVqDOXaedBYGWE7mr6JZ9h8fjqQcwMzc3tx+AkMfjudT21Qsd/G4pgKURn+0A8K1uuE0LFixEgYOjUD55pI7vy8FR8KdYNdx4jDEL3Qcj43FgPycWVxbj1e2fY8pNw7D9wy9xU+FgrH3rU0wqGorvfWc4BmTZ4b0UwNIXPzTk3l18fzGWzr4RSyMcfC+/9Snu+vZwCJKEZWs+0BisWS49V6klExaiIVx+N+08pnPGhW+ElA257KQQkOFk8eTDE+BrCcLOMSCUhFvGDMbWXV/o9OzCiiIAUA9f5pcX4un1H8Pr82N+eSFKrr8KvlYekKDhE4zGOSjB+JDGkve+DUEQ0d/F4cGpo+H1BRAICXC77LKDDFDl8fW/y06zbLcDTz48AV5fAI1NPP78jgczbhuJ9w6c1sjp4spi9HNxuOCTs0/CI6zH5Q3EjNtysWjVbk2E0d7DZ3FT4dVYGMZ/Gw9/bLycmxYsRLMNOxtNz9EEOf2cmF9eiDf/cRwUIZqiafPLC5HlsuGtXcfx2Qmvxp759x/fostGWFhRhNVvHFTXg0WVxTj8RT1e3Hy4QxmPNicAYOnsG+H1BXQZCJlp9k7NpVgdhBZkxMOrHE/beN4DRxPDvV5klpVCP6FvB102FksT0ITSyP2iymKwNEEg4vrx3GusczLWSGllXOsa/DGNazxrjI013kPbWAp8kq5HTpvxPTttFPwtfWMGOzkKN48ZrNkz1lQWw8lRaEkx30dnwFBEPYRUwLEUGIqAT+B1SHfwuuTm5uYC+BOA69s+eh/AfR6P51TCL2aOawCc6M4ihFMe3xzXb7b8Zlqv+00fLELYE8eA16AbZTdWSDSNmlW7dAt4bXUpiJBaSrg3b0zjKEKYFLJrFI0DACEQtASCcNpYrH7joOYEPt3B4t83yOlQiyqLDSNsVj5SCkkC6htb0djEY9POY/Cc8iLH7cDcsgIsW7NH2766FJda9IWxohUI6QsFKDrCZShC2FOhF9cgBr0bLr8cS0EU5GibWJwTkdFLNat2oWpavqE8L509HjRNcOZ8EzZsP6opvjm3rABX57hw8ZIfAzIdYCChX7801F1sRk2bUy+8r9rqEjBAr9WB0XCZixAmhc6NBeFy7bQzCPCiLKc0BRtHoaU1BJqicO5ii6qLFeS4HVg+twS//tPHGjmtrS5VbYjcIW6UTRyOIVe4IIoSlr74ga6PJVXj1U1f+Oe11SUgMcioRFOm8h/L77uC3rY+pJLsdoToReASbxtKNIXVbxzCD24bgaw0u6Ed/bP7boDDxoIQ4NzFZlXHL6osxo59J1X7JyPNhnXbDusyYqqm5WsKLJvJeIdzgqax0OD+Vj5Sqh4EGf4uCrozCyHGIoRJJbvR7jmefVa8e7JY30M8elO1YSQJNCGmEdjdda+xtqU4Gl+fb9bRCFyVnQaR7/y4xjNW8Y5BMsitxNBqAVQFOW4Hah8pBekDzlegd/k+OgOJpnV78B37TmLO9ALT5++M7HZX5uVaAGsg8zVTAOYAeAnAbd10PQsWLCQY5nxfqZeyHX5qDkIASbJScy8TtJGh7emrAiH40a//jtrqEl0F4trqErhddlRNy8eQgenGqXWCCEGQ8Mc3D6Ns4nBU3nkdKIrA7bIDkPAfP52I1/9+DO/sO63yHFpcoRbiRbj8KhuZcFmOtjkK8gIYmgIoAkEUUTUtH+4Mm6E8N/uDcLtsSHdyePyeMWBZCnxQhChKEEURLf4gGpvkeAS3y4ZQKDrNBpEkS94tGCJSZgVeaKcYCAnIdDvBt/gQEkUQAkMZoyjg3u+OBENTKs2AYkMozmd3hg2EAI1NAcM+WIagpqIIHMugJRCE91KgrR8JthicDmIU+U81m8VCz6A7OMMFUUKDj0erX0Ao1Gook+lODr/4w/uqc+wXVTeCYyiIooTZ0/NxobEVf3zzMzwwNU9jCym/dzlZdV652jJcFEqm8PmMaLVcaMq0qFuoC3PJzMbrLMKfx+vzywUiU2DdUu67ztsCmFClCKKo2raKs0fRnXqu5tjbArG/h3jouYK8AALgyjanulkdinj4ouO5V5qm0BwUcO5ii0oPMLCfE2ksrRlbGrJdFE4j4HbZQEPPIBnPvcbHrR3fGCQDzOrpCH2IAzoV31tCQSR8/9YR8DXL+wuWIfj+rSMAkthD4u4aS6fH4/lD2N+/y83Nnd1N17JgwUI3gGWM+b5YhkAypsayYKHTUFJhjbh1KQpqmmrVtHxDuQSASy0BHT3H/PJCrNt6BF6fX6U2OPjP+j5T0dhCz6GjaDqj75fNmWAozy4Hp6EoCJfjmspiZLo4LH+5vZ/F9xfD5bBoZSzEh1gjQOk23v1wblsF4/IGorGJx+/+/IlGXjmWxri8gZhy0zCVUmBJ1TgAxnQw9Q1+iJKE2lf2afqx22hcbAxEvUfl/iz5txAv4nWYdhSBSVME5ZNH4LmN5vbK2fom9TO3y46mliB+++rHGrmfX349GIY2/D3DULjv9lEaqo5FlcXo30Z7o8yVJVXjDH9/7mILfvfnT/Dkw8brD5MktHWpmr0Y631zLG1os3IsDZEPafqMp21c99oN75qhjfePDE0BXYiklWgC78WAjtLGmZ0WUYRQjtIe2C8NFAFEyQZBMuYvj4vb2qQtY9DWfA9NQYqngnQPwtr3d5/spgpoikKAF3RzjKYoiAksAEZ13KRT+Dw3N3eC8kdubu5oACe66VoWLFjoBhBCML+8UHXuKUqIpGAxAsUYrFm1Gw8++TZqVu2GtyUImu4uFdj9oGkKEk2hztsCiaZS+lmAdh64HftOYt4Mrdxlueyq0b1p5zHd9wtmFeGlNz8FHxTVdoB8av3shgMomzgcdd5WrHxlH773neFY1MZvp8jEnNodvUImokGRlxAhvUJekhEhQN1wArL8rVi7F6Eo369961Msur9YI88LK4rw8lufmspx7dq94AOi5vvlL+8FRcucnkuqxqG2ugRLqsZh6ewbwUCrA/uCvFuIDR3JbHi7l978FK40TmcXVN45GivbnMZKH89uOABRlPDg1NGqkwwANmw/iox0DgsrinS2hQRJ01bpJxiUOrxH5f4i1waFSzReWPrSghGi6VFFZkRRwlXZ6ab2ysKKYmzYflTts2zicNX5DLTL/bmLLWBogifmTMBTPyzFospijMsbiB/fPRYZTk43V1as3Qte0M7nDduPGtrxVw5IQ9W0fPz1/RNYVFmsmzNul93w856OAIxVPyUbYr1vUZQMbVYjeoR42sYDxfZO5LumKYIFFTdobJEFFTd0OfAjJBiPQSjCUS4BuNTEY+mLH+Dhp3Zi6Ysf4FITb1hETeG2Dn9+hds6ElRb2/DnqqksBmXQNhX30Kl4z4kGRRmPAdVHgpYEkzkmJPjgsbvWkqEA3s3Nzf0/yHr4egDncnNzDwKAx+Mp6KbrWrBgIUHggwLWbT2iSfdat/UIHr9nbMqloZgZg7XVJUjFJSVVo0KiQUmFnT09H75mHsvnlqC+oRW+liAuNfPqu/Oc8mL9tiMqHcepc02gaYI9h89h+i3fNEydcrUV3KnztoKhKaSxFHjB2KmRqjIRDb1RXpIRHaVnGn2/5/A5zLkrH4+Wj0GWywaOpUAITNOulf9H1qNWrhMMiprIhcWVxQBL9zodaCExiDWlWBAl7Dl8Dg0+HrPuGIUV1SU475X1c1Mrb9hHqC2yNPw7zykv/rr7S8z8t1zVtnBn2PHMq/tReed1pqmvHd1j+P2F2yyuNBZCKD4dZ+lLC2Yw06ORNSWUyGPFXnnioQnwXvLD1xJEayCoFvkEAJeTNZTvdCeHS03aOhULK4qwbfcJ/ODWETHNFc8pL9Zt1V5/3dYjqLzzOqzZfAjzZhQiy8XpKEgYhkoKyqZ4KA+SCbHed8iE8iAk6NP942kb1712Aw1NSBTB81pb5LGZY2Qapy7cqxnNkihqIyrNnGgrqkt1kZdBXkB2pg211aXq89tYAt5vfswR/lw1lcWGbcz30GOSVnZ7076/s0jF95ZI9BT9aneN5QIAgwC42/5eD3m3dLibrmfBgoUEg6YpeH1+tdAJ0JaWRBMkffhBBFLViDVDb3EmRaayUjQBEYDPTlzAjaMHqSfO/oA27dtzyos1mw+pBdyeaKMxMKLvUD5X/i9HYEi9TiaiobfIy+VEZAFBo+I7ZqmsSrFCCsbUA4IgF35b+uIHWFI1HnXe5o7lmKaQO8StKfoGiWB5xHte3uYc6U2c/hYSByOZHZc3EDaWRlCQOcfrG1pgY2nVobb4hfc1xWB/+dB48xRm6Gkxbhs3BAFeRGY6h8YmHqIkc7ua6e9gqGNqDeU5PKe8uuJs8cLSlxYiEW6rGDoBRckw8vjZDQfgOeVFY5Mffj4El5NFSBDxs1k34FfrPkKdt9WQ1ibH7UCandUU61SyuB4tH2NOG2CQPu71+XHqXz7NvPC1BFHnbcVzGw+0FdcS9XU5BBEcR4OHXLMAFAWbnUEgKPaYU7o76CF6AjRFcNct1+LW4mtAU4AgAu/s/VJ33zRFYVzeQF3BLyP6h3jHIhabRW1roxEKiJBdNQSsjYLQ0oX3KhE889p+jew+89p+1FaXGjaPtQihOQUG0VBwiKJxrQJRlHQOaJqmUN8BxZMCQQBqI9aG2rV75TkU2S9lsoc2eLfJgt607+8sUvG9JRLmFCSJlYHuyimbBrkQ4bK2f0sBLPV4PO96PJ53u+maFixYSCDsHGWYlmTnUi8VVTHcwtHujEw9RHOepgqMUlnrfTwuNLYg79oBqFm1Cwue34U1mw/BYWfw47vH6mgKsrPsmFtWAEESTek75pcXYtPOY8hxOzBvRiFWv3EQDS1BcCzVq2QiGnqDvFxOsByNeh+PmlW72mR1F+p9PNiIHE2jVNals29EYxOPhat24en1H6Gmolgnn7/5z/14bfvneOKhCWAZgg3bj3Yox2s2H8L9U/KQO8SNHLcDP39gnGlBqaAg4uvzTX1G3i3EDo4mmnV+XN5AVNxxHb4634RFbbp5wfO74PUFsHTOjWq7HftOoqaNEsBhZ3Qpq4sqi7H6jYP41fqPNd/Jzm35sEXR7xSR07WN9Pe8GYV4/e96GoPIFPFEppFb+tJCOMJtlS/P+gz1KEXpI/2VyONVC74DG8fghU0HUbNqN373508QCkmofaQEq2smwWln8Pg9Y3X63sZShnLYP9MOG0t08j6/vBANTX7dXFxYUYQd+06qf8+bIa8lSn9mFA6R697qNw7ifGOgR2mcuoMeoidgYyncPGYwlq2RKSCWrfkAN48ZDBtLRbQjKJ88Ems2H0LNqt1Ys/kQyiePhI3Vr8uRurqdKkLfNlabBQDsThbnvH5N23NeP+xtGVedgSiZRCpLelmLhx6MhoTFEWOwuLIYdERGGGOy52MM7J14aF6iHeRHwmEz3kM7bMm7h+5N+/7OIs3kvaUl8XtLJBwcZahzHQmWge7S4d8DMMjj8Vzopv4tWLDQzfDzIv6x/zSWVI3XnODfedOwhEYBxXry3RUwkLlRIysnM0CXK3RfDqRqVEg4jIw+JZKgZtUuzee/WvcRnpxbgifmTIDX54ckARlpHPigiEEDWBBC8M6ek5g9vQC+5gCWVI0HRQHNrUH0z7Ljgal5aGzisX7bEXhOeXHizCXUPlKKXz40AWfqm7Bh+1F4fX4sqiwGCCCZVCxPVfQGebmc4GOMejFKZSUgWPqiLM913la0BIKYW1aAgf3ScO5iM/720WmUTRwOl5PF2fomDMpOh9fnV2lm3Bk2ZKZx4FjaVI6JBPTPcqC+ocXwPSsy/rNZN8DXzPcKHWghMeAFCRu2f66mm2ak2XDuYrOaYgyE0Qw8UoqVj5QiJEj4+rwPIUFE1bR8LH5hN9wuu9qHEump0Mis23oEc8sKMCg7HYIg6aI6lci4B6eORksghBXVJRAl4Mszl/D3j0+jOO9KpNkZLKkaD5alQBPo9HNn0sjNbA9LX1oIh2KruF12OGw0ls0Zj39daNbYDQytlxmvgFMq+AAAIABJREFUz48Lja3ol+HAipc/1Mj8b1/9GCsfKQXX5pDsn2nHiupSiKIIiiIyn7RJUc0z9U0YMtAVJu/A1+d9KrVGZPr4tt0n8MCU0Xhwaj6+Pu9T1w+lP4qCZhFQoiD9QQkNPj/cLjvqvK2YVDRUtw52d2ZA5Ly22xhIQSHpbbNAUIrJZggERZN2+jGN1NW+liA2bP8cc6bnG7SNPVK3NWB2D/q2sYIixjqUMuASDgF4LeK5XjN5LkEQkRWDnqchYfH9xbpCzTT0RUbjyYaMJzq0NSAY76FLr03aTJqe2vcnM5oDIj77oh7L55ZAkiQQQrD38FlkjB7UJ8agJSDiXUO5TawMdJcD+iiAhm7q24IFCz0AQZDw+rtf4PV3v9B8/t2SaxOmOCyuxc5BiQqJHLdUciYZGX1ul900wuBCg/zZ2rc+Q9X00WiKkJt5MwpBEaC2rRhW7hA37rt9FPx+AQue36Xrr76hFQue36VGqdo4Bg2+AI5/dQk79p3EzMkje40c9gZ5uZwwj3rRcy8Kbby3SjpziBDNb//20Wl8d8I34GvhsWH7Udx3+yi1mFSO24FfVN2Ix2aOwTOv7ceKtXtV2e6faTeUY54XwIcE9M9y6N7zuLyBeHDqaDQ287j3uyPB0JSWH/p+mR/aQt+Fwp2sOItrq0tg5xhTrlEbRRCChGVr9uA/fvptSG2RboozSsHvF05SKWI8p7xYtmYPnvvJt8ExtGHf9Y2tGJDlgINjcO5iCwgIduw7iSk3DcNzGw/A7bKjfPIIDBqQDkLL1B66Z4mYe9F0WzTbA7D0pYV2CKIEt8uu09WLKouR5eJABDmtP1Jm5s0ohChKuNQcMJ1PLEWBY2icPudDupODy8HhpTcPYc/hcxiXNxBLZt+IurDAiWy3AyFBgiACEiU7xERJxLI1ewAAvpagYfp4cd6VGHZ1Btwuu8o/bSTX7fNit+Y51m87YspV3d00TuHz2u2y4/x5XzdeLTGI1WaIx/kZqasVPDh1tEHbOGyWONoqtB5n65tB07Q5rQeRMG9GoWa+zJtRCBC93iYEqp4Pb9vVwncsQ2FuWYE6d1jGOIIzngNHlqGwsKJILbqrZBiwDIVgSDsOgmiyh57wjaSN4BdN7vn2kmvRZyxFImHEkH5Y/EK7DvzpvTcYym6vBJEwdtQVWLbmgw7nblfQXXPgOchFCP8GIKh86PF4nuim61m4DOCDArKzXTG39wdC8F1q7bihhaSA6aKcwGq4PcW1KNEE3osBjfNlfnkhnNlpKbmjDI8KASGAJKVcxG6kfOUOcWPWHaNUqoBIufPzITA0hco7r0NTC6+L0Htu4wH8ev7NqnHoOeXFlveOY/b0fMP+Gpt4AGjbkAXw7IYPNYutWQRGKqI7isz0JZjybcbACRcu57lD3Pj22MH48zsezLo9D+WTR6ibLkCW4yfWfIjH7xmDpbPHg2UoSJKEl978FPdPGW14D4QiWP7yXjw97+YIvQD4moP4+e/f1+g8JZqtztuK5S/L/NAW+i4i9bCvJQiWMV77GZqCt5mH1+fHXbdcCz4kgRDjKM2z9U0omzhcwzvrsDGmUZ2KPlYOBX826wbcP2U0fvGH902df105IIxqe1j60kIYaIoY6mpFXhgAF3y8GsWZmc4hM82G/955FJOKhqCxiTeU+S/P+pCZziIkSBrbdN6MQjT4eDT4ePiag7qCZxu2f449h8+pOj3H7VT737TzGBbMKsJT6/Zp+tvy3nEMuyq/Q7k2mhfPbTyAqmn5phztVmaAHuaRshQQ5qg05zTW2xbxtI3HZjHiDc9xy3zi4feq0HrUhh2y1FQWY4CL0zmhCQi2vHdcE9W85b3jeOiuAt1YSRLRzS2VmzyibaxBSyEAS1/8UPdMRnvLeAI0AkEBG9/2aJ5r49sezJ6er+O0jed9JQsok30/RRGgjyx/FCF4+k8faeTx6T99hKd+WGpw7N0LEWU+JhLdRWiyEMAlAFkAssP+WehF4FgaUx7fHPM/uy1Zz/wsGIGmiY5Lbn55oVyMIEHoKa7FkElF5FAKG82CIIIIInLcThBBTLnNcSS3X/nkEXh2wwFs2H4Uj80co+MwHNBmBD3z2n7TCD0+zDisrS7BpKKhuNTM6+Q4nAOxbOJwnWw8t/EAJhUN7VWcn4q8MJKUkvJyOcHRMOFe1LelaQoSTSFECCSaAkcTlbOwbOJwPLfxAPYcPofnNh7AFf3TDOU4I82GZzccgCjKzudJRUPBmOjjC42yMzkQlDc0ynuGpHckPLvhAMomDtdcK9SLZNxC/IjUwzv2ncTAfk5DTmeaIlixdi92/98Z3Fk6DHxQQLM/qOMLnDejEBu2H0VmOqd+tmBWEc5fbMVLb36KBbOKdO137DupOqHrvDLtkhJdrcybSOdfV+rhdGR7WPrSggIGwKAB6abyojht9xw+hxVr92LB87vwi9Xvo+Rbg9SicpEc5gtmFWHTzmPITLcb2h9lE4ejbOJw/PbVj3XUCJOKhqp/P7vhAC5ealXnoOeUF7s++QpPPjwBT/2wFFXT8rHlveO4e/JI1dkcTa7N5oXLyaq875F6IXxnF7n+dSc/dDKDoozXayqCg5iiYdzOwLaIp208NkusvL9mtB5GAdAURfCD20aAZeTnZRn578jnB6JEYBvwRcfK1xzP3jL84P6ln9+G2uoS08NNsS0KfcXavahZtVud96LB8hDP+0oW0CZy25dqhQRDxrITTGF/QTyIh7+9K+guj2Cax+OxwmosWEhhCKIEp43RpDA5bQwESUpYVGhPcS2KJsaIKHbfKZyF6IiMygWgRmbu2HcKTzw0AY1NATQ28dj4tgd3/9tIDOznRJ231TQSRxShpijmDnHj3u+OhMPGIDOdw6PlY0AIkJluwytbD6sciGZppZnpnBXZYwGAnGI6wMWhtro0akV5s+ic/i4OK6tLEQrbaHlOeVHf0Goox/UNrfCc8kKU5M1Og4/HrDtGITPdptHHNo7Gi69/asitGM2RAMgZB+WTRwCQeh3nuYXYYZQdwdEEaXa2jZNWAssQMARoDQpwu+z49tjBKk9/jtuBFdUlqlz6WoJYv+0IvD4/0h0cnp53E5w2FnYbhbqLrdhz+BxcTg7L5kzApWZZv2957zim3DQM67cdUe+rztuq8oi6M2wmzgTARlOdkluL59lCrBAEESxNm8pLuK7NHeJWOf0HZDnAB0P4wa25+PM7HjU6OiPNhk07j8o6vgM9He075W9RBNKdrBqVGRJE2Gw0+lN2uF12PDS9QOa+jWGe0BTBuLyBmFQ0VI3w3LHvJLLdDsyZXgAbS0wjqC1KvXbwQUHHxb1u6xE8fs8YjeOFD4oxtYu3baw2CwA0t4bwxVdeTdtPjp5Dmj07gi4kPloPnhc10fuPzRwDQRT1Echm0eIGUbexUpZ0l35nKMpwfjAU0YVLx/O+kgWm93zv2KS950SDomAcBd5HfPCEwFDGE5j8DqD7HNCe3NzcAo/Hc7Cb+rdgwUI3g6EIOI7GwH5poAggSjZIkMAQkjDWip7ipo2ncESqQCmgVOdtAVLUgRTO7SeFvaPivCvxiz+8r3lfJ85cwhNzJiDH7cDew2c1DgyFs5mh5c3TnaXXIjvLgbMXmvHvrx2A1+fHwooiuNI4cDSFuyePxIkzl1DnbYWfDxnKhttlkxfIBI9zTxTdtJB4BHkhjF9WQNBAQXWU1h+Z6vqnv36O+eWFagScEhlHEagRp+PyBmL6Ld/EM6/tR8E3B+B73xkOhiaQADS18nhgah7cLhscNgqBlvabMtuA+fmQSncTft2+6iiwoOdOVmSbavs3IMuF8+d94Dga88sLdUUEX3zjEH5wa64m7f9ns27AxUutYGgKjcEA0hxp6J9pR47bgXf2ncbpczJFR2Y6hznTC7D6jYPqoSAgy6qNo7B8bgkIMd4Qfn3eh34uO7JilNtw3cuAYOnsG9U07e6yPSz0DtCQTG1VtOlaI6qYhRXFAJEw7eZvYkCW7AB7a9dxFOddiUlFQ9TNfjivb47bEZUKx9cS1PwtSdBEYIYEEZIIUIIoB1gIscs0RxOUTx6po1mw0QRBXgAvwJRnPbxYo+LA8vr8yHJxfW5S0RRlyMUdSYFBU8Sknd5RGk/b+O6V4Ku6Joy8ZgBoCgiGBHxV14TrR+Ro+o2LikySsxXD1wml2KzR9SPtICXqNlJszA5IIscgnsLz8RycsDQM5wdLQ2cTdtf76k4wtPE9Gx0G9FYwNGUojwxNQewD9jHHUoYyzrEUQgl8/u5yQA8B8FFubu4JAAHlQ4/Hoyf/sWDBQtzoKSdWgy+gU8LpdrbjH8aInuKmjcfASQWkcqSJmexyNEFNZTFq1+41jUqmKGDpnBvhvRTAktXva549K5PD7gNndAunUkBn5Sv7MLesAIMHujRVtDmWwuLKYiyPGEsHS0EUpISOcyq/Nwsdo6PonMgDN6/PD7uNxo9+cD0y0jg4bAxeevNTtQDVnOkFmDO9ADWrdqmRp4rzT9Fhf/voNEq+NQhXDkiHjaFBSZJuPoXLWiAYwmN3j9Ec8Ggc5T0/bBZSADRNocHHwx8UdDK+5/A5zPy3kXjy4QloDYSQ7mRxodGPt3Z9gUlFQ5GZzsHPC5Agqeuw55QXazYfQk1FMVoCQUy5aZh6KJjjduCJh8ajoYnHipdlh5bRQc3q1w/B6/PHxGNupHsXVxbjVz8qBR8UrcNACx0iy8VhRXUJRFEO0FCiijmORk1lMRp8fh1VzMpX9mJuWQGyXHb89f0v4GsJ4vaSa7HyFe3mHoDK66wcQoqQsLCiWNf2H/tPA2hPjx/gtrcVWWvX3i2BINIYOm555gXJkGYhlrUhWrHGvmbjKLQWkU4cO0ch0BrGAY3Yg3CM1nSZVoPonJ/x8DXbWAo3jxmsKTpWU1kMG0uBD3tnsT4TED2NP5KBIhBjtLgyBjMnj9SNl9EYBIPaCOzFlcZFl0OAyt+uXN+sBkw88yOe95UsiKfIYm+FnSHIcmmzDbNcNtgZghb+ct9d90M0kfFf/TCxxBbd5YCu6aZ+LVjo8+gpJ1bQhDd5RXVpQmkr4qlc31mYGzipmVbUU8UbE41osssLEja0GYHuDLthpEVIkGDjGLVgIND+7Etnj0e2O023cCoFdFas3Qs7x0CSoKYB2mgKIUFCehqL2rZ0c4qSZTHIC5BoKqHjnKrvzUJs6CjtU3vgBnx93ofVf/kUnlNeLKosxprNh1DnlYsVTrlpGGpW7cLS2eNR521F1bR8nXPj2Q0HsGzOBN1hjDKf/rH/NJZUjQdNAYIIfHL0HG4cPShKKq2UkvrQQvcjBGD52r2ommZc1NXl4PDa9s/xzr7TWFI1Dv/zwZeYctOwiGjQIvzto9PqOuznQ8hI5xDgBXx85F+qrAIEFAFWvCzryjpvK9ZtPYK5ZQUY2C8NX9U1gSJQI6ZDMcitke5drhSRk6Rusz0spD4Uu+W17Z+rBypulw0sS0EQoNouFXfkGepVO8egtm3uAFAdysr3tW32S8UdeZAkYN22w9hz+BwWVRZjx76TGrtVuU5x3pWqHfvA1Ot0hQw7W2Q7VooDw3HqoFhjX7Jx/Lyo2rPh727O9ALNOMQThBNuI2v7NHKUGvM1GxX2CwTNnKratrE+EwCVPklPY6CXgngihYOCsQ0deQiprFdG+l53BwS6tWrejEIYCWw88yOe95UsCARFkyKLBX2GrrI5IOITTx2K866EJEkghGDv4bO4cfSgpH1viURQMOfATiR9ebfsNTwez7vd0a8FCxZ6zollzpsspdxCRNMm6XApSsHRlU3C5UQ02RXainso/M3zZhTqUlkDfAgMTYyjo4k5n7PLyar0AxQhamGcjg5yEj3OqfreLMSGWKKZlAM3EIJla/aovw2X3fCia0pVcjPZvtQcMJhPpQCRUHr91ZqoJnlTJVn8txbihqK7Nu08ptPN82YU4lJzAO/skyMz7RyDSUVDDaJB96mHgQp+v2Ai/vK3Y7ij9FqNrC6bM14jn55TXixbswe11SVYs/mQ6szLcTtAUejQ0WbpXgudhRIhGemkUuyFQJvtMqloqCllhmKHAMa8zo1NcrLw2rc+w323j8KJM5fgcrKqTRSO++/Mg68liE07j8FzyovMdLsuo0UOFimJ21bvytrQUbHGvjTPBFE0fHcPTjXgS44xCCfcRtb2OdqQg9n4PRjzNcfSNp5nApEM1wkQvQzFEwUeMrnXyEPIuPS9RHRr1XMbDxjShTAmNCSMAQ1JPO8rWaDUHYm856ppfccuFEQRL24+jBc3H9Z8XnTdFUn73hKJeA6PuoK+MJYWLPQq9NRGiqGNlVAq8iZHo+BIxaTAVHUgRZPd8GfynPJi/TY54u2qbBfqG1rRGgiCoQkyKJvhs4sSTIsT+vkQ5pcXwsbR8LXygARkOLkOD3LMjM3OjvPlfG8W93T3IHxcJQD9XVxM0UzhfIq5Q9zISGuX63Bn84XGVswvLzTlKm9s0uYE1nlbUd/YisYmXo2oVj5XNlWMAZ/p4spi0DQBLxJLPizooOguRTcrBdXSHRzWbTuM+6eMxqLKYmzaeQx+PoTMdM70MFBBjtsBlqHx/42/RheB968LzVF1+bqtR8K4GQnEDhzQidS9RrrUQu+FIEqGByqKvaDIltnhzPptsqxmpNlAm9jVkgQ0tQbhOeXFlveOo7a6FBIkw7anzjVhzeZDWFhRDAkiGJrgsZljNE5pOVgEYONc97tSl6WjYo06TuNebJPEw5cc6zjQFMFdt1yLW4uvUbOa3tn7pQlfdBzXj7EtTVEm1zfigAa2vHdcE0m75b3jmDM9X/dc8USBR9fjsbbTjpVo4tQWRX3EJ0XDcC9J0dCtQam4TzO7Z6oPcUDHJee9EQQmh0eJvUyqBTJasNDnoSwQ4VAXtQSCZWS+L+VaarEFJvXURjDUXtm3troEVdPysW7rEQRDqbmiKtxike+Go5M7QSia7CobH+V7r88PlqHx0puH4LAz+PJMIziWwVu7jmNhRZHu2W12Cjv2ncS8GYWa7xZVFmPwQBcA4L/eOYq6i/LG0SySQhBlw5CmKfiDsrMjvL/FSuGhyGejKUg0hRAhkGhKjbIOR+Qzhm/uuhNKCnHNqt2YU7sDNat2w9sSNLxHC7HDaFwv+HiZ71mSQATRdEPN0UBNZTHG5Q3EfbePwrpth1XZVQ5SAGDd1iOwcTScdgYLK7Sys7CiCDv2ndT0qzilzSKmRVFLBbK6ZhJWVpeCZSn87He7LPmwYIhw3aXwNwdDItZtO4ypNw/DM6/ux5rNhzDrjlG4KicdWek2Q13v50Pq/+fNKMSHn55BRpreWb1h+1GdvC+qLMaQK1xgaILKO6/Do+VjcEV/J4JB0VTnGt1/eH/x6l4zXSqKyelQsNB10BQxPVBRAj8WVRbD6/OrB+d/WDgJj5aPwfptR+D1+VFTWYx12w7jmVf34yf3jtXI4fzyQmSks9h7+Cxy3A6UTx6Jt3Ydx9PrP9bZH/NmFGLTzmOo87Zi49ufg4Bg0ardqFm1G2s2H8J9t49C7hC3GiwS77qvrA2/nn8zVtdMQm11SVz0fkqxxo7mWW+3SZT1XW+ja9vFMw7hXM0PP7UTy9Z8gJvHDIaNNWprvEewsfo9Qqz36rQZX99pM7Z1K+64TuUlZxmCijuuM9W3giCCCGKHdhPdViAucv7QEXufePZIlMm+hDIQRT5ovJfkg/r7vVz2fldA2pyPkTonwcGvSY00EzlPM5Dz3giatB8eKTK+5b3jSLR7gUhSrzWargFw4sKFpm4xDLOzXZjy+Oa4frPlN9P69G+2/GYazp/3xXWNSGRnu7rcRxev3xNq+BpEkd2e4oCWGBp8SABDUSoPUkgUwTE0SKoVI2Bo/OH1g7rKyQ/dVQCk2rMAkGgKq984pHueOdPzQUxkIBVkV44EIWoEpxLNk+N2oLa6FDWrdqFqWj527Dupe/YHpowGQ1MIhEJgaVqOXqAJtu76Aq+/+4UmGslzyos1i2/FolW7dSf9tdUlIILs0KhZtRtulx1lE4ernKXfGJQBkddHsDQHBV3F7TRWXwTockT9KM9i9qzxoKd1cA/JLdAJm6Gr48pyNHgBqFm1S42ELps4HDn9HIAk09W4XXY8MPU69M90QAIghCQQSi4QyzAEBJSmSJUi42UThxvOEyMdkUj5SDZcTpshGXRuIqCMoaK7RFHetIuShK/qfNiw/ajKx5zjdmDlI6VgGIKLlwIqj7PiAMhM53De267fyyYOx9ArXPh/YRQCSj+P3zMGfFDEgEwHWJaC1xfA/37wJYrzrtTMkVjtIGW+CaIImqLA0dAV5OoIZnPl1/NvRsgfjHtskxW9RXaBrusAmqYQECX8/Pd6GVV0pHZdp/DWruMY9Y0BcDlZZKTZVF5nAFg+dwL4oAA7x6hRy16fHyuqS8DQFOobWhEMifC1BLH38FmUfGsQrspOx5dnfapNBAC/fGg8fvfnT3T3pBQ97J9pw0+efa9Ter0rYxaLjdPZNSeW+0oW2VX1jSSBJsRQ38QzDhJNq7aCtm0piKDv961dX+iiOO8svdZwfGPRjfFcn6YpNLQENcW9F1cWI6uL+1SJkdcBXzOv2tquNJmTnYQFFMWzR4pnTx3v3iseez8Z5FZiaLz13nG93Nw0LPX2/Z1EPHLeG0FxNC40BvDUuvZClAtmFaF/pk2391XQGdlN5oMYCxYsGCCedKWugKKARh+P3776saqEfnz3WAzsLztCUgk0Q3Df7dfhvLcFgHwaf9/t14FmCIQUoxMBUpNbDOhYdgVBhEAIFjy/S/M7t8uu8tS5nCwafFragQYfL8skkXCxMaDjabxl7GDUXWxVnc9KdFC0VFOFLkRJtVWwumaSZoyVyOfW5oDmnlr5EJx2BjS0BigAOdID6LHCVxb/afegs+OqbEp4UYIgtnOCek55VVl75tGbsaK6BE0RG6N5Mwqx5b3jmHrzMPBBCVcMSENtdam8yaUIVr9+EJ5TXuw9fBYzbsvFylf2we2yo3zyCNw/ZTRIGwd6+HphyYeFWKBwldIAQiLBhUa/hscckOVGgoRQCHA5OVU2vzxzCb/fdBCVd16HmlW71fYuJ4v6NpqZ8LTmmspiZKZxCIZEnKlvwobtR+H1+bGwohitgSB8zbzG+abo6ZXVpZBoSkePIdEEF3x8lw/uzeZKMCT2iQJFfRGCIMLB0dHthTAe35Ao4vV35YNvAKitLsE3r87CQ98rgChKEEWgqZXHH/5yCJ5TXuQOcaNqWj4IkQ8Wf7X+I42+//vHpzFz8kgNpVLuEDcGZDkMZXFgvzS8svUwZk/PN9HrAEj3US3FwmncF9YcsY1mgaYIIEmGNEHxjEN8vM4SPjvhVQ9BfC1BfHbCi+9OMB7fIC+EvTMBQcN7FeF22TW0Gpt2HjO8flxFABGHo1aSMxknFQ2FnQOCIQn/9c5RHbWHaLJHqpqar6PVCN+XgMjvyuz6HE1QPnmkShkVHlltNGapBoYmmFQ8BHXeZtXBP6l4CBg6NffKnUE8ct4bwQdF7P/8HJbPLVGDD//+8WncMubqxNK8JrAvCxYs9BBiLVrRFUgiVOczIBsQv331Y6x8RF+YIdkhiUBjU0BXJTzdwXb84yREKnKLKehIdiOfLXeIG7PuGIWvzzchx+2AKEmY873RuNTGfcsyBHO+Nxo2joIowpCn8Yk5E9TNm7JxJIIU1RkeS7ERJXLC5WThDwg6+QIIvC1dd3p0FaksL8mMzoyrIjNKUatgSDDs43yDHxzL6HjKn9t4AD/6wfVw2ljURkQ+nzzbiPLJI3HizCUU512pOp/vu32U7lAmy8WBCDIdhyUfFuIFTRFDXvJxeQPR1BLE8rDIZ4UqxnPKq+PpVyLIvn/rCMwtK0C6k0Nmug0trUEs/I9dusj+la/sxdyyAlzRP83YESyIajR1jtuBpbNvRDAo4qLPr+pnpW1nijebzRWWoRDqIxFifRFBXug0T216GosJ3xqEs/XNmkOWn957A/73wy/x7bGDVb0eeajy3MYDeGLOBKx+46CGl7N88ghTnvSv6pqw5/A5VNyRZ/j91+d9WLZmz2WzR4zGSLm33rLmxBpVG09RO4Y2aUtTukxOjqUw645ROq5ijqVMoxg7AsfSJn3SEHmtd1KUjIttihJ0DuB4IpAVao9zF9uDiRRqj/CnokzGlYrgilag7EuUCHuzEeIFSVevoNZkHempbOVEQpKAxibeYK/MXe5b6zFwLG24xzSS894Iu43G2FEDsfiF3Robzm6jEfIn7vn7BqGJBQu9DLHwzXYV0U7mUw0hQVKNJqC9SngoRQ3dVOQWM0OkLHNtkcnKs5VPHoFnNxzAhu1HMW9GIRw2BsGghBc2HUTNqt14YdNBBIMSfG08nEYy62vlMbesAKtrbtVwGkbjnVOKjURyzVFh1nMIcgq4KMFUvowKHfa0CdOb5CWZ0JlxVWRGKWqlyHWknCnF3IzkeUCmQ3U+K589t/EArh8xEBu2f46qafkYMjAddd5WlE0cbngo0+IX0BwUQNOUJR8W4gYDYGA/p05HPjh1tOp8BmR5W/nKPtx/52jkuOVCbQsqbsCSqnGorS5BuoPFfbePwu5PvsZV2S6wNIWv63yGBy9lE4ejztsKO8eozrdw5LgdOFPfBLfLjkWVxXhs5hgABK9u/xx2jkmIPWM2VzLTbPEPooWUgmIv2NoymQKipNrf4XYMAcHS2TeqMuLgGNRdbNHZCE//6SPc9e3h2PLecfzoB9cjp5/TUEYbmwPYc/gc1m87gsfvGYvfL5yEK/qlGa4dCyvkQqA5bgcuNLYari0bth9V+74c9gjQ+20SZZ3vyPaLxc5U27YVU9e1Naj/IwrGNmlHxVqjQZKM91FGVK4UMeFVNiATDgF4d/9pLKnSU+yMAAAgAElEQVQaj98vmIglVePx7v7TpnIZDIoR9r+BM5dIhlzGIF3b88WzL45VBpIJvW2v3BkQwHCP2VcynIJBCRvf9mg4oDe+7UEwmFgZ6C263oKFPoOeOlWlTCIUUrEarpljUpSklDyFiydlLJlhJsv9XZwaaQTI76rOK1No/PS+G7AojJ9LicxfPrdENXojZdZ7KYAVa/didc0kOeozhnsLLzaipGGt23oEj98zRl04FWNUEEyqaEvJkWbaU7Q9fQ2dGVdFZpQigYpcK3KW088JigIq77wOdo4xlGeOpXXpgZ5TXtAU1LTTp35Yihy3w7QYoa+FR0gQ4cxOgxhHdJ8FC4AsG2ksDWd2GlZUl0AUAYYipmnivCBgRXUJQoIEigB/fvso9hw+J0cpz7kR3x47GF+f9+GFTQfx2Mwxhn24nCxy3A5V5o0oEd7a9YUu4n/ejEKEBDEhEZdmc97ICWSh98HIZlGi7CP5bn/1o1LwQRGiKJkegHAshSk3DcNzGw+galq+oYw2tkXieU55seD5Xbi1aDBmTh6pFj1U1gI/H0JrIAivz69mDABA1bR8XHOlCxRF8PT6j1UOaeUeLgftRW+3SWKl1ojFzmxvK8Tc1qzIdqgLNALBkHGfwZBBn20O4Eg9bOQApmmCmwqvxrI1H2giLmma6BzmsVJ7EImohdSUsdry3nE8NL2gk0/fdq9xRO6nIs1Mb9srdwYhUTLM/q59pLRvOKGJhO/fOgK+5vYI8O/fOqLLhzeRSNY5YMGCBROYnarGm0raEZRquJEGRCpWw2VoY6OBSUFnuoJYU8aSGVFluY0nWQpLO/Sc8po6OERRwpothwxldv22I+rhiYTYNjo0RZDl4uDOsCEr3Yb+mXb8+O4xmjQkxRgFzOUrWdJMe4K2py8i3nGlKYJxeQORkWbDUz8sVYuxrVi7FzluufiQKABvvPtPTL15GBZWFGuKDNZUFKOVD2ooZRReaEGEKm9ulx3zywsNaRIUp8aazYeworoEVCeew4IFABAESS74ShHQkCAZUCg9MPU6iIKkFn1VZLbBx8NzyotzF1o0judImg5Allk/H1L1eZaLQ0YahycfLpGv3RYxWPKtQbqI/+c2HsD88kLd2hDO4RvfM1tzpa/CyGY5d7FFR+8iO8Vkujqaokz1MCHttGGbdh4zlNHXtn8OAGqR2sx0DjRNsLiyGMvX7lXXjkWVxXClcZhbVqDWuwCANZsPycEKALw+v+Z57rrlWtAUhVAXCnN2Fr15HtEUwV23XKsr5hZp+9EUkQtQhtUZMbMRaYoyaaun61DsjMhieV2xPeOiTTFxAM8xcAALArDylX26rJnaar3Dz9RBKmqpPSgamHrzMB1dCEWjS1HgSuS+GRd8OFKRZqY37pXjhSCYHBwIyXtwkEjQFIUAr6eUpCkKYgKFoC+MpQULvQo9daoqiTA1IFLNB00I8Pg9Y/Gb/2wvqPj4PWPl4OHLfXN9GLHIcqTBRwBDA+lMvcx72ODjUTUtH5npHFxODs9uOACvz4/55YV4ev3H8Pr8MWUMcLTMLXepOajhwlIitIO8AAbAk3MngKUp1FQWawqTPDZzDEBiN1Yt9A0oRWzCo30UB/LMySPBAGA4Wi1043bZMbesAFcOSAdDy5pXceQB7Q62FdUlaPAFUPvDEgQCIgAJdhuNnH5OnQwqTjxl49ZXIlssJA7RslcW31+M5S/LsjvrjlFo8Yfwm//cr5PZqmn5WLF2rxodqjiezRxxgWAIf3zzM2S5OMy4LVfDEb2oshgZaSyuyk7XrSlulx3ZWQ4QiqC2ulSO5qKIvKnuJRGXFnoGRjaLWXRzfWMrFjy/C7On5eH63BwsrChSHW2KHm5qDaq/9ZzyqhHNQwam4+yFZlBt68WgAWmYfOM3cKk5gMYmHn/52zFU3HEdah8plR0jbQdAkCS4XXZkuTiZGiadg9tlA0cT+RAobC2465ZrcfOYwahZ1T6PaiqLMcDVd/heuws2lsLNYwZr1vmaymLYWAp8OAc0YrcR7ZzezqypLIadoxBo1ba2sZRhsbzI68cDjiaG1zcqwMfRMCnWB11b8+KK+j1trNzOfFDE3z6SaT2UA4DX/34MP7h1RJf2yfFE7sfzbpMFhAA/vfcGPP2n9kKoP733hj61V45+cHAZb6yHIJjQsBgdCHUFlgPagoUUQ4+dqhKoqYHaFKrEXQKIo/pxF0AIQbqTxdLZ40ERuUCGBLm6a6ouqsq41XlbgDYe11RLX4xFliMNPo6lMb+8UBPZUFNZjBZ/ELXVJRpKglU/m4hHywvxr4vNWLe1PSIolowBXpDQ4OOx+R//1BzCvLb9czx0lxzFQdEEzb6gzlEICfjfD09gyk3DEpZm2hPzxEL3w6iIzXMbZeOOgVwUMMQTtY3bZQdDU5BECaAIaIbA7bJr5kydtxW+Zh58UEQwKOHcxWbs/r8z+M4Ng/HUK7tV2byifxrqG1rV6Dg1siUOo9qSQwtA9OyVzHRO1YW/+MP7USk1AKjRoeGO5/XbjmBuWQEGDUgHy1DgWApfn2/CT+4dCwLgxc2HdHp5UtFQsIw28k+UJNAUhZ//vr0w4fzyQtg4Gv/1zlHcPXkk+rk48IJkybSFDmEUWUpRxofiCnXGi5sPY/Y04Mb8QRqZXb/tCMomDtf81nPKizWbD2FuWQEy022wczSOnfTiljGDsWR1uwz/bNYNaPDxyHIRnLvYjIH9nEhjaQiCiP4uTi3UJooSvj7fpH6vtUco1fkMhBdUS71C48mGQNCsWJ3WiROPQ9PPi2qNB0WGNmz/3DAoKNbrxwNekEyun6/rM8gLGODiUFtdCqGD6HraxKls6PAjwGMzx+CZ1/Zrgj0IBU1bjqVwa/FQzQHAj+8e26UijPEiFWlmCCFwOphetVeOFzRN8OO7x6o0HIrsGFHC9EbEcyDUFSSVAzo3N3cJgB+0/bnV4/H8LDc391YAvwXgALDR4/H8/LLdoAULSYCePFU1jIC+q2scWuHoKT5rAqDBF9ClY6Xb2YRdoydB0xQaWoNqsacctwOL7y9GliN5qysbwUyWOZqAB6UabQDk4j80BRESMtNtmFtWADvHQJLkaI/atdqDki3vHcdXdU1wOVls2H4UZROHazhzO1pMBVFCTj9H1EMYXoBq5Nd5W9XK8nPLCnBT4dVgGAohf1CTZkpxNEKgOzTKw5GK1bQtGMM86l8EaSvmoxiAuUPcqLzzOs1ma355IeZ8bzRW/+VT9UBlXN5AiBLw7xva2y2bM0F1WETKpuJ8XlxZDJaGZq5F2yBZcmhBQbgcK9QALicLCXKRqGVr9qC2uiQqpYbyuSuNU50K67cdwaPlY9A/044z9U347av75ayV+4uR5uCw+IXdqKkoMtTLaXYGW3efwIzbctVI0yVV4/DCJn00z9yyAkwqGorla/fiyYcnaBzUMpUBC0hIeoeBhZ6FksFSG6EDl865EUtXf6iJGuRDguZQfNzoK1XqJAU79p3URUbXVBSjJRDExrc9mHrzMIwY2k/NwgLkiP4AL+jsWWd2OkKiBAoEXl9Al0LtzE4DCQtSMXc0WPLeVUQb20i7M1YqEkEU1RoP4XhwqkGfooiCbw7AXd8erokANro+ENvBsiBKcDk5XJWdDpoCMtJscDm5LjumWJbo5sDCiiKwLEEoYjAoioDjKNX+9/MhcBylc5CKIgx5fFc+0rXDlXhtoFSjmelte+XOQYLLyWpkTD4s7xsueMbkQCjeYJUOr5O4rrqGNkfzZACFkN/y/+Tm5s4E8BSAWwCcBrA1Nzf3ux6P56+X704tWLi86KlTVRtLYebkkbqF1sZS4COtgk6ip/isgyYpJSuqS1My/VwgRHU+A22cgy/v7bJx1dMwkmWOJrjg4w1Tu5XP3S47yiePgDvDDoYmhpQEy+ZMwL+/th8PTL0Os+4YpTOoOoqE4FgKggBDPlElisRsk2HnGKx8ZZ9OvliORr2P16UlDmij9DBDT80TC92PWKL+aYrCuLyBmHV7nhrBA2idZ+WTR6hO5co7R6vOZqXdpeaAoWxelePC6ppJUeea2WbKkkMLChQ5drvsuqJ/Tz48QeNgNqLUWDBLLjK1bM4EbNp5FKfPNWFuWQGuznGBEC3NDACseHkvHr9nDKqm5SPdyaE2gjP0uY0HsKRqPIrzrtTwiZrRI9g5BnZO/r/XF9DJdNW0fKzZfMg6YLGggVEGy4q1e7F8bonGYeF0MHh/z9d4/d0vNDZH5IH79Fu+CYaRHWpXZbvw9Xkffv+Xg+rh4okzl/DkwxM0Mlw2cbihPfvLhybgoZU78IeFk0xTqH1hzrMlVeNM1qJUtIqTC+ZRvXq+5u7o025jcHvJN3SF/ew2BiF/UNtvjE5Vu4026bO9LoqCeGxdf0DAtt0nYqLLEEUJT73ykW4MIgvEhUwKg4e6yOPb222g3rZX7gxEEVi25kOdjKXa/rqz4FhiHBjGEiQyeSCZ5OksgMc9Hg/v8XiCAI4AGAHgmMfjOeHxeEIA/gTg+5fzJi1YSAYIgigXaZMkEEHsls1RICjitbZ0q9rqElRNy8dr2z9HIJi4a0XjAE4kzAtXpOaJZsikSEIoSQtbREOkLPOCZGjg8UK74ec55cWyNXtwtr4JFxr9hmPR1MrD6/PD7bIbGlQdpVKJAiBI5hWhgfYNQTgUx4uRfIVHTCt91bY9W9QxilJ4RaIphAiBRFOg6WRa0i0YQYn6V+QmPINFbcMQzLgtF02tvKnzbGC/NKz62UTMLSswbNfYxBvKJk1Ih3NNu5VsR0/pawvJD0WOyyeP0B3SvfTmp1hYUYQd+05i3oxCeH1+rN92BEtnj8dTPyxF1bR8rH79EB797btYsvp9FOddqepqmhBDne522cEyDNZsPoT6hlZDOeRDAq7O0XJAK07wcCgFDZXvFKqE8L5cThZulx1enx8BUbL0qwUA0XVgMCRh7VufYdmaPVj24oe4tfga9ftnNxxAKCjB7WSx8pFS/GHhJMwtK4AoSXjypT1YtmYPAAnL1uxRnc/Kbym12LEMl5ONqofFKHZLuL7fsP0o5pcXatYiJfLUQtfA0UBNxDqvcCB3FkqkcCzvKxgSDQv7BUOxHyxH2gHBoGTcZ1C//sdj69IUwcF/1uORp3fi4ad24pGnd+LgP+vV7MdwxLqXoyPmDBB20N8F9HYbqLftlTuD3rS/7gwCQcnE95PY50+aCGiPx3NY+X9ubu5wyFQcv4PsmFZwFsDV8fTbv396Qu7PQmKQne1Kij5SAZdbds+2FXXTp3tJuDJB78Dr8xue6NttDNwue0KuAQDnvS3GKSU0QbY79eSpzttiXGE7SZ6nK7Jb520xNfAiP7dzjOpoi3y3LieHqmn5aGw2duJJBMiJIsd13hZQMOZ2pCiC7P4ueH1+/KJqHM63OQX9fAiuNA4vvv4pctwOsAzBgKz2a5ytb1b7ikxbH9A/XS6kYgBlnrhddvU3fj4EmiFY+Hx7AaGfPzAOQ6/IMO0nUejNOrgn9G5WloRfz78ZwZAIlqGQmWbTvLPz3hasfGUfqqblG8qfJMlpkq9s/Qyz7hiFphb9HDBK7Z43oxCggOx+8vsLn2vh8ghC0N9AHi9e0uvrcXkDZY5qwPBZkgG9WV4VdLfcGo1hVpaENK/eGbbn8DmU35aLSUVDkWZnsKRqPFoCQTA0hQXP79K0rfO24ppBGfj1/JuRmWZDfWOroU4vnzwCK1+RHRpmlB52jsG/LjRpvtu08xjmlxfizX8cx6SioWpRNoalsOXd41h0fzEAYPncCRBFCQxNwc+HwDCULrK7I/3aF+SsO9ATOjdR7ybSZs0d4kb55BEQRQksQzD7rtF48XWZHin8vEKxOfq50wAAwaAAhqEgCCKqpuVj085jEEVjewMAFlYUq/KvcKZHtvO18Jq/I/U0RRE8NnMMfC1B7D18FsV5V4JjadRWl0KChAAv4vW/H8PMfxuZ0DFLNJLpvqLJbnqaHSsfKUVIlItEul12MEznD7HqvC2GkcIz/22kzpY9W99kbEcL+v2bmc0NQjRjHW6/avqU9H3G01YUJfz8gXF48o97NLp2QJZTp2vrG8z3cgP6t/d7sbEVCypuwKUmXrXNM9I5MDRBv356+RFFCY3NAdR5W8DaWVM7pqf2rN0NM7ntbXvlzuDcReMxoCii2s69GWfrm419P9MS5/sBksgBrSA3NzcPwFYAP4V8MDci7GsCIK7wywsXmrrl5CaZFsBUwvnzvi79Pjvb1eU+unr9nkI02e2JQlA0TeuKrezYdxI0RRL2Dmhan5K4qLIYUlBI6Htm7Kxh9WaGpS6rPHUWNgdrWmHb7HlSRnZp84Ikyue3Fg3GXd8eDpYhoCh9UZIFs4rwzKv74TnlxZKqcYZyDEmK/u5pCq2BkC51fN6MQhDIuoziaPgDgoZr8bGZY5Dl4nD/lDxQhKChsQW8AJXz2SxtPVqqN01TWDr7RngjuNl+fPdYtSBdnbcVT/5R5l0l3Zgu3tM6uKfX2u6yGYxAAIRCAi5EpMQKhKDO24pNO4/h8XvG4jf/2V4MZX55ITLSWbicHB6/ZyyaWnkMyk7X6dEZt40ERUn45cMTcKmZh/dSQObwn16AixebZTmj45NHitMWAB2XNxAzbsvFwv/YFZMcXw5cTpshWXRuVxFtDKkwfa0cYmSmc0h3cti6+wTe2XcaQFuadHWpsW4nBFJQwIULTQBNqZHT4fJ45YD2yGYjSo9FlcVY+9anaPDxmu+8Pj8y0jkdb+/88kLcNm4o3t5zEp+d8GLWHaPwuz9/ounvv987qongi6ZfL7dtmmj0FtkFEvtuwm1Wt8tuSO+lfBYuJjluh2pz0DSFhpYglofJ47wZhXjvk690duqiymJs3fUFbrjuClRNy4c7w4YBWXZdkaz55YVwu+yorS4BTVOaOaDo6ZowPb2woggb3/Zgz+Fz6vXXbzsCr8+PmZPldxHvmPXEviSWd5lssntl2z17vc2G33N2BoGgpNqINpaA9xvkIdEUfC08vj7fpNqyvhbe0JaladrUjtaNH03FZCOb7gmJvk+apo2DZAzaAoA7jdUVLLxwoUnXjnWwmsMYWZaLwTDavQ/F0aAIpbHNayqLIcForGLnde7OPWsyyC3nNN4rc1xq7pU7A4ZjDIsQMnTifB/JjHjmuYLOyC6RpOQJKc/NzS0BsAnAox6PZ0Nubu4tAP6fx+O5te37+wB8x+PxPBBDd9cAONGdDugpj2+O6zdbfjOtT/9my2+m9QYHdE+Edl2DKLLbU4WgnE4WZ71+3UJ0pduOlpZgxx3EiJ4wWiWaxuo3DuoU6pzpBSCd5GO7nOjM86SK7Jq1UTig391/GjcVXq2J7FxQcQNaWkPgWBpulw1/ff+Eyr345NwJaG4Nxc27TNMUAqKEl9781GCc80EEEYShseA/dumM/CVV4/HcxgP4yb1jIUFCKCSBIgDDUGhqCcLr86uGcfjvojqPaRoLV+mvVTUtHyvW7lU/W10zCUwH63pX5txlcED3VDjtNehGmyEuhL3r5XMngA8KsHOMWszK6/PjyYcn4NA/zyP/m9nw+gIICSL6ZzjQ2BxAYxOPTTuPAZAjRwf2S8O5i83I6efEO3tO4pYxg+F2ykVlvHHIY4gQ/OY/96uR0hlpNg1HtRINKBcpSo6K75fZAX3ZdW4iEG0MFX392vbPdYUBF1YUYdvuEzj4z3odj3+4423Le8cxc/JIjUy+tv1z3Fl6LXL6OUEA1Df41SKbQLisuSCKEvzBEB797bvqd4qM9su0gwBqkUEFOW65IGcwJI9ZZHE4I90KtOvXSB06IMtp6DBJVfQW2QUSrwNYjgYvSoBE1Ij9TTuPqcVdf/nQBLQEQvjH/tOqHaKxc6Ks5UdO1OOO0mshCBJomoAPhUARGn4+hBf++2Db4fZoPPPqfl1h5Qem5mHB87uwpGoc/ueDL1W7JVJPh19PkW9lPrhddridLPr1S4trzHpqXxKjAzqpZDfaPXN2BucbAzr7NDvTpnNCx8OrbHeyOGewfxvotsMfsX+LtV+bg0Vdg77PnCw7Aq3aPuNpS9MUmoMCzl1sUaOVB/ZzIo2ldbIj0hRefOOQziafPb0AVNjeJ549kkRTqImoORDNHu+uPWsyyG1v2yt3BjYHi4ZmHnVh8pjTz4msNE4nu70R8cxdBZ2R3aSJgM7NzR0M4A0AMzwez862j/fIX+V+E8AJAHcD+ONlukULFpICPVUEoTkgGnJ4KQXYEoWeqBIsSpJhSknVNAldoGS7fCCSbrM/b0Yhkr0KRiyya1ZkM8gLcDtZ3Fk6DDVhm7c6byueeuUjrJhbAkIIOBq4s/RafHfCN0BTBATERI6jzxdBEOHgaMNCnAzkYsAhE760loDs1KMIQWMzr3GW/6LqRlyV7TJOT4xSTTxkUPDQ7bJjyBUu1FaXaDIUEIWrrKc2ihY6D4qGGmlMEdLGD6rHNYOyVMdajtuBZXPGqxQHuUPchlHNShEqZXPldrJw2o0LtSnyqGy4IEoonzwCG7YfheeUF7XVJRqHYDxR/RZ6BxR9PWd6gU4vr3xlH2qrSzFzsqTR4bXVparjbv22I/Cc8uLEmUsamZwzvQAhQcSZ8014YdNBuF12XWSz22WHgyUQQMAynBrx5znlxYq1e1UnmzvDZijfSiFChqEMv++f2Z5SrTi8ATm6rsHHayJYf/7AOGTaGUvW+wAEAP+60KKJfFYiiD2nvCCE4LMv6nFn6TDVDgl3Uhmt5XXeVgy9Qo4ie/GNQ5g9PR9+XsATa/Zo9GmWy4YTZxrh9fk1hyPhXOYbth/VRGY/9cNSw+u5M2yav6/KdsFGdc6Z1tuLs3UXAkF9UUuzfZZRAUwzW7Y1IGJDG4+r4kjcsP1z2ZHYyX79fOx7wnjaSjSB92JAE608v7wQzuw0RG4IRQmme7lwEJM9EjEQxmi8zkb2eE/sWS8XBJO98oPTula8MZXg50Use1FfhDDRvo9kRTxztytIJnn6CQA7gN/m5uYqn/0eQCXkqGg7gG0A/vty3JwFC8kCQZTgdtk1hsWmnceiOq86dx1jI1kQxaRSHLGACqNvUKBwOsVH6pMsILrCT89tPIDaJK/SG6vsmhl4giCq9AThqPO2IiSIoCiCoCBpfhsyaS+Kcvp4tCgGUZCQ5eKwovr/Z+/dw6So7n3v71qrqrvn0kArMGK8oEaHi3AyKkOQUfdWwzk7GtDNs8OYi0ziSOKYSBKzI4PZQTyR0bgTt+Z9NVHyBjEJuM9xG6Jw3uDlaJQkDAL7qIijUSNolGsDPcz0parW+aO6aqq6qnq6h7l09/w+T/LIdK+qWl3ru1atteq3vmsupAQEYxCQdjrO/HVVE1GxtLkBQjDXpi2xaAQH4j04vW5M4PLIoMnjkMqxonW2HQnbuesj/P1Fp+MHP++bgDQ3uWHI5OkR00Cx9MiNqNENibUbd+POr10Mxvx9QYXguGete0Ogjw8dt9MuvPxcTxuxak0nVrTOMW0RmBn5kzKkbQ3j1z5ywT2TbcsWzwIDEA4pmD29Dlt37Qu8HumqsgiK/gpql3XD3GA2tw3384LWDYmqkLAtiw4fSyISMl+O7I/34rFNu+1nx4RYNRQBZHRACAYwA3fc+GlXFJ3lx7/w8nN99Z1Ma8hoEp+YUOv7/djakG1R45zQW9E627ViYH/cYc8xFDedGBKE4JCCQdOl6f/Nuev5HqR1TZeejY0feHwnWhfMwOoNr8EwJP6w82+YNe0UczWSLiEiCjQwlxVXrt4MCTuS+qvzz8edq//saU9XtTVh/ea3fK3BHtu0GwDQtSeOtRt3Y1XbXByI9+KkMRHf61WHVdffooDJ58D6X+Qk3mjAstb46GA3hBC+1hq6YQT0ib3jrGLGfrphBOzhM/DzFjMmDE7rzatffbp//U6sapuLXMdswZmvPUDuWE7KgDGSzySaCBgf9hfMUYnkvRdlOVYunkqa+xgIw/X7S+ZednV1LQWwNODr/zKceSGIUiakcl/fuZDKYeSxFCiWoE6y4Bwos6U4gjNfXyshGAwfq7VSJ99OxaUc0T0Y2s03eFN9OoxBHSoJifYH+yZvC7UCsZaJAwAYPAPBZYsbURUW0GXI9SB3RojGohGXl25uZLXnNwuOI91pV4TInUsuxg8e/qOrg92xphN3t+V/CUEDxdIiSGfjoiHs+TiB57a97/E4b1/cCMBbjus3v4X2xY3oeLQT0Wrv5nD7470Q3NyQKnG875qzp9d52selzQ2497HtaJ53nmeyzdogcfWG19DeYm7iFnQ9w4BvmzQc1kvE4JKvTZQIGrh6+wtBbXJYFTiYMJejty6YAVVh9ne5kc03LZwJVRF46qV3MP/Sc/C/X9mLq+ae7fHjB8xJve9+6UL866+2u+rQmFoVf37tI0wYF/Gd0OtNafYGbcsdS7StSXEn1IaWF9ay//hh974Kt7c0YpzDCsZP60Ern844JYo7v3Yxfv+n93D9VVMRVgX0tOaxWbj2srM9PrK3LGrAmqfNlyWrN7wGSPheI5PREU8kXS9jxtaG8ejGXejaE7fTxhNJMDDct24H2hfP8tV3Jlsv++t/OO9ZcP0PeFE6CifxgMKtNcKq8O0TW9pxnbOI/rMI2ktFcEDTB3ReVfE/p6pwyExuGx+8l4snqjlwPAPPBHRIYb6rEkMKR8bxuwwZcE7pHSMpgK+vc3/1oRJhPmOaoMjxSqWS5j4GghLQdig+bceJMPDtWAmCGBEMHb5vi41BbhfDIW76/sTMXbitDlQ4VJ7NhqIw3HHjHPzststxx41zoCgMKNN+MctG3jqZGKsCK/FewmBoV1UZli1263LZ4llQFOY7+Lc6l870y1sa8Yvfve6JLnJ294MihZ1pGIDtuz/GyiVz8PP2K3DHjXOwacu7uO3/eVv6wfwAACAASURBVBm61hdZCsAVIWpFKN20cCYebr8SHW1z81oVaAB+83tzOWVH21y0LpiB48mMfyR4P36E1uSPE3tgQAw7QTpbcu0M1Fap+MrV0yGlaclxzzeacOeSizGmVkUmIz3lGE8kEa1RcesXL8SEWJVvORsS+MrV57uuuXXXPqzf/CY62prwcPuV+FazOXHXcvU01J1U4z/ZUleL1gUz7GW9Qdfz20nemshof3ALlnQ8h/YHtyDekzEHx0TJkq9NDAn49hdCPm8fgtpk3ehbCv7E828jWhPCmNoQljY3uNLesqgB6ze/hQce34krZp2J+9fvxLV/dy46HnXn7b51pk9uPJHEyWMj+N6XL8LPll2BW794Abp70zh4JIlZ007BuDFhPPXSO672dfvuj1EVVmBIiUzGHRGU6Mn4al3hHFJwaIxBCg6R/X/uZ8TIowHYd7jH0x+5K6vnXK3HohHEE0nXihEn1t+Hjybx2aazEQkp0AzTKzzXZmHqWeMBBpfeHtu0G1t37cPYrN4PHjUnf+rPiGF5SyPu+9alePB7l4NxhjuXXGyuzFrTifvW7QAg8YV5Uzz1KawyLG9pxJHulEffT730DlQhcM83mtDR1lSQVVK++h9Up0frC5kga41Uxt0/0w3/6F/dpx9XTP9ZVTiWLZ7l6Serirf9KfS8jDFPW7y0ucF33KGq3Kef3ghV9V5fCahPik/fIaNJrNvs7guv2/wmMppbu9bKsdxz+lpwOGz/fvH9z/TbH6/kNt0wzDHNilZzrLyidQ627/4YxiiKDTDHmD51Rx0dYyTB/ev5YI8RR+uzgSDKliD/OG2Ql0f0JDW88sbHuOumuZBSgjGGF7bvxWUXnFaWDcex7rTnDX+0KjTS2RoQqmCeCNqlzQ1QBSvpF7SDod1kSsemLe9i5ZI54JxBcIajx1OmdYHm7SX5eUqDwbM0MTeCrZBIYc6BSxpOx4qH/+SKFti7rxur1nTi7pub7MjS3AjRrj1xrFy9FQ8vvxJMN/JHWjB4/Ozu/NrFAREuDMgT1U/RHqWFn85i0QgSxzP2pmtWGdVWqxCC4UgijfWb3/REqixvacT6zV14dttefPOfZvqu+tClgeNJ3XPNrbv24Yb5hrkqREr7vCtaZ/vqbM++bqze8BpuWdSARG8aD/3PVz1tkulL7x1Ikw1MeZKvTczoBsZHQ+hoa7JtBkICvhu9Bvn8p/S+50PXnjgeefJ1XH/VVJw6oca2E0j0ZGyvXaAv8l4IfwuQyZPG4M4lF+PY8QwOH0vhfzz3Fj53yTn46b//p2PVyix8/jPn4Z5HX7FXBDTPm2JHPefWgSeef9uj9e9/dTaSGQ13ZL0jrWhaVeWuz8gXvTTQDZk3kt36N+D1tw9aMXLfb3YgnkhiaXMD1m7cjXgiieUtjYjWuJ/90WoV+w/3+m58OX5sFX702CsAgO9dfxFSaR2/+8M7+Nwl56DDsZfE8pZGLLl2BiDNZzqPKPjh1y9GPGFuQrtu85u4bt4UnBwNYVw05IkavWVRA9Zu2oXPXXIOwKRv38nvngXdLyalb50erTovdBm7pgf0iXVvn7iY/nMypWHTlvewonWOaa1iAE++8DY+f+V5Az5vOqNj7cbdLquOtRt349YvXuA5pzQM1FQJ3HHjHHBmejcLAUififVQ9kWJJ6pZZch9fEj4eztLuLfAUQUPGCNxX01atn/WhpFBfeFK30MlrHJc0nCavWmp9XwMq9xeMVHpmGPMwupOJZIqop6fCKPhXhIlQjqjY8KEaFHHJFMaEsd6+084ihguvyrBGZ7p3INf/77LdZ3LLzq97JbUBXuMNZXlMhAJYExtyNW506VR8gHdg6FdwRle/ctB08fWcY6OtrmBx1idy7C17F+XWNE6295MzS8fheRV14G7c6LuLC/IVWs6oenS3ggmFuDDqBTirebjZ/fLp17HssWN9vWdb6n9TmdZHqQM09f67rYmaIYx6geKI42fzprnnec7Qdu6YAY+MaHWnvg4kkijdcEMjK0NIVodQiQs8OpfDgIAojUR302Irph1JlTFe83Z0+sgOId0TD4Dpq2H38TyY5t223pf0TrHFdX/iQm1yGgS//nWPkw6uQYaY+QXWgH01yZm0rrDf1/P60Xv5/MvhHCdv2tPHPev34k7bpyDfYePu6xgrGsnejKYPb0OquC45xtNONqdtn10J8aqkNEM3Hzv81jROhtja0O4YtaZnrb07ke34c4lF+NbzRfg5HERCM5clhu5dSCeSGJMTQi3fvFCAMD4sVUIhzi+829/8ETT3rRwpu+LFoUsaIYdt+0Ph5T+thE8u3lxkL+99fJ6RescqArDhwe6sXZj30uR+9ebbeIDj+/MlneT6zqGlKitUrFyyRx8fOg41m9+y56sZhwYFw1h66596E1q+Om//ydaF8wI9tfPvrzWwO0NaS1yN/bsaGtCd28a1REVjAHXf3Y6nu38K65uOruw+9dP/a/kzdmKpdBl/MUs9y+m/yw49+0nXzdvyoDPKzj33fzSL6+aDvzLz/7kOaefB3Mq0xfVbPVV1gVsmAjJ7Gh+K+1TL72DJdfMdCeDDBgjndgoqdJfnmc0w7VvjfV8vLvE9xcaTARnSPSk8eGBbltjiZ70qLETEpwF1PPB/f3U1yeGjZAq8LlbNxR1zFM/XoDEEOWnXBmuCMZKipSUAX5gUpbnw0SXEj29Gn786z5Py1u/eKE5sBjpzOVhMDQ10HP4RS60L25ETyqD3/3hHVw3b4rrHIVcJ8hnLlqt2g9sayOY+jNinojVpc0NSKY11Kgi7+SD33W27tqH1mtm4Juf/xQUwe231N/94oUejzshOI70ZnDXL/t+y+1facS4KjNqo9zqcyXhp7NTx9cG6iqZ1lwTdFYn8We3XY4/vfo3OzJvQqzKdxOiL/zXKVAVjtu/0mjrwYr4bH/wZfzzly/yROqv3bjbjGzN2hGkMxoWXn6uPdHXk8rYaVeuNjdj++2Lf7HPmRspVIxfKHlFlw5D3SewbDw6cs+vMtuKI/dFyPbdH2PRZ+pdOrtlUQOeesmMGk1nNEyMVSFaE0JtdsWTX92KJ5JY/tAW3PONJoypCfnWgVVtc3HoaBJHu9P41f/ajX+68jz0JjXohoFU2ly54Dxuf7wXtdUhz7XAgCPHM66NPS3vYdL20OD3/F9x46fxnS9ciJ/8ZrvrmXzvY9sRTySx4sZPY8Gln8RJY8K+z99rLvskTh5XhYwm0XL1NHvztq49cXT3pvHlz07FY5t2w5DS1nUsGoHg3HXN5S2NSGU0PPTEq4gnkravviK43e77adbpr9/fSz1dNxAKCRgSuP2hLX19oAI2LrYICf+9VAo9fjQRyVoY5t6rSIgj1asXnQ4o7v5Xhf3PWxXmSPa4ExfarheTV90wMPOT43Ht353riiLVZREbJi7wpmXcuxrwlkUNYNydWcPwHyPVODbeHAiV/vI8yN9eq5DfVwghEeAzPkraueGcYyIIoowIWr462AMX53XAGCBl2Q7+ecAb/tydk8sFJmF3rACzg/DjX28v+bfUg6HdgZ5DAzxRFuufeRP/bc5ke6mqc7l47nUiYQUyo7uuw5n/RFoyrZnRyKJPd1174nhskxkhWndSDT7Y320v0+0veoIzf/1+sC+BjCaxas2f7M94TkccAHTG7MlGIBud98vOktfLaMBPzyxgQ7cJsWrb2zD3O1UVaJw+CfFjvejIbkTpl662KoSQABJJDTctnIlISMHY2jDueMSMVqoOq57j4okkhGDo7k67XmJYE33xYynXNRI9GVwx60yPB6Yd/YnCOriVvty13BjqvkcmrfvaeBi6RHhMFQwAq9qaIKXMeotLXN10jj35DPStQlnROgdrN+3CVz93PloXzMAjT76Oi6ZOxN9fdLpvvbB8nU8eG4GmSd86sHdfAitXbwVg2jKk0rrLysOyXnCuqqmJuCc8JsaqwMDsyWcrz3cVsIEsMXD8IhdXPvJn/Oibl2BV21wYBvDhgYRdfvVnxHAkYW5Q2Lpghq9mDCnR3ZO2rTScbeLRbvPzmxbOBGfAhLFh3w0tnatbLN10rOnEqmyk6OzpdbY2c68vYXpM67pRUBRrWvf3JS40ejPtWNHlXFWz5JoZJR34MBIk03pB9yqZ1vHGuwddVoeduz7CmPMnee5pMfc/2ZNBXSziakvNyeeMJ6+FtuvJtI4/7NjrsiawIuhzrx8JK/js3LM8Vg6RkICWdJ83MArcx7BZGgiMgHanZkMyRhquFcgjRd7fN0q6XBndO1Zct/lNfC0nyr5SGa65H5qAJogyZLiWuhXqi1XqKMLcVOBuh4fessWzoAgGPY9fbqmi6eX7lnowtDugc/j4KN+yqAE1ESVwCZ3zOrFoBAcOuNdjKABub2l0RbItW9yI3pQZjfy96y/EP3/pItz7q1fsKDtVEbh//U7XjvX9Rk8w6Yk8sWwQvjp/OgDYUc1+b6kD9aIXrpdQREEqI/HRwW4IIRBWmWs3d2Lg5OpZCO47QatCAtJ/8lZKiXTGwP95+yD+/qIzwJj03c0cTCKtw/alBYCOtrn2v9MZzfe4dEb3vMR44PGd+OHXL8Yvfvc6AHOy5Ib559u/KyhSKNcvVOEcXACpjAEhuN3Z9XtptG4EJzwoGnvo+x6BNh7Z+2xbZmVvu8b8vZ+7e9O4bt4U9CTNCZfT62ox9ayT8chvX/PV91MvvYPlX2nEvsM9qA4ruO36WbhnbV9/oX1xI372H6/a11h4+bm+tl43LZyJlau32uetjgj8fNkV0A2JI4kUxkVDgVF01qZ1ufoi3RVGvvsUHLloZNsSCUVwXH/VVHDGMKYmbE+ePfH8275++1VhBf/y8z962sSVSy7Gv60z/ftPHV9r7suQxQjIR7Radf19IN6D+9btQHtLI/6wY6+vZn/xu9ex5JqZULKboPk+M1QBjTFougGcYPSmbkj/SNX555d8v3O4KfRe6YbEIxt24ZENu1zpZk07xSdSuLj7n+vrnc/nW6gcWkYCWTdloTJPG6MbEm+8F8fUs8bbz+M33ovjHy726ifIyqHjZq8FB2MS379httkPyNplaIbhu2EgY/4e0Llph2qMVEkrg4Pw7zeOdK6GDxmgMclGz20YjrkfemYQBFHxaJrE4890uSYyHn+my99jrAwIjOj267ERJj4+ylak3IksoVNVjjtunINETxpHu9P4+X+8avuPKpyjukqxI01j0Qh++fTrrsnnQqInBOeIVqv2eayNuOKJJKLVIXS0zUUyrUFROLjPRpScB3tdFtKzCEUUHDia8iy9nDA2TJPQQ0B/EUnWd4Y07YV+8bvXsXXXPsfmaS+jdcEMPLftfd9IodwNkpwRdgeOJH2PW3zVdN8BHQB87dqZ+No/mhu9Wj6kQZsX5vqFhgVHvCftG+WsBbw0GolGm6KxS5OgiK2Tx0bwyG9fw9Zd+zAxVoWVSy7GiodNbTq908ePrQLjwNeumYlkRsOq9X0bzd35tYsBmHI7eNR8gWgRZItQd1INOtrmItGTwfbdH6M6cprrxfftX22EIvwj/jgH2h/c4tLXydEQDiX86wfpro989RPw18ns6XVIHHcfs7S5AWuefgNfnd/X3lmrl1oXzMDkSWOgcIZkRsOR7lTgyw+rD7A/3oMdb+7DpRecjo5spHNQBH7u330Ryk0Ak1jROgc9qQzix1L2Rpz/+Pe9CKsCsWoVsWoVd9/chIxm4G8Hu/H0y+/i7y863X5R0l+b3O89rvDoz8FECYjqVXL8kgtNBxR3/9WQwMFE2tNnG5+z0g8ovH8XVgWuv2qqZ2O/sCqgp939QD1gc0XdJ+hBUTg0TeKHjzqjpRuhKAxaTvdSBvTjc72lA/u8JzhGGq4VyCOFlP4R5jeW6Vh5QOTRGDF40AQ0QRAVjyH9IwdaF0iPX245oAqG2xZfhGPdaURCCpJpDWNqQ1B9Jh8JEyNgp++eVGbAgygNZiRpLBrBlz871bUUd3lLI3RDYqUj0rT+jBiuv2oq3vvbsaKiJwxDYuPL76LpU6e5ovK+84ULEU8kwRlDRpNYu/EN35cqiuC+elEEg1GAXlKZoKWfo6hTOszkizS1vuNCoP2hvuXcTtuLJ55/G1/+7FTX5O2yxbOgqgzIuAeyTzz/tu2v+8Tzb3sGmbcsasCho72uY+rPiKF53nnZQZj5Usy5xN1v80I/reeLcu5vIDCckaGVvvlQuRIUkWZNPgNmWR073jdZ6PROv+cbTZgwzrQzcK4K2LprH9772zHctHAmMprEc9ved0WGJdOab31gMF/oHIgfx9VN5+Dg0V60LphhewMf79Hwm///TU+U2bLFs3DoaK+PvppIdwWQr34C/jq5Yf759guz+jNiWHj5uVAExy2LGnCkO4mJsSrEohEsvPxc238/pJqWF3c88ufAyeT4sZStQ6EwXNV0Ng4dTaJ1wQx07vrIU/btLY1Yv/lN+3hrdZP1O/TsZsFWRLbzWpbVh7XZoBTCjspe3tLoitIvtE0OYjREfw4WXMBzr5c2N4ALuPpchaYDirv/aR0Bdiv+mwAWklY3/Ddz97O1CN5c0Rv0kMlIz2bedz/qn9egfrxhuMdyakCfdzDGSJW82aYqGD7/mfNwrDtt/q2Yf4+msWXQ3j6GLM/5glKFJqAJgqh4Ks0DGgzgjOOhJ151DWJoRBpMkAa6e9IDHkRZy3r3x3vtCKlotYoJsSqoAFI5USDWZlZ3fu1ixI8lkejJIFqj5l0aCZg7el849RT8+7NdduTeSWMi6O5Jo+PRvk1WLIuFXFiAXgoOBglYkuZ3LWL4yI1kdkZluqP2oshoEk++8DY+f+V5qFLdmwnFE0mMi4Zx981N0HQDIZWjo60JB4/24mh32p4MsQbKsWjEM0m9bHGjaxM258ZtRr4J4jxRzkZAFJVhSISGOSK50jcfKlf8ItLA4HnZfLQ77dv+H+1OIxYNA/C3jImEFKzf/Aa+/NmpdmTY2NoQTh4bcW0sl1sf2lsa8fBvX7UjsG9Z1IAXtu9FbEwYW3fts6OwrZcuY2tC+NFj2z3Xz63jfZ+T7pzkq5+Av04sO4z6M2Kel3Xfvu4CfP+G2UgcT7snbL/SaLezQdYcYEDrghl4ccdeXNJwGv776i2utu2F7XvtdlkIjue37cH1n52OG+afjw8PdNvRzUDfpJ3f5KM1Ue3Ug1MvuVH6BbfJQfe4wqM/B5N0xsDajbtddXztxt249YsXuOptoemA4u5/cLth+G4CWEhaLeB5rOnec6qq/4aJqsqg5XS0i5nwCx7LwdWB55AQOX3e5S2N4KisCeNBh8aWEAF77vh5khMDh/efhCAIorwRnOG7X7oQE2NVAMyHyXe/dKE5WC1DNM0/YkHTaEIwCGsA59TA8pZGnHXqmAFPWllLIoG+qLr71u0AQ9a7MxsFYmFFyRmGRKIng+e2vW9a7vWHZHjqpXdwxawzEa1WcbQ7jcPHetGR47H3wOM7AenVdLF6EYJDCg6dMUghAPhHovpdyzpWYwxScAhB3Yxc1JCAFCJ7jwTU0MDiKnL1ZVlpuK6lMGi6xIcHupHoSUNw5trMqKNtLloXzMDajW9ASglFSjBdQghzye3qDa+ha08c8UQSsWgY32q+AN/+wgWeSKi7H+1E87zzXNeOJ5JgYNlzGq46ZukkKMoZktmDTSfWYDMo4lGDW4PmBoonrkERkJdyfYZUEnp2AkRwZk44SobZ0+sAmG3u8pZGTBpf7Wn/b7t+Fna/dxCCc1+tzZ5eh7G1YbRcPQ2MAVc3nY3TJ9aiJ5nBvsM9dh3yqw8dazpxxawz7b8feHwnrv27c/HxoeOYGKuynxftD27B6g2vAVmtOjH1xUl3DoKeL8H1k+Ojg8chhQAXDEw37PbIurcLLz/X0wbdt24HQkJ4ynXVLzsByewytF7y3fONJnS0NWHd5jfxrZ+8iFVrOjH1rPEeH9wHHt+JxumTsHrDa+CcgQH4ywdHcPO9z+PHv94BVRG2DlwRrvbkYxPu+UYTWhfMsCeqnXpw6sXveTAuGgJzzCZxweh5PQQIzjD/krNwzifG4uSxEZzzibGYf8lZnnoruFnvrbZg1ZpO85kVUL+5YOibDWTZv/2uzzF7eh2WtzSio20ulrc0Yvb0OgjuLd/gNoZ70hV6zkxG2hsW/uy2y7GidQ7+sGMvMhlvn9PaZDv3+n52GUH9+NwJ8LQuffsHabKKyQuNLQEBidtzNHZ7SyNEQYM1olDoBTpBEBVPRGGoqerz0E2mNdRUqYgoDD3pkc5d8QRGLEiKigoiMHokPfB4iP6WRFaHeb9RciHB+jbaCoBxb5RobsQpYGpASundTDEgwsRPL35emj/8+sUFRaiQT27/FOPN2B/hnCij57a9X5DeejNG4GZGYUcZxqIR3LRwJk4dXwtVcKjCjNg6GuB9esrJNXbkiFX2qsIgwT2Tz9Y12lsaA7UVYsH1Kx0Q8WhIIN47+Bqk5eeli1+7097SiFPH1+DCqafY7ea1l52NH379YsQTKRztTuPfn+1C87wpePrld/DGe3HXUvjZ0+vQ/JkpuOORPl/Spc0NENm+hJSw65BzE0+L/XHvxnKCmzYIHguGxY1Y9/s3fe0RQoJ0Z5Hv+cKFf8Tl0y+/gydffNfTzgrBcTyjYWlzAxTBfcuPc/+o+J6U5tLRc9veR/O8KejuTbva1CCf8LG1ISxtbsC9j21HPJE0Iwxh6umpl97BD79+MRgYOId7I0XdQCgkEMq+GPTrR4QE7PvgtFayNT1vCtoffNl17PrNb9qR+k7f7GLuPz3f3VSHBU6dGPXc6+qwQG9P370q5rlSTN+hKszRPG+KJ21VmCPZ404bCXHfuhMJcaR6+9LWBJyzJszRk3NOwzB8Nyz87MU+NgaBGyZ772uhUeC0Ymlg0NjS1NK4rMbAGJD97dTGDS6jRU8EQYxijqcMlxcvYL7V9PMYKwdEwCZGolwtRYaJwfZu668z3JMy7Ci5M06J4gdZb0bA6bPXv5enNOCJ0Lr70U7ctHAmVq7eaqebGKsC84kaybt5TY5e/CJLNV36L3vMuRb55PZPMd6M/ZHKGB5v7j/s2IuOtiZISCx/cIuv3vLpwVmG++O9WLl6a7atnItM2tS7hpDv8QeP9OKmhTPxiQm1+PBANx564lXEE0nPJIXzGiFFBGpL1/XA+sVF8HFDoUFafl66+LU7Vp2yJoAAYOpZ423PX4v3/nYMrQtm4MkX38Xajbtx08KZOOWkGhgSLs/d/XHT7/RbzRdAQmJcbcTWn3MTT4uJMe/GckJwxBNJl11TMq2hJ5XBs9v2Yu++7mz9iUJkJx8z6eA6MNrI6/Os++9TcMWsM/Hki+962lnn/g23LGrwLT8J/43Mwiq3dWRF0v9hx15c2TjZlT5IF9HqEO5fv9O22bDydcN8w12+up+3b9B+DDPAYOplfDSEjrYm6IaBkCpsayXBuas+WPekdcEMbN21z30/i7z/9Hx305MyCvNVLuK5UkzfobfA6wNAMq3jjXcP4q6b5poBDIyhc9dHGHP+JFfa40WcU1W574aFqspg5Ab9ZFf4+W2Y7Ech/XjaMHNg0NjSxNLYhAlRHDiQGHUve4cDWmtDEETFk8/jrBwJZT1cnUuE2lsaEVKpSR9udN1wLet1Dhx0w4w0XbWmE/FjycCIjH6vEaDfSeNrXRr49nUX2L7MzqXKisKwbPEsV9pli2dBUbzDRr/IkVRaw61fdFvY3PrFCz0e0H7HxqIRSMBe4quGBJSIMigWFOXIYLZFuiFtfVnLd5988V3ohmH7m3qvI/MuYw2KHDIMZPUECME8x9+yqAFPv/wuYtEIfvG717Fy9VZ07YnbkxQ6HEu9Acz85Hgsb2lEOMRx2/WzPNoSgkFjDBoYQsLHxiMbNZWbB7D8XrB+OOsKhAAPCVuvIYdWA/NCjAi2VVCg1oM90p3potWqvQnduNowVJVDCOab1vTTN21jLP1Zkaa5z+Pntr1v/728pRGPP9OFWxY12Evu71u3A2NrI1i70fRY79oTx8rVW8EYbH2pIQENLDs5xRESbFTpzmW5AfN54sSq235t4ZFEGmecErXtAmLRiN3OWprp2hN3lSXQV16HjvT42rb84nevuybh7lm7DZ+ecSqqq4Qr/XPb3vc8d62NAa3JZ+dvsMhni+H3O7fu2uc6PpPWwXQdipQw0hqgmf8OevbkRuoHtZX9+WwTfeiGgVg04rKrcOrPiRIybc5MWPbvEztnMf0M3ZDoSWquz3qSmqdcizmnocN3w0K/Ta8VASz6TD1Wb3jNtiRa9Jl6KAH2IoVQqFUH4YbGlsRwQXWRIIiKJ3hHZo5y3No3nTZ8IxY+ff6pFIlSQjh1FxQNVUhEhqr461cRzGUrEwpxKJwDAq6lsg+3X4FNW97DitY5EBzQDeDJF97Gos/Ue5ZD+kWOMA7UVCk5FjYKVIVDc2ygmHts/RkxXH/VVDsSd2KsCj+86WJ092j2rucnYkFRjgxmW5Q3ygf+0XuCs7xRV0ERMBISD//2ddsKJhaN4I4b56A3paE6okBROP7bnMmQUnqsPfbHe6EZhh01OHt6HRZ9pt72R509vQ4rl1yM7t40DEOipkrB9376Un59SARGTRVTz/yWlS9tbsDajbsx7awYLr3g9EGxSyEGF2e5tS6YEVinColI1XTD3mTwc5ecg45HtwWeM5nWkNGk7f9708KZOG1iFKrC8a3mC8CYeZ1X3vgYN14zA1+5ejp0A+hJZfDqXw5i775uV/RzRtNck5ETY1WQUtov5QbLrqccyVc3nZv07TvcA003fJ891qoj69iwKqCnNVfb6SzLU06qwd793Vi3+U00z5uCF3fsxV03zcXhY0kc7U6D+2xwuT/ei7G1Ydz205cRi0bwreYLMH6c+eK1uzeNb37+Z3zGAwAAIABJREFUUwipAtHqEOKJpK/X94cHEvZqkyBbDHNfiYFHdgYd64nUD/AfpqjSwgmrwjcC2NKfna5Kxf4jSU8dnzguglRvZkDnBPJEsgqO3F0AqyMKLpp2Cm5/aIsrD9URBWlHHpSAcyo+59QCJqs1n00QNV3ipZ0fuPqnz3b+FVc3nT3g8QytWBoYqYCx5ezzT6WIVWJQIT0RBFHxWB5nuW91I6HybAKFYJh86ljc/tAWLOl4Drc/tAWTTx0LcQIRA8TgY3n0BkXJFRqRwRjzHLu0uSG7THwr2h/cgpWrt+KeR1+BbkjPUlkhOF79y0HcfO/z+Po9z+Pme5/Hq385CO4z0PSLHKmtCuHO1Vtd17pz9VbPhjK5xzbPO88TBaNp0p58tj7rWNOJUTCfAqDPo9MTYTKAIPB8UT79RQAFRe4HHfeL372OK2adaVvBdO2J4/71O5FMa/j40HHcntXFgSNJ+1iLiTHTGsYq8ytmnenanGvrrn1Y8fAfET+WQrTa1Fp/+lAAXDdviitq6rp5U2zP3ELrmd+y8vvX78TCy8/FlY2TfZccjxatljLOcnvi+bd9I1hzteAXkWpFdz3w+E6Xvp94/m18+7oLPG3uxJOq7cjmeCKJk6IRCMbw0BP/B4aUuG/dDqxa04lnOvfAMCR+8PCfcPO9z2P1b1/3iX4OozqieqL4f/G715HWg5fcjxb9BdVNa7NTq0w4Z1i/+S3XM9Lv2XP/+p12VGduOxdPJBFSBf5t/U47qrgju5Hg4890QVU4Vm94LbB9++hgt90uLn9oC77/sz/ibwe6ceu/vYSf/vt/QtMN3L9+J9Zu3O3R6tLmBqzf/Jadz9wNLK3NVf3yXUw/wu9Yv0j9oHNRVGnh6IbMqz+LZNrf1iKZ9o9ULuScABBSuO+qt5DiHfOkAvKQysmD4P79UL8XFsVs2qsAuOyC07Fy9Z/w9Xuex8rVf8JlF5x+wrrKtzqR8EcNGFuqNLYkBhl6bhAEUfGkejOYOC6S9cUz34abG2xk+j+4BNENA+GQcEWkhkMCumFQBHQJkU5qmDA27PJjvOfmJmR0o6iIjHRGx9qNu13Rnms37sa3r2twpdsf74WWPZ87+iRgkxefXZ2dkSOGAXDO8i69VAKOtQZFucdx5r+xU+65KpVcj05zWT0GFNHovN9+m6UMJALIL3II2ai/ay77pKvsuvbEsXbjbtz6xQvtz63JwFytOe3Cg6wQJk+K2v/O/a4/rVm/L9czNxJWIDN64O8OWlYerVYhAjYhGy1aLWWc5WZFsLYumIHJk6K+WrDaMiGAjrYmGIYE58huyqnYZe4855qn38A3P/8p1J1UY24IJzgUwXDjNTNww/wZUDiDgEQqa7V0JJF2tdEZTbc3GOvaE8dTL71jR/rHj6VML2LNcB3z2CYzuveG+X7t+OjSX1DdrDupBh1tc+3nYMvV0+y2aFXbXBgBzx7r+Wi1g852gnOGex/b7rHGiFareHbbXgDAitY5iIS576ZxDz3xqudan5hQi462uZgQq3Kd26VVwfGjta/4Xtf5t2WddCKRnX7HhgTDkmtm4Ib55/d7LooqLRxND4gA1nOeYwX2rYo5JwD0pjTfVW+fv/I8T9pC85AK6Ife+sULPOcsZnNF0lXpoBkGqsPu1Y7VYQUajS2JQWY09GEIgiCQ6s2AAZiU3VTAubtz2SEZDN3AGXVj7EmsQ0d7AEldhFIjndTsDVOM7DLJYjdBFJxj2lkxfGJCLQQHxtSEMe2sGNSQwP/7z5e7li0KzgHkbBooGbbv/tizxPGqprMBITwToNYGHAIAdEAIgWsvOxtXNk72XivHNsK5QYwUHLOn1+GKWWfaAxYeuMlJedrhDIRMWndsoqMjcwI/O99mKQPddFOoHFpGwnxBwRDKWsD4WRiMi4YgOMM932gC5wzRqhDAgLtumgshGKQBpDQNDAz33nIJ4sdSMKR06UJROMZUh8xrc44bF0zHp86r61dr/d0TBaZn7IEDieDfyplHo89tex+JngzG1IRHvVZLFT87ANXP0z6nLTN0uP7O6H3L1f30bb7Qk+Aw29VUxgADA2MSEoDOTM1ee9nZmHrWeJeGFl81HdEajjtunINETxpHu9P4t3U70LUnjomxKqxcMgecM6x2rAYAHBpDkI0OBxPm72C6LGiiRggODSirCZ4gy4cP9ndj1ZpO+2/LQiKeSJr1XkrIgE1KnXWXC5adDZNgYJh2Vsw1EXzjgumYGKvGw+1XgHOGtKZB1znGRcNYeeMcCMGhqmYb99X503G0O40nnn8bXXvimD29zt4QWEpg2lmmx7ilj93vHcQ5nxgLw5Bonnce1m9+y2Ur4muLkbW5OJHNlHOPzWTrQ6HnGuyNnCuVQq22rLajkL5VMfZdgnOcdeoYhFQOKSWEYDjr1DEndF7BGcZFQ65jred/rgWLrhuYGIu4XrRXhTmSPf5BP6SrEkEyZDSdxpbEkEMT0ARBEGVGTZjjmKrYO5pbSylrwhw9PdR1qzRqwhyXZpcoWuV9x42fRqI77YowaW9pRHWYI53SXdEnQoXn+PaWRvzvV/bimc49/fqKVvtc37pWbx69hQRD87wpLn/D5S2N+EHrp3Hn6j+7zhUSOKGJWGJwCEUUHDia8nhS/vCmi/GLDa+7optnT69D87wpaH/Q9D69/qqp+MFjf3QdNyaq4vChFB54vK+8v/ulC3H9VdOw8pE/28f9y8/dx63dtMv2QG1vacwOXh2DYR9/WKdXasG/10ejyxbPwqYt7yFaraK9pdFzL0irI48zws7SkNMbtRgtWOdat/lNW99+5/zOFy6EojD8aO0rruj+7bs/9niFL1s8C89s/SuunH2mGemsG1i94TWXjtY8vQtHEmksbW5wXWfZ4kY8/fI7uPrSsz36W7Z4Fp5++R1cNO0URMICVSEFNarI+zsHq64MN35RlJY/MtBnAbB2425PhKVldRRUd9WQ8PXXBoAnX3wXNy6Yjmlnj/f0sba8sRenjq/BqROjWL9xl+2J76eHXE9dy9fZ2W46vXzXbtyNeCLp+Y1mvhm1OWVEVZj76i/3OVZM36om4Jx+/f6aMMe0s8d7NOiX1rIozD2vuUq0L63fszJIm+EqFfvihXlbE6UDjS2J4YJJWbEbB0wG8N6hQ932cqzBZMKEKD5364aijnnqxwtG9TEDvYYzesmK8hopJkyIDsdrwMkYQu0OhJG+74NJJfwWKYTdQbCYGKtCR1sTWEBk3mjV7mAxkrrxK+8VrbPx0BOvBmrAHfHGffWyonUObr73+X61MxC9mcdxtGc3IHQed/fNTZASjmXAwRYUw6RboMy1O1j6DCzrm5sACSR601CFQE8qg3G1YXuAu7yl0Z5gcx3X1uR7vpsWzsTK1VsDj2tdMMMV5ZirtSBtdbTNBXNMqvV3X4LOs6ptLhiAsMqRysgB2aVUSptbqs9Mq42TgL3RqYWfFgo5FxjsaK98unV+tqJ1jj2B5Py8dcEMrN7wGn70zUuQ1gxIQ5p2D4zhl0+/bm9mV39GDM3zzkPdSTX4YH83ntv2Pq6YdSZUheHVtw/gs3PPhm5I6LrEky+8jWe37bXzAgBn1EXz/s5C64qTUtFubuR2SDCkdfM+KpyDCyCd8be0UkMCaR3QpYRg7udMvmeaVdf9vr/rprngjKH9wZft8i20zbPatKA2z2xzGJ5++R1PNP2Sa2YUrOXBoFTrfCH5KhXtRqpV9KYMW7t+EcDF9K2kEHj4t696VussuWamb9rCz8vx59c/QuP0STmbmk9yaa6YdmSgfcaRZiR1Xwq6LddyGypKtR0cLgr9/QPRLkVAA4iOqUIkTLeCIIjyoBjfOKL88SvvSEjJqwHnksagHckF9x4HeAf9umEgFo24vP+eeP5tX705j0WAh6e0vafN/xqjr19bsgS2LdkB5rd+8qL9+c9uu9xOG+TrHHS+2uoQ6s+I4bSJtYEezIA5Obfw8nPNSSGHXUyQP6xuSIQdGownkhCCF+0BbRgSisxdCixJqyWE1cbpjg0uLZy+ucWcy0ILOGckpHg+C/IKt+pEOqPj0NEk2h/cAgDoaJtrTz4Dpt/0ytVbcd+3LgUAXHPZJxEbE0FvMoMnX3wXjdMn2cf65cXSvMEYMrrs86vmpgVUvrpS6v2F/JYROgw9eNm+ZXVk2a45ozSD+1Ay7/dSSkhmWvsU2+ZZbVrQceaEkMSTL76LJ1981/X9DfPPL/myItwke9y2f0mfCNJi+vJ61mve2XYAwA3z/dMW2mfTDYlHNuzCIxt2uT6fNe2UHL/qwtsRGqOUJ1RuxHBBegIQCSsDiswlCIIYCUSQj67ggEYzJJWGn0dfMq0V7gcYoBfpc5zfcu0fffMSz3L0pc0NCKnC9rW2ruM8dkXrbM91Z0+vQ+J4+S0HHy3k9YPM8RbXjT6PWj//XOs4v89j0TBarp6GfYeP+36f6Mmg/owYvvzZqa4l7pZdjNSl73EhlSOeSBesryCfWcEZBFhZWheMNvKVYa436YmeM+lo76zPnPXA+blVJwTnrvbar67Mnl4HQ8Jl07Fs8SyzvQyoW1ZeQirHke407vplp6t9HhcNY2y1CpkxBv3+lDtK0DNRSnzt7ufwyPIrfb/PaBIrV/8RS5sboOn+95Uxf+1Yvs7BbSWz/01lNTooyte5iH5/WBW+fbawKqDntGGFtp/FtLPF/C6idKCxJTFc8JHOAEEQBFEc1VnPtomxKgB9PoHVIWrSK5GwyjzlXXdSNZb7aCAkvMeHVI5li2e50i5bPAs9qYznOA2wJ9wAM/qhuydjD2Ssz+5fvxO5Fl65x67f/BaWNje4rnvD/PM951+1phPuIRExUvhprb2lEWGV2Z6s1nfPdv7VTvvE8297yrq9pREf7D+G2653a++WRQ3Yf7gX963bgfWb38Iti7zHPbftfSy8/Fx78hkwtdKxphNpHZ68WJPDhu7Vbz59BZ1HgX9dIK2WHvnKcDDP+Z0vXIhoTcij5Wc7/+ppX29Z1IDntr1v1526k6rt+uFXV75y9fm4Z+02l9bufnQbbph/Pp7b9r6njixtbsCY2hDqTqqGocOefLaOvX/9Tuw/3INURg7J/Sl3BGeeMlja3IBjPWkAwAvb93rawWWLZ+HJF962729I5Z5yuWVRA17YvtejB6tNA2Drwq88qKxGF6rKfPtmqupd0R5W/fv9YdXb79cN6dtn031sFwrVXDHajASMUSI0RilpaGxJDBf0TCMIgigzEr0aepNpl1/hvsPdSPQq1KhXIOmkhgljw9nyNm0xwiEOSOnSQJA/bW9Sw6Yt72FF6xx7p/UnX3gbn7/yPHS0NbmO81tmmehJ+y7Ly2hG3iWaXXviWLtxN1a1zYWRzbdRxsvBRwNurZm6CqsM6aQ57RqrVtHRNrdPhyq304ZDAnff3ARN79v1fky1CjC4lgI/tmk3Wq6ehv3xXuyP9+KxTbvt7yfEqlEd5lhyzcy8y0GZlJ68KABSeZbW++lL1w3f8+i6MSjWDsTQk68MB+uclt+wbphtriFNL2fGgElN50DN1gPrczCJJdfMREiYdapGFaieUJNtC83JpLvbmqBJCSNrm+GnNQaGJdfMABjs8zPGIIRpRcF0Gaj5SEjJW1dGcxR/KqNj7cbdrnZp7UazXQKAX/++CwDsti2j9flvA+b9jUWrIISZpjet4W8HjuOxTbvRtSeOfYd7PG3okmtm4Ib559te1h1tcwHGACld5UFlNXpIpvTAvlnuM6YnqeGVNz7GXTfNtb2aX9i+F5ddcJonrab7twma7mPBUWD76Uznp1snqd4MJo6LuOqAuakhbUBYytDYkhguSE8EQRBlhuAcG/7wru9mJLS8rTJJJzWH9yWQ7jXtMvoI9qcVnCPRk8aHB7ptvSR60hCcg+m6yxvTb5llsN1H/0s044mkmW8pAV2CBy3xoyXGJYNbazqc7zRyPVnTzr81A30lKJFJ6WC6ASm4Z9Mtp6a69sSxak0nJsbMDY0s70whBGZPr/O0c9Yy3ty86MizhDSrLyE4dDBohgHOGRRhfs6sgbnDS3YorB2IocFPCwPB6WEvs+djUtp+w1ZconOhCQMQO6kaBw4kkLsAxWpbdd0AdHPZKQcA3XTA5wz4/sN/ROuCGYFakzA3IBQcCDFA13VIHXZdC9L8SWMiEJxDQA7a/akUBGcYFw25PhsXDdk2GQDwTOceXH7RGRCc4/urvRtzCc7ANB1ScNy5+s+u7xPZSGoTCT3jbmMsL2trkydneVBZVQZWW7I/3gMI7jtZKzjDq385aL/YAExtXTev3sfWguGZzj32yxEr7eUXne5rgZHv2ZlLoZqz0vnpNpdUb8bVj0j1kpJLHRpbEsMFxdQTBEGUGTVhjuZ5U7B6w2tof3ALVm94Dc3zpqAmTE36aMHyW25/cAuWdDyH9ge3IN6TyZmUNgmrzFcvYZ9lnn7LLD8xsdZ3WV5IuI8PCX/7Bmc6WmJcmeTTo1+Z151Ujdv70UExurXIpy8hOI70ZLDswZexpOM5LH9wCz48cBzHM7kvc/o/F1F5FNOmDtZ17n1sO5Y2N/jabNze0ohkRus3PwqA27/i1unS5gb85Dc70P7gy0PyG8qdsOrfh9r93kEAbkurkEDAc808V+5zb/b0OjTPm4L2bDszVDoiShdnHb/hh8/krbuFPmMK6V9ZVAeMEappjEDkgcaWxHBB/WiCIIgy43jKQEeON2nHmk50tDUheGqGqCSC/Gk72uZ6NJDKBOnFm9ZvOSYDw/rNb7qWK6/f/CaWXDPDdXxal/2mG4rl8sTIk1ePAWUOVeTVQTG6tcjVVySsQGbMaGkpOO5a4/XKvWnhTJxRFy2oLpBWK5di2tTBus7+eC/WbtyN5nnnYdL4GtNmwzAtOYRg+N5PX+43P7puYFyVintubkJGl/jwQAJrN5pWEMhea7B/Q7mTysjAPtQ/XHyWx9JqfDQUaHeV+9wbUxPGytV/GnIdEaVLoW1JMc+YQvpXFj00RiAGAI0tieGi5Cag6+vrxwD4I4Cru7q6/lpfX38lgJ8AqALweFdX1/dHNIMEQRAjTD5v1JJr1IlBw7k8HEV4Kfv5Oltphc/S0NzlmBpj2LprH7bu2uc6xw3zz/d4QBeUjpYYVxz5NGbpy6/M8+mgv3MG5kU3oAgOcIaMZupaERypgPOZXrkAGPNMAJBWRw8D1duJXqdrTxwrV2/Fw+1XQJHStPHQgYxRuAe53X4zhpWrtw75byh38vahspYrTmuqTFp32Qk4v8t97nW0ze233AqxZyDKl2LakoLtLwrsX5lpaYxAFA/phhguSiqmvr6+fjaAlwGcl/27CsD/B2ABgKkAZtXX1//DyOWQIAhi5LE8H51MjFXREs8KJnd5+IcHuv01wL1xCpaXbW7av36UKGh5cNDxudcqNB1ReQxF2Q/0nEHLnxXu324m0xo+PJCg5fKjnOFqv4J0qPA+zQnBIbOfF5MfaoMLQwSUgeDF1/vce57oyeQt30LtGYjyRQnooysnUMbF1O3B1DcxeqCxJTFclJqibgRwM4C/Zf9uBPB2V1fXe11dXRqAXwH4p5HKHEEQRCkQCXFfL7hIqNSadGKwyF3SuX7zW1ja7PYMDfIO9PMZvGVRA554/m17aaiW59qF+hSSZ+7oZSjKfqDnDFr+zAU8vtNLmxswpjaE9ZvfcqXNVx+IymS42i8u4Gm7lzY3gDt2MNQA/OJ3r3t8ofvLD7XBhdGfr3Mx5N7z57a9j2WLZwWWb1D7RG1O5cA586/jJ/AiqDi/6MHTNzF6oLElMVyUVJ+kq6urFQDq6+utj04F8JEjyUcATivmnCefXDsoeSNGjgkTonn/rlRKTbuVdN/L/bd8dPA43v0g7vIk/M+39qGmaiImlcBvKzXtDhYjqZv98R7Psu21G3ej4+YmAICqcIytCQcOcMaNk/jXpZcimdbx178dw2Ob+jxC98d7Acby/j7r+Ixm5L1WoelKlXLW7ki3a0NR9gM5Z25dMT/rhW4Ak08di3tvuRTpjA7OTcuNjjXb7Lpgpe2vPpQaQ63bcroXJ8KJaLjQe7Q/3oO1G3e7vFzXbtyN711/ESZmz7E/3oOtu/bhSCLtSjcuGsZJY6vynr/c2uDhaHP9yqa2JoK7b26CZkgonCEWjUBRBjbR4rznALDu928Glm9Q+1RqbU4p5cVJKeUrSLsfHTzuW8dv/dKFJ9RHL6ZuD6a+/SilchhMKvV3Ocmn21IeW44Eo0EP+Riq319SE9A+cADS8TcDUJRJ1qFD3TAMmTfNaBdXqXPgQML+94QJUdffw81waqUQ7Q4XI33fB5NK+C1CCHS+sQ/Rmojdue18Yx8+dV5d4G8brdodLEZaNyIksKJ1NiIhBYmeDJ54/m3EE0lASjDdgKbpOJTM9H8ewbF6w2uuAfDEWBUgZUG/b2L2PvR3LQYUnKd8DPfzuVy1O9L6dDJYZV/oOZ3e6IIzCGEuVfbT+KFD3eYxAGAAuuBmPXJQTH3IR6W0uaWkreGiWA0XdY+ymlu1ptP+yKO57FLorj1xO93s6XU45xNj8dHB7oI2xDyRelgp2gX6LxsFAAwgHj9+wtdiAKTgePUvB/Hstr32567yzZbtQJ/BxZDbNhbqNV2qdb6QfJWCdoUQGBcNuT4bFw1BMDYo97WYuj2Y+rYoVX2cKCP5u0pFt8WOLSuZStV5oRT6+wei3VKfgP4AwCTH36egz56DIAhiVBJWGZrnTbF3K7aWSYVVhjTtklVxCMFxJJHGQ0+8apf30uYGxKJhc0BZxLmsZZyrHNqxlnGSdIhyw/JTder59pZG3HHjp3HHI3/uV+NUH4jhphDN5aaZPb0OzfOmoP3Bl13HxKpV2ryuxOivfIerzfFrG0kzwwP10YlyhHRLDBelPgG9FUB9fX39JwG8B+ALMDclJIhAomOqEAkXJ+1kSkPiWG//CQmiBEhlDLuDAJjLNzvWdKKjbS5Kd6EtMVA0AHfllPf963fi7rYm6HpxvUJdNxCrVtHRNrfoqCiCKDX8/FTvWtOJu9ua0NE2F2AMkDJQ41QfiOGmEM1503B78hno8w2mZ37p0V/5Or/vr306EYK8pkkzQw/10YlyhHRLDBclPQHd1dWVrK+vbwHwBIAIgE0A/ueIZoooeSJhBZ+7dUNRxzz14wUYvYssiHJDN2SAx6ks7UadGBBB5a0ZxoDKW9cNMGQ7ALqkSE+ibMlbN6S0lxDm0zjVB2K4KURzzjSaYdAzv4zor3yt7wtpnwacB+onjhh074lyhHRLDBclqaeurq7Jjn8/B+C/jFxuCIIgSgvB/T1OBWeAXn7+tUR+qLwJwh+qG8RogHROFAtpZuSge0+UI6RbYrgYvO1QCWIISGd0TJgQtf8PwPW33/8JotKxPAQnxqoAwOUhSFQeVN4E4Q/VDWI0QDonioU0M3LQvSfKEdItMVyQpoiSJqSKAdlpEEQlM1wegkRpQD61BOEP1Q1iNEA6J4qFNDNyUB+dKEdIt8RwQRHQBEEQZYiuG2C6gYmxajDdoA5ChWOVtyIllTdBOKC6QYwGSOdEsZBmRg7qoxPlCOmWGA5oApogCIIgCIIgCIIgCIIgCIIYEmgCmiAIgiAIgiAIgiAIgiAIghgSaAKaIAiCIAiCIAiCIAiCIAiCGBJoApogCIIgCIIgCIIgCIIgCIIYEpSRzsAQIgCAczbS+SDKhAK1MhnABwC0IcxKSWq31PJzIlTSbwFIu8NFpf6uYhnm+zAZQ69boAK0W855H0pG8L5MRoW0uaSt/qmwezQZFaLd4brGQCnVvJVxviajxLRbqvfyRKjE3wRQf8FJpZZxMYz2ezBU8wtMSjnALJU8TQBeGulMEBXJWQD+OoTnJ+0SQwVplyhHhlq3AGmXGBqozSXKFdIuUa6QdolyhHRLlCtFabeSJ6DDAGYB+AiAPsJ5ISqLoX5DSdolhgrSLlGODEcENGmXGAqozSXKFdIuUa6QdolyhHRLlCsUAU0QBEEQBEEQBEEQBEEQBEGMPLQJIUEQBEEQBEEQBEEQBEEQBDEk0AQ0QRAEQRAEQRAEQRAEQRAEMSTQBDRBEARBEARBEARBEARBEAQxJNAENEEQBEEQBEEQBEEQBEEQBDEk0AQ0QRAEQRAEQRAEQRAEQRAEMSTQBDRBEARBEARBEARBEARBEAQxJNAENEEQBEEQBEEQBEEQBEEQBDEk0AQ0QRAEQRAEQRAEQRAEQRAEMSTQBDRBEARBEARBEARBEARBEAQxJNAENEEQBEEQBEEQBEEQBEEQBDEk0AQ0QRAEQRAEQRAEQRAEQRAEMSTQBDRBEARBEARBEARBEARBEAQxJNAENEEQBEEQBEEQBEEQBEEQBDEk0AQ0QRAEQRAEQRAEQRAEQRAEMSTQBDRBEARBEARBEARBEARBEAQxJNAENEEQBEEQBEEQBEEQBEEQBDEk0AQ0QRAEQRAEQRAEQRAEQRAEMSTQBDRBEARBEARBEARBEARBEAQxJNAENEEQBEEQBEEQBEEQBEEQBDEkVPIEtAJgcva/BFFOkHaJcoW0S5QrpF2iHCHdEuUKaZcoV0i7RDlCuiVKgkoW4GkA3jt0qBuGIUc6LwMiFqtGPN4z0tkoKUb6nkyYEGXDcJmS0+5I3/fBpJJ+C1D47xmt2h0sKk03A2W478Mw6RYoc+2SPv0ZyftSKW0uaat/Ku0eVYp2gdIum1LNWznnq9S0W6r38kSoxN8EUH/BSaWWcTGM9nswlPMLlRwBXfYoihjpLJQcdE9Ghkq675X0W4DK+z2lCt1nE7oPpQmViz90X04cuof9Q/eodCnlsinVvFG+Bo9yzHN/VOJvAir3dw0Euhd0D4by99MENEEQBEEQBEEQBEEQBEEQBDEk0AQ0QRAEQRAEQRAEQRAEQRAEMSR1hxjJAAAgAElEQVTQBDRBEARBEARBEARBEARBEAQxJNAENEEQBEEQBEEQBEEQBEEQBDEkKCOdgUKor6//EoD27J//q6ur67sjmR+CIAiCIAiCIAiCIAiCIAiif0p+Arq+vr4awAMAzgNwBMCW+vr6K7u6up490XMLwaEB0A0JwRnCKkcqI6EbBmqqFCTTBnTdgKJwcMaQzugQnKMmzHE8ZUA3DAjBURXi6E3rgGTgHFAVjlS673tFMEgJ6LqEbkgogoFzBkNKcMag69L8N2cAJCAZwiGO/fEe6Mw8XkgJXTcC8x9SOQwd0AwDgjMogCe9328OSkcQTtIZHRMmRIs6JpnSkDjWO0Q5IojyJFylms8Ww4DgHIwD0pD9tsXV1Sp60gY03QDnHKpgCCus71mUfTb19GQAeNv6mrBwPbfCKkdPUoPgHJEQR6o3Y19LDQmkddjnFQKIKOZz76OD3RBCuK7lJBRR7Oeo4BxVYY5eRx7DKgMY63tGZj9LJ7XBvtUVQzHPbWdaVXAw3td3cd7nfP0fRXAoCkcypfXpKmVAt/spAOeANADDkGCMAUxC4Ry6IaHppsYiIY5kynD0bwApJRTBEVIY0pqZNqQKZDSzv5Xbx3L2xSw9JbO6y9WaIgBNlwir3D73/sM9UEIKoGfPITh0MGiGAc4ZFMGgCu6qk+GQVTcYIiFhfxdSBaQhkdHz97OI4SG3D8zAoOkGdAMQgkFwgOkSAKCDgQtAt9o1waEqHJyZmgHM/xrZPrqiMGQ0aesuHOLIaAYymoSqckhDQtNNXQsBHEkkwVSBjCbNcYDgkADSmgHBGGD+D6rCoRnZPDmuxzmDhIQ0AM2QEIwhEuIAA1KOdl8RZjoA4GAuXRuMIaPL7G9nyGgGOOsb6JVj3z9fOyU4h6oyJFO673fV2XbLemaZzyLd9UxMG0AmY5ifCQbGkC0TjojCkMy2I1LCHqMp3Fw8nMn5XEoGVTHHe1abq6qmjmROO2keL/HRweMQioDgDLphAJJ5npkhAWTSekH3J7dc+7t/+c49WGVWLlobbKqr1YL6TFY6v37ccKWNVKsuzTmfs06qqlVXnaoOc/QOc16tPDjruV8eijlnMVS6vgvVLUGcCCU/AQ1AwLQKqQFwHIAK4IRntYTgiPdksGpNJ/bHezExVoX2lkas3/wmGqfV4ezTYuhwfLe0uQFrN+7GDfOn4ZiquL5rb2nEmBoVD//2VdwwfzqOdKc93xvSwD2PvmJ/9t0vXYhIWEHieBr3r99pf37LogY89dI7aJ43BX/YsRdPvvguJsaqcPtXGjGuSrUbOWf+Y9EIrr9qqus8y1saEatWPR2R3N/sl44gcgmpAp+7dUNRxzz14wVIDFF+CKIcCVep2H8k6Xo+WG3+dfOmBLbF1dUqPo4nXW33bYsvgmDc8wybFIsgldJdbf36O/8rPoq7r7ts8Sxs2vIeXv3LQbS3NGLiuAhSvRmoIYGDCfcz7IFvX+o53rqWs2Maiig4cDRlp5s9vQ7N86a4jrvjxk8jnTE8+Z4wNkyT0D4U89z2S2v1XeKJpH2f9YzhSnftZWfj0gtO99VHoiftKcPvXX8RNE3iJ7/Znvczq0+1ddc+l9bnX3oOJsSqcPhoCu98EMe0s8d7tPXGuwcRCQlPX6y9pRF1sQgMQ7q0ZuV593uHPOdb2tyAWDSMaETB4UQadxVQj15542NURxT7XIX2s4jhIbcPvOQfz0cypbvK5ztfuBAnjQ2jN6nhhe17cUnDabj70W195by4EWOiKo51Z9CTzNjH+rVblpaPJNL4yuemu3S+tLkBY2tDeGzTblvrS5sbEAkLPPwfryOeSOKWRQ3YvvtjXNF4BlIZ3ZPX9pZGKArDf1+9NW9bubS5AdURFYY0NVcVUkxdd6dx1y/96/3tLY1QVY47HvlzWWk33zjNus9WOxWtVj1tWG7a9pZG/G1/Avf+egd+9I250MdE0N2TcWnCaqP+6YrzMKY2hAPxXlc5fe/6iyA4d+klt22zyn1cNOTR0bevuwC1NSqOdac9WlUUhv/x7FtonjcFb7x7EI9s2GXne3w05Jko7u/ZkPu9XzsfdO7BLLNy0NpgU12tFtRnKjTdUKaNVKvY55O2LhZxTUJXZfuhuelOiUU8E8BDlddC81DMOYuh0vU9VPeNIHIpeQ/orq6uBIB/AfAmgA8A/BXAH0/0vBpgNyAAsD/ei441nbhi1pn41Hl1duWzvrt//U4svPxcnDy22vNdx5pOQDJcMetMMMZ9vz/WnXZ99q+/2g6Fc7sDYn3+wOM7ccWsM9GxphNXNk62P7/rl53QAvK/8PJzPedZtcadPug3+6UjCIIgBp9k2vA8H6w2P19bfDxleNruY91p32fY8ZThaeuPp7zXvfvRbbj27861j0umzc5zWocnrd/x1rWcpDLSlc56ljmP23e4xzffqYw80dtbkRTz3PZLa/VdnPc5N92VjZMD9eFXhonjaXsCLt9nVp/K+tvS+v3rd9qT1Y3TJ/lqq3H6JN++WMeaTvSmDI/WrDz7ne/+9Tux73AP0jrsyWfru6B69HcXnu46V6H9LGJ4yO0DOyf0ALN8fvKb7dA0ibt+afanrYlG6/uORzth6Gb0svNYP81bWl54+bkend+/ficOxHtdWr9//U4c607bde+Bx3fiysbJOJDVXG5eO9Z0QjDeb1t5//qdOJJI4lh3Gse60326/mVwvb9rTSf2He4pO+3mG6dZf1vtlF8blpu2Y00npkw+GQBw8thqaJr0aMJqo3702CvQNOkpp8TxtEcvuW2bVe5+Orpv3Q4I5h37/eQ325E4nraPaZw+yZVvv/nh/p4NhbTzQeceKDTONCm0z1RouqFM2xuQtjcnbU9Aup5hzGuheSjmnMVQ6foeqvtGELmUfAR0fX39TABfBXAmgKMAfgXguwDuLeT4k0+u9f18f7yvM9b3WS+i1Sp0wyj6O90wEK1WIaX0/T4SUjyfcYbA6+yP90Jw9+dgzLZBcObfSp97Hmf6fL85N12pU055PRGCtFtOlGpZlWq+Bkqp/Z5K0K4fJ3qfPzrYnbfND2qL/Y6LhJSAZ5G5lNj5XdBzy3rGWMdNmhDFRwePe9IGP/fMY4Ly6fdsypfvSSWg41LTblHPbcYC9WX9208fgvv3RQQvvAyDytW6tvNvZ/8nqM8kpYQe8J1uSADBxwX1wXTD+11QvqWU9r+BwvtZI8VQ67YUfqOT3D6w+Zm3fCydBWlcSunRQFBZ57tOJKQgEgr+zMqDNRYIyquTIG3mjif8dJ2bX78xSKlrN984zfm38znWX1rdMCdSdMPodwzm932+8rOOyy333HRB17WOc7Y/dr6l9/nY37Mh9/ugOuB3bmBgdX44xpmloFmLIO0G9fX66zMFpSuFtCN9/aHK6/9l7+6jJLnr+95/6qGfZnfEjqSZRXYcbSBQAtnEK7EjX4MxRgJf21dRCPHRSpegwTF+wI72IrCjXdkx+IIWzg1wRU7sxDh45dhYyr2yDTI5PliLCdgOzCIWEwtT1sVoA86ifdAgzW7PTHdX1f2jqrqru6ufZvqhuuf9OkdH3fXwq19Vf3/f369+09vVqlt8zco8yk7jdjeZps91FEZ1/pmfgJb0w5JOuq57TpIcxzkh6a3qcwL64sVL8v2Ub1ZZppYWSk0NbWmhpPVyVUsLcwOvs0xT6+Wqrt5XSl2/WWn++9jSQkl+oC7HKSn5rzmWFkpSEOj8+fW2+sfbt5bTtH2Xc27bLsMWF+cnWtdxJqKOsTsB2z3vLMbVpGNo2Po9n90au8MyjLixLKtrzu+Ui9P226zUOvRFhhQETessMz33x31MvN/58+upx+q0f7xPp3qm9U3d6p127uMe/GUudvvstxcX59s+93jb9eifTnaKD89PH4t4fv+fYafPdT3xzzaTsR6PfwzDSN3PMAyZHdZZpiGp836dxmCW2b6uU70Nw6i/HmiclTArOTeTfWbLGDhnp3/ucZx1inHDMNpioNNnnWxHafFVrQUdl8V1iO8FOtU1qVNstt5PpMV1a33T7kEyH7td7tOS75P9WK9trej3ly3TVM3zuvbHafdo3T6/eL/Wz71TTHaKoWT+qdfbSOkfe/UNLes7tYG0srfd5kd8n9lPvbIQu53Ger3GTJ22y8K2kz7+qOqa1DO+Rhjf0xS3u0Umxz5jNMr5hcz/BIekv5R0i+M4exzHMSTdKunUTgu1JR1bWQ4Th8IGdnRlWSdPndGX/ubp8DcxE+uOHD6oRz71pC4+W25bd3RlWTICnTx1RkHgp66/Ym++adk73nijar6vI4cPNi2/+/aDOnnqjI6uLOux1afqy+9783LTXwuS9X/kU0+2lXNsZbntrwtp55y2HQBg+Ip5s61/iHN+t1y8p2C25e4r9uZT+7A9BbMt1+8ptB/33rsO6Q8+/WR9v2I+HA7kLbVtm7Z/fKykQs5o2i7uy5L77b9yLrXehVzL1/8gabB+O23beOySvM6t2z22+lTH+Ej7DOf35HXPnTf2XBaPqeL3cawfOXxQtm3onjtv1OoTZ1Nja/WJs6ljsaMryyoVzLZYi+ucVt6Rwwe1/8o55S3pvj7b0acf/0ZTWf2OszAerWPgK/bm2z6f+Hd173tzOJ6+965DzZ/zXcsyLWnffLFp37SYj2P5kU892RbnRw4f1OJCqSnWjxw+qCv25utt7+7bD+qx1ae0GMVca12PrizLC/yeufLI4YPaN1/UFXvzumJvvhHXb+7c7u9bWdb+K+emLna73afF7+M8lZbDWrc9urKsrz51UZJ08dmybNtoi4k4R/3iP3+5bNto+5zm9+Tb4qU1t8Wfe1ocve2OG+QF7fd+99x5o+b35Ov7rD5xtqneeau/65P8XPvJ853K3i7uM0P9jpn63W6U25Y6bFtq2Xauw3ZzY6xrv3UYpMxBzHp8j+q6Aa2M5D/zySrHcf6VpDdLqkpalfRzrutu9tjtgKSvd/tWSLenAyefvG7bpkyj8ST55BPaLctUKW9qo+JJgRE+/do2tVUJnxhvRU9ZD4LwiddeEMg2wyde+0Eg0zDCJ2EnnhKvQCrkTW1VfXle+IRsKwhSHziUfAK470k1v/vT2af96a2T/mvU4uL8OGZJDqhH7I7b4uL8th5CmMW/HE46hoZtgL9Q7srYHZZhxU2hlAv7liCQZRgyTCnwg565eG4up3LFV80L+4qcZahgGx2f8t2a6/cUrKZ+q5AzVd6syTJNFfOmtjYa3xTL5S1VvMa+liUV7bjfi8tLfzJ2vmjX+9H4aerJp6sXcoZkGGEfWe97jY4PIBxT3EoZjt1++u04PpPb5ixThtkYuySvc7fxj22Zsm1Tm1u1RlxthTEbj1NM01DgB/L9IPy2niHZpiHPD1Tzwp/5KOZNbW75jfGNEcW6ZSpvG6rUAtU8X/m8pWo03oljK47V5Fgsjqf4wUiNWGuMtWqer0LOjMpuLJfn1x/M5clQzQ9kmpJtGcpZZniMOEbzcdswVMxb9XX5nKXAD1T1uo+zEp/JTOTcrPaZrWNgQ4ZqXlD/mRnLlAwvvCaeDJlWON6O1+dsU6Yh1aJtatFY3DYN2bapaq0Rk4V8+L7qBcrZpgI/CGPIMGRZhmwzjL1qrZGfA0mVmi/LMCRTMoLw/qAWfU5e4nhmdJ8Ql2sZYfuRIW0l8r4dlStF3yBKxLVvGKp68bkbqtbCn5mIJ0cGGftnJXa75SnLNJXLGdrc8lLXzcV5K9o37Iu8pj6x4ivMPdH9mmFIfpSjirahzShHBYGi3BfIjr5FXfUCBYmcGARSzg4/x0o1zBG5XBg3QT1PNu/v+43Py/N9KTDa+sy8pY4PCezVN/S6fp3K3kmbH+V9Zp/fgM5E7M7N5foaMzW2ax/HjWvb4lyuKeaS/WxSaS7X1KbmCmbbAwhHXddGHRrtPK0Og5QZ6ye+RhXf0xa3u0FWxz7jMsr5han4o43ruu+T9L5hl+t5vgxFF8ELVEm83ypXG+uqnvz6dp7KZa+xruZpsxa+l6TAkyrVxHpf8qL76uSyOFX50XIrfhOpbHhaij/4mpQ29EjW348GEPG5dHqmROs5D/HZEwCAHrY2En2LJHlhH9ArF8cDQDvax/OkciXRr0R9U73YllxfLvtN/Val5tX329poPnq1kujDPMn3pHK07JqoX0oeK6myWWuq02ayv/S8+gOPkuUP8yFIs2iQfju5bVDzFKj92qeVmRz/qOaploiP+LNOjlOaJsHiMqP/h2VIW9HYKLmfIcn3fG1WGvvXNvymeEiOsZrGYlE8xZpjLWwThqRKdDNoS1q8snkA7SXWxbG9pZYY3fDq12VrIznOqiX2Zfw0aWlj4EYMNw2pJYWfdXJ9rdb8CZrRf/LDdU05aqMxxg4qiRhSOO7fF+XFZCwmt4mDpZo4ZvJ4fuLvb/E+ybycjNeWIuvXor5ddO5Wy3bTOPbvmqc8TzWv87qNcnOf1dwXhX2i1HxvJoWfie/5KidylBLrvehF63JDLZ+7J9W88JjJu/Pk/vujuvmJbVr7zGqXD6tX39Dr+nUre7u4zwyVo76r15ip3NLHddpulNtudulnkzZazmljAnXdaNm2Ux0GKXMQsx7f/cYtsBN8px4AAAAAAAAAMBJMQAMAAAAAAAAARoIJaAAAAAAAAADASDABDQAAAAAAAAAYCSagAQAAAAAAAAAjwQQ0AAAAAAAAAGAkmIAGAAAAAAAAAIwEE9AAAAAAAAAAgJFgAhoAAAAAAAAAMBJMQAMAAAAAAAAARoIJaAAAAAAAAADASDABDQAAAAAAAAAYCSagAQAAAAAAAAAjwQQ0AAAAAAAAAGAkmIAGAAAAAAAAAIwEE9AAAAAAAAAAgJFgAhoAAAAAAAAAMBJMQAMAAAAAAAAARsKedAV6cRznJyX9fGLRP5D0n1zX/fkOuwAAAAAAAAAAMiDzE9Cu6/6mpN+UJMdxrpf0h5LeOck6AQAAAAAAAAB6m7af4Ph1Scdc170w6YoAAAAAAAAAALozgiCYdB364jjOLZKOu657qM9dDkj6+uhqhF3KGMMxDiiDsXvr2z820PaPvv+2EdUE27RrYxdTbRxxKxG7GD5yLqYVsYtpRexiGhG3mFYDx27mf4Ij4aclfWDQnS5evCTfn45J9laLi/M6f3590tXIlElfk8XF+bEdK0uxu93zzmL8TjqGhq3f89mtsTsssxY32zXu6zDOuJWmN3aJz3STvC6zknOJrd5m7RrNSuxK2f5sslq3aa5X1mI3q9dyJ2bxnCTGC0mz+hkPYrdfg1HOL0zFT3A4jpOX9IOSPj7pugAAAAAAAAAA+jMVE9CSXibpb1zXvTzpigAAAAAAAAAA+jMtE9AvkPTNSVcCAAAAAAAAANC/qfgNaNd1/7Ok/zzpegAAAAAAAAAA+jct34AGAAAAAAAAAEwZJqABAAAAAAAAACPBBDQAAAAAAAAAYCSYgAYAAAAAAAAAjAQT0AAAAAAAAACAkWACGgAAAAAAAAAwEkxAAwAAAAAAAABGggloAAAAAAAAAMBIMAENAAAAAAAAABgJJqABAAAAAAAAACPBBDQAAAAAAAAAYCSYgAYAAAAAAAAAjAQT0AAAAAAAAACAkRjrBLTjOPsdx/nH0ev3OY5z0nGcfzTOOgAAAAAAAAAAxmPc34A+IemFjuO8RtL/Kuk/SfrQmOsAAAAAAAAAABiDcU9AX+W67gcl/Yikj7que0LS3JjrAAAAAAAAAAAYg3FPQOcdx8kpnIB+zHGcOUl7x1wHAAAAAAAAAMAYjHsC+mOSzku64Lru45JWJX20106O49zqOM4XHMf5a8dxHhh1JQEAAAAAAAAAOzeJCejvlvRD0fs7Xdf9P7vt4DjOCyT9e0n/RNLLJN3gOM6PjLSWO1Ao5RRYlmqGIdmWcsWcAttSzTAV2JasvC3lrMYyy5JdzMnM2wpsS57ZWHZ+rayaYcq3wv3MvC0jZ0nRvvFyu2grsKyw3OjYZt6WYYevfcuUmbdkWaYsy1RgmaoZhoLofWw767rtM6hhlgVg97IsU1belh/lE9lxng1zZPz/WpRv4/dh/m3knuJcTqW5MKefvXBJgW1pbi7XdKy5aH1gm01lxtvF6+O8bBXiepkychZ5bhfKRfEWx0zc7yt6H9iminPx2MFQYFsyo3g283YYW9E4wIjGDmcvXpai+Iz3i8sI7HD84ZuN8uPxRNjfWspHYxcv0f/G9Yy3yeWtoV0D+vvdIe1zzuWbx7F20ZaVt6PtwrxoF3PyLVOeaUq2JauYk29ZUTxHbSaK7Xis7VthmYraS9x2inO5+hi6tQzZre3BrLezXDHcL14fROUbyfKH3C4wflZ0j1QfL1jp/XJrPoz3sYvh/Vacp4NEHIY5PsznZr7R95u5cNuzFy6F94VRzrYKjdeBbdXHH8l4T4vVRp9gyLcsFVr6j0JiHBJEbS4uN77/7HbusXzRbionX7RH+dFMtXg+4OyFSwosS4VSLnW7uZbPpnWMuV2DlFtq2bbUYdu5ueZz6lbmIMcvtmxbHEK5/Z6T1GjbZy9c7pnTBzmvaWwvg3zGs6rftjurSi0x0K3tbNe4W8Lvuq77kviN67pf7mOf10t62HXdb0qS4zi3S9ocUf12pFDK6dy3N3X8xKrOrW3opuv36/Drrqu/X1oo6V/d9XJVKr4++HtfrC87urKsfM7Uuz78uY77HTl8UPvmC7pUrur9v/t42/LHPn9GN77k+frQw6e1MF/Um37sJXrgodNN2y3MFyRJ74yOs7RQ0rGVZS1EgbVWrur+xDF7rbtqPq+L65XUfTzPH+jaWZbZ8fiDlgVg97IsU5erntaeKTflwGMry7JtUydXz+gHDv49vffBU/V8e/trnfr7eNv9C0V5ks6vbTbl4qMry7pmoahyuaq5uZzOrm3qoU9+Vbf+wAv1oYdPt213Ntp/Yb6on/qn363NLa+pXve9eVn7SuS53SKXt3RhvZIaM3ffflCPfvZr+uk3vExPt8TdvXcd0mdPf1Mvf+nztW++oC+55/SZ0/+zqa9Pjh3ectv1unphT+px3nbHDcrnTb3vwS80xetDn/yqPv/E01paKOndP/v9urxebYv9q+fzqla8HV0D+vvdIe1zfudbvk+Vsl9fdtP1+/XPf/QlevZSpSmOb3/tdXrvg+3xH49zW9vM7a919PCfuPX4PXL4oH77E3+tffN5venHXqrNSk2bW54+/pmvtbWHX3jjy2VaamoPRw4f1PP25mUYhjw/0PrlSlPevufOG/Vbjz6htfVNHV1Z1t49xUlfbmxDY7yw1dwvryxrXyIfxXk7LSe/6uB36TOnv6FXfO93aqvS3L+/7Y4b9MyzGzrwnc/Tt9e32u7L/vQL39APvfy79MBD7fduafeCv/iml6tWC/SBj7bfB37JPacPf+wJveW26/XSF1zd9V706MqyPvPFb+grX19ru1+8b2VZ+/YFbdcqX7R1/tmttnIWn1dQZbM27o8u01rnA+JrtbSvqK2Nan27eAzZaYy5XYOUW5rL6Vsp2z5/oaiN8vbqOsi2xblc23jnaDQG39xmuf2ek5TetjuNdQY5r2lsL6OKx2nSb9udVYO0nZ0Y91dOvuw4zp2O4/x9x3GujP/rsc8/lGQ5jvNxx3G+JOmtktZGX9XBbVb8+gcmSTcfurbp/bm1DT13qVKffI6XHT+xqnPPlLvu98BDp2XIqE8+J5efe6asW5YP1AfUb3jNi+qDieR2Tz9T1tOJ45xb29D9J1ZVk1ST6jcE/a6reJ33GVS34wNAv2qSnk5MPkuNfHJ+LcyV8WSzFObb5Pt42/KWr60tvy0XHz+xqstb4U3p5Wj9zYeureff1u3i/d/wmhfpucQkS7zde36LPLebVDx1jJkPPXxaNx+6VkG0TXLdex88pVuWD9T7/OXrr2nr65Njhxd850LH43zw976o5y5V2uL15kPX1t/XakFq7O9w7lkS/f1ukfY5P/1MuWnZzYeu1fl4LNuUk9Pjv1Obee+Dp5ri94GHTusNr3mRbj50rc49U67n3rT28H/9zhfa2sMDD53W+bUNGTJkm2Zb3v7ARx/XG17zonq7WFvP5Pdi0EOn8cJ7WvJRpUtOPv7gqm5ZPtD0R4p4mw/+3hd13YGrdC7lGA88dFqvf3Ujh3fL5/E+65cr9cnnZDlxnyBJy9df0/Ne9PiJsM5p94vvObGqZy9vtV2rrWp6n7BVbZ+s3u1a5wPia7VZaf4D6+UeY8ztGqTccodty1vbr+sg22502HZjB+X2e05SetvuNNYZ5Lymsb2MKh6nSb9td1YN0nZ2YtzfgL5N0o+3LAskdfv3a7akV0l6taRLkj4u6S5JJ/o54FVXje8Zh2cvXKp/YJI0P5drei9Jxbzdtuzc2oaKebvrfufWNmQa6rivZTbWddo/eYzkchlG4/UA67wg6LjP4uJ827G6ObdW7rusQcueVuOM3VHJ6meV1XptV9bOZ5Kxe26t3DXPJnOl1Dlfer5ff92+LtA1i/P1nN+tjGRe7lTednLmJE1TXQc16tg9e+Fy15iZn8s1xU1yXRy7xbytIAjayki+j8vodzwQbxvrNN7wgjD2d2KQ/n4YZjleY6OO2+1cw7TPuTU3p+XFTjHbmrvj5fH2yfhtfd+6bevytPZQzNsyjcb7tLLi1zV/5+1itxrHeKFT/HYbLyTzUZy3W7eJY9IyO9/feb7fcV23e7dB7yGDIJzYClruzbq1p07rqjVfSy3XrPUet3F+4439LOXzTrHb77Ua1TUdpNxR1HXS2w5WZnrbThvrjOq8xm2ncTvLdvs1GNf5j3UC2nXd7fwbtW9Jesx13fOS5DjOH0haVp8T0BcvXpLvj+evTZZlaWmhVP/g1svVpveStFmptS1bWihps9L4W3vafksLJfmBOu7r+YX6uk77J2UHHL4AACAASURBVI+RXK5o0JK2T7d1lmF03Of8+fUBrpwky+yrrMXF+cHLHqJxDnzGGbu9bPe8J/lZdTLpGBq2fs9n18SuZXbNs8lcKXXOt5Zp1l+3rzN0/vx6Ped3KyOZl3P2EHPmhIy7/Yz7ZnPUsdsrZsLlc6nrPL8Rx4ZhtJWRfB/HXr/jgXjbWKfxhmUYO//8++zvh2GS+X5Wcu62r2HK59yam9PyYqeYjeM/vc00x2/yfc422rbtpz1sVmryg0L9fdpx49e2OYR2kSGzErtSj/jtMl5I5qPW+7t4mzgmPb/z/Z1ldj5GMqa75fNYt7GNEX1hyDD6b0+d1uVss+2adboG1hhjv59clIXY7fdajeqaDlLuKOo66W2HUmbKWGdU5yVNV9zOst1+DbZz/tuJ3bH+BIfjOKbjOO9wHOeE4zjzjuMcdRyn19M7/kjSDzuOsy/a9kckPT762g6umDfD34lZKEmSTp460/R+aaGkK/bm9bY7bmhadnRlWUtXznXd78jhgwoU6O3/+41ty5eunNNjq0/p7tsPammhpEc+9WS4vGW7/VfOaX/iOEsL0e+iKvxLxLGWY/Zal7c67zOobscHgH7ZkvZfOdeWA4+tLGtxIcyV9951qCnfJt/H284VTBUKZlsuPrqyrD2FsOvcE60/eepMPf+2bhfv/8inntQVe/Nt9brvzeS53SRvqWPM3H37QZ08dUZGtE1y3b13HdJjq0/V+/zVJ8629fXJscPf/t1ax+O87Y4bdMXefFu8njx1pv7eto3U2B/G89bo73eHtM95/5VzTctOnjqjxXgs25ST0+O/U5u5965DTfF75PBBPfKpJ3Xy1BktXTlXz71p7eEX3vjytvZw5PBBLS6UFChQzffb8vY9d96oRz71ZL1dLMzzG9DTqNN44b6WfJTvkpOP3rWsx1af0vye9v79bXfcoK8+dVFLKcc4cvig/uDTjRzeLZ/H+8zvyeueO9PvA1efOCtJWn3ibM970aMrYZ3T7hfvW1nW8/YU2q5VIZfeJxRyxlA+i1nSOh8QX6tivnnaZU+PMeZ2DVLuXIdt5wrbr+sg25Y6bFvaQbn9npOU3rY7jXUGOa9pbC+jisdp0m/bnVWDtJ2dMOJ/sjMOjuO8X9KipEOSvk/hz2n8peu6d/fY7yck3SMpJ+lPJN3tum6vHyM5IOnr4/4mXqGU02bFl+f7si1TOdtUpebL8wJZliHbNOQFgYJA8vxAlmEolzPlB4E8L5DvBzINQ7m8qVrVV80LZJqGcpahQOE/rQoCqRZvZxkyTKlaDWSYUuBLXhAob5sK/EBVL1D4oG9Dhhdeh5qiY5uGbKn+kA3LMgde122fQfVT1qS/vbq4OD+OnuOAJhC73SwuzuvWt39soH0eff9tmfxr4aRjaNgG+Ab0roldyzIly1Q1yqm2Zci2Tfm+L8+TLMuQ5wXygjAHW7YhrxbIDwKZhuq5pziXk6HwN7E8P8zhe/Jm08M45uZy4e+jmZKi/GsZhvYUzPqDCi9vhX1CPmfJMKRqzZfvh9/MM/1gqh68NoFvQI9rtH5AY4rdXN4Kf1/QCKTAiOLOkGFE/+jICFTKW9qoNMYOlmWoVgtjuWgbKld81fxAOduU74dxbpmG5vKmLkf77S2FZcQCPwi/JWcEskxTQRCoWgv3K+RNbVX8cAxihm3AtAxVvPDnPCzTVN7Sjh9AGBvm2KGbCX8DeiZy7k6uYdrnbFqGqn5jHJvPhePVWpSTc5YhyzJVqfoKgnA/yzZVrfoyTEmBojiNvvEpKWeb2qr68oOwjZiGoUrVl2UZKuVNVWuBAkk1L2gqwzLDsbVlGmF78MI62ZYhyzTk+4Fq0XaeF8jzg2jcbahS9ertYt/z5mZtXDETsSv1jl/LMhVYhmpeIN+XbNOQpfZ+Oc7bcSxblqGa5yufM8Pxg1SPEz+IYyuQAqlUsFSpBap5Yd+ftw0pitFCzpTnh+sKeUueF7YFywrzeXkrjOs43tNitd4nRPeMpYIZ3otG5RTzpjbjcYxpKJczVK2G9czZ4fE9L6if+5VX7km9Zvmira1qUO8TCjljrA9U6/Mb0JmI3cZ8QHjNi3kz9SFmyTGiZZr1seNODVJuaS4XjXPDbecKZuoDxxplhufUrcxBjl+cy2kjsW2pYLY9gHDQcvs9JynRtqPxe7exziDnNUh7yUrcDvIZz6p+2+6sarSd6L6iS9uRthe74/7Cyc2SbpD0uOu6zzqO8zpJX+q1k+u6H5H0kVFXbhi2NqoyFF3YmqdqzUu8l+J0Vl8mqeZ59WXxH9xqG16jo/UkryUPxvvGyw1J8hrl+tFNpx0t9xP714/tBUoW63n+wOu67TOoYZYFYPfyPF/yfJmK/plPTarVGnnW95pzcPw+zr9x7okHwIaka6J8XK41Z6ZyuVrPv0qUWS57TevDvBwOPON6BVWR53ahaiUcF8SSX7SJl2/W/Kaxg1+L/sbhSeVKuI0tKaiE2z0/EZ/xfptlv+k4yde+mscIlQ2v0Qa8MC69ZDvxPFWHGKz097tD2ufsteTK5BjYVvhFilrVa/wTTV/yatH7aF8rWh6rRuvNaLmvRtvZTOTstDKMRFHJPiFeFq+v168WPrxmFO0C4xeOFxr9cpz/WsV5O3lfZUqqRRPVyThJxpYU5mKpcQw/upe/JjGhakvytmr116pJGzWvqbzE/5piNdknyJO2ys33nlu15rrXEmMev+I3resWzpXNWlOfMKS/R86keD4g/oy3NtIvVnKMKM+rjx13apByN1q23eiwbVxmfTzcpcxBjr/Zsu3mEMrt95ykRtuOz6tbTh/kvKaxvQzyGc+qftvurNpoiYFubWe7xv198mrym8uu625JPPgcAAAAAAAAAGbRuL8B/VeO4/ycJMtxHEfhz2r0/AY0AAAAAAAAAGD6jPsb0EcU/gTHfkl/LmmvpP9jzHUAAAAAAAAAAIzBWL8B7bruc5L+xTiPCQAAAAAAAACYjLFMQDuO81sKn9mRynXdnxhHPQAAAAAAAAAA4zOun+D4K0lPSNon6WWS/ruk05JerPH/DjUAAAAAAAAAYAzGMvnruu77JclxnNdLepXruuXo/Ycl/ek46gAAAAAAAAAAGK9xP4Rwv6StxPtA0tVjrgMAAAAAAAAAYAzG/fMXj0n6Y8dxPirJkPQmSR8fcx0AAAAAAAAAAGMw7gnofynp5yS9Pnr/sKT/MOY6AEAmzV9RUrEwWFquVL0R1QYAAAAAAGDnxjoB7bpuzXGcX5f02wq/AS2FDyZ8Zpz1AIAsKhZs3fr2jw20z6Pvv21EtQEAAAAAANi5sf4GtOM4PyPpWUkXJJ1P/B8AAAAAAAAAMGPG/RMcvyDpFa7rfnHMxwUAAAAAAAAAjNlYvwEt6VtMPgMAAAAAAADA7jDub0B/0nGcn5X0cUkb8ULXdfkNaAAAAAAAAACYMeOegL5XUkHSv0ssCyRZY64HAAAAAAAAAGDExjoB7bpuaZzHAwAAAAAAAABMzlgmoB3HeaPrur/jOM49aetd1/3AOOoBAAAAAAAAABifcX0D+kXR/79nOzs7jvOnkpYkVaNFP+267ueHUbGssixTa+ubqhmGLNOQLcnz/IHLqEny/GDbZYxCVusFYDq05pC8ZajiBTvKKeQljEq32BpW3Pl+oMAyiV/MhFHk+F7HoM1g0iYZk/GxfT/Q+bWyAtuUAtEukCn9tpFB2lK87bm1smSZxPwuly/a2qoGOnvhkizLUiFnqLJZm3S1ZspYJqBd1/2V6P9vHnRfx3EMSS+WdK3rurvi07csU2vlqu4/8ec6t7ahpYWSjq0sa2Eu13dCbJSxuu0yRiGr9QIwHdJyyNGVZT30ya/q8088PVP5EtOvW2xJGkrcWZapM996Tu/+yOeJX0y91jZz0/X7dfh11+n4EPMzOR9ZM8mYTDv23bcf1KOf/ZrueN11tAtkQr9tZJC2RF+ApHzR1vlnt5rGG0dXlrX4vAKT0ENkjuMgjuP8d8dxvtzpv167R///pOM4f+k4zs+Pur6TVpPqiVCSzq1t6P4Tqxok7IdRxihktV4ApkNaDjl+YlU3H7q2/n5W8iWmX7fYGlbc1aT65PNOygGyoLVd3Hzo2vrNoDSc+CbnI2smGZNpx/7Qw6d186FraRfIjH7byCBtib4ASVvVoG28cfzEqraqwYRrNlvG9RMcO5k0XpB0UtK/lJST9GnHcVzXdf+kn52vumrvDg49GefWyvXAbyzbkAxDi4vzYytjFIZRr0nWf5ymMXZbZfWzymq9titr5zPK2O2UQ+ajb5TG70eRL7N2nSdllq/DsGO3W2zVX6esG+QaZ7W/z4rdcA1GPV4Y5zVsjef5udzQ43sUbWY3xNkojGOsm+XPJq7bJPN4t3FV1vqSrNRD6j92s1TnYZnEOfXbRgZpS7tx/LSb47aXsxcupcaD5we6Zhdej1HFwLh+guO/SpLjOI9I+nXXdR8bYN//Jum/xe8dx/mPkn5UUl8T0BcvXpLvT9lfLSxTSwulpgawtFCSgkDnz6+Pr4xR2GG9FhfnJ1r/cSbjLMXuds97orHWwaRjqJtRXueZid0OOWS9XG16P+x8meW4GadxX4dxD4CHHrtdYit+veN+Oqv9fQZMst3OSs4d+zVsief1cnX48T3kNjNr/cOsxK6U7c+mqW6TzONdxlVZ6kv6+SyzFrtZjr/tmtg59dtGBmlLGRk/EbfZYFlWajxYprHrrke/MbCd2B3LT3Ak/L6kX3Yc528cx3mH4zhX9trBcZxXOo5zc2KRocbDCGeSLenYynKYAKX67xEN8teCYZQxClmtF3aPStXT4uL8QP/NX1GadLURScshR1eWdfLUmfr7WcmXmH7dYmtYcWdL+qWfuIn4xUxobRcnT53R0SHnZ3I+smaSMZl27LtvP6iTp87QLpAZ/baRQdoSfQGSCjmjbbxxdGVZhZwx4ZrNFiMIxv8NS8dxrpP0E5LeIOkvJP1b13VXO2z7v0n6VUnfr/AnOP5M0s9E34zu5oCkr2fpW6SDsCxTRs7S5lZt209CzuoTvndSr0n/RW5xcX4cGeiAMha7i4vzuvXtHxton0fff1sm/1qY5XMZZd1mKXZbc0jeMlTxgh3lul55adK5Jysm8A3ocY36DmhEsdsttobVT1911V5d+HY5c/39pE34G9AzkXMncQ1HkeN7HWMnZc5a/zArsStl+7Nprdsk79viY/u+ZFmGAgVSoEz1JX1+AzpTsZvl+NuuSZ5Tv21kkLYUbyvDkIJgIjFP3GZHvmhrq9oYbxRyxq58AOEA34AeOHbH/Q1oOY5jSnqRpBcr7NfOSfo1x3Helba967p/JOkTkk5LelzSR/qYfJ56nudrYb4oOwhkeP62EqHn+TI8f0dljEJW6wVgOrTmkGrF23FOIS9hVLrF1rDizjQN4hczYxQ5vtcxaDOYtEnGZHxsK/C1uDAno+bTLpA5/baRQdpSvO3SwhwxD1U2azI8T9dcvUeG5+3KyedRG+u/MHAc592SViR9XdK/k/TPXNetOY6zR9L/kPQrafu5rvvLkn55XPUEAAAAAAAAAOzcuL8B/R2Svuy67g8o/Ebz/+s4zn7XdS9LumPMdQEAAAAAAAAAjNC4J6CvlfTH0eszkj4t6bckyXXdT465LgAAAAAAAACAERr3BPRVrut+SJJc1910Xff/lnTNmOsAAAAAAAAAABiDcU9A247jfEf8xnGc/ZLG9YR7AAAAAAAAAMAYjfUhhJI+IOlLjuP8saRA0i2SfmHMdQAAAAAAAAAAjMFYvwHtuu5HFE46n5b0BUk/7LruR8dZBwAAAAAAAADAeIz7G9ByXffLkr487uMCAAAAAAAAAMZr3L8BDQAAAAAAAADYJZiABgAAAAAAAACMBBPQAAAAAAAAAICRYAIaAAAAAAAAADASTEADAAAAAAAAAEaCCWgAAAAAAAAAwEgwAQ0AAAAAAAAAGAkmoAEAAAAAAAAAI8EENAAAAAAAAABgJJiABgAAAAAAAACMxFRNQDuO828cxzkx6XoAAAAAAAAAAHqbmglox3FulnTXpOsxLJZlKrBM1QxD+VJOgWWpZhgKLEtzc+H7sxcuKbAt5Us5mXlbvmWpZpiy8rZkW6qZpmRbMvKWzLwtM28rsMNtAitcppwlu5iLlhsKbEvFucbxzLwtI9c4dr6Uq5dt5MIy4v3m5hrltK7Ll3L1+jSdjx0eP65bca65LoW5nGqGIUWvk/vlS7l6veyiLcsy5ftB/brJsmTm47qbsqL/zLwl3zIV2KYC25JnhtcjsE3JsmRZZurnEJcBYHQsK8xhRt5WYS7XyF2WpVKUH1rbbr6UksPsRt4qzeXqeTPeJshZCuwwT3mm2VhmWSqUcpO+DJgyubzViMVE/ym7ue+2or467nc902yO36g/i/u4sK+y6tsaeSvqk035likjb9fbSxzHZr6xfRzj37p4uVF2S/+b7N+TY41c3pr0ZcUYJMdF8dgp3xInjXGmqZphqjjXeH/2wmUpZ9Vztuzm+PItqymGZcfjwzB/WwU72s6UZ5r1nO9bjRi2CnZTTAd2WI4VjTPjthb3CcpZ9fFcPI4LbLNtHByOjy3Gdugp7X4gXuZFOTOwTdlFu6mtKDFmyZVyMqLxTDK2W/uAfDSG+dbFy/Ucrpb7N6vQWOZH7ShXyjXuBQt2U1vMFXP1+sTtLW07qxDWP1eM+4OwnsqFfU8ub2ltfTPlHspquveL61xvd4myjHzvtpeWl9Luz5LXfm19c2ractO9fDQ+6LZdchzRq8xJbdvvOWWproOUOcnzyopBrsWs2u3XYBznbw+9xBFwHOdKSe+RdL+kfzTh6uyYZZlaK1d1/4lVvXb57+vlL32+jp9Y1bm1DS0tlHR0ZVkPffKr+vwTT+um6/frrh97qdbWt/TAQ6e1MF/Um37sJXrgodP17d/xxhv1vL15rT1X0Qc++nh9+ZHDB3Xl84r69npF730wLP/1P/gCveqG79LxE6upZSWPHZfx25/4a73q4HfopS+4uqme8bq19U0dXVnWF77yLZU3q/Xy4+3uvWtZuZyhK/bk9fTaZtu5/u0317RZ8drKj8v8k9X/oXvvOqTnzed15lvP6d0f+XxqHe5bWVapZOv8+bI+/pmv6dYfeKE+9HDj3O6+/aAe/ezXdOfrrtO+qDHFn0O8zbGVZS3M5eR5/oSjBJg9lmXqcs3TxuUt7Zsv6NuXPG1Watrc8rS3ZOu5sqGHPvnVtrabzEvJHJZcv6dk65d+/S/a2vs/ftUL6zkiXnb4dddpaV9RWxvVSV8STIFc3tKF9Yr+57l1fcfSfMf+89jKsrzA11e//ky9P+vUx3/lby/opS+4Wp/54jd040uerw89nN6/Hzl8UMVCOFH8G7//V1pb39S9dx3Sw3/itvXTcV+c7MPvveuQ/suff13r5YoOv+66tnpcPZ9XteJN+ApjVCzL1OWqp7VntupxlZZDj60sK5B0/MSq3nLb9fqOpXl9e32rKRb/9U/epGdrQdOY6d67Dumzp79Zj+G0sdk9d96o+bmc3vWbn9PCfFE/9U+/W5tbXr3sm67fr9tf6+i9D55Kzd9/+oVv6Ide/l1NdamP5374OuVsUw9+4iv68VterFotaBsHFwuWSnlbe3IWYzukSt6XxbFz38qycjlT7/zw5+rLfulf3CTTMPTspea28bY7btDqE2d1y03XtrWbY1E570qU03qf15qb77nzRu2dy+lXf/NzqcvS+oo413/5/7tQ7yO+vV5J2W5ZV+zN6ZnnKqlt7vDrrmvql7q1yY9/5mv68VterK2K13SMe+68UbmcIUmpbS8tL8XXvNP9WXz8O153Xebv0+bmcjqbcr97zUJR5XJ14O2ysO2kjz+rdc2SaazzsO32azCu85+OPyNK/0HSfZLWJl2RYahJ9U711Tc2bgIk6dzaho6fWNXNh66VJN186Fo9/Uy53kG/4TUvqr+Ot/83v/O4PE/1QXe8/IGHTss2zfrksyTdsnygfry0spLHjst4w2tepOXrr2mrZ7wu3u/VN35XU/nxdu99cFWWYapWDVLP9XtfvD+1/LjMsIxT8j3VJ5/T6vCeE6uq1QI98NBp3Xzo2vrAKd72Qw+Hy99zYlW1ls8h3ub+aB2A4atJevpiWc9dqki+oXPPhK8feOi09pYK9fzT2naTeSktxxyP2n5ae0/miHjZ8ROr2qxk9+YF2VLxwom56w5c1bX/vP/Eqp67VGnqzzr18fE2tywfqMd7WpkPPHRaz12q6LlLlXocv/fBU6n9dFof/t4HT+n1r35RPe5b68Hc82yrSU1jSCk9h95/YlXfXt/UubUNveA7F3SuZZ9zaxs6H23XPL471RTD8fJkTIZj03I9xuOcH29/86Fr6xNh8f7J/P36V7e3i/p47rdW9fQzZd186FqtX66kjoOfu1TR08+UGduho7T7gfecCGMrucw2TZ1fa28bH/y9L+qW5QOp7eb+E6s611JO631ea3v8wEcf1/m1csdlaX1FnOuTfUT6dqsyAqNjm2vtl1rXJ9tk3O5aj/GBjz5e77fS2l5aXqrfx3X4POLjT8N92uUtP7W/vbzlb2u7LGw76ePPal2zZBrrPGy7/RqM6/wz/w1ox3F+UtI3XNc96TjOyqD7X3XV3uFXaofOJQYVQdCYNGms39B89Bfg+P/xNvNzudTtTUOpy1vLt8zeZc0nvmofv+9Vz/hYyfJb6+f5fuo6z/ebzrG1/p3OJa0O8XXodm7n1jYkw+h4TBmGFhfnNWlZjN1BZeE6DkuWzyVrdesUu+fWyirmw27H8/366zgP9Gq7krrmmLR9WnNEvMzzA10z4HXL2nWelFm+Dmmxe/bC5aYYlTr3n8W83dRXdeq34uX99MlxOynmG8vS+ulO6yyzc9leMHg7mDazHK+xXjm30ziwsd1GW25u3SZtWWsMJ5cnYzIuu3VMGy/rNl7rVn5cdrJtdDqvUY/tdkOcjcI4xrq9PpvkfVljWSJ2IqbRvR10WtdaTut9Xr/7JNtRpzq0lt/pvmsnbS5e3+0YTfVvaXtpealeTpf7s+Q9XBbaW6fYPXvhUofr3tzf9rtdFrad9PFnta6TsNO4nWW7/RqM6/wzPwEt6XZJ1ziO8yVJV0ra6zjOB13XfVs/O1+8eEm+H4y0ggOzTC0tlHRubUOGYdRfx5YWSlqPvua+Xq4qZze2WS9XU7f3A6Uuby3f89WzrPXEV+zj91fvK3XdNj6W5wcd65czzdR1lmnKD9L3M6KBSNq5pNUhvg7dzm1poSRFE9tp2ygIdP78eupHN84BT5Zid7vn3ek6TlKWz2WUdctE7FqmNiu16GXjdZwHerVdqTmHJde3Hi5ZVjJHxMss0xjoM11cnM9kPI/buK/DuG8y02LXsqymGO3Wf25Wak19Vad+K17eT58ct5NqrdFvpfXTndZ5vjqWbRmDtYNpM8l2m6Wc22kcGEvGWZybW7dJW9Yaw8nlyZiMy24d08bLuo3XupUfl12tBW3lth6729hup2atf8hE7A5JX5+NlX5/Uo+diB90bwed1rWW03qf1+8+yXbUqQ6t5Xe679pJm4vXp7Xn1Pq3tr2UvBTv1+3+LHkPl+X7tHjM0H7dm/vbfrfLwraTPv6s1jU2TXE7y3b7NdjO+W8ndjP/Exyu677Wdd3vdl33eyX9a0kf73fyOatsScdWlrW0UNKnH/+GjkavpfBDPrqyrJOnzkiSTp46o/1XzunI4YNaWijpkU89WX8db/+ON94oy5LuufPGpuVHDh9Uzfd1712N8h9bfap+vLSykseOy3jkU09q9YmzbfWM18X7ffrxbzSVH293713h72LaOSP1XL/0N0+nlh+XGZZxSKYl/dJP3NSxDvetLMu2DR05fFAnT53R3bc3n9vdt4fL71tZlt3yOcTbHIvWARg+W9L+q+Z0xd68ZAZaujJ8feTwQV3a2Krnn9a2m8xLaTnmaNT209p7MkfEy46uLKuYz3z3h4zIW9LRlWV99amLXfvPYyvLumJvvqk/69THx9s8tvpUPd7Tyjxy+KCu2JvXFXvz9Ti+965Dqf10Wh9+712H9AeffrIe96314DmEs82WmsaQUnoOPbayrH3zRS0tlPS3f7empZZ9lhZKWoy2ax7fHWqK4Xh5MibDselcPcbjnB9vf/LUGd1716GO+fsPPt3eLurjuTcva/+Vczp56ozm9+RTx8FX7M1r/5VzjO3QUdr9wH0rYWwll9V8X4sL7W3jbXfcoMdWn0ptN8dWlrXUUk7rfV5re7znzhu1uDDXcVlaXxHn+mQfkb7dsgIj6NjmWvul1vXJNhm3u9Zj3HPnjfV+K63tpeWl+n1ch88jPv403KftKZip/e2egrmt7bKw7aSPP6t1zZJprPOw7fZrMK7zN+KfOJgG0U9wvNp13ZU+Nj8g6etZ+hZpkmWZqkny/EBzRVtbFV+e78syTe0pmLq85cvzA1mWoULOVM0LVPMC+X6gQs6U5wfhetOQYUqGwm8Ke34gzwuXW5YhPwhkW6aqNT9cbhmay5sqR+Xnc6aCIFC1Fu5TyJuqVMN1tmXIMAxVqr4sy9CevKnLlbCcnN28rpAzVd7yZJuG8jkzOp/weDnblO8H8oJApZypjUqjLsW8qcsbnmwrPPZmy3mXNz1ZpqFczlBQ9bVv35wufLsc1s80ZVoK62Aa9QFJYBmqeUH929O+H8g0DMmQjECyFNQfXpH8HOIyuj3YYnFx3ui4cngOKGOxu7g4r1vf/rGB9nn0/bdl8q+FWT6XUdYtK7FrWaZkmfIl5W2j/q1Ozws0VzBVTvw2c9x2C3lTW9VG3ijlE3nENDRXMGUo/O0qL4jyohF+kcYyDVVrvkwzzAHypWLeHPgBhLP2DbftmsA3oMcRt1KP2M3lLVU8hbFYObIJYgAAIABJREFUafSfvh/2zZZpaE/B1FYtUNULlM+bqlZ9+X6gUsFqxK9pKJcL++RCztRmxY//xbF8P5Btm1IQlmEaYT9uSvIl1aI4tszwXxv5flB/X635Mo2wr6xWg6b+d3PLq/fv9b7ZNJS3NPMPIJzwN6Azk3PjcZHvKxqnGdpKxEnTODMItLdoqVJrvI/Hg3nb0FbFD2M+iq9K1ZdhSgrCGLZMQ6ZpNMZmifFiEATK2eGNTM0LFARhDNv1+gX1XG1E+Xur6tfbmhf1CUbUKCyF51yTJCP89nZyHGxaYdo3vGCkDy2btf4hK7E7DP1+Nmn3A1IYW76vKC6D6J5Gqnm+/CBsT3Ht81E78qN2E8d2HMdxXBYKYS6u53DLkO8FqvmBrCjvG0Y4LqpFMZ+zw3YV71fIm+H6qC3mbVMblfA+zI7apR9E94yJ7WzLqI+NKtXmMZMhyTakwDRVrXmSjKZ7KNNQ1JY95XOWfD9QoCBsd14jn5imISno2vbS8lLa/VnztbcUVL2puE+bm8s17uWj8UHaQ7wa2zXmADo97GvS2/Z7Ttmq6yBlTu68pi1uZ9luvwaDnv92Yjfrf0Rs4rruCUknJlyNofA8P+zoJVU2qvXX8jyVy54MSddEg6ZKLbxBNKP/vGiCxpIkXwqk+uCnUY7kR/eVtarXWF6TNmqN935UVrxPZcOrlx1EZcf7lRP7BdXmdZWaF772w9fJ49Wi+huSNlvqshXvF7020sr0pKgImaYhw/Pr18r34roHqt9Ge81f7W/9glfydjv5OTSVAWAkPM9X/O9EtyqN5YakjSj3xeK2W9lozg2byVzhhfvFZcR5KH7vq5ErY1sbtHQMploJYy4Zi639Z7nc6Ktr0XaWpMqG3xSvNS8qq+a19VVBskxJgdfos+I49qV62fH750fjhZqntv432b8n61GlGewKYc5tjCHlhQ/WbB1vSY1tNst+/f3+KLYCNXJ2Mr7iMqXmmIzjzIvHb1F94nZTj31f8mrNMV2vu5rbmpVc0fhfvd/w1dzWAq8xPga66XQ/UI/L+J6q1ohfU5L8RvxVE/c7cSx7tXDXZFxWorFOnLf96Ncq4rzvJ3JzvMyL+oL6veBWoq+ohceOxz9xu0zbzouOFfcjyTFTIKkqaXFxrj5pn7yHiu83wzZZq59rU7urNdpct7aXlpfS7s+S135hYc/U/KGnXK423cvH44NO2yXnAHqVOalt+z2nLNV1kDIneV5ZMci1mFW7/RqM4/x3x/fJAQAAAAAAAABjxwQ0AAAAAAAAAGAkmIAGAAAAAAAAAIwEE9AAAAAAAAAAgJFgAhoAAAAAAAAAMBJMQAMAAAAAAAAARoIJaAAAAAAAAADASDABDQAAAAAAAAAYCSagAQAAAAAAAAAjwQQ0AAAAAAAAAGAkmIAGAAAAAAAAAIwEE9AAAAAAAAAAgJFgAhoAAAAAAAAAMBJMQAMAAAAAAAAARoIJaAAAAAAAAADASDABDQAAAAAAAAAYCSagAQAAAAAAAAAjwQQ0AAAAAAAAAGAk7ElXoB+O4/yqpH8mKZD0H13X/cCEq7RjlmWqJsnzA9mmKdOSKlVflmnIluR5fsft07bptR4AsiBftLVVDeT5vizTVDFvarPikbuADtL6d0mqSZIhKTB07pmyZJlN7YdxAYatW0y1rstbhipe0Ff8EavIsl7x2TquKeQMVTZrE6svZtMgeXKacuqk6xof/9xa+zgK2G0KpZw2K77OXrgky7JUzJva2qgO9RiZn4B2HOcHJb1G0ssk5SR9xXGcT7iu6062ZttnWabWylXdf2JV59Y2tLRQ0pHDB/Xbn/hrra1v6tjKshbmcl23j7fxPL/negDIgnzR1vlnt3Q8kauOrizroU9+VZ9/4mlyF9AirX+/b2VZuZypBz/xFd36Ay/Uhx4+3db3S2JcgKHqNtaUmuPtpuv36/DrrmvK9Z3ijzEssqxXfHYa1yw+r8AkNIZmkDw5TTl10nWd9PGBLCmUcjr37c22/mxpX3Gok9CZ/wkO13X/q6Qfcl23JmlJ4aT55cnWamdqUj3RSdK5tQ098NBpveE1L9K5tQ3df2JVtR7bJ7fptR4AsmCrGtQ7NSnMVcdPrOrmQ9fW35O7gIa0/v09J1b19DNl3Xzo2vrkc7wubj+MCzBs3WKqdd3Nh65ty/Wd4o9YRZb1is9O45qtajCZCmMmDZInpymnTrqukz4+kCWbFT+1P9usDPePMZn/BrQkua5bdRznXZLeIen/kfR3/e571VV7R1av7Tq3Vq5/sI1lG5qPvkVybm1DMgxJ0uLifMftZRh9rZ81s3hOabIYu4Oapc8qy+eStbp1it2zFy51zX3x+6zmrizWaRJm+TpkLe926t+LeVvFvDr2/fXXKetm+fNLsxvOd9Rx22usWX8dmZ/L9R1/szKGnaa6Zsk4cu5OPpte8dlpXOP5ga7p47hZjRvq1Vu/sTuMOg+SJ8eRU4dVzqTz/6SPPwnjjNtpt9uuwU77s35NxQS0JLmu+yuO47xP0qOS3iLpN/rZ7+LFS/L9jP0V2jK1tFBq+oCXFkpaL1frrxWEdT5/fr3j9gqC/tbPkMXF+Yme0zgTUZZid7vnncX4y/K5jLJuWYhdy7K65r74fRZz16RzT1aM+zqMe/CXpbwrqWP/vlmpqVoLOvb98evdMC7oZpLtNgs5dxjq17DLWDN+Ha9bL1f7j78ZGMPOWv8wK7ErDeGz6RGfncY1lmn0PG5W42aa65W12B3atRwkT444pw41Piad/yd9/MjMxu0U243XYDv92XZiN/M/weE4znWO43yvJLmuW5b0+wp/D3pq2ZKOrSyHCU6q/wb0I596sv7bQ3aP7ZPb9FoPAFlQyBnhb0klctXRlWWdPHWm/p7cBTSk9e/3rSxr/5VzOnnqjO6+/WBq38+4AMPWLaZa1508daYt13eKP2IVWdYrPjuNawo5YzIVxkwaJE9OU06ddF0nfXwgS4p5M7U/K+aHO2VsBEGGvumTwnGcH5X0LkmvlBRI+iNJH3Fd96Eeux6Q9PXMfZspknziq22aMi2pUvWbnv6a/MtLryfETvoJsuMy6b9GLS7Oj2NEeUAZi93FxXnd+vaPDbTPo++/LZN/OczyuYyyblmJ3danxRfzpjYrXuZz16RzT1ZM4BvQ47qLP6CM5d1YWv8uhb9dKENSYET/D5raz24ZF3Qz4W9AZyLn7lS/Y9HWdXnLUMUL+oq/aY/VWesfZiV2peF8Nr3is3VcU8gZfT2AMKtxM831ylrsDvNaDpInR5lThx0fk87/8fFlGG3jqHGZ5bidVrv1GhRKOW1W/Hp7LObNrg8g3E7sZv4PPK7r/hfHcZYlnZbkSXqkj8nnzPM8X4aiD8Dz5Hvx60Bez+3bt+m1HgCyoLJZa8p9WxseuQvoolP/nhzxxQNlr4/9gO3qFlOt66qe+o4/YhVZ1is+W8c1FQIYIzBInpymnDrpusbHTxtHAbvN1kZVhqRrovawtTH8FpH5CWhJcl33nZLeOeFqAAAAAAAAAAAGkPnfgAYAAAAAAAAATKep+AY0gHTzV5RULAzWjDe3alp/bqP3hgAAAAAAAMAOMQENTLFiwd7WQ+uy+JP6TKYDAAAAAADMnlmegLYkyTTH8VDR0Zn2+o/ChK/JAUnflNT78dbbN/LYHdc1HOQ4251Mv5zBcxm3Put2QDMQu5M0q+c1qDFfhwMafdxKMxC701z3UZrgdTmgGcm5xFZvM3aNDmhGYndcx9iurNZtiut1QBmL3axey52YxXOSGC8kzepnPIjdfg1GNb9gBEGwzSpl3islfXbSlcBM+geSnhph+cQuRoXYxTQaddxKxC5Gg5yLaUXsYloRu5hGxC2m1UCxO8sT0AVJhySdleRNuC6YLaP+CyWxi1EhdjGNxvENaGIXo0DOxbQidjGtiF1MI+IW04pvQAMAAAAAAAAAJs+cdAUAAAAAAAAAALOJCWgAAAAAAAAAwEgwAQ0AAAAAAAAAGAkmoAEAAAAAAAAAI8EENAAAAAAAAABgJJiABgAAAAAAAACMBBPQAAAAAAAAAICRYAIaAAAAAAAAADASTEADAAAAAAAAAEaCCWgAAAAAAAAAwEgwAQ0AAAAAAAAAGAkmoAEAAAAAAAAAI8EENAAAAAAAAABgJJiABgAAAAAAAACMBBPQAAAAAAAAAICRYAIaAAAAAAAAADASTEADAAAAAAAAAEaCCWgAAAAAAAAAwEgwAQ0AAAAAAAAAGIlZnoC2JR2I/g9ME2IX04rYxbQidjGNiFtMK2IX04rYxTQibpEJsxyAf0/S1y9evCTfDyZdl21ZWJjT2lp50tXIlElfk8XFeWMMh8lc7E76ug/TLJ2L1P/57NbYHZZZi5vtGvd1GFPcSlMeu8Rnuklel1nJucRWb7N2jWYldqVsfzZZrds01ytrsZvVa7kTs3hOEuOFpFn9jAex26/BKOcXZvkb0FPPtq1JVyFzuCaTMUvXfZbORZq988kqrnOI65BNfC7puC47xzXsjWuUXVn+bLJaN+o1PNNY515m8Zyk2T2v7eBacA1Gef5MQAMAAAAAAAAARoIJaAAAAAAAAADASDABDQAAAAAAAAAYCSagAQAAAAAAAAAjwQQ0AAAAAAAAAGAk7ElXYBpYlqmaJM8PZJmGbEme50uS8kVbW9VAnu/LMk0VcoYqm7WJ1hfA7CuUctqs+Dp74ZIsy1Ixb2prozrpavVt/oqSioX+u6DNrZrWn9sYYY0A7Hat471i3tJmxWeMl0HdxuZZlqx3PmfK96Sa7+/4HKb1emxXr/PN5S1VPOnshcuyLEt5S6pWvL73B3Yinh+Ix+jj7juIb2zHtN9bYjowAd2DZZlaK1d1/4lVnVvb0NJCScdWlrUwl5OVM3X+2S0dT6w7urKsxecVuEEBMDKFUk7nvr3ZlnuW9hWnZqBQLNi69e0f63v7R99/m9ZHWB8Au1vreO8tt12vl77gasZ4GdRtbJ7lSZZkvRfmi3rTj71EDzx0esfnMK3XY7t6nW8ub+nCeqWt7V49n1e14u2664Xxyhftic4PEN/Yjlm4t8R04Cc4eqhJ9QQuSefWNnT/iVXVJG1Vg3ojjdcdP7GqrWowuQoDmHmbFT8192xWGFgCwHa0jveWr7+GMV5GdRubZ1my3m94zYvqk8/Szs5hWq/HdvU634qn1LYbfwF6t10vjNek5weIb2wH95YYFyage/D8oN4QY+fWNuT54c9upK+joQIYHXIPAAxX63gvCDqN/8izk9ZtbJ5lyXrPz+WGdg7Tej22q9f59hoj7bbrhfGa9Bid+MZ2TDpusXswAd2DZRpaWig1LVtaKMkyDVmm2WEdlxXA6JB7AGC4Wsd7htFp/EeenbRuY/MsS9Z7vVwd2jlM6/XYrl7n22uMtNuuF8Zr0mN04hvbMem4xe5BRPVgSzq2slxvkEsLJd23sizLMuQHgd79M9+vm67fX193dGVZhRwJHsDoFPNmW146trKsYp6UDgCDyOUtBZYlzw90/1tfodf/4AskSatPnA1//zCRZxnjZUPa2PzYynLmH2yTtwy9+2e+X+/7+VdqT8nWPXfe2Pc5WJapwDJVMwwFlinLavT303o9BhVfA99vv/9Knm/eUmrbzVvhelvSfSn3drN2vTAZhZwx0b6D+MZ2cG+JcSEX9eB5vhbmcjr+1lfI8wPZpqnNak2/+G//rOmH/d/yT75Hlaov2zbkVfmnCgBGx/d85XOmfvYNL1Mxb2uzUlM+Z8rn4SIA0LdODyu77QdfKM8LVMxbOv7WV8rzfVmmqULO4AGEGdA6NrdMQ3a0PKssy9TF9UrTg8He+Zbv03t/7pWqeX7Xc+j1ULFpvB6D6nQNfur13yMFajrfasXT1fP5sO0GgSzDUN4Kl8dyLWOoXI5JFgyHV/W1p2TrnW/5X2Qakh9o7PMDxDcGxb0lxoUJ6D54ni9D4cUKFOidH/5c2w/7/+Rt36P7T6xqaaGk4299hfh+DIBRqXhqykOSotzzSnIPAPSp08PKjr/1lTI8X1sbjfGfPE+J+StMWHJsLi9Q1j+atAeDvfPDn9Pxt75CdhB0PYdODxVL3m9M2/UYVNdr4Plt51uteDIkXbM4r/Pn11X1mstKH0Nx/4adq0n6pV//i4nFF/GN7eDeEuPCn8MG1OmH/efncvXX/Mg/gFHiQREAsHPkUozLTh4MxkPFhnsNuJ4YpUnH16SPj+nEeAjjwgR0B8nfWjPzlmRZqhmdHzy4Xq7WX1tm+m+0AcAw8KAIAEgX/6ZzzTB0fq2sXPzDr2r/HV3bIpdiPDo9GMzuEmtxvJp9PFSs229Ez4JBH6wWX49za+W269G5rNm9fhifUT0EsN82zkMIsR3cW2JciKgU8e+MHf21P9f7f/eL+rvzl3Xvr/2Zfur4Sf3GH3657cECRw4f1COferL+24G/8Ydf1k8dP6mjv/bnWitXGcQAGKpSwUx9wEmpQK4BsHvFv+l8NBqz/f/svXucHNV17/urvauqe3rUklrSjCwHkGwjCpBFPIBGGIFxJKzziQmWHd0TDfYBDbYgtpKgazsnlgTnAAlowM8LucEJD3sk+9oi58gOEPCNjBQbDEYjgxIDlgrZwYjYskaPltQz/aiux/mju2qqu6t6uqbfPev7+eij6apdu3btvfbaa+2qvfbmv/sJTiQ0SDIvsO1sGy2t6SWb7rg3KyOIWuG1MdimgT6ks7qnn+CW1y9962VsGujz3WTQS7Y7zf8IstGiuz4+dc8PS+pD5t6bxJH/RtQCP/mS+dQngIP08Xrcn+h8yLckGgXFgPbAHWdsw5qleGDnAWdJwr7XjwGAsymNyBgYBz7/iUvBGcPD//RzJ41XjDaCIIhqSWVM7Nx9CBvWLEU0IiGRzGLn7kO49aOXkK4hCGLaUi6mM+Adg/dLf3F1wUaDxZuVEUQtMAwTs6Kys8FTIpnFjqcPIp5Ie/oJbl9kNJ7CjqcP4jNrL8Hv9UTBWeGme5XEiG53gmy0OFl9aIZVYEPN7A5hxzOvk/9G1IRi+Zqw0ZdOWZ6C9PF63J/ofMi3JBoFTUB74I6dFI1IJfFw9r1+DJ/6iJnfNMSAaeQqUjdNx3ixsWMuUUUTBFErjLyuKdY3n/qISbqGIIhpy2QxDL3OZbIGRMtyNhrM0twzUSe0rIm7H91XctzLTyiO46oeiePuR/fh4S2rIBRtMlgu5msn2QSVbrQ4WX0YplVgQw1tXEH+G1EziuXL5lMfee+U5SlIH6/H/YnOh3xLolHQN/UeuGMnJZLZiuMoUcwlgiAaAcXpIgiCKKWcbiQbjWg2QWSwXmmnA5PVR/H5IL4eQUxGPfoj6QOi3pBvSTQKkigP3HHGdu09XDbumt91k6UlCIKYKiHJO75bSCLjkiCI6YvM4RP7kmw0ovkEkcF6pZ0OTFYfxef37H+rRG9M5/ojqqMe/ZH0AVFvyLckGoVgWVazy1AvFgF48+TJMZhm8GfknEFHbhmLLDGYRi7ERrmYY8XXTZZ2Mnp6ojh+PDGlazuVZtdJT0+0EVp4EaqQ3XrQ7HqvJZ3yLHJYRCZrObomJAnQ0rpv+laT3Z6eKK7//BMVZ/zUV9Y0u+93hNxUS6ProUFyC7Sg3g0CyecEksyhGbnlpCJnkNhETOda2miT0Wo6d6qQbE1OkDoKIoP1SjsZnSC7dn1AEIB8iB13fRTXl8wFaIbVEN1g06p9q53L1SqyO5n8TYVm6YNiWlU+qqWZz9UqchvUt+xkOlXOK6XS55+K7NIX0D4YhgnByMV5NjUDMHIxAgXDLKvA3ddNlpYgCGKqaGkdgmFgwbxuCIYxbQ0EgiAIN1nNgJC32XpikYINBclGI5pNEBmsV9rpgF0fvbGIZ30U11dOb1D9EbVhMvmrJk/SB0S9IN+SaAQ0AU0QBEEQBEEQBEEQBEEQBEHUhaaGA1IU5csA5qmqOqgoyvsAPApgJoDnAHxaVVVdUZTzAHwbQC8AFcAnVFUda1qhCYIgCIIgCIIgCIIgCIIgiIpo2hfQiqKsArDedejbAP5cVdULAAgAbskffwjAQ6qqXgjgZwD+RyPKxzmDxRl0QQCTOcA5dEGAxRnCEQmW/VvkkMJS/hyHHBbBZA6TMxiMweIclsiccwRBELUgktdDR0+MweIckYjU7CIRBDGNkcPihG1Ugc1j21nI20j2dW4bi8kiBHHC/uJ8wmyVZF5wnSTzuj6f2y4sLgvRmVTb5u7rwTnEsAiTM+gCA0QOJnMweaLfQCyV4+I8mFzaHyZLMx1kdyr6oKDexFxb2Nd3RSQwWZzw58QJf26yvKdDfRPlsWVgNJ4sKwNBxs0gMh7qkgrShrqq9xEqfaagNHosJ/wh35JoBE2ZEVUUZQ6AewFsA/D7iqIsBNClqupL+STDAO5WFOVRAB8A8FHX8R8D+EI9y8c5QzyZxbbhEcSiYdx03UV4YOcBjMZTzo6gO3cfwr7Xj6E31oXN65fhmRfexM9/eQJ33nIFxlNZfPnbLzvpb1vXh6ee/xUGVl+InlkhiqdDEERVRCISjsbTGBoeKdBLC2JhJJPZZhePIIhphhwWcfxMpkQn+dk8tp313d2HcP3V78GDj5faWKcTWon9tXWwH7GIBMYFnEhoJfebF5ULYj7XCrddWFwWiq3ZmVTb5sXXL18yH+s+pOC+7fsLZNa0TNy//WcF95ibl2OvMmwa6MOOpw8inkg7aU8mNN80tw/2Q5IY7nrkpY6VXUnmvvrAj8nq9s5brkBiPIuvfsfbn/PTNaQriEplIMi4WU7Gi+Uw1CVh9HSpj9A7O4xMamo+Qr3kOshzEfWFfEuiUTTrlew/ALgdQDz/+50AjrrOHwVwDoB5AM6qqqoXHa8rOuAo2LUrFzvODwCMxlMYGh7BqmULnd/3bd+Pj31wMUbjKYyeSjqTz/b5Bx8/gFXLFmJoeASZbH12KScIYvownjEdAwGY0EvjGXJuCIJoPJms5amT/Gwe285atWyhM/nsvm7VsoWe9te24RHoADQDnverl7/qtguLy0J0JtW2efH1q5YtdCaf7fyGhkdwdkwruYctx15leGDnAaxdubggbbk09w6P4NipZEfL7lT0wWR1O3oq6Uw+2+fd/pxf3qQriEplIMi4GUTG05q3j5DWpu4j1EuuGz2WE/6Qb0k0ioZ/Aa0oygYAb6uqukdRlMH8YQbArW0FAKbHceSPV8zcuTMCl3E0PmGoRSOS8/fE+RSiriUJo/EU7FUoYVn0TT8aT8EwLSzoiVZclp4AaacL06VOpiK79aST6r3dn+XoiTFPPRNUv9SLesiuljUCt5uWNSBLtVvK1+5yUys6uR5aTe8GoZntElQn2XbWZDaW1zkIAgzD8r6fVXq/WtSL2y4sLksr9Id6y20rPGOj8WtzwwLCYQmzukNgTHDOFddR8fV+sh6WxZJjthz7lcHdPwzTuy+403jdox1l1zQtnBnPIKubkETmtMHRE+O++gDwlt/J6nZSf85D15TL16++W6ENvKByTY6f7FYqA0HGzXIyXpq29j5CvcbAIM/VCFpJvuqFn9y2um/ZDKaDPJSjXs/fjBAc6wAsUBTl3wDMATADuUnmBa407wDwWwCjAGYpisJVVTXyaX4b5GYnT47BNAN+dcwZemNdGI2nkEhmnb9temNdSLiWIvTGumCvPklrum/63lgXOBNw/HiiomL09EQrTjtdaHadNFIRTUl260Sz672WdMKzcM499Uw5/dJqshu0PLLEcf3nnwh0zVNfWVOztu4EuakFja6HRht/raR3g9Bs+Qysk/J21mQ2ltc5WBY4E7zvJxTer2b14rILi8vSLjp3qjRbtpqGT5v/+rdn8egTrxYsP/eso6Lr/WQ9rRV+Q1ggxz5lcPcPv77gTuN1j3aT3XIhAMrpAwBldZBfvU3qzwnldVsluqJV+1Y7l6sVZFfwGQ8Fq1AWg4ybvmk95HAqPsKkTGEMrCjbAM9Vb5op960gt3WRmzamVfVgo6j0+aciuw0PwaGq6odUVX2vqqrvA/A/ATypqurNANKKoqzIJ7sRwA9UVc0CeB65SWsAuAnAD+pdRhHA1sF+9Ma6sGvvYWwa6MspWcCJh7Nn/1vO783rl+H7PzqM3lgXeudE8Jf/7bKC9Let68Oe/W9hy2A/QpLgd1uCIIiK6A6xXDy3Ir3UHaKNbgiCaDwhSfDUSX42j21n7dn/Fm5b521jedlftw/2QwQgSQI2r19WcG7z+mWQ6mRjue1C+35b82Uh2pdym8V5tflt6/qwa+/hipafF1+/Z/9bJTK7ZbAfM2fIJXJl78HlVYZNA7kyuNOWS3P7YD/mz4m0veyWCwEgc3jqH0kS4Ldh2mR12zsngs993N+f89snjXQFwThKZOdzH78MrEhmgoybfjLuJYdh2dtHCMtT9xHqJddBnouoL+RbEo1CsKzmfemTD8HxQVVVBxVF+X0AjwCYCeAVADerqprJb1C4HUAvgCMAblBVNe6Xp4tFAN6c6lchnDPoAEwzN5AcO5mCIOTeiL/n3FnIahYM0wTnDLLIkMro4JxB0w2cHctg1owwBAEQGcsFFLFyA02QDQin+5sXL5pdJz090Ua8QViEKmS3HjS73mtJJzwLkznCIsN4xszpIcbQHWJI6yZMn8BprSa7PT3RQF80P/WVNfQFdAvQhC+gG/XWdhFaTO8GoRXkUw6LyGQtRydNZvNwzmBAgCVYMM3c8nrDtDCjW8zbWBYkScDJ02lkdRNpTcf8ORF0SxwZ08I/PvsGPvbBxeAMMEzg+z86jD+59gKILru2lvVi24WGmfsCWwTKbr7Uajp3qrSCbNWDSjbVcrf5r48msGvvYahHJlyQh7esgmhZvnXkvl5kDKIkQMuaME1A5AIYA4BcSBnDtCByARJDweZbxXkwDmhZs0ACOc9xAAAgAElEQVQGJZkjlTURT2TyoacYZs8IQ+IMPB/JsN1lVxcE3Dq0p+S43QaSzKEZcPSPYZm44+sv+rYtUOjrQQBOnU3BNCd8PdMAdMOEZSEfbsUCLAEyR9kN0irVFa3at9q5XK0gu5bIEE9kkBjXEJZFpDUd0W4ZsWgIgl4ofxnLgq5bYAJgWoAoCggJgqe8FMt4OTkMdUlIaxM+QlhmU96A0F1eHQAEAbCsSfVIpQR5rnrS5C+gmy63U/EtO5lW1YONIsAX0IFlt6kvZFVVHQYwnP/73wH0e6R5C8AHG1kuIKdQBQCMM2z5uxdKliMMbVyRc3J0A1ndgAjAsoA7/+GnHmmvgmAYFFCfIIiaYBjAbX/7nLeuaWK5CIKYvmhpHQLyhmUFNo9hmLDK2FicAV/425/4nBPw81+ewLP73y44d8NqBTDqMxFr24W557NAJl174/dF7dDGFc44arc55wyPPvGq59LkcvJWKDMGdCO39JTlC2BP3ThpdKB4iqg4D9MolUHNsHDH37/o2VfsCaJ2l13fMBv5NshqhvOMFixn8hnwblugyNd7yFsPMXuCzTXPlq1At7V7fRNVYAn44o6fedrobnSgQE4n0q3wtOXdMg7DKCuHmVS2IG0mVb0U2nJtT0zVSq6DPBdRP8i3JBoFfVM/CX6bexgeb45M0/RM245fUxEE0boYPrrGSy8RBEG0KuVsrHLnaJk7US1B7PtWl7cgz9KuBGmDoPUxHeqPaByVzgeQ3BGtBPmWRKNoFdupZZnsjbsb5pOWMYBefxMEUSs4894MJKeXmlgwgiCIAJS1seC9CSFnueXJsYiU+8KzwrACBOEmiH3f6vIW5FnalSBtELQ+pkP9EY2j0vkAkjuilSDfkmgU9AX0JAR5497qX0gQBNEZ0KYdBEF0AuXspslsKsMwIRgmRMuCYJgtMxlItAdBbfZWlrfp4n9U2gZB62O61B/RGCqVJ5I7opUg35JoFKTjJiHIG/dW/0KCIIjOIKsZmBeVMbTxKhiWBS5MvikOQRBEqzGZ3UQ2FVEvOslm76RnqQXu+qhkwzSqP6KWVCp/JHdEK0G+JdEo6AvoCgjy1UMrfyFBEETnkNUMCIaBBXO7IRgGGQgEQbQl5ewmsqmIetJJ8tVJz1IL7ProjUUqqg+qP6KWVCp/JHdEK0G+JdEIaALaBecMFmfQBQEWZ+Dcu3okmcPiPJ+OQ6pibUKl9yQIgnAT6pJgcY6jJ8ZgcY5Ql9TsIhEEMY2Zqj1TfF3OxmqcXUR2GDEZk8loPTYb55yByRxm/j7gnGSzhlTT7yfzA0mnELYMjMaTTZGBWs5VtCPUB6cG+ZZEI6AQHHk4Z4gns9g2PILReMqJwxSLSAVvIyWZ40RCw5Ar3ZbBfsyLyoHfElV6T4IgCDehLgmjp9Mleqh3dhiZVLbZxSMIYpoxVXvG67otg/3YufsQ9r1+rO52EdlhxGQUy8jyJfMxsPrCgvH3jk8ux6ywWDOZ4ZxhPGsgfiqDB3YecO5z+2A/ZpNsVk01/X4yP5B0CtFsGajlXEU70uz6b1fItyQaBb0OyqMDjqICgNF4CtuGR6AXpdMMOB3TTjc0PIKp6PNK70kQBOEmrZmeeiitkWFFEETjmao943Xd0PAIVi1bGCifRpebmD4Uy8iqZQtLxt97vrGvpjKjAzh2KulMPtv3uZdksyZU0+8n8wNJpxDNloFazlW0I82u/3aFfEuiUdAX0HkM03I6nM1oPAXDtAoqyTDNgnTKeTGsXbkYECxYnMO0LEiiANPIpWWMgXPAMADTNMFcGwxUek+CIAg3xXoIsHWHSbqDIIi6wDmDjpy9JEsMpgHopgnOBJgWfHQS0N0lIa0Znpss+dlB0YhUlE997CKyw4jJKJaRaETylXUIAkTGwLmATNaAJDFYZk7ebfsfAAwI0G2fgAsQDKsw9rlpISyLiEXD2LBmKaIRCYlkFrv2HibZrAHV9Ht/+8sCFxlgCbjk/Hl4dv/bjo8YmxkCIMASGWCBNprrcILIl3tcnWwTwkrT1stHsO8/Gk8CnNWkrEHTVgKN61ODfEuiUVQlT4qirFFV9YmiYzeqqvqt6orVeDgT0Bvrcjqecl4MA6svyJ0UORgToGVzS6tuWbME77tgPjgXwATgBy++iUhYxIOPH0AsGsbN1y/BV7/zsrN8YfP6ZXj+wH/ionfNw6wZMmLRELpkDsuwCu4JAL2xLnAmAMbk8eSmorBrreQJgmg8ksjwsWvejWv7F4EzwDCBZ0d+DUlksLLT5BMHgiBKmOoYP9l17iWtsWgYN113UUFogHs+faWnPWOaFhIpHafOpLH96V8gnkhj62A/emNhpDImGCwsXzIfq5YtdCbZ9ux/C4lkbrnnJ/6Lgg9edi4sK/eSPyQJ0NK1+46p2Pazy12pHUZ0FpwzGBBgCRZgCTAtC1wQsHzJfOx7/RgAwLQs3LlhOcKy6EwKxxNp/OZ4Anc/ui8XKuPmfsyISMhkDBw9OY6du99APJHGXbdcgaxu4jv/cgirli10fIJwiAOCMOFrMIZot1TSz7as74cscdqsrEo4Ewr0jmlZECAAEGDy3EsBWWRIZUwYpgnOGWSRQdMNwMd3+83xBCSR46nnf4UbVl+Ij1zzHui6hfu2Tyxnv21dH14++Dv84ZXvgiAIiCfSOZmbYlsG1ffkAzaGSscVzhlOJ7O41xXywC/MDucMGcuCrltgAqAbFgxRQMhDfjjLzVX0L1kAy7IgCAJGXj8Kzljui7ji8lYgF3Ura4B8K0XkzNOuEDkD9Op8pE7uQ+RbEo1iShPQiqJcD0AC8CVFUdxR7SUAdwNouwloEcDWwX5f52rTQB92PH0QF78rhg9cei7ufvSnzrkv3LQM//isitF4Cn/xJ+9zJp+B3Juj+7bvx923Xok7H36xIBbR3Kjs3NN9XAQwWTefSnwjiolEEJ2BJLISPbRlsB+SyKCRkUAQ05JaxmEuvs69pHXDmqUloQEee/I1bF6/DPdt31/w8v2b//yaE8v5tnV9+NYzB7FteAT3fPpKPPbka4hGZKz7kFJw3ZbBfjz3ytv4xH9RcPnF78DtX3+h4FzPrFDNJqHdtl9QO4zoLOyJkO/sPoTrr34PHnz8QIHcAcDphAbOGL6+q9A/CIc4Hv7eawDyoTK+OYLPrL3EmZC2Zf/YqST+/5/+uiT/rYP9+PErb+Pyi9+BHU8fRDyRxj2fvrKknw1tz+Ubi4bJdq8CmQtOHG/b5/t/dr5S4NeFQxx3P/JSgT6zYOEff/gGblvXV9B+dvvGE2lsWLMU24ZHcNct78c9239a0H4PPn4Ad996Je74+xer9sOC6nvyARuHzAVsGewviaUrcwFuE92A4Ey8AhNhdu7beFVJnkziGDudKhlju2Z3lbRfV4jh4nfPKxk7u0IM6WThyFapXAQpqyAxjJ1Ol5Q1PDucm9V0ESTfSpFEVmJXbF6/DJLIkK1iArrT+xD5lkSjmGoM6PcB+AsAvfn/7X8bAHytNkVrLIZhIhaRMLRxBf77jZeVGH0P7DyAtSsX49r+RSXxce7fsd+JVzh3VuEbTzvN2fFMSSwizbCcez68ZRWGNq6oWIlNJb4RxUQiiM4g4xOnK0Nxughi2lLLOMzF17mXtHqFINj3+jEIADasWYqhjStw54b34/Efqs5Xo/bky9qVizEaTyGeyGDVsoXoX7LAcRLtdEPDI7i2fxFWXn6et57L1u7LZLftF9QOIzoLHcC9+fjj9uQiMCF3t370Evz3Gy8r+cjkgZ0HkEzpUI/EnbxG4ymEZdH525b9sCx65r8tL/O2r2H3ES9/IiyLZLtXiWZYjm5Zu3Jxic93/479GD2VLPmYaFZ3CPteP4ZvPXMQf/2nV2Jo4wpsWLMU33rmINQjcSd80Gg8BSZ4hyXy8gen0pZB9T35gI1DMyzs3H3IGQ83rFmKnbsPQStaVaP7hDzQzdIxTsuaJWPlfdv3Q8uWjlWpjLePkMqUpq1ULoKUNZu1PMua9Ri7g+RbKZms4Xn/jEddBaHT+xD5lkSjmNIX0Kqq/g2Av1EUZaOqqg/VuExNwzBMCAAMQfBUhtGIBM68DYpZM2QAAPNZdnNmTCu5xjAtCJYFAfmGMKyKv7iZSnwjiolEEJ1BuRiE1JcJYnoy1TG+kuvcS4oTyaynnXN6LINtwyMAgKGNK5zJZ3ee0Yjk2ER2nGe/SRpJDPuUq7bxCG3bL6gdRnQOnDNk8v3AP8az6fxdfE6WeMGx3liXE0bGThONSEhrOmbNkD3zsP0Lu1+cGdM8+1kimZ024329lrtP9kLN/QLBfcwwc+E31CNxHPldAo8+8apn+/TGumCYCOQPBm3LoPqefMDGYZgWTicK2/l0Qiupa785A8ZQsgQniN0fLG1lclGvsgbJt1JMn2cyTWvKX14Cnd+HyLckGkU1/RAAtiuKsl5RlM8qivI5+19NStZEbEfLjW1U2AZF8bmZ3SH0xrpw8kwKmwb6nDT28ow9+98quYYzoeZlLJfnVK4hCKL14Jx592VOfZkgpitTHeMruc4OVdEb68KuvYdL7JzP3nApot2ycyyt6Z55pjUdt63rc+I825M1xekSySwEwa9c1ZquBDGBvaz6N8fHCiYQ3dj9wa+vzMr7APbvTQN92LX3cEGatKYj2p2L+eyVh+1f2BPXe/a/5fQ5O81t63L5Tgfb3W6XLQ+9gFuH9mDLQy8gnsyC8+r7v7sd/do7reklx0zTwhduWuarB23dtmmgD9//0WHctq7w/BduWlYzfzCovicfsHHIEsNN112ER594FVseegGPPvEqbrruIshSoeyKXCiRoU0DfRA9bPkgdn+gtBXKRaCyMp/7e8hakHwrRfR5/mryBDq/D5FvSTQKwbKmvsRBUZT/DeBdAF4FYGdkqar6yRqUrVoWAXjz5MkxmAGXcXjF+CmOAe2O67R1sB+xmSHohgXAQiqj43g8hRkRGd1hCTMiIk6dyRTkV0mA/Z6eKI4fT1RcxukQA7pcnTTo/o3QwoswRdmtF82u91rSCc8SiUg4Gk+XxJdbEAsj6frqyk2ryW5PTxTXf/6JsmncPPWVNYHS29fUqq07QW5qQaProUFyC7Sg3g1CT08Up06N1y0GtJ3O/hpRlhgsK7cs+LcnxrBz9xuYHZXxqY+8FxZyjtpYUZ5b1vcjmcniyed+hYHVF2Ln7kM4ndBKN1ob7MfO3Ydw/jmzcfnF7yjRc0FiQDez37aazp0qna77LM6w5aEXEIuGceOHL8JTz/+qJEazbbMDKOkrWwb7MTsq4+jxJAQBsCxg5gwJf/3ovoL+lMnq+MaTv8AH+t6JJe+eV5CHvVm5Owa0I+tZC1ljop/ZG3nW03ZvBdm126X4y8ihjSsgBHhuL/mdbFNVvxjQz7zwJqIRCX945bsQT2SgGyYECJg7OwwBwHgqi1TGwKyojLsfeQmxaBgDqy/AO3tmwDIBwzSQcYVSaNUY0K3a5yspVyvILjjH5od+UiK79228qmATQM4ZxrMGjp1KIiyLSGs65s+JoFviJW0ih0UcP5OpaDwMRyQc8/AR5sfCSBf5CEHG/0rLKskcJxJayf3nRWVktdIY1JXmWymSzHEyoZU801yP+7uZTL7qOY/SCnI7Fd+yk2lVPdgoKn3+qchutRPQhwFcpKpqK4a/WYQqjHK3oyVyVrAzdXeIYdzeGZkxMA787kQSiWQWbx09gw8tXwjdMGGagMQFiCy3ikQ3LJgmIDIBHNakyqoSRRh0aVq7797abGXQCgNEM2h2vdeSTngWXRAws0ss0EPdIYazKR2ij05vNdmlCej2hCagWxO7XaY6xk/1ulCXhLSW10OcQRYZUhkdnAkISQyZrAXDzNlDiZQGXTcdW2nl5efBME3IEodl5ibZOBMgcwGaYcEwLUTCIjLahJ4LSUKgDQhpArp6Ol336YKAW4f2AACU82JYu3IxemaHMbM7BNOywIVCm73AP8j7AIAA07SgmxaYIECSBBh6ToaZIIDz3LyTaVpgDK6+ketvnAvO35puggsCZA5nsqTRtnsryK67Xdw8vGWVr53jhZ/8Fr9QM82cn8ZY7qvMnC7z1m1hmU/oPcbQFXKlzeupTNZ02sut02SJwTRysW/DIRFW1phyWwaVi0rTt2qfb5cJ6CCyG6QN5bDojKmTjYfhiFQgk7kNCL0nESstg53OftNWrqySzKEZcO7v1mdTvX8QgtzfphL5qpcubhW5DepbdjKtqgcbRT0noKsN6fJ2lde3LAUxAXUDpv23YSCZNCAgp4S2eLzhXHn5uWCGCQbA0gFb3bP8Pxi12V19KnELKdYhQbQ/nAm47WvPeX4ZBGP6GQkEQeSY6hg/1esyqWyBrZTVDScPLZ8n5wx3/H3pl4wrLz8XomXBzC91t6/LGnDy1Nz5GwYm8R8JIjDu+ObqkTi2DY844ynPx312i11hXzFgGhOOmuikyeed/23mZZrnM9MK8pg47/gaALKum05H293dLjbOcvca2DnuOjXzisXdHmnN8NVtmVShDKSTRomecreXW6e57xWLhqua4AgqF9NRjppBENkN0iZaWq94PEwnsyUy6kelZbDT2fquXFmzWmGfyJZJXA+5DHL/IHRyHyLfkmgU1QbSehXAvyqKcnsnxYCuFHdMRGAi3jMFaicIop6Q7iEIol0gfUW0MiSfrQm1C9GukOwS7QjJLdEoqpWpmQB+CeB817Fp84rEMEzEIhKGNq5o25AWBEG0H27dU8lSOIIgiGZBthLRypB8tibULkS7QjY60Y6Q3BKNoqoJaFVVbwYARVFmq6p6ujZFai86eSkGQRCtS5ClcARBEM2EbCWilSH5bE2oXYh2hWx0oh0huSUaQVUhOBRFuUBRlF8AeF1RlHcqinJQUZQLa1S2psI5g8UZdEGAxRk4n6gqSeawOM+dEznEsJRPxxGJSGCyCJMzGIzBEjkskcHiHJLMnXwNQQCTRVhiLh/kz7vv2Y4bIREE0RgiEQkW5zh6YszRPQRBELXGbQ9B5GCy6Ng8clh0zjGp8Fyr6qRy9h3Rmbhtb4vbdnn5tnfb+kwWAZHnr2ee9ro7PUQOLovQhZz9n7P3J+7plkEmc4BP+AoGyWVDKPDl8j6YH8U6I6f3OFhRHnKXVPDbPh8qOm77g/FEmvRQh2LL19ET42XlK8h4FERmbRl1j9V+VFoGO91oPDlpWetxf6DyOqBxfmqQb0k0gmpDcPy/ADYB+KKqqr9VFOVvATwM4ANVl6yJcM4QT2axbXgEo/GUEwMnFpHAuIATCQ1DrnOb1y/DMy+8iZ//8gTuvOUKJMaz+Op3XnbO37auD089/yvcdN3F0HUT935zBLFoGDdddxEe2HnASbdlsB87dx/CvtePoTfWhTs+uRyzwiItfSAIooBIRMLReLpAD20Z7MeCWBhJn12uCYIgguJlD20a6MOOpw8inkhjy2A/nnvlbXz/x//hea7VdFI5+45src7Eq81tu/yG1Rd6tr0kc8fWr8Rev/vW9yOjGb79ZNNAH0Iyx/969g18fPWFkCSGux55ycn7yed+heuvfg8efPwAyWUDcLevu03nRWVki3Z2K5af5UvmY2D1hXjulbdxdd85uG/7/oLjxf7h8wf+Ex+49NwCebnzlity/uBDL1B7dyCVyleQ8SiIzMphEcfPZErS9swKQUvrBWkrLUOQstbj/vWqV2IC8i2JRlHt66C5qqr+0P6hqupDyMWFbmt0wFFaADAaT2Hb8Ah0AJoBp2Pa5+7bvh8f++BijMZTGD2VdCaf7fMPPn4Aq5YtxOipJO79Zu7atSsXO8asnW5oeASrli10ft/zjX3QS0pHEMR0ZzxjluihoeERjGfIsCIIonZ42UMP7DyAtSsXO3rn2v5FvudaTSeVs++IzsSrzW273K/t3bZ+Jfb6706Ol+0nD+w8gMS4hlXLFuLe4REcO5UsyHvVsoXO5LN9Pcll/fDy5YaGR6B5rDcvlp9VyxY6es+efHYfL/YPr+1fVCIvbn/QPkbt3TlUKl9BxqMgMpvJWp5pM9nSldWVliFIWetx/yB1QOP81CDfkmgU1U5AW4qihJHfeFBRlHcA8F8P0uI4S/RMy+l8NqPxFAzTgmGanufslR1hWfQ8H41IBeeiEck3XfE9CYIg3PjpIcMkI4EgiOpwL3E1TJS1Vdz2j9e5Zukkv6XC5ew7ojPxa3PbDjdMlCzPdo+xldjr5Wx/+++wLDp5hWWxIG+/e5Bc1odKbCg/n9BuK87gebw4TztdJfJC7d0ZVGqjBxmPgtj9wdJWVob6lbX2+dI4PzXItyQaRbUhOB4C8C8AehVFGQJwA4D7qy5Vg+CcQUdOUYmMYTyr465HXsKGNUvRG+sq6IS9sS5wJgAQPM+JIsPWwX5YFjzPJ5JZzJoh4c4NyxGWRczsDnmmi80MY+tgP3btPYx4Ip27p0EKkyCICThn3jqKM0CnLSMIgiiP2/7hTHB2OpdkjrNpHaOnkgjLImbN8LZVEvnlmLbeGdq4AolkFnv2v1V4jjHAqK9OKn4WmQs4mdA8l99a8LbRyNbqXDjL2e2xaBhrVy5GNCIhremYO6sLy5fMx2+OJxCLhp0we5qR+5LGlpNEMuttr0fDUM6LQT0SR1rTJ+0naU3H7/XMwJ0bloPl57vtvP3u4ZZLvz5LBIczHxuKMeimmfufWThyNIEF82Zg+ZL5WLVsIaIRCTO7Q1i+ZD4MEyXHvfI0zEJZAOArL5wxSBwlIRWm/JwkM01B5KxANuyxUSyy0W3dVMl4JPrY/cV55vL1l+/i8ZgzwbOsxWUIUtag9691vkHyJCYg35JoFIJlVdcRFUW5BsCHkbPX/sUdkqPJLALw5smTY56b+ZWLawgAN374Is9YbILEcNIjrtHTP/kP/PyXJzD05ytg6BbiiQzOjGnYs/8trPuQgoNvnsSSd8/Dtnw8uU9+5GKEJNE3XtznPn4Zeud0wdBNMDIaHOxdWZt4f6EBt1mEMrLbDJpd77WkE54lEpHwu3i6ZILlHWXidLWa7Pb0RHH955+oOOOnvrImUHr7mlq1dSfITS1odD00SG6BFtS7QQjSLuXiI1qc4WxSw8nTua80GQNkSSywedy2ytbBfnzXFdt0y2A/fvaL3+GHI0caEjvQ71nsMtn0xrowtHEFRKBhsSFbTedOlXbQfbmvVQXohgUz/1EJhwXDMME5w3jWQDyRKYzjvL4fkS6O5175Dc4/dzbO6Y3Csiw89uRrOP+c2bjm0nMQT2SgGyYkkeHL3365oA+EQxwMAnb+UMWffOgCcIGVjQE9c4YMWeQYS2YRjUjY/4vf4bkDvy0bA3puVIaWfyYLwGNPvub0tXrGNO0U2QUm5Nc9GRuSOOKJDL67+xBWLVuIWTNkxKIh/ODFN5149nfecgVSaR1dYRHZrFkQA/pTH3kvTAtIZ3Qnj7mzwohGpII2smNAX3PpuY5OWr5kPv70jy/B2bHCl2R2XPKB1Rdi7qww0hkdnDHIU5yQriYObqv2+UrK1QqyK4UlHD+dckK02LLQM7sL2fTEeBikjbgs4ujJZMEeU5/7+GVYMDcCQysMLhHqkjB6ujSWb+/sMDKpwvFYkrnnC9u5HnGVx7MGjuVfTqc1HfPnRNAt8ZKyhiMSRj18lN5YGOlk6f3dL73Tmo7eORHMDIuesa1PnMmU5DuvKLZ0pc9UTDPlvhXkdiq+ZSfTqnqwUVT6/FOR3aonoAFAUZSI+7eqqslJ0v81gP8LudAdj6mq+lVFUa4F8FUAXQAeV1X1jnza9wF4FLnY0s8B+LSqqpWE8VmEMp3M4gxb8ps/2PTGurBhzVJsGx6Bcl4Ma1cuxqIF0YK3xmJYxFgyW6CAe+dE8P1//SXePjaGP/3jpQUDzhduWoaf/Nt/4o+ueg+2PPQTxKJhZ3I7Fg1jYPUFeMfcbnAm4B+ffQPP7n/bKctn1l6Cux/dR8HzXTRbGbTCANEMml3vtaQTnoXLIk6cTeHsmObooZkzZMyb2VViiNq0muzSBHR7QhPQrUmQdvGzf4Y2roDIGf7z+FjBRN0X1l+O2IwwdMOEIAg4dTYF04Sjdx7+3mtQj8Rd+VwFADj065O4cNFcCHX8AnoyW87Nw1tWQbSshn0V2Go6d6q0uu7zm2C+fbAfs227mXNsfugnJXJyz6evxFgqW2C3/9VNlyObNfG1777iHLvrlitwOqFBEHJfLdurFD+z9hIsmDsDX/vuK5gdlXHTh5fk+kW3DCYAWtaEwASMpzVoWmGeWwf7IYkCTo9pME0L7+zpBiwBpmnlXvx4fMl/27o+fOuZg1CPxJ0+K5DslqWnJ4pTp8ZLJvru+cyVSKZ1bPtmaf0CcDae3LBmKR594lWMxlNQzos5PtyGNUuxZ/9bni8OZkQkWBagZXWMxlNYuGAmslkTnAsYy5cjFg1j00AfEkkNZ8Y07Np72GnXu255PzZ+cW/ZjeYmo5yen0xmWrXPt8sEtMk5tnrom6GNV5WMh5WOR5bIEU+kkRifsPuj3TJi0TCEoq9TmcyRzhrgAoNlWRAEAYZlIixxmMVyJHJs/rvSst73Z1cVfq3NGU4ns7jX1YcKdGzB/UXEExmcTqSdss6OhhGLhmAW+ShM5vjdyVTJxPo75naVlLVS34fJHCfPpkvSzZ0ZLn1+F9N9AnoqvmUn06p6sFHUcwK6qhAciqJ8DsA9AEL5QwJyk8q+caDzX0yvBHAJAAnALxRF2QPgGwCuAfA2gKcVRflDVVV/AODbADaoqvqSoiiPAbgFwNerKTdQPiYcAKhH4nj0iVedgdpWV9mshbseealEUd+54f34zfGxgg0pRuMp3L9jPzasWYqsbmA0nsKGNUsdQ2U0nnImmD+z9hL0L1ngTEC7Y8SNxnPB84c2rkCjPHGCIFqXrGHh/u0/K9FD2yHMDKIAACAASURBVDZeVXVgf4IgOpty8REtWCUbrt2//WfYtvEqcMawxcOpdk/2jsZTyOoGPn3/XgD5Sd8mPMusGXLBMffyW8MwISBvABsWaGFpe6MDOHYqia/v+nmB3N7rspt1v9iWhlVityfGtZK87PB8xS81wrII07KcFzAfveZ8JJJZ3L9jPz57w6XY8tALAICtg/3OJKad57bhEdy54f24/esvuiYGjZwDZQAaWMlGWvbEp33cMK269q9OwWtTst+MjpW0s12/ABw96I7tvHblYseHi0Yk380ji2Xl/j+/CmfGNABw5GA0nsKZsYwjIzaj8RSYMPH30PBIbuIy4DOX0/MkM/XFDFD3FY9HFvDFHaV2v/3C141uWPiff/9TDx9hRYmPoBveZdWNwrLqgDP5bKe512duQjcKN7Ob7P725LOd71e/87Jn2qxhVuT76L4+UmmexATkWxKNolp5+iyAK5D7OnkmgGj+f19UVf0xgD/If8Xci5zOnQ3gsKqqb+aPfxvAf1UUZSGALlVVX8pfPgzgv1ZZZgAT8YHc2DHa7L+3DvaXDhRlNiH024Bi1gzZiQHml8benMRdFne8MHvgIgiCMH30UDt+uUkQRGPxs384E8rqFj/7p9h2sT+GcmIz1hG/Z4lFQ85xP3uO6AwM05p0Uzc/ORGYUHKdX15eLzXSmg6Wny207XbbzrfjOgPlN6jz9zfKfyjjvFQhJsWrLsttHOluL792zO3tI1ekE8+MaSX5FuftTu/+oDQnx8G/ci+n54n6wrlP3fOp171peesD02Mlu98EuJcYsbwOKi5r8dAdZGM/02fzYq/7Bymrf1qrwnSleRITkG9JNIpq7fHDqqr+POhFqqpmFUW5G8BfAvhfAN4J4KgryVEA55Q5XjFz587wPG6aFu745HLc8419zpKPOz65HHNmhvDYHR+CJDLM6g45hqXNaDzpu8mE36YSM7tD2LX3Dfzlf7sM4ynvjUbSrqUNxfGo7WPhkIhYNBzk8TuSnp5os4vQEPxkt1l0Ur23+7Mc99FDIhfQE2v+s7WS7NayrdtdbmpFJ9dDK8luUCptFz/7Z97sCE6eSflsQiMAlvfGPu4X95vXL8P3f3TYWTo+q1tGKFS/qV+/Z+mZHcGXN30AWd30tec6iXrLbSv3+Xgi7Wt/23azaVq4/eZ+3FsUbuHMWLrkOr+87Jca9vV2HGi7z9jhG9auXIzeWBd27T2M29b14cHHD/huMihJHF/e9AFP+YwnSstmT3K7+2y7y3UjdG44JFbczolkFpI4oev82nHX3sP4v2+4dFKdaMd2XrVsoXPMTr9r72FsGugrCB1j61B3fiJn6IkVRLuclHJ6vhKZadU+30rl8pPd02O5vZyKw0rIEsPsgO1oc+qstz6QJY45M7srShuSPdKeSZXI4KaBPkicYc6cibR++shrbiLQ/QOkPXHa2/eRRAHzZkenlGcxrSRf9cJPblvdt2wG00EeylGv568qBrSiKH8MYB2A3QCcz3VVVd1R4fURAE8hF9v5fFVVb8wf/xCAzwP4GwD3qap6df74YgBPqap6YQXZL8IkscWmEgdQDos47rEJoSwKGEtlSzYW3Lx+GZ554U38/JcncNctVyAcFnG2KKbbpoE+xKIhRMIitKwJkTGks7oT6sP+OoJiQDc/Hk8rxGhqBs2u91rSCc9CmxBWfg3FgK4tFAO6NQnaLn72T6hLwvHTpbpl9kwZr//yJN7ZGy2wf7YO9iMWDSGTNcA5gyQyZ/OsrhAr2XCoHpSz5aZ7TMda0Oq6r6IY0MhtTJXKmgWbhK+/7mJks2ZBXNO7brkCmm4WxAa2N7pKaSZ0PRfX+cxYGtGIjLAswjAtPPrEq84mcwOrL8RQPs7vwOoLcN47okiMF8Ygniy2r98GZdFuCbBQ183JO0V2Af8Y0HfdckVJ29v1yxnDmbGM88LC3niQiwwJ1+aBH7vm3bjm0nML8r395lwM6BOn046cDay+EDt3H8LphIabr19SMDH5hfWXY2YklItfyQQYpok7vv5ixXJSjqnGu2/VPt8uMaCDbNhXKZwznE5lC16i3X5zP2Z3eWxYGCBec6WbAAbaMLGOZa10w8RK83Qz3e0F2oSwkFbVg42iZTchVBTlBwDOBfCm67ClqupHylxzIYCwqqr/lv/9Z8htSGioqnpt/tiNAP4AwN0A9qiqen7++NUA7lZVdWUFxVuEOhk2clhEJptbjlrscIUkAZmsmYvzxBgYz21CwpkAmQvQDAsQAAG53boZA0QuQMjHJbThnEGQeD7f+m2S0240Wxm0wgDRDJpd77WkU54lEpEwnjEdx6I7xMoaCK0muzQB3Z7QBHRrUst2CXVJSGtmzsZhDGGZQdcM6ABCIY6MS+9EQgypFnZMprtDWQvaQfdxzmDxnF1tmoDIBHBYnpMdxRNyACo6Zhim74Te3LkzcOJ00jlu2/vudIwL0Aw4/UrmmHRSsVEbZhbTKbILTMhvpW1v169f3Rcf92rr4nzdaWSJwTRzcWo5F8AZCnxASeaB5aTWtGqfb5cJaGBCfiAIgGXVpO8G0QdB0lYqc0GeqVXKGkR3kr0Q3LfsZFpVDzaKlt2EEMA5qqq+N+A17wZwt6IoVyG3YeEaAP8A4EuKopyP3GT2xwF8Q1XVtxRFSSuKskJV1RcA3AjgB1WWuWq0tD6xYYBuQNeN/OYBBjQDrs0EDJjGxMYC2fw5m9xxwPSwKwzDRM+cbhxPJ2iTHIIgSkgmsxAALMgPEMkkaQmCIKonk8oW2DGZVE63CAC0pHvDJCBFeodoAQzDBIzcxjYMAAx42s1+G35VeszvesYECIZZYu+70xlF/kG2gq5DG2bWjiBtXy598XGvti7O153GzE+WiQB6YqUOflYzAssJ0XrYcmJP4tSiGYPogyBpK5W5IM/UKmUl3RkM8i2JRlDt7jBvKYryziAXqKr6DICnARwA8DKAF1VV3QlgEMAuAL8AcAjA/85f8gkAX1MU5RCAGQAerLLMBEEQBEEQBEEQBEEQBEEQRAOo9gtoE8BriqLsB5CxD5YLwZE/fxeAu4qO7QHw+x5p/x1Af5XlJAiCIAiCIAiCIAiCIAiCIBpMtRPQu/L/CIIgCIIgCIIgCIIgCIIgCKKAqiagVVXdrijKOQAuAfAvAH5PVdUjNSkZQRAEQRAEQRAEQRAEQRAE0dZUFQNaUZQPA3gRwN8B6AXwC0VR1tSiYNON3A7eDLogwOIMnFcbnpsgiE7G1hmj8STpDIIgqoJsEGI6Q/JPNBKy3wiivpBOnxqkm4hGUG0IjjsBLAfwjKqqRxVFuQrAdgBPVF2yaQTnDPFkFtuGRzAaT6E31oWtg/2YPdtqdtEIgmhB/HRGLCLBMMxmF48giDaC9AkxnSH5JxoJyRtB1BfqY1OD6o1oFNW+1uCqqh61f6iq+m8AaNY0IDrgdHYAGI2nsG14BCfOpKbV2yd6W0nUk056q+unM/TmFmvaEp3ZhZ6eaKB/0ZldzS420cLUYjysNA/SJ0S7MJlMT6XfkPwT1RJE7qqVN/KViGbRLn5Uq+j0duurOoDv7j6EDWuWYmjjCmxYsxTf3X2IxkKi5lT7BXRSUZTzkJ90VhTlagDpqkvVxnDOoAMwTAsiY2Ac0LImOBMgcwFZA9BNE7LEYJqAbpiACUdJ2ozGU8hmDRw9OY75cyLolnjZt0/u+3ImQATa6m0VvXUj6gnnDKdTWdz7zQn5uv3mfszuak/5Mk0LsWgYG9YsRTQiIZHMYtfewzBNgDe7cNOQcEjE9Z8PtvDnqa+sQaJO5SHaG7/xMNotARYgcwGaYRWM9155ZGBBz1pgAqAbFgxJQIgzGIZZYDPAtDxtEMO0qjYSCaIW5Bx5AfGE5msnSjJHKmsinsjgzJiGPfvfwg2rL8TcqAzNsMC5AMMADNMEZwwyB7KaAcNH/k0LYJxVbFe3ux1OTA3OGU4ns7jXJZe3D/Zjto//4idvhmmBc1ZWbsr5SkBuAsk0LTDGAMECLBTkJ8kcmkcfqCXUD4LRCvVVSRmCynmQ5wqSthIZNnx8pEbaNO04ryEIwEevOR9f++4rTpk/e8OlEASh2UUjOoxq++FmALsBLFAU5acAFgNYW3Wp2hQvZbNpoA87nj6I2VEZN6y+ENuGRxCLhnHTdRfhgZ0HMBpP4c4Ny9Eb6yowSHpjXThybAyPPvEqNg30IdLTDfjYCO2o5Irxe1s5tHEFSO0R1WIywZl8BnLyde83R3D/n1/l269aGVniBTrE1jWyxGBo7dHnCYLwxm883LBmKR594lVsGezHzt2HsO/1Y74hu5jEMXY6hfu273d0xOb1y9A1O/flvdtm8LNBOBMAgxa1Ec3FtnHjiTS+vuvnnnaiyBlOFk1O37auDz9+5W1cc+m5+PErb+PqvnMK+sOWwX7Mi8qwDMtT/i3Lwpavv1iRXW2aVtvb4cTUMCA4k3JA3r4cHsF9G6/yTC8y5ilvx04l8bf/+G9l5cZvbLhv41U4myyV/6ee/xVuWH0hYhEJjAs4kdAw5Epj94FaTUJ3gj/aSFqhviotQxA5D/JcQdJKMq9IhmWJ+fpIZo1fuPjRnvMagjP5DOTK/LXvvoL7/sxblxHEVKlqLYCqqi8CuALAAIB7ASiqqv64FgVrR7yUzQM7D2DtysVYtWyhc27tysWOUgSAnbvfwKaBPvTGco6hbTjs2nvYyUMv4wS2ylKTaij3RQBBVEtW95avrN6e8mWYVoEOsfUE9ReCaH/8xsNoRMJoPIWh4RGsWrbQOb5teARnxjMF6bWs6Uy22enu274fWtYssRm8bJCtg/309TPREtjyGpZFXzvRyw5+8PEDuLZ/EbYNj+Da/kUl/WFoeASakfsSZ+tgf4H8f+GmZXjsydcqtqvPjGfa3g4npoZump5yqfvYY4yjRN9uGugDY8KkcuM3Nuim5Sn/tu+pA9AMOBN3dhq7D9SKTvBHG0kr1FelZQgi50GeK0jaSmXYNODpI5kN/OCoHec1dMNfvxBELanKv1AU5RIAQ6qqXqcoylIAexRFWaeqqlqb4rUXfsrmvPkzYFq5v5XzYjind0ZBOvVIHDuePohtG6+CaVr49dGz+NYzB6EeiTt5mGbubYHXMpVMByyf5UygL7CIusEYPOWLMbTlF9C64WMIGu3T5wliulK8hDQkCchkTde47j0eJpJZABOT0cp5MaxduRjRiAQta0CURYRFAeMZE4aPs5hzfiwfG2QFTFo2TbQYtm2dSGY9+4WYD5PhJe+cFf5ffN4wLRiwMDsawraNK5zJAc4EnE5oJelNE5A97PCs7t/faEzubJiP/+K2L906H1kLO54+WBAeYMfTBzH4RxcDsOUG4CIDZwyGHW6J52TN715e8hebGcKGNUsdve8XlgCCUJHenyz8QblJN+oHpRimhUvOn4ePfXAxOAMME/j+j2oTKqLScCuVtlklch40TzttpXXgb9eYBWn9J8vNxoXgaMN5DV9fmUJwEDWm2n74dQBfBgBVVV9VFOUuAP8A4INV5tuWSNx7WdWRY2OQRAHLl8zH9Ve/B8dOjZekiyfSePvYWfxezww8+sSrpQY2E8DhvUxlZkRuOyVXjP0FSvGziWjL+UGixZBEhk0DfSXLsSSRteUkC/dZwpnr800sGEEQZSleQrp8yXwMrL6wYEnpXbdcgdsH+wtiLd62rg/feuYggFxf1w0TN374Ijz4+IRO+6sbL0dXWMTdj7yEv/n0lT6TdQIsq9TJiCfSEACIlgUYFqkRomWwHfldew/jszdcWhCfctNAH3TThGl6O86SyNEb64Lhc143TGz84l5nKfdzr7yN7//4PwpC6Nkfg/TGugABJXFQtw72IzYz3PZ2ODE1RC542pciF2AapTr/zg3LEU+ksW14xMnD/YKxN9aFE6dTkGWGjGYU5HvHp5aX+EpbBvt9/c8ZXTK+9K2fYTSewsNbr/UMSwABuHXbnpJ40sVUEv6gHSfdmklXSMSHV7wLdz/6U6dON69fhq6QiGw6O+V8Kw1VAQCij+yInAH6RNogfpSfPEqcwdIL7x8Occ86CIc49HThd9D+vg8DDMOVrvly2I7zGiL3bmNblxFErah2O85uVVW/b/9QVfWfAMysMs+2RWCCbyiNnbvfwM3XvxcPPn4AO3e/gdvWlabbufsNPPbkayVLAW8f7AeH9xLDbcMjYLx0+WC7LZ81DBOxiIShjSvw8JZVGNq4gmKGETXDsnJGzmfWXoKhjSvwmbWXIBzisNrUHg5JArYU9fktg/0ISfSWmiBameIlpKuWLSxZUnrXIy9hVlTG0Mar8JVNH8Ddt16Jp57/FdQjcaevyxJzJp/t6774rZ9h9FQy/8VnqT2yaaDP+cqt3W0GYvpgy2s8kYZpWc44vmHNUux4+iB+MzqGbz71WoldvXWwHykti00DfXh25NfYvH5ZwfnN65fhe/96GMDEUu5r+xc5vx/YeQADqy9w0t+2rg+nzqZK4qBuGx4BZ9SnpiuCYSEWDRXYl7FoCEJ+oqtY53uFPNo0kPMV7b8tWEiMayVhBO55bB8yWR0b1ix1+sDO3YdgwTuMzPA/T4SRiZ9Ne4YliJ9NO7/LhX+oJPwBjS3B0HSfUFl6db5vkHArzMdWYKzQnwjiR3nNh2wa6IPASn2UbNbyrINstjRjmcPT95GLdl9vBTlsx3kNkQuYXaTLZkdDEDn5lkRtqbYvWoqiXKKq6s8BQFGUi9C6L3bqjpY1nGVV582fgSPHxgpCaSSSGkbjKYzGU/jWMwfx1396JeJn00gkswXpbvnoUmzbuCK366hlOUuiDEHwXFKiZSeUXDvvOmwYZu4LLIC+wCJqipY18PD3XsPalYsRlnMxoR/+3mv4/CcubUvDWEvr6JkVwtDGq5w+H5IEaGmKtEcQrUzxElI7rrMbe1wHgM8/8JwTauOj15yPRDKLWFSGlvWOhxuWcxpN103PZd6f/8SlEC2rI2wGYnrgduQN08KtQ3sKzodlEfteP4bTCa1A3mdEJByPp7Dj6YNYu3IxQhLDvZ/J5SFxhu/uPoRn97/t5GOH6nD/nj+nG0MbVzh2+uAfXezZ7zIdYocTwTEME90Sx3nzo55tX6zz3SGPjsdT0PPpPntDHyAI+Np3XsHgH13sG/PcNFHw9TQAfOojBmIRCds2rkA8kUEkJCEkM+x7/ZiTxvQJi2C64rtOhGnyeM4Kwh8U91XqB+UxfMLpGUZ1oSIqDVUBFM5dlNgKRekq9aMqzTNoWbOagXn5l/PlQou0ihy227xGMq3jxX//LT542bmwLAuCIOBHL7+Nay49py19ZaJ1qVae/geAHyuK8mr+94UAPlFlnm0LZ8xZVrV1sL8klMZYUnOWhKhH4jjyu4RnuA0tayIkcfTGIjh+POEorHJLStpNyRFEI3H3TRuvZVvthJbWIQBY0BPF8eOJmm4kQxBEfSheQuoX15YzAcjHglaPxB3d1RvrwtDGq3ztgbSWewllmCir88hmINoJW165x9LutKZ79pM7N7wfaU337Af3fmYFfv7LEwX3sEN1uH8fOzWOux/dV3Kv0lAfDHo6S31qmlJOn3qFDYgn0shoJrY89AKAnAxtWLPUOZdIZiGJ5XW8+1guVrQBgTMn5Ib9BehkY40d+mMiL++vHSsNf0BjS+VUWqf1zLdS/yiIHxU0bZA6yGqGS74MZH2qieQwOJwx/HDkCP6/f5nYyq031oWVl5/Xtr4y0ZpUFYJDVdV/BqAA+BqALwF4r6qqP6xFwdoR97L4XXsPlyw/6Z0TwV/deLlzbM/+t0qWkmxevwyiKJQsJwFaY0kJQbQj3SHmuWyrO1RtFCKCIIjKKV5C6mUH2ON6uVA7IoDbby4891c3Xo7eORH0xrrw/R8dLgk54LVUlSDaCS87uHdOBJ/7+GUlsv7syK8R7ZY9l4L/6OW3PfvHsyO/dn7fPtiP+fn+ZB+bPyeC2z3666zuUANrgWgn/MIGuGXNDsFhjwd79r/lKbtuHe/Oy9br7v5R7Id6jTX2vezf5XzKSsMfEJUTlr19k7BcnW8SpK0qTRukrEHCBJJctQ7kKxONQrCqDIKqKEofgBlA7uMEAOerqvpIDcpWLYsAvHny5FjB8qJ6I4dFZLIWDNOELHGYpgXdyC0T6QoxaLoFCxZMI7fzq8gFMCY4S0QMw4JhWQiJDKZlIatbYAyQJQ5dN3O1bAG6kTsucgYOy3Nn206kJ/+1ZxPv34hASIvQBNktR7PrvRZEIhLGM6azbKs7xJBMTn2Tj1ah0rZpNdnt6Yni+s8/UXHGT31lTaD09jW1ktty9Rz0WYDalq2RNFoXNEhugQbq3eLd6UOSgEzW9FwqKodFCExA2tZdnCEsMWRSWXDOYDEBmj5hD4RFwdFzssRhWRZMywIsAaZl5XYzFyzAQlOXRnPOoAN1Xx7bzLGr1XTuVKlHHfq1v308Z+vmZJYzAYwBWtYC5wJCEkNGM/NyPJGGiwJ03WVL5/tLSGLQ8/F4DdOCYVhgTIAk5jbk1LJGrh/KuXxNM9+f8mUtLifjQkH/lTkwe1akLfW5H50iu0D95VeWGEwD0E3TV5dJMocBAbqR2zBTEgUIggAtazh6OqvnVr8att+Yl92Mlh8beE5mDcOCaQGwANOyIDIBIgM0w8r5lYxBlARoWROWhQKdL3MBmmEV9C+vMcGvztw+rj12NTL0WyVt2WqyO1mZuyISki7fJBJiSNXANwl1SUhrE/mG5Zzd4EWl7RrEj6rH/YPkW2xneYXqKKYS+QpiuwRJ2ypy26m+8lTohLmPaqjn/EJVH88qivIIgDUAwgB+C+B8AD8B0AoT0E3BXhYvAjDzy6REABwWRuNpfHf3IVx/9XsKdq7fOtgPLgo4O5bbcCIWDePm65fgq995uWBH2OcP/Cf6lywo2QF8djSEmWFx2kxCE0RQIhEJR+Ppkh2hF8TC03ZgJQiiORQvIdUM+C4VFQQBox66q3d2GEwAfuc6d8uaJbj43fMwNDyCWDSMm667CE8+96sSm+O2dX146vlf4YbVFzZlUxzOGeLJbMnu8K2+QQ9RG/zaf25UxsmE5mknbxrow46nDyKeSGPz+n48f+BtXHbRO5w0y5fMx8DqC/HcK2/j6r5znE2temNd2LK+HzOjEo6fShfY1ZsG+hAOcTz8vdcQT6QnZNAyAWNiQxt33wRnOJnQSso+M9rVvAolGopbfm09a2/u56fLTMPC6WSh3Gwa6MO//uxt/MHl5zq+X3Fem9cvw4yIhEiIw8iaiJ/R8B2P/rFlsB87dx/CvtePOX1hyEO/ZjUDImeIjwfXv5wznDiTIb1dQySZF4zhdlvOi8pV+fSSzDF6urJ8K23XUJe3H9U7O1wyAcw5w/HT6YpkJYhchbokz+cqLoMkc5xIaDWv1yC2SzvaOeQrE42i2m/qPwTgXQC+D+A6ANcCSFZbqE5ER27TiFXLFpbsXL9teARcYI7RsXblYsdIttPct30/ru1f5Ew+28cf2HkAo6eSFP+VIMownjE9d4Qez7SmEUAQBAEAac1bd6U1E8kivda/ZIHze+3KxXhg5wFPm+PBx3PHtw2PoBnbltr2ULEdRFuoTg/82l8z/O3kB3YewNqVi/P28Aiu7V9UkGbVsoUYGs4dtyef7WuHto/ANFBiVz+w8wDOjmlOvpXIoF/Zz4xnal5PRGvilgFbz06my7zk5oGdB/CxDy4u8P2K87pv+37ouoVM1oIO4F6f/jGUPw5M9AW/Mk1V/5Lerj2aAc/xvVqfPki+lbZrOVtkqnkGTVtpGepVr/V6rlaBfGWiUVQbPvioqqrjiqIcArBUVdV/UhTlwVoUrBNwL71AfvfhaERCLBou2Bl2197DCMkMd254P5KZLKIRGbFoGLFoGGtXLnbScS547hQblkXP3WIJgsgRZJdlgiCIZjNhP5j4iz95HxgTwATBsRmM/BrsddcuxvsumA/DNGFZlqPnohHJsTm8dJ993DCthutAw7R89HHjy0I0Hv/2NyeVWftvzlCQxr6m+DgAxKJhWJg4rpwXc2zr2MwwUumsc8wwLTDOIebjjwr50B22Lc/y+bnvMRpPIavnNrwiOh+3/PrJarEu85N5t7z65cUEgHMBhgH89a3vhygyXHL+PDy7/+2CdAsXRPH3X1gJgZX6irk+IMAQBHAIntdPpn9Jb9eeevkmQfI1TMtzXqJUhmufZ/C0lZXBME2fPL3r1ba3RuNJgDPfUBlBn6vd+gv5ykSjqFaeNEVRPgDgFwD+UFGUf0UuHvS0p3jpxZ0blqM31gXTskqWWH3u45chmdbxN4/tKzgmigK+uONnzrF7Pn2l747I1e6YSxCdjMi9d1kWOQN06jcEQbQOXks3Nw30Yfiff4F4Io1NA32QJYZ0Vse7z4lhy0M/KbAzRuMpMCagN9aFRDLrqfvs45wJudACjXy+fNmKy9SMshCNx7/92aQya/9tmChIY19TfFw5L4Zb//i9MA0LvbEuxKJh3PjhiwrDF6zvxy0fe2+BvW2H5+iSc27SXY+8VHBux9MHoR6JO+WRRAadbIlpgVt+/WS1WJf5ybwFTJqXJDLEE5mCZfGb1y8DAGcSuTfWhZOn09j69RcKxgEg1wduuu4ibM2PE37XT6Z/SW/Xnnr5JrYu9dKxxXMFssRK5iVsG8N0fTLMfcrKPcoakrhnniGJw9AKvwGu9P5B6kv2ub8scSc8qvu5Kg2VEaSs7dhfyFcmGkW1ITi+AOBPATwD4H0ATgD4drWF6gSKl17s3P0GNg30QYBQssTqq995GSdOp0qOJca1gmOPPflaya7dmwb60DsnQrvFEkQZOBNKdhPfNNCXMwQIgiBaCL/l2naogAd2HsDZcQ0S4wXLJW07w55o2zTQhz3738Jt6wp1323rcse3DvY35asWEcDWDM2oAwAAIABJREFUop3Wm1UWovH4tb/Mc8e9ZHbTQB927T2cnzzrx7Mjvy5Is2f/W9gymDvutpMHVl+As2MavvnPr+G2dX0YWH1BafiC7SMl9rYdnuPYqSSOnUqWnBtYfUFB2Wd1hxpWf0Rzccvvrr2HS2xLL13mJfObBvqQzGSd673y2rx+Gf4Pe+8eJ0V5Jfx/69LVPZdmaGFATQQjkfYGG0AGhTEmguybeEHDLwomwrgCCiYSY1YZcIMmgUFz27gJJkoSMBdg93VdNZh9UVhvGBmiZjUIrfECmBiuzdBz6VtV/f7orpqurqqhe649M/X9fPzIVD91P/U855znPOcgCLZl8Ws27OILnz3bciydjFMrdxyAzDfglNojd/9C+l+v3+5+eso28fkEm79g2fzJ+Hz242oqNvn40abX0fL8jX6fmMm3nHPM+roa/D67K0nVdMdjqg6F7wo9PxT+vDSX8zsV3ismVUYx19ofvxfPVvboLbr0HUQikVeAV7J/XhQOh6sikUhT1y+r/5O/9CKyP8qjW/Zw55cmuqbRONm2nbsPsujacXz71qkIgCgKaBooPsErQOjh0QGJlMqjW/ZYlk0Z32MpKwMeHh6DD7elm7kpCFraUgQUyVHPWL2kFk3TeHTLHmZfdjYVAdlM8RUKBhAEnUXXjOuwIntPoqoaoXIfDUumFVQd3mNg4fb+U0mVULmPRdeMAwEaltSi6ZnfRVHgzi9NQpIEfLLAVbVj0AXdlOvoiQSKLHDuJ4bj94msWpw5tiDAsaY4O3cf5HgsydI5Ezqtg+f+9rHqIA/XTzevXfQM9EFDvvwqPpE1S2pJa5prX5a/jyyJiKJAMqXy8H++ZuqmsizyrUVTEcWMTDW1JEilnZfFS5JAw5Jppj5bd+V5QO44MM10uHW0f3WoDJ/DNZ/svr1+u+v0lG0ST6g8veN9Vi64GEkEVYPHn3uH62aMtR037ZJ2IZ2XdqE1nuaPb/2dVYunoes6giDw3KsHuHTix+3HVF2OqdpTORR6fij8eRVz/mJSZRRzrf3xe/FsZY/eolPyFA6HnwIc1w+Ew2EikcjVXbqqAYDT0otoLO66LCaetyTEbVsiqbHkge2WbQ1Lpnm55zw8OkASRaKxTEVmA7flaB4eHh59idvSzdwUBJnl2uWOeoYApNK6Y5/XsKQWIZ0xgPqy51PVTM5cGUDV+/RaPHoft/dvbDeQADTQjLZpSBuqsSRy37o/mPK/vK6GdU+8afkeVi6YAmRkP7I/yoeHmjutg+f+JonZ/NCe7A5KcuXXWHp/sr7MIvNpNSPTsuTYTy+YNc7ctvauyxxlNp3WqV+7w/zbGB/AGAcEZF1DlyTX/X+48bWMg6xAp5jXb3cvkii42CZdS9MgiQJv/OWIJc/3iFAZc2eGbcctNFWEJIo807if3/y/iKXdZReOstlRxaQAKSZVRaG2XHHH7Jm20P++F89W9ugtOpuC4/8Cj3Xw36CnoyWGK26ybv/6DZMIViiWbXfdeCHVoTLbUhdZFvrVcg4Pj1Kgwu+8dKzC39UsRB4eHh7di9tybSMFQX02TcGf3j7o2K/t/eAojz/3jm0Jbn1WB/HwGAjkfydGWplcmR9SqTDylPIO0xysuKmGkaeU2743Y9/83zy926O7CAUDjn34tl37zL9lWWDZfHubZxs/cN0nt69XJGznWDZ/Ms82fuDJch/TU2kaijluoW2d5MhNpyimbTHXWuhxe+L+i23bH/FsZY/eQtD1riVCD4fDw4BPkwmmeb6EUnCcCbx/9GizY86f3qC9ir116YUkiWiCQErVkSQBWRRI63omykPXkUUBQQRRFEmlNHN/RQJN1R2POViorg5y+HCsL8/fG8HmZ9LHsptPXz/37qC83EdLov17qvCLtOZEjPRXCn03pSa71dVBrrrziYIP/NT3ZxXV3tinu+S2o+dc7L1A915bb9LbfUEvyS2UWL+bqz/IoogoQTKVWeLt94kkUjqqplFRJhNPaqhqe7+W6ec0An6ZVFrL6B2iiCIx6NJ19eXYVWp9bmcp5fE/X89WJIGUCmlNRxRBlgQkMgZKWtXRdPDLImlNR1X1zO+6jigJpLTsfoKALAmAnolyhpPq3aX8jDrDQJFdKO13U10d5HhTK0kVVC3TT/t9Aokc28/vE9EQ2u1BScAv29sYY4JTX+9TpOw5Mu0liYz842xDluozK+S6Sk12T3bNbr6CrlLMcQtta8qRriMJQoc6RbvMnVz/KOZaCz1uZ+4fQQDd/bso9rjFUCpyO1Bt5c5Qqv1gb9GT/oUuTdqEw+FrgV8Ab5BZKffzcDh8XSQS+Z+uHHeg0NESQ2hfTqiCueRQAnQtk99kWHU5hw/HsvtDKnuA/rScw8OjVGhtTSEAp2U71NZW7+vx8PAoTaz6g4qmto/7yZzfEg79mvFbOp6yHCPldXkeA4x8PduQcUNv1tRM+g7ILPkUATWZmwoho4OrufuBraiUp3d79BSppGrpp5OqVd6SWZsxV2aTaXubjvp66zky8i3Qt2mYPDL0VJqGYo5baFtDjgx9oyOdIl+uO2pbzLUWetzO3L/hcOuOZ9Vf8Wxlj96gq6sGVgGfjkQibwKEw+GJwDpgYkc7hcPhlcB12T+3RCKRu8Lh8AzgB0AZsDkSidyTbfup7DGHAC8At0YiEefEbP2IjqKj02Qm4Q5FW9FEEVEQQARBB0UWSMadb7+nZuU8PAYCxqzuR0eakSRpUM/qevRPgkPKCPiLG7bjiTSxE20nb+jR5+SO4eUBmURKIzMdLaBlC6oZEZ2ZKE8NUcxEa5ZC1LaHRyG0R5sBuoBmyHH2dzc9NjfyTfFJaJpOOhvd7/MJxBOqp/t69AuMb+BQtBVBkswVLopPRNMyRdSMvl1QdYt9aBQ+1LJjgBH5n1T1giOiPXoeJSCTSOmmzeH3udvvA5Ge8kkYzzV3xYDTcy2FyPL+iGcre/QGXXVAtxrOZ4BIJPJaOBzu0ArKOppnAhPIWFb/HQ6H5wL3A5cCB4At4XD4c5FI5PfAr4EFkUjklXA4/HNgIfBQF6+7T5EkkWhritXrGzkUbTNzCA0LKhyNJdm4dS9XXTKGBze/bv5++/UTeOrFd5k78xyGDw3QGk9bFBDFJ3I8lmRV3jFD5b4B1TF6eHSG8nIfH0XjNOR8H/V1NZwWCngDq0e/IeCXO5XqY/AuIOs/5OoFl9eM4sLzTmWTgy7w9RsmMaTCx72PvGJu+8aXJ9GWUCmXRW+89yhpDDl30nNX1NXg84kW2b534UUEFBlJhCOxJA3rGwkFA8y74lx+tKl932XzJ/P0jvd54y9HPN3Xo6RxsgGXzpnA//zxADNqRvOD375qkeuqoIIsyURjCVa7yH99XQ2btu5l5+6DXHvpWXx64hk2fXd4UPGc0L2EEpA53JSwvYPqKn+vOaF7IgVHMed28nN0tV8u9Ln21Pl76rilgmcre/QWXc0q/vtwOHx3OByuCIfDgXA4vAj4czgcDoXD4VNc9vkIuDMSiSQjkUgK2AOMBd6JRCLvZ6Obfw18MRwOjwbKIpHIK9l91wNf7OI19zlpMDsvgEPRNlavbySpZrZPnzzaVMqN3x/c/DrTJ49m9fpG2hIq3//Na/z1cAvL1r7EooZtvP+3E6bzOfeYg2eu1cPDnZakZg6okPk+GtY30pLs/wqDh4dH/ydXL/jMpIzzwEkX+MFvX+XgsVbLtu/9+lVa2lLeeO9R8hhy7iTbq9Y3WmQ7FAwQjSVY9pOXaG5TzTF89mVnm843Y981G3Zx7WfO9nRfj5LHyQb80abXufYzZ5vOZ2P7mg27UNM67/2tydzHSf6N8QJgRs2Zjvqu53vuPRIp3fEdJFK9s1LJcJTWr93BooZt1K/dQbQ1hSTZ3T7FtC0UNz9HV/vlQp9rT52/p45bKni2skdv0dUI6GVk0hY35G2/kUx0s63eaSQS2W38OxwOn00mFce/kXFMG3wEfBw43WV7wQwbVllM817hULRdwW7f1oaq6RyKthEs9zn+bmzXdd2mgAQU2XEfBIHq6mDP3lAvM9Dux41Sk93+/Nw/OtLs/M2pOqf14/syKLV3U0qy253Pprufc2+9t/563X1BX8lurl6g6x3rAgFFtm0LVigDcrzvDgbDM+lpue2uZ2jIeSGynavnSiJme7d9DX9JX+m+g0HOeoLe6HNL6d242YC5Mp67PRpLWGy8jmxEwPU4ql6cvltKzyyXUrouN9l1tTm03rE5orE4q9fvsDlKv7f001SfUtHptrl09B7cZLyr/XKhz7Ur5++L++ptipbbAWIrd4b+9F57gp66/y45oCORiK+z+4bD4fOBLcA/k5lUGpvzs0CmbohIxpGdv71gSqWivQVJZESozPKRjwiVIYkCI0JlxFpTjr8b2wVBsCkgbvug6wOqgmdfVyTtzY6olGS3r597V5EkyfWb68/3BUVVqe2Fq8lQaGXw3qC73m9Hz7mz91LstfXWeU52Db35zfS28tdn/W6OXiAIHesC8aQ13sboywbaeN8d9OXYVWp9bmfp1meYlfNCZDtXz1U1zPZu+xoroPtC9+3vOlI+A0V2oQTfjYsNmCvjudubmpPmvzuS/1h2ibzbcSShcH235J5ZlkKuqxRkt69tjrQgODoS44k0h+OxTrc1OOl7cJHxrvbLBT/XTp6/r+7LOHdvUapyW2qUaj/YW/Skf6FLKTjC4fDIcDh8dfbfa8Lh8LZwODy+gP2mAduAZZFIZAPwIXBaTpNTgb91sL1fIwPL62oynRaYOYQUKbN926593H79BMvvt18/gW279rG8robnXj1gKiAGj21/h6VzJtiO2dUQdw+PgUC5X6Q+75urr6uh3N/VLEQeHh4eXSdXL3ju1QPUu+gCy+tqGFKpWLYtnTMBSRK88d6j5DHk3Em2V9TVMPKUcnNbPJk2//34c++wbH7m+3DSd5fNn8zjz73j6b4eJY+TDbh0zgSLjBvb7543mW279vHY9nfM78VJ/o3xAuDZxg8c9V3FtibZo6fwK842h1/pHZvDCGjLxZyo7kLbQnHzc3S1X/b7BOfn6rNea0+dv6eOWyp4trJHbyHoeudnncPh8O+BrcD/Aj/M/ndTJBK5tIN9zgBeA66PRCLbs9sCwDvAZ4H3gd8Bv4hEIv8RDof/DNwSiUR2hMPhh8nkiv5uAZd3JvB+KUWR5uKW8N+nSLSlNBIplTK/jKaDJAgI2VhwXdCp//EOxyIU9y68iIBPdq2KPBAqtfb1bFR1dbDzI3LhnEmJyW5PPvfeqCisBGQQBRIJrb1ysl8ETe/3VamLmKEsKdmtrg4WVVDvqe/P6lQBvt6KgO6Na+ut85zsGno5srA35Bb6qN/N7f9kUUSUIJnSKA/IJFIamUVgApqmIwgQUCSaWpIcOtZKQJEzTrpTyjk1VE5TU9tJzjb46OMI6JLqcztLdz9DQ+YRAD0j26LYviRTRSClahxvjqNp8MONr3Eo2sa1l57FlbVjUDU9U4Rb00mrGpoGLYkk6bROKOinzCcWVGytO3WPvtZNu5uBIrtQmu+m/RvIyP+xE21oGpQFJMr9PqKxBE3NSfa8f8QsKBgKBpgzcyynD68koEioqu5q7/l9IomUbuq7ikTBBQglSUTwScQT6ZKzHQuMgO5z2ZUkkbQImppJqSUIAqIEstY7z7KYYnmdKaxXyHvoKdtOCcgW2fb7BEc7zqdIJFWK+gb68r5KQW4Hsq1cDLn9M7peUn1gb9KT/oWuTtoMi0QiPwyHw98FfhuJRNaHw+HbTrLPN4AA8INwOGxs+ylQBzyW/e1p4P9mf/sS8Eg4HB5CxnH9YBevuSRQVQ2B7AtQdYwuMZVU8Usikl8GQUBCR0ZHTWUEX5ZEGpZMMxXwNUtqTQUk84GoyIBPEmlLaaYSs23XPubOPGfAVGr1GBj0VkXhZDyNv8yaMUjQITGIBlQPD4/SoqP+L9mWIlejM4LXkm0aVeU+Ar4hFgNMUezqXG9M7nl4FIuh/xpIACrkugYCksjwqjIQoGFJrWEHomX1XUHVQNXwZx09QVExZbxQ53Nv6B4eHk4Y30B1dZBjx1oYXlVm9tOKJCBVBQgF/YwdVYWmwXdunYooCsiSgKDqqNlUNYYNmVKx2JRJi42pkiqwAGH7d7HD+y66gKpqyIikBUAUM06sXnI+G+cPlftMf0FH438xbYu9Bic/R1dJxtMW2Xbq7iVJ5Ggs2SP9ezH31d90MM9W9nSD3qKrDmglHA77gM8B88PhcDnQYTWJSCSyFFjq8vM/OLT/X6Cmi9fZr8hVTA4fjlk6t9yOT8v2uvmdoFPHe/v1E9i4dS+LrhlHb4WTeXicDLeKwg1LpnWrnEqSyOHjcW9A8fDwKBk62/85GmB5Kdg8JdqjP5PrpJYkkWiLuyx3xsnRW7qHh8fJyJdhw5nsl0SiLk60nsL7LrqPjmz53jx/IX1jTzmL+4pSkOP+qIN5tnJpyM5goKtJXZ4ADgNHIpHIq0Aj8NsuX5VHUUiSiC6JpAUBXRJREWwfz4ObX2f65NEgYGkrSaLrcXJ/8/DoCVRNd61o3J24DSiDZ07Xw8Oj1OjJ/s+9zxOyY7yEz0sK6tEH5OuaPkVCVCS07DYkyaJ/9sT43Vu6h0fpU6jtY7RTs/2nLos9aiv1hd7qfRceAwFV0xn/yeH85J8v46d3X8ZP/vkyxn9yeLfIcaH9RX+0O/vjNXc3Xh/YO3QpAjoSiawMh8OPAH/NbrohEom80fXLGvh017IMpxm2b98y1fHjGVYVIOYSRQL0u5k6j/6PUfzCqeIuavd19pqO4zeh6e1L2z08PDx6k670f/k6RH4+P81FiT7enODOH71gFpcZHlQKzg3q4dFV8nXWKeePZN4V53E8lrDUNFlRV8PQrP7pZhBqWufH797SPTxKm0KjFJ3a3X79BJ568d2C0ht2xubryBHSU0XPvO/CoyMMOT4UbQVJLNmUEgG/xOenfYL71v3B/F6XzZ9MwC+R7kI6iWKimvvi++0qnq3s9YG9RaembcPh8Jez//86cB1wR/bfM7L/98jBKdoj2pqifu0OFjVso37tDqKtqU7NojvNVv3tSLNjRduhQb/rzFZnZ728qGmPrtBbFYVFwbnKsyh4C2o8PDz6hs72f4YRlKtD7Pv7Ccv4K4qiY583pEKhYck0Fswax6ate0mquI7d3vju0d3k65rTJ4/m0LFW0/kMGf1zVVb/lCQRyUWWdfROy2Sx3573LQxMDHkMBQMsr6vhjrkTicbi6JJga7dx614WzBpn9p9Pvfgu0yePPqmt5NRfF2LzGY6QXEaEyhDFnpPB3tLJPfofuXJ883eeOakc92WfmUrprNmwyzKmrNmwi1TK2YFoXOuhaOtJo5rz+4GNW/c6fv9u368klq7d6dnKmT5wRV4fuGKQ9YE+RUKXJD460tJjqyU7+zw/mf3/ODIl2j1ccJst27h1b7fkl3GaYdu09W2W31TD6l+2n3PpnAkkkmlCwYClfe6ygmJn6vpjfiOP0qKnil/kI0mwbP5kUyExZsMlWUDzgv88PDz6gM72f04Txt/5xU6rDiHo3H79BB7c/Lqlz1v3xJvs3H3QjODT0Vm+dodt7PbGd4+eIF9nDWZX4DnpnwhwvCXFb7futcnyHXMn8vMn/9zpuibFfHvetzBwUTWdUDDAjZ8/1yJftvcrwFWXjLG0uf36CVQE5JPaSh3lFO0IwxmcK3dL50zgu796lWgs3iMyaHwX31v6aeKJdL8onObROxSTG7ev+0xV01x8GprtOy3qWl36AadByOn7NSZzStbsdNAb3e5vIOPziSyePZ6AIhNPpvH5Bs+Es0+ROBJL0pAjtz2xWrKzT/TScDi8HTgz+98ncv47szsubKCgSwLRWJw75k7k27dcnHEEp1Tmff58wqNCZrvO5pdxmmGLxuIMrfSzePZ4c4bu0S17uPeRV5gzc6ylrTEb15mZOi9XkEd3oKoagqoh6zqCqvWIcqKqsPmZiGXWevMzEdS0N3/m4eHRd3Sm/ysoR50OT734rtnnrVxwMZufibBz90Gz/YObX0fPLrnMRPEIHdaS8MZ3j66Sr2vGWlPEk2lH/RNdYNX6RnbuPsivnt7DglnjuP8rtdy78GI0XWfn7oMZ53GROXyNdkBB356n6w5cJFFgzsyxpsMFXN6vLtjaPLj5dRSf3G5HuchhMTlFc4+RBoYFMytWHq6fweLZ43l0yx4i+6M9KoOqqhEKBnpUJ/fofxQjxz3ZZxrRmR3VsnBbNSOJ9rGhqGt16QfQ7b6S3EnOh+un07BkWulPWuqCRW80VnoMplDTNPDMzn2MCFUQCvoZEargmZ37Bs14n1Qxnc+QkfGG9Y10d6a+zkZA/zj7/2uBKuAXZN7ZjcDxbriuAYGUrWD80GNvWGavf/HkbqKxOLdfP4FfPZ1RJjqbX8Zthi2VUrlv3U5b+9OHV5q5bfKXVhU7U9cf8xt5DE7UrLFqOF8Mbp7lyaqHh0f/opAcdTIwd+Y55ph+/1dqbf3foWgbzW0pwqNCXHXJGOrXvtReS+JW51oSXcm76+GRr7Nu27WPeVecx9I5E2w5oLWcKLbI/iir1zcC0LBkmrlUWPFl9OzO5PAtNCLP03UHLjIZu+hk71dziaiMJ9Msr6tBkQSOusihDu79dQ4dyii6zabzZNCjNykmN25P9ZmFRmcqEtTX1djaKRKk8pwaxVyrWz+gabqjXqSqGgJZZ5uql27kcxa/IjJn5jm25+ZXRJJtpX713YMkCVwy4eO2/OGSNDhWTBezeqArdOpYkUjkMYBwOPzPwNRIJKJl/94C/KH7Lq9/4zSr9qNNr7Ng1jhWr2/kwc2Zf2/btY+br74ATdMRJRFFEkiqOoeirQiShChBMqW5LoUaGlRYvWQamgayKCChk6Zd4QmPCjH7srOpqlSQJZHvLb2EeEJFEgXzXKqmM6Rc4YGv1nZ4rly8RO0DH3+Zj3hS46MjzUiSREARSbSl+vqyikYSXGR1EOW18hicJFMq1dXBovaJJ9LETrSdvKFHr+JTJJJqRkFcvWQaW156j8eff48RoTLu+acpyICYbZPQNIYG/ay5rZa0qruO14ejbcy+7GxbVI+q6o7tRVEo4fWjHqVGfvE1yER1ZuRSQxQFfJLIkHKF1Utq0XUdURQQBNA1Z8ddPJmJRVpeV4OmFrYsvJjl47Z78HTdAYuqavgk6aTvV3SRgaGVfmR0Ui5yuHrJNAKKxP1frUXN2lqxliTlAdk0wI1vJIW7jEqiwJTzRzJ98miC5T5irSm27drnyaCHK7n6giSKGedrF8IYi0kp0VN9plt0ZsOSWks/nkqqDA8qNCypPen9F3Otbv2AKDo8BDpXfLQvUZMqI0IBy3ML+EXSicGj9KkqjvnD82VsoGKsHrB/D2Lm4XQTXXVmDwcCQGv27yBwShePOWBwm1XLzXl35ulDmDvzHO756cuW2aZNW/eaeRqXzpnAo1v22HJ+dTRbbgwUG7futeUrMmYLNVV3nbFXVe2kNma/zG/kUTD+Mh+HjsdtM6Ejhgb6nRPaNQf0IJnRHKx4zldQfBJX3flEUfs89f1ZxHroejw6h1vkz6xLx6CqOsOHlnMi1mZrs2z+ZF58/UMunzKaFXU1rMrLJ/rolj3809Xn23SVRDLtkgvQc3Z4FIaTjnrvwotItWmsyqtRMjTo59md+5h07qmmzE05f6Qtis1oGyz3oac0Eq7ROtbota5E5Hm67sBGQj/p+3WXgUx/mFKd5TCRTBNPpmlqTloj/G+qAZ+EpunmN3LH3ImuMlrmc45MVCTBFtHp4dETeVxzU0pkZgh1V4eqIgkuEchdk1f36Ex7P55KqjnRx6rreYu51mLa9nUe7M7g80scjNrt/pGhAGpraV5zd1OMjA1EAoroKOOZAMTScUD/FtgZDof/k0yK8i8CD3f5qgYIbrNqsdaU+W9JsOdZbFjfyIJZ49i5+6Atajo3YiM3osOIckaANAJqNvpp0bXjqf/JS5bjb9q6l0XXjEfVdAKKzPhPDufZXQdyZttrkSXx5MsS84q4yKKIKEEilXGOl/pMn0fHxJMam7LVfo2IC0N2+tssoKrCi69/yMoFFyOJoGrwbOMHXFk7pt/di0fheM5Xj/6IlM3BnNYyq5Eq/BItSY1kSmXBrHE8tv0dIvuj7ZE/2UhSp+igNRt2sXLBxdz7yB944Ku1eeO1wD9dfT6VZYpNVznenOC///CBpf9/6sV3WXTNuD56Kh79Daeo4+OxJP+66TXbysCvzZnIlbVjONLUZsq4kTJm9ZJadPSM3qtm/p9KZ3RLxecWrWONXutKRF5vFUv26BsKeb9ubSBjc2maxsoFU9i09W0i+6MA2RQxMh8eipmpGCEj86t+mbHlmloS5jcSa00xIlRGKBhg9mVnEyz3IYogSyJtKZ1yv2xOGkb2R9ttOUEw7a9kSkPxiWgq5vhRrKxm0kfGSQuCJ+v9lEIjhYvFSClRXR3k8OGY6wRcUtVd7EfnorGFRgq7R2faV2YVGgFezLUW0zYNbMxru7GDZ1AKtCUGjt3fWYqRsYFIPKnxwmsHetxf0iUHdCQS+WY4HH4VmJ7d9PVIJPL7rl/WwMCtgvGjW/aYM2EIHUdJ5/+dOwtjRHSER4W48fPn8tSL73LVJWNYnRNZ8p28PI5OuR6Xza8BMJ3Qx5sTSKJQWG687GDkl0SirSfPw+fRjxBdqv26LDUqZSQJl5xOeBHQHh4eJYMkiRxvTZmRytdeehafnniGJRoht35Ebl42t8gNScz8P5nKFFwzIoKSmsDdP36J8KiQLdp55Cnl3DDzHEvEtBf16VEM+VHH4VEhQkP8jjIaGuK36KWGjO/cfZC5/5hE03TLCqbbr5/AUy++yw0zz+HehRdx7yOvdCinXY1i7m+5PD0TgUlIAAAgAElEQVSKo5D3m98GhwjH3BWry+bXmIE+bhF1pNv77Me2v0P9/Mm0JtL8aNPrhIIBbrrqfJb95CXL8RdeewH//fIHfGbSGZZvZumcCfzPHw/w2QvPsERbF2OLtUdt7vBsuX5Mb+VxdT+/S92dqy+wnb+YSGGfzzkC2ecTSOd8tMVEgGs6jte6YNY4W15nzeW+FlxtbysIzja0UMqpHweQ3d9ZiskfPhBRNY3Hn8+k98vlc1M/0fc5oHOJRCJPAMWFmA0S3CKE7/zSRHOGL60757gzoqTz/55y/kgkUczObGdmaYz8jUvnTCCZUrlj7kRirSke2/4O6bw8jk65HtdsaGTlgot5dtcBRoTKCCgy963LREvpiKiabptRz80dLYkCumSP5C40v55HiaLhWO23YUltH19Y8agqxFoSlrxWHx46QVWF35NPDw+PHsUtusdpexpMpy/AjJozzYkzaO+HVy64mOa2JFJWr9A03TVyQ9Xs0Z6SJKIjcP9XamlqTvLcqwdYMGscVZUKw6vKMkvLfZIX9enRafKjjmdfdjZ/P9pi2RYeFWLOzLGgY4nuN2qkrHviTQI+mW8+/LLtG1gwaxyr1jeyZkktD3w1k+tc03RkSUQXIa1ZIzi9KGaP7sQpwv/JF95lWd2FqGkdTdcRRRFRdC9C6JPb++zI/ihlfh8N2YmWBbPG8YPfvmpbLbB49niu/czZtnHhR5sy40L+9pPZYrnjkI7Axq17i7bl+luu24FOb+VxdT9/4StOiokUTqX0gqIzi4kAFwXn/Oqig6NYFEWuvfQsZtScaTm/U20MXRdcbeiStTsHkN3fWVJJlRFD8/Jg99P6V51BEkUWzjqfmvNPQ9d1BEGgcfdHJZcD2uMkWGfMVTTVOsMuS6ItKsPIAQ1YZtSnnD+SOTPPMWe8jfx4yZRKKBigzO+zzHrffv0EdvzvXy0zOVWViuOsqCxnDNEhFX40LXO849n80KFggHlXnGse27iOhvzZymDAcuzBlDNnIKLpztH5mu5c7beUKfeLDKkMWKJF6utqKPeLtLUOgilNDw+PPsEtumdoUCHqUIMhWOGzOOKM6OVcDkXbaG5LcvePXzJ1hLaESrBMtkVuLJs/mWcbP2BFTrSn0zUZEaVzZ56DjG46D7yoT4/Okh91XFWp8Isnd5symq9b5kf3V1Uq3DXvQsB9peChaBu6oHM8ZpXn+vk1tCZSPPlCRqaNaDpPnj26C6cI/y/OGMvfDrfYIpDvnn8h92/4o2WbIgkMqfBbvpFcWTfkO5dD0TYCiuw6Lrhtd7PF3MaC47EkgJkKRAfX1Iw9nes217kdjcUzKao853aH9HUUZ1ErTlwihZ28tJpLdObnp55lsUuLiQCXJLj+8rBDjSD7ClmfT+DTE8+wrKZ1isCG/mlDuz43ffD4cnyKxOHjcZvsDutC/vT+RJlf5LyzhrPioR0WGS/zi8S70V8yWOSpZHGKyvD7RBZdM56bZ+nIooAoCtmoaZH6tS8RCgbMmcK2eJrTqyuYM3Msm5/Za8vX+H8uPpNhVQEabstUYHarpKyqumnM3j1vMv909Xnmx7dg1jhTmQKYPnm0bWZx9fpGFs8ez33rdpr35lUJ79+ILrIiigL0M92vNaH1SD40Dw8Pj45wipJbvT6z6shpuxH1ede8C2mLpxFEwTG3aFNz0tzPiIpTRgZzKr9nxntJEriqdgxSjlPZ6ZqMKJdc57OHRyE4RT9KPpFESidY4aNhSW3G6BYForE4rfEUX5szkepQmWnkQEYOn3rxXW6/fgLNbUmGV5Vl8tomNdeVgiNCZaA71FLZkNFJr7pkTMnn3fTonzhF+MdakrZ8z6vXN7J6SS2rs3be8ViCjVv3svCacTS1JKgeGuCBr15CWtXQNMz+PtaactTB48k0Prm8w9Uu+dtFUUDHvnpVc6hD9ODm1/nqdZ9CEASLU9DNqew2xnXHCtj+WMitWArNVQyFR5qnkirVVX5LFKffJ5CMp7t0XONaPzrSgiRJrteqqhrDCz1/B5HC+YgukdViXoqIYiLAVRXT+Wycf82GXY72YSqtO9uSt3UtsrpUkETR8ZqlEr7m7ialOfdna24bHFHgbb3kL/Ec0CVAblSGhMCRpoRj5PH9X6klFAxw4+fPtSgF9XU1nDEy6DiDOHJYOT/5jz/xxRljkUSRsoDkWEl5x//+FYBQMMCJlgQfq67kq9d9ClEUGDbEGtkcLPdZnOBGuo/Th1eaHb7bbKe3TKv/IEsCN37+PA5HWwHwyZm/ZUlAddZhSpa+zofm4eExOMmPkoOOI9WC5T6un3E2Q8oVVFVn30cxtu3ax7wrzjVzixpRorn7BRQZVdNJWaI82yN4csdht2tSNQ1B9yaMPQrHcBBt3LqX6ZNHU1WpUB0qQ9N0UmkVVYM/vX2Qi8efjqbpfOfWqZxoSYIAx07EbRGkV10yxhJdZuS1XTpngiWqdNn8GjY/s5fldTVoLuN7QJH54cbXWDBrnLcaz+OkFFwITRLRJQFVzcjzkeNxNmx5i6pKBc2lbz0cbaV+7Q7L9msu/ST/9fxfmHfFeRyPJWxR0+VlMnNnnmNxvi6bP5nKch+v/PlvLJs/2Ra1ufeDI47bN22N8MZfjpgrbHfuPsi1l57FP178CcfrHV5VxspHCkvl4T6edP2b60nndilQTK7iYpzxkiSavoRC2hZy3GKu1adIHG5KFJiDufBIYUVyzgGtSIIlslvxiY7fgeITSamdP7+qutiSql3WZdnFhpaF3siC0ikq/CLzrjiPQ8far3neFedR4RdpHSSrhdMu7zjt8I4HIr3lLxkMz7JfkTvYGpHH7c5ehaVzJnBvnlLQsL6R1UumOc4g3rfoYnbuPsj/ufhMHnrsDb563af4t3//k23/BbPGmcUMc53YS+dM4OiJuGUmUdN127LJpXMmEFA6zhc5GGayBxQ6NDUnzGgO4z0Hy3wn37fE6Ot8aB4eHoMTt1yIbpFqlRU+xvhD3PPTly2TyU++8C5fv2EiaVXn0ad3m9HQxn7xZLrgFUfF5Gf08OgII39nfgBEbjG25XU1NDVbHRcrbqohFFROWqPkR5syuZ4f3bKHxbPHM/KUCg4eayE0RGHRNePMvOluEdKHopnUH55se3REofaJJIm0pFSix/IcxjfVUB7w8ddDMUdZrKr0Ex4VAjJyXlWpUFmmcPWnx9AUS1pWmXa0SmbNhl3Z1QMVbH4mYgkE2vxMhEXXjOfh/3rDtn365NE8u+uAae/t3H2QGTVn8tGRZsfrVXxSwU7lnhxPetK5XQoUk6u4GGd8T7RNqrApL1fzpq17WXTNeNsxi83B7BjV7BB1m1R13nrvCKsWT7Pkp73ogtMsx21LpHl6x/uWXNGPP/cO180I2+SmmPO725L2HND90YZOpDMrNPKvuTJQutfc3bhH2Tu84wFIb/lLxG47kke3oGo6oWCAVYunctbHhnDfoou544aJbNu1jyUPbCfWmnSdfXPa3pxNmm5UYZYl0TXqyk351zSd26+fkFnqCAgINmXpR5teR1V1BFVD1jP/L3SZVj8Lph00pFTd8T2nutmIy0STiKQFAV0SkaTu75b82crJhgwbs+Z+30CIofDw8ChVjFyIuX3P7ddP4PHn3rGMq8b2MkV2XBI9ffJojp2I8+Dm17nqkjG2vuysj1chCEK2H5XwKe5ZBp2uyVix5OFRDKqmM33yaEfdcfZlZ5t63vFY3PL7ql820tScsIzLbjVKguU+Ivuj3LduJ7HWJKFgAFFt1zMVSXD8xh7b/g4jQmWEgn5Ptj06pFD7JA0cPNZqdxj/spGDR1sIVmQChXJl8Y65E9mwZTeLvnABdVeex7on3uTuH7/Efev+QFWln1OG+ItaJSMImZWoO3cfZPX6RurX7mD1+kZ27j6IqmmO24PlPnN/49+SCJu2vm0bh5bX1eBXRHObgeloy6MnxxPDuV3IdfRHOoo2tLd1d8b3SltB56pLxrDuiTepX7uDdU+8yVWXjAHB6ZgaoWCA5XU1NCyZZtaJcrovBN1RF3I6riQJ/MPYaj48FONoU5wPD8X4h7HVSJJVHkRR4I2/HOG2727n1vu3c9t3t/PGX45kUnXYjgnL5k+2nN/IAZ2PLxtZnd/W57MfuLds6O6kP15zd+OTRFsfvnTOBHzSwOhzTkaFX3T0l1T4u9c34+lkJYbiE1n0hQuIJ1SWr21PAG4UhWhqTrrOzDhtj55IABBPpi058/LbxZNp14IXsiSy/ndvsWDWOM48bQhuBWHSJwnPH+gz2QMNt2WtmqZ328xVb0XFJ+PpnHxoRq5193xoHh4eHt1Bbp0HTQMdnZ8/+Wd27j5IrDXJfYumkkylKfPL/PzJP3Pz1Rc49rtVlQqt8RSR/VF+9fQeFswaZ0bRPdv4AZdOPMNWzNhpuWv+NXnpsDy6giQKHTqOjX8HFNn2e0CRibUmWDx7PAFFprJMcdRPY60p89+nDAngkwXUrFxLksjRWJLnXzvAt2+diqbp/P1oC796uj36uswnDoriQR6dp1D7RNV0M6Anv63ik3jk8T8z74pzWbV4Gum0xpGmNtb/7i0i+6PmStRc546xPL+YVTLxZJpUWneNUjvZN2T8W9UgGoub40mw3Ec8mWZo0E+w3F9wEbmeHE+KKmbXDykm2rCYSPMeaVtErmbFJzmulFZ8Eloyz+7S4akX37XVsFp0zTiHJybQ1Jy0R+iWKZZWsiTY0jYtnTMBWRJshQVVFdfVBPYc0CqyLHLvwosRBdB0SGsaqbRqa9sbNnR30x+vubuRJYFQ0G/qJfFkOjOJLQkDos85GS0JjRdeO2BZPfBs4wdcWTumW9MeDRZ56jdoKpxoti/HenBzJprkse3OUVMpVbVtXzqnPQIkWKHw9RsmsW3XPu6eZ529u2PuRMoDMsOHljnONMdaM0bvuifeRBQ7PyM90GeyBxqy5BwBIXfjLGBvRsUn42kEVeW04RUIquo5nz08PHoFVdUQVA1J16gIyNx89QXc/5Vapk8ezfrf/RlV0/n9y+8zffJo0yDNxYjiDFZkHHTGeJxKazy4+XXO/cRwexG29Y105HMzrsltxZKHRyHIQCjod9UdjX/H85wORuCEpsF963ZSv3YHD25+vUM9dtn8yejoCDlOEUOHePz59/jBb17j2Ik4pw+v5M4vTWLNklpC5T7P+exxUgq1TyRRMAN68tsOqfATjcVZ8dDLHDsRZ/ED2/mXn/3BTJfk5LgOKDKbtr5ts8vcVsmsuKmGEaeUs23XPsfIZUWyRyPnfkP1dTVs27UPyDgV6utqiMbirF7fyA83vsbQYAAJHVEUTKfyw/XTaVgyrcPAkJ4aT3Kd2w/XT+d7Sz89oNI2KhKO0YZOC5iKiTTvibYd5UrOR9OcI2k1hwhsGZg78xxLZPXcmec4Xmta1RyPm85zqguqbjoRG5ZMY/Hs8YSCfsvY0X6tzqsGnK4VXeA3v9/Dh4eaicYSfHiomd/8fg/oDisDesGG7m764zV3N/GkxjM79zEiVJHVbSp4Zuc+4smB0eecDFXTePz59yyrBx5//j3n1QtdwAs87SNyi12UB2QSSQ1V0xCBynJ7NEkoGGDUqUHqrjwPUYQ1t7VHcurooAu8umefOYPnVySGBv1848uTMnlryChOt1w7HlEU+M6tU4nGEqRVDV2Hqgo/Tc0J7l14Efc+8oplxvDRLXtyZi8zcxaFzkjn3qciCay4qYZVv7TmAcwvHuBRGgQUkXsXXsTBY63mLODIU8oJKCLxdPe8MCPlTH5BSy8q3sPDY6CQOw7qSZWKgIzikxg+tIwzTxuH4hO56pIxqJqOKMLKhRdxX844vLyuhoBfQtMU/vnGC9E0ncoyhQc3v05kf9Q1AlXVdSRJaq9Cr4imriGJomsFew+PjjDkWdNBR0Dx2XW7XN1xeV0NOu3RnCNCZaxceBE+SeSUqgBr77qM//yfd3h21wF+9XQm1/PHqisz5xIFvv6liciiiCQJJFMqaUHAX+ZDTaokciJXI/ujrHjoZQB+fs8MdF0noelIkuhF+Ht0yMkibQ2ZR4CRw8pt0ZXf+PIkWhNJvnVLJgo/P69seFSIqko/93+llqbmJPs+amLq+I/hkwXmzBzLa3sPct+iqbTGU5QHfKz/Xfsqme/cOhVBEBCEjINIkQUWXTMeBD2TU1cEXcs40pKqzvDsar9MCoPMfnfdeCGariOLAotnj+fmqzUkUcjW7qlF1TQUn4Su6bSlNPRYHCDjVAZQ9ZNG//VUkXk1p7BuKBjg8OFYl4/ZGxjP41C0FVz6oFRSzVmdmR2nXVZnqqrGqaGApW2mOFzKsW31UGvbgCKSaHNuO7yAaxAFWDjrfGrOP82Sf9kphiytao62XVq1r5Qu9PwAmobjcfMjdFVVIxiQ8Y8cclJ9RxQF5/sSsTk1FAnHIn2KhM2PociCow2tyAL5QeClQm/Y/aWOqmmUB3woPhFd15EkgfKAr9uL8JUqkig6fg/dnQN6MDzLkiM37cDlNaO48LxTLcVZvrVoqk1xmXfFuXzzZ+1FierranjhtQNceN6pPLplD0ODCjdffUGmExYFkmmVn/3nG46FYapDZaQ1DdAp8/tYs8HqEH7gq7WkUnq2SnkbdVeeRzyZ5tTh5SSSGmlVozWezi5nEQkF/QT8EvGEalH089MrTDl/JPOuOM+yrCGtacTiacoDMoKqewZCCeG2PKI7l00oPtFlmZaI5jlGPDw8+jlOaYbuXXgRqbTGql82EgoGLH3glPNHcvPVF7Bq8TQEQUCS4Mnn3+Xx598zx/6hQxWOHo8z74pzqSjzESxTTMfGY9vfIbI/moli0XXqH9phHnfOzHMKqkrv4eGGkzzfMXciw4YGzGXJoiig+CTu/NKkTG5OQedItI1v3TIVXdPx+yWampPc98hL5jG+ueAiLp8yGoCqSj///uzbvPGXI9x+/QSee/UAn73wDIueUF9Xw5BKHzL2JexTzh9JrMUreO1ROB2lkciV+VAwwKIvXIAsCaY8y1lHxT0PvWyZYPnGlyfxvV+/avbxRgF5Qx6b2xL84sm3iMbifP2GSTy2/W2m/cPpDKlUuOnKC7jpqgs41hRHkgTqf9Lej8+deY7FrnLq11947QA155/Gfz3/F5sduKKuhqHlPkRJoCWeJhpLkEyplJfJ3L/hj536Zrwi81aKKWp5pClR0HMrK/fxUTRue9enhgK05TmhlYDMoeP2ttVVfptj16dIHG5KnFQ3CCgS5501nBUP7bC0CygSiTbrtRZj2xV6fgC/y3H9PhE1aX2uR2PJgp5rMfelqTrptGZJAbLipho0h8hq3SmCuoPtpYAI2PIsCIMrXUJ5mcyF551qk4fyMpmkw2TPQKPCLzp+D5nJLq8IYb8mN+3AZyadYasU+8vf/dmS5H7OzLG2JScN6xuZUXMmP9r0OvOuOJerLhnDPT99mVvv3849P32Z5tY0N189zrEwDDrc+/ArNDWnTOez8fuqXzaiqjqSrqEIMLyqjGFVAT5x+hBizSlWPLSDxfdv5183vYauwy+e3M09P32Zdz9sYlHDNurX7iDamjJnfnOXBU+fPJr7HnnFXG5537qd3L/hjxw81sr7fzth7udRGsTTOtFsNdz6tTt46LE3iMYSxNPdN3hqKs7LtDx/iIeHxwDAKc3QwWOtZrTo7MvONvvA8KiQOZYvXP0sy9e+xN8Ot/DW+1Fz34b1jbS2qTzwqz8S8Eu0xdPUr32Ju3/8EuueeJMbP38uU84fyfK6Gn7+5J8t469TVXrP9+xRDCqCTZ5/uPE1NBXufeQP3Hr/dn7+5J9pbkty+Hgr+/9+gvof7+DuH+/g1jXbWPzAdpJJzSKLoWCApuYE3//Nq9z945f45s9e5jOTziAUDPDg5tf5wmfPdtSBNRVEh5QDN199gVfw2qNo3NJI5Pbhsy87m/s3/NEizyvW7uCvh5ot8nbfI68wtDLAt26Zyj/fOMletHB9I63xNDd+/lxCwQA/+O2rzJ15Dqm0zvd+9Sq33r+Nb/7sZYYPDfDw42+a+/5/08cSjcW5Y+5EltfVcGXtWY79+oyaM/nhxtccC4SuWt+ILgkcjSW556cvc/ePX+Lf/v1PxBMqoWDAco2FfjNekXkrxRS13Lh1LwtmjaNhyTQWzBrHxq17HZ9ba0JzfNetCbuDP5HSHdsmUnb7LalSkG4QTzqf3yk1QTG2XaHnh8JTexQjj/GUy32lHApBIpi6m9F21S8bsWeAzrR1sqGd2pYK8bRO9ETeNZ/oXru/1Em4yHlikKTgaHG5/5Zuvn/P29cH5Ba70B1yKu3cfZCqSj+rl0zjZ/XT+Vh1pePyWqNC8rCqMpuC8YPfvup47EPRNiRJYMGscYwa6XxcI81LrjKmqrqtMzfyUh+KtheYORRtyw6eAqqms2DWOMKjQgCuRQ4DikxAkQe1slKKFJprq0vncCl4kO7mXEMeHh4efYFTcavcXKC54+Lsy852nDSefdnZ5r65Y/+J5iQ/3PiabVxedM14ghU+du4+aO7nNv52d143j4GLJImkXMbsppaEZbKjpS1FMqVy2rAKix4YHhVCFAXLMXInYYzj5eqXsiQ4nlPXdZIpzZarVsj+nt9eLeHIM4/SJbcP78iOyd8WjcW5dc020mnnbyagyBY5VzWN1esbzZzRoWAAURS45tJPsryuhhmTz8AniaZzaN0TbzJ8aJnt2KFgAL8icsfciXx8hLOdl3aw6X60KZOD3fhWi/lmOiriOBgp+HkIcNUlYyz5j6+6ZIw9CpVMaoBCx3D3tvb34do2L7dztxyzqLb2+0qpzs81peZfa+HyqKou53ewdd1tVoe2rjZ06epc/fGau5ti5GEg0lv37zmg+4DcYheC4Fz4IpFUWbDqWf7lpy8juhTHMCokax10tPn7GUsT1z3xJvsPNjsX3cgmm5ckEV0SSQsCmu6s0IeG+C3FZowIrvq1L7GoYZsZkRUeFSLWmnI8XzyZJtaaGtTKSimiac7v3KngRGfxClN6eHgMZJz6uNwiVrnjoptzI1juM//OjNEZ1c2pqJVpuOnW87qNv5LoqYEehZEGVNWuV44IldHUnDT/rh4awCfLPPTYG9x6/3ZTD5wx+QzqrjyPvx6OWY7RkdwbMup0TkEQkETBFrnqpjN7eoVHZ8jtwzuyY/K3Gd+EYavl/27YPYaci0K7fBqpF5ev3WE6Jj839RNs3LrX4hz6+9EWy7Hz9zt4rMXx3G76fXNb0rTZivlmPF3eiuzSZ8n5460u2CadH9z8umNRO7d+0GkMl1yKyUkOxeRcjyvkF+B0O7/9mKJLW9GhbTH3JYrO31J+02Lksbj7cj6ukxrlbkPb25YKbr6ebjT7S55ivp2BSDHfQ1fwLI8+ILfi7HOvHrBVwF02fzL/+T/vAJkPf8tL7zm2ebbxA5bOmcDRpjZHYTna1Garkpy7NPGx7fYKy0vnTEAQ2/NX1a/dwaKGbaTSmuM5KgI+s8IyZCJZnnrxXctyoqdefJfZl53Ntl37WLnwIlYumELDkmmsXDCFu+dfSLBCMSs0D1ZlpRSRJeeBVu7Gd1RMpWYPDw+P/kZuHxceFWLlgil8rLqS79w6lWsvPYuKMpn6+TWmQ6Ij54YxRstypg/OdWTntpdEAUXCojds27XPpkfU19WgSD38ADwGDKqmo+uamSIuPCrE/V+Zxuol0zjztCH8/J7L+fm/XM6QCr8tvduDm1/n2s+czQ83vsamrW9bdE83OY4nM7VG/vT2QUfZFSXnQjaeXuHRneTK02Pb32HpHKvdtKKuhpGnlFu23X59u1308ht/5d482+eueReadk88mWbZ/MnIcrvO7ZR68f5Hd3Hz1eP4Wf10flY/nYfrp1NZrnDvwotc99u09W3b9Wbq9zjr903NSR7c/DpzZo4t6pvxvjkrsk9w7LNkn9V+0lxWKjsF+pT7RcdjlvvtrhyfLFhSeRp+A59sbysIOnfNu9Amn3n+Z3wu9+TzObiSBN3mX7j9+gnZwphW/C7H9fvstqYsiS7ybL2GYuRR8Yl8c4H1+/zmgotQHO5LlgTX78mpbU/b0N2N7E0kEVCcv7OAMjhcpj6fyLdvvZi1d13GT+++jLV3Xca3b73Y+TvvAoN1bOhT8otdlAdkS/XXjVv38uyuA2b7x59/j1mXjqHhttpMpIcgIEoCk887DR2dyjIfd8+bzP2P7uJQ1FqBHGDBrHFUVSpUlinmjDtkKob/6uk9ZjqO/QebeXTLHr7x5UmIgjV/UlrTuP36CZZCFrdfnyko4FckotmKySNOKbMVvLj9+gmcNryCc8/8FMdOxC3J++vratiy4z2isTj3LrwIAYF0NqrFq1ret8iy4FwVXBZJd1M13I4Kv3h4eHj0V3yKRFKFhKYRLFf4wdc+zZHjcUt/Wl9Xw6atezkeS7J49nhGjQyy/KYaVv/SWuBN03UalkwjNCTAD3/7Gnd+aSIjQmUMqVS4Y+5EMw1HrpGVSqoMDypZ3SLTt/oV0fK3W1V4Dw+jjoeq6URj8cyKOACfTCKV5P6v1tKWLWC2fK21WI0gOEdR+eRMKo1D0TZT9wyW+6jOynm+HIeCfnR0RgwtpywgZXXgdllWk6qjruDpFR7dSb48KT6RNUtqSWtapqiamkkj0LCkNuNg0wUe/q83AFi1eCpVlX5OtKQsts+y+ZM57xMhLp8yGlkSibWmSKY0Gr5SSzqtIWBPOxMKBmhpS9GQUzj+6zdMAgFWL5mW2S8vvU1kf5RHt+zhW7dMJXoiTqw1xaNb9nDXvEk2/f726yfwq6f3cCjaxsdHBFGEwr8Z75uzomo6siyYhVk1HVRds63yFYSMgyv3nY0IldmcvwCxtjRt8aTFX3DwWDOxNtnmzIknNJ7e8T4rF1yMJGai8B9/7h2umwdpRdUAACAASURBVBG2tRVFgXRat8jn12+YhChC7ttLp3Teeu8IqxZPQ9d1BEGgcfdHXHzB6bZrFXTBDEYLlvuItaZ46sV3ueWa8ba2iZTGC68dsFzrs40fcGXtWbZMJBI6oaCfxbPHE1Bk4sk0oaAfCd1yraqqMbzKb3lWfp9gK8AIIGAvLLi8rgYBu7NcFgWG5p1/aNCPLAok89u62tAC6RLNN6qjc+eXJvH937xqXvOdX5qEjmNWmAFJIqkhCrpFdprbEoMmB7QiwtG4aisKWlXm69Y0uX3mgA6Hw0OAl4ErI5HIB+FweAbwA6AM2ByJRO7JtvsUsA4YArwA3BqJREr00y0cVdUQyLyAZFvK/LeOzht/OWJpOyJUZuZeueenmSrL4VEhbvz8uaajd8r5I/nOrVPRdB1REEimVaKxOIeibax74k3TeTz7srMtg11kf5R1T7zJglnjWL2+MbOURBAsOZnCo0L4JMlxMJk+eTSPbX/HdHIHyxTTeIb2yJd7F16Mpum25P0N6xtpuK2WL38OmmJJ7s2pij6YKyiXAqm0bhbHMN75xq17WXTt+G4diHK/BVQdzx3i4eHRn/EpEkdiSYsC951bp9pybjasbzTH3vvW7WREqIwHvnoJKxdcTHNbkqbmJOt/9xaR/VFGhMpYMGtcxhkoitx144UEy/yIFdCwpDY79mMx+lNJNadvhWSb9e+U19l6OGCsgMs3nIdX+TlyIsGaDbtYMGscPlkwjXZol+mVCy52dKoYaeEORduI7I+aOueCWeN4bPs7LJ49nlNPqUCSBMp8Iqls5L8EJNs0myx3hKdXeHQnufKkZSft/JJINJa0fSfDqgJcd/lY4gmVZErlcLTV9p2s2bCL79w6lZZ4mnsfecXimN78TIRF1463fUNzZo41nc/GcX7w21dZPDvj1Ltv3U5WLphi2y8ai7P/7zFWr28EsikMdLIO41qONLXR1JzkV0/vMccavyKRjqc6/Yy8bw5ONCfNaHQjMKyyTLG0MSJq89vJkmAr2CeJAv+66U+2frVhyTTIy88qSwJv/OWIJZhtRKiMG/7xHPK9SJoKP/jtqza5WrOk1np+dC44azgrHmqfcFxRV4OE/V2LElx3+VhOZNPQ+GSB6y4fiyhhuy9V03n8+fd4/Pn3LNs/N/UTNieV5pKHNn+7T5E43JSwOdGGBxXbpHsiZc+Hvnp9Iw1Lam22biKt8+iWt5g+eTQBJWMnP7rlLUe7uLds6O5EEkVkWbA42GU5ExQ4WDwxiiKitQjUr33JIjuKIpLspuC7UqbFpdip0/fQFfrEAR0Oh6cAjwBjs3+XAb8ALgUOAFvC4fDnIpHI74FfAwsikcgr4XD458BC4KG+uO7eQAZW1NWwKqfTXJGNaErk5HrOjV4+87QgkpT5ME40J6iqDCBLkjkbf7w5Qbnfxx1zJyDLmaUFuZ2yES1tzHT5fCJ6Cn6+YgYgoOo60RNtXHPpJy0RKsZsuWFIAPx02XTHyJdESnUvDKPqoGPes7E9MwBMcxX43Agdp9n2k/3u0TGqqnE8Zp3TPR5Loqq6t3TCw8NjUGFENBsREYqUMXpUBNJaJu+sLAnIouBYgT4aSziOf7n5nQ9F20imNCrKJJJpiXVPvGkZb5968V3q62r409sHCVYESKU1QkMCyBKUBSSSralBb/R7FIckiWiCQErVEUVQfBKptOZqkD//aiZSzSdnaoOEggGLXB+KttGaSDmumHts+zu2SGdD/4zG4lRVBjjRmmDkKeVeZL5HSdCRHZEGx+9kzW21DCn3c/+GHdwxd6L5Wy7GmNAaT1n2X7NhF6uXTAN0VtxUYwbtjAiVceqwCsfjnDasAkEU+P7XLqGqwm+LujRW2kB7KgIESOsCfp+A32cda5bX1VBV4edoPJWtBSSQVnU0TUeWREQRkimty3ZXZ2y4/oKq4ljMbfWSWkvuU0HVqQ6VWSKlZVlAcHC0ysD3vnYJqaRu6iE+RUBParZx3z2SVbfZ1B0Vg8995qqqcYqxqkrXkQT3VVSqphOsUBhS7jejpXVBR9Xs55dEgQe+Mo1hVeXmfR1tas2kfch7DmlgQ54DeMOWt1h0zTjLcZMqBTvROiqCmC9zqqqxc/dBS4FngJuvttvF/dGG1nWdf3/mbcvz/fdn3ubWL9gj1wcqiaTGpryJg00lPnHQnXRUQLQ75bavvoGFwG3Ar7J/1wDvRCKR9wHC4fCvgS+Gw+G3gLJIJPJKtt164D4GsAMaMvlXcmefjLwrRlJ9p+jldU+8mcnFJEt882cvmwPOvQsvQhREVj78B3Pb3fMv5KvXfYqAX2ZIhcLxWJy6K89D12Fo0M+BgzGefOFdWyqNu+a173fKkAAPP/6GWa0Z2gsiOkW+BBTZzIeU/5skCe5VN10E3i1Cx4iYPtnvHidH8UnMu+Jc28y84hPRBslSFA+PwUgypVJdHXT93em3eCJN7ESbQ+v+j1NE88qFF5FOaxYHwddvmESwwockirbxLJlSHcc/o4Cv8bckCvz1cAtDKxW+Nmciw4cGMsV4BJ1F14xn+x/385v/FzHbL549nvKAj+FDywaFcuzRfUiSyPG2lCnDU84fyfWXn0MqrTrqY6IEl0z4OPet+4NFJ3h0yx5TFxwRKiN6ImFZGTesKsCmrRGe3XWAWGuSNbfVklZ1JCkTWbXo2nEcbYqz+Zm9zJ15jqPzxcOjtzmZHaG6FIBPZ1MlHoq2EWtN4ZOdbZ+m5qRlAtLY/2hTnF88uZtFX7jAtAWrKv1ETyQcj7P/YDPrnniTZfMn88h/vWmmdDp1WAXREwlkWeCfrr6A62aECZb7+P3L7/P48++ZzunqKr8tdYYoCkiSSEtKJXosYbMDjEmjztpdnbXhhg7tH32D5iIbel5uZ1ESaI3Z77MsqKDm+XV9fomD0bgtqndkKIDaan2+skskqyyKqHmxrHK26Fq+XMmSCDkRn5IkctQh4t/p/fp9EoeP26+1emiAZNratsIvccIn2yJOK/wSrXn3JYiCY5pPQRTI9cIX40Qr9P6hvUCbzY+Rd37onza0Do7Pt398dd2F7vgMBguSy/cgSYJt9URXEPI7w94kHA5/AHwGuBi4IhKJfDm7fQZwF7AS+G4kEqnNbv8k8HQkEhlbwOHPBN7v9ovuYaKxON/40Qu2F/+9pZ+mqsLPvr+f4Du/2OkYhbxywRTLMi/AcZux5BEwZ70hk7DfWFJpOLVz95ty/kjmff58BAECfol0WiMaS9DUnGTbrn1cc+knqSj3EWuxLju6Y+5E/uv5v3DLF8bz0ZEWW2d8enUlmqbzuxffZUbNmZYcUNd+9mxCwUBRzykUDLj+vua2WoZVOVfiLZDesPHPpARk93C0lWU/ecnxGVaHyvvwyjw6ScnJ7lV3PlHwgZ/6/qyi2ndln85Qyucp5edWAL3lVz0TF9l16gvdxtavzZnIyFPKTWMK4Ev/GOayC0ehahnHxHOvHuCZxv3mcuuduw+axty7H0bZ/GwmHUEqrbPuiTf59i1TOXYijs8n8sCjf7SMn0MqFH79+z3c9sVPOY6VHn1KyfW5uURjcX7yH39i+uTRVA8NUFmucOxEnMoyhT+9fZCLx5+OpuloGogiiILgqBMsnj3eTCGzvK6GjVv3mjJtRO5PnzyadU+8yT3/NIXRpw4xdTBN02lqSZBKa/hkkaoKf1f0M4/uo6RltzfojJ0x5fyR3HLteHQd6te+RCgYYOG1FyAKAms2tNfpuWvehSSTGqGgnwOHms2ChXNmjuXUYRXIksjTO97j3E8MJ1juI61qDB3it6V1yLUBR4TK+NYtU9n/9xiNuz9i2j+czshTKjh4rIXygExTc8qSctG4HzedPhqL8+HBZv5102uONqSRQudkdpfxe1efrduxHOhz2S3UfjocbeVnj7/B9MmjzWjLbbv2ccu1423v5FC0lXqHYzbcVsuIvLbF2G/Hmtr48HCzzTb/eHUlp1S1F6TLHS9yr9VJ9zh0rNWiA5nXuqSWEad0/r4KbXso2srDDs81k9qmc/dvPIMjx9ss3/Ky+ZMZPrTM9gw6YUP3udwW8y4GKsXI7kDkREuceEpFUzFXL4gSBHwSQypc+96iZbdUVgGIYJlgEcjkvnfbXjBHjzajaf1n7iYt2AtPHIq2EU+kScdTVAVkc6b6g49ipuIBUFmu2PYNKHKHy35zf8vdFiz3WX4Ljwpx1SVjuG/dHwgFA7ZZvWXzJ/P0jveJtSZZdO04vn3rVFRV52hTm5nD8uarNR7dsseyrOHRLXuou/I83vvrcT498QxLZE19XQ0iOocPx8zrqK4OcvhwzPU5JRIqR7VWkinn2c/Dx9s40ZLsdCR0R1GB3U1fy67qEMVnzCDnvpP+hiFDA4VC76fUZLc3r6dYnJ5nR8+5s/dSrByW8jOD4u+nEHr7np1kN3e8+dI/hvnMpDPQXarXD6sKEGtLmKmuLq8ZxYXnnWqL7rlo/Gk88dy73HTVBXzhs2cTCvo5cryVf/uPTPGqgCITUDLHFASB9b97C8BSuO3gsVZUTWfuzHOoqvAPqH6tu+jL/r7U+lwbPomrLhnDUy++yxdnjOXDQzECioxPgU+FR9gCBr59y//P3r3HSVHdeeP/1DnVlxlooJWBaBIkMVoYhCeIDImw8Vlx2WdjIiT8NqCuOlmR6GSjz8Zs4sDuo+5PGZP8svvS7GKCZEN03WB2SSResmuATVZJBGLYB0RtiVEw0XBtoGGmp7ur6vdHdRV9qerp+/Xzfr3yCvZUV1efPnVudc73XOaa59/dE8LagSudGc0L5pyHxZd/ALGhpNNGvenqGbi/fz4kTBw9eirvUhQAqZSOoyXGnW0W7diuqJdat3XL/W0K9ccOx2OQUmSFu5g7fTKWLZyGO//RGni2Y/s+/MOXcOv/MwP33nIZorERpHQDqZTpDOzaA9KplOnE4rVWI2hZA11/ec0lOLdnDP7fWy6DAuT1AQ9FhxE9GXdmQ2c+3Lzzxl68u8fnbDq/sq8XG7fuQ+RAFCndyEufnp4Qkikd4XEBzz6kNiWMJVdcYK2YODYE3TCceO6Z12Snl1va2uew+4TJlF6wj5dM5V9rrmbIu6Yq8sIN/eU1l8DM6dOaqvCYcZrfz9I90kTX84/1Sj+33zqlKHj5N0ezNhf86YtvoWdCV961fuoPL8gL62H/ZkVdq0v/0ft75V9rsef1BVUsWzgtbwa2T1Vcv/9//vKt/A0b/0jLO9ZUBVRVZIVMSRmGa74stQ/dDPm2lN+tXRkev5thdkYaBLt9OHkqmXfvdIVlVfvAYvRD6uK3AM7J+O93AXi7wOttyw6zkclZ3oH0Jg+6FXtr3aY9TiWvTQkj1OXPe288kXI9XzyRQmwomfW32FDSOT6lG1l/W3LFBU4FueSKC/JiW93/3Z340wUXYvmiGThyfBh/882fo/+rW/E33/qF82ReCAXRWByr1+/AwJptWL1+B6Ixa1fmD1042TVe00jShCEFhF9asQoNE6a0su1dy+dCmxLO+l5CAr87fBq/O3zK9XufOGUtH7JXEVjxzQRSigJTCkjZLLdE4wmPvMjZSUTUSexll9f9sYZLP/gurHpoG958J+ZaPpom8LMXf4vuoLUPwxWXTnGt24I+Fbt/fQSqVBDq9uP7m1/Dyod+4Zwns442TRORA1Fnv4W//96v8JvfncQDG3Yh1OVHuNvHcplKZhrAg4/vwtUfPR8jCR3//os3ERtKQkLi0LGhvHbe20fc21VSKBACGEmk8MbbJ6yQcOk2nt3+kwKArjMEGrWMYvpj1kZ+87B2YAFWLJ7plPWRA1E88vQruHXJTPzvZbMwJujHj3/+Bk6cSmDihK68Td9ipxNZry2Yc54z+Gwf8/ff+xUSCQOfHdyCI8fjWX1A+9piQ0mnT7ZgznnOe+//7g74VCvW8y1f2YotO/fjSzfMxrr0Xj9QZX7/x1Tw+6OnXdMgpRu4/mMXYcvO/XjniDVjcMXgFqzbtAfXf+wip282KdwFVeT3q6RQMHf6ZFz/sYuc8mLdpj2InbbiTnulvU9tkT6aqeCJn/0ayxfNwGD/PCxfNANP/OzXgJlbTytO3xqwfqsHH98FtwmFdjskU2Z+LO7Y/PQLBlTMvmgyVj20DSsGt2DVQ9sw+6LJCAay5ygqUJzBZ/tav/7Yi1DcrlV6fL6s7FpVj/OqOWmQdNmHY3D9DiST+YOufp/EH15qTYC75Stbcc+6X+APL30v/K55TcFjP34Fvz10CtHYCH576BQe+/ErLsdZK4Zc+9BK87bVPH+3Dmpf2uFiM1khWTojDYYT7psQDlc5bEyzlOTbAWiapn1A0zQJ4FoAP45EIvsBxDVNm5c+7noAP27URdaDCisUhp357SWNeTGLco5btvBCfOepl3Db0llZ750wNoABl/OdO3Espp47DnfeOMf525ad+9ET7nZiFN2+7My5xo89M7s6d3Y0cGYmtW6Y+KcfvZx3HSv7eqEIZJ3TftK7ces+SOG+SUcypWPlmm343eHTGDFN7P/9SQyssSrJhzbuxg1XWQ0d+zMOHrU2etjw7Gt512B/lv00z44xZp9vYM02RIeSHIR2mK5pSETUSfwSGOjrxf+c/V6nYbZx6z7Xeu47T72EK3un4m+++Qt879lXPWMRmqaJlX292P7SOzh2Mo7dvz7inOf2ZbMwYWwAW3bux8q+XiR0Petz7rhutvM3FSYH9agsdt48e3yXs+/Huk17oBuG6+q5Dc++ltc+vfPGORhKJKEowN0Pv+Da9nJrwxI1u2L6Y/akINU088r6yIEo7lm3HVCA0/EkZl/0LqzbtAfHTsRHXa3q2c9SgE9e/n5MOqvLsz9lH5u7wa29Ea42JYw/vfJC/P7oEFau2YYVg5tx5z8+j+PD2f0fwzBc7+fbl82CENbA6YI557kOoC654gLn2HgyldevUgHcdPXFee+1Jwh5pf34MYEifrnG80tg2cJpWYPryxZOg19mH+cVK9ptdmrAL/L68wN9vQj48/usAZ/ifqwvfxAtmTLyHnbc/92dSObEak7pXjHP86/Vp7pfq9+Xf61jAu7HjgnkHyuEkpfv7fyYyfBYoWa4hJ3VDdN1w0jd5TeQQsHSP9Kyftelf6S5D04qHn3oJh7HHOORx8a45LF2pSjueUxp4gcH1eS5J1uV9+ZoijZhJBKJa5rWB2AjgCCAZwD8W/rP1wF4WNO0cQB+BeDBhlxknWQ+UYeiAOaZgPmmFM4mEQCc4+xCcvvegzgeS+BvP3sZoietmcXf/MEevHfyWGfXcikEVJ8CPWUipRv41g/2ZC3p3fBsBH/8kakYP8aPrz/2K+dvY9Ozqw9Fh51ZWbnxcYRQoCjWLOdHnzkTaiOeSGFCKIBkykBPuBv33nIZAAW/O3xm+ZhuwPWcunGmMrj75o848a+BzF2F51nluQIMj6RwKDqMQ9FhPPrMK7hr+UdwajiBE6cSWXHSpFA8d7AetM/XxErZabpspoInn3s9K2TKk8+9jhWLO2c3XOo8hTbga/YQGFQbyYSOiSE/hjNCO0UORJ16buo5Ibz5Tgxjunw4HkvAp1rLGDfvfAvLFmqudZsUAhNCAfSEx+CciWMw+Ln5MAxr+atPKkgZ1qaD3QGBkZSJ1f3zYJiAKhSYMLFi8YxRy/261BPUkqQUMKHgK38xH1IouPqj5zudcCmEsxouM99GY3EYZjovGiakEPD5rJVpdtgzu+2V2a70YfR8x7xKzSazPzZavrTvJ7ey/sjxYZw1rgtPPvcqli+agfC4YN5xufebVz/r7cOn8PH55yOR0p2Qhu+ZNBYHj53OCsdhz4bOfO+JUwkA1orW2OlE1h4Gh6LDuO872f0fe9Vq5v1smsC5PWOQSOrOILfbYMWUyWOxfNEMZ7PC3H6VnrFRY+57dcOEYpquad8qK33sNsNg/3zopgmpKPBL6/VMiuLe93Ub69ITOsaFfFmhMoRqvZ4rEU+lN5ecD90wIIVAwKcgEc/fRazQgFPmIJEQ7tcqBPI24EsmDWx49tWs/uOGZ1/FisUz8/rXp0d0CMXMutZTwyM4PaLnHZtIuofyvOO62dnXqrjfi26zj4v9/oA1CP/crt9mhevYvONNXDX//PwZnS3Yhz49YuDl3xzJymM79r6DcRef2/TjItWSSOpF5bF2VcpGm5VoaFpGIpGpGf/eAuB/uBzzfwH01vGyGk7XDSjIiFtWYLdgRTegwhqcnhTuQuRAFAd+H8vaQDByIIrdvz6C1f3zMJLUoZsKAooCMyMkBgA8vPJK7P71EWze+RZW9vVm/U2bEnbimW3cus/5txMf5sZeKIr11PG2pbPw4OO7nA0q7rxxDh7a+H+dWGS3L5uF0Bg/fKpENBYHYBXgdszMzJgzP/zpmaf5QnGfJW0YJlTTCs2R2YiLHIjiwcd34YarLnLSI3MGw4jHk2e3XXKbSSk7TVeiKyBcY2h1BQTiQ1UshYiaiN8nm2kDPmoSyYQOKWVWwyxyIIp1m/ZgsH8+fKoCv1/gs5+a4YSAOhQdxn/+8i3Xug3CxIuvHMTjm/fhvlvn4ebVmzEpbG10oidSTmN/OF3WivT/YJyZQFOoFK5XPUGtR0qB40NJ3JeTJ8OhYLoNpGPSWd157bwvXX8pFAAPP7EH/+sjU5FMmZh6Tghff+xXzoxHu+1lt/8G++cVNfjMvErNyO6PqQCgm65lrp1/v/fsq07/x87H9iaBE0J+J6ZzZnxo+7jQGH9WzOAtO/fjzhvn5MWAXv/Uy7jjukucyT6r1++ANiWM6z92kdOfsvtdj/8k4vz3l2+Yg+9vtv7bbQ8g+78NA7An6dqzkFev3+Hczyv7ep0wkPYgt9tgxYGDp5z+IwDXfpUd5s99oMMsKu2bWTJhDaCek+7LJ12+gCqVvLxw+7JZUKUCI+d4XTegQiBlJQ+EAqi694O6RDyVkX46XMapAVgDTnOnT87bsC93wEmVwuNaBYyca9BNE9v3HsT2vQezXl++yETOJHBAAWJDKdz3nZ1Z983Ybn/+tcrscQsgI7RH5ti6Yrrei26jqHZYj9x8qOaeE4CUwKUffFfWflW3L5sFKRWYOenbin1o3TDw8Ka9eHjT3qzX53zwnKYeF6kmKUVxeaxNdadXJOTm2+6AcPoj1dAp+amlFTNTN7Oh4DZAfPuyWfjaoy8iGotjVV8vAt2+rPdYM5sTnoPM0Vgc4ZD1NPXICWsjg89/+kNQ04O+oTE+fO3RF7Fs4YX491+86Tw5GjcmgEee2etUQpmzlh/78Svp2QABjO3y41ev/t56qigVqNLaFGDzzrcA2E+ECzdWVACTczpN0VgcwYDE5z/9Ifh9EuFQAF0+kR5MKPSUp3k3rqzXzO1EysRZOU/QVZ+CRKp504aIqFbsZa2ZDbM7b5yD7z37KmJDCdx09cXOAIPd+XnsPyLoDqrpctSayRXwC2x/6R08vnkfBvp68dMX38paIuvVUSxFK6/woerLnGEMRXEGn4EzMf5uXTIT96zbjjffOYXD0dO47H+cm57tbA2UHD0Zx4afRHD1R89HwC/xr5tfw2c+fjEiB6LYsnN/3r1hP+wfLTszr1Iry8y/x2MJfP7TH8LECV34/VFrVnI0FscNV12EgF/ijutmY9wYP6QU+NL1l2JMlw+/P3oaD//wJUwI+a0VooqCVMqAaRq4a/lHMDSSxKmhBAzTRDQWx+8On8KGZ19z+jqRA1E8+dzruPeWyxAbSuLoiTie2fYGFsw5D5/6wwsQ6vbjB/+5D5/4g/PxxtsnERtKwqe696egwIm9W2gGuJregNFt0P32ZbPwyNOvZJ3XrV+V2wctpcxoFxLAhFAAty6ZiaBfdVYMSwBuw8q1GJT3+90HSv1+geTwmU+QMD2u1cy7VrXQw4Xcg033ONiD/fPzrlUVCr5w7WwnXvqkcBe+cO1sqELJTosSZh/7fcI1H/p9AolUzox1KAgGZFYaBAMSCoDcnnF8KInJ4WBWH9oafG7ejXbrNfu1mdlhSHLvhzF+gaFU+yfCiMfYz0iVx344AN0C9CJm6uY2FPw+gfv75yNlmPjd4RgeefrM0qz77Ia9y3uGRs4UrEIA991qLbVUpcDaJ3Yj1O3Hx+a9L2sX3DtvnIMNz0YQORDFhmdfww1XXeQMAH/lL+bnPQE9FLU23Fh8+Qecp/1zp0/GZz5+MRQB+KRA9FQcP9lxAMCZ+Ds+n8Bf//lcJwxHbmNF1w2M8Ul094zJ6jQJAYS6DKfxZC+BatXGTzH5oRoM08SxaByx0wmnog2N8eOsccEqfgoRUWvIXtZqhbH6wX/uy1g1NJIVhiBzIOKrj/4S0VgcK/t6EQwEMf38Hqy+cBLGBCQuv+Q9uOLSKZ5LZMtRr3qCmp/PL3E0lnDaOg996QrXvHHuxLGYFO7Cxq37cMNVF+HL33jeaZ/9+ScuxvixAfz5Jy7GkRPD+NfNr2HZwmkY06Vi7cACSKHAL5WiwhXkYl6lVpaZfyMHovibb/0C2pQwvnT9pbjjutkQwgqrZMLq3/h91oCVKgXWP7UXC+achz+/ejrCoQCkFDhyfBhf/ofn8z7nK38x3xnczdzk8N09Y6EKAZ8EhtIb1B+KDmP3r49g1Wd6MZxIYvevj+Ctg6dw65KZeO/kEBJJPW+i0hf/bDaOnRzGxPFnNuDKHfBEeuP2EcPEhJAfn108E2Y6fIKRvu/jyVTWbGyvflUpIU7aVUI38d+RQ+idfk5WyIMPX3xO3R6+JZLum44Nfm5+1jUkEzrGBVUEJ48rGFYEAHzCvX/tE0Du8GuheM25s6VNmPD5lKwBYJ/PCkuWyY7BnTeoLpE/E900ERqbE9pEWq/nMkwz72XThGtsacAahM6chd6sM59t9n4nRaVbmzo9YiCZTGUNwB49MYTTI0ZHPBDXTRPROoz9sG3XfyXOeAAAIABJREFUIJmzUVQhIKRVCbhVwHKUZUq555NCgaKbVnAiRbE2wciQ2bDPbFwYCR2hgIrA5HHQDQPJlInHfxLB5p1vYe3AAmcg+a2Dp/D5T38IZ4/vgt8n8LVHX3QGt+1G0er++emC3D1m1OHjcWzcug+f//SHMPmsMRDCGixWdBMjuoG1P3gpL/7OF6+bjfPeNa5gY0XXDUDPWK6csh62uj0tbtXGj717tPtyqeo9oTIME/+6+TUsmHMegn4gmbL+++bFM5tm91IionrSk3b9YKL/q1ud10PdPpw4lcgKQ/A33/oF5k6fjBWLZ1rx46SCoF/g1HAKPinw368dxIwP9EA1zcJLZF3i4wLp1YAKAFPBoWNDgBROHVZMu4Han5QCw0kja4ax4pE3FAXOqrSeCV342xWXQRGAKgQAE6oqkNJN9Ezows2LZ1jxn5N6Ov+aMHJaBkIqSEGM2r5iXqVmVyhGuVv+tQZgTaim1SfR02W71deyHjKGu31Y8cmZOHRsCCdOJfBPP9qLvo9/0DOsxcTxXfjqo7/M6m/ds247Hh64EtB1JHXg7PQs6mhsBCdOJWACef2prz36Iu7suxSqlFid7v8cj43g25v2Yvnii103X7PTwDtUjm4NFurAGJ8sul/V6mE2KiWlgovedzZWPbQta1KXdAnBYR1ffKx8n18iocMZRPMaLPZ6AGgY+QPAxYQVsY87246BPcrnlxIHO5G0xgeWXHGB0y9d+4OXcMd1l2QNaGXF4B7l8xPxFIJdPsQNA7ppQigKgqrAyHD+TGXvfvGMtugXFxu7vJ3phoEv/cO2vNfXDizoiEFTs05jP52Qlk3HrRK3n2rbM6TCGbsXjzZTt1CjwIR7we7WsJdSZM2SsWMmvXXwVFasLrtjbcertJ9026KxOBRYg89rn9jjGRMtciCKb3z/v53Z2HZl6xV/RwgrZpgd97oajZVWbPwE/dL1yW7QLzEyXL3Bc1WKrI2JCsUmI6LOVmjzRi/xkRRiJ4dHP7BJZNa1f/vZy/I2jdqyc39efXfNwmlY+8TurJVAk8JduO/WeThr/OiDbG71+6q+Xvh8At99+mV84g/Oz/q8zPZDK67woepKAc7MfNvRE8OuYdoOHx92Yr3ed+s8/J+1P89rO67un4e//ubPXfNbZj6dO31yXjvFK65zq65Go84wWozygM99yXbAJ5AoMKFF1w34gj5nJSjgXY98+YY5+P2x03n9rdxN4BK66dyfAPL287Hfo+sm/vqb2/Lu71CXH36f+zBDsaFyWrFf1Si6DifON2Cl6f3f3YnB/vl5sy1LiZXv80sciSXy8uTEkD9vMNFrwz7FbQS4BPZgtT3712uwupQ42FJ4xIB2aUcV+/lSChw+Hi8qXVWptH2/uNiHDO3KMzyrFEAHhODweYz9+KTiPEitBg5AN4BbJf7Ahl1YvmiGs9nDYP885/jRZuoWahSU0rB3O8+Dj1vXpShwrSCOn47nNZQyN/nbvvcgjscSWbGe1z/1EiIHop7XUuiaCYgnPJZLuTRYKuFTFdd4Xz5VwUiiih9ERC2v3M0bYzW6nlrIrCNzB/G27NyPZQunOTu/jx/rRzgUwP53TmDpH2l44+2TWR3Bn774Fj466z2jDrK51cv3peP1LphzXl7sRGdAoEVX+FB16YaZNTMfAB55+hWs+NTFeXEs1/7gJWezMyGQtwnaQF8vnn7+N675DcjOpwvmnJfXTvGK69yqq9GoM4w28DqSNJ1y355lvOHZV7Fi8cxR2+TjxwSy+jtu9cj4MQH829bX8NbBU579LbsOyZ3NunHrPtcYzRuejeT16W5bOgvfeeolfNYlTq7bue20YKic8umGUXSalhIrP6Gj6H5iKQPAtaDoJsI5fc1wKGCt5M69VlT/YWUp6VrKtVJrKvhAsQMGoAG4xjmvNtYZDeBViWfuTJy7BKrQE+VCjQLFNItu2HudZ+o5IUgh0B30ZWXIgN/qsABIxyELQQqcWQKcfopk74gOwFmOfNPV7uFG7O/Kzog37waLUdUbOpEyEQxITEmHZJFCAMLkJoRE1JEy68hHnn4FN38yexBv3FgfViye6ZSXPp/Aw5v2YuYHJlob7ArAMIGUYeAnOw5gwaVToI8ypcCrXg76VQT9KNh55Uw0kkLJm1EZjcXR5Vcx4jMwfmwAZgyIj+jO8v8nfvZr3HT1DJw9IejEO5dSgRTAD3/2m6zzZ7ZXM/NiqNtX0mAV8yo1q9EGXnXDwPa9B/P2u7np6tHb5EIoef0dv1SwYvEMJ0RjPJnC7l8fwaHosLPZoAJrf5vRQjbaGxTa9/GR48MwYWLBnCnoCXc5m8nHhpLOytSbrnbvazFUTvWVsulbKQ8ASusnmnn9++6gD/nb6tWGvX/TlMmh0ccpajA+UFK6lnCt1JqG4in88uXfZ8UE/+mLb+HyS97TEYOmIyn3MDdf/LPZeSF5KtEJadl0vCrxWHpnVKfyqfB8dqOg2IZ9ofMo6WVmAKCqAueExuI7GTOZw6EgAsIqhO3zuz2pvGbhNKiwBsYLXQs7I968GywC1VwfIQAci43g5KlExgCLHxPHdfH3IKKOI3NCUT38w5ewbOGFOHt8l/Xw1QB0XXeWfCoZG7ps3vmWM8vsyedex8q+XkiMXrd51cvxRArJlMkBASpIBXDNwmn4Xs7M/O6ARCJp4LtP78Wnr9TwlUfOzHS+beksfPtHe7Bi8QwouuEsxdW9lqam26u5IWmYN6kdjNbHqrRNntvfSerICh3gGVNZz5/1WajfpQqgO6g6f7tr+Vw8tHG35/2ci6Fyqq+UTd9KeQBQSp5UdNPp39sCPlHXWb2l9PmrPT5Q6oMVjk+0NykU/GTHATz2HxHntUnhLlxx6Xs7ou2iFgpzU8XnLO0QM73l2JX4pLC107C93GXj1n1lhZtwO185ISsKnSfzqd/4MX4EAwKfXTwTawcWYLB/nmuspMwnlYWOo9IEfAoGcn4na3lIlfdn1Q10BbJzUVdAtTa3JCLqMLl1ZDQWdx6+KrqRV7dlbuiyduWVGOyfj3MnjsGKxTOKrgvd6uVVfb2YfFa3M7O10rqf2pfdDluxeAamnhPCxPFBBISCRDzlvH72+ADuveUyfOUv5mP5ohl48rnX04NW2Qq1EXP/tmXn/rx2CvMmtaLR+lj2IGJum9xfpelium5Ye9+Ypms9k3usV78r92/vO3ccVpVwj7JPV31ZbYSBBRjsn+8apxkora9fSp7M7N+fPT6IKZNDGOOTHfO7VmsMhdpDp+cHv6q4fn+/Wt0xpk5Jz6aSu4REFQJCAndcd0lZyzmqtSRltPNkPvUz0pXjaE8A+aSw+hLxFHrGB7J29w34rA5lNem6gTGq1SiBogCmWVa+IiJqB+XUtVkb4aQpKH7GmNdnAsCKxTMABRjsn2+dlGU0ufBqh9mvGzoQkAITxwcRDgWwYvGMssKjuYUSYCg1anWj5fvMQUS7Te6XcB1ErNf1eoZszOnHTSixPmOfrvqK3SyvlPZHqXmyk3/XzHRlX5c6PT8k4ilMdMaYrHKmFmNMHIBukOzCXoehV1bwV6vy6ORKqJUk4qms/FOrdq6dH3rsJbi1+RgiopbQiDrS6zMz5yOwjKZKFJuvix3cyg8lwPYkta7R7o9iBxGbDft8raWU36tV82QjsK9LmTo9P9hjTHb4tVqMMTEEBxERERERERERERHVBGdAExER1VkiqaOnJ9ToyyAiIiIiIiKqOQ5AExER1ZnfJ/GJOzaV9J4nv76oRldDREREREREVDsMwUFERERERERERERENdHOM6AlAAihjHZcU2v166+FBqfJVAC/BVDd7UCzNWXebbbrqUQ7fReg6O8zFR2ad6l5FZFXpqL2+RZog7zbytdeSw1Ml6lokzKXeWt0bZZGU9Emebden1GuZr22Fr6uqWiyvNusaVmJdvxOANsLmdr1Ny5Fp6dBrcYXFNM0y7ykpjcfwHONvghqS+8D8GYNz8+8S7XCvEutqNb5FmDepdpgmUutinmXWhXzLrUi5ltqVSXl3XYegA4AmAPgHQB6g6+F2kutn1Ay71KtMO9SK6rHDGjmXaoFlrnUqph3qVUx71IrYr6lVsUZ0ERERERERERERETUeNyEkIiIiIiIiIiIiIhqggPQRERERERERERERFQTHIAmIiIiIiIiIiIioprgADQRERERERERERER1QQHoImIiIiIiIiIiIioJjgATUREREREREREREQ1wQFoIiIiIiIiIiIiIqoJDkATERERERERERERUU1wAJqIiIiIiIiIiIiIaoID0ERERERERERERERUExyAJiIiIiIiIiIiIqKa4AA0EREREREREREREdUEB6CJiIiIiIiIiIiIqCY4AE1ERERERERERERENcEBaCIiIiIiIiIiIiKqCQ5AExEREREREREREVFNcACaiIiIiIiIiIiIiGqCA9BEREREREREREREVBMcgCYiIiIiIiIiIiKimmjnAWgVwNT0/xO1EuZdalXMu9SqmHepFTHfUqti3qVWxbxLrYj5lppCO2fA9wB44+jRUzAMs9HXUpZwuBvR6FCjL6OpNDpNenpCSh0+punybqPTvZra6bsAxX+fTs271dJu+aZc9U6HOuVboMXzLvOnu0amS7uUucxbo2u3NGqXvAs092/TrNfWytfVbHm3WdOyEu34nQC2FzK1629cik5Pg1qOL7TzDOiWp6qy0ZfQdJgmjdFO6d5O3wVov+/TrJjOFqZDc+Lv4o7pUjmm4eiYRs2rmX+bZr02Xlf1tOI1j6YdvxPQvt+rHEwLpkEtvz8HoImIiIiIiIiIiIioJjgATUREREREREREREQ1wQFoIiIiIiIiIiIiIqoJDkATERERERERERERUU1wAJqIiIiIiIiIiIiIakJt9AW0MykFUgB0w4QUClQAum40+rKIqA3Y5cuh6BAgBcsXqinmt/oKjetCMFBaEy00rguxk8M1uiIiqgT7BLXF9CU6g/cDlYNtfaoHDkDXiJQC0aEkVq/fgUPRYUwKd2FlXy/C3T7eyERUEZYvVE/Mb/UXDKj4xB2bSnrPk19fhFiNroeIyscytLYKpS9Rp2F5Q+VgvqF6YQiOGkkBzg0MAIeiw1i9fgdSjb0sImoDLF+onpjfiIjKxzK0tpi+RGfwfqByMN9QvXAAukZ0w3RuYNuh6DB0w2zQFRFRu2D5QvXE/EZEVD6WobXF9CU6g/cDlYP5huqFA9BFkFLAlAIpRYEpBaQ8k2w+v4QpZfpvEj6/tN4jFEwKd2WdZ1K4C1Iodb12Imo/LF+onqqd3/xBNave9AcZDYyISlOobd5svMpQVVTvO7RSelSbdx0l8M6R01n9M+c9HZxeVHv2+IBX/qsl9hGoHMw3VC+sbUdhx8MZWLMNKwa3YGDNNkSHkpBSwOeXOBJLYGDN8+m/PY8jsQR8fgkVwMq+XudGtuPosJtNRJUK+iUGcsqXgb5eBOvYwKXOUc36zB9UcfjESFa9efjECAehiahohdrmzcitDF3V14t4MlWV79Bq6VFtbuk70NeLtU/sxorBzVn9M4DpRbWVPT6Qn/9qzS8V1z6CX3Igkbyxb0n1ophm206rnwrgjaNHT8GoYOmAKQUG1mzLWpIwKdyFwf55ABQMrHne5W/zoeh6xTvQ9vSEcPgwtxTK1Og06ekJ1aP2nooq5N1qanS6V1M7fBdTSqx9YjcWzDkPoW4fYkNJbNm5HysWz4Si667v6dS8Wy3tkG8qYddnUBTANMveGduUsmC9matO+RZosrzb0xMqaxPCTs6jbhp537ZLmduMZV+htrnSgM2Sikmj3D6BlAq+9A23srD071Dt9GjFvJudvgJrn9iN7XsPOn/PrGeaJf80470FtPZ1NUPeLbWdU22mFFj7xB6XPsKMquTvZs0fler09kI5fct21q75vFjFfv9y8i6nHI2icDwcr78ZTudcQTqRdROdd+sSUS3ohoHtew9mda4A4KarDRbqVBN2fWY3SMqtz3TDKFhvEhGNplDbvFnLEbsMDaQHShNJr7Kw9O/QiulRqdwBfQBQdKseSaXbSJmy+mcdmF5UP41u5+iG6dFHuJj5mzyxb0n1wrVGoygUD0cK4RlzjIioVlj2UCuSUjDvElHFWjVWZWbohzffiVXtO7RqepRrtBAao9UznZZeVF+Nbucwf1M5Gp1vqXMwR42iUOxLn0/BnTfOyfrbnTfOgc/HAp6IaifgF65xugJ+FunUvFIAXnjpbfe8y3qTiIrUqvuspACsXr8Dh6LD2Lh1H25bOqsq36FV06NcmekIWLNLV6/fYYWJAuCX8IiBa/2909KL6qvR4wPM31QO9i2pXlgWjULXDYS7fRjsn5cXyzllmHhm2xu4a/lHIAWgG8APf7oPn77yQiYsEdXMUDyFX778e9x36zyYpglFUfDTF9/C5Ze8h2UPNS3dMPHwpr247o+1rLwb8Ask4qnRT0BEhMJt82aWGfohciCKR595BcsXzcDUc0IVfYdWTY9yjRZCI5nQMTHkx2D/fOimCako8EvrdaDz0ovqKz6iN3R8gPmbysG+JdUL81MGr00Dc2M5QwqYUgCGiSsunYLjp+IQioLYUBKxoYS1xEVv/CZGRNSepFDQHcwuvruDKsseKkqlG+SWwueXSOhWbDkpBD55+fvx2H9E8Nh/RABkbPxUk08nonbVivus+H0Cdy2fi6BfRWwoiY1b92Hdpj3O5neZ3yG3nPZLBQnd9Cy3WzE9SuVshmuYuGv5XGx49jVEDkQBZIQYSLeBkgkdCoBz0vsWJOuYIPWsY6n5SKEg1O3Lei3U7WMbnZoa+5ZULxyATrPjidlLuuzlKuFuX1ajwe2425fNwvqnXkY0Fk8v8VLq2tAhos4S9Et88P0TseqhbU45NNDXi6BfYmSYnRzyVmxdVw0+v8SRWAKDGZ810NcLAPjhz36TtSyUVSYRtTMpBY7HEnho4+6s/kM4FMgrA3PL6bnTJ2PZwmlZZWmtyu1m5dX/euTpVxCNxUuqS2pZD9azjqXmFPAJfPSS9+Kedb/IavsEfAKJOuQB5kEqB/uWVC8M6pI2WjyxQsc9sGEXllxxAQ5FhzG4fgcSfEpERDUUTxhORxSAU/bEE2wgUGHF1nXVkNDhmk8/Pv98rB1YgMH+eewQEVFHSAG4z6X/EPSpeWVgbjm9YM55eWVprcrtZuXV//qr62eXXJfUsh6sZx1LzWkkabq2fUaS9RkfYB6kcrBvSfXCGdBpufHEtClhLLniAmv5lBRnwnF4xB2zl9pkxiAjIqoF3TA84h8aLHs6SDnLfEeLnVlNBfOpabbtMnEiolyZZa/dxwh1+2DAhE+K7HAaOeV0qNtXt3K7WXnVXYZhQjVLq0uqXQ9m1sUCQDgUzDp/p/1WnU43DIRDQSxfNAOhbp8TbqdebfR6tvOofbBvSfXC/JQmhYJJ4S4cig5DmxLG9R+7CA8+vitv6YoJOMfZJoW7EBtKOv9mrBwiqiVVCtdySJUCSHFIrxOUu8Qys66z1areksI9n0ohAJ35lIg6h132hkNBzz6GXXbnltOxoWTdyu1mVc26q6rnKhAaxCs+NbU3v0/ihqsuwgMbdmXlCb9PwkjUfh5yPdt51D7Yt6R6YQiONBXAyr5eTAp3YckVF+DJ517H8kUzMNg/D8sXzcD3nn0VqZzjADiVysat+7LiWRIR1YoUCr5w7eyscugL1862GpfUEcpdYulWh1Wz3pLpTXpTigJVKnmfZe2TUKUPIyJqEXbZu2zhhc7gM+BedtvHzp0+2SpDz+qqabndCqpZd6kAVuWca1WZ5/IKDbJs4YUVXye1JsMwncFn4EyeMIzKBn8z21emFJDSfRin1u08ak/sW1K9sCxK03UD4W4fBvvnAQrQHTw/a3bCbUtnAQqgp84cpxsmVCEgJHDHdZdwp2MiqouUYUBVFdy6ZCaCfhXxRAqqqiBlGGAzoTOUu8Qys64rJXRHMdxmgt1984dx/+fmI6UbkELAL4FkgjMpiKiz2GVvd1AdtezWdQNnh/y4ZuG0rI0I773lMihQIAQ6rr9R7brL5xNZbSifr7w5WV518bt7Qlg7sIB9ww6U0t1DGaT08kMZlLLqrZbtPGpf7FtSvXAAOoOuWzeYKWXe7IQHH9+Fwf75Wcep1n/A0O1/M54lEdWBqeCrj/wyb5mUXUZR+6tkiWV2HVa9esttJtjdD7+Awf556ZjPOpKsJImoQ+m6AemxzDm37E7oZlZ5un3vQbzx9kkM9s+Dohsd2d+oVt2VAnD3wy+4tKHmlTzQ4l0XA4rOvQ46US1CYHitevPKs7Vq51EbY9+S6oQhOFwYHkHYK106Q0RUDSyjqBmXWBaalU1ERMWX3SxPa6eaaduMdTE1Vi3yBMsDqjX2LaleWD+6EB5PLoUA+AiRiBqNZRQ14xJLbnxDRFRYsWU3y9PaqWbaNmNdTI2VHdZTAUyz4jzB8oBqjX1LqpeGzIDWNO0uTdP2pv/31fRrV2qatlvTtH2apt2bceyHNE37paZpr2matk7TtJoPmvNpNhE1M5ZRBKSXWOoGVNO0lmQ3uMPLfElENLpiym6Wp7VT7bRttrqYGs/OE5PC3VXJEywPqNaYx6he6p6nNE27EsBCALMAmAD+XdO0awB8BcDlAN4C8LSmaX8SiUR+DOCfASyPRCIvaJr2bQA3A3ioFtcmpUAKwIhhYkLIj/v75yNlGHyaTURNxd6gaLB/PnTThFQUbu7WAew6qllnWXEmGBHR6Iopy1me1k6j09b+/Q9FhwAp+Lu2oWr/xo3Os9T+2LekemnEQ413ANwRiUQSAKBp2isALgSwLxKJvJF+7Z8B/KmmaS8D6IpEIi+k37sewD2owQB0od1lASv4v64ooxb4zT5AQEStT0qBWDyFg8eGnJ2KJ5/VjTE+yfKmTZWyA3ojCamkl+qZABQICehsuxIRAQB8fomjsURRZXkpG4mx/5FttAHA3LSFFDClqHn6tUpdTuWTUuD4UBL3ZfzGq/p6MaHC35jlAdUS+5ZUL3UfgI5EInvtf2uadgGATwP4BqyBads7AN4D4FyP16vOa3fZr35+Po4X2VBko4KI6kIKRI8N4aGNu52y5vZlszCmZyzAsqYtlboDeiP4/BJHYgkMZtSBA329mBjycwYFEXU8KQWGk0bVy3L2P7KVmh71TL9WqMupMjoUZ/AZsH7j+9bvwP398+vy+SwPqCzsW1KdNCysi6Zp0wE8DeCvYNXHF2b8WQFgwIpRbbq8XrSzzx5b1HGHokMeu8u6NxT+v9s/ip6zxmQdH43FsXr9tqKOLVZPT6is97WzTkmTYvNuvbRTurf6dzl0bAgPbNiVVdY8sGEXBvvnY1ITfLdmy7vV0sh841VHQVHqfl1en3c4OuQMPgPW9Q2u34H7Pze/Ze65Vs+7rZLO9dQJaVLrfNsJaVipYtIoGosjeuR01cvyWvQ/6qUWebfU9Khn+jVTXV5IM11Lpma6Lq+8+/uj7ve4bpp4Vx2uvx75uZl+h2pq1++VySvfNnvfshE6IT8UUqvv35ABaE3T5gHYCOB/RyKRDZqmXQ7gnIxD3gXgbQC/9Xi9aEePnoJhFLE7rBSuO3/quuFaicRHUjgcj2W9nlKUoo8tRk9PCIcPl/6+dtboNKlnQVR03q2DRqd7NbXDd9E9yhrdMD2/W6fm3WppeL7xqKNgev/mtVAoHbzqwJRulH2N9W78NUveLfd7t3rZVm2NvG/bpcxteNnXAopNo5Si4MSpRNXL8lr0P+qlFnm31PSodvoV1CR1eSHNes8Xc13NkHcVj99YUepTR9c6Pzdr/qhUp7cXyulbtrN2zefFKvb7l5N3G7EJ4XsBPAFgaSQS2Zp+ebv1J+0DAN4AcC2Af4pEIvs1TYtrmjYvEolsA3A9gB9X61oy4yOpUHD3zR/G3Q+/kLVcRRXulYgUihUzLPN8Qin6WCKicqkejVtVKtZ6Emo7fqng3lsuQzQ2ghOnEtiycz+uWTjNiuvX6ItLk571pWAgaCLqeFIo2LJzP25bOgsPPr4ru7+BwmV5oZiu7H9kKzU96pl+KoCVfb154RFqUZczDnBjqFLBl2+8FCdPJZxYuuPG+qFKBUYdmkIsD6gc7FtSvTRiBvQXAQQB/J2mafZr3wTQB2tWdBDAMwD+Lf236wA8rGnaOAC/AvBgNS7CLT7Sqr5efPXz85FIGk5FDZhFNxTq2aggos7lVxUM9PXmxdr1qwoSbCS0HSmF66ZVZzdZbGW/hHu+lECyeS6TiKghVADXLJyG7z37KpYvmoHxY/0IhwLo8omCZfloMV3Z/8hWanrUM/103UC424fB/nmAogCmWZOBYcYBbhwJQCoiK5buyr5eSJQYR7RMLA+oHOxbUr0optm2T8KmAnjDa5mBKQUG1mzLe8oz2D8PisvmgplPkP1SQUI3XZ8o+/wSOhSkdAOGYT0FlaZZVmXf6VP/3TQ6TXp6QvXYI2QqCuTdRmh0uldTO3wXUwq88NI76J1+DkzThKIo2LH3HXz44nPyyi9bp+bdamlkvimlvqqmzLovGJBIJs2MehCuAyY+v0RCB3TDgBTC87hi1SnfAk2Wd3t6QvjEHZtKes+TX1/U8mVbtTV4SW1blLntUGdWQ6HZpMWmkZTC6iMYBoRQ4PcJpJImUoZRcIZqMXVANWe7NmveLeU72scWO8jrdm7AmvhnGCaEEIBiAiaqNmBcy3urknZDs97zRYbgaHjerVWbrZz8X4vZ79XOH80yU7/T2wvl9C3bWbOWg/VSQgiOkvNuwzYhbDTdMD3j3OQmiq4bUGAlloTiOhMt3O0DAMTiKURjI04Qd3tm9QQ+cSaiKlGlgovedzZWPbTNKWfuvHEOVClYzrShUuqrasmcPTXzAxPxsXnvw/3f3Zm18gyfAAAgAElEQVQ1K2KiywzsZEJ36kvoOmc+E1HLq8ZsUrdzDPT1YsOzr2L73oMFz1lMHZDZV4Futt1Mx1J/Azs97E70aOmRm35w+bzbls7Ck8+9jmsWTmv6mcSNaDeQxfBIe8OwZkeXo9z83+zlAWfqNw/2LaleRKMvoFHs+EiZnPhIBaQAp5AEzuwsm0r/7aDLDqL3pf9ORFQNKR3OYCBglTP3f3cnUozt1pbKra8qkVnXffJ/XpCX3wbX70ATRf9oaqFxXejpCZX0PyJqHoXa/pWcY3D9DiyYc96o52xEHdBsqvEbVPp5Dz6+CwvmnFfTz60W5pnGEen9MDJNCndBVJD29c7/9dKu36sVsW9J9dKxD0HLjY9U6IkyAAT9Kp84E1FN6YbBcqaDNCKeX2ZdJwU88pvB/FaEYEAtK5wGETWHaswm9TpHKL2CstA5GdO1/jN6C/1erdDeYp5pIMXM22j0tqWzrBAuZWrXGe3t+r1aEfuWVC8dm58yN4EoJeZQwZ1lAcQTKe48S0Q1JYX7TsVWOdPAC6OaKLe+qkRmXacb8MhvAtCZ4YiovRVs+xfZtvc6R2woOeo5G1EHNJtq/AbV+LzYULIl+nXMMw1kAk8+9zqWL5qBULcPsaEknnzudaxYPKPsU9Y7/9dLu36vVsS+JdVLx4bgANLxkXQDqmlC0Y2iKmX7ibK9tCbzibIKYPJZ3bh92aysv69K/52IqBoCPmun4sxyZqCvFwEfl1a2q3Lqq0pk1nU//Ok+3HnjnLz85i83mCERUQsp1Pav5BwDfb3YsnN/Ueesdx3QbKrxG1T6ebctnYUtO/fX9HOrqdPzTKOoAK5ZOA3rNu3BwJptWLdpD65ZOK2iPFPv/F8v7fq9WhH7llQvimm27dOlqajRzuCFdmuVUsCUClK6CcMAVKFAwiyr0u/03TfdNDpNmmGX2kZodLpXU7t8F39QxUjSdMqhgE9BIu4dNa1T8261tEu+KUVmXRcMSCSTJnTThFQU+CXyNiCshTrlW6CGebenJ1RWCI5y3tNpeXQ0nb6rfTV0YtnnplDbv9g0yj2HXypI6GbTzVBt1rxb6DfwUkn+tT/PMGDF71VMwETVfqdmvbda+bqaJe/aeQeKAphmVfJMOfm/FqqdP9r1e5X42U2Rb0vtW7azZi0H66XY719O3u3oGdCFWAPJAilFAVQJ4VeRUhSYUkJIxfOJsq4bMBI6hG4gIAATJhKGCVNKmKp1TimZ7ERUmTObmZg5/02Uzx9UYUrp1GP+oPf8Erv+G0k3UANCgZk0AJjpcFMmDC6NJKI2k9n2z22vlzObNPd8AKDoBgLp+no4aZ0jkB50SQGun13ptbeLas7olVJA+CWMrL6edNLP55dZg2JSAjrrPSqSnVcnhbsL5tVS7ttS8r/PL7PafL46L1nrhPKoHbFvSfXAFQ4upBSIDiWzNm64fdksPPL0K4jG4hjo68XEkL/g7C+3c9y2dBaefO51XLNwGsLdvqaY5UBErSfY7cPBaByDGeXLQF8vJoeDiGfEkyQCrMHnwydG8vJLz/hA3swGt7rr7ps/jGTSwH05mxmxHiOiduFW9lVSznmd7+yQH0djiazXV/X1wucTuPvhF8r67Gpfe7uTUuB0Ukf02Age2HBmo7gvXDsb33lyLyaE/Fi2cFpWnZnZD2TaUjXU6r71+SWOxBJ5bb7Rxi6qpZTvxbKrebBvSfXCx1EuUoBTEALWDqAPbNiFJVdcgEPRYQyu34HRym+3czz4+C4smHMeVq/fgc5czEBE1TA8YjgNBABOuTQ8wsYa5RtJmq75ZSSZP5vLre46eGzIGXy2X2M9RkTtxK3sq6Sc8zpfQs9//b71O3Dw2FDZn13ta293KQAHjw05g8+AlWZ/9y8vYskVF2DBnPPy6szMfiDTlqqhVvdtQodrm68OY88ASvteLLuaB/uWVC8cgHahG2bWDqCAdROGun3Ov3Wj8M1Y6BzW+7mMi4jKoxuGa/kyWrlEnamU/OJWdwX9qsf7WY8RUXvwareXW855n8+9PA76VZdji/vsal97u9MN07NeC3X7nL6a29/sfzNtqVK1um8b3Uco5Xux7Goejc431DkqGoDWNG2wWhfSTKRQnB1AbZPCXYillx9MCndBinRcI1XCF/TlxVgqdA7r/YypQ0TlkUK4li9S8Jki5SuUX3Jj87nVXfFEyuP9rMeIqD14tdut+L/FxTPNPK5Quev2ejyRcjm2uDK20LXnXhfjsVrp5VWvxYaSTl/N7W/2v51+INOTyiSFgrnTJ2NlXy8G++dhZV8v5k6f7HnfF10ONbiPMFp5VO6xVFuNzjfUOSrNUR+vylU0GRXAyr5e5ya0Y39t3LrPiYez9ondWDG4BQP/+DwOHx/G9ze/hoE1z+NILAGfX7qe47als7Bl536s7Otl8G0iKtuYgMBATvky0NeLMQE2Eihf0O+eX7b+8gAG1mxDdCjpdGTc6q4p54Rc3x+s86Y2RES14lb2rezrhV8qiA4lMbBmm9XuzykzbXYsU/u4tU/szis3V/b1IuBTXMvTCWMDeccW21fwunbV5bq8rr+T+KWCSWd14/Zls7LS7It/Nhsbt+7D4ejpvN/Isx/I9KQyBXwCyxZOw7pNezCwZhvWbdqDZQunIeDLz0ul3Mdebb6gvz55tFB5VMmxVFvsW1K9KKZZ/hIHTdM2AegC8DyAU/brkUjk7yq/tIpNBfDG0aOnYJSxjENK4ex+rEoBIRQkkjqkEFj7xG5s33vQOXZSuAt3Lf8IPve1rZgU7sJg/3wouu6cwzDSu4gqJmBahW0xgfV7ekI4fDhW8rW3s0anSU9PqB6PZKeigrxbC41O92pqh+9iSolX3zyKaVPPhm4YkEI4/63o7kHeOjXvVksr5xtTCrzw0jvonX4OTNOEoij46Ytv4bH/iABAut6aByVdL2XWf1IogKJg7Q93Y8Gc8xDq9iE2lMSWnfuxYvFMz/xWLXXKt0AN825PTwifuGNTSe958uuLynpPq+bRWmnkfdsuZW4rl32lyi37VFgxSgfWbMtampxbZvb0hHDo2Om84+ZOn4wVi2em6+kz51v7xJ688vTqj56P08MpjB/rx8TxXVBhlrQJl9u167oBU4pRrz9Xu+RdwD3/mlJg7RN7cPVHz8fZ47tgGCaOx0YwcUIXjp4YxsQJXVi36cxvlNIN+H0C4ZA1M9OtH1goPUu5tmbQytfVbHm30DWbUmJgzfMu9+b8vLZVKfexKSWeev51XNk7FVIAugFs3vEmPj7//Kq02Yr5HbzKo0qPraVOby+U07dsZ81aDtZLsd+/nLxb6QOmY+n/f1/Ga20x6qDrBhSkEyilw0j/O2UYOB5LYGVfr9Nw3Lh1H+wHkIeiw4ACGFJANwGhKBDSBEwTqmmdt/NuYSKqJt0w8PaR07jwvLMAAIZp4u0jp3HBlDBnDVAe3TDxX7veRk94DKZMHovDx4fwwfefjcH+eU4dphumZ97Rdavey3Q8loBuGMxvRNSy8h62AVD0dLmmm9AB6IriGaM0s/zLjGWqTQljyRUXINTtgwkTAaE47X9dUbB978GsAUwAWHz5B7B6/Q4AwNqBBVBMM73kXkFKN2E4E2KARNLIG6jJ6rekrz33ugpdf7uxf9tD0SFAiuy0Msys30CbEsaf/ck0ACbC44JQVQXHYwnn97CtHVgA3TBd+4Htnp5UmkL5z1Yo5m5uXirlPtYNAy+/EcVF75vo5NGX34jiTy6rrM1WzHdyrsGjPKLmxb4l1UtF+SkSiXwGADRNmxCJRI5X55Kam98nccNVFzk7J9vLskQ6Ps7c6ZMRO51wdnS1Q288+dzruGbhNIS7fQ15qkdE7aM7qOLSD74Lqx7a5pQzA3296A6qSAwnG3151GT8PuFab61/6mVEY3HcvmwW/D4BI6E7yzwz67Cvfv4PXN/v90kYCe5VTkStx62sW9nXm9dOt2OU5s48lEIBdDPvuHAoiOs/dhEefHyX63m9zpcdX1iBhILTSR3RYyNZZe8Xrp2N7zy5F9FY3PV6875nkdffTkb7bTPTRJsSRt/HP4i//96vsuq3FZ+6GGt/8BIiB6IAzsRClRIe9aFVhxIVW7aoUrjem6oUQCo7L5VyH3uNVVTSZiv2OzXLeal07FtSvVS6CeGFmqa9DGCvpmnnapr2iqZp06p0bU0jM+i/bphOgQ5YTx8f2LAL0Vgck8JduOnqi51C1P77g4/vwoI552H1+h1gV52IKjWSNDCYU84Mrt+BkSQba5TP0OFaby254gLn30a6r5MC8uqwZMpwfX+7hVkhos7hVta5tdOLjVFqH7ds4YXO4LPbeUfbZ8Y+dwrAwWNDeWXv3/3Li07ZbZ+30OZknRhjdbTfNjNNllxxgTP4bB/7wIZdOHkqgWULLwRgTS6695bLYJgmdI/61ODYM6UVW7ZIoeTFIb992SzXDfhKuY8Nj7GKStpsxX6nZjkvlY59S6qXStsf/wDgdgBfjUQib2ua9g0AawF8tOIraxK5T+a+8hfzXZfAhEMBDPbPx8mhhOvfQ92+jljyRkS1p+sey/Z0li+UL+WxzDPU7XP+nUov+XRb5um19DPF+oyIWlSxS9p13UC424fB/nkFY5Tax3UH1YLnzT2fKgSEBO647pKsc+uKgqDf/VyZZTcUIHraewZhsdffTkb7bXPTxO3YoF/FWeODWLfqSpwaSuKvv/nzgv3AFENSUVqxZctIUscjT7+C5YtmOKEyHnn6Fdxx3SX5YTVKuI8L9RHK3Tq6VqF8OjVEUDNi35LqpdJtLc+ORCI/sf8jEomsATCuwnM2ldwncydOJZynjzZ7WRYADMdTrn+PDSXPLKsrMFOBiGrDvu8ORYda/r6TQniUQ/Xar41yNXO5bi/dzOS25NvrWK/3S4X5jYhaR2Y5LYXA3OmTs/4+KdwFVeSX5bpuWLGhTRNKelDXTWZ4h9zzSiGgp88JwDkfdB1GQs87txQK4gnvPoX9b5jKqDMIi73+duH9G5yps+w08To2nkghlTKR0s2C/UBtShh3LZ8LAE1X91NjFJP/7OOisThWr9+BgTXbsHr9DkRj8Yrb8sKjjyAqOG+x36nW523mtnars+vElX29GOyfh5V9vZg7fTL7llR1ld61pqZpQaQ3HtQ07V1A2Q/XmlLuk7mNW/fhtqXZy2UG+nqx9ZcHMLDmeYwP+fGFa2dn/f22pbOwZed+rOzrhV8qiA4lMbBmG1YMbsHAmm2IDiVZgBLVkL2SYWDNNtx0709a/r7z+wUGcpbiDfT1wu9vze/T6jLzVzOW60G/zMsvbku+AfdlnkIgr967bekscPyZiFpFfjn9PJYtnOYMQk8Kd2FVXy/iyVRFZblbGXrnjXOw9onduLmEc6oAJp/VnbdE/wvXzs4quw3Pjcw6N0RSKeEKVACrPpNfP046qxuPPLMX0ZNxz36gNiWMG666CA9t3N2UdT81hl8q7m10qZR1HFBiO1MxXdtsUMovE2oVyqeU8zZ7W7vVBQICyxZOw7pNezCwZhvWbdqDZQunIRBg+lJ1KaZZfmGkadpNAG4AcD6ARwFcA+ArkUjkoepcXkWmAnjj6NFTFcU8MqXAwJptWY2PudMn44aPTcfJ0yOIDSWxZed+J8bzpHAX7r1lHgzThGmaUKDA5xOAaTox3XLPNynchcH+eVByZiT09IRw+HCs7GtvR41Ok56eUD2GXKaiCnm3mhqd7pVyu4+97rtWYEqJF156G73Tz7HKGUXBjr3v4MMXnwtFdw9E2Kl5t1oK3QPNnr9MKbH2id1YMOc8hLp9SOkG/D6BcKgLUiBvGae907m9zFOBgm9lvN+u91YsnlHz71enfAvUMO/29ITwiTs2lfSeJ7++qKz3tHI5XQuNrLvapcxt9frf5l1Oz4duGOkVigq+9I3nSy7Lc9PIKkMVHDkxjLFdfjzyzF5s33uwpHPa5zGlgpRuwjAAVSoQAkgkDWcJfin9imK1Q9616zEoitMH85r5LfwSbx8ewoRQAEIo8PsEvvWD3di+9yBW9vVi3aY9ef3AFYtnwoSJlRWkfbPeW618Xc2Qd00psPaJPaO2mYo9zj622Pu8lPOWopR7qpzzjhZapNZt7U5vL5hSYmCNW/0337Nv2c6atRysl2K/fzl5t6IHV5FI5Nuapu0DcBUAH4CbM0NytAMVwKq+XtyXEVtt8eUfwIOP73J2RgaAxZd/AIA16+DoiWEMrNnm/G3twAKopgkdgK4ojHVEVGdtF2NMMfH+d0/I2qn4L6+5pKLZDVS+ZsxfmQ16GAaOxxLO34ZHdPzzj1/FHdddAkW36qZMum5AQbqBoJuQUuCahdPyYoyqQN57iYiakXc5bSAgFKRgDewuXzQDG7fuc9r45ZTldgznL//D8xjsn5c1+Jx7zkKDL7puALq1XFUAQAowcKZs1gGoUmBlX29bls/FDky5sesxuxNdKC0SSQMrHzrTb/vml69wfjN7xrO9seSkcBeuWTgNKkyMeOQpw2iz5cBUEt0ws9pcAHA8lsiPL2+Y2L73YF75cNPVF+fHgDZMhEPBrHjRG7fucy2bVKAmbbZS7qlyzptZrrke14Rt7Xaie62mMZm+VF0V5SdN074HYCOAeyKRyFB1Lqn5+HwCty6ZiaBfRTyRylvmnhuPLZXROHLiGOnWwJAd6yj36VLmMURUXe1230kh4Pfnl0tSCBho/IzbTtNs+St389y1K6/EDVdd5OyKbi8x9vskjMToe41nbn5T7ZkvRET14FVOq0IgOpTIGqy5beksPPrMK4gciJZdltufZ+8B41Y/SChZZXXuBoLFaNdNBnPrsXLSpujPyskbugHnvyMHonj0mVdw65KZeHdPKGvVkJTC9bc1YTqxw6nz+H3Co80lYCTODK+W0nYs9pxAG5cJTdbWbjeqR3mmCgXsWlI1VRrU5SkAfwrgN5qmbdQ07VpN00JVuK6mkQJw98Mv4J512zGwZhvuWbcd3//Ja7h92SwM9s/DXcvn4ss3XurEY7MrA8A9jlGtYigRkbd2u+8Mw8T3f/IakimrwZVMWf/dbmEvWkWz5a/czXOjJ+P40X+9juWLZmCwfx6WL5qBH/3X6zBLyC/2hk2Twt0dsYkVEbUXr3JaSORt4vfg47uw5IoLKirL7c/bsnN/XjzWu2/+MBQoGDFMRGNxhENB57NzNxAsRjtuMphbj5WbNsXIzRubd7yZFZs3GotjQiiILp/iLPU3pQCU/Dx129JZ+PaPXqrJdVJrMHQ4A8WAlXcf2LALRs7U3lLajsWes501W1u73Uih5O1j9oVrZ3MTQqq6SkNwPAbgMU3TfLAGogcBfBtAV8E3tpDc5R7alDA+8Qfn4+6Hf+E8gbzzxjm45VMzcPh4HI88/QruuG421g4scH3i2K5PJYmaWbvN4DRh4hN/cH7WktDbls6CCRNsJtRfs5XrufWWXxWu+cVgfiGiDuFVTo8k3ZcdTz0nhMH+eWWX5fbnrVg8A1CAwf75MAwTPp+CE7EE7n74edcZ11xSbqnncnu3vBHwiYz44AJ+CSQTet7M7LnTJ+OeFZfh1HAC0ZMjzu/oFkaBOkPKI5RByjCyQ3CU0HYs9pxAfVcP1FOztbXbTcowoKpK1upaVVWQMgz2FaiqKpoBrWna5Zqm3QvgeQBfAfBTADdV4bqahr3cw7bkigucTjxgFf73f3cnDh+PY/X6HYjG4tbyrJxZCNZGIgIpxYozpyL/GCKqnbaawWkqePK57BmtTz73OmCyidAozTQDLbfe8vtU5hci6nhu5XRueQmcWdZdaVluf56SMqDoOqRpQNdN/Muzr+aVx0uuuCDjswWkrHSRamsr9LvUQm7eSMRTUHQ9/d86kukwB7kzs7fvPYi71v4c0ZMjWL1+R3bYFrfvldEfjMbiHf87t6NS8m6xbcdSzlnP1QP1VnR6ZdxnpmR5WhRTwb9uzl5d+6+bX2Nfgaqu0oez/wngHQD3Ang4Eom0Q9mWxV7uYRfk48f6PWZKjMOaL10BVVXygvy365NIImoMRcB1Rqsi0Pq7DlHFcustRbjPmGd+IaJOl1teVrphl5QCOqxZY0IoUKVibfZqt/cV9/p7TFB1/r32id24ZuG0ju4nVPt3qRavmdnjx/oBWIOCA3298EsFyZwLZX+wM5SSd4vdaNMvFQz09WIw45xe+azTN+vjfVYe9i2pXioth94N4H8B+GMAf6Vp2h4Az0YikX+s+MqaRP5yD/cA7QBw8NhpnDNxLHRVydp8wutJ5GD/PC5pIKKSmSbyVmI8+PguDH5uPssUyq+3pMCDj7/A/EJElKOay7qlFDg+lMR9GeEZPvPxi6EIwCclJEykTMW1/l6dng1th3B44+2THd1PaNbl9gGfxF3L5yLoVxEbSmLj1n2IxuIY2+XHYP88xIaS2PDsq/js4pl572V/sDMUG/avlIHSpA5sSK+cCHX7CuazZtisr9iB9VrgfVYe9i2pXiqNAf0OgO9omvYUgKsAfAnARwGMOgCtado4AD8H8PFIJPKmpmnfATAfwOn0IfdEIpEfapp2JYC/gxVX+vFIJPLXlVxzOXTdin2jAjAVE7ctnZX1dOhLN1yK0/EkHtq427UC6fQnkURUXYZHmWIYJmSDromaS2a95VUHMb8QEWWXl9DNsid7pQBn8NneM+b/rP15Vt9gTJfPtTyOxqwQDpmvdXo/oVq/S7VIKXD81EhWf+/2ZbMQ8Es8+PguRA5EnWNvujr/t2N/sHPYebenJ4TDh2OuebeUgdKUYWD73oPYvvdg1utu+azRqwcaPQOZ91l52LekeqnoPtQ07W8B/AmsmdBPAbgDwOYi3jcXwMMALsx4+VIAH00PatvHdQH4JwCXA3gLwNOapv1JJBL5cSXXXQzPJ3cmoOv6mY0ppIBPFfjiA/+VV4F87fN/AFNRIEzT9UmkKgVMmICpwLCX6qHxT/eJqLkJRcHc6ZOxYM55zkyILTv3Qyh8Rt2ugt0+DI8YzoZIXQEBAeB0xmtjAgJDQ8m89wolezaMNiWMZQsvhGkCphRF1Tv+oIqRpIl3jpyClBIBn4JEvO2ibhFRG7Lb9IZplYdQrLa3vbGczeeXSOiAbhjw+yRM00QyZUIIQJUCqrA2LZRSgZ4+TgqBVMpwPuMvr7kEsaEkxo/14+uPvZjXN7j/c/Nd+wRnhYJ4eOWVODWcxOHoMLbs3O/MWCxlNmEjZx42UqnfO/O3lkIg6BeIJwzX/1algBAKkrqB+76zA+FQEMsXzUB4XABjgj4EfAK3LZ2FRDKFw8fjSCZTkEJBylCcuvn0iAEB9/6gFAIpw+io36vdFdNm0g3TyUt2W37j1n2uA6VCKPjk5e/Hlb1TIQWgG8DmHW9CuIRH0HUD7woHszbR9GofZl6rfaxX+86+xw5Fh4ACbccUgJ/96i3ctfwjWdf68fnvd51JW+0ySwr3PlI9Z4C3IvYtqV4qfRAUAvCXALZFIpFS7uibAXwOwKMAoGlaN4ApAP5J07R3A/ghgHsA9ALYF4lE3kgf988A/hRATQegCz258wUkxo0NYmDNmd2rB/p6MfMDE7F551vOOcKhoDObIRwK4vZls/DAhl1ZT8xPjyRx6nQSf///s/fuYVLUV/7/q6r6NpdmpoGZUVTwhsVFMIOZQWE0+QKyG2+Y5bcLmiiTFVFGI5tkNzrgrpKfMmiyuZgEI2CCmI2YZ0k0Bjc/jHw3EYyg0QRksXUNkcQL1wZmmOlbVf3+6K6iq6tqpnt6Lt0z9X4enofuqfrUp6vO53zOOXXO+zz9hstR5MKFi5wR8IssnDvBwgUX8IvEOgc7R8hFXyNQ7uVgJGp63o8sayJyIm6RgTNDAYuT4fEI3LuogdVPvkYoGOCWayaa9qOe9h1fwMPhEzHLtWqq/G4Q2oULF0UNO5v+7gX1PP/yeyycO4HRQR+JuILXJ3GkPaVTQ8EAX7huMt/8ye9Ndnt10M8fwoeYeN4oVj/5mkkfVpR5uO+xV05Tb1x3sSWbLBQM0BVLWnyCf/78pUQ6ojyy8XXTmH6vSByBqKLy4ZEONm19h0h71FFnD3bm4WChu99th8xnnXm/N219m517DzJ9cp3Fxlq2sB6fVyIUDHDz1RMtPKnPv/weC66aQO3IAGiC4SdmjmXnD7Y2N7L22d3s3Htw2DyvoY5cbSafV7TYYyk5E1HjZlu+zC9x5bRzWLn+d6Yxy/wSsU6zrJSVe/koy2ZsbW7kjFCAriz7MNe5SpLIqYTCwWOdBHweovEkdSPLqfBKFln1SAJX1J9tmuu9ixrwZNCTZo7b1zrLJwm2PpIdX7aL03B9SxcDhUJbgv4zMAPYJsvydlmW/02W5R6D2uFweHE4HH4546szgG3APwKXAVcAtwJjSDU51PERcHaBc+4R3XWP7YqpxsLU/9a2YRd/93/Gm8ZYOPciY4zwgQgbt+xj6fyprG2dw+J5U9i4ZR9HIl1G8Dn7Oi5cuHDhhGjcXg9F467DMhRht+8oCWxl4FTMJhtF0XjmxTCL503hSzdNM5wd/bye9p1YQrO9VizhZpK4cOGiuGFn0z/6zJvMbhhH24Zd6HGeuHJap86fNd4IPuvnfGfTmxw61knj5DON4LP+t7YNu0gmT5cvz24Yx0dHOoweMToWzr2IletfZeOWfSyeN4W2lpksnT+V6sqAEXzWx9y09W2OnIhx75rt3L76JR7bvJubr55IKBhw1Nnd+S9DGfn+7sxnrR/ftmEXsxvGARiykf38y/1eFs69yJYndXbDOFY/uQufx2OaS+ZY2f7g6jubjKB3LvN2URrI1WZSFSz22Hc2vYlqE+vLx+7vdIhVdNrYh7nOVZMEIu0p+pnWNTt4bPNuIu0xNMmaHZtUsOjI1U++RtIm+7g/dFZcsf9NcTf7uVu4vqWLgUKhGdAPATErW10AACAASURBVJ8AvkMqmL0E+DqprOicEQ6H/wR8Vv8sy/J3gVuA/wQytYUA5LUKRo2qzOdwIFVaYseBgyCgKKrt3zzS6RLn6ZPrOKum0nRc+ECElet3srZ1DgDN106iqtJvz7WjQW163jU1wbznP9QxXO5Jb2S3PzGU7nup/5aPjnTY85spGmcWwW8rNtntKwyW3OjPe07DOXz20+OR0q+Or2ocy7gzqyylm9ky8PHRUwZ3YFvLTMf9zen3OcqbzbVKHaUuu6Wu2/oDw+Ge9LfclvI91G16eWyI+bPGG/qypjqQ0mNaSo99dOSUoeeC5fY8zdWVfjTNnifTI4k8ds8sBEHA6xX58FAHK5dczsdHTxmZy2eOTvkGepBFx9rW2ZYxZzeMsw2cL543JXWujc7uzn8p1mfYF7Lb3e8Gq/xmPuvM44PpjOlguddEjaBqWooHVRIs/p1+7tm1lUy9cDQ+r2jQsGze9q5FljL9QQ0snL7F8rwG+/pOKKZ5OclurjaTk9xqAtRaZDZ3uz8fmy2fudoFy9vubMp9rjbXz1dnqarGiVMxEkkVr0ekqsKPKApZ5/deDxaTfPUX8pbbIvEtBwPDQR66Q3/9/kID0J8BPhkOhxMAsixvAf5IngFoWZanABeFw+HN6a8EIAH8FTgz49AzgA/zGfvo0Q5UNc83XpJoy9GFpuFx+JsGLJ43hZrqAIIg8MHhDvsx0Fj/3B4ORbq4f/F022P+eqidU10JzhtTxdGjHfnNfYhDb+YwmNcfKPRKdvsJg33f+xJD4bd4PJIDr7zg+NuGq+z2FQZTbiRJ4rOfOt9S0phZMpxZupk9TzFDXto7E477m9PvkyR7eZNEZ3nrKwy08dcfsjuQv6HUdVtfYzDX7VDRuSW/Z0oi0yfXcd0VF5hoE+65pYHpk+uQhJQey9Rzdnpy+uQ6NHC07z843M7K9TsNyoXv/vQPxrXuXdRIIpkEDdtzE0krN3BVpc8xSOqos7vxX/J5hiUnu938brDqRac9rT1NT6BqmkGNoNNW6c/TyXc7eOwUV888j3XP7jH25LsX1JNUVMf90+vpm+fV1yjWNZ/LvIpBdnO2mfJYr/nYYfn4CLmOqwiiY3CykOsLDscK2K3bHOk6eqkHh7u90BvfciijWPXgQCHX398b2S2UgkPUg88A4XA4RipwnC8E4NuyLIdkWfaSyqT+ObATkGVZvlCWZQm4iX7mf4bT3WP10rnM7rGSJLBsYb3pb8sW1nPyVJxVG3ZxvCPG01vfxiOJrFxyOfcvno48NmSM8cQv3jIW9qat71jGuntBPZu2vsOqDbs4cSrW3z/VhQsXJQhJFPjyTZeadMeXb7o01WDDxZBDmV/kmqbzbcu+9ZJhPRtFd7gzIYkC9yz6JPcvnk5NdYB7FzXY7m9O8HsFWrP2xBQ/qStvLly4KG54gFuvv9hCm/Dwxte49fqL8Ump43wShp7bvO1dyx77hWsvZvWTr7Fp6zvcvaDesv/6vBJtLTO55erJbNr6dlb5+S6qKgL85zZ7u//n//2uZcxQ0G+h8KgNlRGNJx11dnf+y1BGvr8781nrx7c2N/LSa+8DICAY2Z7zZ403ZX5257utfvI10578/MvvURsq52u3zzD5g63NjQR8qezN4fi8hjpytZnykduAT7QdM+CzhnKcYhWSDV1GznNNV3pnQg9OWq4vOlzfxkcRHY7NzmqG3Ok6PMCKrN+0wl1XPcL1LV0MFApdi3+QZflbwPdIUWXcBezOd5BwOLxbluU2YAfgBTaHw+GnAWRZbgY2AwHgBVK0HP0KRVEJlXtpa5lp6ciaVDWDu00v49u4ZR/N106iNlTG2XVBS5bFvYsaCY1Ile1lllrpXGCrWmZyOJ1x8dQL+wgfiACQSKq23WJduHAxvJFUVTwegaXzpxrNQDwegaSquTpjCCLamUAVhG5LhvXP8aRq2dgVVSUeV3ls826DJuprt89AFAUkgR47jsejSWqq/OmO6qk90alLugsXLlwUExRFRXDQn4IgkEiTQCfiCqODvrSeU/F5JR6+q4lEUkMUU+/2dPqMp1447QfUjiwn0h7lmxvNTemOt8cNe/5QpAtJEviHOTJ+r8jqliaSqsqfP2o37P6/HOxg8bwpnHvmCCQx1UhreXOjJduvOuhDUDRbnd2d/zKUke/vzn7WkigS8IksuWEqt16vAqflxY5CozvfTd+T5bEhrrviAv718Veynp8fryQQ60owojIwLJ/XUEeuNlM+chvrSlBbHbDIbKzLmvcXiyu2sYqvfG6axT40zzU1rt1cNTS+dOM0o3dVbaiML904DQ2r3xFL5HH9PI5VVHv6I0XVLMd6vaLJR/J6C825HPpwfUsXA4VCA9B3At8FXiGVTf3/AV/M9eRwOHxuxv/XAGtsjnkJuKTAeeYNRUkFfz0AiobeD0ASRSLtURN3W22ojJpQGW0tTSSSqiXLYvWTu2hraUIQUiV8sxvGGUr2pdfeR0AwNSPUx/RIIkrS7TrqwoWLLGiCqWERpHRGW0vTIE7KRX9CsikpnD65jtHVZfzgnlkoKvx6159TmQrZjVY08x6zc+9B9n94MrUvKQq57DLxaBIBODNdkhV3tyYXLlyUCERRsOjP2lAZkmB2qxNxxbD9BUVN60YNAQGP5/QY4QMRVm3YRW2ojIeWzrQ0EHz0mTf54j98gn99/HfGtURBQPKkgjOSKOARRYOSD1KBzfXP7aGtZSaCopJQYFRWkNQnYQTMneDkvwx19PS7JUkkCUagT1U0BCX9wlZRiHWdfvZaRl8fOzqWSHsUTcPiu02fXMeICj9tLTMZUeE3KLPgdLZmW8tM1IymXtnzFn0SSaS8nrmL4oOSSD1jSRJA01Acmjbns16TJjnQsj6fhlOsQhJFUKznaFl0DNmf05fj2d/8rylQ/Oxv/pclN0yxub7gcH2rfZrPXCVRsI2jZI+bBB5Y96qNjzTTDaR2B9e3dDFAKCgAHQ6HTwKLZFkeCSTTn4c09FKVtoyMhGUL6/n6U7+nOujjH6+/2PbtXCKpIEoCt15/MU/84i2DH2x5cyMeSeCB2y4zlKVe8qCoKoogILpvxF24cJEBpyZImua+pR6K8Jd5UdB48I4ZRNpjnOiIs2//Ea6cdg4rHtth4oT2e0XiWXuF6iAvqqYhDeQPceHChYt+QnaAMdNu1kuyH8qy3aOJJMGAh7iimc4DLFyjK5obTbb69Ml13Hr9xagOWXmjq8uQx4aItEe5e0E9a5/dzfVXXsDGLfuItEd54LbLLHPSy++V9O852h7vme/URY9w4o4dFfRZnr2SDkrr2eebt73LsoX1Bg2H/twTisIDt13GwWOdBHweNA2qgj5WrksFnR++qynnbE0dXp/Ekfa4ycdsbW5kdNDnBqFLCDlzFffTmDrFTLYc+SRIZIlRrjLnAW6cO8FyfV1fma9vjZWkri9Yrp/PXP1e0XYO2XZvPpnSLk7D9S1dDBQKWoeyLE8AngI+kf78CnBzOBw+0AdzK0rEEiq/feMv3L/4cryeVLPBjVtOl17Nu/JC2yyLj46eYuX6nZbSvFUbdrF0/lRGjgjwlc9NQ1UhqaRKIO793nbX6HThwoUFTtlcoiiAqyKGHBQNjp6ImQz0B++YwX0/eMXCCd3W0mQxFCUht+w/Fy5cuChF9BScURSVqqDPKC3Wy7yrgz7bgEZ10GfhGn1owy5WtzTR1jITBGg/leC+H7zC4nlTbPXrx0dP8eWbpvH+x6dpNvZ/eJLF86awasMuHlj3Ko98scmx/N6J79TN4ssfdvfy6a1v2z57XWYyqRF8adoUTdCM5x4KBvjCdZMNaiv9pUYoGOBQpIsTHXGH5m4iEvYUKnEFY5/X5+m0r7soXvTH2s1nzERcyYlWA3KXucw1gZDK6nZKjosrGpu2vm3Klt609W2W3DDFdq7ZdDhOWf/xpGZ7D1bfac7Q9Yj2TQg9DhngLlJwfUsXA4VCCXF+BKwHyoFKUvzMTxQ6qWKGomr8/Dd/4s6vb+PEqTgr1+80gs8AT275H0tDgWULU80p4HRp3vxZ45HHhlg8bwpnjqrg2MkoSUWjdc0OumKKpZzPjmTfhQsXwxOCQ6MIwW0UMSSRSKgWByHSHiMUDLC8uZG2lpkpxzkYQFGtVqIoYd/kxU1/duHCxRBALs2p4gmVlet30rpmB6s27CJ8IMLshnH25ynmTDDdXk+m9askCsZ5m7dZGwjqTemOnYwa19LH1zmCD0W6iCfUFA2ElqKDUHLM4nORH+zupeOz189RTj8bNa6kAlfaaTmbP2s83/zJ703nf2dTyr8DHOVi7bO7iXQm8PokIu1RkoKAJolIkoiiqg7P3I3+lBL6Y+3mM6YkiRw5EaN1zXaWtL1E65rtHDkRQ5KsYZ98ZE5fE7Whcou+yp7rzr0HWbVhl6Fvd+496Pj7E3EFQVHSelBxzPbP1sv6XJNZtB6ihK2P5Nq83cP1LV0MFAqtRCgPh8OPZ3z+rizLtxU4ZlFDyng7VO732vKCVQdTbx2PnOiiqtLPt37yhilIfSjSRU11gJuvnmhqVti6qBF5bMjS8EI/xy0dceHCBYDq0ChCdRtFDEnYOQhJReWWayYaZcF6UNnnlVDj5teV8YSac5MXFy5cuCg15FJyLdlkd1VV+uzpiVSMY+WxIYu9rr/w0/mg9aaEY+sqOXCwg6deSNFsRLN0cW2ojPbOhPF/W87+bubb0zku7JHPs+/O18qUMydfTX/BED4Q4fmX3+P+xZfT0RWnsszHo8+8afiD9pn3fsesaTdzs3TQH2s3nzHzyZaWHLKFC5G5/tJdoojtuKIomHhAFFWz9ZEU10fqFq5v6WKgUKjv+bYsyzPC4fArALIsXwzsL3xaxYtMXrB4IsndC+pNRundC+pR1BR/mN8rcexElEh71DRGbagMn9dD25OvmUtenkzRcdg1vHCNThcuCoPOD3ko0gmSWOK86g6NIu50G0V0h+44QosZdg0IBQS+vekN0x7ynU1vWkoRIb+GMC5cuHBRasgl4OGTBBOP/kuvvU/IIeDnEQXD1p8/a7ylubhOn7dy/U7gdAPBpfOnGs0Jv3zTpabGhfpLwo1b9nXLn6oj09/oiXPVRfewu5dOzz7XlwJOvpr+0qE2VMZ1V1zAxhf2ct0VF5iCz07Z1w/f2WSR0YVzJ9jy4booXvTH2s1nzHw4kPPhYO6PueY1riRyz6JPcrIjbgRIR1T68EgCqqk/o9tMr3dwfUsYavGC4kShAehxwG9kWf4jqRdu9cDHsizvBgiHw1MLHL/okMmBpAGbXgybssqef/k9ltwwBUVRGRX04fOKpuYVtaEyVnyhEa9HsN0czqoJIkmlY3SWakDHxfCCJImcSihGs5hoPEndyHIqvFJJyqtT0yNVdZvKOSGTIzQUDLBw7kWMGV2JV5Ic+RiLBV6PQOuiRtqePL0njKwKOJQiqpaN3Q1kuHDholSRi53Zk45zauhX7pdsz5PQTBzAdrp2zOhKU3D5SzdOQ9W0lH+gwRmjylFUlbaWptTeLAqIEnzlc9NyspezeYhdG7v3sLuXPun0S4ZMm0BASNFh2Nzn7poT6v5dVaWPda1zUlmZgsaSG6ay9tndpkpYp+zreFLlXx9/xSSLo6v8tty9LooX+fAl92bMnvRBPhnI+XAw98dc84FHBEkQTbzry5sb8YgQzzhOdaAVcfKR3IBjCq5v2T8NREsNA7EeCg1A39MnsygxiFKq1ENVVW69/mKe+MVb7Nx7kNpQGQ/cdhkCQorTS4EH1r1KKBhg8bwphEb4qQh48ftEwGlzACGZUtzfWHYl0ViyaI1Od5G6KBVokkDkWMzSLKa8pqIkI3BuU7n8oZckhoJW+qNi11taQiVU5eORL15BUlFRVZCk3B0MN5DhwoWLUkSudmZPOs6pJH1Vy0yqgz5WtzSRVFXLeQL2FSi1oTK8nlRjOkXVQID1z+0xfIHlzY2AhkcDRVGQAAmRZJ72hqKoCKSdNUUrRXOlaJB9LxMKhMq9PPLFJo7bvJwYFfQRVzSLPNk1JzTJTlzBJ4kk0VAUDZ9X4Ka5E9j/4UkORbqYPrmO0VVltjKlqJpB7dIdbYILF90h36SDRFzJWBvKgGfb55rMFkuoOVGLODfTw3ID3FjGabi+pdv8d6DWQ0FNCMPh8G+ALmAC8DsgHg6Hf6P/64sJFhu8Pomj7XGD2P++H7zCjX8zgR/edxWPfLGJRELl3vTfjpzoMvjhNm97l1hc4YF1v+ORjb/nyPFOS7PCFenNAdJGTjBg25ikWJBL0xcXLooBSUUzslTgNF1BduOKUkGZX7Toj+XNjZT5C+0rO3ShZ7E5lVMXs94SJYF4QiUaUzh6IsqfPzrJL19+j1YbGXDkrlScm125cOHCRTEiVzuzpwCGUxbz4UgXX/3udk52xvGLgqEbJUlEk0SSgoAkCaz4grW5eKQjSjSRxCdCbaic22+YyuP3zmbp/Kk8tnk3X/3udiKdCaR0g7lIZ4LWNTvSTcF2GH9zMXhQFBVF0WxlLJrUOHCwnaMnohw42M6phGJkRmc2J5RIyZyiaiRJ+YmZz/qr392ONx2ofuK+Odz0NxNY++xu2waFT27Zy81XT0QeGzLm4jadLD1krvdbH3yxT9Z7Pjok80XJ2tbZtLXM7JMAkq4XD0U6jcaZhc41r9+VYyNGvboh2z72SdYQohvLOA3Xt3Sb/w7UeigoA1qW5WbgX4AA8HPgOVmWV4TD4XV9MLeiREK1eTA/2sXqO5tQFI2HMv52oiNuvEnSgx569t2//8cbhIIBls6fyhmjKoicjFEV9KEUUPIy0MiHY8qFi8GEc1lRgW/hBgmxpIaiqaZGEYqmEksOjw2yN9BLEkuxyauCQEdngtXpvgG6s/rbN/6SUTbpZjW7cOFiaCGn5oI5ZOw4laS3dyYsGU524z1w22X808JpCAJGE9dIe5Sl86cyti6IKApoaAZ9gg59XP3/wzWrqpjhJGNJRe2xas5J9p7e+rbpWT+w7tWUHGjw0I9Sxx5vjxsNCk90xHnqhX2ED0TY/+FJFs+bYnCJu70aSg/9kUWZ75h9XT2RT2ZkPnPNr2FibpV/Ctj6SHb3wI1lnIbrW7rNfwdqPRQ61t3A5cBvwuHwIVmWLwV+BQzZAHRSsecVSmVSmh/a5m3vGk0K9aDH4nlTjOy7Q5EuVq7fSW2ojMXzphBPWLk7ixnDfZG6KB14HLo8e7I6J5cKkorKw09aG0WsamkqyYD6QEAvSYy0R0tObyUV1Qg+Q2rPefSZN1k8b0qq6a2mueXZLly4GHLIxc7MJYBhV5J+94J6nnphn3GO7mDZjffAuleNoGAmAj6PkRnVU+aUG+QoTjjJ2MdHT1mq5la1zDTZWE6yt3jeFHbuPWgcZycH4QMRTp6K0bpmh2k+hyJdBMu9bq+GEkZ/BHEGO1CaT6A4n7nmc2yu1CJJRXPwkWZafCQ3lnEarm/p9swZqPVQqM5SwuHwSVmWAQiHw3+RZXlIVy048woJCOn/ZxoXz7/8Hm0tTWho3Wbf1Y4sQxJFE49YTxjsBoDDfZG6KB1IaKz4QqOReaI3i5EozaCdqsLUC0fz2U+PRxJBUeHn//0uqqoNGyMhX+iNYauD/qLQW/nob1W1D15UVfqQRBGUnmc+2PuFCxcuXOSLnOxMAVMz8M3b3iV8IGIKYJi5e+GDw+1GxinA9Ml1hg0ugsHDq0PXtwDy2BDzZ42nqtJHsDzVbBx6cNzADXIUKZxk7LHNu03H2VXNOQXPzqmtRB4bMuQrWw5CwUBahvzcv3g6m7a+Yzq2JlRGW8vMHvdpd18vTvRHEGewA6X5BIrzmWs+x+p2fE8NE1VVc/CRrFWvbizjNFzfMkvGNA1JEApuyllKGKj1UGgA+pgsy58ANABZlj8HHCt4VkUMryRauh4vW1iPV0opyuyHduPcCXjQECWB1uZGjttk302fXAcatK7ZbnrY1dXOG0oxkOa7ja1clBK8HtFUVuT1lO52WhaQuHrmeaxc/ztj/d+7qIGygESiy11/dpAkkaPpRkM6/dGY0ZV4JTH1ImIA9Va++tvj0HAwFPTj9wr0ZBcVw37hwoULF/miJztTkkQipxKsf26PKbP5+Zffs5Zlp0vS/ZJIKBgg0h4FUjb4wrkTTDb4soX1bNyyzxQUDAX9TJ9cx3VXXGBqYruiuZFQsKxbx41u/jY83Nriha2MSaIhHzrsquYcs6ePneKWayYaVC2ZcvDAbZcRaY9Z/MjMY73peXUnG+6+XrzojyDOYAdK8wkU5zPXfI7NtOO7k/kyv8feR/JLJKLmtZG5/hEE0LRhG8twfcvcZWyoYqDWg6BpvX9rJsvyBOA/gQuA46QaEs4Lh8N7+mZ6BeFcYP/Rox2ofUgcLkkipxIKB491GoGsupHlVHglo3GJ3dtoTRJZ++werr/yAsr8XlY/eVqwH7xjBvf94BWLUm+7s8nxwWuSSOuaHdZzWmYiDOEFUlMT5PDh9sG8/kDQ9Z1LP8huIRjs+14ohtp60TwSrd/fbqszhKS9KTpcZRf0LCHBCDDoyEcG+nIN5CuPevPbbIOoNhQg2pno8+t1h4HWBQMkt9CPsltTE+S6rzyX1znP//u8Xp1Tynq6PzCYe9dQ0bkDfQ/zyep00m0P3jEDvyg4nme+hmi7NyydP9WgyVve3MiooI+4gu2x31h2Jclootu5l1K26lCRXeid/EqSyPHOhNHXR3/RUJ0VhLALAuvULpH2KKvS9ASmZy1J3GsjQ7bHdoP+tGuL1ebPZV7FIrv6eu/LIE4+OqSv9U2+Lzz6Y665yrwmSQ72fhNCNxWDw91e6I1vOdQw1OIFvUG+uqs3sltQBnQ4HH5bluVLgIsAKfVVuGdvuIShKCoVXomxdUFbRakoKh5JhHRHZEQBn0+iK6Gyc+9Bdu49iDw2ZJQL1oTK0DT78upDxzr51tNv2Cp4u1KYUDCABiiCUPTGrQsXAwlV1QgFA5YyXVVNKa5Sg+LARa8opcUj35/QN1BV1UiqGp2xhEP5YCojbiB0ZaaRnQ+XXyKuMLrKbyo79HuFnILP9OJ6Lly4cDEYcGoAGPB6TBR1hs3toNsErMHnTP2rkXKABE0jqdrvp2fVBFnbOhtJFPBJAnFFc7xeLK7gS+8jTo2/+ropmIv+g6KoVOdQ4amXa69qmcnhSBeqpqGqGs3XTqK9M4EgCHg0c4WVk7ypqpY61mFO2UE6p+ba7r5eHNDXux7U7Iv1nqsO6Y/s+HwzI0VJrxbQAAFRcmaLy/V35WrLKg5rTFEL95FK6UVivnB9S9df6inRtq9Q0L2UZbkOmB4Oh38hy/JqoEGW5S+Hw+E/9s30ihPdKcpspa+X9mVSb4QPRIzuxovnTcHryb07t3GdrFIYeWyIW66ZyPL0W5vhVjLgwkV38HklbrlmoqXk0ecVUeKltz4kh6aKufIBD3XYGd/33NLA9Ml1psZAtaEyPjjcTigY6HddmTmnxfOm5MXlJ0kiR07Eeu1MDDZ3oAsXLlzkguxGVymqjBjf2fSqre5z0m2iiJkqoZuAjIYTPzMIioaEYFSgOOnuvx4amH3ExcAh16BYXNH4y8F2fvW7P3PdFRfw3Z/+wVS+HvBJJufduSm2s/1mJ78P3jHD3ddd2CKfhoH5INegutcncaQ9TluGvLY2NzI66CuISzfXtdNfPtJQp72RJIf7JokwTDKge6OfhxI0SSByLMZjm3eb4iXlNRV9yvVTKAnqBuACWZZnAZ8BngIeLXRSpYxspT+7YRxtG3axaes73L2gntpQGYBRprV527ts2voOyxba/w3M3ZN16JxJ+jkL515kBNf0c1Zt2MWQ7gjpwkWOUFTNsj6+s+lNy7oqFXi9AvcuajDpjHsXNeD1DhRbQXHDzvh+eONrNF97sUXPbtr6zoDoysw5bd72rmU/yOSIzOX35DPn7P2ip+u5cOHCxWAgO/to/qzx3dq2ueq27nRoT2P0pLsHch9xUXxQVI1NW9+h+dqLDW5wSMnY6idf4+CxTpNciBIWn2/ZwnrEbsrx7OT3iV+85e7rLmzRXRbnQCCuYASf9Wu3bdjVY7+SnpDr2vFJ0Jq1NlqbG/EVWPJaqC1e7PB6RHvfsoR7JuWL3ujnoYSkYh8vSfbxS81C96lR4XD4W7Isfx34STgc3iDL8p19MbFSRbbSD5Z7U3QakS6eemEfi+dNYWxdJQcOdpg6cG/cso9VLTNRbbpz273Rzm6aAfY0HooKuJQcLoY5kg5lRUmlNEtqojGFF3bs5/7Fl5s6Ff/DnItK8vf0NZyM746uOF+7fQZd0QQ+r4fOWIL5s8azedu7/V5elTmn8IGIsR+ce2awR/1caEmY2zDWhQsXpYDsjObQCH+3ui9X3dadDhU0rdsxnHS3nS0/XMp0hyOcSu8lUSDSHqWjK24rYwGfxyQX8YTKxi37TJRwG7fs4yufm+YoO3byu3PvQZZ8doq7r7uwYLCr3vqLAiPXtZOIK4wO+ky0dT6JgrKvYejTM0RjyWHvW/ZGPw8lOFE7qWrhWcuZKPRe+mRZ9pLKfl4ky3I5UFn4tEoX2Uq/vTNhod64f/F0o2O3jkh7NNWdW8TUnVt/o+33isQQLIpULw/THMomPjjcbtNERSv5BikuBgZen0RcgY+OnEKSpD7ZwAcDHof14ZEESvHVtSQK7P7fI/z6tb8Y39WGyrhxrjwsSi91uXQyLJ2M78jJGKqqklQ02p58zVRC5/NKSGi9lu+e9Gf2nMIHIqx/bo/R2KLbbvd94Ey4/KMuXLgoBmTqSo8o4vEKxBMqqqrhFQTa7pzJkeNRkopKsNzXo+7L1m1IIpokmnSxM81GapxuqfUcdPfS+VNZtWGX47xclAZy8X2cSu+rgz4k4IHbLgPs9+loPGmSCz1grcuOlutflQAAIABJREFUPDbEwrkXIQgCmiiCoIFmbkboZAOggaDzs7r7uos09KqObHn1UFgVvb5WDkU6QRId4wT9RoEhClQHfabvqoM+W70rZH0niIVXiA51egZJFDlvzAh8XhFN05AkgfPGjBhW9I75yNhQxEDFSwoNZj8HHAaOhMPh3wO7gJ8UPKsSRnYp30uvvW8pA6kbWc4Kh7KpzGyOJ+67iraWmYyu8nP4RIzWNdtZ0vYSrWu2c6Q9jjejlsSuhHDZwlRZIJwuE+lKqLSu2ZEeZweRzlTXbt24svubi+EJncMrJXe/tpW7UoEkCbYlNZJUmpQVw5lSwSyXuevD5c2NXHD2CGqqy21Luvd/eKLX8p2L/izkmQ3n5+3ChYvihZQO9iYFAS1tS/Z0fKaufPzZ3Rw5HmV5+vM939/Ox0c7+eEv9hJPKPzo+bfyoity0sU+SehT/buiuZG6keWuTi5x5Or7KAi2pff7PzzJsfY4AE9u2cuXbpxmsTPrRpab5CJTnvT+PY9t3s1tq1J29kdHOln77B7TPFwboPSg68ZDkc6cdGNfIjOWsLZ1Nm0tMwvmKc5cK7c++GK3cYL+osDwSQIL505g/XN7aF2zg/XP7WHh3An4snw5X8BjiZscPhHDFyhsxQx1eoYyv8ik80ez4rGUPlzx2A4mnT+aMv/wiQX5vaKtjPm9w+MeiKKDjPfxzxc0rffRfFmWpwGHgA/C4bAmy/LUcDi8u89mVxjOBfYfPdqBOsA8r9lv0zO7Z+tv10VJMDL4PJKIJArEEorp7btO8q9JEq1rtnMo0oU8NsT8WeOpqvQxuqoMD5pNBjNomsY3f/KGURao4+G7mrjne9uNz7WhslRHW6A13cAw+29CEWVB6/dkEK8/EBHLcxkk2c1EptzpSMlEE0KJvQlNCiI//XWYz356fFZZkYxHKx75zgf6es+lGzUMHdnNVS6dspqSgsCStpcs47a1zORbT7/hKN/d6R5NEln77B5mN4wzSrZeeu19ltwwxaQ/C6kyKZYKlYHWwQMkt9CPsltTE+S6rzyX1znP//u8Xp0zmPtjMWIwbYahonOd7mFvGjJpkmiyNZc3N1oqAnWHZ1RVGZGTqUxoURQQBYH2zgQXnl3l2BApe3x9vLaWmXigT/UvGeMF/B60hDJkqgaHiuxCz3t3T76PJIlEFY3bVzvbDUvnT2Xl+p3IY0N8/jMTGF1Vhtcr4ZFSjSxT1xJIKhqqquGRRMS0Ldr6fas9s3jeFFOFlD6PgbIBBtvXckIu8yoG2e2vZnX5yEBfy0suayUTPVUq9ucceuu/9iRfSUHg3//jDebPGm/Y+pu3vZuiZ7CJp+XzDIpBbjVJYu2zu218makl5/f3FkMp9tEb5Cvj0DvZLfTl6X+Ew+GJ+ociCj4PGrJLC0UJuhIpjrByv0RShYSioiZUTp6K8/5HJ7jkolriMQVFhf/atZ/PzDwPSZLSJS4SiqqyeN4Udu39iE9feo7R5ELf0EZX+YklVOKqhiiKSJLGx0e7WDj3IgI+jyE8kfYoJzripvlmNiUYyrxGQw0DYYg6c3iVnkx4vQJzLxvHyvW/M9bOP3/+UrxeAS3e8/nFCMkrkkxoaQ5CEckrlKTzm68s5yqXTiXVdqWs0yfXMbq6jH+5+ZNopJzDXO+l1yeRBBbOnWDq+P2lG6eBYDbCNUGgzCfSFVNRVBUkEX/AQzzac12TS6HhwoWLYoJTQ6a2lpk4eSNOfVL05IpguRdV0ygPePm3x18x6VNV06iq9KFp4PF5LEkbYM9fGAoG0BBIChpoAkJ6cqIkkCRF1eHzimgqJBQNUUyVoXpEiCVUFFVDI6V7BU1DQjDtWX5RIBQMcOzYKQv1RynuycMJuXC6JoEPj3TYliW3dyY4FOmistzH8uZGw2H/9qY3+crnLiWe9gWTmsrhw51G9ZXuvwUrfKYx9XUwtq6SxfOm4PGIJNP0ixoCgtC7wKP+O1QNREFI03wIeQcFi+VFeLEjH92Y6z3NJ6gtSSKnEgoHj3US8HmIxpPUjSynwis5jt3THPLlP1ZNdAUaajdi5i/zEo2rRrA64BOJdSUsx+U6h/7ioM6HnkGSRI53Jngo43mtaG6kusCXEP0KQeP/mXURX//x68ac/+Xzn0zpi2GCoRT76A2yKaKgf+jFCr2Xu2VZvgnYDnToX4bD4WMFjluSsNscli2sZ+OWfUw6L8Sc6eM43h4zDJDpk+tYOHcCy9Nv8/QSlURS5Yktb3HdFReYgs0rl8zg/rWvWDa0B++YwRO/eIudew9SGyrjX2+djtcjmgydZQvrCQX9PLnlf0xzNoSK7vnxXBQP+uvNejaGFG+yBt/48e9Na+cbP/49D9/VNMgT6x308rLMgGdrcyM1Vf6cgpnFgt7IcqFymc2NN31yHQuuklnx2I6815NOB9IVTfLtTW+Y5OtbT7/B6jubONIeN56TrvNL/bm5cOHCRW8aMmVzaLZ3Jpg+uc5k796/eLqFJknPMl3x2Csm2zrSHjX0NUBS1Uzj6xQH657dbbGpW5sb2bT1bY63x7nlmokWm7k66Gfjlv8xbGu9j8rR9rhlzxoxQh0Qu8xF3yKX/gqKqrHjjx9y76IGVmf0jrh3UQMv7NhPbaiMYLmPrz/1ukl+OrrifPnbv6U2VMbXlsywpf5aueRy4/ry2BA3Xz3RkNHpk+uoCZWZ7IW7F9Tz/MvvcePcCY6Bx2w5fOC2y0gkVFMgTB9n4dwJjA76cgpCD5TvMRSQq27M557mE9TWJIHIsRiPbd5tksnymgoLCXSuc/B67G1vr0dESyi9GhNSwedDx6MWu7i2OmAJQjvb/6KpKkZyOE7KOi5f6PQM2XP1e0Xi2QF7BGPNQep5PbRhF6tbitfvFAXBCD5Das5f//HrPHxXE8MlEjSkYh+9gE8SaG1utMi4TxJI9GHmU6GMHvOAHwN/Bo6k/x0ucMyShd3m8J1NbzJ/1njmNJ7LoWOdJgNkdsM44wHrx7dt2MWhY53MbhhnGCH6306eitluaJH2GLMbxhmfjxzvsgTbvrPpTaJxheuuuMCWQ8zlFysdOBkhfa0XRdGeN1nsg0YOA41E0t4YTJToy5VYQrPVHbFEaf2e3shyb+Qyk6c0CYwK+mi7s4mH72rilqsnG05lrnPQEVegbcMuqoN+W/lKqubn5KTzS+25uXDhwoUevMtEZlKDHbI5NF967X1uvf5ik70b8Hls9WnA5zH+r9vWmfo6CTzxCzNn9MK5F/GdTW/a2tRtG3Yxu2Ec82eNtwQHv7PpTcMW179btWEXccV+z9IzhvrbLnPRt8jF95FEgZmXjOGZF8MsnjeFtpaZLJ43hWdeDDPzkjHcu6iBHz3/lkV+yv1e4/MJB/8NTq+H+bPGm2TUzl549JmULDvJlp1NdfBYpyUQpo/TlpbpXDBQvsdQQK66MZ972l1QOxtJRbPVaUkbnyfXOQiCve0tCFZ9nwSe3vq2ab08vfVt298Vjau2dnE0bn2pkav97/OI3LuowXTcvYsa8HkKC3vl43slHTJpk4NIrdkThpqv3BsMpdhHbxBXNDZlrd1NW98m3scyUFB8MRwOB/pqIkMBTptDsNyLJFqNar30MPv4gM9DwGelxDjREbd9K3OiI04wnf0BzsZ7LK7w/Mvv8eAdMxAQUmWGnC6z0RsWuKVVxY3eZB31BvGEwsYt+1g8b4pRVrhxyz6+8rlLS+7FhCjaZ/iLNoZTKaC/yssGGr2R5Xzl0qkMrjzg4Z7vbaetZWav15P+HESHLCpRICedX2rPzcXwRnBEGQF/fhIbjSVpP9nV84EuSgbZ1SSZwTuD8iirtDuRUC36W8OsJ9s7E450Bzp021r/vx6E2bn3IMfb48b4o6oCxrFO9rn+/+y/6bZ45ndOe29SGRi7zEXfIrNZm5Pv4wHGjK5k596D7Nx70HT+omsmE4snLd+nZOG0w+7kv4miYKyHsWcEc7IX9O/tZMvOpnLyCU+Pk5v9MVC+x1BALroR8runuWTr67CjIjoU6UJVrZmHuc7B2faeZn3+ApaKk7sX1GPHzZSPP5PrHLpiSV7YsZ/7F1+e1ffnooJkNZ+5OvoFIpYs9GKBs688iJMaYAyl2EdvoKqa7V63+Pop9GWvzYLupSzLIvDPwGcAL7AVWBUOh4flC1GnzSE0IoAkCkTjSUvpod3xVZV+jp7osvztpdfet2xoehmVnqUBWK6jj1sTKmPp/KmoSurNHAgmDrxMjjuXY7R4kY8RUtB1JJFJ54U4q6YSSYQRFX4mnRdCKsEyFI8ksmxhvaXE1iMJ3fKSFSskUWT65DpLowhJFKGEmiT0RpYlSbTnp3KQSwWB/37jLyZD9Ne7/sy1TRcYQY3ericpXU5+9ESXg3xZy83tr9X/z83lbnTRVwj4Pb1qkFh8La1cFIKegnd2ZdgP3jHDor/XfHWWSS9u3vaurT7duGWfcU5mQFoP5Amk9pPwgYgxvj62k+4dUeFHkuz7AlRV+oFUIEnvo+K093psxnBp7EoDPfVXUBQVryTZPt+/HurA67F/9vr2Ko8NMbo6YOu/CZzm29QzsXuyF/TvPZKIpGmmfTzbppLHhqiq9JtoPvRG9iNHBPj2lz+V6iGC1qM9MFC+x1BApm7srlF4Pvc016A25EcjIImCgz9hnoMkOtjedvarJlgqTh595k3abOgnJNGBLsNmXEkUHTiYzcdKosDu/z3Cr1/7i2nMG+fKtrKq28ep3luiMw93Hr6XRxJKzu909pVF1GHiL+TrYw41iA7rURSFPn1xIj3wwAO9Pvl73/veauBy4PvAq8ANwLQvfvGLv+qT2RWGauCfurriODRt7HN4RIGGyWfyRvgQp6JJY+E+/rM9/M/+o8y4ZAyfuKiGt947wqlokngiydL5l/BmxvF3L6jnp78OM/MTY7j84jGmse74u6n86YPjLJgzgSvqz2Ly+aP51e/2s+Aqmedffo8PDp+iNlTGVY1jaZx8Jrv/97Bx7vLmRoIBD5H2OPev+x2bXnyHV9/6iEsnncGTW/6HH/xsD6++9RENk8+k3O9BG6iblicqKvx0dg5e17iKCv/KAbhMt7JrJ2fLmxup8Ep9+twCfg+jQxU8+MOdPPPrd3l938d8/jOTqKrwoiT7biOSJBFVFFAAQRLxiEKfy1+ZX0ITBCadN5K/vexc6uUaRlWXMSLgIZEovU01EPBw3lnVPPrMm/zi5T8Rfv8YS+df0u2zKQbZzUZvZNnnEbl00hkmvXn/bZfhlUSSmlWGRK9I7cgKHvrRaTn+3N9OJFjh45MT6/jFy+9xy9WTDL3sNAc73VNW5qF+Qh2bt73L1PE1TL1wtEW+PjGhzpirnc5vbW5kRLmnT9dUNvRA0APrXjV0f291/UDr4AGSW+hHm6Giws/TW8N5nXPT30zo1TkD8Wx6+3sGY+8eTJuhGHVub9DdPdQ0DUHTEEklL2TqE1UUeGDdq4Yjcyqa5FCkk5b5l5h0/v9pOIfLp4wx9KIkCnxq2tk0TDyDf5hzEbMbzgENXtx14LTeXNTIUy/sQ0qXyz72n7vZu/+IRb/+zYxxTJ90pqOe/49f7eO/f/9Xbv+7Kcbf9L4AD/5wJz996V3C7x/j9r+bwmcuP5eKgMT5Z4cse+8ZI8uZOr6m3+2ygcJQkV3oGx0g2dgqdy+o51e/28/MT4xhxhSzv9ba3Mj/ff0AmgZfuG4yX//x67z13lG+cN0kFl4lc1XjOH77xl8YM7qCaRPqeOu9IyQVlX+8/uJu7QX9mouumUQy3djY7zktY5k21dm1QW65ZiI//XWYW66exKmuOH8/5yIe2/xHnvvtn9j51kfUy7U8vfVtLhw70mQP2N2zgfI9ukMuz7JYZFfXjbWjKunsiNneI5/Xas+2pv11NStQqmka5X4PsxrGcm3TeVzVONa5qaBHYsqFNaY4wJdvupQRFT60rON9XpHzz7HqtOw5lKft3ey5hiq9JLP8KFUQ2PTiO6bvTkWTXHfF+YhZ96HMYdxqm3HLyjzI544kGkvikUQ0TWPGJWMIlntMx+Yjq5n28dNbw93ax36/vf63s+FFBCSPyMQMv3N0dRk+wd7HLQa5HWq+cm/g84hcfskYGibW8beXnUvTJWO4puk8yr2iZU0OSUgCk88fbbKV7l5QT7DCi+BAH9Mb2RUK2TRkWf4j8MlwOJxIf/YDfwyHwxN6PWjf4Vxg/9GjHagDyLdzOssMPjjczqat7xA+EAFSGRVL519CUtFQVJXj7THKAh48okh7Z5wTHXE2b3sXgNv/LsUvNrthHFWVPkZU+Nn66n6uaTofVYVIe5RguY+Dx06x448f0jj5TILlXqLxJOeOGYEARqaznpWSBFrTDQ911IbKWDxvivGmpzZUlmpoUKRvumpqghw+PHh5VDU1wYEoRDmXHmR3ILIZNUmidc12i7y0tTQh9FG25kA1NdEkib8eOsnZtSOMLsv65776LQMJTRJt13J3a7dYZDcbvZFlr08irqTK4fxeieMdMR76kb0MdSfHHrTUC20B0ARUVbNQE+mw0z2aJPHL7e9xbdMFaGhoGnR0JTgc6eKl195nyQ1T+eX295jTeK6Rff3nD48z4dxRqd8rCfg9Qr83IOyNvDhhoHXwAMkt9KPNUFMT7FXGcL7nbF59LT5vfkVyvaHG6O3vGYy9ezBthmLVufmit/cwKQgsaXvJ8v0T980BjVQ5uCiAAGt/vptbrp5MR9dpWzh8IEJtunIvWOGjK5oKOkTjSc4bU0U8ofLB4XaTDaxpMKamgnhCQUpnRR8/FeNER4zqSj8+r4fOWIKOzniahmkHkMoUXTj3IsbUVCKJgtEcXEdtqIzVLU1oaLa69BvLrkRLKEOmymSoyC70nQ7QbZXTcquBlrIXJK9ILKEZ9uW21w8w7swqxp4R5N8ef8UiL0vnT2Vs3QgA1j67m9kN4zj/rBGcOBXjZEecgM9DNJ5kdKiMcr8XVdVIJFVi8SSHj0eNjPzUOEHTPq7PUwNDjuWxIe5eUM/K9b+z9QHXP7fHZA843bPBrqTK5VkWm+x2N2dNEln77B5LRu2SG6YU5IcnBYF//483mD9rvDHu5m3vpmgENPO4udqHmiTx6lspXatpGoIgsGvvR1x28RiLH5WP/6hJIr/c/ieTnZyqUjzfcg9En4cPDndYMnTPqqlEjZvt6FxlNR/7OF9bOp/1Ugxyq0mSoY/M8ji1JH3l3kCSRI53JUw+5YovNFJdNjyarfZGJ/VGdgulMxH14DNAOByOybKc6O4EHbIsjwBeAa4Nh8N/lmV5DvBNoAx4JhwO35c+7hPAemAE8FvgjmKm+NBLuRAEVq7fafrbzr0HufX61AK+PW2Uy2ND/NON9dzzve3GccubG42mWDoHi24kJJMqdzy8DYC2lpnGNTLLTNa2zsajvw0Ho6RMEYRuOfD0zy6fV/Gjp5LBPrmGI9dV38lHPl2dC4Giqty/bqfl+7Wtc0pS1ocSF19vZDkRVxAAvyQSSyiGoQBWGepOjgVNNcmZBKDkXmWkqCo//82f+MyM82wDLbden/r7z3/zJ9P3ho5OQnwAdrOhJC8unOHzSi41houigVN5ORoIimro26QgsHPvQW741IW0rtlhGkPnYn5k4+vpQNkfWN7cmHaENDZtfYebr55o4hrNfAGZFARiccVij0PKhtYRPhBh5fqdxne2PM+q6vi3RFJFGAC7zMXgQbdVdLk1vsdsx2hovLjrAIciXY49JgI+D4qqpnR2miv3a7fP4OEnX7esl6/dPgM0jZZHtlnmlBrHvI/rc8n0+cIHIo6N7LvjlHa6B66M9w0UB77VW6+/uCDbTBQFWxoBO/7hXO1DRVVZ99xe1j2313Rsw6QzrHzVEty7qMGIZdSGUk0AJQkL/YSiarZ28mdmnGcZN6mots0VV7XMtHJb5yir+djH+drSpbZeFFV1kMfh4ysoCBaf8qEf7WK1DX3MUIQHuHHuhJyofgq9TiH4gyzL3wK+R+pl6xeB3T2dJMvydGAdcFH6cxnwQ+BTwF+ALbIsfyYcDv8X8GNgcTgcflWW5SeA24DHCpx3v0F/2yViT+QuiaLx/0ORrrRRYG5O4dR4oqrSR+YLq0x+sExeLyc+LydnILOxS2/4vAb7jfhwxEDcc2euq77jARqowJjkwIdWqpxOpczF1xeyq48RU1NZx6FgwHQvMmXImV+ucDmWRJF/+dy0bjnsBovz2TzP0pUXFy5clCZ8kpDmfI5xoiPOS6+9z41zJ+ABxIwqFt3W6I7z9lCki3PPHEHbnU14PQJaIlXdt3DuRRau0dQLyCaQJDyCc1+Uqko/8tiQUaVYGyqjOuh35E+VRBEBe9ve6xFJJos9vOCit8i2W3ySQFzRbO2YTK7ebJnWM+2rK/1IooiiqpxdW0HbnU0oiv3LcjQNRbWXu5EjAkiSiM8jmDKwvV4BNaHmxSnt2gMDD0kUuG3eZEtWcaHPwiMJfPXmT/LIU68bQaSv3vxJW/7hXO3DfPiPFQWeeTFsauT2zIvhVCatzT3IhYMaUtUHdmvErrlirsjn+kPdlh5qvnJvkHRqNDxMEnYURWV0lZ+2liZjP/F7+75St7frVcedQAjYQYoDejRwVw7n3ZY+98P050bg3XA4vD+d3fxj4O9lWR4HlIXD4VfTx20A/r7AOfcbdDqB1jU72LQ1zL2LGlLZHmBwGr361occPNZBa7rZxJyGcxhVFaB1UaNxrG4sZ6I2VEYo6MfvF42/vfTa+7Q2NzJ9ch03Xz2R9c/t4Z7vbad1zXYinQkkyfx4daMoe04vvfa+8Vl/y9Gb37yk7SVa1+ywvbaLvsNA3XOvV2Dh3Amsf24PrWt2sP65PSycOwGvt+9yk/XNPBPGZt6HqPCJxprTr9Ha3EiFrzTl1G4t57t2BwN9IbuSJHI8Y4x/ffwVbrlmIvLYkHFMpgz5JGyfva8P2vlW+EXG1AZZ++xu7l5Qb5Uvv8j9t13G/Yun09Yyk/sXT+f+2y6jwj+wcleq8uLChYvShCSJHG2Pc98PXuGe721n/XN7uHHuBEYFfYiSwJH2OK1rtqf3ge0snDuBffuPWPTo3Qvq2bztXWpDZSSSKkeOd9EZVYhpGj5JYMzoSltn8VCkk9Y12+mMJagOBvjyTZdaxn1yy15j79B19oZf7uUbP/49yxZa57H22d1EE0lW2OjSqgr/AN5dFwMJO7vlSHuctc/usbVjMhvQXXD2CFZ8ISUv8tgQt1wzkcc272bpI9tSFAXHujh6MsqR410cPNZpbw9LIn945yBfunGaSe6WLaznmz95g9bvb+fwiRhrn91trKcjx6P839f/YpJjvZF9tlzr37v2wMCjzC8x6fzRrHgsJVsrHtvBpPNHU+YvzED1SiJlAQ9L50+lrWUmS+dPpSzgwWtjawd8kq2NHMgykv1egVuumYTXk7KtvZ7UZ7+NT6hqqczuVRt20bpmB6s27GLn3oOodjzYkv24Psk6rt7wMxOpF4C99xl9kr2va3d9nyQ4+BMDxRTXvxhqvnJvIDrEJcRhcgu8PonDJ2Im++zwiRjevnCaM1AoB/RMYCVQC8ZLLS0cDk/N8fw/A58m1cjwmnA4/Pn093OArwL3A18Ph8NN6e8vBF4Ih8MX5TD8uQwwB3QmN9DydGA3+43aLVdPxucV2bX3Iy6fOoaTHXFWbdhFKBhg4dyLOGNUBV2xJKIgmNLf77mlge1/+CszLzmLMr/X4En6wzsHuWzKGFq/b+VaevjOJkRNs31rr3OYSVLqTWV33Kd2yMwG+OBwh4nruj95pF0O6L7lc+0OA8EFNVAc0HgkHv+59bfc/tmpUKJZS/oa7K7DdiaGjOxKEvfacMstnT+Vlet3Uhsq44HbLqPM5yGhqIiiQMAnEYurxttcnwSqovWYiZ2p5wJ+D1pCMR2TyXOXWYUyuqqMV9/6kJmXnMWR412WLJTR1WUWvrr+Rl9VTbgc0PljoDige3tOvs/T5YDO+dqDrnP7ArncQ29WRrMowj3fs+cABRz1piBCMqmiavDRkZRtGWmPcu+iBp55MczOvQcNO2FklZ9kQrPdD/TeJnqgDqBuZDlHT0QtHNNfu30GHx3pYESlj7U/e4vwgYiRqXrGyApUDR595k3j+Ee+2ISSlf06cmTFoNqmfY2hIrtQuA5wslsy++d89lPnc23TBQathqpqJJVUlr7fKxJNqIDAchtZ/erNn6TM7yWhKKiqZqEt0DNHETQEBJKKaut3ZffzuX/x5Tz6zJssnHsRZ9UE8XkFOqNJDh7rpLLcR0XAi98rIiBYqmYH29dyQilxQOdio+fbaydbz/qkFCVdNvLjYJYsvUpSHMwXmI4VfRJHjkdt7NkAatw6Zq7XF30SR09GTdznIyp9jBphHVf0SRw9EePhja+Z4iOjqvyWY3NFvhzQ+fDjlhoHNB6JffuPpvvUpGTs7T8fZeJ5o0rWV84Xks/DX214xs+uqUQZYL9tMNCb2M9gcECvA9YCfyBFwdFbiFnnC4Dazfc5Y9SoygKmlR8ORTpNNBp2PDq33TAFQRC4dEId0ZhiBN4ORbqM4MnieVM4/6wRRulKUlGJxpM0Tj6T6qCfdc/uMYzwuxfUozrQGCQUla5Y0kSkft8/Tuec2iB/OdTOgz/caSJYH1s3Ao+n51c8qqrx/scnTeffvaCep17YR/hAJDUXQaCmJth3NzcD/TVuscFJdg8d67R93n19zw8f7zS46TKfsyjC6JF9d53qao1vLLuSRFLF6xGpqvCnGrz0IT46esp2PS6ep3HGMJGngURfyq6qapw4FSORVAn4RDqjiu0YZ9UEWds6G69X5ERHnAfWbTcZDyNHBBgzuhJRFGx12H3/OJ1xZ4wwZE9VNQ4d70TTUi/rumJJPKLIyFCFoSc/OtJh4ljUnb+1rbNZ99yynJHhAAAgAElEQVRepl98pmGs6/N85KnXWdUyE1/A2y+yPhAYyjp4IG2GYsFAPc/BkpuhLK86+ltuu9PPp7piHIxEaUvbs9Mn1/GF6y52tEsFASP4nM3f3LqoEZ9PZOOW/2F2wzj+8frJhIJ+/uuV/cb+rdNsPHjHDEZXlfHAkss4eLTTCF7UhMo50REz6DV8Xol/efRl2lpm2nJMR05GTfb3qg27TJzQHo9oauYFArUjy3O+Ry66x0Do3EKeTaZvd/q70/1z5jScwxX1Z9O6ZjuhYIBbrploCl4sb26kKugjFrO3XSrLffzo+bfYufcg0yfXsXLJDDqjCcoDXjZveyfdPyjVtFgj9eIjm9M8cz6QoiXz+0Sar51Ee2cCn1dEFAUeWPeqJdD2jWVXEgoG+vSe9SeKaV5OsquqGh8e6eDjo6cMvXTGqArDBtWRaUPq0Cnkzsz6ncmkyp8/PmnoWT079dwzrL57PuMeipzi0olnGA0qdV8PwXyvjxzvtLVnH76ryfJMDkU6+dKN0/jW028YY37pxmkgQk2W/3j0RBfRmMJjm3ebbHaw+gWR9igBv8TS+VON+xrwS4iiyKgaq07OBU7r284vORTptPUll3x2quXYXPyMwYKT3H50pIOv/8cblu/Xts6xyM1QxYkOexmTJIGRw+AeHIqcso39ZOuDQlFoADoWDoe/3Qfz+CtwZsbnM0jRczh9nzMGMgOaDO4cO66t6ZPr6MjI9nz4riZHrmcQWP/cHkLBADdfbTZm7l5Qz/H2OOEDER595k3aWppsOXsEwUqk/uAPd9LW0mQoRP37h36UMub9otBjVpwmiZbzH33mTVPGCZrWL2/PB/ut/EAaPo6yK0m2zxuNPr03mkeycCs++sybtN3Z1C/PQACSSYWj0Zz6mOYFyWN/z0RRKMosj3yQ65ooRdnVM+Sf3vo2N/7NBNpPQbDc5/AsYfmaV4yO7ply+51Nb7J0/lR8HhFBUW11WEo3ns548Aa8xOJJTnTETfo3sxuyk1zp3M9OfHWHI1186+k3+ifbv58xCBnQA3Yt6B+boZgcZjv0JgN6IK7TFxjkDOgBu9ZgZEBLksiphILPKxlBEYDZDeP46EiHrW488HG7UUY9f9Z4i43R9uQuls6fanLy9cBwZpOqQ5EuIu0xJEngZEfCFLz40o3T+MVv3+Pmqyfy/MvvUVnmQx4b6pYDVx8zM4hXGypLBY6ClXw9I+tPb4JYChmjvcVQkV3og2fjwIuqy81nPz3eCN4tnjfF0iRt1YZdPHDb5Rxvj9mO89GRDmY3jDNkfv+HJ3lo6Uy+8ePfG1n3mqZx64O/Zt3yOXx89FS389GpPpanszp1mR1RYd9fKBZTOBw1359ileccM6AHaDbOsiv5PBw7GbUEVcv9HlMWpeRgE0s2vokmmfXsoUgXbWm+++zMRI9HsuU19kg2Po/k4Ou1mH09RRRt5SepWP19yefB5xNNQTyfT0QSrNdXJcmhsaCNr+mRWGnzEmV1IX6pw/q2jWPkcWwufkYmikJunXwaO7kZolBEkbU/e4v5s8YT8EEiqbH2Z2/xz5+/lHjXMLgHvYj99EZ2C2U0eVuW5U8WOAbATkCWZflCWZYl4Cbgv8Lh8PtANE31AXAz8F99cL1+QSbP5uZt71o45G69/mIj+AxwoiNuyzMTCvo5eSrGsoX1tg1WHn3mTebPGm98RtC4e0E90yfXsby5kYfvamLlkhl4PakOyHMazuH7/zKLH9wzi/sXX44k2QdFIu0xFEt7ACucGscFy715cYtKkogmiSQFAU0SXd7oXJF+3tk8bgh9a8ArinODwFKDKAqW9bhsYf2gv4UedshRdnXdEFM1YokkC6+S8XmkVIaxoFm4EL904zQ6ulKNqkIj/LZyO6amEkXV0KTUm+yeZDueUDmcNoSzX9bp7kPAgS/t7T8fTfHCOfDV6U21Vm3YNVz6erhw4WKIIQkcPNZJMmlu2hMs97Jp6zsWPd3a3Mjmbe+y448f8rXbZzC2zp6/OeBLWZDy2BDLmxv50o3TGHdG0MLzf6IjTjKp8c2f/N6ko7/19BvMbhjHo8+8SfO1F7PhlylnUu+bkr3/bN72rvE5mg4O6TZC3chyNvzyLUtA0dXbwwc99c+RxNM+lVMTeY8kUjuynP/39hncv3i6wTt+76JGfF6JmuqA6fiTp1JJRtMn1/G1JTPQ0nOIJxU2bX3HtueEPp+Fcy+yDYJrmhO3qWsH9zUSimYbVE1kNaoL+O1tyIBNnxDFoTmanU/m9YgsuEo28RovuErGa1PlrGoaoWCA5c2NtLXMTCVGBAMWvmZRcJAfwYYDWtV4+MnXWbl+J61rdrBy/U4efvJ126CnUxW3HV900sEvTRbSsBEsvP4rHOIY+RzrFCspZh9aEgW+dsflrFs+J1XJuXwOX7vj8j7vy1TMEAWBSHvUxF8eaY/ayvlQhON67GO57VUGtCzLe0hRYwSBV2RZfg8w0hZz5YDOOD4qy3IzsBkIAC8A/5n+8+eAdbIsjwDeAB7tzZwHApmNJxRVw++VWH1nU5oHTETVzA9187Z3uXtBvSnN/Z5bGtjzv4eZcmENj/9sD8sW1jsGeyGl/OIJld/v+5h/mCObeJGWNzdy27zJTDxvlKm0prW5kc9+6nxTNoluzFdX+glIYrcZeU5dYGvSnEm5cIsOGPfvUIQGz7/8nqm78PMvv8eSG6b06WX0LKXs5+yVBLQSe0SJhMrGLftM92zjln185fOXuo1XBhI5yG62bnj4rpl0xpLE00EOVcM2s+JUVwJ5bIjKMvsM6Q8Pdxhl1k46MLOTtaKqBHweRwPSA5zqSvKnv0ZM3YIRNILnjqLCLxKPKaz4QqOJBkmnK8oey4ULFy5KCYqqEfB50MCkc9s7E1QHfXi9Zj3tkQTOqatkdsNY/u3xVLWKna6OxpO29BzLFtazccs+Iu1R7l5Qz/Mvv8eiayY72siHIl10dMXTFAZTWHLDVPxegbaWmagqaGg88Yu3jCzTFc2NVAf9rG2dk+qJIqXomrLLrV29PbyQ7dvp/XSW3DCFW6+/2Kh4OhSxr35NVUNp3PcDc0ZyLJHk8Z/tJtIe5Z5bGgzKmNpQGQGfh2/905WoGvzb2ldMPmJ10MdTL5y2Z6PxJNVBH7MbxnHDpy6kqtL+JbyiaRafsz+SV1yA6hAsVlXNlPlnZ0P+4Z2DVARqLPpFcsi+lSSB7DdisYTCMy+GTbb2My+Gue2GKZbMQ49HsNDGLFtYj8cjoMUzDhRwkB/r71cUh2C5opHdyswj2fuaHlGwkK6KIrbHiqIABVAUZ+9VXq9zQlyux3ocnpdHEouWT1mUBLraFdo2mGNGFQFvfvy3pQxheOtJ0SHGJ9qsx0LQW/vprr64eDgcPjfj/y8Bl9gc80egsS+uNxBQFBWBdDO/dCaFJ/UHxCxlFD4Q4fmX3+NrS2bg9QpAiv5iyoU1+H0Si66ZBMD9i6dbmk3oRs69ixrY/ocP+NsZ5/GvP3jF8sa7raXJ1AjgUKSLTVvfZskNU/mby85FA2KJJAGfF9Jcp0mElIIEW/J8PRsgO3jsTzc4jKkakiR2G4hOgikb/PR8Z+aQgz284QEWXTOJg8c6gVSgeNE1k1L3uw+vIwgC9yz6pKUxhCAIBRG+DwY8ksCV9WM4b8wINE1jdHUZV9aPsTVwSgV6c4tDkU7oYb0VC3KRXV03hIIBFv//7H19fBTlufY1z3zsbpIlWSAJqIDFakAINWCCkKhHUd5z/ACUcyTUCqGECLHKsfZVIragFQK2HlttwSKtgFXQ81KLqOccFGoFtCQKHhA1UlSgioSPTbJJdndmZ+b9Y3YmOzvPJLv5Ih9z/X7+hGV25tmZ57nnvu/nvq9rei4GpXvw8Jq9RrLiXH0IT774oeXl+OBdV+K+WXnY8PrHhvMQK+56pi5oBHiVG6qwsrwQew+eNNmw2HGwLEFIjNi2R0JWwRKCqk9OwZvqxgWDU+GJVq0wABRVex4ZHj1wBb4+HcA7H54w+ERDYgQCT9otnuLAgQMH3Yl4ESyXwEKFgoqSAqM9fGf1McyfNhaPxPikgGY7Hy2bhGXrtOCWVoTxcEkBvKk85t58OX61Zb/xfZ/XDVGS8cCd48EwDEJiBKXTc8EwLQkJXdAwa6AHqW4eTy6+BiluDhPHZEPgGSgy0ByWQQgDnmfAIjaJyBh+u+azA4qstVG39g5w0D8QG9tBViHJ2nvexRLIDPDzuyfjmzON2Pu/32BxcZ4pmbdkbj6ep1TRl07PNeK61ZuqDTHMwT4PzjUEkelLwc9+Z47rVm+qxopFhTjXEMSgdI2+TFE1Yfmd1cew7/AprFg0GSsXFSLDq2lMnK0P4oNPvgXLMPC4OCwrnQRRiuB0XShaADAOEYYx1gCgce3GfpaMb9lZgse9GRxLbCkwYpPFLGEMH1I/ruqTU7jisiyLfeGinZyWRDFhLLEfwwC3X3ep4Stn+Tx44M4JYChVnKoCarX2qnuKTMcRhl5AsvD2cZaYkBC63aQlinlCzynwJKayUT+WJdR7wLMM4vXRXB4eoRjxcbdAEA5aKR4jAJUbnZaPSOZYQugxNCFMjw07JUnBlh2fmZ7xlh2foey2cf0mN8OAoc7zu29Lqra21yIZO9Oh67TnSzU1NX/txDH0G9ASt8VTR4HnNQ67yo0tn/+sdCIAoL4xDAAou30s1v3pY/gDISxfcBU4lhicYteMvwiqSqfViG/ZyRnuw61XX2IkpfXdLVnWxDG8qQL+8+3PMffmyyFJClbYVCjTqgHOBkQjcVQ89TJcMDgNPMeCEK2dXT9OjKqHl07PNZTIW8brVJUkAklSTNxiS0sKAD5+X7ljkBUFoqhYuBVlRe11L6IUgeCKnCz8ozZgOAJX5GQhRSBo7qE70a2hN3cQtDV3ZUXFjQXDkX/5EKzcUIX7Z4/XKouaQkaVyKofFaGxWQLHEiPA8nldOFMfwr7Dp1AXELG4OA8eF49VG63VxzXH/VAUmGxYfJDk5rWW2XgHMntQipGodvEMiqeOMonC6JV5xVNHISvDDVVVEZFUsKyK7IEpuO7KYWZO6ZICZPSC5+bAgYP+DcHN4Ux9GJt3fIYp+SOQniZg4AA3JFnGu/uPRyneGPAsgRiRTULahDAg0YRWbBGGXsl58VAvvj7diLVbtYrQn9892TiOVg19/+zxaA5JSE914ecLJ+O/3/sSE0YPwfbdR3Hr1ZdgZUzXyfIFV6G+UTR1oiwuzoPP60Iqz4JRVUBWqcGVXcFFZ2/4O+h9oPlhS+ZqgoMP3nUlUj08CMOgOSRRq+gvykpDZXkhAs0Stu46An+UM7iipACv7/kCM679rm0V7eAMD+obJZN/83BJAS4YnAoXz+GJF1q6YR+ak49brh6JM3VB1DeK2Fl9DLdefQl2Vh/D7Kmj8Pqeoxj9ncEt6zkiY3kcf3QivqVGncbAH40Fe5tv2plwC8TiG1aUFMAtEIRiYg4XTzB76ihrQRdPIMbdLzGi4C8fnNDsLAFkBXj1nSOYdWOOpaqYAWMq1Kj1B/Hkix9akspA4tXKAkcw5+bLURtTQDLn5sshcARhyWwNWRb4WelEnPYHY8RhPWBZbWMvFpIoIzPDbaoCt0sWcywDn9dlqkD2eV3gWHNyzOXhUVsXstz/rAy35byKCqPgRU84bt11BIoKy32VFZV6LC130TtjaJUuQNePwLIM5k0bA5YhRsHavGljwLIM5H7AvZWMnekIHNLdboSRuL2nCE8uvgbLSichwytAklQj+QxohrC+UcSvtuxHxZq9WLv1IEJhGUtKrsTqe4vQ0CThkWffQ/kTu/CrLfuhKjAEX2KR5dOECGM/p4q+bKgCIQRrtx5EWJQx7ZpLcOpcs5F81o+L5b2TZQWMrIBTVTCyAlFWjeTzXTeNxtqtB3H3qp1Y8ts9+Pp0E558cT/W/fkQzgREVKzZi7LKnVi/7RDuumm0wetnVJU4aBURwPJsVnQJJyFjKBjr13lqs1UdtzcgFFFRFwhrzn10TdUFwghFemcFk10HQU9/NyYydz0uDteOv8j4fYFmCQumj8HIi3yoWLMHT764HyfPNOHxP+xD+RO78LPfvQcACIoRiJKMLJ8HNcf9aApGjOBMv5bOn6/bmlgbFh8chUQZH9XUgmWIad5IUstxYUmxiMI8/fIBTMkfgcoNVQhJCk7Xh1GxZg/mP/426gKilVO6Fzw3Bw4c9G+wLEFzWMbmHZ/h1qsvwfpth/DQb/Zg6dq98DeE8cmXftzzi11YuGonztQ3IxiOGPyjz7zyEQBgw+uf4ExdyOST1hz3Y/22Q/g6SpFUc9yPWn8Q38T4tDS/9anN+9EcimDh6l346bPv4drxw7B991GD+zn22FPnmi2C3L/eckDjr27jd8cWXKyrmILK8sJ+l0xzQAfND1u1sQqSpOAnT+/GrzYfQERWEQzL1PisLhA2+Hnn3DzaoGms3FCFKfkjjE7X+O+drQ9BllWLf7NyQxVuKbrESD7rn6/eVI1TZ4N46Dd7sH7bIdx69SUGfQ3PE1ydd5F5PQfC8HndpvO2tU70ZPyX3zT0St+0sxEMW33Dyg1VCIbjk8oq9X6JlNiE5wiuu3IYHl3/Phau3oVH17+P664cRuV1tuVKpvC46i33sdCqlc3HSbKCQJNk8ocDTRIkqi1k0NgcMR3b2BwBja+DF1jU1oVQsWYPyip3agVydSHwgjXdFRYVvLXvGLJ8qfB5XcjypeKtfccQFuP9d/r9D4nWsfIcwZybR5v4sufcPJp6XwWefqxApeHojTE0Q9Ud609gGAbBkIyla7Vc1dK1exEMydTugb6IZOxMR+AkoLsZutN6tj6EhqYwZFlFSIqYXhQzr7+U2g4jR7Q2q3jBlfqmMLbs+BxL5lrFVd758IRJ4CA9TaCKDZBoVcqvtxzAoHQPlfvU53VDBaiigTrZPi1Q+PUWLemjJ2XsEkLLF1wFBowjStgGukvYQFFshCl6sICCHSKygg8++RbLSicZYpwffPJth4Qrzid6o7gFkNi4xYgCfyBsHLd11xFcNfYCoy3s/u+Pp9rH0/4gCGHw4Jwrsax0IoZnp6F0eq5JuKrWH0R6mqAJBLaxlSsrKjJ9qZbgIFaEULZZIzr/qCyrJpvHMKAe39OfmwMHDvo3IgD8gTA1wav7eDpEScGqjdXUY1wugiVz802+6pK5Bdiy43MALcKDaSkCHl84GRPHZNsKu2UPTEXOcJ+RtNHb3eOPjfdnc4b7UDo9F0MHpUIF2vQ14wsunOSzA8Den1EU1dgIf/rlA8gc6KGKYKtR4oJafxCvvXsUAwe4UVleiNLpucjMcKPq8EmL+OGSuQXIHpTSqi9F+zzD6zL+rG+S1zeGUXuu2Xatxp+3NejJ+NZ0M/oTEhUMTEZUT1Howoa0mEznSo6FnWAgTWRT7/KIH2t8/uE/XvqQOlZZVqjHypRjRRnUZDGNmS6iKAg0myuYA82SJbGejGCjbHNfaccqMvDauxo9g75WX3v3qKWqG+idMXRvHHNnQ6IUFlVuqDIVH/VlJLMeOgKH7aCbwbIE/iYR67cdMtobHpqTj4ljso0WLTtnW4ooRqI4FvWNIvyBEIJhyWhLCTRLeOFNTaxl/OgsrCwvRCSiwO3iqGID+u6dz+sGSxhkpLmwclEhVKggDANF1fhOH462ZU0ck43508aCYRiQKLVGls9jO3ZdNJHWunLxUC+euLcI9QERy59roQbpj21bicBOBLKzOQk5jqDs9rFoaNRUKHiOQdntY8FxBCplF7kng2O1Ko9YMc4lc/PBcQzVcejp6K450NlIZNyyrKC+UTQfx7S0hemUHLGo9QdxUbYXLMNAiigINIl4avMBQ6xKp93I8nkwOMMDgWhtf22NdejgVHqAGW3N06sh4u2poqrG74r9vqKqtvbX4YF24MBBT4WsqKhvFJGeJlBtYmaGFrR6U3gMztDsGmEYw88DgEsuTEdEVlDXGMK9d1wBgWcxINWFxqDmw9KoNh6ak4/0NLqw7KlzTbjrptGGfR+U7sbZ+pDl2Fguf9o1HF/TAZA4d7FONUFkFWsfvB5n6oP44399ZvgYZ+uDBodmzXE/GhpFqgh2yS2azo9Ojahzo+tzsnhqDs7WB421kp7qwu6P/oHLRw4yur1ovpQt924U+kZ8faPYZswWe97WfEs98W0nwtjTfdPOBmvDgczGcSCzNgJ8NGHB1jYdrMJ+dK5kjrXGPDRaTdrcV2w3XKzVjLJiRwtKp6qgH6tYjnW7WNxU+B1LLOd2sYiEWsab6P3Xfr/NfZWtY1UZOkWFylhpNXibGJrnCJQeGkMLNmMWOAK5h465s9Ha5kV/SJomsx46AqfEtJtBa9lavaka824da+w+6s5yLLJ8HpypN7cl6thZfQxL5ubjtXePgudYPLV5P1ZuqDKSL+v+9DFOnArg+dcP2+5sSJKCnOE+zLl5NJau3YtFUXoPQGubFCXZ2M3UnaVHnn0PCyp3omLNXi15vOAq27EHmiUj+UJrXZFltVXKDwctSHS3uqMgAEJh2dRCFQrLvdJoyIpqqfJYtbGauhvfG9Bdc6Czkci4WUKws/oYHpk/Eb9bMgUP3DkeUFvawmhtqbddOxKNzRKW/HYP7l61E2u3HsRdN42Gz+s2dVlUlBQgVSBtJp/1saa4uFarSBSZLt7CgNH4/lzE9H3bChYn9+zAgYMeDIEnSPPwGJTuttjEiWOywTCM4dstXbsXgOY7rt92CCW3XI6y28eiYo1mn598cT8YhsEfXjuMZevegyjJuG9WHoqnXmaprl69qRonzzThvlnmCtL7ZuVhy47PTfY9PU3AzupjlmO9qQJ+8oMJyPJ5qF16jq/pQKeQ0CkCK9bshb9ZslTHsyxBkyTj69NNeHiNFis988pHKLnlckwck437ZuXh9T1fID1NwL8Xj0dleSE8Lg7+QAgrN1ShYs1eIz7TKznt5uQXXzeY1srP1r2HyeMuxK+3HMCWHZ9b5rleVEGrtj5bb07EDUh1YeuuI7YxW0iMGH9OxLfUE9+6sGhv8007GyzLUDo98rXEcuxxhP68aHSUhLGhyqBUNTPQkrWLZo5DZXmhVpzmYjvEPcwRQr0+Rxkrx9LHyrHWY1miCTbGVt1OHJMNNp4DBIAUocdyUsScHHW7iKn72+DgdlnPyXP0sfIc7W7ZUVTQnwEthu7pRA60MfcnsCx9nsev3b6KZNZuh67TqWdz0CbsdjCbghJ+vnAyztWHoKgqHrhzgkm99v7Z47Hh9U8AABVzC0yChcU3jsIbe7/AlPwRSHVrCsfNYQkDUgT8x0v74Q+E4E0VMO/WsaiLaW2PvX59k4jiqZdRkyOl03PhFjijevmirDScOtcEn9eNWn8QtX6Nx3TFokKkuDksmVtgEsZYXJyHTW98alE118+/qryo1Z1dZ5KakehudUchyarRaqRXbbz27lEsmDGu1yWhWxPZ6I3zK3YOgGEAVe0VSuOJzF23QDB/+lg0BSP46e81BfhnfnKd8fy27jqCB+dciUBTizDgxRcMwEPP7LE4haXTc7FyQxUuHurFstJJeLvqK9xSdElCDqAsKwBHcN+sPKsgB6NtXERsdsoHZ3iQ6mahKioqSgqMdi6BZ+ntlpRKDwcOHDjoCVAUFfUBTZfE53VbKutKbhmLZeveo/qOKzdU4anN+7Fo5jhb+0wYBlWfnMTNRSNx/+zxRtW0zgfNsQR//uvfsbK8EKejm5B61TOgUcvdNysPYUnG9Gu+i+zBKVhZXoj6RhFn60P4z7c/xx03XoaV5YW2VXyOr9m/YaerUVleaPIXIgBOnWs2hMX0Y5/avB+V5UUAo2LBjFxs2VGDgjFD4U3h4Q+EcP/s8QYfrJ4M27LjMwCw7SrQq5Nj1wpLYMRduoCnN4VHps+D5nAEO/52DJO/d4FJpC3D68KmN7T4UU8Ib931OTK8Ai7MTMNjZZNw8mwTtuz4HP5ACEvnFWBQuhvrKqYkHF/EinW+8OanWDRznCZEzxKwUHu8b9rZkBUVuw/8wyTk9XbVV7jlarP/KUUUanX8T34wwSr4xag2/qj1+mJEwbo/fYyZ118Kt6Albtf96WPqeRMVNedZuiArzwJxGoQghH4sIUD8TKCJeVeUFMDFMxYajkRjuZAoIy2FNwkbKlAREq0JYI4BHp5XYBKufXheATgGiJdBtK8Ct1ah98YYujeOubMh8MQUt+nzUeAJpEjfT8a7BUJdu/ECqh2F4291M+zao9wCB1WF4aDoHHXpaQLSPAKefvmA0d7VHJawrHQSQmIEboGDSyA4+PczeLv6hOmcleVFuOffvgeOJXDxBIoKantils+DjDQX0lPtnSBa63hsa3utP4hzDSE89Ju9prFnZmhCBg/cOR6AlT5ET770VkqB8wVZVsAguoBt1Ns7Csam1ag38vDrO5qJtLn1FuhzIDPTi9OnA10yB7oCbc3dcFACy7EG57M3hTcqP/TnJ0lmZWmdpyz2+eq2K8vnwVcnA1i5oQoA8C+TRybx4mOwfbfZGdu++yjKbhsHoDVKERhK25npLpMT7Ng5Bw4c9FTQaAjqm8JGh1qtP4hNb2gJpgsz0yBFVITEiK3vqP/ZLXDUf8/yeZA10IOiKy4yKN5i/Ut/IITBGR4smJ4LWVZNok6AZj/TPAI2vXkYC2bk4oLMFCiylijISHMhw+vCyAtzwbEMGFlFxPE1HVCQaBGMrKit8BxrPOFgzTFZznAffjjtcqwsL4KiquAIA54nKJsxDvNuVYyKs/g5qVdIx64VWYFxbM1xP1ZuqEKWz4Of3z0Zv33lf1Fz3I/xo7OQPTAVhAFkxYX3/vdr/POkizH35jE4da4JGV4XZv+fHASigvZGkmVuAQak8RAIg/Q0N2SxCREAYUUFy5JWE9H2xQVyr0yykssAACAASURBVPFNOxMsASaOvcBEFfHj708AG5eAJQyM6ngdWlWz9ZwMGHz46beWpPatV19iOTaZ8ya6+SLKKjbH+OWBZgmbd3yGshm5lqQuAwayqpg2QmRVAUPJltPEvCsp19fua4LUGipQ9fFJFIwZCkCjv6s6fBJXjR1qub4kyhiUJmh+uqqCZRgILJ2mj2FAvT4tLu6NMTTDoNeNubMhSQo++eIMViwqhKqqYBgGVYdPYlLuBed7aN2CULOELJ87GrdqttzjIgg1x2/HdAz9ZUOjx8CuBf3tqq/w1Ev7jVYcXRmc5wg2vXnYSD5XlBTgtXePoqEpjHWvasrhgSbRcs77ZuVh3Z8P4tuzzfj1lgM4fioAhoGlPXHimGw8vnAyGppESBEVE8dkm8Zr0GdQWsf11kf9uPooZ1DNcT+27jqC+kYREVmBLKvw8ASASm9riDoqvZFSoG+j76jhegR6O5ZHcExgTwPLEgg8wfxpY5GeJiDQLOHtqq+MdsaZ119qUZZeuaEKxVMvM51HbyNdMjcfVYdPGp/R2gXt4OIJiqeOMtEGFU8dBVeUMz8RuyVLChAVG2JZYKlj5xw4cNADYUdDEF/1VXPcj73/+w0IYcASYECKYEu9pv9Zb+nXMXFMNtLTXHi0bBJEScHqTdUWX6N46mW4b1Ye1m87BDEiQ1YUS7v6/bPHY9ObhzF76ii4WAZ1ARFL1uyJ0sPtQaBJBA9AEWXIskK12UtLCsCyjgB2f4a+mRwLI7EVd5wdbYV+rD7HJo7JxopFk7Hw9nF48sX9KF3xFh5ZuxfNYQnfnG5CxZo9WLhqJ5778yFqDKdzp+vrZ3FxHl5954hlDTxcUoDGkMajDgCvvPU5QmIEP1v3Pu75xS78/R91uDAzDYQBLsrygoUKqNakY+XGKjAqA0mUoSgq1RbwAguVJXQxekes0wAjqxiQypsoMAak8mDiNrmSiX1dPME144fh0fXvY+HqXXh0/fu4Zvwwwx9t73nthefMx8mKirqAaPqsLiBSxckkWcUrb30OKaL9mxSJ/t1GXDGR62v3gKHGci6eiTuO4PKRg7F0rTZ/l67di8tHDqbeKwBQZBWAGl3DavTvVgg8S6VWEXiaqnnvjKF745g7EyqAy4YPNM2dy4YPhNqPkvChZgmMLGPo4FQwstzpyWfAqYDudsiygkFeTd3bHwijvlHE5h2fYfbUUbjl6pFgCWNQUrAsA5ZlUDZjHOZP13bl3AJB2YxcEJYxVST/6sfXmnYlX3jzUwDAiKFe/PvsPHx7tglbdtRg1o05ePmtGpROz0XWQA+gwrQDvmRuPgBg3+FTJvqMH04bY1vhor/YNkfbyexEXv66/4SlfUh/IXYXrYSDxJFMq1FHkKjwS0cQFGWkewVTJSrDap/3o3dKrwDnYnHaHzK1/9w3Kw+7D/zD2JGmzcsLMtOMygTdtoABNv/PZ5iSPwIH/35GSzSA3jFAm4fNoQg++ORb0074Ox+ewLXjL0rIbtFaG5cvuAqryouMzg/Hzjlw4KAnwL4SrshU9XVD/jDcVPgdkyh1fMuq7jvqtlhFS+XYxDHZKJ46Csuf06oDV/+oiGrTL8z0gmWBshm54ACoLAOOZQ0qDRVAc1DCghm54KFV6LVVyRdrsxUFUXE2FV9+02BQEDiihP0PsRQSlvgk7rjsgSkWGpqlMcfqcd7sqaPgD4QsxTun/UEThce+w6dwweBUPL5wMgLNElLcPDa8/nFL4dHcAkQUBYqi4F+nXAaeY/BY2WQEgiL8DWF4U3g88cKHphjwzb1fapQgUFHfJJrivKUlBUhx06u4I4oCF0twtj5IXUuPL5xsOpezVuiQZQWpPIvh2d5WafKSiX1Dol2lcJEljknmvDzPUsWxed4slmcnuk0T0batpCWwiAAKNtcXeKvwnRiKWLoKXTwDMWTe4AxLqqmLMtAsYcuOz1A2Y5y1qjpBChIAkCOKkUPRz/vyWzVYGO2KjEV3xdCdCdVmzKpiFVnsqyAM8Is/fmBaZ7/44wdY/aMiOD1SnQcnAX0eIMqq8QLX8eU3DZpDHDXiLIBMn9Zab7SrA4iI2vRXlDjxq3NBrN92yPi7Lij40xhH4b5Zefjw01NYMCMX/kAYqW4eS9fuNS2yVRursax0EubdMgan64LI9KXgh9PGID3VRW07yfR5UFleCIFlMHvqKHz5TYOtoEbp9FwTZ1mgWYI3lYccFQ9IllaiOxKX/RnxtAeAveBFe5HMi79DUBn8butBTMkfYcy9ndXHUDbD6jQ4OD/Q13MorFgCn6dfPoB777gCdY1h+Lx0W8SxxHBKCWEQFCNobJIwJX8ELrkoHZXlha2q2tPmYYZXwFtVx/Hi/9SYrnX9lcOMdu3W7BYtobP8ub9pY1HVLqPPceDAgYNkYUdDUNcYNpJzPq8bs27MMfmO+w6fAgBUlhfhXCAEBsDAdDceuHMCpIiC9w5+jctHDsJjd0/WcjEK8LMYzuj6RtGWyojR/UMAKggeeXav5bjK8kIAWuUSjUPaQqMgK+BYAn9Ismx0vvDmp9T2cwd9G4km7PTEYkpmanQjBOAIY+E51jdD7p893hSXzbz+UgwdlIrS6bnGHAWAV//6BW65eiQamsIgDDDnpjH41ymXQZIUZAwQUHsuiDSPC89v/9hYb0AL3SKNbmH2VOD4qYCFr3oFZVNJ/w5LCJqkCALNEtUW+GN0hOyoGhxo6GyaPNlGc0S20RBJNKaWbcSxK+8xJ7btRLdXlRdRz0urpK28x3psJMHr6xBDkZjfJVt4ogEANvQXun6L6fpIjIIEACRZoVaBS7L1GXRHDN3ZYGzGzPTgMXc2pIhqaJ7p+YKtu45A6qWaUT0Vzr08D2iv4F5skiTWqQFgKA/rxpYmKPj0ywewrHQSztaH8OSLH1rOoR/XGBRR3ygaBnjD659g4e25lh3/h+cVgGMJZEWBKAMD0gQ8ce/VkBWVGgSkpwkYlp2GCzPTwBIgPc0FniOIRJJPNHZb4rI/g0HCghftRTIv/o6gN3Jx9Sfo6/mv+0/gXyZ/x2SXbsgfhu//8ygoCqAoChiGoc5LRVHBMMDvX/vY8qzbsg1283BVeVFCVVF2cMRVHThw0FsQy2mvJ8vS0wS4BQ6pHhaV5YUINEs41xCCz+vGQ3OvRHqqy0jYsSzg87pwtj4EUZTBcSw2vXnYSJjdkD8MxTfmIAKlVf+1tepTmj0WWAZnAyI1mewPhKj8zjSbHyv25tjo/oeEE3ayAsgahyUBANnqD+i0AgNSXfjVj69FqocHA41f9HRdEDurj2HBbWOhqoA3RYCqqAAYbNnxuZGU1vHskinwpghwCQSl03Nx+3WXor5RxM7qY5g9dRRUqHi0bDIamsLG5/92w2VgWQYXZqZRfRAwqmUt6dSN067ROIVpiSidajH2XM5aaT+SiWU5Gy0bjiUARRws0SItWVZwY8Fw/NOEYaZOv3hhPzvRbZqItiwr1CQeba4kKxKf0O9S6fQXlZRkuaKoGPfdwbjtny41uLVffecIFAWWSuVkqsDtRSN7cB1tN8T9PR08T1B2+1g0RG0dzzEou30sBM7cEdCXkZLCoyms4OSZRrAsi1QXQXMn03A474zzgPYK7sU6zIFmyXSOmuN+bN99FMsXTEJ9YxgZXpdtcpnjGCydV4BzDXRBwgGpLvzhtcMAgJAYwdybL0flxmrTyyQkRtAUlLDy+SrcNysP23cfxYxrvwtBIFi98QNqEJA90IObCr9jEmWoKCnAYK9AJftvDd2VuOzPYBlQBdgW3T7OomLcXnRXgk5txRlx5sv5RwTAX/efwNV5F5lERm7IH4bbr/suTp5pMhy+ZaUT8d/vf0UVBuQIg3m3jDVV1yViG+zmYURpqYpqrY3SDo64at+Dd4AHbpfjOjnoe9ATvJt3fGbZxFsytwDpqTxWbqjC4uI8/GjW9yBJikk4sKKkAADw5IsfWj7zpggabcfavbj3jiuo/utjZVqFNM8SS0UpYF+lSqPeePrlA1g0cxx8Xjd1w9DO5uu0co6NdtBesCyBygBzbh6NTW8exr/dcBlqzzWbklYP3DkBLoFFQ6OIn/2upVP1x9+fgOe3HzaS0Fk+D06eacSWHZ9bEl8PzcnH4S/OYMzIwaYE5pK5+XAJLL4+3QhRkqk+CFRE11IRztQHUd8oGqLyX37TgHvvuIK6KaRTLcaey1kr7UcysSxLGEsh2OLiPLCEsdi3ZBLbKW4OV14+xOhq0e12ipuFGGw5lrMRAOQIAWTzCGxpNShJPPvEulUkPtHfpdhQ9Smqlf7C5eIsuYklc/PhcrGIhMxjVVV6FfjqH1kT24yNiPndFLqOngJiE/cvvH1cv6GfIABCYdkkdL+4OK/f5ApSUnic9IdMlGoVJQUY6nN3ahLaiaI6GbzAQpRhcBPRlFQT5RoDzDt9BIDP60atP0itGJl2zSX49ZYDqDnux8/vnmS7e71+2yH8cvE1SE9zUcex429fGtxj2QNTwLEEtf6g8WLUUVleaKoaeWrzfiyaOc42CJAVYNVGs9CMHX9VW3AqC7sBDGPMqVgj3Jllw92VoFNsdu57MhdXf4KsqLih4GJ89PkpFH7vQsMu3fZPl+LUuSZTCyktENOdcBUqCEHStqG1ediRNkoOwNJ5BVjxfIuNXTov8QpqBz0PbheHWx/YltR3tj85vYtG48BBx6H7mWFFxeAMN8pmjEPFmj0mX23VRo3/1XhvMgQ/37jP4s/F+4C6j6dCNZLVhJJImXbNJXC72Kh4mdxq9Wl8larMMFSbf2GmFy5i3jDUfysBvcIzJEaS6nJx0LeRDNVf7DoCtERV6fRcBJpECw3Gky9+iOULJlmSWf/x0odYNHMcHl2/z8SlPvP6Sy3Hrt5UjRWLCk30i9parTbO7fO6LbHi0nmaPkZE1RJ1D/1mj+l31PqD4FiCDa9/gtLpuUhPEzA43QOBBb4fpVo0ztUNayX2GfgDIbAs6TOdrsnEsmFJxqY3zDSWm974FA/cOd5ybDKJ7bBkwy0dR4FBWOChuVeioVGEW+AQEiMYkCaAsBo9RywURcVr75oTma+9S0++JpNYjwDYHMftvHnHZyibkWseK6MJ3cbTLtI0yCUbXuey26x80VKE/rykiPV5EUKPoQlhOq2Iq7NBCIOZ11+KX/6xZRP5Jz+YAEJ5Fn0VkkynhFlZXoT+IE/cFE6ca74jcHJ1nQheYHEmIFp2DeIrfBPlGtNViGMTxLozUnPcjxfe/BSLZo7DhZleEAL84oUPUXPcj5zhPnjcnMWg69XIPq8bZ+tCqNyo8fktmjkOQwal4kxdEK/v+QLFU3NwVe4FqG8UsfGNT7BgRi7VUdcVzvWqkVp/EG6BM1o3dUM+fMgAcIRBWJJtXrR0/iqa42f8m1NZ2OUQJRl/+eAElpVOMrUl3XFDTqcZjmQ2YzoC1maHnaXssDvoPuhr3COwcKvA5SMH48FndmNxcV60iodg6KBUSyvfXz44gcfungx/QyjGCZ8Al6C5B8nahq6ah4RlwHEEi2aOMxx2jiMgLBNfMOLAgQMH3Y7YijKf1405N482ig5iof/9tmtHYsigVEMMNt7fy0hzWb53pj5o6sojDGMktyyJFDV5/82uMk9jhW4JmeJ/q4VWLsr7z8jW6msH/Q+JVFvqPoyiqIgoKn7/2seoC4j499l5RmwE0DfFCUP/fMigVFSWF8I3wI2nXtqPmuN+I8aKXW8RWQFhgJJbxuCWopH44399ZlAe6ueu9QdN2juZvhS8secoXv3rF7jt2pH4l8nfsd2IqTnux/pth6KCzipkMOB5sz/D812bkunrdIscIdREKa2qmCUMleub5tvaC+BZaSXsKTDMsbmiAiQuBUXAQKGabBsOZgqSSazDRtwwPjPm4gnmTx+LSEQFYTTKz/nTx8LFE4iWeZP4WAmhxxeEIq5oH0Nf1mOTb6KkoD4QMok8fvG1Hz6vu98Ua7UmHtkfEtCykjh9TkfQU9dAr4QoIymF2ra4xuqbwpYdzF9vOWDsjvsDIQg8C4HXkhn+QAgAMPP6S/HEpg/g87qxrHSSwemst1ctK52Iyo1VhnOi77SXTs/Fwb+fQeH3LsCj6/cZ47j9uu/aJrOBlmS0boTvumm0yZA/NCcfr7yt7Sba8VdFZMWUiLcVBcvQ3nTdlbjsz3DxLK67cpipLWlxcR5cFGXi9iIZpeaOgFZx1dN3ovsyWJZABoOQrEBWZTQ0S3DxrKFanZnhRlNIxsNr9mJxcR614vlsfRBL174HoMUJj0S0ADARPtFYdNU8FGXg0ef+ZrF5DvWLAwcOugutVXHGVsqVTs81qjapSSlJxjXjh2Hp2r0onZ6LiWOyKVQd+cgZ7jNRCNQ3ivCmCMY5A81SwomUREBYYMncfKPDTvdRf//ax6bKuNjfWusPYtMbLUUcLNH8SjlJOjgHfRdtVZHS4hRdW+Tbs01GbMRz9IIZRaUnsxhoGzQzr7/UiOsiskJdb3pRkj8QMlEexp675rgfKzdUIcvnwaKZ4zBh9BB88qUfU6+6mOovLZ1XgEHpbjxXcQNUaD7VvsOnsKx0oqmSWx9vV1If9nW6RUEgmD11lJXXXiAQg9bu6eULrsKpc83GBkD2wBSqb0tsNuUIYSwHszbHsnFJcMIwaA5HLNQEvgFuSxylwob2kCIsyBFCfR/QkvCJcjsrYCBKMk5HC+NCYgSZPg/cAi3tZT/WeHAsocaSHEugxMUL9jE0C1nsmZVPKW4Wg32pRgdULB1LuLl/RMscS7fXNEqYvghb+hy+czmwnQR0JyJZhdq2IEXo58semGqIwei7hB6eoKKkAJUbqoydct2I3nXTaKNFK8vnwZBBqdTzpqcJqJhbgDf2foGHSwpwYVYq3DwHWVXBEmDVvUVQZBWKoqIxKOEH/zIKAs9iQKoLO/72Je6fPR4+rxuPPGvmX129qRql03NR3ximGm4VKr46GcBpfxMmjh0KhWGggsFf95+wOB2/XHyNdq+TSBgl00LnoAWyouKrb+qxYlGhIUxRdfgkhg5KPd9DSxpiMjvsDroUsUHbjQXDcf2Vw7F83R6sLC/Ej+64AqKoQFYUpHl43FgwHIqi4plXPrJsxP178XgAaHHYWSAoKdh3+BTqAiLuveMKDErXHG6XwGrlB60gUQGiZNDZ7wQHDhw4SAZtVRDqLeA5w30YPsSL+2ePh6KqeHzhJHAcC0VRoShaUEYIgwef2Y1av0YD9++zx2NZHN/+qo3VJgoBPSk2LDvNKBrYuusItfq4tU3C1vw4UdJst/5+j8gKGAaYce13oUJLGsT+Vh01x/14dP0+rKuYolU9d+FzcND70BY9Ai05un33USyYodFu6MU3/3bDZZb5vmzBVRB4gkfLJuHbs03YsuNzZHgFzLt1LMJiBD/5wQREFAWV5UWIyAp4jqDstlxU/HavxRfShTOffvkAVi4qBBjAJbB4fOFk+AMtwoS3Xn2JkaB+7O7JIAwMf6l0ei58A1xIcfFwuwgUFeB5Bg8+03I9t8DZ3g9XF8VZfZ1uMSwp9AQ7JVELaCKWsQngpSUFAE+pTU1CAM8tECxbcBVqYxLbWQNT4BYIwjFJ8IisJExNYPfcaLSHSVF7qCq1OjOe21lRVNQ3ipZkeZpHsPz+1ipe48fKswwyvC5TF0CG1wWeZRCm3IPeFkOHxMToWPoyOI6hFzhyBBGK2Gdfg6rS5+0Fgzt33vYF+91jkOguYqLgOfr5/lHbaOwU6lUjkihjsFcwuPZid75fePNTLF8wCYFmrRL6TF2Qet7B6R7UNYVw3ZXD8Nq7R6M77Vr13sQx2SieOgqVMW2aelJIX5wDvAJkG34k3wAXwiI9CVhyy+X49MszuGb8MJOgzZK5+Qg0S3i7+oRxHimiGEYwkYRRX2/f6kpwHIPLRw62CFN0phHurufD2uywt3dtOmg/9KDtxoLhuPLyIUaSNtXF4luK8EGqm6XalEEZblSWFyIkRrSdWVk1bDAAMAyD5c+9f17XfWe/Exw4cOAgGbRVQSjwBKt/VAie4wwxtAXTxyDD68LpuiZLkljXIak57kdjULQtklj9oyKkeQQ8/bKmS+IPhHDXv4w2AndFVXHvHVdA4FkMTveAo4gO6mjLT2AJg7rGMNZvOwSf1427bhptHXcKDxXJ0zM56L9oi+ovPsmWM9yHW6++xIhjJo7JRsktYyHJMgYOcOPxhZMRkVU0hSQ0NIp4avN+Y44uX3AVwqJirMGJY7Ix68YcLNtoFm3X15+OWr+Z5uN0XRBPbd6PJXPz8fJbNdh3+JQRT72590ujM8HfENISjT4Pao77sXXXEdx102j84oUPqOsdgNHpGn8/BJ7AHxC7xI/v63SL9vQX1gR7BMCKOFu+wq4aXKWLyZXNyLWMQVVVSBFzYvvheQVQ1XhaDzplDI2agDA21f0UDSFFUSGK5uvfP3s8FMrU4TlCrc7k48QNk0mWt1bxqsaNQZEVpHo4DM8eYFBUsBws1c9A98TQnY1k5mPfBQOPm8XyBZNAGI16hu0v/CMAOI7YzltJ6rx52x/oTLoNAgtUlBQYCRD9oQntnLjpqZpIYPz5dlYfM/6uV40AmtghI8vgAdP3/IEQmsMSFEXF+m2H8Mf/+gyLi/NM5324pAAsyyAc1oz2lPwRxsursrwQc24aYySGaGIYKzdU4dg3DWAYGOfVkeXzwOsREBIjRhKwYs1erNxQBX9A43C9oeBiy67bqo3VuO2fLjWdx8UTqCxBhGGgsgQs2/oUtgu+9C4Klk3ufP0JUkSl7oRKkc5L4LX1fDoLnb02HbQfsqJVMFx/5XBUbqgCwzBYMH0Mmm123lnCUm1K7blmVKzZi0fX78Py5/6GCLRKjoqSAhRPvczSUtcV86otOPPOgQMH5xOtVRDyAov6RhHNoQhWbWyxvQVjhqL2XDPVzyueeplxHn9DmGqb6wJh+LwuhMQIZl5/KSaOyUZFSQFYoglpP7V5P5aufQ/PvPIRXDzbavIZaNtP4ABkD0zB4uK8Vm2/Tt0W7/v2n8DaQTJoa77oyVEdM6+/1DT39h0+hWXr3sPZuiDCooyyyp0of2IX/A1hI/kMaHP01LlmgxoRAKbkj6CKtseuP31Muh6P/mc9fpqSP8L47qqN1SgYM9Q4ThAILsr2YmV5IX5XMQU//v54bN99lLrec4b7tITyABceu3syJo7JNs6ztKQAitx1fnxfX7MkpmhCh0GVEYfWbHk8OACzp47C+m2HULFmL9ZvO4TZU0dR71tYUrHy+bjn93wVwpL5vDzLUMfKs9ax8lGqitjntrg4j3qsqjKW9fDU5v2gyQEoCl0gTlESTJZTTsowDHWstGS5KAPb3jmKkChDUVSERBnb3jkKGnOTJNnE0FLPLX7T9ZJikeWL6iX1E0iSgp8++z7Kn9iFhat3ofyJXfjps+/36OfWmRBtREnFTv79fcWG9wjEViHrO2MCC5MAYTIghLHQTAgsg7IZuZg/baxtm5OZngL46mQDnnv1YwAtLYpDBqdiVXkRIkoL9zKBioHp2m53ZobbxDW2+kdFxmTUKT5iUesPGtxK988eb9rZX1ycB0VVMHyI16AJiW0JeuHNT3F/VLAj/px8tBViZ/UxzL35cpxrCOPxP+xLeJe9tRe2y6mObhXdsRPaXe11prWpqmAZpkNr00H74YryS0kRTZT0tL8Jl48cbD/fFMXSvnr/7PHY8PonccepCAclZGW4keqm26jubtvs7HeCAwcOHCQDuwpCjhAEJQUrnq/C/bPHo9bfInCmqqptu/3QQanG+XZWH7P4dA/PK4DAEYOKTferBqe7IIYi7eLab8tPkGUFqTyLlMxUiJLde0QFo6rdojnhoG+gLaq/eC2a9DSBOveGDNRE3nWxueFDvJbj4tebXZx1weA0Y/3FckDHxlP6sXpldOzfs3we/HzhZI3iMWaTSf9+XUA0qqRr/UEMz/ZaKk6XzM1H8Y05qGsMg+dJK1RjHfe34p+B28VBleQ+s2Z5lq5Pw1OEqpOpBk+GpjLR50dUFUvnFWDF8y32fum8AhDV2oFMoMIXR1Xh87pAYD3W9voqpQrcJk6IxMWldrzrPKWq2Z6icYLl+iwLXJ1n5nVeMrcALMdY6EK6cl10FTwCodJPeASCUA+t2u5s9Mbn1pnorir4/nAvuxWSKMfQQsjoaLV6PM2EJCMhnlLjeyxr8D8DMIQoVpUXAbJsOo8sAwLHRluqOFTG7L7XN4qGMbdrwwqJEYiSAkJgvHR0Q+4PhPD4wslQVAWLZo7DBZlp+OZ0oyGMaNcaKUerth8uKUCqm8f/jfIPAomJUbT2wu7r4hYdhb4Tarl3nUjE353tdfraHJrpxenTgQ6vTQftgxytYFhWOglZPg8yfalYunYvKsuLbOkqOJYx2rV9XheqP/nWCJJajtPmTDgo2c/d89C22dnvBAcOHDhIFHaCzYQF/PVhw6e77dqRKLriIqzeVI3K8iKjPT/ehhLCmJIKHGe2zS6ebdVPaw/XfiJ+giwrgNyK36LTJnQB17+DvovW5os1yUefeydqG1F1+CRm3ZiDVRurqSKf8evNLs7iWILHyiajvknrMgDD4IE7J+Dr0wEjntKP1Suj9b/7BrixaOY4sITBN/5mk6BgrV/TDNL5pPXvAIyl4lT/DXo8ae+7dY6/FfsMfF43Tp8OdPicPQUqVLhdrMmmul0sVNCrmqnctKBz5ydq6+yp4syChbKsIMOjzXcwDKCqtkltSZThdXNwxVBV2BVfcDY2myMM4tUNEx2rXtUcn9hnGMZyZ1lCkOE1c0NneAXLOQFNSia2W0hbD3SO5O6IoTsbYkSFHM3T6PNRVhWIkd5Pd5MoWJYYm4X6hsTO6mM9+rl1Jrpr3jp8A30M8ZQSPAssjWtfWjpPCz5otBOsquLhKKfvWwAAIABJREFUkgJEZNk0+bbuOoL7ZmktKrqATHy7SvagFHAcgwGpLjy6fp9Bs1Fz3I9afxD+QBhpHgGDMzwIhiPgOdZQeA5LESyZm28655K5+WgOS0YAI9lWSNobxtbat5JpZ+qPcEWFLePpA1x855mNvt5e58AKvYLh1XeOYMncfKiqtg7rm0LU+fa3j7/BQ7/Zi5/+7n089Js9eOTZ9zBx7NBW54wzrxw4SAyiJCMz05vUfw56D2KTZOsqpqCyvFDr8lJUpHkEPLn4Glw81Itbrh6J1Zuqoz6QjKwopUW8Df39ax8b/t2j6/fh5+v3IRiWDdscluRO96uSseeO7XfQnZBlBYysgFNVcFCpc29n9TEUjBlqUGrExlP6cYMzPPjx9ycYn+2sPmaJie6blYdAUATDAAJHcLY+hEfW7sWTL34IgW+Jp/T4KZau8f7Z4/E/73+JCzPToCj2HQ7paYJp7IpNNWAs7zQY+u9mWcahNWwDoqTgv/Z+hSxfKnxeF7J8qfivvV9R291tbXkHq8E5G6o4jkK7oM/3LF8KGFlp9dqKrAJGuleN/t0KltApMFgKDQnPM1gy1zzWJXMLwPPmY2OrmivLC1E6PReb3vgUIqUCxO0iKI6jKymeOgpul3XutlYdGo/uiKE7GxFZweqNH5je8as3foBIH+BbTxQegT4fPELPfW6diRSBPm9TOvn3Oz5ZH4KdUMsgr2Ds0HOEICRF8OAze6i0E/oLLgJzxUnNcT+27z6KyvIinKkPIiIreODOCfCmaLuEgkDAEgaKoiKsKNTdk/pGEQCgqkBGmgubd3+G0um5uCgrDQyArX85gmWlk8ASbZfx1XeOYEr+cAC60EHyAjKttSH1pCrJnojmkIwPPvnWpIT6zocncO34YZ1HwZFEm5iDvgG9guHt6hPIHpiCa8dfhCyfB2fqwshI46N0FdpcAFHx3LbDpu/rtqC1OePMKwcOEoPAs7j1gW1JfWf7k9O7aDQOugLxlXBgCQJNZl+xYm6L4NhXJxtx2t+Eyd+7ACvLizSRKaKJ8ew7fMp07vhkFOmCrqak2skd2+/gPIE29wSWweypo0wbM7o4fOn0XAzPTsPxU42IRBQ8v/0wFs0ch4uyvCAMg9f3WEXkpuSPwMVDvWgMSoYQfK0/iE1vfGp0lwJaVen8aWNRcvMYuAQWhADXThiGR559D6XTc20pCgane7CuYopBdREBPe6K5Z2GCvhSeKwqL4IkK/jmTCPWbj0IfyDk0Bq2AYEnuO5KM6XD4uI8TVibUi3cFR0culWOrXrVPm9/HJyMwHzYlgJjvCXWZBQVaSmcSSCO4xgwcRucyQjPh8J0ztvKckpVc4IV2ADQHIrYxNAX9djkWzJCk30VdvNhVXnReR5Z96BZVCBJERN15Nn6ZjSLSqeyA/SX+dQvYEcpIcqqsUOvQsXy5/5mOSa2ql6WFYBRLTv0t159CZrCkqEu+9Bv9mD5c+/jdF0Qz249iIikgpFVyIpC3bnfWX0M9Y0i/uOlD3G2IYgZ134X67cdQn1jGCdqG3Hw72dwzy800vd7frELB/9+xuTkcITBIz+cmHRlS2yFQuyOrVMp0zo4lsFbVcexYOXbKKvciQUr38ZbVcepu+Idgd3zcdA3ESvMN+7STPz+tY9x36w8pKcJePA3ezF/xVt45e0a1DeFcfxkwFifOnRnr60548wrBw4cOLCC5itWbmwRONu66whGDE3HQ8/sQaApjFp/Mx5esxdffdNAtccmP41lusSvSsaeO7bfwflC/NyTRBm+FB6D0z2mtVNz3I/12w7h+KlGrN92CKfrQqg57sej6/eBYYB1fz6ICaOHmKrw7rghBzurj4EQBpkZ5iSY/l2owE+ffQ/zH3/L6Ejgozk3XWhu664j8KYKlqrTpfMKwEEFp2pC0bKsUOOkxcV52LrriLmjVFagQsVPf/ceHl2/z+h8PR/iz70Jigy6qF43cgMpMlC5ocpU9Vq5oapDY0hGYJ4ljJEs1jun/YEQtQJalFU8svY9k0DcI2vfgxi3uZmMALii0ruhaYKFPE+o3do8paqZJfQYmva7ego4G6HJzo77ezIiNl0fkX7SHa8oqhGLl1XuxPwVb+HB3+y1CH12FE6urQ8hEUG3hEXfVGD7buvu+z9PuhjZA1Nw7x1XgGMJAs2SwTs2f5oCqCpSeRZpKTweXzgZ/kAY9Y0itu8+iluvvgQvvPmpViXDMNjw+ie4944rkOlLwZ//ehD3zcozRA91o/67Px0yXhw8C4wYMqDTKlucSpnWwXF0MQKOI4j0EzECB50PXZjvl4uvQXMogn2HT6EuIBobXrX+IN6uPgEAKJ6aY5mDuhPpcCk7cODAQfJQbPxAXWBQ73h7fOFkAMCWt2pQOj0XmRluPDQn36Dq0JNRugjawyUFYGRH6M+Bg1jIsgKOtfrT983Kw/bdR03igVk+DxRFNfyi2BiMZRnMunEUnvvzIUzJH0GtxORZYqnAPhsQLRXYz736MebcPBorywuhKFriiVXVNkTttS5awgIP3Dnesra7S1S8L8E+2aV02z3rijEkMxcElrGI2Wp+PmPx8xM9bzIC4AxDr/JnKDlXRVHB88RUgS2rCjU5lyxnd0+AwNPjfoEnEPtJ3N8bubs7E/broXM3IZx3Qh9CIkItiYq+cQBmTx1lMUI6Uf/y5z5qVeQFsgIXSzA4XdtJm5I/wkhU6xUzNcf9eOaVj/DEvVdj9tRR2LxDo+RITxOQnuZCU0hEyS2XI9AsYcuOz1A2IxdEr3wEOqX9yBGksYeiqGA5JqEXrQMHyUASZagscPJMo5HwePrlA6ZNqIN/P4ObCr+DrIEerFikBT88a+9EOnDgwIGDtkFs2ogZhmnxwVJdeOXtz3HiVCPuumm0YZcnjsnGY3dPRqBZRCisRWM/nDYGg9M94NCSwHL8KgcOWhCbyFUUaLEMA5ROz8X6bYeM2OjhkgJw0fVZc9xvEgRcsagQ67cdMm3axxbtPFxSADa6BvW1J4Jg5YYqi/BhzXE/fr3lABbNHIfh2QOAiNy2qL32FygyfW13p6h4X0FPuGddMYZkzinKKrZE4399s0WP+a0UGImfN1EBcI6lCxZyLEOtAhclGQ2NQYOuZECaAHisx/XGIrewKOPr2oApcf/ZV2fhTeE7lX6hJ0PnJI+fDyxh4jUx+yQ4liS1Htp9nc47lYPuBssSRADTTndbu22J7sjZGk5RBhu3k3/btSNxc9FI7TiWNRJE+q6/i2exftshasXMfbPyICsKBnkFzJ82Fv5AGOlpLjz10n5DzVnH/Glju+O2OohCUVWERRmBppYXrTdVQKrbcSQddByyomDLjs+NICq26k5Vo3aNZbBh+yc4+PczWFpSAE8KD0nsDy6AAwcOHHQRohRrscmr+2blgRAGw7LT0NAkQlEVHPz7GdT6g9i++ygeK5sMhgA8SxASI/jFCx+YfEgwKiKqFrz05ADbgYPzBT2RywJGwJU1KA1lM3Ixf9pYI85CVMwwvlq6KSQZHOyxPNIXDx0AQgCWZaACYEEsVcm68GHsml9cnIcMr6vTOsqSrfiMj2F7enKuK9AT7lkyY9CvX+tvBlhie30OwPIFV+HUuWYjfswemEI9pxyt+I/XF5g/bawlSdUVVcWMrMLndZk4sH1eFxhK8l1WFMQzc2jxCp0ft7cVuXEALszyomLNnk67v70Nkg0n+U/unAAKg0ufg6KqcAmsaT24BJZKSdMROAnoXopEBAfbI84V+3JToU0QRlUN4RqVJQgrKjK8AlaVF4GwQFMoghOnAsZEzRqYggFuzkhCt1wPOFMXhArVqGzevvsoymbkQhLlaMW0GypgqDnrMHY4k7xH/d256QhUFdj70de4oeDiqDCkC29XfYVbrr6k3+yEOug66CIhehDlTeEREiMQeBYRWQEhDFRVxawbczD7/+RAIIxT+ezAQT+Cd4AHbldybmooHEGgIdj2gf0MJn9IZagUa3NuGoOnXz4AfyCEJ+4twqp7ihBRVBCGAcsCBAwIVIDn8PjCyZqgE6Ni3ataVWZrQlMOHDiwwq6rM75aGgwAFZYq5vXbDmHRzHF4dP0+I6ns87qQyrOa2Hq0YjQ2YZ2eJmBwugc8TxCJKAhKmih7R2MkPd5bVR61G0SrLqVV0SYiUkeL4foakqmSTUbYL9kxDEqAriLZ60sRBWu3HjSOXTqvAOCtKbxkqpoTHWuyvz+VZzE826vxD6iq/VpQgf98+3NMyR8BtwBIERX/+fbnKJuRSz13b8tDmO6vqoJlmH7XdcoSYnT768jwClShyT4Jmzl+923jOvUyPcqe5+Tk/AVAFgAp+tHdAC4B8AgAHsCvampqfnuehtejYEfwX1le2CZFhd2OXGsvFwDUf8vwulAXCJteMouL85Dq5i3Xc7EEKW7OdufSGBeFKy1ZEZuuelH3J7gFgmvGm9WZK0oK4BYIwv2EC8pB14BlCSJENXjfVm6oMubXh59+i+xBaZZKnQszU8/3sB04cNCNcLs43PrAtqS+s/3J6Qh00Xh6K+L9oYljslE8dZSJc3NxcZ6RfNZa+YH6JhErYo55aO6VYBli8qsWF+ehLiACiPNDz+9PduCgV0OPhwSWwN8sYuWGKvi8bmpr9KY3NP5oXcBOo9XwGnGeHk/pCeuHozoaZ+tDXRIjNUTH29p5W41hYR/DZWT0vQ7MRKtk27pn7QXLEpwNdPyZmX4TGKx43nzsiuersKq8yHL9ZCuwExlrstCfQWamF6dPB2yfgR09qd1Ye1seoqvub28CxzOYdWMOVm1s0bpYMjcfHM+gP6Q+OI7Qf38n63/1mAR0Tk4OA+AyACNqamoi0c8uBLAFwAQAYQDv5eTk/KWmpuaT8zfSnoGuEHto7eUCm39bWV5IVfBdWV6IeE3YRHd6O4M3qate1P0JobBiBKiAdg8rN1ShsrzIuYcO2g2tIoDB8ZMN+O/3v7Lwvs25aYyx6QG0blMcOHDQOyBKMjIzved7GP0S8f6Q3uq8srwQp/1BRKK+ld6Z5k0VIMkwks+AZocbGkWj2ED/7NdbDqB0eq7BVeuIjjlw0HmIXbu1/iA2vfEpFs0chwszvSAE+MULH5roCmv9GmWevgbt4ilRVqkxEi1B2N7xxp43PvZqK4a1O88vF1/TofH1ZiQb9ydafdtZz8x0TlthQ+uxycT85zu2701jbQ9645g7G6KkGMlXQLsHqzZW95sYVJRkvBwVntZj85ffqsGCGeM69ff3JB8xJ/r/HTk5OYMAPAcgAGBXTU3NOQDIycn5fwD+FcBj52eIPQddIRpgp4yuKIAK+3+z+5w2URPd6W0vbxLLEqgsA1Giv/wUBf2Cw6czINs4EE5w6aC90CsCwpIMt8BZeN9uyB8GniNJ2RQHDhz0fAg8265qZgcdBy1xsO/wKZTNGIenNu+3+JHLF0wCSxjT5znDfcgemEq1zd5ol1zOcB+Kp14GAFA7oaXfgYP+jvi1W3Pcj0fX78O6iilgwFDpCkNixBQL0uIpmWGoa1mSFbjZxDwtPcGp0YMwUKI8uD6v23RuWtygx7A+rxszr780hoKNQBFl22SnFKFz7fYHJBP3J1N9m2hiOZnrE5tjCQGVxiDh3EAXFN8BAC+wEGXg5Jkmk5YVdQzneaxdid445s6GqtLzWp1MgdxjoSiw4WRXOzUG70nxvA/ATgC3AZgCYCGA4QBOxhxzEsBF3T+0nge9ZSXLp0mvtkZTwQssVJZFhGGgsix4gZ6C1ZXRY6G9MBjjxRP/bxxr/3l3g2UJmiQZX59uwtenG6njUqGCTdC56u/gWPp8OB/P1kHfgL67Xt8oanzxMfPrhvxhuKnwO/j6dIA+75LkgHfgwIEDB7D133ieQUWcH7lkbj7+9JcjOFsfMj7PGe7DXTeNxqlzTdTzhMQIcob7MOfm0Vi79SDKKneiYs1e+Jslx99y4KADsFu7euXl0rj1u7g4zxB7a895vznTqCWV9eOi2j9a/Eii8SSBymkJznV/PoSTZ5pRsWYPFlTuxMNr9mLOzaORM9xnGW8sdJG6OTePxvpth1CxZi/Wbj2I+oAIliX2NovrPfZEv3e1/maoLOmwLUwm7rerZI1Qjm1tjrX3+hzLYHFxnmVudjR+5GzyFBxp/73lBRZnAiIq1uxBWeXbqFizB2cCom2uJFEkel97EnrjmDsbPSmvdT7QXb+fUXtoSj8nJ+d+AP8B4PGampqfRj9bAGBCTU3NwgROcTGAL7tuhOcfiqKivikMKaKA5wjSU12aWEUMIhEFX33bYOL6qygpwMVDBoCLe5GfrQ/iH7WNFmX0i7LS4PO6cezbBjz+h33Gvz3yw4kYluXFidqA5fMRQwZYxtLV8AdCOPqPOqzdehA+rxt33TTa8lu27z6Ke/7tCvi87vZepjt+1MXoAXP3bH0QX59utPDNXZiZhkHpnrZP4KCn4bzP3Vp/M+Y//hZyhvuw4LaxCIuyMb/WPHg9lj/3PnXtVpQUYES2FzxFwMRBn0d3vUguRoJ2tz2VvM53uu87PQjn3eYCmq9I899GDBkAWVbgD4QhRWTICvDqO0fwdvUJrFg0GQAMio312w5RbfMjP5yIgQNcCEsKKn67x1Lx9svF13TE33Jw/tAj5m5/R2trlxAGiqKirjEMUZJBCAMXT+BNscaCtPN+ebIeK5+vMsVIL7z5KR6ccyWyfCmWa8dyx+s2Qf9//LqPFUa0iwnPNYTwf59+l2oz0lNdrf7uNnDe525bz629SCTuB4Dac82Yv+Ity+e/f+RGZPlS2j3WRK+vKCq+OdOIb882wS1wCIkRDBmUigsGp3Xo99c3hnDs24AlLh0xxIv0tPa9Z077m7GE8u5adU8RMuPuVTJoxxzos/O2N6G/5z4iEQXHvm2wdE+MoOQNY5D05OgxFfU5OTlFAFw1NTU7ox8xAL4CMDTmsCEAvknmvGfPNkJRemaSvS3oZPhtgQEQicg4G5Is/6ayrD2PryzHHUuoyuhlM3Jx9mwj0t2chffI72+ifn72bGMn3AErWrsnEYaBW+AMrjRd8dmbwsM3wI2nXtqPmuN+zJ8WwelQ+2SKupPD8nzP3QjDYNMbn5rmw6Y3PsUDd05IaF72VCS6rnoLEv09PWLuRqvqa4778dyrH2POzaPx87sngxAGqqpS126mz4Pn/nwIZTPGWWxWd6KvzZv2orvvQ3fzBrdldx0e456P2Pl5Ptdtj7C5Udj5aSpLcPxUwMTtDGgdUH947TBKp+dieHaarW3mAYhBybalPxRuv7/Vm9DX3g89ae52FD352SQytkRiLBYAFECMyDgbtMaCNGSkCVg0cxzcAodAs4QX3vxUo/SIFqadqWs2ElEAMCV/hBFPelN40/9jUesP4sJML9ZVTGk1Joy0YjMiIYn6uwlh2rxfPWHuqiwx3btafxCP/2GfxqXbCbRErcX9AACWpVJgQAX1/iUbx2dF563t9QGkcATDs72dmhuwj0vHQwy2b43bzcOIrHTYbiRzX3vCvAVaxgyGAVS1S3M6PRF9NfeRDAZ5BVSWF0FWVbAMA4HV8n12aM/c7TEJaAAZAB7LycmZDIAHMBfADwD8MScnJxNAE4CZAMrO3xB7H+x5fBXLw29L3dWO96i9fM2dDZYwRlt/rT+ImuN+rNxQhSyfB6XTc1Fz3N9hnuz+BJYl8AdChrgQEG3FYRlQ+7gcOGgD8Yrsv95ywOClizCEunZXLCrEvsOnMH+a1WY5cODAgYO2Yeu/KSq27Pgc983KM1U2D0h1Ge9/ve063jZXlhcafKJdoUviwIGDrouxGFmFz+umxnyAlQ82NtkcaJaQ5fMY/7eue+38rY23LZvRU2LL9uC8c+kyqsWm3zcrD2Dotrgr7nVXnJMlDD0u7cB7hiXEZh4SoINFL71xDutj1jfHesOYOxNO7kPjP2cADI3OAakLJkGPIVOqqal5HcAbAA4A+BDAH2pqavYCWArgLwA+AvBSTU1Nlf1ZHMSDteFLYil8SbHqrusqpqCyvJAqWNBTwQHIHphC5Z3auutIq3xVDqxw8cTCD1lRUgAX32PMhoNehtZsjIvCR1pRUoB3Pjxha7McOHDgoCPwDvAgM9Ob1H/eAX2nDVMP6PXK5jUPXo9FM8dh6y4tKZ3l82DrriMWvyrel0qGH9SBAwfnH23FfPF8sHqyGQC27jqC+2blYWf1McNOAMmt+75sM847l64Ko6O5srwQpdNzsX33UaCX7wV2xZwRWFBjjw5SQDvopXByH92DHmXno1zPP4377CUAL52fEfV+6IY1ngNaYEHd0eiNu3U6ZFlBKs8iJTMV/5+99w9zqj7z/t+f80kymYEwBBjQtgJbqweqsEVlqM5oW7Fc36dWoct3H8a6wriOtEy3sq3PqiPtKl1ltD9X+nxxi7RF7KPQfXhWSrXPRQvtLlLLoNIVWYgsa8Wq5WeADEwmyTnn+0dyDjnJOZkkM8kkmffrurxkkpPkk5P73J/7vs/9Y1VnC3Q92UxdUYB7b7/KKn+ploD6cGNoOsaM9uLRZS0wDANCCCgy+TghxeKmY2LRBJoa69D9pdbkMULgN6++g1/2HMmpswghpFj8dZ6i+kzXSiFmelXKqvU9Vp/X1//zBN452otlC2fiAxNGw++TeKyzFQldd7Sl0oNZ6eXGtLcIqVxy+XzpuuFYuA/b97xt+ZOhI2Fs3XkYd916JTxSQXdnK3TdgKIg7+u+lnVG5rnLrCgux+fnqmiuVkohM/GYhgkOLQfisWo+U6RYGPsoDxUVgCZDj02x6jqkotS0YtU0HdCSqf0KACQAHdUZUB9u4jENPp9ETCQNFkUI+ETtyg4ZfmLRBASAep9ETAM+cdWHcOM1k2taZxFCyHDh5ND7pLD9PWFsg9UDMpctVc0JDIQQO/noBg8ALZEs15YAoBUW4KxVnZF+7tJ76ZYruF7Lwf1SyEw5Wg6Q6oCxj/LAAPQIwFSsSWWtUbGSvOGmTIYD6ixCSDHE4lrWQBQOjsxNpkMf12D7WylX2TghpKIYSDfQNHNnuHvp1mpwn5BSw9hH6WEAmhBCCCGEVD0+ryyopcbW78wv4WoIIYQQQgghJuyoTQghhBBCCCGEEEIIIaQkMABNCCGEEEIIIYQQQgghpCQwAE0IIYQQQgghhBBCCCGkJNRyD2gJoOqHp1T7+kvBMJ+TqQD+CCBRws+oSNmttPUMhlr6LkDe32cqRqjsDhW1+r0KpcznYSpKL7dAjcsuGVpGms7ldTEwNXaOpqJGZLdcn1Eslbq2Kl7XVFSY7FbquRwMtfidgGH9XlNBua04Rvo5KJWtKwzDKHJJFU8rgJ3DvQhSk/wZgD+U8P0pu6RUUHZJNVJquQUou6Q0UOeSaoWyS6oVyi6pRii3pFopSHZrOQBdB2A2gPcBaMO8FlJblPoOJWWXlArKLqlGypEBTdklpYA6l1QrlF1SrVB2STVCuSXVCjOgCSGEEEIIIYQQQgghhAw/HEJICCGEEEIIIYQQQgghpCQwAE0IIYQQQgghhBBCCCGkJDAATQghhBBCCCGEEEIIIaQkMABNCCGEEEIIIYQQQgghpCQwAE0IIYQQQgghhBBCCCGkJDAATQghhBBCCCGEEEIIIaQkMABNCCGEEEIIIYQQQgghpCQwAE0IIYQQQgghhBBCCCGkJDAATQghhBBCCCGEEEIIIaQkMABNCCGEEEIIIYQQQgghpCQwAE0IIYQQQgghhBBCCCGkJDAATQghhBBCCCGEEEIIIaQkMABNCCGEEEIIIYQQQgghpCQwAE0IIYQQQgghhBBCCCGkJDAATQghhBBCCCGEEEIIIaQkMABNCCGEEEIIIYQQQgghpCQwAE0IIYQQQgghhBBCCCGkJDAATQghhBBCCCGEEEIIIaQkMABNCCGEEEIIIYQQQgghpCTUcgDaA2Bq6v+EVBOUXVKtUHZJtULZJdUI5ZZUK5RdUq1Qdkk1QrklFUEtC+CHALx18mQvdN0Y7rUURTDYgHD4/HAvo6IY7nPS1BQQZfiYipPd4T7vQ0ktfRcg/+8zUmV3qKg1uSmWcp+HMsktUOWyS/l0ZjjPS63oXMrWwNTaOaoV2QUq+7ep1LVV87oqTXYr9VwOhlr8TgDthXRq9TcuhJF+DkoZX6jlDOiqx+ORw72EioPnZHiopfNeS98FqL3vU6nwPCfheahM+Ls4w/MyeHgOB4bnqHKp5N+mUtfGdQ0d1bjmgajF7wTU7vcqBp4LnoNSfn8GoAkhhBBCCCGEEEIIIYSUBAagCSGEEEIIIYQQQgghhJQEBqAJIYQQQgghhBBCCCGElAQGoAkhhBBCCCGEEEIIIYSUBAagCSGEEEIIIYQQQgghhJQEz3AvoBqRUkECgKYbkIqAB4Cm6cO9LELICMLUQ8fC5wGpUA+RkkJ5I+WCNlblEBhTDwBoagrk/ZpofwKRs32lWhIhZYF6iJQDyhmpJGjrk3LAAHSBSKkgfD6OVet7cCzch4nBejzY3oxgg5cXKCGkLFAPkXJCeSPlgrJWWfjrPLjl3i0FvWbrd+YjUqL1EFIOqIdIOaCckUqC8kjKBVtwFEgCsC5MADgW7sOq9T1IDO+yCCEjCOohUk4ob6RcUNYIIcMN9RApB5QzUklQHkm5YAC6QDTdsC5Mk2PhPmi6MUwrIoSMNKiHSDmhvJFyQVkjhAw31EOkHFDOSCVBeSTlggHoApGKwMRgve2xicF6SEUM04oIISMN6iFSTihvpFxQ1gghww31ECkHlDNSSVAeSbkoaQBaVdVfq6q6X1XV36f+m6Oq6udVVf0PVVUPqar6pbRjb1JV9fXU44+kPf4xVVVfUVX1TVVV16mqOqx9qz0AHmxvti5Qsz8Om2kTQsoF9RApJ5Q3Ui4oa4SQ4YZ6iJQDyhmpJCiPpFyUTKZUVRVmjdAAAAAgAElEQVQALgcwJRQKJVKPfRDARgBXA+gH8FtVVX8N4C0APwLwCQDvAHhBVdX/FgqFfgHgJwA6QqHQ71RV/SGAuwE8Wap1m7hNpdU0HcEGL7o7WzixlhAyLGiajvEBH7o7Wy095JNAPKYN99JIDUJ5I/ngZjcVAm0sQshwM1g9NBS6kNQ+pdzvKIOkUGjrk3JRypsaaur/21RVHQ/gKQARADtCodApAFBV9X8D+H8B/CuAQ6FQ6K3U4z8B8Jeqqv4HgPpQKPS71HutB7ASJQ5Au00BDYzyAoaA9Apo0UTy5GkGeFkSQsqJlApO9cbw6I8v6KgVdzZjbD0nFZPBk+m4+KTAyUgsr8nY+Tg9dIxqj6Gcnq5pOgSSBqqESMqKEHnLSrp8hSNRSKkULV+UVUJqk8xru86roD9uQNN1SEWBIgER0wr29YZSFxJSDFIqON0Xp49ACoK+JSkXwjBK01hcVdVrASwD8GUAXgC/AbAJQF0oFPpa6pgOAM0Afg3g5lAo9Fepx28CcB+AhwB8KxQKtaYe/wiAF0Oh0OV5LGEqkpnVBROORPE/nvg3WyP2icF6dMyfgXVb9qGrvRlTJgXg9cpi3p5UN+VohDQVRcouGRmcOtOHv/v+ziwd9a0vX49xjfVuL6PskgHRdQNv/+ksHvnRbssAfXRZC1Y8uStL3r69/AYEA/6cr/3aX8/BlIvGQEn1kMvnmAzK1XxuKii7ReNmN2XKSCEUIStFvWYoPz+DmtC5t9y7paDjt35nfolWQspITciuG07Xdld7MzZuO4jd+49af0+9aAw8nsK6VZZCF5KCqBrZHcr9Kp1TZ6P4u9XZMvite27AuDGUwQpl2OW2SN+SkIJlt2QZ0KFQ6GUAL5t/p9pnfBfAI2mHCQA6kr2ojQIez5uTJ3uhFzi9MyGE7eJTJwex8MbLMHnSaHTMn4GN2w5i6YKZEFppc5+bmgI4fjxS0s+oNob7nDQ1Bcr2WcXIbqkY7vM+lNTCd0kIBcGAHx3zZyDQ4EXkfBybdxxCf1x3/W4jVXaHilqQm3wwpGI5Q0ByAvaps1FHeYv2J3A8Gsn52kd+tBvdnS0QqeyJfI5Jp5xyC1Sv7A63fGbaTUDyt82UkXQGyi4uVFaKfY0bg32vWtC5xX6HkaAr0xnu62+oqQXZNXH6bZyu7e71PeiYPwO79x+1/u7ubIXQtIIqIQrRhZUqN9W8rkqT3VxrHsr9Kh1XHyGmDcnvWqnyMViG83tVgtwW41vWMrUq5/mS7/cvRnZL2QO6Fcls5+2phwSAPwC4OO2wiwC8B+CPBT5eUswpoMfCfVAnB3HHZ6Zj9aa91t3JexbNAkT1OaiEkNrA6xVYfPN0PLHxgl5a3jYLXq+AERvu1ZFqRtONLOc5oemO8ubzKtDTesM5vfZYuA+abljGRj7HkOoj3W4ysaana9n2Uj5l6sXIylDKF2WVkNrE7doONHhtf2u6jroCW2oUqgvJyKVUewx9BFIMlBtSLgqrKyqMsQC+paqqX1XVAIAlAP4KwFxVVZtUVW0AsBDA/wWwG4CqqupHVFWVAD4P4BehUOhtAFFVVVtS73kHgF+UcM0A7FNAF954mRV8BpIbw+pNewGjXFXBhBBix9BhGQhAUi89sXEvDLboIoPEdJ7TERCO8qZrA7/WcrwLOIZUH4VOT08AVkAHSMrUqvU9SKQdU4ysDKV8UVYJqU3cru3I+bjtb6koeemqdArVhWTkUqo9hj4CKQbKDSkXJQtAh0KhnwN4AcBeAK8C+FEoFNoFYAWSPZ9/D+DZUCjUEwqFogDaAWwG8B8ADgL436m3uh3A91RVPQhgNIDVpVqzSfpU2qkXBxzvTuol6p1NCCEDkStrgpDB4OQ8j2/0O8pbQtcHfG2m403nvDZJt5vWds1Fd2dLzqFb+eiwYmRlKOWLskpIbeJ0bXe1N2P7nrdtf/tk4fZWobqQjFxKtcfQRyDFQLkh5aKkdnQoFPo6gK9nPPYsgGcdjt0O4M8dHv93JAcVlhVzCruUimMpFRNgCCHDhSJAvURKQrrzbPa7FMivpNjptZm9MvM5hlQnpt3kAQDNQK4pGfmUqRcjK5mv8dd5YMS1ouSLskpIbeJ0bdd5FSxdMBN33apDKgp8EojHNFc/MFdLjUJ0IRm5lGqPoY9AioFyQ8pFKVtw1ATMgCGEVBrUS6SUaJoOoenwGAaEpkPCyFveMl/r5EjlcwypbfLVYcXISvprggH/oOSLskpIbZJ5bceiCQhNS/2tIZ6ab0B7i5SSUuwxlFlSDJQbUi5GtEzlM9XY7S55f9yAJoTtLjkhhJQDTdPRNNaP7s5WaHoyW8fvU9DfFx/4xaQqyWe/KhWapmN8wJeUN8OAFCLvfW84100ql3TbStcBRRGAMJAwAI9UCpKRwcoYZZSQkcdA1735fL9uYGzAh8c6W5HQ9arSEdRtw4d57o+FzwNScT33pfiNbDabrjNWQfKCviUpFyM2AJ3PBHaT9FIqr1Rw/Ew/utNe19XejAkBHxU7IaQs+PweHDsdzdJDTY11iEXdRuOQaqWQ/apUn38yEiv484d73aSy0TQdHqkgHC1eRgYrY5RRQkYeA133Az1fDd4eddvwke+5L9VvVKzNRkY29C1JuRixLTgKnWpsEtOAjdsOomP+DHR3tqBj/gxs3HYQjD0TQspFf9xw1EP9cQ6KqEWK3a/SkVKBIRUkhIAhFUiZ//Zf7OcPxbpJ9VCMjA1WRob79YSQ6mOg636o9cJg9t9ioW4bPvI996X6jQp93+GQT1J50Lck5WLEZkDnmvSZ86QIAws+8RF877nXcCzchzlXTMKdt1yZLFXIKLFh6RMhpCQIA4s/Mx1CKFAE0Di6Dos/Mx0QNBJqkaL3qxRSKjgX13D01Hn4fR5EYwlMGteAOp8EDAw82E1P9tLtmD8DgQYvIufj2Lzj0ICfP9h1k+rB65Poi+sIR/pxpjeG7Xvexm3zpg2YcaXpBmZ+ZAI+98nLIBVA04F/+c3AspX++sHIGGWUkJGH03UfDPhhANCEgJL6O/2YdL2Q6d/5pEBMMxz9vVxZruX+jtRt5SHfc1+q36gQm62QLGyvTyKmAe+fOAcpJdt61Br0LUmZGLF70EAT2N2CxwLCCj6rk4NY8ImP4O9/8FtLaa9ob8bYlFHB0idCSCnweSTO9Mbx2NMvW/rlgSXNGDNKIp6gfqk1pCIw54pJmDt7iuVMbN/ztrVfDYQhBcKn+vHk5tcteVneNgsA8MTGvQPuTT6vgsU3T8cTG/faXu/zKtAznA/73qnk3GdJ9eFkGwHIKve9Z9EsPLftIJYumIFcA9T9dRKfafkzrFyXrstmw18nkcij5HMgW27g9Rf2ekJI9ZOpN9TJQSy+eToeXLPLtsdteOEAQkfCAC7oBQlh8+/mXDEJbfOm2crW0/dUt2zU7s4W57UNUfJSobqRDB35nvvB71/OslGIzZYA8Fwq69W0L532bq9P4kQkxhakNQx9S1IuRmyNhdOkzxXtzZBSwPAk7wZ2rdmFpd3b0bVmF8Ln40nFr124W/lX/22aFYwGkkbFo+t7oEGw9IkQUjJiCR2PPW3XL4893YMYDYSaxCcF2uZNw7ot+9C1ZhfWbdmHtnnT4JO5QnsXSGiG5YgASXl5YuNejG+sz2tv0jU4vl7P8DnMTBpz71z7/Ovo4kTtmiHz9zVtIw0iy95ZvWkv5s6eAk3PHeiIxw089vSeDF22B/E8Sz4LmdrutP5oPIEVd2bbgpRRQmoXnxS2valt3uWOe1zbvMsB2PVKpn83d/YUKyhnvja5pwokhMiZ5ZqJm44tpiVCIbqRDC0eACscYgyZ536w+5ebbORrswEABHDL9Zfa7Mtbrr8UmXeOYxqy5Lx7fQ9bkNYQ9C1JuRix+1D6BHZNN+BRFETjCdz3/ZfQMX8G1m3Z53i3WlFg3a00nfd0joX7kNANACx9IoSUBk3TnfWLRv1Si8Q0w9Hw7+5syZldaqK7OMB6ygFO/huQLq9P6M7yltB1m7xlOua79x8FgLSJ2mxFVc243Vh/5IvXOcpH42jfgNl2motsaRmy5fr6DFsul4w5rf/hp36H7i+1YNnCmVZ7Gq93xOZmEDIiiGkXep0GGrxoHF3nqIc+2BTA2q65Nr2iCWE7NtDgdXztiTN9uP9/voSHOua4Z7lmkCtbOr/bzRcoRDeSocfrVQbcVwa7f7nJRr42GwDAEFi9yR6sXr1pL7o7W+1rHeReTSof+pakXIxoeUq21EieBAMGHn7qdzgW7nM1JjTdgM+rYHnbLDyxcS8Ul9IZRQEEyzoJISUid2uDYVwYKQmD7RPocZGXk2f6rH8rOWQn3zJRp3Xu3n8Ud92qw2MYgGZQPKsYNzl0s4WCgbqkM53jPd11mQJo+UlLui2XS8bc1n/idBQr1+22fX4xAR9CSHWg6QZ27z9q3SQ1s1Cz9RAgNPvelbkfRs7HHV97pjcGANi47U3Lb0xv0eG0dw91T+B8dSMZWhKAFVMwcdtXBrt/OfZ1LqC1h+4SWNZ1w5aUMBR7Nals6FuScsE0jxTpit00JtIxL0ChGQgG6rBs4Ux4pMDytlm20pnlbbPgkYKlT4SQkuH1KnhgyWybfnlgyWxm7tUopjORjlsGlePrYWSVg37181fjJ784aPXrzTVkJN/9bLDrJJWN2+/rkcJRPuq9yoDZdj6JrDYtXe3N8Lml45dg/WagyMStPJ4QUhtk6oLNOw5l+XNuPlvmfrh9z9tZOuyeRbOwecchAEDoSBgbXjiAVZ0tWNs1F92dLa4zF7iH1gaFtF3Jl0Jko5AYhOLyvkqGO1HOvZoMD/QtSbkQhlGzRvZUAG+dPNlrlRnnwpAKulLDJ9TJQdzxmelWScqcKybhrluvhEBSUdd5FfTHDUAY6I9pOHrqvFViM2lcA0Z5JTRNt6bFJkuPlYKnxTY1BXD8eKToE1CLDPc5aWoKlMMKnIoCZLccDPd5H0pq4bt4/B7ENR26BhiGASEEFAl4peI6uGukyu5QMZxy4zalfHzAh5hm5FVe6/N70B83oOk6FEXgfH8C587HrYGGSxfMgMgRLDSH30AIwDAcP6uQaer5Uia5Bapcdsshn7l+XwDJ+Re6AUUBPFJAAnnJZy5babADudLPi9v6n9t20MqEBNIy1QZZql4LOrepKYBb7t1S0Gu2fmd+1e+xhVILdkU6tSC7Jk6/jZMuePjuj8Pv9SDh0C4qUw/5pLDpNtMvNHXYz186jH/51/+yPm/OFZOwdMHMrFZUmWsrxR5aDJUqz/msqxJkNz2mYDLYfUVKBefi7jGHTPKNQRQicxfsSFPuBWJ5DAuuFoZT7itBbovxLWuZStWD5SLf71+M7DIhN4V5t3DV+h6EjoSxdedhPPLF6+CRCs6ei+Fr//Rb29TXjSmH5UJwWiSdLiSNCimVrKnww2FEEEJqDykETvVmG4xNY/0cdFqDOPUJ9EmR9x7j9UkcP9Nvm17+wJLZ2L7nCF7/zxNWZkyu26NmmahpkDgdy56TtU2u31dKBWfP2+Ux3VbKJZ/xmJZWgqwhnhKuoQ7GuF1Ht82bhrfeO5tVHs+KU0JqE3ddpmW1Qsilh4RhQELgxJn+LN333olzlp/YNm8auta85HjjLr91cQ+tJtJjCkO5r8TjOp7c/Lr1nivamwFvdgpyITGIfGVOSiVLzhnXqC3oW5JywQzoNJwybRKA413Mhzquxdlz/dANAwICE8b6IRUFdV6B/rgOA8CDg7z7OdLvvDgx3OekEu5QDgfDfd6Hklr4LoaUljNjktQvrRAuvdhGquwOFZUmN4Vk2BhSYu3zr2Pu7CkINHjTsp5nAnDOZs7kQjaNGbgrrKKnWJgBnR/DJZ+m3eRm83TMn4FV63ugTg6ibd7l+GDT6AGDKgO951DbUYPNss7x2VWvc5kBnR+Vtj8MllqQXZN8fxu3LOeB9JAhFfz8pf/CTc1TIRVA04Hfv3kUH7/yA1b2qbO91oKJ40ZVpNxUqjxXSwY0kF/VWPpxA+0/hlSw9vl9DnZcdvVaoRnY+ayhFFndlcZIz4AuxresZSpVD5YLZkCXCadBAHrGtGMg2cepty+G9T//Dyy+eTr+ceNrVquOtnnT0L2+B1+57aohHSRBCCEmuuHcX0437ENDSO2iu/QY1HVky4AwcMv1l1ptpay+zwog4vqAGTlen3TMphkf8BUVhC5VwI+Ul/TMQDebpylYn9XWLFfmVD7vadpRQyVHuYZAUVYJqU7Ma/dY+DwglQFveqVnORfiz0kpcP2sD2HlupdtrzUDOY//TeuQ9wMmtUFBVT4C+MubLkfkXHJmgdcj8Jc3XQ6nabmF2If5rqEgm5NUJfQtSblgV/EBUFITQdMxh9YsvPEya6oxAMydPcUqcc41yJAQQgaD+9AQ6peRgtve5CgDhrCCf0DSoFy9aS+QZxwtrsNyTszXr1rfg3gRcTjT2elaswtLu7eja80uhM/HISXNkWojgQty4WbzjK73YuGNl2XJ36r1PY4lnfm8p1REWeSIskpIdZJ+7d71yC8HvHbT9Q6Qnz/nSU1p0zTgsaf3OL4WAM70xugPjjDylb9Mucu1N0pFQX9Mw5ObX0fXml14cvPr6I9pkJnTAlGYfZjvGgqyOUlVQt+SlAsm46aRnuni8yrQNQDCwD2LZtkyd75y21XQDQPjx/htd4oCDV7r7807DlmvCwb8+OtbP4rxjfXQdMAjJSQMZtEQQorEwNfumgOPokARgG4ACZ36pNaxZWM67E33LJoFiOysKk3XMfMjE/C5T15mlQj/y28OQTPyq8hJaLpjVkRCK7yix83Z6e5scUrkIRWMlpYRlW7zmPJ43+JroAiByRcF8FDHtYjFEzh+OorNOw4hdCTsWBGm6QaCAT865s9A01g/vn7XHJw43WcbumS2R8slR7myH/PNaqasElKdFHrtahnZnYEGr6WHgmPqsHLpdVj/8zesfvZf/fzVUKRATBeArrv6gkC2bjRnB+lGUj9Jn4TQ6BPWEgkA//raO3io41rL5vpVzx/w2dYP2+QvU+4A92ppTdNtSW/Hwn14YuPeZHuEzAUIA1+57Sp877nXbLELZ/swzzUUYHMCw189NNyfX53QtyTlgQHoFOklKMGAH4tvno4nNu5Fx/wZ2L7nbXTMn4FAgxcTgw04fS6Kx59+BR3zZ2BisN5S3OZd8mPhPoSOhPGbV9/BP3zhOnikwJlzMauHmDk4YCwb9xNCisDrkdC0OB754cu2oTdej4JEYuT16RoJZJZJrlrWgq07D1t7U+R8HFt3HsYXFszMem19nQc3t37YKhE25aXeJxGPDrwHyVTmS2ZfOKmIrIk6Axn9bmWcbE9VfchUtky6zfPw3ddCSgEBgdO9UdvgrXsWzcL2PW/jjs9Mx9adh1PyY3defV7Fsr+CAT/uvOUKx6FLuZzmepeWMebQr1zlxunyC8oqIRVLrr2mkMAeYNdlAODxXNBD6YN775o/A0dPnoPHI3Df93fiWLgPD3XMcfUFASB0JIxXD/wJ3Z2tgDAQORe3DbZf3jYLwUAdRnklfcIaIbMtiyk/Uopkcpt5XIbcAem2lX1v1HTkLdNSEfD5FCxbONO6eevzKZCKyCp8y3sNBhxtzqULZjh8/8IGCA91sHioBxiPFOhbknLBOsIU6XfL01trbN5xCLdcfynWbdmHrjW7kNB0PP70K9Zz9yyaZZUrbN/zNh5YMhtzrpiER5ddh5tbPoyv/+C3OPzuWVt51rFwHx51KbEhhJCBSCR0W4nnsXAfutf3IJGgYVWrZGZ0GTBw6w0X9qZ1W/bh1hsuheLQqC3uIi/xPOVFSoH7l1yDhzrmoLuzBQ91zMH9S66BlCLjuOyWBefiGiAlEkIAUgLCucSP5cjVhwfAg+3NmJjq8/ypay7Bw0+9jC90b8e7xyP49k9ezWr7Mnf2FKzetBd33XqlYyBI12DZXwtvvAzfffZVR9tJKgJzrpiEB9ub0d3ZggfbmzHniknwKAr64rprSbF7ubGAJgT6dQNrn9+Hpd3b8e7xXsoqIRWElAoMqWRdq5ktDjwu7QI8Du0KALsuA4AxDT7rJtiD7c34ym1X4UxvP8709qOvX8M3N7xi6ZCN297E8ja7L9iV9l5zrpiEG666BF1rXsLhP57N0j9PbNyLo6fO0yesITQdWX7/Y0/vQWbsM1PuzECp097okc62k0dm70e6Bjz+9CtYuW43utbswsp1u/H406/Ygt+FrsED4LZ502w2523zpjmutaDWIlLB6Qy78fQgW10V8vnkAvQtSblgEkeK9Lvl6eVToSNhPPPiAXTMn4GpFwcg0oYSpj83+aIAjvwpgrfePY3b5k1DOBK1nKjMciyAWTSEkOJJaM7ZPQnqlJpFSmEr5wR0rN7077ZslA0vHMC9t1+VJQODb6FhQBGKLRO1q70ZgD1DJ9PoDwb8CEf68cTG31mvu3/JNVmloaazw/yKyiBXNlL6cwaA8QEfujtbYABWlRcA+H0eR5kz7SEB4dz2Iq2cPZftVO9VrCFh6TLp8QocP9Hv+jrz35nPnTjTh/v/50tWpvbpSMwKLKVnQVJWCRkenLIazWs1dCRsa7GhSGRdu8vbZkGRcAzCaZqOYIMX3Z0tlp4IBvxZw1MfWNKMiePs2aKhI2FseOEAHl3WAk3ToenAb19/F6s6W6DrBqSiWJUgbjrN7/MM+XBVMnxoLjaXlmFzZcpdrt/bgIGuJbNxurffymoeO7oOBoysFhwJ3cXm03XH1h75rCH9OAgBpFq4Oa21oNYiEHg0I+j56PoePNbZmvW++VJoBQRJQt+SlAvKU4r0EhSn8ql1W/alDJvsUhWvR8AwDIyq92DyRU34+x/81jY1OfP9APcSG0IIGQjFpWROEczMq0W8Li0FPvpnQaxa32Md57avKC4tNBSHFhpOaBocsyIyew9mGv2Zg3qPhfvw+NOvYHnbLFvgPDDKC40ZFhVBrtJVwL19RX/Gb+9m95iPS0XAgJLl8Oayxcz3kIpATDMcZXJVZ4s19MvR5kr9O/O5M70x631Wb0q2X1u1vgcbXjhgBZLM2SD9erJVBwNDhJQPp6zG9Gs1GPDDAKAJAUUz8OtX3snrBq2JpukQSDrGhpRom3d51vDUx55O7nuZOiQcieKt985a+/HEYD1uvOYSeAzDFgx002njxviTf0iJc/EEHn7qd1k6lrqmeiikbVm63EEzXE0yqQj0x3VbIsBXP3+1a1uNOVdMwtzZUyz5377n7UHHHcy1NjUFcPx4JOda8417JFyCxYMJehby+eQC9C1JuWALjhTpJSibdxyylVOlZ71IKazn1MlBqz/hssd3IBbXcKa33+Y4Achq1WH2MWT0nxBSDOl6CICV3eNUikeqn7juXE54y/WX2tpiPHz3xx33Fa+LvHjzlBfdcHYQdMNuyJtGv4lbtpciBFat77HKOEF/oGLIVbqa67nMkncnO8rsAb2ivRnReMJWcmuWz+dri7llOOl6sgQ+0+YyX+dUbnzPolnYvOOQ7X0CZsA9EoUAUKcInI7E8MCalxxL/gkhpcXtmg80eKFODmLxzdPxYEqnPLhmFz51zSXYvOMQutbswqr1PQhHonm3z6nzClw8YbSLjjGydEhXezO273nb+ju9hUH6vujkDy5vm4XTvVEs7d6OB9a8hHCkH8GA3/o8tg6oPqQUeGDJbNvvbPaALhZdR1ZLqu8++yqcZsT5pHBsl+Fz+Hyn1mmD3dsKaS2iKHBsLaIMotVVIZ9PLkDfkpQLXospMktQfF4Fj3W2IqHrtuychG5gwwsXWnKYgyTUyUFMGjcKfzzWazlO5rTY0JEwtu48jEe+eB2EEKnMHwMJA/BIhXe1CSEF46+TtgEj/jrJOF6N4tpCQzccB7RlYsBAYJTPJi+BUT4YeUqMIvLLijCNfjNIGY0lHF8XjSWsf7OlQWWRq3QVSJalp2cVbt5xCJpuoM6r2Erew5Eoxo6uw2NfaoWmG0lZEQaWLpgBKQXu+/5LWYHs7s4WiAxbzF8n0d3ZCk3XIRUFPgnEYxqkdM4w8yhJx/u5bQfRMX8GGkf7EAzUod6rIB5LSpm93FjB2udfR+hI2PY+ZhKBKZ9uwXez5J8QUlrcshoj5+Nom3d5VrXNExv3YtnCmVi5brfrXuPW7qI/rqP3vHslxZhRPjzyxRYoStKP8yjA0gUzcNetV2a1MPBJga72ZnSv77H8wZVLr0NvXwzhs/3Y8MIBtH/2o7Z1m1nd5mNsHVBd6LqBnXv/aGub9queP+CW6y/NOjbfliuFtFKLa877lVNbi1LsbZqmY0JjnW3vrvMKxKLZt1LMoGdmuxwpAaNIw7CQ1ibEDn1LUg64n6WRXgajpxyVzJIYRREIR6JYtb4Ha+670Qo+3/GZ6Th66pyVebN601488+IBLFs4ExeNH4Xw2X6c70/A51GwgqVVhJBB4FEE6jICjXVeCY8iEB+mNZHS4VYWJwB8+b9/DB6pIHI+jme3HcTSBTOynAZdNxCL2y35WFyDrnvz+nwpgQeWzLaG6lzI5rH303S6kbuivdnq72cGyRsDPqztmkunoALJVboqpcDim6dnOYo+r4L+uIZfv/IOvrH0OkAA75/oxepNv0c4Er1g56TarMR14ehI6zogccEWq5MKTp3pd20Hkn6zw3xOwkCwwYulC2akAtgeGHHNCj4DdltPwsBt86bhrffO2t4nMCopx6Z8asJ5zQwMEVIeMm9wpl+rmkvv0g82BfDDr33asV9trnZDMd3AU8+/Yflz1vN3Jqs30ltkPLBkNkY3eFEnBIRhZLVRiGkGNqZuiH1o4mgcPXUO//jca9ZNLzOInr5uswLDfJ6tA6oLaRj45NWXYOW6ly/YPnc2QxpG1g0QNxnMtItc2yMoyLqD794D2iqIRboAACAASURBVKEHcwn6JXt9Eicc9u7xAZ9tLwaSNkfjaHuCRONoH6QiBpX5n29rE3IB+pakXNBuLhBP2p06czNYeONlWL1przWw4tUDf0oOo9ANaJqBzTsO4Vd73sHEYD3+tu2qIb3LSAgZecQTOrb3HMFNzVNT2RV1rtkVpPrxSiUrAPyV267C2fP9+P5Pf289ds+iWXDaTAxD4J9/9Sbmzp4Cvw+IJwz886/exNIFM/PaezQN2PTLkC3zddMvQ46vz7yRO9YpCyWm0SmoUNyCPMngDRyzDB/rbIXXq+BT11yC90/2Wln5Jpl2TmZPcnVyEG3zLocBwEjrrZwzM2uADCdTBoMBP44fj7h+X9dMqVSw3JRP9pQkZHjJda26VURIBZgYbHDsV5uuX9TJQSy88TL0xzUk4IPXqyAciVqD5gMNXmvo233f35nRF3oPvrH0Ohh1ChIOukjTDezefxS79x+1EpbCkai1xuVts7DhhQO2dbNKqLrRNB1j6wce2JcArGod07Z6ziWRwOOSKeyRImuwZiHB6lLsbW5t4x77kkMGdtxw9mda6c+UG/qWpFwwAF0gQjMQDNRh2cKZ1mbgkQqOhftwLNyH37z6Dj7T8mdY8eQuW1DgnaO9CB0JY2ygzvZ+zKAhhBSKAQNXT7/Ill1xz6JZMOAYfyTVjqajcZTP5qTohoGf/vJN22Nbdx7G0gUzs14uFOCW6y+1ZXLds2gWhIMz4oSu65YDnU7HrQayG35kLJ1ZKFVFrtLVhEMWcDDghw4DejyZVT92dF3O7GYAgDCszMJgwJ+VVe022NB8L9NmGirZyud9cgbmi/xcQkhhuF2rua5P1/dK6RczKJyZ6Xzf4mvwzQ2vYNX6HmvPjMU1R5105lw/PNKPc31x1Pk80KSAlBIQBqSRHAi3e/9RhI6ErerYDzYFUOeTONcXswWkWSVUG+Q1sE8422ZOhrzQDFw0oQGPLmuBYRgQQkCRycczKSRY7QEcK9UGs7cV0i7EEC7+jDDoz5QZ+pakXDDuWSCapmNMgxd13jGASAajgQt3Dz9x1YesLDXAPqV53ZZ9WQMwmEFDCCkckTWdffWmveh2yC4gtYFuAOu27LN+8+/97Q15B5UNA67yko9RWUg2DaldPA6Zy+bgL1MG71882wq2mEwM1ieHL5n1tAawdedhdMyfgckXBfD3P/itQ5ZzqzWcqBKyjtlTkpDKpZjrU2ZUsdp00I978LdtVzne4HXSSWd6Yxjf6Me5aALdaZVK9yyaha07D6Nt3jQAwO79RxGORBEM+FGnAOPG+IGExiqhkYohrL1woEQCRQpEowkcO3XealUxcVwDxvg90DKEJD1Zzjw2GKhzDFYDgNer2I71egc3XNfdZhTZNqPh4s849KsmpYa+JSkPDEAXiNcncfxMP7pTdwrnXDEJSz83w7rzPr6x3vGuX+NoH5a3zUKdT8FDHXMsJT9pXAMzaAghBaHrhuMwMF0fOCOVVB8JJIPP6f0o63webMxoi2E6Lk49oJ2zUvOTF2Z/jhxy9aQ0JHD/kmtwtjcGv8+DxtF1ePqF/TZn5fENe7By6XW2nsrL25I3RkzX1wPgtnnTsGp9D75y21WOsnniTB9+9LP9WVlcwyl3zOYnpHIp9Po097V+l6zm8Y1+/OPG12y6xyeBB+9sxqof92QFmT/8wRmOwcS5s6egO3VT7a5b9ewWHdQrIxYhDOdEAofMAA3Cas1iEo0lMMrvPMvD41EwadwoKALQjTp4PM7pBgnA6mluMjFYP6j2oF6pOGZge6XICpbrhot9ajjbp/kObSSFQ9+SlAsGoAskpsEaJhFo8CKh6Thxug/BMfX4hy9cBymTpVZzZ0+xLt7te97G+EY/4pqO070xqz+iWeYCLy9rQkj+eDwKlv7FlTjbGwMAeD0CS//iSng8CowYDbFKphjj2ewheToSs/Yer1fJOwNaUZz3JUURQB7ikp5dlqufIaksipE1t77L3/xyKzTdQCym48nNryMY8KNt3uVo/+wV+Gzrh/GTXxxE6EgYx8J96O2LYdnCmbh4wmic6e3Hj362H/fefrVlcKbLkwHnLOczvTGEjoSx4YUL5epSAeWOEDIkmHooAZ+jDvJ6FHR3tkLTdUhFgU8C8ZiG8Y1+PPLF6xCO9ONMb8zKcBaKyNqT71t8DcY0+PCV266CASQrQYax4JXBu8rCyJH9mxn8NWAg2q/ZYgjL22bBcBAoQwqcOhnFd5991Tr2q5+/GheNr8+yD0sxhBCajqZgPR6++9pUABzJALiDrAnhbAM4BeELGdpIWS8c+pakXAyuxmIkkrpbuW7LPnSt2YXv//T3SGgGvvOTV/HeiV709sWw6NOq9fy6Lfuw6NMqpBQQENZdcyCp4B9d3zOoKa+EkJGHAliGaNeaXXhy8+uI9mtU6BWOaTx3rdmFpd3b0bVmF8Ln45Ay9y9ntj4IHQlj1foedK3ZhXhcd3RcDAcb0edV0DZvmm1faps3Db4Cyiw1TYfQdEwMNkBoOg35CqdYWXNyRoMBP05HYuiP6fjec69ZA5ef3Pw6lj2+A9//6e9xx2emQ50cxMRgPYIBP+IJAz/e+gbO9Cb7m3oy2o+Z8uRFMgtxYrAeAKwbKZt3HAIAhI6EsXLdbgAG5Y4QMqRomg4PjCwdtKK9GdFYAl1rXkrpz5dwMhKDlAr6Yxp++LOkbgs0eDF39hRs3HYQ0O2troIBP/pjGr72T79F15pdeHDNS3j/xHmsfX5fXrp4qCl2TyClQ9edeyXrenZQ2W0IcGZGMZCM85rBZ/PY7z77qlP812pFk47V6moQ9PUl8PBTL+OLj+/Aw0+9jL4+52iHmS2dfv0ls6Wz5dLtBnnmO1PWi4O+JSkXJZcpVVW/rarq+tS/P6aq6iuqqr6pquo6VVU9qccnq6r6b6qqHlRVdYuqqqNTj49VVfUFVVUPpJ6/qNTrHRCHu5VPbNyLhTdeBr/Pg8i5eFYP6Mee3oNYTMeZ3n7Xu4yEEJIvCc1wNEQT7CVf0eRrPGeiSGQZ6IqSPRDuWLgPmpEtA7GYbrWNMo/rXt+DGDMaapZiZc3JGW2bdzlWre/BuWgcx8J9jj1TV2/ai7Z5l2N52ywABlat78Hu/UfRONqHFe3NkC5pf+nZ0Gu75qK7sxVbdx5G6EjYOmYonGFCCHEiWwe1oDHgs7UlSNefup7s5WzeDDZ1nZYRTFx442VZdtrqTXsxd/aUvHTxUFPsnkBKh+IS/FUcojOurdQcbD5Nc8lqdhpYiOybwAMN8ByIBGANNTQ/2y3hzoABf53EsoUz0d3ZkuxFXScdM7tzZWtnfj5lvXDoW5JyUdIAtKqqcwEsSXvoJwD+JhQKXY7kQM27U4+vAbAmFApNA/AKgK+nHn8EwM5QKDQdwFMAnijlevPBrVeRWdbs80rH54UAzvTGSnKXkRAyssh0dADezKoG8jWeM4nFdfz6lXfwUMe1+Kf7b8RDHdfiTG+/837iULdIeRl5DCRrUiowpIKEEDCkYmUGOTmjH5gwGsfCfQifTcpcoMHr+N6Txo3ChhcOIJ7QrddOaKzHWIfyWNtaU9nQHsOABwZumzdtSJ1hQgjJRboOEpqOWNx9z/RKl4xRqdged9OT5uPl3n+LtT9I6Sgk+OtxCVZ7HGy+QrKanW7AOLW0KIRCZC0W17H2/7yBeCL5XDxhYO3/eQOxePbn5/u9KOvFQV+BlIuS2fSqqo4D8CiAVQD+XFXVKQDqQ6HQ71KHrAewUlXVdQBuALAg7fF/BXA/gJtTzwHAcwD+P1VVvaFQKF6qdQ+EIpwnu5qN2u+9/SqXqe0Kxjf6sXLpdVj/8zewe/9RTAzW4+G7Pw4BgYQQ7FFECMkLj1Qce/p6pABv8Vcu0mUyuFQEkCPDwOdV8KlrLsHKdS9bfe/uX3INVtzZjEd/bO+F55NAPKMk03SMzc9VJwfRNu9yAIAhFe47NUguWZMQOfsoms6o2TtRIPlePfvfx8ql10Fx6dkYjkTRNu9ySEVgzX03wuMR8MAoSLacPp/ySQgpJ7n0p1AuDGMdG/BjdL0XipLMqnp46cfx8Npk5nQ0lnD1F4cj+ahY+4OUjkL2OwkDK9qbrcxis1WMRPbgSimF4xBAKQV0p5YdQzwI02wblylrHkVBZs8QqQiEI1GsWt9jO9ZJLvMdiC1d5p5Q1nND35KUC2E4lG4MBaqq/jOAfwJwCYBPAvgBgG+FQqHW1PMfAfAigE8A2BMKhT6UetwD4HwoFPKpqtoPYFQoFEqknvsjgOZQKPReHkuYCuCtIf1SAMKRKE6c7rPabEwM1uOBJbPx4q638Pp/nsDjX27Fe8fP2ZT+A0tmY9MvQ1bQ+f7FsyGlwJhRPvSej+ORH+22jv3aX8/BlIvGJIdDkUqkHD/MVJRAdkc6um7gzLl+xBM6vB4FjaPqqvY66z0XxZ/CUautwsRgPbram3FR0I/Ro/xuL6PslpB85EvXDbz9p7MF6/xTZ6P4u9X/ZjPm51wxCUs++1EcPXkefp8H0VgCF40fhQ9MGJ31Xmd6ozh+OikvwYAfi2+ebtujKnzfKdeipqKGZDeXrJ0514//8cS/ZTmH315+A4KBbP2h6wbeO9GLU2ejeGLjXkcZum/xNUgkDNvQo672Zky9aAw8nhHbQbAmdO4t924p6Pit35lfopWQMlITslssufRnOBLF+yfO4XvPvWYbALx152HcNm8adMNAf0yDbhhQhHA87vb/Z3rZ99xi7Y/M96gCO7pmZTff83/yTB9OnOlD5FzMsg8Do3yY0FiP8Y31Du88tJ9/9lwUR09lx0omjavHmAwfpVC5zGcN8biGt49GsnykKZMC8Hpl0d+/xAy73BbpWxJSsOyWJACtqmoHgI+GQqGvqqrajmQA+ikAj4VCoetTx1wGYCuAuQB+FwqFLkk97gHQGwqF/KqqxgA0pAWg3wVwdSgU+lMey5gK4K2TJ3sdm/kXiyEl1j7/etbdobsXzISAAd0Avv2TV7HwxssQaPBizKg6bHhxP3bvP2q9x8TUZFivR8GKJ3dlOYHdnS0Qmo6mpgCOH4+4rmUkTngd6JyU4fPLtkEMtewOhuE+74NFSgXn4hqOnroQrJs0rgGjvLIqrxk3PbR0wUwIp4kkGLmyO1TkugZyTeYGYNPTPikQ04yC9HZCCCzt3m577MH2Zqzbss91/0jHkBI/f+kwbmqeCq9H4N3jvdi47U2rz67b6wo9D6WgTHILVLnsmr+LaRfougFFChw9mWwBlq7z+nUjS57UyUH83R1XQ3eTSynxwJqXsrLoLx4/CpoO1PkkHkx7HjDlqtVVJ5WD4dy7akHnNjUFigpAV7O9UAzVbiNlUguya1Lsb5PuY3kUBYpMtguQioIuB13XMX8G1m3Zh4c6rsWXvrUDQFJPLrzxMky9eEwy+1IYgAFLvw5WbgbyAzOfz9f+cFpXLjunXHZ0Puer0mR3KHVDvn6/IRWsfX6fg48wIy87z+2z8/WjdKngKYfPv3vBDChO2d1DHM8oxkcCaC8Ue95qlVrb1wsl3+9fjOyWqgXHIgAXq6r6ewDjAIwGYAC4OO2YiwC8B+AYgEZVVWUoFNJSx5gZzu+mjvtjKjAdAHCyRGvOC13XcToSsz12OhKDoRvwKkC/YdhKSbo7W2zBZyDZT0cRQCLh3mtnoB+mEgwBQqoFQwqET/Xjyc2v28rRGppGYdC1ZsOBMHDL9Zdag8DMzJqy5YoSG24DT7755VacjsQc9bQwjLxLHT1Ksizu1hsuxfjGeui6AY8UCAb8tj3Ebf8QwsDV0y+ytfC4Z9EsPPPiAYSOhPPed0hl42QX3LNoFp55Ifk7mzcaMkux1clBLL55Oh5cs8smp4FRXitY0p/RGzB0JIyV63bjB11zcexkLz7YNLpoe4YQQioNsy1BnVRwLp7A0aPJwNvYQF3O3s5SXjDEQkfCWLdlX+oGb9pun+rBfyx8HiiyDdZAfmCu5wuxP0zc7JzuzhaanmWgEL/fA+C2edMGbFVRCIX4UbpuOMZKdL3Ew8dMXH2k6ksuKCv0LUmZKIkeCIVCnw6FQleGQqGPAfh7AD8LhUJ3AoiqqtqSOuwOAL9I9XPeiWTQGgAWA/hF6t8vpv5G6vmdw9n/GQC8XgWLb56OdVv2oWvNLqzbsg+Lb54Or1cgAeCHP3sD9yyaZTXJN3uApTMxWA9NB85F40UPJeSEV0Lyp/Ym+wrLQAAuTFcnw4PbwJOEZgyJnvZ4Be74zHT46yTeP9GLM739ePd4L+5p+xj+4QvXoruzBQ+2N2POFZMc9w/DcJaXhTdeBsB933EbVEcqEye7IP13Tg8Ipw8+apt3eZZ+XLW+B/GEgSNHI+g3DEhFweN/04oH25uhTg4CSMqNAPDk5tfxh/cjZR+yTPkkZGQxHNe8IQXCkWTgrWvNLvzp5DlHXWf2dvakDSl0GipnBhLXPr8Ph/94Bqd7Y4gDgEcp6DsN5AcOtZ/IwW6FIdNuMgyFrBbye5ZisGAhflSdVzrGSuq82efAvB661uzC0u7t6FqzC+Hz8cGdLxebFwYjqbmhb0nKQ7kTU24H8JSqqmMAvAZgderxTgBPq6r6NQBHANyWevzrANarqrofwOnU64cVXYOjAn6ssxW6bmD3/qM4HYnhG1+4DuGzUeiGgXtvvxrf+V8X+iLes2gWftXzB8xtnpw1JGBFnncocxkCzDYixI6uGwgG/OiYP8MqK9q841D57sYPMbrL9a/rBiq2u1kFMdTlfm7Dfdx+p0L1dCyu41xfHAnNsLJP5lwxCYs+reL7P/29rVebT4qsIYS6y2TrQIPXdd9xy7YZO5bOZqXiZhcEUq1gzICwpukYH/Chu7MVmq4DEI6vC5+N4v++/Acs+rRq6+WY3u/0hz97A8fCfdi84xDuWTTLljnzwJLZkBKOQ49ykc/1ySowQkYWQ33N52sHZAbeNm57M8t3M3ViV3sz/F4l51C5BIDnth10zDQ09arTd8pc70D2xVD7iRximD+l2J+G2+939zuy/SjNJVj9WGdr1vuWIrNeN1zWatBHygV9yySmrh1MhQrJTcl1VigUWg9gferf/w6g2eGYt5HsE535+CkAt5Z0gQWiuTjymm5AUS5MDwWA7z33GoIBP+7+3JV4+O5rETkfw5neGJ558QAW3ngZvrFuty0oFo0l0BjwQYsN7K3RECAkf+q8EnfecoVtQNZXP3816rwKtFj1bSqKcL7+FcG7+wNRCidWg8A/fOE6vHci2Vs5HIkms55cJoEXqqd13UDjaD/+/ge/td5r7uwpVlAQSO5D3S5Gu+KyXwTH+LFs4Ux4HbJS3JyCby+/Ie91k9Lg1Jv0WPg8pIu8mZl5ViaeVHAyrTXMQx1zXF/nJGerN+1Fd2crIAyrxVjoSBjPvHgAHfNnYOrFAQgh0LP/fXz8yosLciLzvT5ZDl65xOIampoCBb0m2p9A5GzfwAeSEctQXvOF2AGZQZnQkTA2vHAAqzpbYaQGDUIYWLpgJnwSiEUTEEg52A5tLjTdwNzZUxwzDTvmz3D8Tk7rfeSL1+W0L5z8xGSVlIKErhd8892snBnKtg61Sin2p0L8fikVnD4fx6Npv9WK9maMdbFz87kZ40mLcaT3B/YoIksAcsVKMgOZpQis00cqDp43JjeUCybLFojiooAVRUBKYNGnp+Gxp3sw8yMT8MCS2TjT249vbngFX7ntKnSt2WW9j9krzNyUTNZ2zc3rR6EhQEj+6LphBZ+BpHHz3WdfxeNfyr4bXxUIIyvbkH268qMcTuzYgA9CM6BIoKu9OWuitFOWci48ioL+uGYzCs09JB0zGyXTwHfaL+5ZNAvfe/Y1W2/g9O+f6RSYw5Si/RokMwKGDSeZW942CxteOICxAV+WvJm9nLs7W6zfzJCK7Rpwy+h75sUDaP/sR12cQz3LITb7nX7jC9fh/eMRfEydWLCs53t9Dnc2GHHH55VFDS4cuaN+SD4MdM0XUtlUiB3gkdk39sKRKBQBIOP9TV2Xay1SEWgc7XOtVnHSY07r/eHP3sADS2bbqlPS/cDMfX/OFZPQNm+aNUCx0MBKeluHoaoeq1U0l6rLwexPhfj9GoQVfAaS8vLo+h7HDOR8A25eCbTNm5Zlz3olsvb4XLGSzMWWJKHO1Udigl5O6FsyuaFM0E4vkGSQ2V6OapWZ6sCmXx5Ex/wZ+NDE0Tgd6cdF40fhWLjPykA6Fu6DOjmIMaPqXBTuhTvTuSbrFmsIDHXpOSHVQFxzdlziWrUGKwRePfAnPNRxLaSS9IF+1fMHfPb6S4d7YRXPUAauchoqmo4EFGzcdtByQnTDQF80gT6/p6AgroSBujqJhzrmwO/zIHI+joSmO+8hUiCzKWBmy4V4wsC//OYQQkfCrt8/3SlQJwdxx2em24xSZgQMD04y98TGC5lzH5gwyvqdPVKBVAT645qtF3PmNXAho68Fug6cON0HAwbaP/tRV1tFUQQEBB6+++N4+Knf2YLh33v2NYQjUSxvm4XRfm9B3y/f65NVYISMLHJd8xLCNYjmRCF2gKIg6wbd8rZZUBRASAWGFEhoRnI4sKLAK2GrMMncLz0AggFnvWr6ipl6zGm9u/cfRdunVSxbOBMfbApAKrDZFNl+ooK1z79uC4o+t+0gli6YkXdgxRzM6JbdTZL4UvOiMmXG51Wg51Hl7EQhfn/CJQM54SDf+QbcYpphBZ/N49yq7nLGSjK+fqEJdXnFMQyBrTsP22R9687DWLpgpsM7FkZtx1HoWzK5oTzwXBaIpiGrHPWxp/ekylGR1dNr5dJkiZTZH3HrzsO45fpLseHF/Vl3mbram7H2+dexe/9RTAzW42t/PQeNfo+rYivUEGBZARmpKApcgyjVaEHX+xTccNUlWLnuZZv+qPcpiCaq8AuVkaEMXA1kqGipuQC79x+1grhPbHqtKP0bORezTR+/f8k1WUb7A0tmQ3oEtIwAtMxouWBmNbxztNfKgM78/ulOwcIbL8sqF2ZGwPCQq8+zOjmIq6dfZMtwM7OjzbYwwQYvDGTrw3AkCgGg3itQ7/dYjuacKyZlZVUvb5uFbz3zKsKRKFa0N+ObX25FLG7g3eMRbHjhgHVj44mNe7Gqs7WgPvv5Xp+sAiNkZJHrms8VRHOiEDsgFtex4YUDtmDWhhcO4L7FV6M/riF8qt8+y+fOZjy37aDrfqlpOup90rEqaevOw456zG29x09HsWp9D9Z2zYVwaveR7icKw7HvtBhB5fXlIte8qMGQr9/v1nZNUeDQLiO/gFuuHtBZbTVyxEoypa2QwHq+cYw6r3DM1q7zChQZ/y/o86sVv4tv6fcp6B8hviWTG8rDgH6BqqrzHR67ozTLqXzchjlphgEYyHLS1//8Ddy/eDbCkSieefEAltx8BVZv2ovd+49a/RIf/5tWPLqsBRu3HbT6KR4L9+GRH+0uelqxE0M9EZmQasEjFSxvm4WJwQuTyZe3zYJHVqfh3devO2Yi9PVXvwFUakwnNl0WMqfU54tpqKRjGSoZz7sFcfPRvwkAj/7Y/ns//vQr0A0DHfNnoLuzBR3zZ2DTL0OIx7MNJCfdv3rTXiy88TLX75/uFEy9OODqoJDy4iZzkfNxRxl7YmPyd06Xt1zXQGaW0+79R7Fx20F0d7ZibddNWLZwphVkNst6Nc0AYGDlut1W8Nn8fLdKLikVGFJBQggYUrEm3ud7fabL59quuejubKkZJ5AQkk2uaz5XEM0JJz2z4s7mZHajEDCkhM+f1DpSEQhHkoHerjW7sGp9D8KRKGAIHD11PivQ+OiPe6xZQG5ricc0BBu8+PbyG/BU103o7mzFxRMasHTBDEc95rTeexbNwuYdh2w2Ry4MQzj2nTa4jQ857hnIg9uf3PbNTDxS5O3zDGTHmiipGROZxykOsucWK3GzBzRNh9B0eAwDQtMLbp2TaUf3x3Wr+tC0jzduO4j++ODOf63HUaIuvmV0BPmWQ+kjEndcz6eqqrcA8AL4lqqq6Te3vABWAnimxGsbVtxKLNzuKv7hvbMY3+jPWSL1gQmjbZNZQ0fCVv/ntV1zreCzSdJgAeqkMiROFcsKyEhF13WMa/Tj0WUtMAwDQggkdD1nm5tKJteAD17LuRnKPoYeAA/f/XEcPXUefp8H0VgCk8Y12Howms9fPH4UOubPwOYduVtfOKEbcPy9+2OabYYAANx1q5F1Z9ktc2XqxWNsvYEzMbNtpEMPTGYEDA9OfT3v/OyViPTFMNrv3Bd88qTReLC92epBKQzD9RrQhHC0Y+66NSkfK9ftznp/TTfg9TjLiNcjYMTt32GgLKL0tfm8CnQN6Nf1rLY1LAcnZGThds3nzFpzeZ/0tlRSKtB0Hfd9f6ct86+psQ5aXHfMvNZ1HX6fx1HnNo722R5zbKuh6WgaNwrHoxFbVqr1ndL8UAPA+IAPj3W2Iq7peO9EL5558UJli08KxKDktGlc7UaDduNQ4zaA2qMoyfTgIigk+1YCaArW4+G7r4UikjakxyMgAWRaenlXExXQV7mQDOxCyDeOkV59mM5dt145KFmv9TgKfUu7jwghgJR+ZHLD0JJLnj4G4EYAEwF8Oe3xBIDvlXJRw00uJQ84D3N65sUDVjZZpsINjPJhzKg6/Pylw5j+ZxNcWwE81DEHG7e9aQUnJgbr8e7xCIIB/5Bk9rCsgIxUPB4Fcc1APKGnjDEDUgp4PAKJKrx17RoQdOj/S7IZysCVx6Ng0rhRKbmqg8djd3jjcd3WOsPcL9xaXzjhNpk6GrP/2EkHx2HIi4u8eKQAEtqA35/tDioHu3EM9Mc0vH+yF36fx7G1xsRgPSAEvB6BpX9xpdWDsthAjttzQgh89fNXW8NeJwbr8dXPXw0hBDKle6Cek+ba6qSCcI5eqoQQAuTeo5xwaku1vG0WgoFkFfPDLQAAIABJREFUIpGZ+dfd2QrhctM6ASAaSzjqxPQez8Xsl7n8UCkFJk8K4N7br4JUBHxSIBJNZN0Ir/NJwLjg6Btw0e1swTHkKNKlb7hDD+R8KWQ4mgbg1Jn+rP34ovH1mW+bfTNGUeCTySx9GwZc+irPyHrPOq/iOIC7zqsg5tJaI5++ylIRjsMNM+3oUsU7aj2OQt8yiWmDNjUFcPx4hH5OCRDGALU3qqp2hkKhNWVaz1AyFcBbJ0/2FpzlaEgFXWt2ZV2A5mCpdEX5h/cjVkab06Cm9P6L9yyahd+8+g4+efUlWcf8+pV30PLnH8BF40fhxOk+/Pyl/8It119q3eE2P3sw1ELvIlMZDOPnl8NSm4oiZbdUDPd5HywevxcnTvdlDcSYMLYeiWh84DeoMLz1XpxM9f5Lv5bHj/Uj3uf8fUaq7A4VTteAx+/BidNRB7nyIxFNuO4lHfNnYPuet3HXrVdCCAFFIOcddsOj4P0T5237xlduuwqNAR8eXnthANyK9maMTenz9H3K55V493hvljP0gaZRMPJsiGe+X7kzAsokt0AVyq7ik3j3+Dnrd51zxaSswT/pNsjytln4UNNoaLFsT8L8ffVUtt0Pf/aGNY8i/Qa8mw3Rrxv4zv96DQtvvMxyDDfvOIR7b78KHsOwyaOiCHzrmVdt7TqAZCWYJ80mHcgWy4fh3LtqQec2NQVwy71bCnrN1u/ML+o11WxjVLuNlEmlyG4hA7/cjnX6bXLtzelVRZk6KfPzzsU1hCMZPaDbmzEu4ENMM1zXPdB+6q77WuGBYTs2cx8w9T6Q7EP88N0fRzyu49ltB7N6QHe1N2NCwJcVbKxUec5nXZUguwkh8NNfvYnPffIya5jbv/zmEP77TZe7ytNAJITA0u7tWY87yaguFTzoID+rOlugZGZLp+Q48wbGKK/Mktl8YwiGlFj7/OtZgeKlC2ZCZGSAF/K+Xp/EiUgsK7CdKcP5fqdMBpKvUsZRKkFu/Q1eHAtn+5YTg35Ez1efrzxYKlUPlot8v38xsptPRv2PVVW9HclMaOsDQqHQdwv9sGphwMFSZuaQlFi3ZZ+tpcbWnYdTk+QNvHu81zaQZ/Wm5KT637z6Dv7hi9dBQODd4xH8+pXsoPSD7c2IazraP/tRRM7HMRSTnoay9JyQaiIe1/MeiFENxOLJdkDLFs60jCtFEYgl9Kr8PtVKPG7klCu3veTDHxyD8Y0qvvZPv83PiDWcJ1PP/8RHHPV5ppH8+N+0Og5Ruvf2qx2NADdHnhkBpaHYqeoJzbD1HzXLTd1sELehgG5O1dLPzbAy6Mz1uGVKSSmtPqkmycwgBRJG1vubgXHz5n3bvMsBJAMvVjuQGi93JYS4U2iwJ7OqA6l+ucfC54HM9j05hrmamPors2WCqa/7dQN+rweXTPKmdC7gUQQkDMRjmmuVVT7fy219J870oc4rbcdm7gPHwsne/9/4wnU4Fu7D0VPnrSqs05EYOubPQONoHyY01jtnug7wmxSzV400fF4FNzVPsQ1z++rnr7YqkIqhkOzbXAMDM/d/QwqET/XbKvWWt81CQ9Moe2uYAloTaLru0gJDz9q7C8nszpxRcaFSIfvYzOrDFe3NgDdzXGJh1HocJZZIJiCk+5ZG6nFChpJ8bPhnAVwC4A3AqqSsaUnMV8k7ldjcesOlOHkmqeSdeiU2jvbhU9dcghOn+zAu4MfKdbvxYHuz43CqZQtnYuW63UN6h409E8lIpNb6WgkAj/xod5aOeuxLg5uwTQojl1zVSQVwKXmNJ3Q8vmFPXgY3kJzo7TSZ2qMAekzP0ueZBv2Z3phjcNCtXUe1V8pUE4M5305O5ulIDKmZyI42iK5n9wjP6QBquq0naWbZurlWRcKx5NYngZiW/f5PbNyLZQtnYuO2N7H45uk2O8p8T7eWIrVS7koIcaeQwFQmA+lVNz/PbGt1oWWAQHq8MOf7GjqgDdxmI5/v5ba+M70xrNuyz3ase7AxqSPT+1TbZw/dhD7dyOqtX+w5JRfQdVjtL4Dk7/HdZ18dlI1eSCs09x7U2Taf2w2MVZ0tWbZCvokIHte2bwqQsL+qkBvNuQLr6aHlBIBHM66xR/PUHQNRy3GUREaAHzAz57MTFwgZDPnEXmYAuDwUCo2Y3SVfJR+L645ZZWbWspPynTSuAZpm4MSZPkCkekQ3OA8OGt3gs/6dr9GVDu9Uk2qgHHLqbgxVZ1+rhOZshCWqNKBeCRQjh2790sze/eMa6/D1jjmQQrEGwWhG8j0LuSHSH3eeTO22J2Qa9Jt3HMoaHrOivRkS2cbzYJx+UjiGFAhHovjKbVdZbSvyPd91XomHOubA7/Mgcj6Onv3v41PXXIIVT+5Cx/wZeeu8fB3AnLIR0zDBJTvaabDhsXAfPtgUwN/dcbWtVDj9Pdl7fOQQi2toagoU9JpofwKRs30DH0iqksFUQAy0jznplhV3NqNxdB3Wds2FVJRk8DmaKOh9h+p7met7bttBzJ09BY2jk7OENu94M/tYl2DjyTPJv936VL97POKY5CSlgnAkioQQNluItkH+JDTn5ISEVryNXkj2rYSBFe3NVhA2l81XSLZ0vkjl/2fv3cOkKM+08futt6q659Az0+AwHiJoiBYHhw3gDMKMugHDb1fkYPgSDq7MEAeU8UDEXWUgG8dEGDRRN2YXI2BESAR2PxKVwP4WD0lEjAwCuxyE1rBGTNDh1Aw9hz7U4fujumq6uqpmqufYPVP3deWK9FRXvVX91PM+7/M+z30TSw5syhDT9Z3yOgPOOYolWYHf5zXkZzQRZneNZA9bW1DMhQv9Gdp60Kp7xkX3wMl72ACAAxDp4bGkDZw6ecoQy6oyzdElL/hrF92ExlBEnxDuvPWrqKksxcVQ2NKhZnvaWsFSrdZ0d6pdZAJ6y05TCYYyAQxjXRnIuGIynUJn7ZBjCZZXlGLNK2ZR2mAojDX3lyMWU/CjV9oql5dXlMKbbRNE21R22i1Yk6s+9PtJqp7S6KHakoP2CxeX9qD3QOMie1YilR09b0oZXAwZ22Z/uHgSfrBOpXWxikFqKkvBswTJFNB21XYMQ6CgzVY6sg1j27mEmNT++VU6GftzEkXp1+2uLtrAc7RTvNEDl52x/6Mrgl9OqBQtfUtU1P2XBJXCI/HvkW6YH53clyYMN2/qCJPofaglajjWKtm4bP54vLzjmF74lPx3jQJJG7++6afHQntNsVB33PtAAWM7p6JLu6dOq28lSUaBw7kzlWppp4jEJBvat3EmW/FwDOZOHeFIsJAw1uKOhDG25vMcY+qsWjp3bJcoUAYCWGpttyxDgAESdrn5s96B7ZwhCMKy+H9+CeD3giC8BkBnIO/PHNCAMydvtYOeyGu4eddxLJk9BlcV+kAZgICgdv17+q7cV4bk4mIogmuuzLNMYkRjbavEVNtO3Z1qF5mA3rLTSEzG7z783MChqwqCCBkZOHOUsQzCOEqS6QpdOEBn7VCJycj3cairLkdMlHS70jh3RUnR/bp23jWv1OPpB29OqbLTbsGqQG2fTQ6KrOameVNHgIWa1Gtv4dLfVb7TCVZ29/w2lZqC5xhI7YhYWbWYNjZHDJsOm3epC8ChRbk41dCErbtPYPGsYpNN28UyP958AMFQuMuUGO1VMqMDe+vP7a4uXLiwR1c6IJwmeVPlac7L5rs8P3Z0XzrHtKToxwhD/Zg9+TpwLINFs4rBU6Jv8CUnG1mGAUOBR+4ap88d4CjqqstxrrEV+bkePPfqQYMIrJZIBkNsYyE3NnAO+xjdHK/1FJzOnalUSzutDLUr0LOylUjMjtfZrNETs+k8T9YzkSVY0oqsqe46TWF/7i5XANRUlOBiU0TngC7I9UAh3SJFlhFw82e9g/ZyL8Xx/78U/9+Inh9OZiF5B51lGIiKjLlTr4eXZ6EoQNHgLEiSAoCBpKgtIZV3jMJzWw4mVMSVYM+hvxgc6o49J/Gtb3wNKypLkZ/Lw+/zGAKODsfm7lS7yAD0lp1yHMHUm4yCIP/4D+PBcQyUaOYFDoQBCnweg1BEgc8DMpB6pLoRXbHDpuYYVr2cULlRUYpv3Hg13nj3JBjGmmojGktNyIQFsHJhqeE6j9w1Hv/5/qe4o3w4pKRW2a4Ipbi0Bz2PxAWMlX0MLfKhMRQ1LAjtRKq0xIQvm0NuFo8Jo4t04Z/AqSA2vH4EVTOL9YXgPTNuMNm00V6Av54NGcQLEykxrBaqmm1wPEVUgpmCw8IeeUoQlRTIsoIn75uEl944in3HGlx7c+HCBYCuCX5ZUmxUloJSgqhMDD7I6twigC27TxjWZVt2n8CS2WNM561ddBMIiImywsl9gRAQRdUUisRk8DzFxbjvf3jeON3H3337SBOFVr6PRzSmdjQBAJE0PQgJsmROPEqE4LF/fQ8rKksRDIUNY9KSg+3FQh6GtOv/XbRBgQJfDm+I0X05PBQLCa2+Tmg6rZamlEFzTELDhRb9nooGZSOHo46KIOzmdXstFbNgIcswGHWtH1cV5oIyQF6OB6Ou9ZuqtUWbc4oW50wF/b06VvMBiZ11j959IyghA6UA2s2f9RJsn2UgEFjYmwPJVCTuMFIoCLWKhhf3sYob0dIqgucoBud7sWjWDXh684dJFXH7sbq6DJ83qM2EHEtwz8wb0BoW8czrBw0TfUFcoTl5skr+jLdpo3B3ql2kE3qtokIBfvJLoyDIT355AE89kLmifTlZLIYW5enJHurOjB3CLtDvrB1KhOhJYSBeufFKPWoXTcSCaaPAc9RwXmGoH9+dMQoMQxCRZbAMAw8TTxq3M26GErAsg0fuGof8XC8UWQHLMpg68RrUrH3PMhDubOVof1f57mskLmDseJoBYqpuXr2xHqury0DiVUcKgAmjizD95uEmmg0A2HesARNGF2Hh9BsQaoliRWWpgVcx+V0A1AQGCNHFCxOT2wpUO+Q4xrCozspiIYFAYhiIMdmQSK6pLMVlPl5PQrfFSsRSzHDxncWAAtfeXLhwAaDzgl9WSd5wTMSjP3vP4Cu37j5h2PjSE0kEJt/60JyxKrdswvzo9VBcaIygdr31PNwhFDVZ9tJrqt/8l2W3gOdY/HDxRFDK4M5bv4qR115mEqpf1Qmhei3OsaJnctSRApj8P8eZqx76OqGaDmAYoj8zDZQhYBhjIi9dEppO3jOFEgQvGGm/ls4di+zCHFNWOSW+ahsKEMowSG7p5DmCb04YhoYLzboNfnPCMPDJYqEp8Eqngv5eHasoiilH9fTmDzN6rZwq3E6P3kGHKQtBED6FkVpHAdAC4CiAZYFA4IseGlvGIbkl1u/zIhyR8LN//2/dWf/o3kmWOytykk2LkozfH/jcsPv+6u4TqJ79NwiGIqZdfY5jULv+A8NntYtuMnzmVhW5SDf0VrWlrWifpFhy6KY/CC40Rgy0PcsrSnFZgbevB5a2aC/QBzpnh3Z2FWqJQpRk5GZxWF5RgjWv7Iff58Xib92AcETCioSksbax2N5iIyoBm3Z+hOk3D8cPXnzfMMalc8eCIUSv0LKiWEgVLu1BzyFxAWOXCJDtqndEGV+cb0bRoGz4vCzumXEDvv/z9w2Lha3xqr1Fs4rR1BIz2EtNZSl4SiDD/l3QaDb8Pq+p6k4TxdIqrIWhfhPP4mMLSjD3mwLOXgzHKT/GmOyx3UVcB5sxLly4cOEE2jxWWOjDmQvN+noIaPOVC24fjVm3fs08fyrElPR9ftshVUdBUrnuPZRBOCKlnJCyikUemjMWX/tKARSFoHb9Hw0+myHWnVRenk3pmhIIfnTvJJw+14TfH/gcS2aPwZWX5YKjjEq3IMlgKWMbC4mA4RkC6lyReN10Saj2NWIxBU9YPisjrURPJjS7eyNAlBRLWovV1WWWAnVO40gPR1BTWWrBAW1MKgMqfWIwZE6C53g5w3E8JZa80nad5E6pRdoTbMzMtaQR/W+tnDp4am2PqbAQuOgYTmrmXgPgA/BvUCnI74n/+zCAdQCm99joMgzJjmn25OtMzvr0uSbriidFMTnU6bcOx2enLwFQq6K/fdv1EGXFNFlpO+HJn62pLner2FykNXqr2tJeECQzhRViomzJLVx3v5kzzYWKjpJenbFDatNp0tgUxYbXj6CuuhwHjjdg1ZIyEAJ83hDS/bw2hlUOFhuSLGNKyTDTgnh1UhXUQ3PGDhyitgxFYntfIk/zNVf4dLsTYc21fKqhCRteP6JWHXlZEBgTE8JQP6bfPBwrX9iLqpnF2PD6EWN1ftzWAPt3QdsUDIbClvZWNbNYT0BbxThPbdqvX1u1R+eimm6LowsXLnoCyT5H85WJtGyJ86fdJqAstyViRKjJMCe+LDEZqMDMs6wlt7WOJu1zjQvXaj4ItcTavWbita2SwgU+HkRSIElSG01HPBb6ydJbEI6IRmovQjq81/5eIeoU9rQSxt+op+bCntgIaC/5apWAdpoAj8RkfeNcK7iz06uI2SbByw1jiEp2vNJmO0zlWTE21dpMFwQb0wn9ba3cGUQlxbE9uug8nDCG3hwIBKoCgcChQCDwP4FA4CEAowOBwHMAhvXw+DIKmmPS4MvmTM566+6PUVNRqh+ncUC/9MZRk0MVYzJe2H4YNWv34oXthxGJSrYTgLYTnviZKMsqJ5iiqFVFbvLZRRpCknrHTh+aM9bw3mVysk6SbIJbtz3IFu0F+kDn7JBlCJbONdvV9nc+0c896quDsfKFvTgb99N2AXx7oAyD/Fy+wyqo57cdApQMNeoBAq29T4PG00wZotudlgS2s6ufbj0EUVL0xYKG2ZOv05PG/jyPta0p7b8LWgLiqsJcy2Pyc3n931Yxzplgq/65nT0mPwPtHpNbll24cOGiO5DscxJ9JWCeP5N9K6AlYtr+LckKGpuiHfoyLcFVs3YvFte9jXONrTb+1zquAwPUVJrXjfXH2hqQ2/OfdklhKS7ymgxJkuH3eU2xkBO/3VGcNVBAk/IBgPVvpFFFrKgsRV11GVZUlmLC6KIuz4V2v7loN17KQKEMREKgUAaUmtNDrM09sRZjTbb5mrV7EWyJWZ5XkhXsO9aA1RvrUbN2L1ZvrMe+Yw2WNtPexpDxOPtkeTJSelZEsVlL9h/7try/AYRU7NFF5+EkAZ0nCIJP+4cgCHkAsuP/dFcLCaAUWF5Ror+44ahoctbBUBh5uRyWzB6DuuoyLJk9Bnk5vF5RpOFMsNWws64tOhliHQCEo6LpM3cx58KFCkUBduw5iaqZxairLkPVzGLs2HMSSobuyTgNbl20oSeSXpG4IvfjVRPx1APlqJpZjM27VPG2If4sUEr0ao1QS8xyTtCrC9oBTwG/z2P53eQqqORA3EV6wSq5rPNvxpHYGbKu5jaDXQFtCykPxxgSE4mbFNkeztrW4kJZ7b0Libzoycck2qGdPWs2qSa8zfbo5Bm4cOHCRXch2efYbehq86cTH0UZgrf3f2ZK2CQfl5zgsk9aW8d1kKFX5Gnx67Y3Ayj7myv1Y1a24z+7Kyns9Jm4sWkbrUTis9JoJRLBU4IF00aBY+Mc26z6b5527Xml8ps7TRZTKFiZdE8rK0tBLYQVU0nqpmIzLLVJgic9r+SCQO04q1g7pffDZi1p8QgyErZr5X5yf07g+rDegZN4/xcA9gmC8B9QE86zAWwQBOFBAMd7cnCZBkkCtr0Z0Mv2ZUXBsvnj8eyrBwwtXu/s/xw3j/0KGpsiiIkKLoYitm3ciTgTbIUMxcwZubAUPNvWFpIYFPSDjhAXfQinvFjpDpYSzLhluIGrdOncsWApgZyBL4m22bXmlf36/SyvKAHN0PtJFZ3htusJvnGGgU5VcPftI3XKgyH+LCybPx6UaWtZ3f7OJ3hwztexdO5Ygx0+PG9ch9UTsaiEbC9rGv/SuWOxaWfbNKxXaA0AG8hUOKUd0vg4FQYGKg2greooEjO2Cg7O9+pxQDQmmmIFrVKHVTp+F+zelyyO0cfOcwxWVpbq2hfaNTbvOq6P0ypmd4UuXbhw0Zsw+xy7VnoAkjMfxQKYN3UEtsR9cH4uD7/PgyyOQSyBvDY5wWXF/f/QnLFoicYs4zrCqKKyyYVKldNG44XHJiN4KYJ8Hw8pfk1zfNQ9olodPRO1ipb0iq5LuiMaFlGY71E5w+PPysMRRMPGFKwEQFYUFA3KAUMAWfFAUrqug5CKkJpT2hRJklGQIOwJRbGdtyVZgd/nNdAYbH/nE0tqkVRic46z5ijnOAYxMeFoYs6V2FUqp/KstHe+v9p3f1srdwYsYIprV/aj3zhdQBQH2xqCIEwFcDtUP7UzEAj8ThCE8QA+DgQCoR4eY2dxDYBPz59v6rWKMIkQLKp72/CZMNSPZXeNw8VQBHk5Hmx/52N83tCERXfegFBzFAU+L/JyOJxvDOMnv2xLVCeqM2sY4s/Ck/dNwtmL6i49SxmEoyKuvTIPRFK6XXU4HZWMCwt9OHu270yusNDXG1tg16CXbdcKlDJojklouNCiqw0XDcpGDkf73A5SBeVZnLvUiktNUf1e8nJ5XJaXBSlq15SWxqAUO947idtKrwFlVGGOt+r/jOnlw02q0Rr6i+065Wuz8l8AOu3TrHwPw1P89Wwzfrr1EPw+L+ZOvR6XD84Bz1GAKKCEwaM/26MHtquWTAJLCfJzvZBlBecbW/Hb9/5X5RZzMI7Ee2IZBuGYaBKa7Wmxn972wb1kt0AP2m5n51KGpzh/KQxJUgzxwcqFpcjP5RGNyfjzFyFsf+cTBE4FsWrJJFCGwbOvHkDVzGK8vf8zkwr84lljIMkyWIYBQ4FoTLYdk5Nxa8fIMqBAwUtvHMW+Yw29Zo8doS9jhv7gcwsLfZj+yOspfWfHMzN77Tt9GQ8moq9j0+5Gf7BdDVa/TXdx5DrxkQplULN2ryHBNWF0ESqmjQYAhFoiiMZkeD0sBuV5IIoKFEWBrAAsS8AyDJb/23umBNnjVRPx/LZDCJwKYl3NFLCKot/Xlt0nMKVkGPJzeVxW4EVTSwyrXnZ+r6naM8dTtMZkBEMRRGMSeI5BQa7XIHDYHXAyrnSz3fbGTL0szl8MmzYdBhd4IYU7vzbheIpzoahJSO0yH2/YHAEAkRAsTspdANBtyjDeuL13lIBOjI0TE5lXFeZATlYWhPMYieEpwjEJlDBQFAWEEEiKDC9HDedVKIN1rx2xiH/MsXaqa92eyo2kg91SnsWFUAQXQ2H9WRT4vBjk82TmWrkToJRBRFEgikp8U0j1wx5CMi730VU4nQc6Y7u2FdCCIIwIBAInBEEYB+AcgE0JfxsXCAQOpHqx/g4r8vZgKIzPvghh9cZ6DPFnoWpmMUpHA09v+hBngq34+WNTUPNve/WdQn+eB/k5PCKihBm3DMenpy8ZKuqe+dVBBENhPDRnLF77w58wf+qIuIiEM7VZK9glalwl44ENhRK0NhsnnNaoiGwvm3HbgBxL4OVZXEJbV4GXZ8GxBFK0nS+mKSgU/O34qw0COisXqq1wGfbTpIyOqjU0tfewJOP0uSZs3f0xgqGw7r9InGM3VT9pBQqVGmPJ7DF6sNYcjmHjb49hzjdHYIjfa6h4fuPdk5jzzRH4wYvvd2pn3ejnJeRw1K0iTXN0ZcMkJin49zc/xrdvu95gYxzL4IXth7HvWAMmjC7C9+aNQ1NrFNGYBA9PsWT2GAwuyDJV6tRUlmLda4ctE8RW9uckrtCOofF7WDyrGPfMuMG1RxcuXKQtuqsLQ/N/nrj/jshqEjjxXFYVnlqV4dVFubi97Kt45lfG+YFSgi/OteDt/Z+hYtooy04TLfmcWK0pAtiy+wSm3zzcUP1Zu+gmrKkuhyjbbzgmw2mijVIG50NRw/09NGcsfrHjKBbPKnbngHYgxhQ9+QxoguL7UVfdNUHxVITUnFYAp5KolSVYigWuqS63HK/THAaRFMiSgi8uhAxjIIwxmerhGMydOsKUgPdwDKIW9hiLa24lxuXgqOm4/g6eJeA5I/UKzzHgWYLWDFwrdwYKJQhdipgK1vg8b8blPtIZ7VFw/ATAHQC2W/xNAfDVHhlRBsMqyEhsRz0TbMWQQVnw8ixqKkrAcywYBqiaWYzt73yC7e98gtmTr0NeDo8fbdhnaF8JR0VIsqxzQD6/7RBqF01EZ4mH2nYxgWCztTqyq2Q8sKEACEckw6S8dO7YjKS6ao3K2LTzI0wpGQYvD8REBZt2foTFd47JWHvO8rKoXTTRsEM7ENAeX5uHMrjYErOkBOgJ/yVB3ZT5yhAfLlwKIyYqWP+bowicCuLT05dQd3+5nhD08iw8PEWBj8eP7p0ESVbplzjOiRSD/WKwsxuPLnoHdhsmq6vLQOKJCsB6w9eXw2FKyTB9w1qDVv32f6Zcj9wsHht/21Z1/IOqCRh2eR4kSQGXTfDDxZMAqEmRDa8f0buqOjunt5eUcO3RhQsXmQKN5ggMUfleGQKWMiknTDvaZExOdjMMwY83H0DgVBD/8PcjsOYV8/zw4He+jtUb6wEAn56+hKcfLEdddZkqokaA5nAUD88fB0VWwLIMeEogg0EMwMLpN+ib3No5a9d/gLrqMrWi1YFvlmXFcRGS1Rz3/LZDqJpZbEm5kCoS55xgKKwWGfSTpLad8GRXn5smpJZM23LPjBs6TYGRSlFSe/fVlbSuJMnIy+bg4fIgyTIow1hSm0RiCt49+Dker5po6BK9o3y4Kd4RAX3NoI1zlU1s1F2dE+mKln64Vk4V/Sn3kc6w9W+BQOCO+P9f23vDyQzYLcCMQQbw17Mhg3DQhNFFgAK8vOMopt88HHUJLTcPzxsHjmPw9KYP8fC8cTgTbNUDEQ111WX6f58JtqKxKYLnthzs1AJSc6BVM4sN/JKJi+OemBRdZA7sdrC7ujPfF5Ak2SYYy0x7VijBhfNhA7/8svnjcfngrH6/Q9tetYYEYgoktYWo9P/XAAAgAElEQVTQ6o313e6/REnBU698iB/eOwmP/et7hr+dCbZCkhT8x1sfY0rJMBTkUhBCsPxf3zMkx1/Z+ZFlVYrhni0S6ysrS1HQT4Le/gy7DZOzwVY8t+UgVlSWIi/bbsO3HIPzvZbfD0dF/NPze3Q7uhiKInAqiB9u2Ie66nIwDPDoz9ratuuqyyzFjq3eCbsYp78vvly4cDFw0JE/c1oB7IRDN3FzTgFBMBQGAAzOz7L074Pzswz/jsZksIoCPl6F2twqYvXLbWvIlQtLwbEMatd/oK8hk8+ZSvzT2BxxXIRkN8fl5/Ip80wno7/POXZc5Opz68p5nfMaO+0GUADEYoohMbds/njLxBylNvdFCSyVCB2C4ynONkY6phYhCsaPvNzQJWrHAd1eUUvy++KULztT0d/Wyp1Bf8p9pDM6tCdBEHIBrAEwEsC3AdQBeCQQCDT18NjSEk4nQwIFVxXmosDHA1Ad7+I7i3HqyxAqpo1Gw4Vm+H1ePdH83JaDWDJ7DM4EWxFqiVk6bk1ZPvHfnUkKJzpQXzZn6XhlGYYxCEP9mDv1egAqt5LbWtv/kcqknO6gDIMJo4tMfGBdDfL6CpIMPfkMqL/Ls68eQN39/X+CtKvWAFETwmeCrRCG+jF78nX6b11Y4I0LtzFQoKTUctueEKccf0dsg31KUDFtFBoutCDLy5mqkpxWCVkl1ldtrLdtZ3SRPrCzjVBLDH6fF8FQGNleVu+E0jas1XlYQX4ub/n9vBweddVlCLXEsGPPSSyYNhLNrSJ82Zx6EIHhO3ZxhbYgbeNyViDK1lzO/X3x5cKFi4GD9vwZm0LiU4uVk+OORKdo0G8AQe2im1C7/gNL6sYh/izwHANhqN+SYqPhQoueBNTGverl+g7XkFbJR004UJQUVVuIUTmbY6Lzyly7Oc7v83RZuKu/zzk8BR5fdBPOJNBaDBmUDZ4CsS48uFRFt510L8kprDsIAyybP95UJEOYzvZtq4hKsKEWSarQVYhOQaON9fltahIxGSkl61MQV8xE9NSGSCahv//G6QInz/J5AF8AKAIQBpAHYB2A+T04rrRFZwKWxXcWgzIEF0NRw86h1hoeOBXEmWArvLz6c1gpJC+vKMG2NwMAYPhup5SMExKLdoEKy7QpGft9XiyYNtIgJtCfdqBdWINS+6RaV3aw+wIe3oYPzMMg2pJ5s6ok2WwOSP1/gkys1kgWPntx+RRMGF1k4j9cXlGC2kU3pSza19GGIxsP1qIx0VK9njIE0Ti3nF1VkpMqIdGmnVF0A6K0hx011+8PfI67bx9pUmnXYgJtblds1NybwzHUrN2rd1D5cnjTHD1hdJFeybL9nU8MfOSJC1JY2HliVbUW4/SnTUkXLlwMbLTnz8AQx4lPyhDLuEOLFQAzxdLKylI8/WA5AGLyyw/NGYv1rx3B3bePxI49JzF/6ghQShCVVb41L89ajltbQ9Yf+8IUj9RUloKnxJDU1Dh9gxcihuuvrCzF4PimvZOknF2yM4tjTIJ3qaK/zzmypEAUk/iHF5ZC7kLVOKDGyYN9POqqy3WqCp6iS7+H7brD4reQZQUsSwzaFSxLIMvo2sYBUUzvmVrZnHR9xXqssmKmAEklWc9zjCkfsnTuWPAcYymumGmgFNZrGUogZ/7tOUL7v7Gb8+ouOCGfHBsIBFYCiAUCgRYAdwH4es8OK33R3mQowjpggaI6biuOrNmTrwOgTuzhuMJo4FQQm3cdx5LZY7D20cmomlmMXXs/xd9NvAbraqbge3PHgRDguzNG48n7JoGnqblzbbcPaEt2a//WBTCg6Emef7p7vKkdYfXG+kzLQbpIEZQhWDZ/vME2ls0frwagGYZITNaTz4Bqw3Ub6xHJ0MmEMtB/Fw364mAAQJJkEEkGwwDf//n7epLtXGMrFt5xg6nyYc0r++HlWT35rH3ekR+TbHy69h2OqoHrmWAr9hz6Cx6vmoifPzYZj1dNxJ5DfzH4fW2zLxGJVULtgUnw2YnfZZzRR6cd1KorBiIhUCgDSjP0RhwgccNkXc1tWDJ7DDbvOo7S0VdYVujMnnydPg8zFIhEZezYcxJVM4tRV12GqpnF2LHnJM5caPvec1sO4mywxWSnC++4QbebYCgMv8+DNdXlWFczBXXVZfpGih2HpxafaDEOtbHD/ux3BpKtunDRn6C9u2eCLZbvbnv+rN3kdBJYqNy6yf5cixWs/OuqjfWQJLXSzu/zoHbRRDz1QDmqZhZj867j2HesAc9vO4TFs8YgK4vFp6cv4XxjGKKkqFWyFuPW1pClo6/AtjcDhjlj6+4TiCYlNbVq6uT13aqN9ZBkNbZJXhtaxSrGOa5tbulq8hlo/zdKd3Rkf0Ccf/jlJNt4uevra00Ysmbte1hc9zZq1r6H86Go7fzlZJ6z/S2IxW+hEDy96UM8sWEfatbuxRMb9uHpTR92rfw5fl6ruCn5vAyxXiNZmU2i/b70/W8aYqNk2NEz9JfkrCTDei1j4ff6K2RZsf6NB9Az6A042UBMfq0ogMzM2nQD7Fo1/vxFCPm5fLsBi9XffNmcnthj2bZzF/h4XD44F+GoiKsKczGt7FqcvRgGyzLI9rJd4sNK3O0LnApix56TePK+SSAgYBgYWswJAImQfr0D7cIakixb7mBLspxxrW+SJFu31GRoxTBLGVPVzNK5Y8FSAqmfBEJOkLxI3LTzOB6eb11pLEqpib1QyiAsyRjztctw599epwuZ/Ob3ba1YUUnBlt0ncNffj8Qt46428M3VVJaC49p8p1Vni9MqIZaaq6S03zvTAt/+zuloBa291UMZnXbDjv7qmit8qKsuA08JIpICQoBFs4rR2BTF+cYw3t7/GabfPFwXN9a+p1W/JX4Wao1iyewxuKrQB6rP7ZKpzdYu2aLReWgL/lTbejMdA9FWXbjoD3Dy7rbnz5Aihy5pZ52k/bfV3xiGIIejiMiKpY6ErCg4G2zFC9sPw+/zomLaKFxZmKvq9FxoBSFAOCqiaFA2WuOJaV8250iATmqnmlp0yAtseAbofgHaTJ1znM4dPVXhnQp1idOxMhSWcShDASXpx0ilAjkVOD1vZylICgt9OHs2ZGtbomy9lhRlOSPXkslgKcFtE4bhzIVmfd1/24Rh6tpygFQd2q0V3Y7T7oWTZ/muIAhPAcgSBOH/A/AAgN/17LDSF3bttJt3HdcrlywDFsDyb4PyvXjwO1+HJMso9OWgrrocodYooACPr3vfcI2393+GIf4R2LL7hKNJxQ7tCg5IFjsOKQRiLvoRFOgCapoa7n+89TEWzyru65GlDJ6j/aqlRoECr4caNge8HgplgOn0JvumwKkgLjSGbfxVe9xm5ucmAmiNxnB72bWGxPLyihJ4PRRiWNSVxqeUDMPb+z+z5KXTrpnY2ZKYEHRSJUQkBX6fx/B7+30ekAz0v/2d07E9SJKMAh+PJbPHwJ9n3+LMAjgfimLL7hOWbd1/OPi5zhWtfU+rfjN8FhERExWo5UH2T7c9nmrDwi3FpESmYyDbqgsXmQynwoB2/oylTEoJLDsuZ7v134TRRaCMWh0LQKfzSv4+Qwh+uvUQ/D4v7r59JP5l60FLWsRl88fjrXo1BmlvbkmMdShD9Grq5GM5loEYjvVIUjkVJP9GXg8LJSal/ZzjdO7oqfV1TwjrEQDZXs4Qh2Z7ORCYC5sZYn1fjFW1dApo93klmERPxSr9nYKDkDbawMRiGpI9cCIe27ViF23XhRFOegn/DCAEoBHAkwAOA/gnJycXBOGHgiB8JAjCMUEQlsU/u00QhMOCIHwiCMKTCcd+XRCEDwVB+FgQhA2CILDxz4cKgvCuIAgnBEF4PS6K2GdIbjXS2qUCp4K2dBYs2hLXiX97bEEJWlpjaI1IeOPdk6o4K1Hg4VjLdtgpJcOwemM9ppQMM4xJb4+lDBieQo630YBS23YbrYWdVRQQSW7XKVuN3a4VqytwW13TCyyAimmjwLGq0+VYVVAtE3cAZVnBG+8a29jfePdkxrbURGMy1v36aDy5pG4OrPv1UURj6R2UdzesfFOBj8dKK39FVeEfp35MklVfrHGhAW10HrGY+ty1YNif58G+Yw1YvbEeNWv3YvXGeuw71gBZUQzjC4bCKBqUrVYTyKqgkBM/J0kycjiKoUU+DM73YmiRDzkcTftFmBVSaW3ujyDxtuuXdxw1xQsrK0tBKUEMwJbdJ7Dg9tHgWAZVM4shDPXrC8O/m3St4XsPzxuHosHZps+yvCw2vH4k3oK7F8GWmKW92c3xw7+SZ2pHTSV2yHQMdFvtj/DlZaGw0Of4f768rI5P6iLt4PTdtfNndrQSdv6uvXVS8t8mjC7C3KkjULP2Pdzz5JuoWbsX4ZhoGbeAqPcxe/J1+kbkgmkjEY1JeHjeOLVS1efFs68eQOnoK7B6Yz2ee/Wg7Vo0ecxFg7KxdG7SPLSwFAxB2qzFEn8jv8+bEXOOU/vrqfW1xku+orIUddVluiaEFXWJ07FSqAnYRPAcY13RHNeuSLwvlau5a3MnZYjJXpfOHWt9XynEKk7oUoDUKTgyLa8RE2Vd5DGRvicmpv87113wcAQ1Se9kTWUpPJybgO5OOPFxxQBuB3ASwH8A+HUgEAh39CVBEG4FMBnAGAAcgI8EQXgbwC8A3ArgcwA7BUH4+0Ag8J8AfgmgKhAIfCAIwksAFgF4AcBaAGsDgcBWQRD+GcA/A3gsxfvsVmitGpQy2PD6EUP13Y49JxNI/407brpwVtz/XrjUCllWE3sLpo0CR1XOUcC6/F9r2c3P5Q1/G+LPAsswaI6JlmISBV1sF+2Nqie31TU9EbMQxwDXlQaqvoECa+GK9usC0xeUIQiGwirHfBwDqStBU5SPyAoKfDzWVJdDTPC54CjWVJcjJsk4fa4JL2w/jGAorAv/RGOqgCBDVX5wShmTT6MMgazYt84mLix5jtpWe/Aco1eMMIy6eVC7/r2U/VxPtbj2NgZ6R402ny6eVQwQoK66HHJ8Xg3HRDz6s/dQU1GCWbd+zVB5nyhQqMjAquoyxGIyGIagsSmM/BzeYPOyouDpTR86qt61nePji45MtbWuYqDban+E18Ni+iOvOz5+xzMzEerB8bjoGXTHu5vKnNvROilZEG7da4cNvrl2/Qd4+sFy8/dBdFqNM8FWCEP9yPJwJtHCzbuO65RJgVNBHDj+ZYcCdJIkw+dlke3lsDou6sxRgtaoiGX/8q67FusCnNpfqutrLfbt6FieEkvh9WQhylTGGpUUbNr5kaErdtPOj7B4VrEppiAK0bUrtK7AHXtO4t5ZY5w+QktEYhI27TxuOO+mncfxyF3jTAktp88qlfyDvSC4mYIjU/MaTkQe+zOiYRFDCrxx/6najpdnEGmN9fXQ+hU63IoJBAKLAoHAVQAeBXA5gL2CILzv4Ht/APCNQCAgAhgCdQ4vAPBJIBD4NP75LwF8WxCEYQCyAoHAB/Gvb4x/zgG4BcD/Tfw8hfvrUVjtXM6bOgIsFMsdN203jmMJzgRb8MyvDqJm7V68sP0wLoYiUBiCxlAUX55v1s+pIbEd1u/zmHZLGUpsxSSSaXs6syPXnVVPVte3awEaIJRDaQkJBK/+l3En9NX/OhHfJOk+9M4OsY1wRYait7oS0hGUMogoCiIxGbKsoCUsQVRkeBii+yZJkqFAwS92qFXilXeMUu139wlIkgIPQ3CpJYpHf/aebXUoC4BjGUtfzMaFX7XFg4dnbKo9gNr1H+hCLI1NsQHv5wai7Sb7OAAgkgwiyiCSBKqo9qqJZGZ5ODy35aBuJ36fFzFRwtK5Y/F41QQ0NoehyApq1/8R9615G8/86iDON0ZAocBLCYYW+VBYkGW5ULKr3h1Ilc1OMRBt1YWL/oDefnfbS3ZZCcJNv3k4hKF+/ftngq3qxjigiyCKAFhWrfjUqDJmT74Oa16p15PRVTOLwbGqJogmSjxhdBFuHvuVDgXotHE9+rM9qFr1Fl564wgkRUGoJWbquhlIMUp3oCfsT0to1qzd22FXU1RSLIXXk4UoUxorAb592/WGrthv33a9ZXKSocCMW4Zjw+tHULN2Lza8fgQzbhkOpov1S4nFN1q3YTAUNlVAp/KsRKgdZ4lr3S27T1jafCqimKmcN51gJXo9kNgdKWXQHBERiUmQZRmRmITmiJj21euZhg59oSAIHgA3A5ga/58C4KiTkwcCgZggCE8A+Eeo1dNXAvgi4ZAvAHylnc8vA3ApnqxO/Dwt0NnKYEkyKmz6fV5EYxKiMQWrNtbD7/OaxKoemjMWO/acxLL54+H1sPo1z10M44Xth/HdGaNRkOvRdwVlRYEsK2ApAwWqaJkkyX2+I2d3/RyvtRCGK3TYh2BsdkIZdFtJXG/Zo2zTYibLXRPE6Csk+h4QAihKv+ZiTQThGDQ3RkyVHd58j6oSGAelBN+5TcBTm/Yn7eQTKIxxw0sThsv2soZqaJZnLUVXKEPaBNwkGaCMZbXHolnFBruzE54bSH5uoPEId06MqO2/haF+3H37SIMffnjeOLRERIMYzvnGVuTl5KuJbUmGh2fx1ANlyM/1QpYVnG9sxW/f+19QhgGNryacVAcNZAw0W3Xhor+gN2OkRB/v93kxd+r1uPKyXHCUgkIxJKIS44PZk6/D6o31EIb6MXfq9aCUINhsnCtqF90EljIYlOfFk/dNQqglpiefk+eF5RUleO57t4DnWL17BrDvfkks/BGG+jH95uFYsXavZdfNQIpRugNO7S+l6ls41yRIhQPa6TxHGQbJNLiEqJ/LMB4bjcn43Yef4/GqiQYB7+/cdn2X7IinKj1CR5XdKek3EOdVvyyAlZWlWJVw/ZV23PApnDedkIlj7k44XWP2Z3A8RVQCvjjXDEqpZQdNV+HED1wEcAHAswC+EwgEPk7lAoFA4PG4iOEOANfDuI+i8k2oKS0nnwNASr/+4MF9ShltiS/ON1suLh+eNw5ngq04E2zF5l1tLSaXFWShORzDwjtugChL+Mefvmt0DFBVOzlWpQTRBCp+9u//baBOGFqUh1BrFKs37jU55Z8svQWFg3JSvhdZVtDYHEFMVK+fn+MBY7ETqEHduTRff8395ZYtQF4PC7/PazhHYaEv5XFmIvrads8EWyyrhuvuL8eQbvoN7Oyhs/Zoh7PBFtsWs8LBA8OeehM9abtngy2WlR1r7i/XfYMsK2i40Kwnn7Xjnt92CLWLJoLlCJ5YPBEEAMMwCIbCeOZXB3R/+f3vTsCwy/PQcKHFut3vH8bjioR3QBRlVEwbhYYLqqCQxpfu4Y3UHFoXixM/l87ozz64u23XqY8LhtrEMyltE0FJ5P7Uvv/cloN4YvFEUwJiRWUp8nw8/Hk5+OJCE0RJwQ9efN8QRDc2R5Cfw6OpNYZVL9ebbD5x/k51fk9X9Gd71dDX8UJfoTd+W6fXGAh21hNIB9t14uvsjtF8vCYUmOiTv//dCSjweSyTOrnZHIShfl3UrGpmsYHWUd0YN9IqPnnfJL0SOnleWPPKfj1WsUo+ghCDjZ4JtujHWZ3v+W1tY0qnGCWd3rOu2m4qa6AzF1oc/a4AcOFSGBNGF2FKyTA9dn17/2fw8CwG5Zl/R8220Y79n29sRTgiGWgZl85VcxDJ129sCuMbN15toBFbOncssr0s8nM7b0fBUFjnKE4U/L7/219HYWF2wnNx/qzOBlus6ULuHGM6VpYVtIhNBiFGnqcoKMg2Pa+zNmvoxLVKX8HObttb9/f1mHsLTtaY/RmiKOOzLy+ZNsWGXZ4Hlu2+KnAnCejvAvg7ANUAbhME4b8AvBkIBI619yVBEEYA8AYCgf8OBAItgiD8GsD/gXGT6HIApwH8BcAVFp+fAZAvCAINBAJS/JjTzm5NxfnzTWknNMay1HJxmZicCJwKYvXGegzxZ6FqZjFWb6zHizVT8MN1+wxVT+GoiAXTRkKWFV0sq2pmsZmK4+V6PHnfJBBCLJ1yOCLibDg1prvOVK+KNtdXoFgqTisxCWfPto2rsNBn+HdvozedT1/brsQwtlXD3fUb2NlDZ+yxPTA2lawMQ/rUnroDTt+J/mK7IrG2S1FS7VLbuQ2GIjbHyQhHZd1fPl41QQ+otWOe/MU+rKkuB8PAmmubGO2G9bKIWChH5wEGv/b2/s+wvKJEv7adn0tn9LYP7u2Ar7tt16mPo5TRbYUQ6P7KLplACEwLhdUb1Xk+GAqj4XyLya7rNtZjyewxOHcxbEh0aDZfV10GktA2non8hcnoy5ihP/jcdF9wpfrbduZ+nM6vmeLDnaA/2K6Gjn4bJ76uvWMi8WrTqpnFJp+s+tVy66ROdTkeXXAjav5N1YRI9vWzJ19nWsu99MZRPLagBKJkrxXEMMDjVRPg5VmEWmLY/s4nCIbCgJIUuydsdNrNM/m5fFrFKE7es3Sz3fbGnNIaiFprjUAx+yjWy2LONwVDrLm8ogSEmNdvTud6mVJLAb7V1eXm+7M5do3VsSlAJAT7jjVg37EGw+f3zEh6XpSxTMCb3gEAYBnrql9Y+H5K8fi6P5p+gzXV5YCUxLFus4aWbNbQ6WC3ci+s+9MdHa0x+z1Yatk9sOb+ckC0roLujO064YDeEggEKgKBwHCoooCLARx2cO6vAlgvCIJHEAQewEwALwIQBEH4miAIFMB8AP8ZCAQ+AxAWBKEs/t2745/HAOwBMCf++QIA/5nC/aUlFCh4eN44g7AEAGx/5xNLHtHt73yiTzLaDrvGq/TC9sPI8nDIzebh93mxorIUQ4tyLV+eYEjduXfKX9QROsPbbMefBAUpKU676HmwKXBddRap8Gl1BdEE4QqN12rTzuOIxlz7yjQwNjbDMAQcT3EpLEKUZOTneiyPy/KwelAOAF7emv5HjNMWWCpuJ/G2xGLWfHuRqJzg127D3028Brv2fqrb4ZLZY1Dg410/14/h1McltsFSSpDt5fDo3Tei0J9l+X2OUlNFmjbPi5Jia9denm2XCkaDq8vgwoWLgQAnvq69YzQfb+9XrZPFkqLo/w20dUhpsDrfvmMNoJQgL4e3nBeKBmXDy7N4YfthnXt3wbSRKpVH0n0ncv8mX1s732X5We5arAeR0hqIKDZaI+ZEYjQmY9ubAcOaZ9ubAcs1j9O53pbKUDFfvz2xvq6AMgQTRhdhRWUp6qrLsKKyFBNGF5melybCmMhBPXfqCPDU/FwVxVojyOK22rkv88GEwPK3TaYxSScwxGZ9lc6D7ma0t8YcCBAl6/dc7Gbh7Q4T0IIgfEMQhKcFQTgC4MdQqTTKOvgaAoHALgA7ARwCcADA+4FAYCuASgDbAXwE4ATaBAbvAvCcIAgnAOQCeD7+eTWAxYIgfASVi/r7ju8uXaEQvPaHP6FqZrFhcRk4FcTmXcfxvbnj8GLNFNQumogcL4u5U69H7aKb4OEpvjtjFGKihIfnjVN3KH1erHmlHgU+HgumqYnpUw1Nli9PY1MUkahomsA6K4zQHseUHdoTO3BFkNILLMtgeUWJ4bdaXlHS7S0YK5PsYWUn7bE9UIZBgY83fFbg47s90e2i58FRYkoKL5s/HiwlkEBwMRTBP7/4Pp579aDpuBWVpSAEBr9lt/BiGOAXb3wED0+xZPYYPWHs93lAkiZiSZb1DUAtKPb7vJBkRfdrHkbdQDz8p3NYvbEez205CL/PazqXi/6FVMSI9DkQAM8xyM/1YP1rRywXneteO4wF00bqQlbCUD8er5oAX7bq1xSlbfEjDPVjRWUpnnqgHHk5HsiK0uGitzPzuwsXvYloTEJhoS+l/7lwkQwnvq4jTt0VlaW6UGAiVL+qVhprfriuugyPV00AzzJgaZvQ8fZ3PsGjC27E41UTUFddhkF5Xsvz5Xg5XGqOWG6Oi5KiUytpY/zp1kPwcqxpTZW46Tn8K3k285TirsU6CU18+EywxVZgPSWxQsW5QByJ8w8nJmCn3zwcxCKR6HSup9R5sryniot4SrBg2iiDEOKCaaNMieVURBhlxXli3T45aR4rRxksmz/etFbh0ljMjqXEcsysReK+v4KzeQbcAHkGDGO9cdLdCXgneZ5noSaM5wUCAUfigxoCgUAtgNqkz94G8DcWx/4PgFKLzz8D8LepXDfdwVNg7tQRqNtYjyfvKzNQAwRDYXg8DC6GIgZO0trFN4EA8HAsnvnVQUObyOZdxxGLyfo5tEpqKxHDKSXDUH/sC10YgOMoOJYgHJEM4ltOoE0wya0olCGATVLFFfXJHERjEvYc+otBROKt+j9jWvnwjneuUgDHMQY+LY7r/snZwzP6O5dIkeDhGURbu5dY30XPo8DnQe2iiVDnQwLCAI/+bA9WV5fhjXfbuNxEScYjd41DXrYHlBKIsgwCo9/a/s4nJnqWFZWl4KjKDb3+N0cxe/J18Mb3L3K8HKSosTaE56jO45hI8cJzDOSo6ttc3zcw0ZnfPSop2LTzIyyaVYxZt34NoiTjyfvKEAyF0dgU1YWhPj19CT+6dxLONbaCMgyeffWALoJ1dZEPyytKse3NE6b20ofnjcOjC27E05s+NNg8pQQEccHiTszvLlz0JniOYvojr6f0nR3PzOyh0bjIVDjxde0dI0kyBvt4FPg8llSCXp7BmgfKcakpavjbyoWlGFzgxdK5Y/HGuyfxrW9ch7xsHr4sHucaW9EcjllSx/EsA7/Pi2d+ddCkT/HwvLG2lae2m57x/06cp7weFkpMcuOTTsKx+HAK8QELYN7UEaZzWgrgKcSS13jxrDHmsTqc6ykDLJs/Hs++2pabWDZ/PChjFufSEuuOxhp/Xk5EkSUA4aT4OxwVkeM1WncqIoxapXLy/VsV/bLxApjkd5KlBHLSjREG8Ofx+lpFVgBK1c/TFjZjxgAq1mIYAn+eJ+kZEDAJ4vP9GSxlHNt4l67T0QGBQGBs913OBaAqSQ4p8CiMFjgAACAASURBVKKuuhwgil5h5+VZKAqQl+3ByhfaRAn8Pi+ClyJoiJk5HZ/fdghLZo8xlMxrldRVM4sx9HIfTn0Zwo49JzHjluH43Yef42/Hm4UBNu08jmAonBLHY6oTjAYt4GEBQFIGxAudiWAYgvEjLzfYykNzxqo7vd30o4kAatd/YJr4LZWKu3IdUbbcDX/6gfJuvIqL3oAoy7jUFMVzWw4afJjf59WrPpI333iewbrfHMG+Yw2489avGlS0g6EwCnwerLm/HGI88cZCpUrSgm2Nj3/Z/PFQLMpNZFmx5ru732hfru8bmEj5d4/b8Yq1exP4G0vxizeOIXAqqB92JtiKC5fCiMYkvLD9kEkEa8LoIiyaVayfR/vOc1sO4ntzx+HJ+yYBAP56tgkvbD9siAGAzs3vLly4cJFJcLKWae8YUAbnQ2py2e/zYsnsMbjyslxwlAFHgXMXw7gQCpvWb6tersfq6jL87sPPTXy9S+eOhS+Hx4u/PmIWQb5rHKgufmjUp5Bk62Sak43DxHnK7/MODL7THoIIYEuSWN6W3SeweFaxaW3jND5IJVlNiGIZC1smVeFsrieSgrwczlAwlJfDWXbxpTLWVPQmFMBSCDF5BKlsoPMcY9JmWV5RAp5jICZdn0gK/D6P4RlYdUUCahL83MWwKZF3ZWGO6dh0gSLDcsxXFXZ3X3L6QpRlnLvYmlG/W3eCowQFSTZe4POAowSRbrzOwLGoNALHU5y5GEbdxnpUzSzG2/s/w5SSYfDyQH6uBy3hmMFpakIUD88bZ7mjd+VluTjfGDY428CpIDa8fgR195fjmivysPjOMYhERZT9zZUmriNNfXn1xnqs3ljvOPnnVvT1byiKWeRKU8PtruRwKrvUXUFUtL5OVOre67joBShETz4DRh8mioqlzdYumqiLlvzmD/8LAKirLocky6AMA56qG4OJCwCFMGBZYpiEWZZAkhXLBYSlHUsKkuiiXbjoGBachGteUQUEn9iwTz9M4+7UuEKTRbD2HWvAt75xnaVtFvg8eOmNo/i7idcYzqnHAO787sJFp+DLy4LXk1pkEY6ICF1q7fhAF90OJ2uZ9o5RKKMnz84EW/HEhn16IUVUAlZtrLddv0VjMsr+5kqDLoUW09QummiZZGYYAkoJVlaWYlVC0u6hOWPxm9+bO2DdjcM+gE0xRFcXT06T1Xa8xnXV5vWb07W8JMnI4SiGFvnUsmFFaTcmcDpWOw5qq1yEJFkXe6yuLjd05qZSICfGFJ0vW9ss2PZmAPdaVIsnPoMOq7UdjjWdIGbgmLsbmfi7dSfCURlv7fsMt5VeE+9+9+CtfX/GHeXDu7Uw0M299AGiEvTqu+3vfGKoWHrqgXI0NkUNyWRtcalxlSbv6LVGRRQOykJNRSnqXjFSDCiKgr+eDWHr7o9R4ONxz4wbLIMgX7ziKTH5l9wSw1OCqKSYnK5b0df7cNqu1BVIksprmzgpb3/nk25NDvdWm7fGaZR8nYEkrJCuSNWWZdnaLgsLvLZcbpQhWFFZiu3vfILAqSB+84f/xd+XfRUehkCEgtaYYqYgUohOUaBBXVSaq+YZhrG2L4a4qz4XKUF9H1ShYs22A6eC+mazZmeJFFyzJ1+HIX6zCJYw1I/cLN7SNv9ypgn7jjVg1q1fM1w/MQZw53cXLlKH18N2ihrErTftO3TF10myYhsrA2h3/Xa+UfXrVnFLczhmWtctnTsWP958AMFQGLWLbsKa6nJIsgIFCl564yj2HWuAL5vD6uoyyLJi2GBPRm+sIwYs2kkA9wba4zW2KopIpQqbACgs9OHs2VC3xASyAsv3R1ZgGqssw/a+EpODqRTIibKMfcca9CIVDffMsKatcXxfss19yUraJjJlG1+W/Hz7M2xtLI1/t+6EJMsItcQMn4VaYpBsaJw6CzcB3QdIVEROpMu45gofKMPg178z7mBrwhZW3M7L5o+HKMlYuXavofWLYQg2vK62nCcuVC81Ry2DIK3NREv+URBDS8yE0UUmDt1U6DpcdB9SaVfqCjw2vLYeloEU7Z7rdJbGJVXwHMXyilKsSQjkl1eUxlus3LRKXyFVW6aUAUMZk10umz8eHMfgy/PNlv7tsy9D2PD6Ed0PBkNh8KzKtZ9YQbSyshQF8WvLNmrXsmwRwMfVyc3VLi5XrgvnsHofEm32TLBFn+NDLVGsf+0oAqeCOo+5FiucCbZCGOrH3bePxKZdx0y2ubyiFK2RGCaMLjLxKbo8zy5cuHBhhF2sUuDj4WUIFk4fbeLG5TkGkqTYrt9WVJYi26tGE1ZxS1NLFFcX+fCjeyeBEIK/ng1h087jOg1T7foPUFddBqrIoJTB4lnFWHxnMULNMQN9k1VM1VvriIGKlOLHHkAqvMapgOMpohLwxblmUEptNzdSOidrjumXzh0Ljm3TUNHAUuuiJZYhZiJqh2BtCkhYhgGS1oepvDcezvq+PFz3raG7Gx6+nTFH0nPM3Q3WpjCOHSAFRV4Pi9vLrjXQry6vKIHXw0IMxzo+gUMMhGR+2kFTRNag0WVQhgELBfOmjtDFA556oBzXXJmHZfPHIxgKY/Ou41gyewxeeGyy3kKuVekFTgXxxIZ9+OcX38dfzoT03Txt53XBtJHI93kslZM1ZdcVC1UhIgnE0BIzpWSYiUN39cZ6xABbdV8XPQO7diWx/a+lDMmG1zZZGblL10jYpV5XMwV11WU9EgBLkoxtb54wqEdve/OEG2j3MVKxZY6nCLbEcPpss8kun331AM4GW7F198d4aI7Rvz22oAT1x77Q/eDcqdfjoTljIcuKnnzWzrNqYz2keJNRKmrXqaiTu3BhBbUajZjeh0Sb/eV/nsATG/bhFzuOwu/z4rszRutJkJwsDkMvV4UHh/izMHvydXh+2yHsO9agb3I/9UA5ahdNxIu/Poyfbj2EuVNH4KohuYb3Rec1deHChQsXAOxjlU9PX8KFSxH85vef6H/z+7xojcQQjSkAIahdfJNp/ba6ugyyooAQBi+9cdQUt6yoLMXVRT6cbwxDlCQwDODlWcyefB2EoX59DJKsGCqZoZjnEKuYqrfWEQMVDEMwYXQRVlSWoq66DCsqSzFhdJF1/NgDYCnBsvnjDTa1bP54sLTzGWiOp7gUFnGq4RLON7biVMMlXAqL4PiupdRTWWsq8e6wxPt6eN44Mwc0ZXCxNYaatXuxuO5t1Kzdi4utMctcBUNhnRexuK1U3pveWEN3N+zoJ6QBVJDAUbUwLtkfcwOETzEmyjolTVu+JICY2L35Ened0QfgKQwiWJpxE6JAVIDBPh6LZxUDhIAogCgqeHnHMUNLxL9sOYR/+ofxYCljucvq5VnTZ4Pzs3CpKYpNO49bKierYoYyXth+GN+dbqTqSG7t1c55NqgKGrk7572H3uJNFm14bcVuvk5vtHnHRMW6xWqmywHdl3Bqy5QyiMoKgqEwrrBpV/XyLAKngvj9gc/xxOJJuNQcQWNTFP/+VgDTbx6OzxuaEDgVxBWX5eLlHUdxz4zidu2bBVC76CY0XGjROaCLBmVbVuezAOZPHWGqpnZ5F13YITFpwDIMmmMiQi0xS5ssGpSDn249hMCpIIShfky/eThq1r5nqE7wZfM49eUl/P9//LMqQFzU9p4ETgV1HtG66jK9gq5uYz3W3F/u8jy7cOHCRTuwi1W8PIvVL6t6PvuONeidJ8mdqmseKIcoKmAYtSPv9NlmPPvqATw8bxz2HWvAxVDUsC4DAapWvYU7b/0q/n7StTjfGEaoJYa393+Gu28fqXfF0PjcoYl5P/VAuU1MBRBCwMR9fKSX1hEDFTwlWDBtFM5caAEAcKz6b54SxJKCwp6gQqEAfEmCgb4cDhSdLhSGBIKLoYhJADDHy3VprJJkY4sWGj2KArz2hz8Z3pXX/vAnLEoSd5QIwaqXkwpMXq43CYMDQCwmW+ZF/vGu8aZq9fbodpLH2ltr6O6E7ZgHkF6SDAJJkQ3vjqTIkLuVATmdoWDWrV/TtZa0TZ7uxkCxp7RCLCrhMh+vVjArCqC0cXcltnMU5Gfh09ONCIbClkIUDcEWDMrLsmwVSGyrFYb6MXfq9WAIUJDrMZxL/xtDMMSfg7fq/4wpJcNw+lyT4bx2/GXagnn1RlXNmSTzqLrodvQWbzKl1m1JtAutTn0F22fmckD3KZzaskIJLoWieGH7YVTNLLb8jhI/vHT0FXh83fuGv396+hKqZhZjw+tHwFEGFdNGgdrxgjPQs8axmGwItldWlsJuG5zjGEPAwnFuV4gLayS3cU4YXYSFd9yAAp/H0iYbLjTrSWOtsjlxYbXmlf1YXV2GKy/LRcW0Uahd/4Hte5LI7aYvLBTZ5Xl24cKFCxvYxSraGig/lwdg7Z+fffUAahdNBEsJWMpAFGUQAlTNLIasqBQdiZuEQ/xZqJpZDGGoH+Vf/wq+//P3DZRMO/acxNyp14NjKda9dhgzbhkOv8+LM8FWk4YQAEwYXQRRktHYFNE30r0caxN7MaBu61aXIQE2yVpj2qWnqFCikoIn4psSGjRhzM6uekRJ7hFxNltaDUqQXFrMUoIZtww3UUSwlEBOCF5Em6S2VSKVYRhbsc/kgMjDUUu6HQ9HISXRmVEbao90Xnfarvstfov+ipik4KlXzPo/A0WEkIDoyWdAfW+e23LQcvOmKxgIzzItEYtKIJIESoDv//x9A13Glt0nIILgTLAFwVAYe//ntKnlZOXCUrzx7klsf+djPLagxNQqUOjPxhB/FoShfiz+1g0AEG/lkvFYxY363xZMG4kXth/G4rq38cSGP+LmsV/BdUPzwXMUKxa2tSC8vf8z1CS1JDw0Zyy2v/OJPu6zwVbUrN2LYIt1m4uL7oHGm9zTbdMsJVheYbSt5RUlXWrh6itQSixbrGgG3kt/glNbluQ24db6Y19Y2uVVQ3IwxG8WYgNU/3T1kFw8sXgiGrSKFI6xtAnNvkWgXYqORIhQ+Rif2LAPNWv34okN+1C7/oOBEq+5SBGJbZxaRfMP1r2PZ3910GSTy+aPhy+H1+fsq4dYdwCcDbbin198H5eaY3j6wZsx/Op807u1dG7bnK191lstwS5cuHCRjqCUgUIZiITYUgpaxSraGmiIPwt5OZ5244/WiIiGCy1ojUj47MtL+MUbx7Dh9SNgCMGjC260PO/sydfhqU37TZRMU0qGoWhQDjbvOo59xxrw062HMHvydQCgc01r55swughzvimgdv0fUbN2L17YfhjBUAQsRyzvZ91rhxFsiUFOY5qATIBoQ2UgJhUJ9RQVSnvdhZ1Fe+JsXQFVFKxcaLTFlQtLQRXzeYmkwO/zYMnsMairLsOS2WPg93lAkp6rJjyfCNt4J67hkvwuWGm4yIqiJ5+Btg0m2WKsLGuzhmbTd93p4RlTrqemshQefuAEiprQfSJ9jt/nHTA+UewB32EFtwK6j5E8SVi11z664EZQQkzVdXffPhI/3LAPnzc06cJE5xvDaA7H8OFHX+LxqonIzlJbvRJ3YR+eNw5L547F4Pws/ODF9w2OdM0r+7FqSRl+uvWQrrAsyjIoQ+DhGJ13+q9nm7B5V5sYRnI1dF11OVjKuJXQPYBU1H27AlFSdB4grdVo25sBLL5zTAY2oijIz+UN75BWseKi7+DUliWpre1t6OU+vLzjqMkuF80ag9pFE8Fz1jv4X15oxhMb9hnaBgvigaxmEwU+j96iaBfAxyQZ3iTf1lu0OC76B+QEe0msmDsTbMWmnSpP6OWDc8CzFJeaI2iJiPinu28EZRh8ecFaaFObf5999QBWV5eBpQSUJahdNBEMUTkpY5KMYCisf2d5RUlciNWdpwcCfHlZ8Hpcj+TChQanFaharLLm/nLERBmnz6lroAIfj0fuGodITMYTiyeCY6mlf872cnoyWUtwbd51HM9tOYgHv/N1LJk9BlcV+kAZgnWvHUbgVBCFBeZW/8CpIPJzefzlTJO+/joTbIUvW6VBCJwKYseek1hdXYazwVYU+rN0QULtWLVqtSwee5XjXKNaOa2t6T49fQk/WXpLL/4K/Q/2yVpj5V9PxY6pCOs5PmcKlcpAm2ChJMugDNOuYCHHJnUQstYJT0mS4fOy8BTltXtenqP4QdUEnI3T5ISjIgr9WeA5ao53EjRctHdtx56TKhVqEmKizbpAtKLgyLw1dCQqY+vuE4Yxb919Iq3H3N3wcDTjxCO7Ewyxfs9JN1fuu5FoHyO5rcuqfSvUHNUTyBqG+LPwvbnjDE7iw+Nf4qYbrgRDgVvGXY0nNvwRP7x3kmkX9rktB1E1sxgMCVs6UkVRSf4bLrTg2ivzwEoKKAjONUawemM9/D4vFkwbaVjIPragBOt+c0Q/x7nGVng46vJC9xB6gzdZlq15k6tm9o6Kc3dCkoDNu45jSskweHk1iNi86zgWzxo4k2q6wokt8xzRAwKNM9HKLhsuNKMg14PlFSVY88p+Q/CwaedxAMa2wU07PzLYxKadH2FxnEvOruX29LkmDC3yGeymt2hxXGQ+KGUgyopuL8kVc5qY8LqaKXpi4O39n6Hyjhvw+Lr34fd58dCcsQaOUS2ZAbQtcmUAP9qwTz/3ispSvL3/M9Ni6N5ZY/riMbjoA3g9LKY/8npK39nxzMweGk1mIBqTUFjoc3Ss0+NcpA/sKlCtqAokSYZCGfxix1FMKRmG+75VDFmBgSLjsYob8fC8cQb+zMcWlGDjb4+aKpmrZhZj9cZ68ByFh6fgOfWK3/nm9Zh5y9eQ5eXAsRFs/O1HCIbCOv1Gfq4HoiSjrroMoZYYjn96Dvm5HtRVl+kUG0y8jbp20UTbRKikyJAIwWP/+p7p7zFx4DCe9gScJmt7KnbUhPWSk2gMhYGqQh+HAx5qhiF49O4b8fTmD9sK5O6+EQxDTKyMHE9xLhQ1aF3VVJbiMh9vShZrHYRO6EIoZXA+FO1ww0iRFTS1iIbiu2Xzx8OXZS48YgFUTBvV1h3JElRMG2Wp4cLYUveZ6ToycQ0tSa5ekp14ZHdTUKQrWBvfkUxz0+XrdN+pXHQGWluX5kzzc3lTsFCQ67HcBScEBi7nu28fiZq17+lcp1prjFXwkZ/LW3KFDfFn4a9nm/RKQc2xJwZphiqtQTkghOD/vvOxoRq6sSmKDa8fwdMPlkMB4wocZSDsdsGYNOavsoOsZF4g4KINsgx9MrTjo1cURQ82J4wuwpP3TYKiqAH+j395QPdPQFvboGWgNeMGXYQw0TcnJvoeuWucYfK0OnaFK0LowgIigJfeOKonKezsOXEefmjOWLSEY/r8u3lXm2DOoDwvnn31oGH+ZRkCUTaKyfiyORt7l91A0IULG/AcdZP2/RipVqBKCXHDispSfa2lfe+pVz7E0rlj8XjVRDS1RpGbxSMmSSa/q20+DvFnYVCeFxxLQCQFIgGiURn/svWgKe54ftshPHnfJIC0xUMTRhdh7tQRqF3/R/34lQtLkZutUmxcDEVsKmHVhJldApRjGYiiG710FgoU00bEw/PGQYFiSKqmGjs6rSqO2gjrJceugPMuAFmWAQJDpTIILKkJolIbbR6g2ntdvDvatLGTwjsoAtiSVKG7ZfcJvXBEQ0ySLakyVleXdZGvmnGcnMvENbSrl6RynVsJTYrSwIiVCQi8Hmp4z70e2u0bkgPhWaY1zC3oxrYZYagfCqAHOVowsmPPSYPQYGLldGJF1fnGVktncll+Fi4ryDJNfMmVglolQPIEoVVp/fyxyZAVGYf/dE4/txYs+X1eXHSwU+kiPaHxJidPtLSbd8F6A5kYCLhog5SgzKxxHCZWgK6oLMVLb7RVGO071oBPT1/Cg9/5OgoLsvVuDQ1aJUp7lSeSJKPA10bbEmqJJSjPG6tTeosWx0XmQ0tgzLhlOJbMHoOCXA8eW1BiaM9Onoef33YIj1dN1O1VE6wa4s/CD++dZOhGWllZCo4CCoyxhF2i263Sd+HCxUBFqhWo/4+9cw+Tojrz//dUdVX3XHpmWhiIGvGKLXJJABmUQZOIYR+vqDwroCvgOqKMiUTJqqAJaoRBXc3msmiQRMREIFlFo5hdRDeKqIDi/lSEBvECUWEYaKDn0peqOr8/qqumq6tq6J6ZnunueT/PkyfSU119quqtc877nvd839Tj3fSeBcZwtCWGeUs2IjgogJ8ka/ik/0Y0ruC2KSOx7MWPMG3iWejnlwEVtgJQqdnSjDHc83i7pMaEMSfbAn0Ln9qM2ZNHQJZEeESGeTNrLJmo99xQAxH6bjO3AGhlmRcHowkQnYQDL7zxqSWI9cIbn9pkHbKZO2aTVSwKzLGwnpNdZxrU5Zzh4RX24mzOQWXN8d1QNXsQL6t3kAGXn3+6bQdYegPcku/SJVAAvch5+JC9YGRpdZltFUAEd5Hu47Ys8EL0oQuxzd2N7CLBIUsitHjxV/Zx8wa620ugAHQekLoFXQS3TAamTjzT3EoOtE9GHrxlHCJt7RnMqZnTqY7mH/+2w7YKO/e60WiNJfCL32/CiDP6m9WZBYHhkWfsmYLGoOg0QOzZ34zTv13hqCO2oG5sxlvbiPxD1Ti8snUVzCuLUDkvuOfnleyT8Hkza+CVGFwkyYg8InVhLrQnjGde2Z7UTCxHQuEA444ZRqU+Cb9cudXWB86fWQPZw46ZecKS2tOZZKf0hCwOUfgYY6nAGO5ftgmAvtBsOH/VgRLHcTihqjZ7vX3aKPzXazv1GhDV5fAkM6IORuJYuW6HZaHGKCTcQFn6BEEQALLPQE093m1RL9KaMP87tCdsFotPXWScN6MGrTE9K9Xo66dNPAuxhOoYODOypRmzagu7BcF9sgfLX/4E118yBM//rx5crCyXEfB7USIJZsDSLQAqCIU2y88vPNCfZ3fOHbPJKpYdFh7mzayBLDIk0n8g46Cuc1BZ0+w7SdOT6QAjqGzXoM7qHeTMJlP669UfoKHeKo/groFtl8pwKxjplC2tqRySR8DA48ogMEDjXng8DJrDYpWqac4+tJa/PnQx+f2dRevjEhzxhIalz3+MyRcONuUplz7/MeZeN7pbg8YUgM4zUicDGgcUxbnD5xxY+vzHFqd1Qd1Y+GQPFFXDndPPwcMr3kNoTxgvvPEpHrxlHDQOfNPUDEXV8Ivf69qQ67fsxfotezEgUIJF9bWOmYLGhMRpO/pLG3bj9BOHwwMOryRaMrVP6F/e4baaTDSnCGd66t79Zf1Oi0buX9bvxKyrCk8zNJZwKayQtsJP5J5sbVcUBbQkFMuqfDgSheQR8eifdOmB//y3Cx0nm5XlMsKRKJa//IlZ2K3pcJuZ3XGszJPU/hiMAZxTP0V0CWMsDUeijhnNDfXjHcfhUq8Hf16/E3WThuOU4/1QNY7nXt+FvfubkVA4jCLsKtrlsg5H4mbgoX9lCWQRlKVPEASRJNMMVGPeEtM4KkplPPzj8VA17rqLtMovmwHA9Vv2ItIaNwsDBip8+GWKbBKgZzIvWr4ZdZOGd5gtrXGrBm1HQXBjsX7yhYNxyvF+89rSs2Vp8bz7UVUN/fwyGurHm3bVURG+jM6ZRVZxXOWZ+zwZBnUFl0Q0QYAtqJtN0k82WeCZBsFFcNwzswYLUzP/Z7Zn/lvPmXm2tALg3sffdsgCtyfWMc4cfeh8rrvB4Oz331yAfn9nUVRnG1P6SFF5UWA4+9QATqwuhygAFWVenH1qILkjoft+py/cy4KFc2BvY7PzKqLYvr0mOCiA2ZNHWLaP/PRfRmNhfS00lePgkSiYwBCPKRgQKIMoMseXqzWm2LZemKuQycH0wVvGIRyJ4UhzHC9t2I1pE88y/94eqAHAGVSNY0HdWKxaZ9WHFgUGESwjzSnCTqZ6Xd3B1T8YjEf/9L4le74QI7apun2pGHq/RM/QGdtVwXDfk+9ixBn9saBO363hEQW8/NZus1+JJxTHwmxMYMmFtRjCR2P4j5UfmN+58YphYMlV/Y4cL8M5q67248CBCDlnRJdQVQ39K70I+L224MXdM8Yg6mDL82fWQJIE1Aw9Hq9t+RITxpyMZS9+hJ/dOBaKym1ZzQG/zxLYBoCl8yYgoR7b3gmCIPoSxwrAdjRvSQ2ceQQBggjMvW4UvvgmgrVvfWYJAGoaxy9XbsXc60bbFhmNXaxOEmOp2dInDSy3FFl22tly14xz0NqmmEUKX9vypR54VDVAFMDF9ro8XklALMGPqSlMZEemxfKMYzNJysgmqzgbn0fj3FHzVuPWoG42mcqxhIY3t+7FgrrzIAqAqgHrN3+By8af5uhCZroIIggMY4cOxIQxJ5ttfW3Ll7YguKpqqMpU2kR0vq+SyMDTFBey0asWwXHtP52FhU85y9/kI5wVj9/fWVzfsz4i2emVBVww6iTcv6y9rsC8mTXwygLibd1nuRR7yTNSJzo/vua7eG3Ll47OaGtMwQOzxuGplz/GZeNPs0ld/Psf38ed15+DflU+eGUR9yzZaH7//lnjHF+ufU2teO71XVhUXwtN4/CIAgSBIZZQIYr6MOQVGPpX+hDwezHryuGWDl1VNXhEAeGWhGNGQDgSxX03nQsGhpimIRyJmo4yyXNkTjZVu7uCR2Dwl0m476bzkluNADAOD2N5O3i6katK00R2ZGO77ZNyTa/ezZhlQLx7xhh83dSCTdv243BzDF8faMbC2bXgnIMxhs3bvkF1VQk8HoZHnrHr1tGzJ3qCdOfSKwk4cCSGhuWbLRJYCYVjzd93oWbo8TgQbjFtWRAY1r71Gda88Zlp969s/ByN4TY0HW4zF56B9vdp9uQRprwHQPZOEATRWdLnLQG/D7GEggQk3VcSBHgFQFVVaKre5xu7Qddv2QsAyUDOKNw5/RxUlXttyT6VZV5TssMoMltZLqNfpc8iyWT8v55ZqweNfbJg/tsriQhHYvjNn//PJr2gwRpIv+p7p+GCUSc5agqnbTUMUwAAIABJREFUQrtVsydTXeVskjJ8suCYVeyTBcTSAkPuEhQOEhge5qh56/Ew8Hj7cdnsCFQ1jjVv6POWVC4ed2qXAk+yyDB14lkZSYtkGtRmgrPuMROYTfdWFBiu+t5puKjmFEtg3W1+VeLzWHxojye/IxwCc/b7BcZsGtfFChPgktCEPqFXF0tozlI/t9qlfroCBaDzjNSJjiAwXHHB6fjrm7st+l1/e/tzizNaXiJbBhlAN5iKMhmxmGbqjhmfL3/5Y0c5DaPAFgNQIgmuxQ5YXE3qVTN9UsKYOSlxCjAZWkoekeFIJI77nnzL9ruhPWHXVUTCSrZVu7tC+GjMNiiXDZC6+VdyT7Y6f0RuyNR20yfli2bX2grzLH56CxbUnYcrv3cGysskVPl9ZnEeo79684N/4NN/HKZnT/QKbs7lqnU70Bhuw979zfhHYzNOGlCOxnAL9u5vxsDjjuCcs79lseXbpozEJ5+HEdoTxuKnt6Bu0nCs37IXPtnj+D6d0L/cdD7J3gmCIDpP6rwlOCiAm64ahlhcxfyUxJ57ZtagKhk0dJpv3j5tFBSVI+CXcfBIFCvWbrcEJ//r9Z24e0YNFj+9GaE9YSx78SPcNmWkvnsrmS0dHBTA5AsHo7JcDxB7BQaA48DhqPlbC+rG2hYlG5KL/IDVP7uo5hRzUd96bLv0Qk/uuCwmmIuuMkvLoswmKSMad5MSHGE71iMxS6a8ES/wSAxK2kSAa8hY8zbTHYG5SvqJp+z4Mtra0MUErHhCtb2PK9Zux9zrRtl8aq8k4KKxJ6PxUIupkXzR2JPhlQTE094HLjIcONBq86FPdChumC8IrHj8/s7COfDSht0We3hpw27Musr+nhUjqqo57ohQ1e6NMVGsLw9IXV0WADMrWGAMy1/+BJMvHAx/qYTyEhm//+vH5pYaIwizqL7WsaPnACJtcZuDumnbftx81Qgsrh+PhKrh66ZmM/hsOKrHKnbgNikp8zk7xJqmrz4uTDtnanXn7srSKvbV+p7K5nUrzOBU8CLfIS3f/OBYtmu8uwm0T8qDgwIIVHgd+xWj0vySOy/EL5bbHakFdefhT/8TwqyrhpP2LdEtZDO+uDmXdZOG43AkjusvGWJxUOdMHYl+FSX4+dK3XcfJxrBekAoAonHFZeuoQPZOEATRDaTOWyZfOBiRlrgtyLsw6R+pTM+a1PV/jT5YwNIXPsSmbftNWQxDQtFgQKAEPzjnJLNWhSgwPPqnrZh+6RDMmToSf31zty2gOX9mDar8smWMcVuUVDVu/nf7dcHl2PaxItc7LovVX+Md6Cqn3rdsEopUTXOR1XDQgE5oWP1qyBJEWv1qCDddOdyma6y6aN6qqr24YKbkKuknFwlYoiA4vo9O0iaKBhyOxCySp3OmjkSZzx6gzaa4Yb6QUNzaPD5v29zdiALDNT88E0eb9fR/yaP/WxT6Rha4LImOOyJkSYAW7747QAHoXkYvsKVi/6FWczVt1tXDsPT5j22TlIb6WtvA0xhugygKjttHmg7rHYhRnNBYxQhHotD7VX1776CBFbhz+miAtxdJUJmzTrRR7IAnNahvnzbKPO+i5ATMLcDkNnAY1Z27Y3DqC6v1WVU37gKq5qwLVqhZ6qTl2/t0ZLup20NvnzbK7CsmXzgY+w62uBbaMTTxnWxVFPTjwBmYqpL2LdElsh1f3Ma8fpU+3DZlpC377FerPsCDt4xztGUj6Jxq9/4yGbdPG2XuDjDaI4KbmXhk7wRBEMfGLRiaGkwz+mGnfr3pSBvu+u1blnGBcQ4RHDMuPRtX/+AM9KsswQtvfOoorVheKiEW17Du3S8w5NT+CO0Jm4lITuOF4XONOKO/KfXhVpRQEBgYdP1cw49UNbj4a+2hplzuuCxmf821YCBP2+2XRUKRRxQc9Y89ooD0tGZN4zgciVs+OxyJOxbWy6a4YKZkU1gQyEYHu/sTsLwSw4KbzkVjShxmwHGljgUTFVXLOKisufjQTs8gX9A05+xXLZkg2RfQNA2Kwi2LDHOvGw1N6xsSdprmvAjhtCOiKxRiHKmo4CJD+JB1Ne2Oa0dj7r+MRktr3LKFxi3bSc+a9mL25BFm51lZLuOdD7/BiMHV+OubuzFhzMmoLNedVUlmONIctwjj3zZlJF7asBvXTjwLVaUSONw6eQEiOMKRuKXNhpQGY+6rnnAZOKoDegXZ7lj57il95N4kq+rGXUCWhB5ZBSP6Dh3ZLtD+7qY6Uf5SyXTA0h02zjlmTx4Br8uKrSAKrpW3CSJbsh1f3Jwlf6mEcCTm6KBKHud+V+M8uY22BpXlEh68ZRx+/9ePcTgSx+zJI3B8/3LIHgEi5wXvvBMEQfQkxwqGGsE0DmDv/ohjv34kmTHnNi4oKseTL3yEy88/HS9t2I05U0eif2UJGAOOtsTxyDPvIxyJ6vJhW/WAcjSuIByJ4miL83jRdKQNl9SeCgBYv2Uvnnt9ly0h6e4ZY7BqXQgfftqE+TNrcPUPzoCmAbGEYvPX9ISAlPuSwx2XxeyvZVrIzAPgnpk15u5gQ87FKRlL8giY8sOgTVZD8ghIpAWgvZKIWVcPs2Rxzrp6GLySADXNf8tVUlOmGsxOiXgDjytFmSTa5jK5aKuicURaErY4TJnPHiLTNOfFJ6egsptf4vQM8oVCbHP3w8wijID+fB/90/vdHoDNVzpadOzsjggnKADdyzht0Xjs2ffNIkJjhw7Eg7eMAwODJDHcc0ONraKqomko80n49gA/Dh2NIqFwPPPKdtxw2TA89fLHtm1bD94yzjyH8ZvGFt+Fyzdjcf14yCJcOnldniN90vDr1R9g9uQR4NzYetZeIMPIqvaIgmNwWoI+UHVHfKgn9ZF7i2yqG3eFnloFI/oOHdku0D6xS60EH2lNoMovg3OuF8YQGI40R1Hpl7GvqRUJheNwc8zVVivKJaiJvjJxInKJ+/gCIKUWguE0pW9DHTt0IG68YhgirQmUl8iWbDRAd1A17qzHuHB2LX58zXdRXiphz74I/vudLyyZUE+99DFuunK4ntaWRrFucyYIgugOMg2GMgYMPK7UFuS9fdooLH/5E/O4gN8HDiQlORgOR+Lm8YcjcUy/dAhKvRJ+9ru3bYk8Dcs3Y/Gt43HxuFMhSwLumVmDQ5EoBgRKEPD7TFnGaFyBomp49E/v4xc3j0PN0ONRWS5j4HG6nxeOxHCkOY7Vr4Zw+fmnY+/+ZkuR2gGBEtw361wsvnU8FNXqrxnksn6KVsT+mizCMatWFmELlEqSYEkgkyTnXNN4QjWDz0C7DKebLGI0ptqkIhzPq3K8uXUvFtSdZymsd9n407q0EJDpvMMpEW/O1JEoddBKzkUClqpyPPasNeD42LPvO95Xj+i8IOMRmd6JpJ3X0S+pz18fWuPObX7oR/nb5u5GUZ37JaUI+qVMEATmuNNCEBi6U4OkL9zLvMZtAPbJ+qPZtG0/Pv/6KBbV10JVOY4r13XFNA3g4KYmtNFhr1irF/S7aMxJYAIwYczJNh0qt8wrf6mEgN8HDUBbQkO/Sq8lkCxJDG0x1Tw+/fvf6lcGSRJw8EjUNlmp8suAyl2D091FT+kj9yY9dY09tQpG9B06tF20bwc1KsHPnjwCp55YiQGBEluWjsaBeUs2AtDliZxsdf+hVvxy5daiyKgheh83+/3qQMR06I2sOUD3R/xlku7IJLPc7n3ibYsdG9lo0biCKr/PVY+x6XAbfva7d9BQXwuf7HFZyLHrOxbzNmeCIIjuoKP5rjetDx07dGCyrsR4PVgqMvxuzYcI7QkD0IsFTr90iFmkcOzQgZg9+TtYOLsWnOtzdCYwHDoSRd2k4Xju9V0I7QlbtP4VlcPDObS4iv5VPlT6vbjvpnMRjsRsge+A3wcAWPbiR2gM2wsRAsDnXx81z234l43hNty39F19ByrngKragqPZSilkiigKUDRetP6apnIoimYJqt5zQw20tOtSALy66UtcVHNKMvjrxaubnIO/qkv2bbqsBwAkXKUiHLR8GTB6yLdMiRdjMcRp0mwElRvDrYAouEtliAIOtyZsmd1VDvOObLSSVRdpka4sWrjeV4dzipw7JgKK3J7h7SrDksc+dEJxbnNC6RvBVwAQXRYZxG4OwOYrosAwdeJZtgTU7tbA7iuSLnmLJ7lNJxVD49GgMdyGA+E2zFuyEQcjcXigG8i9T7xtKUj4q1UfYPKFg3HRmJNwSe2p+PpAMyrLZVtncqQ57vibiqolJ01vYVbDa/jprzYg0hpHqVdEpDWOn/5qA2Y1vIavDjQ7ft/jERCNKY5ZBJ9/fRQtCRWHInHMS55/3pK3cDAShyh2nxkaq/VG+ywSIEVCT12joQuWiqElRxCdoSPbNbbWGX8LR6IYeFwpVJXb+pSG5ZuBlHm8IU+UitGPGhM+gugqTvY7Z+pIrFq3E0D7eMdFhnBrAvOWbMSND67HvCVvQVE1RztujSqYt2QjHn/uQ/03xI7nBJHWhKu9exz6ZrfMPsV2JEEQRN9EdJnvigKz9aGbtu3HvP/cCIDDKwDgwL9ePgwL6sYiOCiAqRPPxK9WfYARZ/TH0nkXYcalZ+OrA8245/GNmNXwGu594m0cPBzFH/66Dcte/AjXXzIEwUEBNIbbEKjwYkHdWAAcXBTgLZHQeDiKO3+zAZHWhC1Q98uVWzF14pn4uqnZ/NytEKFRbyfdvzzW/EhVNTBVg4dzMFXrloVLBcDv//oxbpsysij9NQWw7TRe+JR93BVFhvNHfhv3L3sHtzz0Ou5f9g7OH/ltiKJ9LDeyb1NxG/fdktscdWxdCiaCW89rLGbr85pXMW/JRoRbE44+vAqGZ5OZyg31taibNBzPrtsB1SGqbWglz59Zg4b6Wn2B3O+D5mBmhjTkshc/wrwlG7HsxY8w/dIhkB2yxkVRABcFKIyBi4JrrEF0ua9Oz0AQGSSPnrHeUF+L2ZNHQPIIEJyOdYnv5LMP7Tb/9DhcX7HilQTcN+tcLKgbi4b6WiyoG4v7Zp0Lr8vOhGJD07gZfAbafZXu1sAuhn6+oBHBbfpPRiazQWogxdgS5rSyFvD7cNqJFTj9xEo0HWmDLIkI+L22lZzXtnxp0ZY2Bv2Kchkvvbnb3NoiywJKfR5EE1bHedW6nZg3owYNT1vb3HioFR5RcM3o3n+o1VY5urv1vnK1Wp9P9OQ1puvuuq2KE0QmdGS7CgSsWrcDD94yDhoHvmlqRktUce1TVJWjob4W0biC/sk+LDXL09jOWiwZNUTvk26/gsDwyDPvm5lvgD4Oqw4yVW47j1Kz0RqWb8biH423yV8ZeufzZ9Zg87ZvcHHtKbYt4PfckCw+mN7mIt7mTBAE0R10JDURc+lDwYBwi313ib9Mxogz+uOS2lPx1YEIANh8n8eefd/MSDYyn5e9+BH8JTIeeeY9S7/OGMcDN4+DwJwzNU/oX47Hnt1qfuZWiDAaVxz9y+6cH2Uqu2DIsR2OxC1yCv4yCapS+P5apuOuqiJjWQ3Rw/DzunNxINwu61EdKIXoEaCmaUC7Fyy0S0Vo3CVYza2ZutlodnPGcd3FQ+ARBAgMqCz34rqLh4AzuwuZje6wpjpLlKXLWmSz88sjMNxx7WhThsPQgPYIzDafSmjAfU++a3u3HKUpGS84H1oUmG1uOWfqSIgO96JYYQyIx627F+bPrAEr6+2W9Qw9JUFC/kcvo6oaqlIcWo8gIJrQi04AsARSgPYBLL1qbXBQALOuHob9h1otHccDN5+Hu2fUYHFKsPiKC07HKxs/x4+v+S76V5Vg38EWPP7chwhHorh7Rg02fLAX5w4/AWU+GV8daEZVuddijKE9Yazd+BkWzq6Fxjm+PtCMFWu3Y+ZlZ5vZ1emdc6Q1gUCF12WQAzhjenE7FVA0LaOgqttExyh84E3+PaZxiB1sFSpEMi3u0BU4B97fvs+mC3Zpv9Pzefwk8hw32zW21gkCw9/e+gz/dO6pEARdV9dJK5dzjnlLNuqTgxtqsPsfYSyqHw9V1fB1UzOeeWU7wpFot+kVEgRgtV9BFDB14pnwyR5EWhPYvO0b/OCck9B0pM1WSTyeUF3HRoPGcBviCQ2r1u1AQ/14HG2No0T24KmXrVJb/lIZVWUMv7hlHDSVQxAZSrweKNGErb19QZaKIAiiK3S0OC6KzgXlwJlLQG48rv7BYNz35Du4fdooeDzOi+jVyUzDxnAbKstlzJ9Zg6de/tiWNfvArHF46qWPMf2SoS76s/o4tGrdToT2hLF52zd4YNY4HGnRNaBf2/Ilpk08CwG/Fy3RhMW/7M75UTZBP2NcCu0JY9HyzWZ7GuprLedL9/EKhUzH3Y5kGtKvV9OA5laHYnklku33ZY/zNnrZwxBPC0AzBse2ptVL7CCrGjZJCckjQlHiePDpTebvGwUTlbRguZpFvSFV02xzq+de32WTtVAAvJGhrjUHR1mpR68xwwCNAyrXwGGfHykuEmmK6hCcc/GhLxt/uu28+UIsoeGTzw6ackGMMfz9/b2oriotqPevK0RjGlam6YyvXLcDs64c0SdiH+nxRcC5gGpX6Sv2lHekDqwc+oNgSQ2uMknUdZ458I/GCJ55ZbuZYWUMYKJoXaWaOvFMxGJ2zaef/+4dLExuE/HJHhxX4cNjz25FaE8YNUOPx4Kl71iMbPWrO3DNRUE8tKI9O/qu6WNsAaBIaxySR0A8oeL+ZZuSn+krrOkrfndNH4Olaz7C7dNGORq1qnL8cuVW2wqo08RF0/RtaZrGoWhWDezU4zOdCBVqcaaeaLcsCebWsNQJhCwJUFQK5xGdw9WpYAxzrxsFSRLwvVEn4edL37bYHQBLEK7pSIrT95ReWIcB8IkMgwb6Mfe6UQX1ThOFhSgKOByJW5zBu2eMwepXQ7jigtNt49nP6sbaMpudstFUVc8Mu+FyDbJHNN8DoN0x+/mN5+LrphZLZtOt//xdx3bmsogUQRBEseC2OO7Wh2ouwUON67rQjeE2RFoT+PaAckffx18qITgogHAkin6Vuo5zuq5/Y7gNLdEELj//dKx4ZZvNv5ozdSQefuY9hCNRzJk6Ep98dhCjhwy0zJ/umj4GVX4Zalwx/UswAJxB0zQo0DNmuzpPyiZD9ljjkpsPV1VVGIumhqScLQAsMovOtig4ZyrrgWrrOVWVY83fd1kCY2v+vgs3XTnCpqcaS2iO2+idnoUsCc5tlQQoKTZhSEqk27Hg0NZEQss4szuboK7kki0tSQK0eKpvzxz9V1Fk0NLaqmkc0aiKR/7YHiz/t385B2Vee2DfLTjndA8kSXRsg5THPrTPK2L0kIG45/GNljb7vCKUaB/xoxjH5eefXlCZ692JR3TJgnd4d7r0O913KiJTjhUcNSZBA/qVo6XNebUaKkfA7zUDy/2rSlw78aPNcZxYXQ4OfVXWOJ+/VLIdP2HMyWbw2fj+Qyu24P5Z4/D510fRGNYLakydeBbu/M0G1E0abnbGz72+C9dfMgQvbdBlPCrLZVSWefFfr+9EOBKFJDHH7SiCyDD5wsG24Hn6xEUUBXy57yge/MMmy/cPR+LmKrpxvNtEKHWbTqEWZ+qpdicUlwnErc4VlwniWDjZ7n03nYuEomHhU5sR8Pvw038ZbXt3Fz+9BffddB6u/N4ZiMYVeGURT6752DyvsQ1VL2qa+90BRN/FWEBJAKZ0FtBup3WThkPTOH7z5/+z/K0p3Ib/fucL03lUVA0+r2gZ3+fNrMGav+8yNffctlxLHgHbP2/Cmjc+M/v/yjIvDjpkQPcFWSqCIIhc4daHKnDOHP1HYwSDBlZgQKAEz72+Cz+9frSj79PclsDUiWeistyLJ1/4CBPGnOx4Pp/sMf0yQ7LipAHl2HeoxSw8D8As3GYUPwTafTgj8KeqGjyi4CgdUuWXwVSe9diQmlSQWlTR+H2nbN5jjUtuPty/z7kgq7b1FnGVY1VaFuWqdTsw68rhFv/JKwuYfunZaDzUCgCQPAzTLz0bXllAvM06e2UMjoExJsAW/MwmW5lrgMY1M54QjSvQuAaebgaukhL2RYFsMrvdg7r263LLlk73S7ORNgEYHvnje5ZjH/nje44Z2JIoOAbnJJEhPaZciD50Iba5+3HWRG9wklkpQjTO4ZVFS3/glUVonDSgC55MV4kFgXU4QJdJIgYN9OuSHIxh38EWx068qtyLl9/ajTVvfIaHflRrdp5OOmFORQsbw21obotj9uQR+NZxZZAkAfOXbETA70NZiceU+AjtCeOlDbsx87JhiMYV+GQRgsgwdWIQUyYGoarcDE4bA/JLG3bjhsuHucpzqBoAxpL6Q8wMPht/T60anTq4uelvJVQNvuRKfzar9flET7VbdVnQUJ22GhFEBjjZrqENH/D7cNNVw8Bd9OgAIOD3QtW8WPHKNovu7oBACSSPgEScws1E7khdQLl92ihHO60slx2dP5/swaZt+y0ZbsFBASycnZTfEhn+9729+PDTJtw9YwxWrQuh9jsnOI7pXzc147Lxp+Picaea84KOCtv0hGQTQRCdJ55QUV3tz+o70ZiCyNG2Yx9IdBmnPtQjCq61J84+NWBmle5rasHLb31m830mjDkZpxxfgd//9SNTDzk1wDd26EDceMUwcA5LYNeY7xu7Tw30ACPH7dNGmdIEoT1h0zcCY/AIAlQ4+xCzJ49AwO9DoNSe+emGU1KBcQ9Ce8Idyj11NC65+nCKltf+mYGhcZ2e0X7jFcMs/lNC1RBN08SIxhWU+py9rEwDY1llK6saHnr6Pduxi+prrZnVHI4+/Kwrh9t+X3T5fafMblkSbFKhd8+wZ2ADmful2QTAO9LrTg/WQ9UQqPBagnOBCq+ur5F+aAH60IXY5u6mowKeNnsoQjQN+Mv6nZgw5mT4ZCChcPxl/U7HnRZdoa/YU16RTVGgDgfolL+pmoZV63baVifnzajBynU7UPudE3DxuNMAAI/+SS9+ccoJFbZihE5FCwcEStDcGkdluQ9HW2Oo8vsQ8Ptw/SX6NpiA34fZk0fghP7lOHgkiude34nvjz7Jsp3n9mmj4JVFXHHB6fZCA7IAj+j8u18diOD+ZZswIFCCX9w8zvG++ZOTpdSJjpv+1tdNzRg00K9nAhRocaaearfHRffOqYgFQWSCk+2Wl+qLXnOmjgRjegEEJ7vbf6gFCYWbeobGjgxDA1rkFFgjckvqAopboaeA34ujLfZaCNG4YvssHImCMeBAuBWCwDBicDW+P/okrH41hPVb9mLv/mZbto3h3M+9bhQ8nFNAmSCKAFkScfncF7P6zkuPTkIkR+0hjo2RxbuovhYHkmOCEXgN7QnjoppBWFB3HnxeAdMmnmUL0r60YTduunK4GaQM7QnjmVe2o27ScJx2YgWONMdx7xNWKY2/vf051m/ZC86ds6+bDkcx//GNGDt0IH4ybRSa2+Jobo0jHGnDXb/d2KEv5ZM9ZnA7U5ySClKLKs53kJ3IBDcfzklDOB/JpvZCNKZapLzmTB0Jp/3+WQXGsshW7ihbOjXg5AFsduwm5yWLcJEggc0WVIVj9avWbPHVr+7ALVeNsLU1U780mwC4wFwysB00b0VJAE+7AM71z23FDQvQh3bTuxfzuM3dTTb2UIx4Zft4NX9mDbyygERb9/W9OY2zBYPBBQCuSf5zbSgUujMYDF4E4DEAJQBWh0Khe5PHfhfAMgAVAN4EcEsoFFKCweAgAH8EMABACMB1oVCoOZftzjW5KAokJKU1jMmLv1RCNK6gNZbAh5824bLxp8ErACrTj1u0fDOCgwK46aph5koe5wAYbA7vvBk1aI0l8LvnPwQA3D5tFKZOPNMc2BrDbWaQ+P5Z41D7nRNsq7S/XLkVDbfWwpeW1u/zili8/D1U+WX8vG4sDiQnQXp13xL8ZvX/M8/xdVOz430zAgGpA6GTvpjFcc/Rc+gJeqrdXtme3WF0QrECmAAS+Ue67QYHBXCc34cBgRIMPK4UR1piiLTGMG9GDRpSsiH0hSoRf1m/E1MnnoUqvxcP3jIOispxOBJDVbkMlbKfiRyTuoDy3Ou77Au+M2tQ6hUhSyWWcXTs0IH49gA/Hph1Hr452IJV63RZqjuuHY2DR9rws9+9Y/av//7H983s/tCeMFas3Y5f3DwOh45GzQBHOBKFKAiw7fkkCIIgegxV1cBEAb9cudUhgaYVi5ZvxsLZ4zCwXykevGUcwhG9MOBLG3Zj6sSz0BqzLkyG9oSx7MWPsKDuPPx5fcgSlPvz+hBmXjYMkdY4qo8rcZQC4OAIDgrg8vNPx4IUHejbp41CcFAAoT1hHDwSdfQhDBlHVcvcj3BLiBn0LT/qJg3HynU79AKIWUoEumlEu0lN5RuZ1l5QVThKSiyqH2/LNhRdAmOOxcGyyFb2uOhQe9KCtamyKWAM4NxVzktTOcpKrIX9PB4GzcFHTahu2eJOch3AHdeOxmPPvm/e1zuuHQ1BAFJb4ZWcNbi9EoPNVWBwDtY7SIDEEhz3P/mu7Rk4SXt4JRcfWspfH1ryCLbERLfikcUKE5ztwUnqphhRVQ7Rw2xFOdVujovlLACdDDRPBDASAAfw38FgcBqAhwB8D8BeAGuDweDFoVDob9CDzHWhUOjdYDD4ewA3AXgcwBIAS0Kh0KpgMPgzAD8DcFeu2t0T5KIoUOo5Fy3fbJ7TX6YPFqnVnO+5oQYLn9IlM/6yfiduuGwYIm1xlPkk3Pv422aF2UCFFwG/F5GWOFraFJw0sByTLxyMlmgCp5xQgR9f8114RMGy1QvgOKF/ufNqqgr4fR54B1ZA1Ti+OhDB0uc/RmhPGMFBAbRGFcsq8PyZNZZzrFq3E/NvqMGip6z3LfUaAYCLAmIaR1WKRrbVcdcDtYVanKmn2h1XNHg8zLJg4PEwxJX8Dc4T+U2q7Qb8Psy6ehgEEVhQdy5EkYFrwKN/2mqTfp01AAAgAElEQVTZVeERBTBBz9KYdeUIQOC48zcbLLbP8njBiCgeUhdQjGy12ZNH4MTqciQUjvWbv8CV3zsD8YSKFWu344Gbx6EtmgATmKWoy/wbaqBpHIEKLxIJDU/cPQFHmqPwl0mmJrRBOBIF5zADHIbNO2USEQRBED2L05x83swarFq3Q/+7KOCRFe/jR1O+g/6VJcn6Pd+BxjlUlWNRfS2a2xJoaUuAgeG4Sh+8soArv3eGpd+/bcpIxBIKpl8yFIcjMaxYu91SUwAA+lX4cNuUkVjxyjZbEtCPr/ku2mIq+gfaC+ZOGHOyWa8nrqrtySwZIIoCOJyDonv2RbBo+WYAwOdfH81aItBNI7ojqal8ItNgbUdZzbYANDjumVlj1p4YECjBPTNrIMK+CyqrbGWJYerEsxyKENqDtcbO6+pqPw4ciLj6nAqA37/4sS2ona6BDehBZacAuJMGdDyhYf3mL7Gg7jyIgq58sebvu3DNRWdaAlqxhJaRBjeg55o7BetvdsjAdpf20GwBtYTq7EMn8rj+RjSu4pWNn9vu75QfBvuE/AQAgAPvb99nuQfrN3+BE/qf3tst6xE4OGIxFQ8/015v7c7rz0F5Sff+Ti4zoL8BMDcUCsUBIBgMbgdwJoBdoVDo8+RnfwTwz8Fg8BMAJaFQ6N3kd5cDuD8YDC4DcAGAK1M+fwMFHoDORVEg13MqGmSfB7EEh8oYOBj6V/rM4zyCAEEEBEEGAAT8Pky+cDCqq3zQOMxiFkbhwQVL30HA78P0S4eYRZZSt5O1xRSUl0jOW6ckhoORuKmfmapf9i8Xn4V//+P7lgnTouWbMfe60bjrt28B0B3x1LanXiMAIE2LzGhzg8sAXKjFmXqq3ZwDDyzbZHuOi3/UN4T4ie4n1XZFUcBXB5qhKBxxRUOJBjz8zHu2XRWLbx2Pu3/7lmXC/fCPxyOe0ArmnSUKH1EUwNKqxYcjUciSiEf/tBXhSBR3Tj8H4eYYwkejCEei2LMvguMqvHg4rajLoqc2Y+HsWhxpjuMnj71hjk0JRbMFMu6ZWYOyEg8W1Y+HltSKlgSQ3jlBEEQe4DQnl0WGWVcOx41XDIMoCKjyy/jmQCuqq3zweAQcOhqzZHHeOf0ciIJgfragbqyZkAO0S1ssqDsP0biC8NGYZTerIYmY6pMZBdqN7/evKsGCpe+gMdyGq753mmOA8sHZ4yyBgdQCg8Z1xVUOMCDcksDKdTscswWfeWW7eY7OSgQWeu2CTIK1HoFllH1snK8qQ99PVTX088toqB8PVdMgCkKySLe9FbEEN+c0gP68GpZvdinYlyEuBROdTihLIqb8MGjLupUl0aYB7ZVE/OCck3D/snesOyQlEWqKlrbmosFdd8Vwh0Aqx9U/GIxH/9T+Ps69brTjZbnLaghAWoawprn40HlczE4UGD78tAnrt+w1PxsQKMG0fzrLmmJexHhlAReMstrYvD60+5vzdl8c0PuDh595r9tjPzkLQIdCoW3GfweDwcHQpTh+Az0wbfANgG8DOMHl8/4AjoZCISXt84zp168867bnE9kWJUknkVDx5f6IbWXz5IF+SJK1Gz7SHMUNlw/FY8++b+p3GQY4YczJ5jnqJg23bRn69eoPcP+scWAAfv/Xj20Tkvk31IAxwVU/s1+ltUM3zlvl9wLQO8B7/3Us/KVeVJY7D4n6ZKy9+rMx8Cy+dTw49K0llWXegllB723b/aapxfGZKCrH8V20y96mq+9VvpFv15OJ7TYeasWvVn2A+2edh4blm/HgLbWu9pba1yxMVkI/vn9pTtreEfl2n3uLYr4PTraraRxf7juKg/vb8N/vfIG6ScPx7QHl2H+oBSvWbjed/EhL3CyoaSzK3njFMOcMJ84hiaL570XLN2NB3XlQNY6G+vEQRQZBYFmNWcX8XLpCX7gvvT1fIHqHYrDtnrDdnr5PiqLhcHMUqgaIIkPdpOE4dDQKWfLgSHMM/7Fqq2VeY4wdxmc+2eM4bkTjCirLZfx5fcj0syZfONixOJ1RoB3Qfah9B9vn9ENO7e9YiHDxreNxXECfW/XrV44v9x01i7+nJvWk+oiHI3HUTRqOynIZ/atKsHTNh7Yi0T6vBwG/r1vubT7ZfKa269ZmRdEcs48DFfpCRWcx5izGszN86JO/VWGbT7j5eqpm9/U0jeNISwyN4VZIPsl1fnIg3Opok4tvHW+7F4eOtJnBZ+PYxU9vwSM/Pt927IFwq6NkSfp5D4RbXbWMqwP2cz7/v7ssGdDP/+8u3HzVCHtbj7Y5St+IIsNxaed19aEd7mtP42a3R1qirtKblYGe97l6g8Zwq/OCzK3jMSCP+p5c0VOxn5zXWgsGg0MBrAXwb9B3ZZyZ8mcGfU1FgC7TcazPgSzXYA4ebIaWhZ5VPmGsmnYFLoruK5tp+pHMI5qr7/5SyWKAqf9O/5tx3mhcQYksmtWcUztzf5mMaEwxv5eun+muayxg6bwJlu1XbvdEYczWrk3b9uPGKzR4OIeiqF3WDuvJiU9v267gstIrMPdnUAh0x3uVT2R6PflmuyoT0BhuQ3NbAo3hNgiCc1EdIW0O3hhuQzSm4EC0Z59hsdlNZ+np+9DTzqaT7XJRwIN/2ITbp40ys2oa6mstu3iA9qBBY7gNz7yyHXdcNwpCB5r9R2Nx8zP9XYjjrt++ZU76A6USDh7MrOQF2aczvXlf8q3PzafADdE95Mq28812u0JP9wGSLJq7PY0do6kBq/tnnWfzVdIDzm5Fbkt9ElpjCiaMORnlpRIWzq511WGuLJfN782fWYPHn/vQ/LubH6eoGg4ciKC62o+mw61mABOwJiKlfj+0J2wGup+cd5G9SPTMGvCE2i3PIJNnmW+221GbuSi4+Oi1YF3Y2WfMWVLP++AfNjmeVxRF1zlKarvFtF3GqfOU9CxsVRBcg9rp90JhzsfGEprDsXY/P9VuzbbKoqOWcfo1AQD3CI7Z2hxObWUW6ZtIawIr1uq1pdKPzdaHzge79fg8kCTBIhsiSQI0h+dWrKguNuZku8VIZ2I/nbHdzi+vZUAwGKwF8BqAu0Oh0NMA/gHg+JRDvgXg6w4+bwRQGQwGjVTd45OfExnSkV5ROomUTENj8mOQ+u/0vwG6ccoeEaqm/7cxIZm3ZCOWvfgRoMEsIgjAop+55M4LwRjDnKkjzb+3ryoi4232RhA7vV2Z6pkRVjyi4PhMPCLdT6LziKIALgoQRf19DR+NmVvYnOwtPbuC3mmipzEc/GONg9G4YhnjGg+1QuNwtGuNA+GjMfO7AwIlONKsB6SNbLQ+UnScIAiiaIirMIN0ky8cbMvY3HewpcOxA9CTdNLHjfkza7B1xz7sa9KLG/7ksTfQdLgN36T4VgYDAiXoX1mCpfMmoKG+FlV+2VJfwM2PE1NW/NMD26lBZ7fvCwJMORLjt7MtQNiXcFs8yKYQpNt5A34f5s+sQUN9rR4o9vsczyuKwN0zxlhs7e4ZYyCmaVUogGPWvNM8RWDO/rjgUDDRSD6xHesQoRIF4Zh2C+hqGKtf1Yt4NtTXom7ScKx+NQTFqV5MSsFG49iXNuy2pz8mf9+Qvpm3ZCMWLd/cXhQ6jUL0oeMJDSvWfoJEstZTQuFYsfYTxBN95/11tTGnYp9FSE/ZbS6LEJ4E4AUAU0Kh0OvJjzfpfwqeAeBzANcC+EMoFPoyGAxGg8FgbSgU2gjgegB/C4VCiWAwuAHAFADPApgO4G+5anMxYrxITpnFimbVUE3NQHzu9V24c/o5iLTE4ZM9EIT2IhvGxChdbyyeUPDmB/9wrDzLGMeqdTstWc+GfuavVn2AKr+M6ZeebVl183lFLF7+HsKRqLnK2hFuxflkkSEOoaB0nvMBn4dZCjlG4wqq/F74PAyt8WN/nyDSMTIoVq7bgZuvHoE7rh2NNX/fhbnXjca23U045cRKm70pmmb2S4VSKJQoLozFzc3bvjGzapzGwYHHlZpFfkec0R+BCi9iCQVV5dZ+VM9M43hty5cA0K26mQRBEETvIIoCEuC4fdooRFoTqK7y2QKMq9bttPlJ/jIZd1w72tyFGo5E4ZVF/GTqKAQqvGg63IaV63Zgyg+DeGXj5wD0cSMaV/RxJ604nT5P4mCcAyoHg2Dxj17b8qWtDXfPGAOVa/CKejAtfWdqpDVh6hVXV/lw1/QxeGjFFvvcrMC1m7sLQz+7MdwKiIKj7+m++5cBXSiuLUuCLfN+ztSRkCUBWpoOtKoCGz74h63o2mXjT7dINncULLfNUxh31AYHc8i6TQa80tvqEQVoNg1o5hhj8KYVTFQ1zVED+sZJ9rZ6ANw4aRgUhUNgQGW5FzdOGuboZ2T6+wAgic4+tCQyxJCfMBftbtZHgq8AwATg9mmjLAVgb582CsyhKGYx0lOxn1z6Nj8F4APwWDAYND57AsBMAM8l//YKgP9K/u06AE8Gg8EKAFsB/Dr5eT2Ap4PB4L0A9gCYlsM2Fx0+WXDsLN/9+Gs8+eI2yxYaIWUQAABF4aYm2YBACe676Vzcef05qCjTV9J/fM134REFs2LsTVcOx4jB1Xjvk32476bz4BGZWfggrnKEI1E880r71hXOgROqyzD3ulEQBQavJMAnVUDVOL46EMHS5z82dcQWJbckdYRbIRBjK9yxtgwRVlrjGlpa4xg0sMIsYnHwSCtKZAqJEJ1DAbBy3Q5cfv7p+N3zH+K6i4fgXy8fBlkW0BpNwCeLFnvzygL+9vYX7fqClSXwgNO7S/Qosqg7HYcjUTOrxl8qQeMcP5k6Cv2rSiAKgFcSEFM0cwy694m3EfD7MOvqYUm7Ti6CSgzvbduHCWNOxtU/GKxnqr1g183sqhNKEARBdJ70InwdJbA4SRTcNX0Mxg4daAmEhSNRVPllLL51PBKqBlXliMUVqEzDg7eM088lCAAD9uw7iv9Y+Yk5Nnz+9VHUTRqODz9twvyZNajyy2AqByTRtTidcQ0V5TIW1evFbw8eieKTz5qwqL4Wisqhqhxr/r4LH37aZPpaxrhn+I/bP2+y6BWPHToQD94yDowxCAyU3JNCpnIV6ffY8NFlkSHRhWCXpsJZK7neXkjMKzHnomtpQdWsguUceH/7Poeg9mn2ewXuGPASwW2aq/GogupKr6W4oldiiEetediuyXdOGdgiQ0skYXsGJX4ZaUqliCU0vLl1r+N1pZ851pZAVZkMn+Qx2+qTBcTauiYFmlM4c9TubnCwm2JFZAyybJUhkWUBImN9If4MlQP+MikZj9PtVpJZt7siuSxCOAfAHJc/f8fh+P8HoMbh8y8BfL9bG9eHiMZVfPJZExbOrgXnHKIgmMFnoH0LTUN9LURw9K/y4YFZ48AY8LPfvW3phO578l001I+HIAKKyvGbP/+fdeVbZBg0sALfHuBPmYyoSKj6CqexQr9o+WbzOyrX4BUYVFVDPLlqDsZs2pqZbklKX3mPQ3DcMtRQX9v56r59BEXluPO3G22fL51/EWXlEZ1C1TgmjDkZv179AQJ+H1paE3hw5Sb82/Xn2N55AHjoR+MxbsSJuH/ZO7jnhhoKPhO9QlzlWLVuB2ZcOtSSVXPRmJNw1fcHA+AQRREHj8bw7P/swPRLhqK5LW7qQc/9jw3muRrqa/HLlVvNIk7zZtbAJwuOupmU6U8QBNE7ZKN5CzhLFDy0YgsemDXO0rfrcxkgzjl+9sTbtiDZ4vrxgKpCcfGFTjnej4b62pQMYwZF0yAIDLIkgKncEnx2CopXV/kw6Ft+PPLM+5aFTwCmr2WMe8aCa0WZ1wxSAnqNnc+/PmrqCtNY1Y6bXEW675l+jyOtCaxatwOzrhzeJR9VcZHfVDTN5r/FElpGbfAAuO+mc7H/UKsZmBt4XKnjPEUWGb6XFtSe7xJY11QOySNg4HFlEBigcS88HgbNJeLF02t0OMQGZBEugX3Yfj+uwr1WVtp5VY1jzRufYc0bn1k+v3jcqY5+cawtkRKTUBFry++3RHOxG03jEF2+U2xonOOhp9+z9csP/ahvBOGjcQ3zlrxlu36n96ErUBypyFE1jidf3GYGnBvqa83/NkgN7kZaEnj4mfdw+7RRrlttVI2bIvzVgRKU+Tz4/V8/xqZt+81Ovr9fRiK5dGqsvpeWSFhUX6sXHTvUvp2sf5VPX0ZM0uEqayeuP+MtQ4QFQYC53c6YlLy25UtHDS+CyARRYKgsl9EYbkPdpOF44Y1PMWfqSAT8Xjz0o/E40hzHc6/vQmhP2NTEDSQzI6QuVAQniK6gahybtu3HhDEnm2PTRWNOwiW1p5oO1oK6sfhw1wHUJZ02j8eLBXVjsWrdTtPBHxAoQaRVL7o5aGA56iYNNx299N07lE1GEATRe2QaRDRw8zcEkeEnU0eBMV3rWfII0FTueryi6TIYHMxxXiQKAlRNAxMERBMK7nvyXYt0QcDvRZkkQlU116B43aThAKIWXWjA6msZ456x4NpQX0v+VIZk6num32ODG68Y1qV7mk22cjZtSCQ0y87oe2bWAJI9NJlQnd8dpwxsBcC9j9sXYpzeM0kW0RSJ2wLLqTEHAEjEVfT3y5ZMaVmE5Zj263evlZV+/aLAHP3iYtmt5i7byvpMNkRCcX53E0rf6Oc0zjHijP646vuDzSz/NX/fBY137yIEefRFTnphPvfCE7rD6y+VbcWWUo/76kAEX3wTMUX4ZY+Ie5942xy4jJVDo483Vt/nLdmImxatx/wlG9HcmsBzr+/Cpm37sfjpLUgkrJ22sSUpVQDd2JLU1etPvV6iY3yyiKkTz9Kz9JLFJKdOPAs+mboNonN4AAT8XgwIlKC6yod/vuhMAMD8JRtx12/fwrIXP8L1lwzB2KEDcduUkXhty5dgySyg+558l4qyEb2CMY489/ou3DZFL85x1fcHmxXWAaCq3IsLxwxC46FWzF+yETc3vIbHn/sQ0y8dguCggJl59tzruzAgUII9+5uxaPlmbNq2X1/YVTUwVYOHcz2bjILPBEEQvUa2BeLc/I2vGiOY//hGzFuy0TKXcTtelgy/6S3bvOjuGWOw9IUPMavhNdy95C2EIzEE/D6zbb9a9QH2H2o150pu1+AvlVwLHRpBlmz8R8JKpr6nx6XgmcepAl8WGDWR3J5tZ9qqAKbOOKDb0UKXIoTuGdj2dyeb98wtW9khroxEXAVT1eScSnUMPgOZFzYEdJk1J7/YKxWHXyyIzkWzhb6S/oyOimL2jX7OK4tmcs0tD72O+5e9g0tqT4VX7l4jKI43hnAlfRAyCk84DUqqqkESBZujbRw3Z+pIrFq30/I3UYDryiHgnEHw69UfYPKFg1OOtQ4yqVuSjIq0q9btQLwTq4vZDMKElVhCcxzoY32oGi7RvaiqBp9XxN0zxsArexBpidt06n69+gNMv2QoXtqwG1N+GEQ8oZh/62plcILoDMY4YtQx0DPymWXskyUPmg63Oeou3nHtKDwwaxz+9vbnCEeiuHtGDZ57fRcAcuAJgiDykWwTWNz8jVXrdlqOS82Evfdfx9qO1xyyR3+9+gPUTRqO1a+GLAk/v1rV7k8Zn/lkjzlXcruGSGsC4UhUL/Z+63gsnTcBDfW1FnmRbPxHwkqmvmeuAn6pNZGcnm1n2ppNoFhwsTunuHo271lH2cqdxZDrsCe92Y+NJbiLX1wcvkk8oZk73I34y4q12xHvQ36/URQz/Z30dCIJshBJKJoluaYx3KYniyrdawM0bhQ5boX5XItVgJuVkg1H+4T+5fB4BDy84j1zK7FRTFDyiC7bNQRAVTtcfW8/1rq1o6PtQN1x/bS1OTNU1WWgV/vGNhQiN0RjKn73/Ef4ybSR8MkeRxuTPAzTLxmK9Zu/wJBT+wOgomxE7+E0jgDWLa6tsYSrPTMGNB1pw4Qxg1D7nROQUBRzOzVpPRMEQeQfRmAuXQParb92GidEkbnKXKiqhpO/VWHzT2JuQTZVs/lFqf6Uce5oXDHnSk7XcNf0MRAYUDdpOJY+/zHmXjcKHs6TetIdX09H/iPRTuq9A2MA5473KjXgZ0g6rFi7XX8m3dCG1JpIbnOMTP3kbGQ9PCLDnKkjzQX51CCeltaQbN4zd4kIPebQGSxyHZzrRei6Qa6jEBEFZu5wN+hrvlc2RTGLkZ6K/RTD+0Icg/RBKKHCdVByG4gUzlHllzF/Zo1F9+isU77TodC/24BlbOVyKgrQnRrQTtdPjn5miKLLQC8ykBYC0VmMCU7T4TYoquZoY198E8GyFz/CnKkjsWLtdgrUEb1O+jgiyaJl7GtujQOAoz03HY7insffxoBACe64djQGHleKpfMmkANPEASRp3QmgSV9nGAQOgyuCQLTpZeSx6twn3tr3Hl8icYV878NDWjj/NZrAL46EMHSNR9Z6hJ0FFzKxn8krBj3rrrajwMHIo73qpACftkEipnKzfotRhAv4PeCOVxTNu9ZNsUFsyERV8EAHJ98Vm7nykUAPJ/IdtGtGEnEVVT4PPANrDjmgkQx0lOxHwpAEzacArZyUg84vdNnGu9Q6N+tM/OXSWioH+/4UnfUARI9h08SHAd6nyQgpvSNjpjofoxK2m1Jpyk9S+KeG2pQXiph8a3jIQoMc68bRYE6Iu/QVI6yEg/uu+k8CAwQRAGKotrsef4NNYjFFTTU1yIaV1BRJplaz+TAEwRB5C9dTWDpTBDbyQeaN6MGbfEE7rh2NB579n1LEbgqvxdL510EQdAzT5nKLec3rsErCgj4fWZGdl8MLuUb+RDwM2o1pbchXbIj06xu49gyScSggf6M7D7T9yyb4oK5IFcB8Hwhm2dczGS6IFGM9FTsh2J6REbEVWfdo4b6WjDLwKHaXtQqv4xF9bXQNMAjMIjgUJNaMk4vNclm5AfRuIo3t+7FgrrzzEqo6zd/gcvGn+ZYAZwgMkFVNfhkD+578l0E/D5Mv3QIHrh5HLjG4ZVFiLy9f1BBmTZEfuJUuX3s0IGYPfk7lvFOEoG4KulpY5xB0zQo0HXmaEwjCIIobrINYjtlLT/x/IcI7QkjOCiA2ZNH4MRqP0RBP6caV5LnBjRVDyhyUbD5T+Rb5R+qqqFfLwZUAedaTYsM/96hvcfK6k4/trvn8EZw0C3mkEt6OwDeE2TzjInio6diPxSAJjKio+IDbkbU0arqMX+PZDN6HU3jWPPGZ1jzxmeWzy8Zdxr6UEFcopsRRQFRRdeYagy34Z7H3zb/tnTeBIDn17ZDgnDCaUzctG0/brxCr7ouAICqL7J6RAHhlmNnGBEEQRCE4QOBMdy/bJP5eWhPGPcv24Sl8ybomc5p3ztWNiv5VvmFKAo4GIn36tygM/59X6Y3A+AEkWt6KvbjUI+UIOxkWw0acF9VJfngwkBIal2lolcxpvxnovMoAL5uas66PyGIfCKbMZHGQoIgCCJbsvW9aKwpLPLheXXGvycIojjpqdgPBaCJjDB0qgyjzESXuaNVVaIAYBy3TRlpeea3TRkJMHp+ROdRNY5V63babIt03olCIpsxkcZCgiC6m3hCRXW1P6v/+StKjn1iIm/I1veisaawyIfn1Rn/niCIIqWHYj/UvxAZ0RntMGNV1V4tNv+q+xIOcOClDbtRN2k4/KUSIq0JvLRhN2ZdOby3W0YUMEbV72de2W7aVjSuoMovQy0iHTWiuMlmTKSxkCCI7kaWRFw+98WsvvPSo5MQyVF7iO4nW9+LxprCIh+eF2mDEwRh0kOxH8qAJjJGVTUwVYOHc7CkllhH0KpqYeMBMG3iWVj24keYt2Qjlr34EaZNPIueH9EljH4hHIli0fLN+OXKrQj4fWDkHBEFRqZjIo2FBEEQRGfIxveisaawyJfnla1/TxBEcdJTsR8ak4icQauqhU3q8wNjAOf0/IguQ/0C0dcgmycIgiByDY01hQU9L4Ig8omeiv1QBjSRU2hVtbAxnt+AQCk9P6LboH6B6GuQzRMEQRC5hsaawoKeF0EQ+URPxH4oAE0QBEEQBEEQBEEQBEEQBEHkBApAEwRBEARBEARBEARBEARBEDmBAtAEQRAEQRAEQRAEQRAEQRBETqAANEEQBEEQBEEQBEEQBEEQBJETPL3dgBwiAoAgsN5uR5co9Pbngl6+J6cA+AcAJYe/kZe2m2/t6QrFdC1AxtdzCvqo7XYXxXpd2dLD9+EU5N5ugSKw3UJuey7pxftyCqjPJfKMvjhfyOf3I1/bVsDtOgV5Zrv5ei+7QjFeE0DzhVSK9RlnQ1+/B7maLzDOeSeblPeMB7ChtxtBFCWnAvgih+cn2yVyBdkuUYjk2m4Bsl0iN1CfSxQqZLtEoUK2SxQiZLdEoZKV7RZzANoLYAyAbwCovdwWorjI9Qol2S6RK8h2iUKkJzKgyXaJXEB9LlGokO0ShQrZLlGIkN0ShQplQBMEQRAEQRAEQRAEQRAEQRC9DxUhJAiCIAiCIAiCIAiCIAiCIHICBaAJgiAIgiAIgiAIgiAIgiCInEABaIIgCIIgCIIgCIIgCIIgCCInUACaIAiCIAiCIAiCIAiCIAiCyAkUgCYIgiAIgiAIgiAIgiAIgiByAgWgCYIgCIIgCIIgCIIgCIIgiJxAAWiCIAiCIAiCIAiCIAiCIAgiJ1AAmiAIgiAIgiAIgiAIgiAIgsgJFIAmCIIgCIIgCIIgCIIgCIIgcgIFoAmCIAiCIAiCIAiCIAiCIIicQAFogiAIgiAIgiAIgiAIgiAIIidQAJogCIIgCIIgCIIgCIIgCILICRSAJgiCIAiCIAiCIAiCIAiCIHICBaAJgiAIgiAIgiAIgiAIgiCInEABaIIgCIIgCIIgCIIgCIIgCCInUACaIAiCIAiCIAiCIAiCIAiCyAkUgCYIgiAIgiAIgiAIgiAIgiByAgWgCYIgCIIgCIIgCIIgCIIgiJxQzAFoD4BTkv9PEIUE2S5RqJDtEoUK2S5RiJDdEoUK2S5RqHJfFdUAACAASURBVJDtEoUI2S2RFxSzAX4bwOcHDzZD03hvt6VTBAKlCIdbe7sZeUVv35Pqaj/rgZ/JO9vt7fvenRTTtQCZX09ftd3uotjsprP09H3oIbsFCtx2yT6d6c37Uix9LtnWsSm2e1Qstgvk97PJ17YVcrvyzXbz9V52hWK8JoDmC6kU6zPOhr5+D3IZXyjmDOiCx+MRe7sJeQfdk96hmO57MV0LUHzXk6/Qfdah+5Cf0HNxhu5L16F7eGzoHuUv+fxs8rVt1K7uoxDbfCyK8ZqA4r2uzkD3gu5BLq+fAtAEQRAEQRAEQRAEQRAEQRBETqAANEEQBEEQBEEQBEEQBEEQBJETKABNEARBEARBEARBEARBEARB5AQKQBMEQRAEQRAEQRAEQRAEQRA5wdPbDUglGAwuAHBN8p9rQ6HQncFg8CkA4wG0JD+/PxQKremVBhIEQRAEQRAEQRAEQRAEQRAZkzcB6GAweBGAiQBGAuAA/jsYDF4F4BwAF4RCoW96s33FhigKUACoGocoMHgAqKrW280i8gzDThrDrYAokJ0QeQ/1bUQ+Q/ZJELmB3i2CIAgCoPGA6DwU+8g9eROABvANgLmhUCgOAMFgcDuAQcn//SEYDJ4IYA30DGiygi4gigLCrQksWr4ZjeE2DAiUYP7MGgRKJXrBCBOyE6LQIJsl8hmyT4LIDfRuEYWCv6IEPm+7+11d7T/md6IxBZGjbblsFkEUDTQeEJ2FbKdnYJzz3m6DjWAwOBjARgDnA1gMoB7AEQAvA1gZCoWezOA0pwD4PFdtLGTCkSh++qs30Rhun8wMCJTg3+dcgIDf14st6140jeNISwwJRYPkEVBZ5oUgsK6etssnyIBTkAe2G45E8Z9/+T9MGHMy/KUSIq0JvLblS9z6z98tKjvpQxSM7Xb23e0rfVsfoyfsFuiE7WZrp2SffY6C6XMzJUfzqi5D71a3U3S2m09cPvfFrI5/6dFJOWpJUUK228fJl/Egy/GS7DYPoNhHp8jadvMpAxoAEAwGhwJYC+DfQqFQCMBVKX/7DYDpADIJQAMADh5shqblX5A9E6qr/ThwINLt51UYs3TKANAYbkM0puBAtPt/rzvJ9J7kagUrk0yF7qK3bZd7BFx+/un49eoPzHt425SRSChqTuyyp8jVe9VbZHo9hWK7XXl3c9m3FZvddJaevg89abdA5rbbGTsl++x5evO+FEqfeyyMe5jPmUG9Pa8ttvevWGwXyL9n09l729Pjbj7dM4NM2pVvtpuv97Ir5Ps1dXY86M7ryna8JLvND4o19tEZchlfEDrToFwRDAZrAbwG4O5QKPR0MBgcHgwGJ6ccwgAkeqd1xYMoMAwIlFg+GxAogZgHWSzdhQKYnT6gDzyLlm+G0rvNKiw4MztgQL+Hv179AcCLx06I/KMr725f6NuI/KAzdkr2SRQy+TyvoneLIAiCAPJjPMjn8ZLoAIp99Ah5E4AOBoMnAXgBwLWhUGhV8mMG4D+CwWAgGAxKAGZB14EmuoAHwPyZNWbnbKzK5V06fBdQNe64+qkWaDZ8b6BpmuM9LNQdBURh0JV3ty/0bUR+0Bk7JfskCpl8nlfRu0UQBEEA+TEe5PN4SbhDsY+eIZ/mZj8F4APwWDAYND57AkADdD1oCcBzoVBoZe80r3hQVQ2BUgkN9bVFWx3WWP1M138SBQao1IlkguByDwUBgNp77SKKm668u32hbyPyg87YKdknUcjk87yK3i2CIAgCyI/xIJ/HS8Idin30DHmTAR0KheaEQiF/KBT6bsr/ngiFQktCodDZoVBocCgUuru321ksqP+fvXePb+I68/8/M0cayRdhCzBumgaaG4IQszWJTYKdZBdSftvcIMtuMe0mON8YEpwGmmbbYOh3gXwDJr2kS5qFlrAbIN1g9vWlCU1h90sC2ySQBFNCF0JAZXMju1CuwsiWJc3l/P6QZzyjmZElW5Il67xfr74a7PFoNPPMc57znOd8HlkBJytwUApOVoZckJ4Lq5/5DruHjMFgoHY31H0bIzfor50y+2TkK7keE7B3i8FgMBjA4I8HuT5eMqxhzy07sPvJGJLkwupnvqO/h+A4gFJ2DxkZh727jHyA2Smj0GA2z2AwGAxG37DxMj9huY/swBLQjCEHITwkAJEeh+/iOciywnZOMBg5Dnt3GbmOaqOyQkERC6I4SgGZMjtlDHlkWQGHnslDnM3r3w022WYwGAxGPqCOXWcDIYDwaRu7Eo2XDEYhwxLQjCEFITwCIVHrPKtunfAWO9lEKEXYvWRkE2ZvjFyH2SiDYQ17NxiFiGdYEdwuNpVmMPIVNnYx9DB7yA5s1MxjWLWJGQnQnAYQ61y6amM7WpvrwA3upeUd7F4ysom9vdXDQfiC922MwSeRjcocx8ZhRsGhxqEiWLzAKDzcLgfufXJ7Sn/z+k9nZOhqGEMdNu9PP2yuy9DD7CE7sAR0nsJWaKyRFWroXArEnIesUGbsKaLY3EtFAcggXRNj6GL37p7v6IbLSQretzEGn0Q2+tQLe9k4zCgo9HHoE3MmsdiLwWAwMgSb92cGljdg6GG5j+zAD/YFMPqH3QqNNLiXNegQntM6l6qM8haB8GzdKlV4nre8lzy7l4wMYPfudnRGmW9j5ASJbBRg4zCjsNDHocGQyGIvBoPByBBs3p8ZWN6AoYflPrIDS0DnKYlW7AoZB4AljbWa81BXiNkqZj/gKBbOrjbcy4WzqwGusG2MkRms3t2Fs6uxbc8J5tsYOUEiG1VhtsooFPRx6LY9J0zxAou9GAwGIz2weX9mYHkDhgGW+8gK7P3KU9QVO/1gpK3YyYX7ksiyAm+xE63NdUwja6BQ4PV3PkbTjCp4ip0IhkS8/s7HmD+zarCvjDEE6X1363G+oxsdnVG8vPMY/CcDzLcxcgLz+MJj/WuH4T8Z0I5htsooFPRxqP9kAC/vPIYFsybiygoPCA8WezEYDEaaYPP+zKCP68BxAKVs7CpkWO4jK7AK6DyFrdjZI8sKOFmBg1JwssIGkX7iADBn+jhs2H4ELWv3YcP2I5gzfRyzMUbGkGUFDlC4nAQbth/Rks/MtzFyBf344gDFnOnj2DjMKEji49BAMAyvxw0XDxZ7MRgMRhph8/7MocZ1o7zFbOwqcFjuIzuw+5mnsEpfRqZhq8KMwYD5Nka+wGyVUcgw+2cwGIzswPwtg5F5WO4jO7AK6DyGVfoyMg1bFWYMBsy3MfIFZquMQobZP4PBYGQH5m8ZjMzDch+ZhyWgGQwGg8FgMBgMBoPBYDAYDAaDkRFYAprBYDAYDAaDwWAwGAwGg8FgMBgZgWlADxEI4SEBA9aFStd5GEMD1R7OBkIA4Zk95BBD6dkUkt8ppO861MnWs2Q2wxgsCOERCIYhcVze2x57jxj5SlSUUVHhSelvwhEJwcvdGboiBiMxg+1vB/vz8xWnQBCVgdPnu0AIgUAAMSoP9mUxhhgsAT0EIIRHICRi1cZ2nA10a51xyz0COJkm7XDtzuMtdjKnXYAQwqNLlHHmYghuwYFwVELl8GKUOAmzh0FmKL2rdt9lhEdAVKZDKngcSs+t0MmW3TKbYQwWvba3L+99M3uPGPmM4CS498ntKf3N6z+dgWCGrofBSMRgzx+Zv+8fToHgQjBqGdeyJDQjnTAJjhyAEB6U8JA4DpTwICS1xyIBmrMAgLOBbqza2I5PT11GICQmfT6780gpXQ1jqEAJh+6o8el3RyVQwg3SFTFUhtK7avVdtuw6jgvBKFrW7sP81t1oWbvP4MsG6jMHi6H03Aqd/thtf5DBMZthDArZsvFMED9GsPeIwWAwssNgzx9ZrN0/RMX6vokFlrNX44ezgVBezTHzCXZHBxl1lW4gwbysUM1ZqJwNdMMtOFJyuPHn8Y32omlGFWSFshewAKEp/pyRPezeeVnJv6dj9V2m1YyxDR6T9Zm5mKQeSs+t0EnFbinh+mWLhPAQZYXZDGNQSNU35wpOgSCiUJzvCOOz00Gsf+0Ie48YDAYjS1AA4YiMddsOo2XtPqzbdhjhiJy1+WOqsTZLOMaQbMZJSS6ccVKt3j95JoizF0M4eSaILlEuWJvIFOxuDjLpWKUjPIdR3iLDz0Z5ixAMiSkF2Prz+EZ78cBd47Fh+5GcrnLpi1xMQuUTokgNAYQoUgCsAnqwsXvnCZ9/zybe7yxprMXoylI0zaiCb7RXO071Zcn4zHQs7GWCofTcChV1TAGAZU2TDTZaViqYgnevx41L/awYlQCcOt/JbIYxKKTqm3MBQnhcCEbxw1+8i6de2IsN24/g3tuuxaXOMHuPGAwGIwvIMsWatkOGOH1N2yHIWVJxSCXWJoTHpZ75wsPPvIGWtftwKQfmC4MBb3Pf+AIaJynhcLlLNOQ+LneJbPd3mim8tyvHSEdFnAPAksZazWmM8hZh4exqbNtzIqUAW3+eWVOvx/NbD+V0lYsV+oQzLxBtUMmlJFS+oCjAc68cNNjAc68chJIjE81CxuqdX9JYm5ei/up3mTyhEo333IAN24/g0Wf3YMP2I2i85wYt0aH6smR8Zq5uvxtKz60Q0U9U5rfuxrpth/HQvRPgG+3FKG8RvB6XKXhvmD6237YoKxRtu/6IhbOrmc0wso5AODzz6BT8w/fuwPz7qwy++YG7xpt8cy5g5fuf33oIUVFhvpfBYDCygKLAMk5Px/wxmcKyVGJtGRxWxo0ZKze2Qy7AYisn4bGowRhvLmqohrOAkq+yTe6DSYenFxZ7DTLqKp3eUacazMuyAm+xE6ub6yHKCk6d78TLO48hEAxrDjeZRUf1PK3NdQmTPLlqNPFNB5Y1Tca6bYdNE//W5roCHFZSR5bzzwYKBf27Co4DKM2LRlBWqN/lkfsnYvE/7jW8rz/b8gEe/+bX8PN//UNv8JjIZ/ZsE8tV/xXvY/OlgRcjhtVE5blXDqL1sXqAUgiEw5LGWkMDly+PLO23LRKeQyAYxss7j6FpRhU8xU6EoxLKPQJk1hCGkUHUSuJVG9vRNCOWfI5P6qo/TyXOzDR2vl9wEpR7BOZ7GQwGI8M4bOJ0B88NaKBItrlgKrG2pNjIThTiXFdW4PW4sGDWRK15pNfjQiFlX1nuIzuwUtBBJl0VcbKsALIMN+EwutKDJ789Ca3NdSl3fJVlBZys5OVW8fjKF7fgYJp/A8B2Kw6XuzZQSKjv6ihvMThZyeuJtCwrttpjlcNLDL4sGZ+Zy/5LfW4OSvP+uRUadhMVWY49SzEqa5Oe9S3T0NpcByfh+22Lqq0HgmGs2tiOn235AF6PG1wB6fExBgd9POUpdlra/Vev8PQrzswkdr7f63GB63lPme9lMBiMzEFAsTQuTl/aWAsyQBXoVHY3Jhtr28tODOhS8xJZVlDiJBhd6cGo4cUYXelBiZMU1FjJch/ZgSXzB5l0V8TJsgIOPQ9Wpv1eaFQnvvGrjLlS5WJFfOVLMCT2WSnJsIcQYFFDtabjpW7FIQSguWoEjLyFt6mY4HnEgseenyXjM/PRfzFyn0Q2qhpW/BhMCN9vW2QV84zBQh9PJYql9L45F7Dz/UVOHiLbNcBgMBgZR5YVlGcgdsnE7kYH4Sznug7CQSnAIUONYSsqPDh3LphT43s2YLmP7MAS0GmGEB4SYOlw9b8TnDwUOVZRRRF7EBylA0oa9+ea7Ohr4tufc2aaeDmTbXtOmJwIS0IlDwfA7SKGrThuFwEHZK2TcToR3A5ERIrT5ztBCIHLySEaHmxV4MLEyn/wtkEgDw5G/9LXQttgJO7sfKJTIIjKgKwoIDwPgYAlQvIQQnjwPfp4qU5Uyj0CVjXXQVFiW1OdBIjKFDLHgfAcBMLF/m1hqwNZVM7FcZqRe8TbiUA4UBlaPLVtzwksnF2t9QXJ5VjKzverPtfqnQCQ0ntidw4Gg8HIFVIZ/5M9Vo1nT5/vAiEk6/FsQtnSfhaWcTK1lJ0o1J1mhT5XHmq5j/6g+oOzgRBA+IzMHVjclEYSaRMB0H7n9bjx4N3jTYnRTGxjTFYvyQq7ie9AzplJ4itfAsEwvB4XVjfXQ1IUNgFPEQfPwS04cBlR7WduwQEHz+l+kh8IbgfOdUTQqrPZlsZaVJS5CmpgzQXs/McIj2AKAl0Cwbpt/4k508f1T04IA98Nkgx232lkmcvS7kZ6BJaEziPU57tl13H8zZ1jk56oWNnF8nm3IBhSNC3pyRMq0TB9nMFG0jGe5uo4zcgtrOykpbEWb3/whZZ09p8M4PV3PsYzj04BBw48j5yOpVKJXZfPuwWi2Ps+9vWe2L1X5eWFMjVlMBi5Tirjf7LHOgWC88FoUvFspuKPTOxu1MtO5HtPnYHC5soAASxzHwRAIVhEtuYOBahwkzkSaRPpfzdr6vVa8jn+uGxeUy6dMx3oK19U/c0SJwFkmWn+9YOISLF5x0cQpdjESpRi/46I+TfRiohUG1CBmM22bmzPy++S79j5j6hMe4LAYeB5DqJE8eKrH2L/0TM54V8SYfed7OyO5Z7zC/X57j96Bi+++iFEiYLnOYyuHJZQH8/KLs5cDBkaGU6rGWOykXTYe66O04zcwspOWje2Y/zVI7Xml89+px7zZ06Ei+dAqJK3sVQy72Nf74nde9XRFcn8F2AwGIwkSGX8T/bYqIyk49lMxR9W8/x0JMaGUk+dgcDmyrHdiVa5j2iBVMRna+7AKqDTSCJtIvW/Adg2dMlEh82B6CXZbcnJhAZTushm1eNQR1YUXAoaa50vBaOQFWXQn3OqyHbNw/Lwu+Q7ifwHR2OyBE+9sBe+0V7Mmno9PMVOBENibF9UjmL/nZjdDQX0z9d/MoBVG9sBAOtbpsWksxL8ndfjRtOMKs2Oy0tdBpvIVDyQy+M0I3ewsxNPsTMlW9eTq9IvVt81UbNqq/fE7n6JkpLLQxSDwSggUhn/kz02lXg2k/HHYM/zc3V8Sweyophi1m17ThTUnEVWKPYfPYP9R88Yfv7wfTcWxD3I1tyhEO5l1kioTYRePb1sNsfrr15SohJ8qvsumb5+xuAhOIlJKmZRQzUEJ4ESza86OsLzNjbLAzJbpsgmiXwSAQcKDpMnVOLe2641aY7mqnyA/XdidjcU6O84Kjh5kw9dPLcGkydUasFtpuKBTGglMoYednYSDImGfzuS9Fm5LP1i9V3DUSml98TufjkdPCSJ+XQGgxEjWR3VTCQ0Uxn/kz02lXh2qMYfuTy+pYOhNO/vL0PVdpMlW9+fSXCkEVWbaJS3CAAM2kT636nN8ayOSzcuJ4+WuGtqaayFy9n76AnhQQkPieNACa8NhnYl+Im+J2PoQCnFb97+GE0zqtDaXIemGVX4zdsfgyZZBZVLuJyczXvAapayjZ3/EAiHQEjE+tcOo/GeG7XkM5DaFiArf5Zp7L6Tnd0JJOOXxEgjArF7jon9hyLDJLe1etMBPHTvjdq5dh/4HC2NtZg8oRJLGmvx7Hfq8cyjU/o8d1+wcZqRDHa2vfvA59q/FzVUIyxKSfnSwZZ+SeT/rd6JyuHFWJrCe2L3XpWVuDL4rRgMRj6hJipb1u7Dw8+8gZa1+xAIiSYfqj9ufutu2+NSJZXxP9ljBYKk49mhGn9IALbsOm6YF2/ZdXzISJtRSk0x65q2Q3k57+8vAuGwbN4tWNY0Ga3NdVjWNBnL5t0y4Jg8X8jWu5vvviCnsOu+ra6K6X8nOPmsNMeLiBRtPc5S3U7Rtus45s+cCA72q3meEvttwRylCb8nY2igUJiqUBfOroZCc1oNwZJoWEJFmQutzfWazRZaZ99cwc5PRmWq+aG/nja2X1uABqs6wfY7hSWM9Ag9dqeA8HzWu4YzBk5UthtHqxL6Qslmy2owFEXTjCqUlQoYWVYEl5PDnOnj0mq3fcUjDAZgbdscR/GXt34VM++4DsGQiM07jiEQDKO1ua7PsX8wpV/68v927wScJOn3xO4cPJ9vURGDwcgUdgtx8T402eNSJZXxP9ljxajcG89SCsJxtvHskI0/OOt5cd5Nim0QJeuYVZQKR7pNBhDsErFu22HtGX/vWzehxF0Yd0D/7mayKSergE4zqpC9VdM7/e+UqJyR5nj66g8QAp4AM++4DgCw8bcfaY2UZCX2eXaDHyinrX6o6OVEEn1PxhCBwlSF+vzWQ0CaF0KzVbEaDUvgZBlXjCwBJ8ss+TyIWPkPfeIicDmS0P/YYefPRKBP2xqoHdr5RDEqg9N8vcySz3mIqgm3amM7Wtbu042jiZ2hupVNzyhvEQKXI1i1sR1PvbAX4CgiIk1YNdpf22TjNKMvrGy7OyxjxYb92r/9JwOGfiaJsLP5vnx3siR6F5Kpvta/E46ev4n0fC8XzyX1nrD3isFgJKKvnlCpHteva0jBTyV7rBrPXjGipM94drD9ZCpxk3rs2UAo8bGUs5kXD40MNG8zfvMFlC2UZIrnXjloeMbPvXIQUgHIb6hkoylnTqXzfT7fMgDf7PnnDr/f/wOfz3cngOcAFAHY6vf7fzhoF5jjWFV/LGqo1qpXFs6uxss7Y/+tajbZDX6KQrGksdZUSeIAWGO/AkGhNrZBKdKlIDDU9bQYyaPXndq25wQWzq42aUD35X/s/Nm5QDd+tuUDW9tidshIRH810dStbHq7UsdhAJg8oRLBLhERUbadhLqYbTIySDp0kfVY2Xy6Ysc+K5xTqL5mPp/BYGSK5HWVC1tvNlOk4t9TOVax2dWmKOmbFw8mgpPH4rk1WL3pgKFvieDkIRXIuEgpLJ9xAamQZIWcWdPoSTRPB1AN4GsAbvL5fHMA/DOAGQDGA6jx+XzfGLyrzG2sqj/WtB3CrKnXa6t0DdPHGjSb7KpVeL5XMmR9yzS0NtelLTAfDI1WRurYr4Smb6V3qOtpMZJHrzvlPxnA6+98jBXzp+Cni27HM49OwQiPoPkfOx9i58+CITGhFulg65Yycpv+aqKpW9lWN9djfcudWNVcB6ejtynxw/fdiFUb29HRGbWtGmW2ycgk6dBF1qPfvpnO2DHWm4RDRJTRNKMKvtFe806BFKqv2XvFYDAyRbIxw1DVSh5sUvHvqcxDh3qFsCRSbH3Db7gXW9/wQxILJ/vKczbPmBsaVe65Qi75uNMAnvT7/VEA8Pl8xwCMBXDC7/d/2vOzXwH4GwD/NmhXmcPYVX94ip3af19Z4UGRk9O2zSSsVpEVcD3HQKZpqXxmVSf5BDVVoS6cXY20anAMcT0tRvLoExeK0qOpyVEIDgEO9OrMJfIhQOKKU7tquMHULWXkPgPVM7wciprstdwjQBSVPiv+I8w2GRkkGV1kt8sBKspJ23u6Y0crn6/6df/JgPYupFJ9zXw+g8HIFMnqqA5ZreRBJiX/nsI8NJM7fHIBSVGw/+gZ7D96xvDzh+9TCmZcJASWVeCExBqLM9JDRuzJ5/PdCmAVgOHQvcJ+v3+i3d/4/f6jur+/HjEpjp8jlphWOQ3gK6lcy4gRpakcnnNUVHiSPjYQDFtu5QmGRO2/3S6Cco9b+72iUMARwcoFdeB5Di4nD0+xy1TlqigUHV0RiJICp4NHWYn5mGSvcdXGfaZVyZ8suh0Vw0uSOkcq9ySfGWzbDQTDOHjsT1jWdCsID8gK8Gb7Z7j/L66H15vcs+qLc4GQpZ7W6sfq0/6cVRs+GwjB6Xb224ZzkVx7JzJhu+rzi0RlBIJheD1unA10G3yI1+NGeTnFc9+9HZGogqgo43yH0R+6XQ54dT4QsPed8cfm2n0eLIbyfUi37SYa84rcPJY1TYZbcECSFSxqqIaD8KgoL8KIsthuk47OsHZMMCRi254TCATDlnY8lJ/LQCiE+zLY8UIqpBpPWr1Dz289hKYZVdiw/QjcLgfKSlzo6IrAU8xh9WP14HkOPM/ZnjtZn58KhWBnmSAbtluoz2Yg3ztX71kuXVc++d10k0vPIZ5U/Huq89BhwxSsfqwekkzhIBy8HjccDusS6HTlTtKNnd0GgmFMnlCJaTVjtKbEuw98PqBxMd84FwjhnUP/bcp93HvbtTlt85kiU985UwsavwSwEcAHSLFc0ufzTQCwA8D3EdsZMVb3aw5ASsuCFy50xpKsWSK2TRADWslUzwGOg9PBQRQpZEUB4XnbjrPq38WvzKka0OoqHRVlnDsX1I6PrypZ2lgLSVYQFRU4eB48iX2XYFd6qpYljrNclQxHJJwLB/v8+4oKj3b9g0E2nU+2bTcep0Bw+6SrsGLDe9pzb2msBacoaXsGso09yDJN63MmhMelkIiVcbZePgQq75N9J3LFdvU+UvUxUVFJ6C/7qoDT+xDB7cD5jojp2Nff+Rhzpo8z+ED9+a2qGvTH9tf3pGNMyCWy7YOzHfD15XdTfZ52Y54oKQiGooZO24saquEpdoJQigsXOuEUCC52mo/xelwmOx7ssTFXGcz7kis+NxkS2XVFhQcXL3alZPdOgSAqwxS79mcXnN07VFYqYElPTPLZqQ7L8f3ChU7b79uXz0+Fofb+5ZPt9kUmn02uJyT6+71z1Z6Tua5cs92+rjmVmMLOrw702GTR5yfsqrpzgVT8eyrz0ExpSwO5YbdOgaBh+ji06q453fP+nMfB46bxXzLkPmK7v/vvT/OVTOYXMpWAlvx+/3Op/pHP56sDsA3Ad/1+f5vP57sDwBW6Q74E4FSarjHtpENeQn+OideNxF11Vxu2AbQ01mKkR7AcROK38qjJnSe/PclyULPSSFq5sR0LZk3Eig37tcmuolD8/F//AK/HjaYZVfAUOxEIhlHuEQA5tcFT33DBN9qLWVOvR1mpAMLzIKA5OZAVKhIF2np0sdSV0LZdx/HoX9luZEgZnrduwMHzSOt+JhmcNjkFem19dXN9+j6E0SfxPnLyhEo8dM+NiMoyHDwPiXBwEmLyBfG+yutxQ5RkLGqoxn+f7cTuA5/DwfNwEiAUkU1+7fmt+xx1BgAAIABJREFUh9DaXA+HjY/J1DZIJjk0tLCy34fvuxEcF6u2tLIZwclbVjBHojJWvWTu2fDjx2+DJCtQOA6KTPHK/ztuOmblgrpsf3XGEMbKTy2fd4tWmX/2YgjggPWvHsH+o2f69GNOgeB8MGqaxI70CIjK1FKb80eP14OCt/S/do26RpS54SIcRBkpj+9s6zuDwRhsUokRE/nV+JxAKsdm4lozSTI5h1T8eyrzUDtt6dbmOpNih15bWp1Db9l1HPNnVuWsyqSk2Mz770/fvD/XoeAsK+JbH6vP2eeWbtTFq9Pnu0AIScviVTyZSkB/6PP5qvx+/5Fk/8Dn810F4DUAs/1+/56eH++P/cp3HYBPAXwLsaaEOUkqjimZc9z/59drKzDq+Vo3tqO12f4lMGrvyVBkex0+O40kt+DQ/ntN2yEsn3crvB43HrhrvEmjcoRHwIWgWd/SbkBS9ZO27Dpu0lxiiZncgsJaFyuddSrZ0tOSbDoXS0zvMavo/ZtvtBf33nYtXvrth336Ar2v8o32mnzR4rk1cDg5REWKQDBio/2mgEvQxjgTmvfpGBMYuYOV/f7wF+/a2i0hPC4FzRXMbhdBV1g02anX40YgaK7evxSMwn8yACBmQxcvhzGyzM1siJEWrBb4LneJWP7i+wa7/Zs7x2q2mMiPRWVoiQ/AGLtaxZ1ejxuXEsSRVnHCwtnVePG1I5g/s8o2lu1rfM+Ez2cwGIxkSSVGTORXB3JsJq41U6SSBE/Wv6cyD1VsxhpFAUj8ifOwx5EC62tWkNOXnVbsnzE1P+MhSCYWr6zIVN/OawAc9Pl8J3w+32H1f338zd8BcAN4zufz/cHn8/0BQGPP/7YB+AjAcQD/N0PXPGASid73BSE8KIlVf6gdvgkP20RKOrDrGK5qRqufx/McGqaPNa0IrdrYjqgcW+GL/7ldJ3F1VXL+zImW5+urA7miUFDCQ+I4UMKDkCHSejYHoQosVwFpGtcH9KvU61umobW5LiOLEEO9c3G+oPeRs6Zej+e3HsK0mjF9+gK9r1L/Tn/86k0HEBUVyIqCjs6o5bMmGdZdU3243jcNZExg5B5W9pvIbiWYKzPXtB1CqFtC4HLEZKcN08daVu/Pmnq9dswobxE6OqO2NmRlhwxGIuL91Kyp1+O5Vw6a7NZTJOCJOZPQ2lyHphlVtjNS2WbBN7YV3DwWW9m9/l2SZQWeEieaZlRpn/3yzmPYf/QMZIX2e3xn7wqDwRhMUokRE/nVgRybiWvNFPqqYnUs2LLruGXuIFn/LssKRngEtDbXY/2SO9HaXI8RHsGmWpq3GWssBkNqXUkLmsOpXGo9709r5VmOw3GwfMZcDj+2dGK3eJXmAuiMFf8tTfUP/H7/IgCLbH79ZwO7nOxAeM5SvJ3wHCBTS+3T2NYQ3lT19NSDNRAEYrkthPA8JCWxZqrtNRIeMjhIigIn4bD0oVqsfMmsGa3/vAsd3fjSiBLLged8Rzfuve1aU4VWok7isqzYay4l+DtCeHz+p8t45p/3s6rpLCBTm2CDprdqOBtVSIKTR8vcWrRu0q3oza2F4OQh5antZGOLTLrRb6X2FDtxNtCNijgfB/SsNlOAchwIz8Pl5LQKBfXvTMcrAOF57D7wORbOrjZVVGeqS3UsyOUQsKjgK/cIhjFB6kmk8DwHSQEcWZQeGmpa1IOBlf3qUe3QKRBIMgWl1ovIw0oE/OrfDpvs9MsjSy2PLysVAECr9n/n0H/j2ivLQDkKUA6KoiAQDMMpEATDEs5cDMEtOBCOSqgcXgyXQADK5YWPYGQP1ScQzrgF2TvMZeOTKZatf88UfwEw+hbOeksz4Xk4QE3VZnZ2b4gHKbBh+xGLc3IghMOihmqsaet9l2INPTnbrvV99YXQx8qE8CA8B1GUbaV29PeT+VgGg5EMhOdw/x3X4M7arxoanql5A+OxvK1fhSz3+1ggOb3oVK41VZLWq06yqjhVreakd3JzFD948GYEu6JajOUpEQDO/P0Vm0WAXK6kVWzm/QrN3WtONw7C28QTPJQCGM+zZQMZSUD7/f63fD7fZAB/CcAJ4A2/3/9WJj4rlxAIZyneLhAOCszO8Km5NyMaVRCOStoWXSD2oJ/dfACPf/NrppegpbEWv937MV5965OUE7BWAfdTc2+O6UnS2OASFiUEgmEAMCSkF/z1RMvBrKMzig3bj6BpRhVWbWzXft7XgGSn6Zfo7yRASz6r94ltZ88cCZ9RnvlgWaZoeyNO1+qN9OpZZ5NsbZFJNw4ASxtrsXJjOyRZwShvEUqLnJZ2xnHAvFW7te9WUeZCa3MdAGu7dPAcBGfMB6saZmWlArweF0rcDkS6RfMFDRA1yA0EwyYfvmpjO1Y/Vm8aExbPrcGLr/VqqWajGWauaPflO/qtmsGQaGmH4ID/OdelaTVbHVNaJCAQDOPlncewYNZEfHlkKQQHD0phebynWEBrcx3CUQkugeCOSVdh/WuHTROx1sfqEAhGTJIfALCm7VBe+AhGdlB9wpZdx/E3d47VYk2vxw1PkWBph3+60GX2cc31uBwyTt6Xz7/FlGRuaayFy8khGpZN2pycjU/Xx4OJtklDpvB6XFgwa6KWFPB6XOASxKCJ+kJYxcpqLBwIhi19p6JQ5mMZDEZKuG2avbsFgkh3XD8JArQ01lrkGAAxbkh3OTnLY11OzlTJmOx8wuXkLa/V5eQRHYCPS2k+Y1NV3Bqn95+qVnOyxxKegyRRQ4z1vW/dBMJzpmlxtnocpZOhNO/vL07CoTwunij3uOAkHCKDfXFZwN5u02sDGdlv5vP5HkBMKsMLYBiAV3w+37xMfFYuEZWpTdk6tXRwlzujeO2t/8IVI0vRMrcG//j9qfjxwttiQavHDQfhY8nfWROxviW2LeTtD77Aq299op0jGdkKFavtwM9u+j3++2ywxyHKKHESTQ5hdXM9rqwowZPfnoTyUhcWNVRr2xLUVcdte06YKrS0SUEC1MmE/nxLG2tBCGe7XSYXtv8UEoQHFs+tMTyjxXNrMi5lkAlEicJTLODKilJ4PS5cWVEKT7EAUcpP28nWFpl0I8sKyjwCFsyaiCtGlmBRQzU4Dlg42+xbOI6Db7RX+24RkYKTFThAsdTKd4AiIip4+4Mv8OBdEzC6shSlRQL+7d1PEc7QjZEAvPXBF7iq0oOn59+Kf/z+VNxZcxWAHg1SizFh9aYDmFYzRvv3yhR8+ECuM9EWd0Zy6CWDrv3KMNMYtqSxFl3hKKKijCfmTALPW9s2BUVrcz2e+NYkjPKW4N/f+7SnuoCazrmooRq//o8TCIZEuAUHOHDYsuu4pXSNJFFtwVr92Zq2Q6gcXoymGVVo23U8530EIzuoPmFazRj8aPPvsXnHMTTNqMIT35qEl377IVri7LBlbi3adv3RcI6zgZjOcrxvWb7+/Vi1dNOt+MVTU7Gs6Va8/cEXiIg9mv6yEvPlNObTrew+Po5MJNclywpKnASjKz0YUebG6EoPSpzEkPiN345NOWM86RvtRdOMKkiKAskiOb2mLSaFY+c7O7oizMcyGIyUCEcVy1g+HDVnesSojJGqVETLNLQ219suKKuxsJ0P1pPsfCIiWuc4IuLA5lGpzGcURYHX48aSxlq0Ntdp+RIlLg+QmrRJ8scqMkzyVM+9ctByp41VniOZ/MhgwvGcKdezqKEaXB7O+/tLOCrjzf2fY5S3BF6PC6O8JXhz/+cZm0fmHtRy3pJuMvUefA9Ard/vPw0APp9vNYBdAF7M0OflBH05sfjflZe6Yk24Xo814WrddMBQbaFQCv/JAFZs2I/1LdMgK1RLPsefP5kHqV6fb7QXs6ZeD0+xEwqlqCgvilWi9CR8uZ6mL/omhlFRxuYdx7Cs6VZ0dkfR0RnFyzuPwX8yoFV0rW+Z1ue2Q3WLYkShKPcIWN1cD0lR4OB5hEUJP/j5XsN2yDKPgKioyo2kXjXN6D+yApw+19nTOCi2Ler4ZxdQVurKu4pzl4vHXXVXG1buF8+tgcvFQw7n37JuIn23XA5uACAqKlixYT9am+vwH7//AnOmj8Pr73xsqE5//Z2P8Ze3fhUP3DVe8zPgYvrvEYXCW+Yy2KW6XU/mOLz61icmP/mNKVdn5L4QwuG26q9gydp9BrsCgMP/dR68jY6/p2fbuvrvZH14f0k0NuW6veQaqmQQAFMlp9PJ42JHbwXysqbJ+Pf3PjPZ9ryZVRBlBT975QP4TwbgG+3F3fXmcZHnOfzH77/ArKljcbkrgo6eRet7b7sWJW6H6ZnaPedzgW5s2H6kZ6sqGysZvbbiKXbC63FrMSEAeIoFOB28oQKotMSp7Y5TUau5rGwuEpXxvX942/BzvR/WS1woAIaXuTS7t4sjE8l1Jfqd3Q6QyRMqsf/oGVNj22e/U5/Qb1v5TlGyG5OZj2UwGNakGsuLUVnn52RT5XPveWnSsXCy15CpeUcq53U6eTx493iTPILTyUGJ9h6Xyi5rB7GWK3EQHpCMN9i+ob35WmVZwci4uUpsF1DuLktKkoKPPrmAlQvqQCkFx3H43cEvMMpbXDASHLJC8dGnAYy/eqQWt3/0aQDfmFIoYzlnOSeff396d4xn6l7yavIZAPx+/ymfz5dzSwfp1muzc3i8zRZDl+BA66YDaJpRZapkWtN2CI9/82vaOdSqU/05fKO9aJg+FgBACd/n9asa1eq2Xa/HjQfvHo8f/uLdPrcMxnSqw3j1dydwT/01mhafujKyeedRzJ9ZBU5WbHeWJNoGLoFqHdfVe7ByYzsWzJqIFRv2x7Z1zrsFP/xfk00a0JnSdi10XAKPL4/yoGVt76JAS2MtXAKPqJRfd1ySKFb3LPAAvZWorY/1vyP0YJKqvlsuofpJhVL8xc1X4f0PT5lkKhbOrsbLO2PbnZ9+ZAo6OsPoDIlY+VK75rf0AeiSxlqM8AigMk2ow59uZBmWdrV83q24p/4aOIj1mKBv9JrsItpAxqv+SB4x+kaf9CLgEI7IhirItl1/NNnq4rk1aNvlx+H/Oo+Fs6vxu4Nf4M7aMYZFjKWNtRjuESCDQ80NX8Ky9e8a3o3X3/kYD941wfRMLwUjtvZmt1V1KMC0d1NH74fjbfTp+VPw9z02pzJ5QiUWz63R/J1qp3ofpxY3lJUK8BQL8I32ar1B9P7GKRBc7Iya+o94PS6tcjmdo5jdDpCn50/Bp6cumxqKqo1s7fy2Otbqk+Wcw25MTp+PZXbOYOQP6vt6NhACbObomYrl++pJ1Z9rIDxvc86BXmvy90BRYLnLa/VjxrgmkWRT/JXyvHUPAd5CViOVWNopEJzriOSVVKJLILhpfCWWrjMW1bgEAikPi7X6g2CzyCE4eSg5+tzSiYPncN/t15o1sHkurXFZplo+X/D5fDPUf/h8vpkAAhn6rH6hJkNb1u7D/NbdaFm7D4GQOKAu2FbbLRY1VOPHLx/E+tcOm7Y08jynVVVYraipq3Kq09Sf3zfaiwfvHo912w4nff0OAA/fd6MWaM+aer3JkdttGRQIsGzeLfiLm6/SOtA++516rJg/BQeP/Qlzpo/rczUj0TZwu8ott+DQ/nv5i+9j+DCX5RZMRvqJRKy3hkUi+Xe/ZdlmhT1Pk2+qvpthi3SPvluuo/oxDhzWtB1ChbcEb3/wBVYuqNO6WqtVz2cD3QhcDiMUlrRkhZ3f6hYVuJw8GqaPw4btR9Cydh82bD+ChunjIJDM3Be7yg0H4eEtdoIApue0eG4Ndh/4XPv30iS25A10vMrHrYD5hgQgEIwY7MF/MoDNO45hVc+W2eXzbsXOfZ/izQNfaAnhv5461rSlc+XGdoQlik9PdZjGzOe3HsK0mjEIRyXTNrlyj2B6zqpUlvr3Cs1Pn2dHJmK5QiDeD+ttrKMrYvJr+4+eQWmxgMe/+TW0NtdhwayJKPMI4GSqVRM/cNd4bNh+BE+9sBfLX3wPD949Hr7RXoO/IYRHt6ho/lz9zDVth3DmYigjkhV28SXHAwtmTcToSmMTxG17TpjerUUNsfdIHWvXv2aMvT1F5ncvnT6W2TmDkT/o39eHn3nD9n3NVCyv9qRKJhZWtaXjr0GIK3l1C9bxtVsYmA8SnLyl3KPgNJ9XspnLSXFzuUSSTfGoO7ybZlRpc5DNO44halFenkosnY9SiaKkWBbViFL+zfv7iyJbL3LYNTQeakQkxfp9SLMNZGr++TiA7T6f74Wef0cB3J+hz+oXqYjOJ4ve4ckK8D/ngti845hWAQJA24rh4HmA662qsFpRGzW8GCsX1IHnAEo4cDLVzk8BrWKqr+vXy15wHKf9jV3i22rLoBiVUSQ4sKLtfXg9bgCxpit/utCJGXdcF6t87iMRHD8JUKtlYtUcvLYdUn8P9JWCZwPdiIiKTiKEssrnDGK/LSr/tqHYr7BzeVk+r9d303ejvqf+mpyv6Fb9ZHGPhICn2IlX3/oE468eid0HPse0mjFovOcGrbpiWIkLDgffp98KBCMgfJFlwDcQv6525z59vguEEEN37oR2BYqoTLWGiJ5iJyRZAc8D82ZOxMP3UTh4DgS0T9850PHKODax6rlMICvUsnIyEAyD44BQRMLZi92YVjMatROuQPvR06idcIWthIGsKKgcXmL5u7JSAeWlLniKnWhtrgfHAQ4HD0VRsGnHYTTNqMJXRpXizMWuXgkb9Cx857qDSJFMxHKFQLwf1qO3Y31Vsygq+NW/HdfsaX3LNDgoxQiPgPkzJ+J8RzeaZlRh254T8J8MYE3bIazqeQ6qv6GENy3UAL0FB4niC7sK4PifC4RDVKa646yr1pw8j9GVHlAYdxf6Twbw+jsf98TrFIRwIDyHv/v2TeB5DutfO6zFqqq9/WTR7Rn1sczOGYz8Idn3NRqWehpsp1emwa4nlZW/0GtLx8va6bHTq25tHthO0u6IhIPHzphkH+6YdJVpLEilsR9P1PkdBcCBJ9aF2uoO71Ub2w3ntKrAlmUFI5K4V0B+SiUmKtbK1WtON6nIrAxFeA644WovrqwoBeGBYSUu3HC1FxyX3kgjI/fS7/cf9fl8PgBjARAAx/1+f06J3mRKE1PTh+Q4rNiw3/C7/UfP4OH7Yo1XIMsghMeSxlps2XUcC2dXG7rZL55bg/WvxoLc+O2JnKxA1iWSE12/XvbC63Fr4vJnA922iW+7LYOSHBP/12vl6WU0+kK/dSVec09dcVXvk/qdN+84Zrg2p4OHlGfyD/mKvS4Wh3zrrON28ZYdod0uHpFQ/tlTKvpuuYiaNNAvwLUfPY3ZX/cZtngvnluDN9s/w/Rbru7Tb0VFGXJPgxK9dtW2PSf67df76s5t1ZV84exqrH/tMOZMHwdPiRP7j54xLKwBvYkbyMmtf6RjvEqkkcoYOITnsPvA56axfEljbWyRSKYG6arFc2uw9Q0/ptWMsZ5QcRxOX+y0/J3X48Jv936MV9/6RPuMq79chjMXuzR7U8dYVbd3qEpWMX3z/qP3w/p7uPvA51psqkq26f2bKo9EeA4EHC4Eo4atzuox/pMBKAqFg/b6G1mhiIqypV2Ho5Ih/tQnlh08jy5R0qTa9NJL+s+fPKHSJOm0fN4tWNpYqzUW1N7LnsU/R08srv8OsV19FBxVACnW/J0AkBTO5M/PBrohSrHiiEz5WGbnDEb+kMr7Gg1LBl3ndFTIpuovktGWzlRC1enkccM1IwyyD6quM40aj1WrpePnCYKTh6Rb7Osrdjeck1jLdQgEpvtACG8a7+ykS1PRls4VrOKBUd4ikDyc9/eXQpcsdAsEt0+6ytAzq6WxFm6BR7Q7fXab1rjF5/P9wO/3/8jn8/0csSUn/e/g9/sXpvPzBkKmDSyZ88uygnKPgL+89asocTuwckEdLl4Oo7RIwOadRw0VFmvaDmHBrIkYUzkMlPDggaSuX12FVRPHm3YcxQ8evBnBrijKS10mR55ogkp4Dg3Tx5r0qvWrurFO4xwkmcYmHoQHz8cajwmE0yYB8Zp7+pXUh+/rbUoYP3kuK3HhQli0uDpGuiE2ulgkzTpA2SAcUQyVqMGQiLZdxzF/5sS8rB7K9wFS9RP6BThRki23fi1ruhW73v8Uf980GecC3ZZ+a1FDNRyEg+AkBu2uyRMq8cScSeA4DrQfFWl2W+jUig995cj5jm5Dc9ZPT11Ga3O9TbUGB4rkryffn3ch4AAwZ/o4TaKqrFSA1+NCkZOHqADPbjbbtlotarUA/U+/+RCeYgHL5t2CzlAUZaVuUIXC4eDxx88vYvzVI/HqW5/0jsGP1Rsq8v0nA3h55zEsmDURV1Z4eipDrSt18hn2bvRNIu1gK63MOdPHaVXNag8IoFcC5rsNkzCy3A2FAhLHYcuu46ZjFjVUa1uYKeG1qmQesSTC9751kyY9M8pbhO996yZUeN1QZAq+5/j4iX6sEMONs4FuXexZb6g0nFYzxuSzl7/4Pn78+G221clqNXisCSIFzyO20G5hP3b2luniCGbnDEb+kMr7moq2u7ojr6/q21T9RTLXkCm9apqkrjMASCLF1jf8hrnc1jf8eGSmsUFaX7G7HkWmKC5yYPm8W8FzgEIBh4ODYlWIh+R3ouTjHNrlNC/GLmmshcuZf72f+osDwNKHag09KpY+NPSKN+yIiDY7HdLcMyvdC+cdPf9/3uJ3ORUhpSJQ3x9cTuuKS5eTR1Tn1DmZwutxY9XGdjwxZxJa1u5Da3OdZYWFW3BAlBX871++q1Uzxzfiir9+pWcVVG106PW4IYoK1m07rCVonnl0CjhwsaAb9gOfA8CXRxq18tRrkxUKF+HRJcoIXIwYrut737oJL71+FIFgGMvn3aJ1OrddSe2pEC9xEtOEgU/z/mHW1MWeqE4HSB3oN+84hr/725vyrhuurCiWlagP35ef1UOZ9l+ZJH5XRsP0sbhiRDEUai1F0NkdRd3XroQsU4PfevqRKQiGoghcjmDzjmP43rcmQVGo5nt8o72497ZrtWZaiSoV7Eim4kOMypA5Dk+9sNd0nEKpaRxQ+wIEguGkryefn3ehoCax5s+sMownQExXz8qOPMVOLVHcNKMKoytLwXEcHCRWZblywRRERQWSTPH3v+y148VzawyB4NlAN85eDOG1t/7LYG+BYBiCk+Cn/5KaveUT7N1ITKLmz3KPdJqVdITq16zsdkSZ29C8euHsalwKRjVpDq/HjWKX0xAHtjTWom3XcVwKRvHdOdX4hy2xRtsjymK7qiKijJZ/3Ge4xvjE9pq2Q2iaUaVtlVZ9sf4a7SSawlEZbsKBo9S2OvlyqO/KNjt7y3RxBLNzBiN/SPZ97cs/60mtqpezzEEIhLOs6k3mGqx2/Kla0VYV08mSSPYhfq4p2c7ljFXYqVRrU8Lh4oWwaUH0SyOKTM41pcr2PJxDyzRW5LBg1kS4BQfCUQkOB19Qa5w84SzvAU+4gayz5A3ZkmFJa+7F7/f/suc/z/r9/nX63/l8vqfS+VkDJdOamBGRWlZcLpj1Z6CE1z4TgKYnREETakKHoxJOne/Uqj8279BXN1knj/meFUs1KG+aUYWfbflAO/f+o2d6KvXqYtIefdwzJyG2K6CUAGfOhLQkERAz2udeOahNGpa/+D5am+uSrhDP5JbxVAb+QoTnOUtdLJ7nYGoNnOMMNQ1ovf8CxwGU5s3iib6C4GygGys27MfkCZV4+L4bLZ9RaZGAjs4IftT2e5PfUv2KVoGmGzitdlmkqpmZTMWHUyCgsvWOFJ7jtHHgK6NKEQxFEBUVTeN6y67jmD+zqs/rYRrO+UH8mIWehmsXOsKW9qH2OPCfDGDD9iM9DdGGgevpDzGirAinz3eaxlR1Z4D+XBXeIjx41wT84Y9nNO3a+D4UQ1Ezlr0biUmmYssu1rKL02SFGiqRn99qTAw3TB+L1k3mChr1mPOXYosj//uX7wGIJWpUeRr9NTbNqDIkGtRFG/21xPtou/j51PlOjK702Nq+BGDLruNaUpznOXR0hkGJYIgR7Owt3cUR8TA7ZzDyh2Rj9FQqalOp6o3vP9K769McbyZ7DQataEpBuMS7qgS3AxGR9qltnYquc7KV3alUa8sKTI2gn3vloGXFZyqV5TxnM4dOs5ZuOpFEiqc3vG/6fgPV+c4nojKw4sXCvQfZkmFJtwTHowCKATzh8/mKdL9yAngUwLPp/LyBkskEp1XFpW+0F5eCEUPCc/m8WyCGFKzUaTT/5u2PTVtyVQ3oNW1/0JrCqIMKIYCDAjI4SBwHno9VUHEyBSXA4rk16OiMGBLReuxW7+IhhIcM4P88MgWnzneibdcfEQiGDZqnpcWCbaWX/rNcPDfo1RysqUtfUE2uRV0F9JQIg31R/aLIRgO6yMUjnIca0ECv/6qo8ODcuWDe5NGtKgim1YzBP/3mQ0t727zzKBrvmWDrV9QKPApqCA4H4utU+qr4UCtS2ix0/Jc21gIc1caBn333dogSxc//9Q+GysFknQ3TcM4/JMSaDwoCj6fm3ozLnb22PWp4MTbv+AgAtDG+2OXA+Y4QRpbFtvxRhcItmJvExSo6JcPfqlX1i+fWgJDYexbfhyJV+88X2LthT3+1g2PxHmcZ723acRQP3DVekxs6G4g1xgRi9niFzU45NQ781b8Z/WVZqXXcqJ5TRS3EUP976UO1EOK2DO8+8LlJoknVpH7y25PsvzMHzLzjOq1AQ32vQmEJJU5iSBwNlr0xO2cw8odkYvRU/HMqVb2yQm0qhW+0PDbZa1C1oq/o+U52lc+C24FzHRFT7FxR5jIloVOp1nYAWD7vFpy5GNJiqcrhxabcQSrV2rJs8/0tKj5TuVZwNnNoLnfLie1tbOjFjXbkY/PIdOJ08Fg27xac1b1jo4YXw+lMr8xYuu+lCKAKsSR0le7nEoAn0/xZOY3V6lvD9LGmhOeZi70Vw2pVc8P0sRg1vBgr5sd8bZT6AAAgAElEQVQqnHiOg0sgACjKPYKpKczyebegU1IMejVqwrqYOLH1DT/uu/1aLJ5bi45O60qsvnTkrKqFW+bWIhQRtQqrT09dxvJ5tyas9FI/KxeqOVhTl8QQnock9coeqNuSCM/lWwE0uqMKPvrkvKHLcvvR0xhW9WW22JBlrCoIykoFXApGDfJAo7xFeGLOJFwKRvGnC12WfqXCW4RlTbfizfbPcE/9NYZtj3aVcI4U9OoUmaIkgTacviLlUjCqaf8OH+aGKMmG7yo4HWiN07h+fushtDabde4Y+UFfEk6yQuEkBFv+33HMvOM6g20veagW8++vwsP3VYHnYxpl3REJP910AF6PG/P/6ka4+Z7gz8KO3YIDa38wFWcudhmqnFdvOoDW5nqmGcsA0HfFlpUNA0gq3tPvQCktEtDaXIdgSMRFm4p/SVawpLEWnmIneB5Y/Vg9ZJnaVsCVlbi0n6sJ59JiJ579Tj2iogxJUfDkmrfh9bixfN6tCIai6OiMYue+TzVfXFok4Pmth7Smiba2TznD7sCzgd7eK4kqpxkMBsMKVa/59PkuEEIsq4VT0opOoao3tfOmrld9NhACCG87b49INGkd2VSqtQGY5glLG2sBp1HUwlCt3adedqKeWv2/Vvs5NA8lR2fRrAlh5rTO8wVFoejqFg12+3d/exNK3M6+/zgF+HSezO/3/5Pf738IwMN+v/8h3f/m+f3+X6fzs3IddfVtlDdWCG5XFRJf3eQ/GcCKDftx4VI3Fjy7Bwue3YNHVu9GVJTByRQP33ejlnz2jfaiaUYVOHC4eDkMr8cNoDdwPnMxhIgoY//RM1i67l388teHITgJFs81XpdaeZwIq2rh1k3t6OqWtIlvLMnCY1FDteH83/vWTdi254Tps2Q51jXcQWlM/iPLWwnVQVePNugwoCjUcluSouRf8kKWFby4/SjmrXoT81t3Y96qN/Hi9qOQ8zgRE2vkx+NsIARKeBCSVneeMdQksd5HeD0uNEwfa0oA/GzLB5g19Xq07fojWuL8llr1uWLDe7h90lUQiHFh69qvlGHx3BrT3/ApiK9JAH647l00/2gPHn12D5p/tAc/XPeuFofpV8r9JwNYtbEdT72wF7KsYPmL70ORe79rKCJaLnhl631S7UXiuLyyl1yF9PQ8OHkmiAsdYZw8E0SXKBvuK+E5hKMSptWMMdn2qpfaIcsUP/zFPjStfBNL1+5DsCsKr8eNWVOvx7Obfo/nXvkA5aUu05j6xJxJeH7rIXR0RrBiw35tDFbPrS6ixr9nicZ6Zh9Dk0R2YGfDMrik4j39DpTntx5Cy9p9WLWxHZt2fGTy10saa1Fc5MCG7UfQsnYffvovHyAYikKUZax/7TAWzq42+er/u+ePaJpRhdbmOiyYNRFlpS60/OM+PPXCXnRHZDy7KSbL5D8ZQEdnBE+9sBerNrbjzQNfaL74cldE0z9PFOcqNlVPbsEBWTEnYdi7wmAw7FB3x7Ws3Yv5rW+iZe1enA9G4RSMAWgq47RVXkGt6o0nlfMme6xaiNaydh8efuYNtKzdh0BItPR/iXRkTcf2VGuv2tiujSH7j54x+V0gFpOvjBubVm5st8yNxgpF1HNQy6aCAOAg5ryF2th8INeaj3Noh4Mzjd0tc2vhcBTOGOcWeMv3zC0Uxj2QZYqf/Mpotz/51cG050syVei51+fzPQGgFLENxgTAdX6//9sZ+rycw2r1TdV11DtlfXWTKq2hVm3cWXMVaidcgbJSIbbyAgoO0KQ6ilxOrN7UW6GibjNUt0S6BQdkmWrn958MYOm6d3H/HdcktSqox65aOF6P7+zFblQOL9Y0KB0k1tzwyW9PAuE5rRO6zHGGirHBaAbImrokRpIpJl43Evf/+fUgfEwn69XfnYCUhxXihOdx/x3X4M7ar2rf5c32z/JWAzqf9cutdj8IhDM1OFX94ejKUjRMHwuOp1jWFKtE/lNc1WerhXSOLCugoFgx/1Zw6LXfb945Nmn77WuXBOF5TJ5QiWk1Y1BR7obgjFWsEsLD63FDUnq/K4WNTrSFzl26yWd7yVUo4RC4GDFUCSxqqEZxRQkg90oYeIqdKHY5DGO8WjkTikimiku1skbdFbVj36eY8/+N6x2zCY/3j5yC/2QAwZCo2Z96zt0HPu/ZZSQnvcuI2cfQJdFuM14gljY8ykss/d7oylKsXDAFikIhOAlGlhXhR4/fhnXb/tOwCBIIhhGKiJoth6MSvB4Xvv/zd4yJg5fasWDWROw/esawg2REmRs/fvkg/CcD+OJMJ2ZNvR4V5W7IMsX3H7gZgcsRVJS7Dddo9y5UeIvQ2lzXZ6xpV4UdjkqGSkC7d6W8PHeTCgwGI7skq9ecym7gVKp6UzlvssemIluZSiVtKhXYye5cJoTHpZCoJavVSulyi5iGgsLtIoamc24XAYXZpxOes4254q9VkhXrObScu1IOkkQxrNRpsDFwFJKkFMwuoHBUwdsffIFlTbca8gX31F9bEPfA9h2jOdyEUMe/AugGMAHAGwC+DuCdDH1WzqJqJcX02mKVUfEJz8rhxVjaWItXdh03SGuoTbmCIREXOsL49X+cwJzp4zC8zIUH7x6PqChrHcaB3u3c+i2R4agEqUMxaO1NnlCJ2yddhZa1e1OaaNoNEHo9vpg24Ecx0f3mOjioAkixfnUOAAQcLgTNXcZHeATLn2d68psLMiC5TJGL4K66q7Fiw3vac1k8twZFAoEYzq97VOTicefkMTh7sUsLMO6cPCZvNaDzXb88XstSlAGngxgW4x64a7xBaqilsRav/u4EptWMttW2demSA16PGw/ePV6rklOTK4KTh9LHgptKX8Gmy8mhYfo4tPX471ad7qj6WbI6Dlj4/2wteOW7veQikkxNY/CatkNY1VwHZ1ySalnTZEyeUGmSz3rqwRr4RntNVaWqfIzX48Z9d1yLjs6IQdN2SWMt7r/jGrQfPY3ZX/cZftfSWBtr9iMnrxnL7GNoY2cHdjbc2lxvGe9duBwGAIOO/dLGWsy9+wZ8euqyoRhCv0AIAOtbptlWGAO9O0gAYMPSOxEIhrVx4PV3Pjb518Vza/Dsd+qgKLHk8+enO6zfBcJBjMqQkXihBQCWNtYaEhaqlJ3eR9u9Kz9ZdHuanhaDwch3UtJrTkHbPT6vYKfBnOp5kzk2FdlKp4M3afEvnlsTaxYepyObSjGYwyax7SA8oDuvDM6yUnq1heRdVFSw/tcfYtbU6+EWAFGiWP/rDy17BgiEw4N334CzF0M93zP2bysNaLfNHNrtIpAsmjHmAoKDx7lLYUNx4+K5tagoj8kKFgKyouDVtz7Bq299Yvj5N6ZcnbMLB+nEQazzfQ6eQzqVYzJ1L8f4/f5rfT7fWgC/BLAcwGsZ+qycQl9dITh5KDIgKYom62DZPdvtwPyZE7WksG+0F/fedi1++It3DQH9ll3HMX/mRKxpO4Qn5kyyrUjWB86Ck0BRgB88cDOGlbhAeE77HPVvVm1sx6rmOnA2ek6x7YbmpoFPfvsmeIe5sb5lGmSFYtueE9qEQ1HM+i72k9x6LWGkVswEgmGUe4SMZ2ZYUxd7opKiBQ9A7Hmt3nTAUsMr14lKVFssUQlHJUSl9GoaZYuhpl9OCA/Kc1jUUI01bYcwa+r1WqIO6K0eefqRWPXd5AmVhuYqarWE3sc0zaiyTK5YBaB2CCSWYLZrOBIRFbRubEfTjCrT9aqfpeoARhQF5R4Xfvz4bRBlBYpCY3rUFlUW6Wao2UsuoNjcU0UBJN441rXt+iOemDMJf7/+XYONPLv5AJY13Yrntx6C/2RAW9TdtucEnpp7M0rdAgjhsOSf9lmOmxQUS9buM70nqSaOmX0UJnY2DM6cEFg4uxqK0ttEVT1WndTH/BqFoiigFLiqslSLB1X9REud51KXph29bc8JBIJhOHrizUAwrBVWxPvX1ZsOYPm8W7Gm7ZDWfHPrG37Tu7D6sXpQwkNWKKiFtIi20CIrKC92YnVzPSSFgudjEzECGHbt2d0zsYAqxBgMRmIypSOrxpOp7GBOBsHtQESk2nldTs7ULDCVSuVIVMbBY2cMfXd+d/AL/PlNVyFeMUSWFYxIsrKb180T9AuFfFxvIslmAcBqBy/huVi+QUe5R7CuwAZwKWjeNVTiNkdKokit59DNuTuHjs372+Ou2Vq7e6ii39lqLDwqDA3oZN+xgZKpucWfev7/BIAb/X7/v/h8vvzM9KQAsai+0z9AtdKCU7dfyBQgPC4Eo4iIsvbCWyVf1CBcXVW1a7BV4S3C049MQSgsQnASBIIRdIaiGDW8GCVugq6wZOmUzwVieqvxlcf67zTxupF4ev4UdHRFEBVlFBc58D9ng1pF6T23XYO/uPkqlBQ5Y1uAFQ68rqrYfpKrwOtxmyoelzTWwlPiBChYZfIgkEjDK9+SEhQU4YhsChryddPsUGkwpsoUhBUFUGRs3nEMTTOqMLrSWo5DXYB9eMaNAID9R8/oqj55dEd7bVaVMdATC0CT3/4WlW0aqfQk+FSfZvdZMqXoCIraOSZPqDQltLOx22Oo2Esu4bCYYE6eUNkz9hl9p/9kAMHuqGGRVU248Rzw3TnVCFyOoNwjoMjtwPcfuAmdIRH/+5fv2i42y4qCQDBiY3cA4mSuEsHso/AghAcFh2e/U4+OzqhWQKBWung9LqxqroOiUHx2OoiXdx5D4z03mOzN63FDQSxZcOp8J/b95ynU/dmX8dfTxuLrk8dou/dUXcX4xbw39n+GV9/6xFA4wckU3mInit2OhP41GIrigbvG4+Wdx7B60wE0zagyLEyqieF/fv1D7D96Bj9ddHvChRb1PYnF5wAHHhfiKqafeXSK5btiVdnHYDAKE1Wv2Vy8gIRVy4lQdaXjzznSIwwoCS24HTjXETGdt6LMZUhCF7mI5XcqchGEQ8YYw+ngcMM1I7B03T7DnMvp4KBEjZ9PCI+LnVGsfEknl/FQLcqLzHFxVOydJ6hx1OYdx0zVynaSSlaSd30VmuhJtPMtvujOtgo+zVIG6USWWTGCy2ld5a7uLBzqiJKCjz65YFo8GuUtNi0eDYRMKWqf9fl83wdwAMD/8vl89wIoztBn5Qz66rtZU6/XnJTaLDAiypDAGQT71b/p6Ixqgud2wfZVo0q17Sfb9pwwNW1ZPLcGHV0R/OyVDxAKS1iydh++//w7WLftMC4FIxBlatt4LxgStWoQ/Zqn/ju9eeAL/GzLBwiFRXxlVCkud0axbtthtKzdh3XbDqOrW0RJkQOdIREta/diXutuQ5MC+6Z/PBqmjzUl3VdtbMfH/305YaMDRuZQbU1PbKtT/q2DyjIsg4Z8bUKYaoOxXERd3Fq8di8ead2NU+c6Y/I9G9tx8kyn9t3Ubdgbth/B/NbdWP7ie+gMiZh//0Q8+516rJg/BW9/8AUiIgXP9foYdZFOj+pvnAJJqolUospQoDdxZ/9ZnCGBPa1mjCmhHe9zM8FQsJdcg4Biqe6eqosLLWv34rPTQZM9KIqCB+8erzVh27D9CB68ezwuhyJY8Owe/EPbB4iKCn6x7TA++Z/L2rhrZ1scxxniBpXJEyohSYptY0Tt+nWN1AjhDN+F2cfQpreZ1F489cJebNh+BA/cNR6TJ1Ri+bxbcCkYwfd//g6aVr4JSabYsP2IpjmutzffaC8evHs8lqzdi0dW78a6bYdxV93V+Pf3PsOjq3fjp/9yEHOmj8MIj4BwVEHbruNaU8GmGVVo23Uc468eCSCWyI6KMhyEaP6wL//a0RnF81tjO2bOBrpRViqYjjl1vhPTasbAN9qLYrfT1k9boY9/1Tg+FJEsfWlZiav/D4TBYAwp9HrN65fcidbm+gEniu10pQeaFIuI1oUWEdE4P+qOWPvw7oh5gVtRgN+8/bHh2N+8/TEUi7VwmeO05LP6+StfaofMmf0y4XnccLUXV1aUwutx4cqKUtxwtbenT1YvDsIl3VjQrtAkajE/pBSWcwJqMZUkNnNou/EmF7DN01g8i6GKpFjvmJZyuHlkOnELBDeNr8TSdfswv3U3lq7bh5vGV8Jt1e10AGRqfvEIgAa/37/X5/P9HsDTAJ7K0GflDPpkhZpEttIyXT7vFridDkiKAh6xwFtNKD+/9ZBtdXMgGEH5MEHTqXt55zEsmDURXxpRgvOXurH1DT9mf32cofIaiNOmhPXWypd3HtOOBQdty6J6feq5/CcDWLFhPzYsvdP2M1a9dMCUYFGbwFjpPAkEpgZk6t+q91E9ByN7EJttGITn8k6qRKHWiURqFTXkAXr9cnAcQGne7RKIl+Rp2/VHzd627TmRUI5j9aYDWNVch6de2ItR3iJtdwghnOZH9efQ+7r1rx3WdJvVCmq7KmSrKtdYhWBsK5YDwDMLpkCRgf/z6BTIMsWv/+MEDv/XeSxqqAbHG4NV20rpDFcXML379CP3bNnvvae8Jm+lH89V2/N63JqsFtA7Zj7+za9p/17VI+fiHebSjrM615LGWkRFyfS7yRMqMfvrPix/8T3t3w/dcyOiFOB1EltWWrjL593SIz+gMPsY4ljJoT2/9ZC2NXj5i70ybZGoZOtTG6aPNcWB+kpkfewmKxT7j54xVCgDwMw7rrOMk9X+IEsaa7Fl13HTO6DGrWqcOMpbhOHD3Jq/1h/z6F9VYeHsaoTCIlbMn4Jd73+K8VePRFmpEJOqs6h0A3pj+vjrmzyhEs88OgUcYk22HYhV3DEYDIaKqtd8RYUH584F+135rJKKrnQmzisriqUPf/g+8+dTUFPPi4WzY7tO4z2lZFN1K1nstnUL1v183AKPSHfvDeZkigpvEZbPizUuVyjgcHDgLJLKqUiQ8TZ9YXgbfVyrcSuX4Xjra+ay0Cw9V6DAkNoxnSrZkl/N1Jz3J36//0EA8Pv9T6EAks+AcRurmkSOT554PW4EghGsaXvfYNibdxzDyztj20pGDS/CkodqsUq3HWVRQzUEgcem336EBbP+DK3N9YiKMv50sQv/sOWQprX36anLePqRKZbOVNWmHFnm0rSWKAUuh6JovOcGBEMizgW6EOwyTkxb5tYiFBG1pjKjvEUJHbbdzzlKLZMgYlSGkxDLRE8wJBrOwcgeEdutTjflXWWcnag+IekV1c8mqn55RU9wm2+xQbyWpv9kAJt3HMPTj0wBpRROB4/vNkzCyHK3rU9R/7usVIhVeVLg9Xc+1mxWEHisaq6DJFPIMsWrvzuB/UfP4NNTly2TJOZO3jZaWARQZIAnHLp0Ehvq1r1v1H21p4nJTQa7s1tczIbUAdO7zyz6SZz/ZEAbz0dXluLcpW4oNpUzDl11sppM8xQJmp2o51owayKuGFESm+jwFP9zpsvwOWWlAkaWFZl6Sai60/qFFqsE5PIX348tFFPK7GMIou9PggRyaOp/q5y7FMbuA59rCyPlHhe+2zAJHAeUl7osz+Mpdsad114TVaEUC2dXo7M7iqYZVZoUiOqTvcVOzJ85EcHuKFYuqMPFy2F0dEbx8s5jBt30RQ3VEBw8FsyaCLfgQDAk4uWdx1DuEaBQaI2g7GSQRniEmNazLjalPdcYH8erY4iqHc3elaGDZ1gR3K58i3AZhUCq2rTJ6Dqr5503YwJqJ1yhbblvP3radN6UdK0pZykl2mrRg4XnkbRchihTGw1ms8prd7dkaCq7tLEWrmLzcalIkBEepma3i+fWgPDmqSSl1DAfCYZEvP7Ox5g3sypn9ZQ5wPKaH/2riQWTgJVtZVbqMyYbkUtkS341U6Ps13w+H+f3+wvFXgEYO7mqlSIOwhse5N9+Y5ylYf//7H19eBTluf79ztfuJtkky0cCWj4EZZEQegImgSRAC8jpURA4nEqoLcQaosQKtbZC0F9BigTbWmvboxWwAp4WPOeyiCg9h6IVASvgR4tEWC0fQgETAptkk/2emd8fk5nM7LwTdiEJCex9XV6XZOd7nnne533e+7mfhbNHKczibZ+gqqwAzhTeEEhvevMIvL4gymfkovaiH3/e/wX+Y/IwPLF+v3ZsVSeVEOCp7xUjHJHAsYw2SHl9QfzxL5/jvrtGwuePwB+MIC1FwM9e/sAQiG/eedRYirLxABbOHoWyaSPw2u5/YObEm7VSd9NkQpITSrAwLEEUDEBkEzt6ybx8MK3NcJRBtru67GsTLMtokggqtKRt92zgawmGEPzgW2Pwiz98qNnXD741BgzpWFH9JOIDyzKIxvgK90AXSqcOA0MAlmXxxt5j2Lr7OJ5fMsnS16j/73LaIEoSUm2cpt3FcQycDgEXGpXFt2A4im8UDcbp2mZ4TnkNSRKX0w4Z0BpNqQNjMNy+3lxYopdELi8fB68vCJ4jWL5gLOou+mEXODBM/N2+k+jeiGURLy8vNNip55QXbx38At+dPhLZvVIsNQnVRVb9v7N6OQwLH15fEALPgudZ1F7wo3eGHV/JcmrNONdv+wSL5uQhogscZ0+6xTSRUBoZ5yabDl5nuJStAortKU2rZU0X+kDNObjSbVgwMxeRiAQQYP1rn2gMOFWK4lI2fea8D/v+fhZL5+drE/fCnGylckWUcO5CC7bs/AxeX1BjLXtOeQECRGVFvsaZwqOxOQyeY7F+2ye6yX8BAq0EiYpZuch02g3J5UfuGW2oPLCSQVpdWYx1rfcWy8DW92hRkfxerk3YbRymP7It7u23Pz2jE68miSTaYBcYqlZxLPsXiF/XGQBSbQxGDOlj0GuuKitAqo2B3992XIeNruPvsDEI+o3nt6o6lWTZpCPLsQyWzL8NTc1hjdWcniaAYxlIMVVYUVGKKzkYBbTks7rdkxZEE4ElFnrd5soYUUTcjQU5lmDW124xzTs5lkDqpgE/IQSzJ92Cn/9X2zX/8NtjQAi5bhLQkmQls3J9PAFVOoZK2OvA3E9nxU5nAdS43e73ATSrf/R4PIs66XzdArFlzgLPQBTbVvbcA13ok+mgGvaNfdOw/rHbwbEEz7/6d1NyWUVGmoDG5jB2HTyN2wsHGY6tlgiqDRDVbuXqCh3PM5g+fqgWjKsrh6rEhhqI0xq52AUOr+3+BxbMzIXXF8LFpiCWl4/FE+vbmNzLygqw8/2T1HJhgSWQYJwExTJRZk0c0tr0RlkRfXPvca05jToYJNF1SBEYQ/IsGI4iq1cKUgQGgR7WbEeUJHAc0RZ1guEoOI4ozPyrfXHXIaIAXnz9sOYrVJ/1+rvHMDl/EDLSBNxRPAQTxwyAJEmm4HBZWQH2/u2M9v+7P/on/nzgFH760HhEo5KJHaEu4C0uzdMkitQkSZuO6T7D8TOdAs7W+uiLMK0LalYrxaqmLssSBIJRw/U8fl8h1jxYgqiYlDroyaBJyOgXuRQ5jOH43fbDuGvCUPTNdOAn9xfhbH2zlmyrKivAlp1HAUAb5z49Xo+B/Zz4ywensbx8HFgGECXgvUNn4LDx+OWWjwyLtPPuHAGBY8EwBARt8UbfTDu1BJYQAtaCcZRsOnhtoj25I9U2Hp13G7684DdMlpcvGAueZdDYmhiIREWUTRsJZ4qAXQdPU+VhYm1a9b9qld6qhcVgGYJoVMI/63ymxLPacPutg19o1Xj6pt4up90gPffCHw9pTOgLjQpbW63wYxliqjywkkG60BjEzIk3464JQ8EQAq8viEyn0FoxICS/lySSSOKqIhiWLJpim5OfVrrOtG1bQvEdNxAS8enxekNzsgM155A+sr+5erA9VnFMuMuzBCxhDHHysrIC8CxBKOa4VslBqVUyVEUii+wREZq2tbpYv2XnUdw/cxRikVBinSFIT+UN8870VB4cQxDTh7HbgBAgxW685hQ7j+tIAtq6YtpCZuVag80i92MTGIQ7MPfTWQnov7b+d91BX+YshUVwLKMx3mZPugVfXmihGvbJcz6s3/YJlpUVoMEXhrcpRN0uPdWG371eAwD43es1WvCvLxEsn5FL1eVbXVmMn7yy37RyWD4jV0uw1HnpjVwkWdFz0idpHp47Gk89VIJIRAn0BZZgWskQsCzBqgeK4PWF0NgcxuadRzF36nA4U3l4fUEt4a1norgHujDm1n6G4y+ak4dPT3jhOeVtHQyTGtBdiXBUhq8lYggKfvCtMUijlDp1fxD8dNMHpu+p+kFzOVgSnQ9VD7TBF8bi0jz065UKXyCMu6e48dSmttI2NXkxIe8Gk0+Zc/twjBjSG8+/ekhLbkRFydTMRO/jnt3yMX5yf5EhSULTMVUZcVt2fkZdUFMZy1YliTzLwJHCIxKV8dOXPzAce9WL+1H9YElS6qCHI3aC4znlhShJeOSe0chIs4MQoMEXROntbvhDUcPCr8KqjMLlFDA5fxBmTrxZm/SUz8jFxaYAvn7bAE02IMvlwMqKIk1OA1Bs6alNBzXbfvHx2yFDxsNzR+OZzR9B4DlUxzB11BJYq34MSSb+tQmarf7lg9OorixBJCpClAB/KKIlnwGlKqTBFzJp6G/ecxR3T3FrlSTb9xzD8vJx4DmC2ot+g0270u145g8faf75dG0z6i76qVrOauJ59YYDyEgTcN9dI7VvRh/T1nkDeGL9fmS5HFg4e5SWfNYzp++7S9L8KxPD5rGSQWpsDmP9tk+0akS9ZA0Hc4Ve8ntJIokkuhKJaEB3xraiJGPdthqs21Zj2DZ/RD/TMRPpIRQMSyZJsNUWyXK2PTlFHTszEVmNqKW2tTlZnUhiPRyVsWLd+6Zt13TjeWdUlLFyvfmaO1r/tztDH0fr810yrg/CWjQqmzSwl8zLR6q9YxfbOyUB7fF4nnC73Q4ANwOoAWD3eDz+zjjX1YBeS0/gGUgiLBv3xLKin/79R6aExtL5BXjhj4c0p7vs3gK0BCJYOr8AazYaA96d75/Qgnk1+FfZHqrDaK/RFe3v+lL0wpxs9Mlw4OnFEzQ2yfTxQyFJssaoVvd7Zl5/L/4AACAASURBVPNHymS2NdCPiIp+kATWUPIIQNNcVUuFX95xxHCdtEZj+gmJev1JdB0iomyYkNZ5A/jFHz7skTpIsXrDQNuqecf2de068AKLsAicq28By7IQWFxRh+2uhBrEDchOg13gUPXcXs0/0JLHACx9iuoPC3OyLdkRqo+r8wZACEEfp4CKmbm4766R2t9j95EkwOsLahq7zhQewXBUkfto7ZDM84zJT1eVFYDnGTCQEQp1nJaW+r5VPb+e9L6vRdAmIqkOHs3+CH78gmKry8sL4UcbAx5om1yteqAIZ863GNj1AHDvtBwwDINwMIyH546Gzx/Bq29/jsaWENWWBvZz4oWlk5XmggyD42catIQg1fZkGUS6/KaUSTvseYi1VfdAF75+2wBNL1yNQ/UNp2dPusW0MKfGZOrCx/ptn2D6+KHYtKMG904fiV//999QXVmi+fFlZQXw+oLaOWlaz/o4T20m2CfDYWCaWcW0N/RN0+RC9JrQAs9CFCUlRm+tRlHLsd86+IWpokbf0NAucNrxtd4AHdTEVT93iPcYl7NPEkkkcfWhfrt1Xj+gawJ82cdLQIM50W3j0ZZOJKmbSA+hRJLlDAO6nKLSAkYDB2DFgrGo1bE4s3ulUBcNreTRaBrUjEVinWHMco5RiwpJWnPF7oKu0v/t1pAJXtv9D4Ptvrb7H6igMOKvRYiSrBHBgDayS49oQuh2uwsBbIWyHlUE4O9ut3u6x+N5rzPO15XQa+npywL1SWJXCq8NMsbgkUGmUzAlNAKhiJZEqfMG4BA47P7wNO4saZOkIAT47IuLmDB6AM7Wt2hl6hmpNmzeeRTFX71Bc6DtNbqi/T3YmkxRSobdMZOSfOzYdwK3Fw6iJ2l0ZScsy0BmCUJhugNTJxHqhEN/nVYTDDVxpA1ySXQZLJO2styhCeiumGBZaZYzPbSuiBdY1PvCJs2yPk6hRySD1OCQIYzG6nSlt9/QivabWq0xa+IQTBw9AGfO+9rVJM1yOcCxBIHWqg0bQxAFXY6Aa9WFU5nSDENwY980RGVFW18UJTCQkZbCYWVFERhG0Y+WIaPZHwHHkXYmASQh6lxPf9/XImgs4jQHrzUPBmBIZulR5w1AloEPPv0SQFv/how0AXYbh0AM+2DRnDyEIyLVliAD/2/te3A57SidOgy3jegHWWpHx63V58XTlDLWN9t4hqormbTD7o1YW6VVfaxp7fWhSr+154/rvAEM6ufE8vJxCIajuHfaSATDEU2PVE34qr1QXn/3GKaPH2pg9OsZy2riORiOoqqsAKFoFDaeu2RM2+ALISpKBk3o5QvGorE5pFXCZLkcWLFgrMI6I4AsASAyqitL0BwIo/ZiwJC81utXq8QHNSa5kiausTrctPlCR+yTRBJJXH10xrdrF+gazDQN6ET0mlNtdG1pswY0a3FMFkG/8Z4YhlDl62hJXc4iVuFYBogp+ScAemXaDDIgMlGYqbEUtUjEKMf3WFkBwJspRxxLLDSozVrNkahkkkfb+s7nmHO720RmYizifybB+L8r0VX6v90ZNp6g4t9zIYlKI8k+mQ5U/HsubCzB9RDmdhVhr7OIjD8HMAXABY/H808A3wHwbCedq0uh19KjMURWbzigfaPqAFT13D5UVL+Fquf2onTqcGQ6BazecKC1TJbFpjePaMfPcjkgyiImjh6AZc/tQ/mTu/D4b/ehyR/CsEG9YBMYzJ06HOu3fYIlv9mLH699D18bM0Br8JLlcmhBf5bLoR1z0Zw8vLn3OKpam8aof19WVoBB/dPxyx9MxLw7cqji+v8xeRicKYK2H6BMmJeXF0KWAZllwAssvP4ITpxtwtn6ZsO26rnUwF6dxKhMFPU3q3305Y5JdB1UHSQ9slwOcB24EGD+RvbB64+AZTvWNaW0BmN6268qK0CKradxuRWERXrzu54yOIqiBDvPGVidKTbe0gdY+Yde6XasWzYFdxQPweoNBzTJDP17Xlyah1ff/lxjSpxv8BtsTWCJ1kxL3WdZWQEYBnj3o9O4e4pb87eP//Y9NPrCkFv16EMRCY8//x5aQmE0B8IKk/vJXVix7q9o9kdgExiTL15cmqcEcwmgp7/vaxH66qa1VZNRXVkMOUZv1uePKPppFNs9W9+Mfyu6CYU52fjOHbdqNnb2fAuqNxrf9a9e+RgCz5jsdMm8fLz0xmG4nHaUTRuB5189hPur38KP176HlqBSRRXr8wSeQZQQyCzTrp+l+eb6xhC2xDYpTtpht0esrd7YN406wejXO1Wzl/SYmA8wxmSEAE+s/yseefZd/HjteyAgyMq0IxSIIMMpaI2rAeC7d42kVrjNnnSLlnheOr8AAs9iy86jICCQZLndmHZxaR6cKTwGZjvxs4fGY/1jU7DqgWLYBQ5/+D+jjW5881NEJQkNvhCqntuL+1btQtVzeyHJwJET9VryWR0r9PfbUcSHWB3u2PlCR+2TRBJJXH10xrcbDEuaTFd1ZTHKZ+Riy86jCIbNCe1AiL5tIGTe1koDuiVm20SOqSZ1l5cXorqyGMvLC7Fk/m3gKLGvwNPnZwJvjk8EjoHfH8VjzytxyWPP74PfH4XAGbe1akJIe/4soGlQVz23D8+/eggsYajJNp5jNHm0B556G0+s/yu+ftsA8Jz5WnmWUMctvhv3s+I5Rht3AWhERNr9XbMgBE3NEYONNTVHcL0IYauEPT06g7DXWTm9FI/H86nb7QYAeDyeHW63+8lOOleXQi9j0Z7UBQf6AKQK+8+c6EdUlGATWK1EMcvlwI/Lx0LgWKxYu0/bz+W0IxgS8dTGfdQydZVRTACNWS3JMlZWFKGxJWQoTzxb34KfPTQeEVFCJCKhvjGAzTuPYvr4obAJdOYyQwCbndU0cayY35t3HsXMiTdjwxufWsqMqPfZ1+VAxcxcCCxRtJ0JXZPSmapMmpJlh10PliX4cflYnPe2lTD1daWAZQnEDpoBWQVptE7FVwK/LnDTN5momDWqR2o6JVKy1l0RlSQ0Nod1q+0Sls7P1xbB9BUYPn/YxLxYXJqHn//Xh/D6glh5fxHqvIo+6DsftrETOI5BY0sIZdNGwOeP4KXtNVpSRG9rtPLqUETCrTf1MZUiqXr6DNrGg4xUm6Zfr9+u+sES2ATW0NDDJrAQJSkhu7sW3ve1iFhWZKze7Ktvf47vzfkqxa4LsGPfcfzruMGYd0eOxgwFFFmXOm9AY0Wr/qp3hgNbdno0H+ZKtyMQjGB/TS1+cv84Ta8OaG2q1hDA//71pMnnfWPcYJPGLW1stfLNtCbFSTvs/tDbqmzBcrLxLNZUlkAmMkJh0VRq/PDc0Xht9z+wZF4+Xnz9sHFy/1LbuB2OSBqT2j3Qhe/PzbOsYFk6vwCBUERrJuge6AJDCOpbqwSerCxGgy8ESZLwk/uLcLEpqJVze31BrKksQZM/bIgdF83JQ4MvrFUWTs4fhGhUNhEsqjccwJMLi1GQ0x9ZvVIQCkcN8XhH6jwn0hTrSvZJIokkrj4649sVLbWK6brOiWwbT3yZyDH1SV39nJ6FuY9byGp+NtM8P7PSi47VVU7k+YdFOe55qCjJJuLhs1s+tpAnUNizKxaMA0MASUbC5JOuRigs4sMjtQaG+TsfnsbXxgzosXKViSIUsWjKeZ3oYLMsqHNxllIRcCXorBgm4na7XWitiHCrmehrAHoZC58/YqGbpOghWdHYRUnWJovugS4sLs1DnwwHQJTji6JxPz3Tuj2pClGSDcnp6spiVD23z7Btgy8Mry9kCta37zmGeXfkWJaLSCJwoOYcymfkYmA/p6ZxqZ5fnZj6/BGqbmokGtVYJsvKCsBDmRCputEA6Bp7UWWoShKsuh6yDDT7KU0IHR3XhLCrJliiGH+TiZ4AlmEwa+IQTCkYrJWB7Tpwkqrv1l3BMgRvHfxCW6wCGLzyZ6Nm3Ct/VhJuJ842wW5jFUkiGThT58OmN49oCQZJkpHlcsDltONrYwZYlnpblVgTWTaVV7Msg4w0waIUSSkfUscDSzsWZfzPrs8wOX8Q7AIQicrY97czmFYyFFFC4tcBjVOjL4mri1ipA68vCDvP4b92fBJj10oiOBKR0BQ1ajun2HgU5mRj+vihpuaXdxbfhPMNQby2+x+Yd0cObAKH5eWF6JPpMNmfXeCoPm/mxJsBXHqxz8qmaU2KVTtM6tX2DFg1oWRlWXlfLIMV696Hy2k3xHFZLgfKpuWAZRmTXSm+FABhwLEEsyYOwa039cHg/k6cOd9MjS17pduxftsnaPCFMXvSLeibaYcM4P+90Nawc3FpHgDgseffo8a0UcmcPNDrSgNA7ww7GEKXwhFFqbWfSTFSefaKdZ6tIPAMlpcXwi5wmq671xek6qeqSERzNYkkkug+SOTbtafwCIQkra+CIpMRiT1kwrrOC2bkoCCnv5ZIPFBz7or0ohM5f1iUsTkmqbx551FUzMylJHXjn5/Fq6ucyPMXJdkw1qn+mTYPjc3PqOenbSuDwNsUMulV9+udgu4KjmMwYkhvPPb8PsMYzHEMZArT/lrE9a6DLUrQ5t76uXhHE/Y661k+CWA3gH5ut3szgKkAKjrpXF0KfeB+oOYc5tzuNqwSVJUVQGAJJDCItiZFVEN2D3ShdOowSLKMVQ8U4cXXD6PBFwbQFnAX5mRjwcxcw37OFF5zjq50O9Wp8hwDh53TGCsupx0ZaTatQYva8KV06jDLYD0iihTmcj58LWE0NIcwpXAQNr35Kf5j8jDqx9k7w461Wz/Bknn5eGrTQazecEBjzQDA2qopEHglmR2SJLAxTRmuVGMviY6FaNGEkNaZ+HLRVROsa03Xyi4wmFI4CHUXWzRm7ZTCQVQtuO4KgSUonTpcYz5wLKEGoaVTh4PnCFhCIEoyAqGoxq5TEZUkLJqTh0hUtGxmun7bJ1hcmmeSPLKyNRvPwOW00bXpWjXcBJZg+YKxlnbMtd6juppemJON0qnDDTr78egC2vi2Z6Xq/99318ge9b6vB+ilDiRJ0UEUZZlq1/dOHwmWIYiKEpaXF2LLzs/gOeVFOBJF2bSRWL6Wvsi7ftsnqCorwK4DJ7F193FkuRxY9UCRyf7UhGGsTaan2lBdWdzuJItlGcggpvhBWeRp+ybaYh5AQlKvtqcgtjk2yxAILEFYlCESArQ2c1XtTsVvl0xCSzACX0uYaltnzvs0dn1VWQHe/eg0BmSnadJI+tiyan4BXvmzB3ffPgzBkKg1nKU1ol15f5Em1aGHQpAwJ5ZdTjuG3JiO5x6dBH8wgkyngFBYol6zJAOPqUznS8Sgl7vAwrIMGnxhw2L+4tI8uJy2dhnWVgsFHcXKTiKJJDoHNp6uwWzjGYR1PsOewqPWGzRtl+2ym5LQNp5YHNOsTeuwMRgxpI8hkdieBjTtuLEa0IloUBMC0yL6ojl5IJQy/kTmZ/HqKnMAHru3wNAL4LF76b7TxrOo+PeRaGpW8jE8R1Dx7yNh41mt4XjbtbYzZ40ZCkRR6vQ5dEdDtmB4xzLMr2VYE7y6r3Z3R0KS6HOWBTPk7p+A9ng8291u9xEAt0OpxFjp8XiOXGK3HgF94C4DprLr6lZGEQC8+PphLeimyVasWDAWNoFDfUNA6wo+OX+QptWsOnmGgbavy2k3lUUuu7cAmWk2PPrrPdrvDhuPFeuMLMDte46hX+9US1YTz7JISwGqK0sgyTJEUcZLbxzG/ppaLWBeOPurEEWZ6oAznTZ4fUF8dLQWqx4ogtenyH+8tvsfmDt1OBw8wQVfODlB7SGwZHXKHbcK2FUTLIdF4OQQGASjPW9EiYiyaTIeDEcRsXccO72zERZlLaHqTOHBcfTA0mHjUL3BWMY3a+IQbN19XNuOYxhs33MM8+/Modrs4P5OrKwoQiAUibvEOhSR8af3TphKkR67twAslOSECMDXEsHf/llHtS+BZwylhempNoPcwqVYqG3XopQoxgb0KxaMhZ3nEJWkJOO0m0AUlSaV3mBESxrT7BqAYXKoys00NIeQLtPZmmoFVHXrcbfuPo46bwAvvn7YZKfOVMHULX7p/Hxs2lFjGNMFnoGkm73Smic9Ou82hMMSemfYwbMMfvrQeIQjIliGgcACkbAImWW6RE4piY6BPtnKwhibrV5YbDnJ/+mmD+By2g0J5cKcbNw7bSR8gTCWlRXg1bc/R/WGA1hePg5fXmihVsW50m24e8owcCyDpRv3GuxbjzpvALIk4+G5o8HzbWOE5r9jEgLugS7Mu/NWLTZXWWcZTsH0jVSVFSAcjSLTaYN4CSFzXmARiEhaXPvWwS8wd+rwuOJXmh7ps1s+xprKEojtVLDQFgqSPj6JeBGOiOjb15nQPsFQFL6mwKU3TKJdhCJyXLISAQv9ZVqSMhSR8O5HxgZ4uw6cxLSSIaZtEzluSyg+CYxQIIKsTDuqK0s0f6Qkn81sbRmESgahyRjwHMHS+QVYs/GALlYpAM8RRKPmbWNzIItL88BzxEDCZliCtFTeICXBsIq+Lc3lRiKyqdqXBpaBKa76wbfGgGXM0iKiZFF1040llNqrSr5eJDjsNgYTRhsraavKCmC3MQj5e16+IFGoGtCx8R9t8ehK0KHfgNvtHh3zJ5Wm5nC73aM9Hs9HcRwjHcB7AKZ5PJ6Tbrf7JQAlAFpaN3nC4/Fs7bCLvgyogbtIiOWHCgD7a2rR4AujfEYuhtyYbkhWu5x2eH0hPLvlfUOSONXOYfXu4/D5I1hZUQSGUSaEVf+5V2OkbHrzCBbOHoV+vVPx5YUWZKQJCEdE7feWQNS0gvWrVz7G8vJxqG8IUA3L5bTh6d9/BM8pL9ZWTQbDEPw4hoH17JaPsbqyGIRA04PWDwChcBQ/XzweFxtDePH1w5icPwhZvRxYMDMXsqw00kpOUHsOYid2gI792UHzn66aYPlDIjXAWjBzVKd1Yu1MyJARDIkmRpVs6gPdfSHqVlndA114dN4YU2C5dH4+Xtp+2OQzVj1QBAC49aY+yEgT4LCzmDnxZtRebKHabCQq48GfvQ33QJdmA31dDk0KiH59ErbuPo5PT3gNdpOWwmvSQNHWKgG1GYvZvnINK8nPfH+CqczPc8p7yYBUlGRMzh9kCOhpY0hyQa97QK+f/Orbn5vYn0vmme16zcaDWPVAEcJREQLHWrKX3QNd8JzywpnStti0v6YW5TNzsXD2KKSlCHA5bQCA2ot+PHT3v4BjGWQ6bdjwRo1mi+qYXv1gCaCrRorVfnY57QiFRVPPB8XORERa4/GkXm3PRew7lyHTJ/mt+uR13oCWUO6baQchbfGiXvaIZYAtOz/T4kW1Km7ZvQUgogROlg0l1WqDQ9rEZ8MbnwKAyX9DlA2L2KVTh5ni31/84UM8dPe/IDPd1po8UUrd3/7gFH7/fx6srZrcro2yLGMiTyyak2dZUh4Lq28jGod2erIyMInLhcCzmP7ItoT22f70DPg66XquJ8Srl5xIfw9RkrF193ED+QIA/q3opsvWdU7kWgElCU0A9O/rxPnzPssKPCsJUomSyAyGROzYd9yQWN/6zufKAmXscWUZzlTB0FfFmSpAko1zH5lh0NgQMOnY9sl0IJZyEpVkbH3nc0NsvvWdz6mSA0SUkZ7KG86fnsqDUKooLefQ3bjylrGo5mQ6cN7f3REMJzWgqRrQXPfWgH61nd9kAEPa29ntdhcCWAdgmO7PtwGY4PF4zl355bWPeMvr1O0gyYbSWQAozMkGyzAQpbay2tUbDmDtsikG55bq4AxBsstpRyQqIsXh0PZ74Km3AABPfa/E4Aw8p7x4Yv1+VFcW44n1+7G2ajIEnsXPF41Hip2HwDNwOe2Gfeq8ATS1hPDG3uOaRIZ+MgnIWomtev30wUORTWAYaA5Y3xCmurJEC9Jv/komBvd34kJjEI3NYfTOsFsMiDJYlmkrAU0yPboFZACP3DMGT/++baX3kXvGQCboUCfcFRMsSQY1wCqf0XMStnqIIvDBp1+amBB3lgztMQl1vWzF7Em3oL4hiL98YGR3ABJVa9Tnj2Di6AGGhMCS+beB4wQTk2LRnDxEWikPnlNeLQlSXVncrn+x0l0eeuMobRs1yHam8BYBfK52DDVRo5aYF+Zk4/tzR8MfjIBlGIVVbXE9LEOQ3csYFOp7A6jPpb0FvaQ+b9chVlNQEBg8UTEOLEMQicqwsmuvL4SMNBt8gbApAFw0Jw+bdtSgYlYuWJbAxrN4cmERGEIQDEfR4AshEpURjUogIDhV26QtUAFKTwjaOesu+vHM5o80Nr0oSVpFlueUN247o8nQqPHQ1WDoq/Ze5/UbEuxJmBGbIGUIwVsHT1ETAuo79pzy4tW3P8fi0jz4/GGDzfzqlY/x/dLREAQWj35nDEIR0TBhFzgGUQIQlgVHoB2TtlizdH4+XnrjsBZjt/nvEkXKjSHo7RS0ReyohUan3cah0RfGT17Zb/imFBslYEEs/SOtIacq7SRJuCQzTP02XE671liUYQCOZRAVJUMlQRJJJNHzEW+flkT6e7AMabfvVOz56VIRl68BDSiVIGEROFffApZlLf0WIaBeK1WCgyHw+cM4c75Z29bnD1PvS5Jk8ByDr2Q5NWZzVJIgScZ5aSQiYc/H/6SwxYdS42OaXAgNoijBaedgy07XFjKtnoEMmT6HRsdKGXQkWIbgh98eg5//V9s1//DbY8Ay5HrJP1trQF8nZApRRELfzuWiQ5+lx+O56QoPsQDAgwBeBgC3250CYCCA37nd7hsBbIXCgO7w74BWdkpjk9G2U3VFM52CSd9zcWke/vLBaTT7I1riQS0vUZPE7oEufOeOWw3OTz2m1xe01CFV2SLnG5S/x7JV9E26VP28Obe7ceTEBTxRUYSmFqWUcPPOo7h7ilvTJ7XxBOEIfUAiBFj72iGUTcvBkt8ozWDcA11aUC1DmXiPurkP8kf0w+O/bWPFPFFh1qnMcjlw8pwPbx38wqCVmmTzXX1wDAHHEcPEkeMIOEJ6HAvHSqO3p67qchzB+LyvGEqEls7PN5WhdWfo5VecKTyiooSv3zYg5p4KUJiTbUicZbkccKbwOHO+GQ/PHa0xiZ/a+AEWzh6FLTs/w8LZo9C/dyrONwSwfc8x3HfXSHPpNtqXeVF1l9vT2+NaA3cr5h7PtR1Dr2/qHujC9PFDNZ3fS/k7G89AlDjDOazK1a00fZP6vF0HgWcMslnz7rwVT238wMCAptl1Y3MYUmuT4qr5+VhdWawt4KqNNE+cbTJomr918BSKv3oDnA4ewVAUr+3+B747fSQy02wGFquVJrTPH7GsyHp5x5G47SxWTuly9c47Akl7Twyx46MkyyZfrMi1sNrkVLXrWKk31U57Z9pxrr4F4YhoWAgBFLv70XduQyQioXemHSsrijS5t+17jmHl/UVoCUSQkSbgzb3HMX38UJw422Tww2tfO6RJyajvFgyxbHiYaufxs5c/MCWRVz1QBIFtXx6uvYacMhQCRXvgAKxYMLb1G2vzCUv/c6/hnvo4hWQSOokkrgE4LMr4YzWYU2wMNc5MsTEIxJT7Cyw9JhVYolUiqUhEr9lhoQEde628wKLeFzZtR/NbAs+aemQtnZ8PgWcQFS//vniORWNzBGs2GucJqZk8ojo5Ras5EkebI8mgy4VUmnWPWZaBLxhF7UW/Ni/O7pWCVJ41xRasxRy6OydzBY4g1WFkeKc6eAgcQTB8ta+ua2CpSd5D8wWJgjBAQU5/w7fz8NzRIAw6VBuVyHLHMwDdbncagDUAbgXwTQDVAB7xeDzNce5/EsDXADAAngZQCaARwBsANns8nnVxHGYwgBMXLjRDki59jzLLoEonkQFAY8kRnVOx2m51ZTEIiDbZMh6jhPr3hbNH4Yn1+7GsrMDQeKXtmCW40OiHwLHwh6Ja4Fo6dRj6907FxaYQBIFBIBjFr//7b5bHz3IpDV9c6TYwDEEwFNUSw/rtl5ePUxhWM3PpAfm9Bdj8f0exv6YWP7l/HH7933+Dy2mnJs/7Zqbg8d8an1NhTjbunuI2sK/VCcvsSbdQn0Hs8+/bWvYDXB1GX9++zq5YuByMBGy308Cx2gRJRZbLoTQj6Gm6yTyLi01B+FrChrKtXul2mCKcHgCZYzVZHhVZLodSImTxbrqj7arfsAyg7mIAv9zykemeVj1QhDPnm7X3lum0w8YzWLHOnCz7fmkeGppD2vv9n12f4a4JQzEgOw2RiJSQr4hnTGBZBg3+CP6w8yi+OWWYwb7S0wT0Tnfg0V/vUUq4KotR9ZyyaGfl82P9Xdu1sFj72iEDS2N5eSE1sUM7RrzjWzzQ++CuQBfZLdCRfpdl8cJrhzA5fxAG9nPixy+Yx9tVDxQZFmjVPg2T8wdpLM+V9xfhgTVvmQ6v2hJtTFXLTEVJQt1FPzLS7GAYAlmWIUsyzl1owZadn8HrC15y/C2fkQueI3HbmXFMZiziocRtLlF0pL1fCbqjz6UhNmG/emEx1Rf/5P4ipKVwCIYkiJKEM+ebDRWAqs2s3/YJViwYhxXr/oqH547W/J4K90AXFs4eZaxgmZcPhgANzSGk2Dmk2Hn4g1GkpwgQBFapHohIYBiiJZ/111ZdWQwOQEtE1BK9etZZr3Q7FqzeZbr3dVVTwDBo116s7OmJiiJseOMwKmbmIqtXavt+kWWxtPV7sPb/JSDdZAW5p9huPEhkzOrb15mQbMX2p2dclsxFd97n/Hlfl4/z8SKe6+oOtiuzrGU+QP+NJzJWJbrt+4fPoSCnv8YUPlBzDmNH9qduu/a1T0xs5YqZuTH5j/juCUBi80edbzRsW1liYmDHO/eRORZrtx4y39OsUaY5ksgw9LFh2RSwkvFZMQKLM+dbTIS/G/umGnppqPelxoH6a7h/5ijTfQHdxG45Fm/sOWZi7k8bP9RybnnNgWdx9nyLiRF/Q9/UHpkvSBiXRlCUUQAAIABJREFUkfu5HNvtLDb5rwCcA5ANIAggHcBaAN9K5CAej+c4gFnqv91u968BzIMi0xEXevdOi2u7Oq+fynAAIYYmDlbbqWUlVkwh2t/7907V2Hy03yVJRmNzBOu3fWRoLqgvL3947mikOej7Z/dK1brd//aPh/DovNvQO8OBugj9HvyhCPbX1KJi1ijIDIPNMZqmALSg/7/+dFQpbY+KppVDVSs69hz7a2ox91+HY82DJYhKMk6ebdLYMq50W1zPH1ACEEmS8cWXTVj1u7Zyyse/W4hB/dIVVus1gHhtt7Nw7kKLpT33S7CxydVGfaMf0ai5yQQhBH162L0AwLn6Zvq7EWX07wb3k6jtSpJs6Sclyfjels7Px8Y3PzWxFRbOHoXTdc1YveEAgLZFtV+98jEenXcbsnqlJnRN8Y4JmZkyHpj9VTT4QobrfHjuaMM96VnSVj6f5u8ApdxR31PAmcJDkmVTA8/Hv1uIPpkpJh8Y773Ei8vZp6egI/zuhcaAtljw8NzR9GcPGJr1bt9zDHNud2PHvhPaNpwFE0MdjyfnD9KSz+o+azYexOrKEhACLPnNPmqF1bKyAoQiUfzu9U81PWnaNQ7MTsOFpqCp54OVnRn371ibSwRX89xXC1dqt5mZMn6+eAIiUclSvzMiSqjzBg1MNT3rWWUFV80vQCgcRZ03QK0OKZ06zCRp8dSmgyifkatJ1rUEIobSZTW+q28MUKVkQAh6uVKQKclIsfNYeX8RJEnGhcYAfvd6DWZPuoX6LdlsLCJResmtai+SJOPx7xYa4s0l8/Lx6tufaTEz0L5f1NukZVWB3D3G765GV8S61+p33xlQn1V3fWbd6bqsbNcyRpeM33giY1Wi267bVoN122oMfx+bewN1W5qEXMWsUYZtz9VbzAkpfstq26hk3vZLq7mmbJ5rxjv3qfO2WMpqxN7/ea/fkvHat7f5WcVKkqm9NLJijquPA/XXwLAEvXtdXRu2stsLjX4qc5xlCHp3o++uM3He68f2PccM+a/te47h/pjv4VpFIt/ulaCzEtB5Ho/nu263+w6Px+N3u933ADic6EHcbncugGEej0fVliYAzO1W20HcK+sWEz3IsnG11WI7WZZBYO7GXTp1GBgG1H0uNoVQPiMXvdLtdOfHEmSkCajzWjcXfGbzR1hePo66/z9bkzFT8gdgcWmeomtz0W/oJK7KZ2SkCejltGPBjBxAVjohT84fpGn6uQe6sGhOnraf55RXYxzSk0Yw6WNnuRzITBMUbVdZxley0jAgOw2eU16k2HiL5w/D81dXv2WW0SYD6jlX/W5/pzOcutL5XG0GNMuyVA0vhpBuyYxoFxyrdS0G2hoTrXmwpOfdC5R3YxUwWd1Pd7ddzqLx2rkLLQY93cbmEO6+fZjBLl99+3Pc0CcNv/hDW59bZWEQ+O5dObDxDOou+jXNNhtPIEak9iso4h0ToDBDYptWPLP5I1RXlmg6gDaB0RLGVpIdtGMDbe9b1bBWt69+sNjQVMvGE1y4QCk0SuBeLoWrwIDusnMBHeN3ZZbVJh1W7zoqyvjYU4uCnP5wOW0on5GLAzUKY2nXwdNaDLCiYixqLyjlnjaBRUaqDRFRwn/+aBIk2apXgwxAxnOPToLPH9YSeervaiNPry8IAJbyHKdqlRiiMCe71c7kVu1agoYGv6mKwObgEQxLmj12lM0ljA609ytBd/e5gLmSTGAJwjJdssohcFi5fq/mj/tm2mETODxyz2hEojJ2HTiJ9FQBF5uCCEdEZLkcVF3nr2Q7qc1YnSk8CnOywRCY/Omq3+1H9YMlEEUZLyydjPrGgCYzp8aK5+qbFT1nlhiqDtwDXchI400LdsvKCiC3MpouZS8Zdg7VlSWobwygsTmMtVs/0Z1bhiTJqG/wU8cTRaKj7ZlGRYl6Po4hqLvYYnmMrqz46wm2Gy8SZUBf77gGGNBddDXWtht3jM7StaKpY1Ui41onbGs1J2Qpc0LL+6dsS1gGC2bkmNjahCD+45qeK2spqxF7TEbgqLrHDGU+JRJimQCnzQvivQage9itzLGabArQRmqo7qFz5csBI3C4a8JQE8udZg/XIhiLb6y93M/l2G5nJaBjOdosLk85hQD4pdvtfhtAM4AKABuv8NqoiNUvVIPTWJ1QgSUmraSl8/OxZacHPn8YVWUF2LLzKKaVDEGfTAe+vNCCLTs9po7iKgPJmcKjoTloOvdjZQVwpdkg6gJVK9YErWP50vn5eOXPHkzJH4A7im8y6PRVlRXg8fsK8fs/HTGtzsVq6y2ak4d3PjyNr40ZgE07agwTCa8vCNaiy+vJc02aRqWqZb1qYRG8TWFUbzQ+OwCIiKJpkrJoTh5A6EGpFVvyehGJ7wrYLLTJbDYGYX/PKkNpryNzT2nap0eqjaH6q1QbA38PezeAOrmWTT6gan4B3tx33MTgrCorwFsHv9D81OJShdmgLnYBih+qvdiCf5xugMCzJjtOT+NR9Zt9huen14iNd0wAFM1Uqm+WZYMOYGFONlY9UASOo78/K11qG28ed6rKCiBKMh5/fp/hbzQ9PhvPYPmCsajT6dZl9UqBjWcQTmridjgkXRNfWgJu0Zw8cCxw60298Zju/S2dnw9b6wLx0vkF4DmCRp+E5189pOnG6jUVl5XRNdLPnPdp8lsr7y+i2iYhRGvcJvAMHisrwJOU3hZZLgdKpw6HKEsGyRC1aaHaYNBhY1GrY8cW5mSbbDYe7fWOQCLf7vWMWOkNVbd7y86jJptdXKrEY6rs2vY9xzB9/FBU6+yxqqwAIMBfPjiNbxQN1uLSl3cc0XT5fYEImluMPVFU+RmeYzDn9uG40Bik2mx9QwBLfrNXu54Fs0bif3Z9htKpww1x62NlBVixYCxWrHtf+26e/r1SSbhw9ijc0CcNPNvW+JVjL+2P1e1sPGu49mWtWqVn65vx5YUWky4oAHj9EWxufabb9xyDw85R5wTBcNQgLaVpW7ceI6lpnkQSPQPx6irbBZaqFW0XWIQCMQu8PP2YtDgukW3tAmuhF228BofFnDD2npTz02NWfQ+VtuOyGDGkjyEWUo7LIuQ3XivPM6YGzUvn54OP0ZZuLyY3123JSLFzBt3jFDsHRRzQCN4iWc+zBHLUuK3VNUiyfMnGtVcL7VWidtdr7miEIyI2vXnEsEi+6c0jeOSeMddFfslu4bvsHZz76axn+a7b7X4KgMPtdv8rgIcA/CXRg3g8nkNut7sawD4APIBXPR7P5o69VAWiKMGVwmsTMiuGQViUsSVGmuKVP3s03cYb+qRi7tThhkBx0RylEeHC2aNwY18nWIqGnZ5hxDEELGTwPIsUG3tJ1pxD4NCIEB79zm1wpgqQZaAlGMaMCTcju1eKQbOpzhtA9YYDWFlRhHl35GiDnv638hm52F9Tq63WLS8fp22nloBnpAnok+GAwMI0cVVLMuu8bXIcBEqnWjX5rJ5PKRcuRigsYfueo6aSh4qZudT3ZdVUjtY1N4nLQygsmVhI1RsOKFpbV/naEgUhdHuhdWTuCQhGZciAIWCSW//eExEFWn2AUvb0law01F5sgT8UQfFXbzCxCGL91LNbPsaTlcXaO85yKRIrL22vwQ+/PUYLbPX7P7mw2PC31RsOKBUUrdcUOybYbRzkiEid8DOEzqAjhBi+of01tThxtgnVlSVxjTcqQhEJ73502tCV2G5nUfUbs2+vrjR/n1EJ8LVETBI0qXb+8l5YEu2C0Y1ParXQwtmjkN0rFf+sa8bLO47gR98ZQ2eaVJagfEYuXvnzUVTMatPJLZ+Ra6qAUpnM+gZtauJY3eZcPb0pG0MIiCiCAyCFRWTq7JFjGTAMwSP3jAbLMOB5gh8+u0c7Bq1p4bLWxXe9rQPQMfS7pk8DYPx2QQggy1127p6EKGCQwpicP0jzV/pYr3eGHQqpnqB06jD86pWPUT4jl+qXVywYh4Kc/vifXZ/h/n8fhZX3F4Ehyhi8fpuiM6rXP1bjzJUVRbDbWDz66z0on5FLtdnG5rC2z7NbFNmlipmjDPF0nTeAJzccwJrKElRXFkMGsKxVN7XOG9AWZqorizV7iDf+t9wOwMWmoMG/Li7NQ0rfVIiirD3jBl8Yi+bk4Yn1fzVU9Si9DWxavwD1PtQxCTHviTZeJZFEEt0HgZA5Ztt14CSmlQw1fLNBq3kWJY4LRcz5hy07j6Ji5qgr2jYYluLaNhBK5Fqt7n+I+fwJHFeSZPA8gxULxoEhgCQDoiyZ2LxMAnM+UQRWrt9v2pZ2fsIQ0+Lh4tI8EIaY0tVW18B043mnVW6FuU4a8AEAyzDIdAqGv2U6hdb80lW6qC5EyMofzDL7jitBZyWglwBYCqVx4JMA/hfAqnh39ng8g3X//xyA5zr4+qgQRQkErQ9FlKl2JkoyVStp5sSbAQC33tTHFCiqwfoT6/djbdVk7Rh67K+pxX13SeBkGRDbbDwcjOomUsCSefmmJn4gMh57/j0Abc2JVKytmkxdzWIYoLElTP3NmcIb/s1zbSUn+hLwtVWTIYGA5xmN3XKqtlnTA1T3V8uBZYuVtago41evfIx5d95qYoRYMZaSDKfOhyjSy7tFsQeyzImZXdsew767IyoaAzagrRlqT2R0i5IMWZa0rtkqa+2DT7/EN8bdFJefkiXZ1G0aAOR2mBCxf4utoNCPCS6n3bL8yMofgVhVakggrUyM9sYb/fPZuvs4tu4+rv3tt0smWR479vuMihJVgqan2kt3R6w9eH1BZKTZsPHNGuyvqUVhTna7OrvqGHvfXW0+2FI3HATfLx2NXuk2cByDn738oaESYMvOz0y2SfN9hvgnKkJS/18UEZSMZaezJ91CTYari0IqjHFN+zbe0VDvRy3ZTsYFZsSynfQ2po/1qiuLwXEMMlJ59O+TpvlfamxJgIw0AdPHD0WjL4SHf/kuqiuLseGNT/HtfxuOrF4p1P18gTAYRpGco1UNLJ1fgBf+eMiwj13gIEoSVRM6Kil2Z1ky3Y6vb89WadtJYKi6oKsriw3fueeUF00tIS0Zrj5fwDpWF1sTK/HcQxJJJNE9IEqSKWYDgH8rusnodySLeRYljlN9Xay/u++urtk2kWsFAcbc2s/A7FbijtgNEztuJCrhN6/8HbMn3WKQcPrht8cYGbqWcz7z+SWL89NYv5bs2G9T2LEJXEP3QU+85o4FzxMq058XGEQD134kKYpW/qBj441OiV08Hk/E7Xbv9ng8P3G73b0ATPB4PMHOOFdHQq+xJvAMJBFaeanKhmAZQtVAUpsC9c6wWyZLFIauMt2ns3cZQBS166jz+kFYFjILiBFFU/G/d3lMLOHyGQpLuDAnG+mpNlRXFiMqShB4xlKHkWUYOFME6m/qvajHpF3vrIlDwDIMQpKE2ot+bNn5GWZPuoXa2ZttbVQUFWWL0hUGj9wzGgLPYE1liemZ0xAvYyWJywfLMnS9L5Yo1KmeBJlQmwpUzBx1ta/ssiBJ9AmpJKFHJhRZhiBKGOz5+FTrpB3w+gKYOnYwALq/1PspRXagGU+s3w+gTX9/cWkeWIahyhTEMiGupILCyh9FLbRUVV8fL1iGmHQCec7i+6Qcuz17ISwTl+/Uj0uIc5/rFW320KYZu2PfCUzOH4S7p7iRmWZDqFUjN9Y2xFb7U+1E3UZfAaXv3aBo3gKNLSH0Sndous4qvL4gMpwClpePgz8UgbcphO17jmHh7FEQWSZOvVliOPdXstKo9pSRZmSNXI6tXw10tb5ud0Es28mqyi4YjiI7LRUnzzUjq5dD87+0bSUZSHMI2LSjBt8YNxjugS6kp9rw3btykOYQcPa8kZGv+uo0O2/w1S/vOKIxsJ0pAnz+kEliKRiOthPjKr78SqrlLmUX6u+WEl8yABBDPxSr59ZerC6DHjtb3cP1as9JJNFdoPqyS8Vn7X33seNmotv+6J7RGD64t9aT4ejJC5bbdvS1JjLnsmq2zLEMEDUelyEEXl/QsHhHYxXLMqjnXzAz15RLZSxZvzAx2liGoZ6fpbGaZYIPj3xJZcF3W1xjc+XLQTQqUxn5ax4sucpX1jWw9gcdywDvlAS02+1+EkARgK8DSAGw1O12j/R4PHGzoLsaei08lX0Xy8Z1pfBgebMG0tL5+dix70RrApie1A2Go5r+EQBLbSQRjEnrTS2pnZB3A3VV5p0PT6MwJxvz7hyBuouKBl1UlJDi4HD8jJeuHQoJz2752LTStaysAJt3HtWuu2zaSLz4+mHDdrMmDsGE0QM0aQ91heydD09Tj6camcgRk3aTpgkVlCG1CkPFwwgE4mesJIPxy4NdoOt92QUGoWj3TijEwmFjWr8PPwCA5wjm3TmCql3WEyBw9IBJ4AikhNq0dg9wACAwKP6XG3G61oe0FAEZqXa0BCP403snqH5ji85PLSsrwPOvKuw490AXVTMagKYRWlVWAIZtS2x3RAUFzR8JFtp6AgtEEjiRjWdMOoErFow1yT1p32fMKj1vodXfHAhjzcaDl9QTjdWK1Y+JSV9Kh14z9q2DX2By/iD0zrC39n0IWdr11nc+157vG3uP4eG5o/HM5o/w6tufY3FpHl5/9xi1s3p27xQwrDm2WDo/H7//01Ec+kc9Fs3Jw1sHv8D8O0fAH4yiVqcJHqtZq3/XKxaMxWNlBfjDzqOYPn4oai+2UO3J5bQZvqmqHqBLfz3btp6p73La0SvdhpX3F+FcfTO27PwMXl8QS+fng+MY2AQGbx38At+cMkyLE02a/WUFYFngf987iW9NHQ6Hg8O8O2/V/NbTiydgy87PNJumxdp6X632EeE4ApfTjqe+V4LG5jDeOvgF7powFC6nDQLbfjXc5VbLXcou9L9bSYacqWvTYlfj+LcOfmH6RheX5mHzzqOm0m61J0uDL0zVjKbdw/Vsz0kk0V1gs2BRxmogp1jorabYGARixk27YKHNSon5UmwMbshyGubpVseNV9s5Xl1rACAMqHEKoSZ16bIWLENM/o1YsIpj878Cz2gVlfpYSOAZRGP8IAdgxYKxpniI5l8JkfHovNvgawlr2zpTBdP51fdF1/c2v6/ugkR0vq9VtNdj7HrQwe6q/l8ktgy5I+B2uw8DyPN4PJHWf9sAfOjxeEZ2+MmsMRjAiXi7K8ssg6pWnbhlZQVUJq+ixUYMmsrqb6sriyHLwMWmAGSZaOXO6ovLSBOw/d1jmDFxKERRhiQrK3kgMiCT1iRs1HAd+uOXz8jF6g0HsGBGDsaOvEHrRh8IR9HcEkHvDAfON/hNDrxvZgr+9N5xU4fdaSVDUfWc0s1cLWUJhqPISLOhviGorXr0zrDjkWff1RhXzhQefTIdBk1V/TW++vbnKJ06rFXrGiZWFcOzCEck5UNmCWycct/xItHOzB0djPft6+yKQpTBSMB2Owsyx6LqP822Xv1gCUgPS0AzAov6hiB++vIHmh08+p3b0CfTri189CQwAof6hgDlfhyQwvTvqbvbLiNwOHO+2eDDVj1QhMd/+x5G3dwHs752C1hG0XwjDCCJ0P5t4y/tv1VmNSHAgZpzKP7qDRBFOe6FqcvtCs8LLMIiNBaKwMLUJPBSkFnWNO4sLy/Eoc/PU307iWGjMAKLLy8EDOPSD741BqIk4bHn39PGN2Jx/1bjUnv7dBS6yG6BTvK7vMDigi9sGIOeqCjC8rVmu7bbWdTW+xWWZC8Hvv+L3XhyYRFcTjt8/jAYhqCX006NQaofLAEnyyA8g2Z/FCxLIIoy/viXz7Hr4Om27SpLwLEE/4z51haX5uHGVs1a2rv+6UMlEEVg7WuHcNeEoXDYeKzZaBxXQWT4WiLgWEZjTVTMHGWyx67Epb7bzrTt7u5zASVGklmChhgbXTq/AIFQBK+/e0zrJwIAvkAYGWkCTp5tQu9MpQeJJCl+NDWFw3P/83dUzBwFDjJEECzV2ep//miSpn/87X8bjn69U6mxpNoPhWGURIK3KYQnXzLaWq8MG+SIpCWD42EqJ0JCuJRd6H+nLXounV8AGRLCYUn7FlT2ncAShEUZogScOe/T2NEqG5zWK8b4GyzvoaPsuSfYbrxIZOzu29eJ6Y9si/vY25+ekdD2PWGf8+d9lx3vdDbiua7uYLvxfocyy2Lta4dMbEPauCmzLN7YeyyumI8WM6q+9XK3TfRa4z1/lBA8/fuPTLIaj9wzWpHvinmua1/7hHINuUb/xrJ4gXKt988cZWaAswwa/BFDL6vHygqQSckTWMXS/Xo7THPJRH1xd7BbRlCaL4q6OQvLAgCxnFtea0jEdq9JcCyWUnI/ax4sMVUkqLgc2+0s+TBeTT63IoxuLl+uX/Gw0rYTW7WMab+db/2bXWAhykYtUlGS0OgLweePIByR4PWFNBbH3KnD4UrhEQ6KpuvQH1/VO123rQb5I/oBAMqffEvb5rdLJ1tq0Kk6VGoSuSCnPxgWWHZvAVa/dACrNxxoY0ZLkqHD9xMVRchyOQx6gFbao84UHl5fEC6nHTZGCY71piqKUozOJNDZ/iy2yU6dN9m8JV5YlpX2wFXAqChryVpAuY+fvvxBD9bAlWG3sQY/Y7exoHVt7imIipLJh734+mGNvbbr4GmN7aDXmQeAZ74/AUvnF2DNxgOW/vtCYxDhiIhASMSQGzMRFWXwAEgn69NGwqKOGS0mxHxWQdPIy3I5MD7vKwZ2xdL5+WBZJTmvRzgi4aXtNYayupe216Bs2ggAl9YTbY8RkNQgbR9hXRMyQHluG944rPVzMNj1FsWu1QVdQFmobmwOab0dfrZovKVfFiUJUUlG5U/fNvWDULcTJQkyCDVeUBN/6mKyvpdDOCKBZQmmjx+KZ7d8DJfTrvR+6JOGi41KAzavL4hFc/Kw4Y1PtX1pmpPdCde7bYuiBBmMyUbXbGzT9C6blgMQGaIoa4siAs9i1Yv7DQsYXl+b7jeRZURj9JfDkajGfv5/L/wVT32vxOLZt/VDiQBa8ln9XYvh4kyoxlstZ9jnEnah/91zyot3PjyNJyqK0NSixPev/Pko7powFJvePKJ9F4QAJCohIrZKahKiyUapx7HqFaP/jbSnUX2d23MSSXQHxPsdJqrVHI+utLqtpW+9zG0TvVbqMWWzH+IsZC04irSHwBJqNavAEkNsHU3gWqOAlnxWr/NJizyBJILaT2VNpVmeoWf6YhnNgQjOewPa3LKvy4E0x3XUsJyAroNNYe9fi4iKFv3aOthuO+sb2Od2u38P4EUoGZH5APa3v8vVhV4nzkqjjWEICOilzH1dDsgyIIoyfrz2PdPvqx4oxh3FN+Hx374Hl9OO0qnDUDYtB96mEFxOG1iWQSgigmUYk9bnrgMnIckylrUyqdUVKf11tJcsXF5eiH1/P4uvjRlg+KBWLBiL75eOBiGK9p8/qLBd9EmKne+fMJUvchxdr6mvK0VpVoPuI3HRMweAS0OwcwhFZG2FUmXQdyRIAl2Euztkma6B2wkFIF0CUQRe3nEEk/MHwS4AkaiMl3ccoXa47m6IZQTbeIJQRKL6sP01tSifkYtVDxSBEIJIVMLGN2sAKGXVauVGOCrCJrBY82AJJJmul9nYHMbAfk78+IX3OqQaIhbxsOxUtmFUlCFJMjiGAQv50vrLFE0uh53HE+v3GgLhNRsPUjt3swyhdnVWdbSzXO1rol6Jjur1DtoYtL+mFqW3uzWN294Zdqx77RMt+ayXwvL5I0hz8Nrz9zaFqO+CYwlkqe1d+fwRUyzxt89qwTKM5eSwvjGAJb/Zi8KcbHx/7mg0B8LwNoXa9N9kosUQdd6AJi9QPiNXSzirTZfVhW31fHxMX402FmjXS2MZv1W6Rvz1ZNvtER+yXA54m0LI7uXQeqAAQEaazbAAahNYrNt6WIuVZRi1wwHAH4oaFk6t+pCwLAOGURbOYBnbAiysq9ycqTwgm5nC8bKhBZ7B8vJC2AVOY+R5fUFLbemCnP5YHhP7nzjbpH0Lv3pFWeDRX4dsMZ/gWAayrPRgafCFDVWKAs+0W7WV9NVJJHH1YdUvKvY77EwN6Hh7hMR73ESOaanrzBATHZFhgSXzb0NTc5usRXqaAIZCphAB+FoieP7VQwYGcqrdOKNPxA8mkieIWsROUUpivyf6YlGymFvO6v5zy44CAV0//P5Z14cONsPQ+y7F6qxfKTorB/cQgJUAnoGyuPTn1n93W+h14g7UnDNpMy4uzcPPXv4QI25yUfXbfvbyh/D6gvjJA0VU5wTIWLPxIFxOu6lUb1lZAViO4Dev/B0T8m4waQYtXzAW/kAUz25r2+exsgKsWDAWK9a9jzpvAP4gPWl+8pwP67d9gpUVRYbEeJ03gBXr3tdK3Ou8ARTmZJt0X5aVFaC3UzA017LxdB0oGwtEwlK3WiDqiQPApSDYOZxvDJmef98MW4cmoXmWrsvFs6S795QywbrJhDkY6hEgMlVfrbtHCLzAot4X1mxX73MeuvtfLL9VVgYYliASlVE6dTj8wYhJD1PgGHzxZRP+968nNXap/tls33MM3xg3mM6ku8L7ikfqh2UZtEREeC+GDNduVeqnB01PcFlZAVxOY9Nbq6DZYWOpml713hbtWO1pol6ujmoS1mPQ+YYg1m/7BIvm5OHNvccx/84cfHf6SPAsA54F5k4djhNnm3Cg5hxmfm2o5otp8cnS+fmISjIEVqnnWFZWgJrj9VT9wbWvHcK0kiGWizTugS5MHz9US6ap+wksQTBCn3ypFVr6f6s2cvTkBby+54RB69cq1ugKnVratxqrEX+92baVjQbDUSwuzYMr3QYOig9W35vLacf8O0egV7odZ+ubsW7rYXh9QUM8rGqHq+wyAoKnNrZVIrkHukwxxqI5eVi79RBmfe0WvLS9BqVTh1GvTYasJZNpVW7lM3KxftsnlrrN7dkdyzJo8IUNSY7FpXlwOW2W2tIZaUKvG3YpAAAgAElEQVS730adNwBJlg1J8807j+KRe8bg6d9/aDiPPxTBWwdOYd6dI+BriRjKvS81XiR9dRJJXH0ILF0DOpapm4iuciJ60SkWOq60bW08sexLFatXHe8xE9F1FiUZ4bBk8LcPzx0NUZJNsbkoylQGcizxQmDp9xT7/NVrjTdPkMi2PdIXE7p2d3efW3YkWJbgrglDzbbLEojXgQoJxzLUb5djiWlB6ErQKRrQAOB2uyd4PJ533W53LwATPB7Pa51yImsMRoLaYmowKwNY16ox9JWsNNRebNE02gDodJglRGM0FpeXF2pOVEVhTjYWzByF814/0lNt2LSjxsS2WVlRBEBZeZBloL4xgE1vKuW4tGNmuRxYU1kCGQqDSOBZk36qvlT9qe+VYMlv9prueV3VFDBMGxskXlYSTddUEmWNWcIxDBhWYbB0JLspqQHddfpEMseiJRDBhca2UpzeGQ6kOvgepwEtcyzO1beYBtX+fVJ73L0AiWmxqeiOtqvXa3YPdKFs2gg8s/mjtsn2vQXolSZoPoljGbAMwY9+vcdk/yqrruq5fZiSPwCzJw3TyqFVuaM39h7XfLWKtVWTTTpzNLTne+LRepNZBqdqfVRffqlycqvjL5w9ylDCbeUH2rMXQI7LP6vjIwgB5Pj26Qj0dA1oqzEoLYVH7UW/Ns6vrZqiSVep++njkWklQ9A7wwGBZ/Dm3uO49aY+hnf5jXGDMTDbCaJp4tL7VajyGrHfmhovzJ50i2UPDJYlOHG2ycQKXV4+Dk0tIYPWbTgiIRyNou5iEAAMx2yvz0ZHa4rHfrfW32pJazzTofHKVfe58cDKRjOcAggAl9OBCxeaTc/OPdCF+XeOQJ9MZTH3QqMfv3u9TX4ly6Vqh8utEnYwaX0eqDmH0qluXGgMorE5rEm/6G01tlGhuqBYMTMXoiSjovot0z2pEjQGu2JZgya1eo1mXVa6jeg1EFmWAVgGEVGp3mFZBmu3HjLF9vpqANU364+/tmoKzpz3Ub+pOm/LZY0XHdF4u6fYbjxIakAntk9SAzouDEYHaUAnouvcORrQ8ekqJ6rr/N+7PtN6XIgSsPWdz3H3lGEUXedEjsugonqX6XmvrZoCTjb68KMnL2L44N5anuLoyQsYPriXyXcmkidINKeQiC/uHnbL4v3DZ1GQ0x+yLIMQggM15zB25A3Xh/4x2tMkH2OwsWsVnJ1HU0vIJMOSnmpDNBih7tNtNKDdbveTAIoAfB1ACoClbrd7pMfjWdUZ5+soqDpxIiGadlB1ZbFhgu8e6IJ7UG9DZ9lFc/JwurYZnlNebNn5mWHFS2X6LIvZvsEX1pqOzJ50CxgGOHO+rfP44tI8LJg1Euu2HoZd4KxLPloTAeGIiE1vHkH5jFwMzE7Dqdpmg05qY3PYggEKEFHStPFUbbpLaeXF6ppKMDtlteu31xfUHDSAKw6ME4EoSnCl8AYGd3eSCLkcJKLtdSXgOAKbwCC7VyoYAkiyDSyr/L2nrQLKskwtqVGbAvU0EAb45pRh8LWEASg6aN+cMozaYbo7IdZ29XrNnlNebHjjU5TPyMXg/k5tQSy2gZsV8zczzQaB5/DU90rQ2BzG3r/9EyX/8hUAwOT8Qdi8U9HkVH010HHVEPGU8ImSbOnLLyUJZHX8G/qkaX5dY60IDMKxHbZjGPOFOdkomzZSC8wZ1lRBab6G1vFRnQB2YzPrVlDHoDWVJYiIEs7WNxv0kgHVDs3JZ7E1sfXNKcPw000fGGKIWB30mRNvhigBNpZR+i3E6O8CbYxM/bc2MDsNogRs2lEDzymvpYY6iBJHxLJCM502bHpTWVQvzMnGfXeNhM8fQYMviKxeKeibaQcAg+91pdssvwOWZTpVnsP6W1V1hztPD767wjJOCouQoVQQAcZnpzbe++UW4yKG+ps6efv/7L19nBTVne//Oae6unsGGmhhhpgHIRrTKMrNgDMYQd0rhvvaxMfl7jKajYxXIJH8IhvdjQzuXXFvZDC7ca/uXswi2Qx6N8HsJWrU3NdlhU0ibMKgshdFbbnGp6yEx2HoYaYf6uH3R3XV1GN3VU8/z/f9epkMXVWnTp36nO95qHO+X7OvfR4WHJPJa7s7oKhwLJAwa/WJF97EX371CgyeSSM1kjO0f8cNl3iuSNPdC+m6iggcadmr72T3y+qukZykIJrfZZBRVSgZGUOmj5zdS+cAGFtJr/eBjZVvIgPAkVFUxGPRvJ931TLG0BE4Sm8vSvB5TRBE+QjiAzqIX+fK+IBWPfwlX+LIq980wyLHf7zMugNrbXeHqwshxSNdt3hDguCxAllg2uRCHlHk+Hh7zDJX09vTBVHkkGx9CXP7V2yBRTPOKZgRwxyfS7Tjt8dSxuTj5xLtEMMckn1c0aRwzlx9kvM6H1+Xi3RGwjM/f8f08Sgy9vGojPeplAuOGwF0AEAymfxtIpG4GsArAOp6AlqnkD/or//hf8C3/2GfZfuH2efhYCqN1hYR3/ijz2HGNM0v9IbHf+V6/o7dhx3uOPSB5SPbD+DOZfOw7JoLkc5KRbd8CKYK47a6aNf+9w2/kks6Z2Hq5DDisYjrdhSjHAJ8uXPbBvnI9rFy2dg/gO98Y7Ej0no1tt02W2c8iB+w8aAqwOCZrEu038bznh0SOP7oC5/FmeGxCds/+sJnERI4lAbsOHDGkMnKjskgzlhdexSxa9duX5MfDGLrs68Zq0SyLsGxNvYPOFb+an5JmSMgX//zr1s61e9+dMa4tj2u+cFnYJAYG1dHMuRRJ81BVATOfNly93Jz73SfHLKuPt2+8y0PX21jvnu9XCzMiIWRK+BblCgdWVagChz/9e+tPmIffUpr5+OxqLEt0y0i+923LjA+utj7HMCYu4R/P55CPBZFvFV09S+7cO5MTJkUQd+aRZaV09t3vo2vfPEivPvRGc8YGFCZIxjcI9sP4E+652PfoaOGrnSXXrpN+tj0VpxOZV2DG9vvobsM6+3pwvadb1lcYpTNV3sTuuUqB376SeayW3bNhYZNAcb6tno7ZNnJYriNgGvwy//2tStc/S3rk8iDqTSOnBh27PbQbbZ9m7PejzafJwH46MSwr3fvpZGPTgzjvJkxCALD8JmsxQ3OXcs7sH3nW1jxpbm46erPQAxxxKdE8F9umIuh4azxATQei2Byq2hMxK+88VLXe8mKthuyWLtCEET94bed8fSVLHBjt4WRpse5gsu5gdL1mdcgY09Vcbf1D33dGazP20UiHBN+Ame4+9YFjnGpwK1jn5ykGu439Pv39Q+g7+vOGClAZRZYlHsHdjVQZNXDx/bECUIY8nA/Wm4XFPWKwDkO/r8Tlt3C7fEW3LJ0Tln7HbxsKVkRk8mkeZ12Fg3kaVXv0LbHW7Bj92Gs7e7QBmDwXpEQnxLB/SsXGqs0OGe4f8uvMDSccT1/6uSwZwd+2TUX4tjgKGaeMwnnzZyMT7bHsGH15UYe9A69eRqwUJ514cyYGsEtS+dg67Ov4d6/24M//96/4mQqqzVeNnTD2bt5L1b37ULv5r0YHMm5ngsUDmKj/y3JqutkUoMtpq05ur8u8/vV/XWVE1lx97UlV3CrZKUQQwwC43hsx0H0bt6Lx3YchMA4xFAjrn8GJFlx7dxJdT6BEhZg0e6u/e87tLzeZNs8V/62Tcb9Kxeib80i3L9yIVbddCn6tllty6Zt+3Hd4vMd137snEn4+3VL8FffuBK5nIJ1m/f4snGF4AIcNndtdwe4aflGCMDMc1od59ltuRtm+65fd9fyDmx74Q2cOZtB7+a92Ng/gH2HjkJ20YA5wKNbu9PXPwCae64sXlr+RFvMMiCRwRwR2R/+4Sv449+fY7lu6mQtqKSutdikMLbvfNtoUyMis2hG3431wNZfoXfzXmx99jV0L52DT7RPxmAqjSd/9ibuXDYPF3xiikNr63u6PFcp6XFJ3HT1yPYDUFWGH7+YtPze//zrrnresfuwocclnbOM88vZT3CrS+t91EHCWnZeK+XbprUYk8/6bw/2D0AG8wzgpKow2uatz76G2750EXp7urBj92HDRs48p9X1nZlXpD3eey2+/bUr8NxL71gCeoag1b/tO9/GXcs7HOmEBQZV4JCY9v9hgbnqc/vOtyErKiRZNSaf9Wd49KkDWNI5C0PDmj0+dSaD3v+xF//wUy1o7k1XfwbZnIzUSBa5nGq03zt2H3bkad2KTrw48B5mTm8p2q4QBFF/+G1ndF/J9joucOfYRI+/5Bz7uYzhA6TrN6/RsPv9o2Hn/XOyxy4Sl/5pCMB9tnS9+sWyoiAUYrhz2Tz0rVmEO5fN03blKs5VzV4r0MdDkLkRr/gE9Tzn0Uzj/lJhsoq2eAs2rPo8vnfvNdiw6vNoi7eA1fn4ulzofunt9bwlUt4p40r1ufcmEol/BPB9aC4MewA495jVKfYtFmGRY9OaxZAUxTM65NRJYfzVk2NbZNet0LaJe60mOmdKFPGY+zZUPYDP0VNnjZV663u6cM+X50NRgHRWgmhrcArlWV8lkskprsbQLQiXl+H0CthVbBtke7zFMglift5i2wkJK9m0hLapEZPPSq4FiyhjAEJAC/bg+r7kxntfoxnZVc8b1yyq2Fe4SqIocH03iqrW9fPksjJmxMIO7XptZ/OyKwywfKH/tkfw1xlTWyy/tcdbEBYFhKBCkhXHRF+pQQmzOcVwgaS7GXjihTdxz5fnj02mywomiQJa2yZh45pFUBQgxBkEqL5WQ8QmiYafa30bunmVoP582qoV67XmFSZeE0flduFDWPFeZWRdde81STfdpOX2eAtmTG3Blt5roagqTg6N4vGnXzdccsiKCjmnIBzmhmamTo44dmP19Q9g05rFjvrntsVUgnvfJ53V2h0vXeVyMq6/8gLD7RiguSj46s3zjHu8dyRlcSli/nit/7tc/YRm30JbScxlp8JdD4C76xcpX9Zu1xw5Mez4cLHp64txz5fnG+8HgOc701ev6QH+Vt90qeGeQz9PELjxoUW30+mshHgsgpOpjGOV2jlTI672VuDM82PS1MlhDOV3WcVaRdeg4+tWdAFs7PrkB4NGnmafO0ULuisA1y0+H4qsFm1XCIKoP/y6dciY3GcWq+MjaQkvv/E7PHjnIsM/789f+RBXz/+k49wg6fptEzOjObRPi+b779p50TBHZtTpF9avqwwdURzrq7jNcegwMPzTi29jSecsRMPaSud/evFtfPXmedb7e63WZuNbdBRkbsSvG5Z6opnG/eNhdFSy7EK8r6cLkdaJsQo8PZLDzLi1nmtBUd39P5dKpfT0DQB/CeBhaPX1RQAPVOheFcG8HVHA2Hg+JHDH9o91Kzrx/Z++blt9p20T11dcmDug997WieGRLMJiyHNAp/uO09PTo3qbt9zqBk8QOGQwZBQFnDOERQ4ma76TVM6gKCokzi2dXh0vYxjUcLptg3T4v/PcvjOxt76WgpwzdwxUyLnyl1/hbVllv11FURQV8z4zwxEQQ1Eqtw2kkoQ8Onchzup+r4liqeuads1+6M3S8ooibbe3XluVRXHM5oyt6lDzdwb+7CuXoTUiIpuTcPx0Gjt2Hy6pc2h2gWS+v922ybICyJrmOADI/qqSBGDL06/h+isvsGxt190q6fdbt6Iz7+POOyK510fRYi589MCzR06chSAICAsglx0B8BsR3Ws7qr5yyaJjDnz/mdexpHMWeq672HCrobsc2Pb8G8ZAjXOGeZ+Zga6551oCq5hjSZjrn90dQ0jgWH97Fzb+YCz/37xlPsJhbnxsdsv3iaFR/O2P/83Rf1H1e3LuGpBQ/7CSOC+O7qWfBaAFFrJ8oCox2FqzueWqJkbZCdxVz179Bs4ZwoKzDqxb0YW//8lBi9/o1EgOKlSEVBUCmOUdRzjTfJwXyx9gebfm+qcHBVzf0wUV7jvzNq1ZjHgs6ni+aFhAOqe4L0SZHDFWPKdGcuhe+lnHroBN2wbQt2ax5Xrd9dR3vnElFFk1gneLIvfVrgDlCTxIEER1EbhXHXf2xwTOMJLO5e2DVtdH0jl3exAgXcB/m5gZzYEBODfvqiLj4RfY3Oc0zwnYXWUAWv92w+O/dthTt0ldgTPccNUFrumac8I4HHMvdy3v8IyTo9vPY4MjgK2fYSknk/9+cz/KbdzQiO6+vN2hNN64v1QkwLE46cESFyc1KrKkFPx3OajUBPQ8ABcCOAVtHHMFgCSA8yp0v4ohhgWcSGUNX0Lt8Rbcv+py/En3fDCmrUaeMinscN5/bHAUH5s+ybK19eNtk8EAfP+nrxsBe3p7uixp65HHN/W/bAkw5LUiKOLiL3Jtdwfa4i2OLzj33taJhXNnOqJ0uzdewQyn/QtqKB/YyrqCRfU1ACcKUy2/UkLerYCjoW9AP0gtkRC+uOjTDh/BLREBuXTjDdJUBtzz5QX47j+OfQi758sLUO+tY1Dtuq3MYAwOe3tiaNRVq6LAHas6ADjycNfyDuza/z5u+9JFrkFSiuF3crFUlHyQmNOprKXjG5sk4r9cfwluv24uFFULEMrhtM85aWyFdtu0KO69rRMPPTHmv7S3p0ubUPbIrFs7SH6jg+F3lZGX/7lIWMCW3iWW68JhAd1L5zjeS1hgyCrOwJPLv5Cw+K31Cgrklf8praJFf/3PvwEA+G9fvQIj6RzWrei0pP/NW+aj//k3jNWhgNXtwcmU5hvXPlDUfUAnzos7gtaZAxo3mn/FZkKWFUyPhfHtr12BwVTG8HO8+uZLXfV7cmgEETGEabGwRUOjmRymxcIWrervcnos7B6EtoR37FX/Mjn3HQeS4ny+X7z6Ia6a/ylsd9Psii7kZBmDqTQAzb3U7ddf4rlTyd5efOu2y/C7kyOWxS339XRhw6rLjckZr3alEX2NEkQz47dO6m7pnG24sz8WETmumm8N7Ke74Mja6vmk/DZ6e7qTIhwjI5Xvs5n7nOYV2H/6xwscgQW9dkcrChzn+l7ZrcA18PxXb5oHO0HsZ0R0D6QbEQXIWevS7kqPCyqB6NH/FAU2YcIOBNFjM1Kt8V6lJqC3AngCwH8G8D0ANwHYUaF7VZSsDIcj+wce/7VlC/loxj2wFAPDn3TPx4xp2jZbBoZ1+YiswNgkir7MnXMYTs71Tqw5Pbet1m5fah7ZfgAbVn3e8ftDT+zHA6uvwLsfnSlqDEsxnNYvqDIU2fk1lba+jp+g7lFKhyEaESzboqKRxjS/2Zzi8Nm4adt+9K1xD0hR9yjAT/7lsKVz9ZN/OYzVN7kFoKsfStGufWWGKggOe/vTX76DFV+62KLVeCwCyIpjdbUqOAMb6kHdHtl+AJvWOIOkFKPS2/p5fvdI8oNByyrSO5fNQ3t8khGEcNf+97H6pksdZcmZdYV24rw47lw2D59oi0HIr0ws1LFwawf7+gcat/7UCD+rjJisIh6LOLTMZG2lsvm6rOwRaGfNIpgDTwLAks5ZDhsYVO+qCtfVyu//LoWN/QOGrj52ziR8eGwY/c+/Yfji1VyGjE2gZ00xIfQPK1MnhzFjagvCArD6pkuhAli/ea+rvQCq1Q4SXmRl1Qg6acZuiyNhAY8//ToGU2n0rVls0VDivDi+ect8/MWWf3V5l4vL+o7d6p9nYC/OHM+33jShY9esIDAAKh76+mLkFBWcMXDmvjOHM70vvBgnhkYxNJzFaFrC3/743xyrrtxc5Njbler1CQmC8IPfOunmls6rP5bJebX3zn7Y2YyC7TvfsowRtu98q2pjBM7gugLbxQW10b/1s+rW725DASpuXTrH4UZBgLPfFdSthlv8nU0uwRUb1d1Xs4z7SyWIHpuRao33KjUBrSaTyYcSicQMAG8B+CMAL1foXhVF9vDHmJNkfO2h3QC0DrR5snbh3Jnoue4SpEazGB7NYeb0FjBJgcScvvH2HTqKO27QB5aAImsdYvvkr74iCLAGCMh4fKnhzN1H7PBoNu9rLuZpDPWtKLFJWgdZ0SfHXc4NXJ609XXcVMuvlKwo4DZzw6EFe2i0QY1XPW5Yv7dMxU1Xf8bijuGbt8zX/DrUcaXyq92C24mZ6lh9dv2VF6A1GsJ5M2NFO3qFAqbqK99K0URFbRtTsW5FFzZts67afvJnb6LnuovRu3mvceodN1zizL+tzAZTaYghAYIAMEn2XPlsPJuieGw7bND6U0OKbZXXfYWPR8uyooLBeszLR3MQvbt9mNZ1CGiuBB7Yug+P3XuNMclodhnCTBPosqk/ZP6wsqV3CXKyCmY7x/58AHxvhSUKU7IrExf97Tt0FKtvvhSfmhnD8cFR5CTV4p9cUayrfwdTaTDu3l/1brfL944LLbaw96/Ndciu2fZzJuP48ZSRJuDel9+w6nIww2Ue8A8/PYTkB4PoW7PIu366uMgx04i+RgmimQlSJ3NZ2bJ4y6s/FmQcIysK9h066tgteMcN1emzhQBsWHU5jp4aMSYyZ57T6r6Ijan45i3zneMZ5h6w0M/iOFlWMM3n5G+QdyV5BDeUZPdybbQ5j0xOxpafvI5l11xo+Nje8pPXcc8fL5g4bUkAPTYj1ZovqZSeUvn/fwfAJclkcm8ikaj3eueKmyP7m68+H2JI2w7LGMPAoSOYmv+CmRrNQpZV3J9fzdEeb8F9t3dh6uQIBNndb5zAuSUQlf7V7K/XXoV0RsqvUGOegVXc0lQ8fKIOnslg67OvaV/2XPzo0Va++qdafqU0X13W9BSorj686h3PgBRF/N7WKwLnlgBj6ayEcJhDYPX9bvxot6gNUt231q2+6VJPX9J+8qD7sK1L/2yq9j/+gxCqjuvfPzJkCV4zcOgIzp3e6uv2YY9th2FRgJKt55je9YXf9tXvoKVgfYK1D+Dt+9upF93ft31FlnNFD8eWZw5a3IXpad6/8vNIZyVMmxxByCXQZshz5SlHWGTI5FRwqFg4dyaWdM4y6rru41oQmIcmg7vQmciMp8/npT+omjcofQBnPsa5cyccg5eO/ccN8TuJrp+nLazgkJiKabGwI2i3W/+6kP98RVGhCtxx/+n66kZVRTjEcXo4gw2P7xlbXLKiC5KiYOqkiK9ndXtOr4CQ9naVfEQTRHUIMk4LR0PI5NSiQeWDjGMqNebx6ysZAEIhjpnnTAJngKJGEAq5L10SOHMfz7iMNYOsKi5LP8rFt3YzjSXthAR33+GNEF+oXATRYzNSLY1XKv7WvkQi8RSA3QD+NJFIfBeucU/rH90/U3tci0B/89Xn46r5n0Lv5j1Y3bcL9z22FxefPwOvvnkUz+95B62RkOFbE8hvo/vBAN79aAjfefIVrO3uMNLSVw9teeYgBkdyEISx1yHL2oqzkKoF6MplZWMLLpMVw9iGANxnyp8+CAuFmON33c+p/rXQDa+tKA358poU/Quw+d0WeqelosjAQ9texgNb96F38148sHUfHtr2csP5fwac9djsZ60hUVXXdwO1ziZObfjRbjEbFAJwy9I52Prsa+jdvBdbn30NtyydE3gVZ1DbWEsiIkckLCAsCvibH72Kjf0DGEylsb6nC7v2vw+gsB1oiQi4+PwZuO+xvZZ2q8Xn1jpVdd92qNa53uqNcrevheqT/diu/e9j3YrOou2G7v9N7+P0bt6DE6ksxLyxlHW3NqqKsAB0L51jSXPdik78+MW38cDWX4FzhmiYOwaHYljAaD7Ysvnab94yH79+/SMcH8qgd/MebN+ZxPIvJCx1vXvpHITzrsrcNNmI7VMtGY8mg+jPslLNpCEmKxDy8UHs5+pBC4tpVp9E7928N6/ZvY4+tf28VXltHzkxgsd2HMSZkSwinDn61/Y65NaPiIgM7//ujOP+YljAyVQWW545iGOnRvDuR0NGEE+9rPu2DWB4JItTZ9L41m2XFXxWr+cMC6xgOfktH4IgykNE5B62wlrnwtGQ0d7p7e3xoQzCUWdPLhp2TzMadtbjIOf6xWxH7vj2Pxe0I0zkGB7JYcPjv8LXHtqNDY//CsMjOTDReW7Qsaa9/Rjvh7QgY+qmG0va0GOQOOaUhEbb91w6zTT3UQrV0jirxAAykUgwAAuTyeSvE4nElwBcC+B7yWQyWfabeTMbwLsnTw5DUcb3jPbVQL0mP86A9nIevHMRVm18EY/cfTXWPvwLRxp9axahd/NeI6K77iNxx+7Dho9EfVWyTls+ymwxBIFDBoNk8iPN8l/ttJUeWmRTMBVQC7vSkBjD6r5djt+39C7R3ITUGL9lUsH7V8MKz0YR7VZjNUu9ayEoRj1WVQisuN/bekZiHKv7XnT8vqX3WoRUdx00inb96G68+h9bBeffNtbS9qiCgN7NexCPRbHsmgsRaxWRzko4b2YMyLvFKVQO+vX2dqtvzWIwH1+0S9FbuaiSboEy9hm8qIRN1bUMxoD8Vn1dA3adC4K2gKGQW60gWlEFji3PvOZYobykcxY29g8UuM5dz2FRwLTJUSPI0vqeLlef0/rqJz9lWct6Wy82txB+NFmoDAvZ4iB22utcP2moAkevyVc4ANc+tdd5K2+81LIzsFC+IiK3rFYMC5ovbPf7L0bv5j1G+t+8Zb7FXZJO35pF+JsfvYpv/NHnMJqRDb/S9p0DhZ4zBHiWk9/yMdMI2vVLEBvQ1hbD9fc86zvt5757Y6DzG+Ga48dTNR9reeEnX/WgXVUQsOWZg462cfVN8yztYbD2VsDze97BtV2zIXBAVoAXB97DdYsvcG2bn9/zG5dzz/es88UIYkeCPFc9jDUL9aPseO0QGy/1oFuJcXz3H18x+mW6a7N7vryg4n39eqEe9Fhrgs6XlKLdiiz2SiaTKoBf5/9+AcALlbhPtTD7Z5I8fKPoE/lHT416bu8Gxnwk9q1ZZNniMB5/bebVGrofaR2GfNRO02+FzGS13DsQ46MafqWaTQt6PT4334Et5ve2nuHcI7hRnfuABopr14/uxupU5bsAACAASURBVKt//fogtrGW6D659NWJOt+79xpwzhzB6byuNxPEp1cj662eqIRN1bWsD8ztfhDNOldka5/A3Z9sED+TqqufyZuu/kyR69z13LdmEQSTP2Avv9X6JFsztU+1YrzlWMgWB7HTXuf6ScOvD89i/v/d+uD2+2dtwbZzMgr4Klcs6Xu58EiN5HBscBQhgWNj/68AaINdZhvsFvT5rqre74F8RBNEVfHrgzmoX+enf/EbPP2L31h+//0rPu3aNvs91y9B7EgQ//310JYX6kfZ8euzuxHh3CN45ATq69eDHmtNNeZLaP+VD8SwAFUQIDFm+EYxo/tGWd/ThTffPeHYyrG2uwM7dh+2nD91cgSJ8+K2NGq/xcHvVhRB4FAFDokxqAKnrXxNiB5E4v6VC9G3ZhHuX7kQG1ZdTgOWOiAkcI9tUo1fD0t1MdPMNsmr3VFU+Go3CrVbrufbyjIkcNx96wLLO7n71gWuemvm9zBeGqF9DaIVvaNuP1f/4O59nfUeifPiuH/lQi1WhsCxcO5MAGM+d515YQXL0lx+g6k0abAA5XLpVVvNuuvQbhsL6VU/39zfVwUBoWio6LN4359b0t+x+zDuWu50w7dj92GXejOWd71sAeD+lQs9xw72vOtuc/yWD0EQ5cFvOxqsvdXaxvU9Xehbswjre7qwcO7MQG3zeOp8kDS9n8t5brXcStaCRusPhwSOe1dcZhn337visqYYW/qlmfVYT1TEBUedMBtl2Nql+0Psy/vIW3XjXFx8/gzj37pvlJff+B3+eeAD9PZ0IZeTMHVyFIxpW1+zsoxvf3+fcf5dyzvw3Evv4IarLsATL7xp+PK0B32p1RaoYlsevYLWTIuFwWRnsKFyUuttYfWwRaZaCALH6ZEcHjS95/t6ujCtQQNSBtliVe+0too4k5YgSWo+wAcQCjFMiYYwYgpKZ6aRtBvUxUbQQFqluPCope2JtIg4djrtaHfiU8JgUnGbG20VMTicxTFTRPL2c1oRnxxG2qYXt7L86z+5EqmzOcf150wOY9R0fSWC2DaTCw6g9Pa1WBmWS5/mPk88FkX30s/i3BmTEQ5xCKpaNK9ruzuMfk1vTxfap0WRGbVqTPd7qd9j9R9cgjPDWYu2nnjhDZxOZR2BBnt7ujAjFkYuK7uWJYC6CaTcKDa3mCaLaavSwavLVWfcztP747csnYMZUyOGLvXj61Z0YnKriAhjBd2HDKUlfPsfxvr5G1Zdjmg4hJyk4ORQGuEwx3eeeBnxWBSrbroEsdYwPjoxjO0738ZgKu2oN2aNF6pj+nNygVnGKua6oshq4PfTKNr1A7ngCHYNueDwxWwU0K65jTPXx7apEUuAwWiriKODzr7dzHjU0Tfzm2bQc/1inw+xt8elnqufXwm3Fn6pxPgwaLtYD7ptbRXxu8G0I88fi0c9x5bNSDPNF5RC0OcvRbs0AV0ENz9Gq26ci8sv+bi27Y0BP3/lQ/zj/9HcWy+cOxO3LJ1jqbzfvGU+FFVFSOCY3BLGo08dMPw+b1yzyNjKYX+51e4A+J2U8fIDdeeyeYjHohUd6NW6U1QPDUS1KMVvYL3SbJPpPBzCidOj+M6TLxvP862vXIYZ01qgZN07l82s3SBaLfUDWq19QPvxJ+gFDws4OZQxAuS2x1tw722dmD41AsXWyXcry/tXLsRjOw4W9edXCZvRbBPQxSi1DP3o028bL4YFyGA4ncr4mtTT0gROnB6FChWcsYIaNfunbG0R8NHxs5ZJ5rXdHTh3xiTkJAWMMQyeSUNRVFOal3qWRT21W81ic4tpK2iZB/UN7Xdy2U+ahfz/S2Cufks3rPo8IiIvqJ/p0yfjxOkRyIqKsMgxlMpa+hvrb+/CtMkR5HIKOIfJl7QKVVVxZiQLSVIcGvcqW/vYoZjP1aAfXZtFuwBNQAe9hiagfTEbBX1Au8dHsLddQf06B/HBPJ4+oxt+n0nH76RypT5gBmkTKnH/oO1iPegWIQHr/oezHdn09cWANEF8cJioVztYSUqpD3XjA7qZcPNj9Pizh9B58ccAAKs2Wh2V68F3zFGu/+ZHr+L+lZ/Ho08dQM91FyP5wSAS58Wx7JoLoeQNY60JIjgvP1DRcAgb+wc041rNzBMVoZn8BspgxmAQ0J7jwf4BbFqzuMY5Kw1JVvBPu97GyhsvNTqC/7Trbay66dIJ6VcpiFYlwGGjN/YPVOUDWqnIioLTqazlt9OprG8fzpKsGpPPgPbMDz2xHxvXLHLoxa0so+EQ4rGoRW87dh923L+ZbEatqFQZBmnjc1kZqsBd68mmNYuhCtwY1AHQBlOMYf1jzgBrdp+X+jPq/im/t26JMfms3+eR7Qfwl1+9AoNn0uh//g0jIA6g6967LEiD1SdImQcd3HjZa3s/U5YVbZswZ5AVFeAMIYE7+68F/P97+XLmDEX1wzkDk5X8JIfq6G9s/MEA+tYsRphredB9SYMxfHWTM+DRHTdcoqXlUbaKoiKkmnxlF/ElW424IQRBaHjFR9Dr9dh5wfw6B/HB7McHdRD8PpOOX1/Jfm28jp+J5SDtTND7+6UR+yJSgTwLNcoTUV0qVR/s1GsdqDn6lzsoCjbeuciyquf44FkInENWFGz+1jX4yb8cxov7PwQATJ8ada28w6PaVlJFVZE4L46vfPEiPPrU2IqfWq/GDCI4LwfteiCVejauhH+ayRG/V/BQqUG1yjnD9VdeYLEhdy3v0FZ0TcCRZRCtNuIHtLAoOFwRrO3uQDQSQi6nFF1hohSYxGACt3Tg3cqSc7jePywKlhX3btfqPgolRbG4SQjqAmWiUCm7G7RT6VVPcrKC//r3/+oY1KlgWDh3pmN1lMA5YFtxZX5GL20yBkyf1oKv/cE89G0bG0TefesChEXuWLnvlraOufy83HZ46bEUdz0TjSCaLZcO7f3MYhMOYyufVXDOLSufx2wfd9WwogJRkUOWVV868MrziaFRRETBMglSrOxCeV+qzuMcAtSieXerfwRBVBaBe7WHVpsoFKjfbu2mnzT1dB9YtRCfbJ9i9A9/e+zMuOxBpfom3v1TOCY9/U4sB2lnKvbRvwHH0F555pwB1O2ZEFTrw8lEXCxXFN13Ue/mPVjdtwv/ffurAID+59/Arv3v4+LzZxjHNjz+K3xx0adxbeensHDuTMRaRVfH+0PDWTyy/QAYGJZdc6ExcQSMrcaUazjtUUhwdtwctJsDqdTDim5i/DSTI37uETzDIwZb3aOqcNiQR586gOb1qFSYIFotFIjKy+bVGkVRXVeJpjOS0Rb1bt6DE6msEXjKTEhwDwhz9NQIejfvxeBIzgiO4laW50xpcb2/fQtfWGCa39/8tQvnzkT30jmmPO7F2ZyM0yM59G7ea/xmvv9Ep1J2N0gbD3jXk49ODDsGdRKAaJije+kcbH32NfRu3outz76G7qVzEA0736v5GU8Ojbre5+TpNI6ePGtMPuv3e/iHr0ApMBAqFpxw0Ka9Qnp0O5+06iSIZsulQ3s/02vCQYL1va/K28ojJ0aw5ZnXLO8zIjJXDbe2CBhKZX3rwCvPQ8NZI09+yk4QONI5yRFw+K7lHdjyzEFfeY+I1B8niGoTFtzrY1hgtvNg6TPpvpJdunG+0wSASRGOKZOjlv7hlMlRTIqU3nZVqm/CPQIWcpe5hEJ23kyQdqZSQVobdQztFii37lblEBWjWkGL670e1ISsDMNxPjA22F5546UAnMc2bduPvvxW/i3PHMRdyzscKxOf/NmbODY4ihnTWjBjmvsq6Vquxgy0glBWEG8VsWnNYuRkBR+dGMaTPxsLiBLChFyE2XTo77lvzaKGd8QfEhjWdnc4VnCGBAalAcUqy423tauSmLVabIWa3im0B6J68mdv1u3qBEl2X8E/mMpY2qK+fm2bt72bwDlc9a9fZ14Z4laWXitUJNm6nTMrq9i+8y3DVceUSRE8sPVXljwePTVi8Sddqe1djUoQLQch6Goct3qyvqcLj+04aDlPtzvprOLoG3np0dq2APfd3oUHf2Bd5fyD5w5hbXeHh+4KuOCwlV80EoKak/N+cp1uRQrpEajOVsRGJ4hmy6VDez+z4IQDZ473+OhTWp/e/D4zOW8NO1xqFNBBoTbG3k4XKjtV4Njw+K8Rj0Vx/8rPY3g0i6HhLJ782ZtIfjCIdz864yPvpFWCqDZZWfVVH3NZGTNiYfStWVx0J5vfNAHgbMZ/e+yXio0Jmeo6bwLmbA9874gJ0M74bWOCUqm+XCVRVeC5l96xuNt77qV3sOqmedSOTBAqVR/c7kPY8PKlpvsgdDd+mkHZd+goTqeyrp1FzfgBKry2OKBmM7dBBacb0KjAcd7MGO758vyGMK5EMHS/gboj/gacqwUAMFlFPBbBncvmIRoOIZ2VEI9FwOpsotEvnp0rNnG7CH59XDbiBzSv9z00bPULrbdF9oY9m1PwxAtvWjqVT7zwJnquu9h0nXVSxFyWXPDaJmrtzNt9BPatWeRoL6PhEH08KUIl/LWW0sbbB0+CwDCYSlvO03VQzAet1zMCwLSWsftwzvBXT76C5AeDxs6VoH0lc/nFY1EjiIyXf/NCK6VIq/7wq9ly6NDV52eBCQevSYtYq2h5n96TG17adtfBWJ4X48TQqMs4wGY3PcpOz8+xwVGcOZtB72arj3V/eSetEkS1CVIf/fpKDuoDOkh77JeKjAk9Jj1X33Sp41S/E8tB2plKLrZqNN/7IYHhhqsuaJrFWkRwqrX4kPolLnj5ZEqN5Iy/ncaPA1DRHm9B8oNBPPrUAXzlixdh67OvGZX4vp4uCILWGV63ohObtu2vmwpe6pe6RjOuxMRElhVMEgWcNzPW8Ku5AUAUucOGrFvRCVHkkMjfY1Ea7QNaCMB9PV3GKrz2eAvW396FH/2ftyznFfIdOJhKY2P/gOVcc5tWaOW33868fXCQGsk52st0Vmo4v3jNQCltvL19Z+AOHd6nbykN4Muy0H1UjE1ynxwaLevOFbfBa0E9wqu/R1otlXLo0O3VF7JR8Ji0mDIpkvdRn/cP7jm54e8DnD3PIYEjIgqWcUCQj5zm/LjZUnMeGtHfKEE0K0H8NQdJ028dD+JbutaEANyydI6vyWLfO2ICtjPNsthqvHDOMHVy2LJYa+rkMDhn5AJ6AlGN+sDU5nUaOhvAuydPDjv8VBZD9wHdZzJwa7s78MQLb2JaLIzupXMsx3p7ujAjFoYiqxbn+AvnzsQdN1wCBgaBM6RzEjY8/mvj2O3XXYLUaBbDI1nMPKcVk0TBYhz1F0+MUesyaWuLVWOJ6WyUqN1KUetyLyfN8CytrSLOpCVIkgrOAEUFQiGGKdEQRvKTinYmqnbLRS11IwgcZ3Myjp4aMTqFn2ifjJFRydIR19si+/ZNt8Atepumr/x2iw5uT0MCCn7Asd9H9wFtbi83rLocuZxinUwvcP8q6RZocO1WQ59uOtT7Llxgjn6Tlx6L3UPXUDwWxeo/uARnhrOO+/n9UGQuF7d6UEiPAHwFPCpw76awuY3SZnoFjHR773ct78BzL72D7qVzDI16BbiaHgvjZCpbUAdeZTSeIJb2umAPBGsPsjgerdppFu0CwfTb1hbD9fc86zvt5757Y6DzG+Ga48dTdVvn/eSrHrTrNo9QSntoJkgdj7aKODqYdtx/ZjyKtMcYIQjl1kcQO1nJwMC11H096LaUsWUzU692sFr4ff5StEsT0B6IYQFZWdvGEhI4BM6QyckQOENE5MjkVFd/TV6GURU4ejfvdXyN3Jj33eRmQCe68N2odZnUQwNRC2pd7uWkGZ5FFThODo1i+tRWww6dHBrB9KktYB4dsYmq3XJRS914tR9/9Y0rIcnubZEdc9sU4hxc0FxzBO3AFysHexsYFhiysmppEwH4HkDQBLQ/qqFPLx32rVkEJiuWflMxPRbCrKGwyKHIgKQE1yrgLBe3PhrgrcfxDHabxeY2Q5upvUdmuMTYsfuw4RJD1+/Yee6T2IV0UKkyClIXyjkx0yzaBWgCOug1NAHti9kooN1ibWWpBKnj0VYRoxnFaI9bIrwsk89Ac7QJbkz0CehSxpbNTLPq3C+VnIAmFxweWHwySTJkjG3/y1q2A1r9NRXz5Wbm2OAoFEVFSCXXFQRB+EdWVHzr7/Y6ft/Su4SMehPi1X5kcjJC+dXIhXwHaoet7ZYiV8Z1kr0NzMlwbRPJdVPjUcwHpV9flkXvY9KQkp/ALpdWvPpoXnokN2PNgSwrkBnDvX+3x/J7Mf/3cpHfq5Fvv3WBtEoQ9UGlfLIHqePpkZylPU6PkEUgCkNjS6Ja1J2eEonEFAD/CuC6ZDL5XiKRuBbAwwBaADyVTCb/vKYZtGFf8RMNc6SzimMFEPlnIwiiXAic4c++PB9zZk83bM1b750ke9KkVLv9GO+28WLXlmulLFFdKqXDcm+pNac3mEpDEHjJ6ZFW65egunHTr+YDmpe8wn68+R7vCn+CIOoP6qMTjQjplqgWdTUBnUgkFgJ4HMBn8/9uAfAPAK4G8CGAFxKJxO8nk8n/XbtcjuHl42n7zrew79BRh3/oIJG/CYIgvGiJCPh4ewy9m/dYbE9LREB6hAavzUaQiN7jZTy+RP1cWwnfiER1qIQOy+27tpzpkVbrl1Les12/uo96czs6Hu0FzXcxv84EQTQm1EcnGhHSLVEteK0zYGMVgK8D+Cj/7y4Ah5PJ5LvJZFIC8D8B/GGtMmcnK8MYmADa9pq+/gEs6Zxl+XdWtkZk3dK7BH1rFlEnkyCIkhjNKK62ZzRD9qQZqWb7IQHGBA2gaWtj/4AWgLAM13q1mzSfV/9UQofj0Vul0yOt1i+lvGe7flffNM/xfsejvaD5XnbNhcbkc7XuTxBE5aE+OtGIkG6JalFXK6CTyeRKAEgkEvpPHwdwxHTKEQCfDJLm9OmTy5I3N46cOOvq4ymWj56u/1tWVZzbFivpHm0lXtfMTJQyqaR2S6GZyr3Rn+XIiWFP/3Kl2ppyUm/aLReNrhs/HBsccdUWGDOe36sc/Fzr1W6Op50sJ42s3UbUpx/N1Cq9eteqmUrrtt60VY73XG7t+bnGfM9Yq1jW+zcq1bC5E6k8x0uxdr7W1FO+vLRb7330clBP76GcNOtzmZnIug3KRNBDISr1/HU1Ae0CB2B2OsMABPoMU8noyoIguPpDnDIpgr41i6CoKhgYoALHTp0dd/R2ovZlUk1DVOnI4EGodbmXk2Z4FkEQsHDuTCzpnIVYq4jUSA679r8PgTPPZ5uo2i0XzaAbXwjctV0TQwzHTo0Y/lZdfeF6XAtVNcrOq90UmLt2q935a1Tt1oM+S/Ll7EMzwTJRvvTsWk2cF0f30s/67tM1i82tB205KMd79mWv/GnaTxkJAgfA8ND/txhDw1koqlpe7ZeRZtEuEEy/E32yAQCOH0/VZ52Hv3dZD9r17OcU6KOXm3LHVjBTr/oYL7V8rnrRbdCxZTPTrDr3i9/nL0W79eaCw85vAZxr+vfHMOaeo+aEBaC3p0vrMAKGr5wnfnYI/c+/AQD479tfxeq+F9G7eS8GR3L5DihBEETptEY4upfOwdZnX0Pv5r3Y+uxr6F46B60Rsi/E+ND9pJrbtW/feQVODmXQu3lPvj3bgxOpLMSwUPRa3Uewjle7aUuKaDB037a9m/didd8u330eP5oJQjnTM2s1cV4ct33pIjy24yD16eqAcrznYmmUqmk3xtLag3v/bg+2PvsaOGO4d8VlZdM+YSU2pcWymtfPfwRRDloi3LWf01KlPno5bRcxcaCxJVEt6r2fsw9AIpFIfAbAuwBuhRaUsC7IZWXMiIXRt2axES10yzMHse/QUazv6XL17da3ZhFYjfNNEERjM+Lhp6tvzWKyL8S4MPtJ1VfOAMyX3tyuta+6cWs3XVdTEw2Fl0/eYn0eP5oJgj29aCQENSeXlJ5ZqypUrN+8l/p0dUI5dFMsjVI17YZbWn/zo1ex6euLsWnNYkiKUvZVihOdaCSE6+95NtA1z333xgrlhphIjGZkbN/5FlbeeKmxknT7zrew+qZLq9JelNN2ERMHGlsS1aKuJ6CTyWQ6kUj0ANgBIArgZwD+V00zZSOXlcGgFaSkKNh36CgAb99usqLWd6ETBFH3yIriYV8Usi/EuJFlxWjXIKuQGPOtN/u1btPK5nYTsowczT03PLKiltzn8aOZQHkxpRePRce1hVLXquxZB6hPVyvKoZtCaYxH0477eKQlyQpCqlo27RMEUXtkRcW+Q0eNOQGdO264pCrtRTltFzFxoLElUS3qUk/JZHK26e9dAP5D7XJTGKuPpTF/cqmRnKf/J8iN51+SIIj6wWxrdDT7wgGZhrBEeSmn3irpl5CoHQJnTdHn8dJnszwf4Z9yvnPvtDgEqGQDCaKJqHV7Uev7E40JjS2JakFOXcaB3cfSlmcOGj6fduw+jLXdHeTbjSCIshMRmat/uYhIm6SI8lMuv83kl7B5Kbcv51pQSJ/N8HxEMMr5zt3Sumt5B7Y8c5BsIEE0GWHBvY8eFqrTR6f2iigFGlsS1YJs0Tiw+1jSt9rovi3DIiffbgRBlJ1MTsEvX/0Q96/8PAQOyArw4sB7uG7x+eSniyg7Fr/NqgqBsZL8NpNfwual3L6ca0FBfTbB8xHBKKemx9JajBNDoxgazuLJn72J5AeDePejM2QDibogm5MtgRv9kM5ISJ0ZLX7iBCIrqzX1Ad0M7TFRfWhsSVQLmoAeB24+lvYdOoo7btB8uin5wTn5diMIopzIioqnf/EbPP2L31h+//0rPk1GnagIui/cc9tiOH48VZLfZvJL2NyU25dztSmmz0Z/PiI45XznsqxAZgz3/t0ey+9kA4l6ISwKJQVuLN3LfnNSax/QALVXRHBobElUC9LTOBA4w8K5M7Gkc5bxhfPNd09A4JxWPRMEUTEEznDz1efj2q7Zlq/U5N+NKCdu/nDHlwbHwrkzLYMy8ktIlEq5/Inr6UBRcf/Khdi+820kPxgEQPokxiiH3sIix/0rFyIaDiE1ksOO3YcxmEqTxgiiiaA+OtGIkG6JakET0OMgLDB0L52DvvyWzYVzZ6J76Rz0bt6DY4Ojhs+leKtIk9AEQZSNiMhx1fxP4YGtvzJsjeaniyNLtoYoA7o/XN0lgd6eTZvmvxPqlkZvTxcAbbeQ2S8hrc4hguClz6D9Lbd01nZ34IkX3sRgKk36JACUR2+CwHE6lcVjOw5atBaPRUhjBNFEUB+daERIt0S1oKgX4yArq8bkMwAs6Zxl+bfuP1CqZSYJgmg6MjnVYWv6+geQydEXaqI8ePnDHTqbGVcaff0DWH3TPGzpXYK+NYvoAy1REl76DNrfckvnke0H8GdfWUD6JAzKoTcJwIMuWouKIdIYQTQR1EcnGhHSLVEtaAJ6HNj9BcZaRU//gQRBEOVCVhQPW0ODWKI8ePnDzUn+NebtU1eLk8BkhSZeiJIo5K+5HOkoCumTGKMcevNKQ6J2myCaCuqjE40I6ZaoFuSCYxzYfUBPmRQh/5YEQVQc3Zeu2f/8rv3vQ+AckGkjLzF+f6UCZ2iPt1g6o+3xFoghDknypzGvNKhNnHiUy1+zkV6ZtEUaJYDi+iyHTkhrBDExoD460YiQbolqQRPQ48DuA5r8WxIEUQ0iorvtiYgMWTI2E55y+CsNAVjf0+VIY+qkCE6mc+NKg9rEiUW5/DWbKZe2SKOEH32WQyekNYKYGFAfnWhESLdEtaAJ6HFg9wGt+8rpW7MYd9yglGWVD0EQhJ1MTvGwPYvAapw3ovZ4+SsNog9ZVhBvFdG3ZpFlVSDn/hXmlQa1iROLcujRTrm0RRol/OizHDohrRHNSDYno60tFuiadEZC6sxo8RMbFOqjE40I6ZaoFjQBPQ6K+beErNKqBoIgyk4hf5Rk1Ily6UOWFTDkOwoltmflSINobCplr8qlLdLoxMavPske1o7YlBZEI9S7qUfCooDr73k20DXPffdGpCqUn3qA+uhEI0K6JaoF6WkckD83giBqAdkeohCkD6KeID0S9Qzps/6JRkIlTXISRC0gm0I0IqRbolrwWmegkdH9ubXHWwDA4s+NIAiiUpDtIQpB+iDqCdIjUc+QPgmCKCdkU4hGhHRLVAvS1Dggf24EQdQCs+0BY4Cqku0hDKhtIuoJ0iNRz5A+CaK6lOo3ulGgPjrRiJBuiWpBE9DjhPy5EQRRC3Tb09YWw/HjKbI9hAVqm4h6gvRI1DOkT4KoHqX6jW4kqI9ONCKkW6IakAsOgiAIgiAIgiAIgiAIgiAIoiLQBDRBEARBEARBEARBEARBEARREWgCmiAIgiAIgiAIgiAIgiAIgqgINAFNEARBEARBEARBEARBEARBVIRmDkIoAADnrNb5GBeNnv9KUOMymQ3gtwAqGY65LrVbb/kZD830LIDv55mNCardctGszxWUKpfDbFRet0ATaLeR815Jalgus9EkNpe0VZwmK6PZaBLtEs2FD73MRp1ptxk13ozPBFB/wUyzvuMgTPQyqNT8AlNVtcQs1T2LAbxU60wQTcmnAbxXwfRJu0SlIO0SjUildQuQdonKQDaXaFRIu0SjQtolGhHSLdGoBNJuM09ARwB0AjgCQK5xXojmotJfKEm7RKUg7RKNSDVWQJN2iUpANpdoVEi7RKNC2iUaEdIt0ajQCmiCIAiCIAiCIAiCIAiCIAii9lAQQoIgCIIgCIIgCIIgCIIgCKIi0AQ0QRAEQRAEQRAEQRAEQRAEURFoApogCIIgCIIgCIIgCIIgCIKoCDQBTRAEQRAEQRAEQRAEQRAEQVQEmoAmCIIgCIIgCIIgCIIgCIIgKgJNQBMEQRAEQRAEQRAEQRAEQRAVgSagCYIgCIIgCIIgCIIgCIIgiIpAE9AEQRAEQRAEQRAEQRAEQRBERaAJaIIgCIIgZ4kZLAAAIABJREFUCIIgCIIgCIIgCKIi0AQ0QRAEQRAEQRAEQRAEQRAEURFoApogCIIgCIIgCIIgCIIgCIKoCDQBTRAEQRAEQRAEQRAEQRAEQVQEmoAmCIIgCIIgCIIgCIIgCIIgKgJNQBMEQRAEQRAEQRAEQRAEQRAVgSagCYIgCIIgCIIgCIIgCIIgiIpAE9AEQRAEQRAEQRAEQRAEQRBERaAJaIIgCIIgCIIgCIIgCIIgCKIi0AQ0QRAEQRAEQRAEQRAEQRAEURFoApogCIIgCIIgCIIgCIIgCIKoCM08AR0CMDv//wTRSJB2iUaFtEs0KqRdohEh3RKNCmmXaFRIu0QjQrol6oJmFuAnAbx78uQwFEWtdV5KIh5vxeDgSK2zUVfUukza2mKsCrepO+3WutzLSTM9C+D/eSaqdstFs+mmVKpdDlXSLdDg2iV9ulPLcmkWm0vaKk6zlVGzaBeo73dTr3lr5HzVm3brtSzHQzM+E0D9BTPN+o6DMNHLoJLzC1WbgE4kEn8MoDf/z/+dTCb/NJFI3AjgAQAMwLsAbk8mk4O2664G8BMAH+Z/OpBMJm+vUrZrSigk1DoLdQeVSW1opnJvpmcBmu956hUqZw0qh/qE3os7VC7jh8qwOFRG9Us9v5t6zRvlq3w0Yp6L0YzPBDTvc5UClQWVQSWfvyoT0IlEohXAowA+C+A0gL2JROIGAI8B6Ewmk/+eSCT+EsAGAGttl18G4K+TyWRfNfJKEARBEARBEARBEARBEARBlIdq+YAW8veaBEDM/3cSwNeTyeS/5885COA8l2s7ASxNJBIHE4nETxOJxKeqkWGCIAiCIAiCIAiCIAiCIAhifDBVrY6vw0Qi8Q0A3wEwAuAXAJYlk0k1f6wFwEsA/jaZTG6zXfc9ADuTyeRPEonE1wB8JZlMLvJxy9nQ3HoQRDmpho+m2SDtEuWHtEs0ItXyAT0bpF2ivJDNJRoV0i7RqJB2iUaEdEs0KoG1W5UJ6EQiMQ/ANgD/CcAQgP8JYCCZTP5VIpGYCuBpAO8mk8k7fKR1GsCsZDI5VOTU2WjggEIA0NYWw/HjqVpno66odZlUKUjAbNSZdmtd7uWkmZ4F8P88E1W75aLZdFMq1S6HKgYhnI0G1i7p051alkuz2FzSVnGarYyaRbtAfb+bes1bI+er3rRbr2U5HprxmQDqL9jy05TvOAgTvQwqOb9QLRcc/wnArmQyeSyZTGYA9AP4vUQicS60lc8HAay0X5RIJHgikbgvkUjYvWBLlc4wQRAEQRAEQRAEQRAEQRAEMT6qEoQQwP8F8J1EIjEJmguO6wG8CuA5AD9OJpPfdrsomUwqiUTiZgCHAfw4kUjcBmBfMpk8W6V81x2CwCEBkBUVAmcIAZBlxXEsLHIoMiApiuVvgTO0RASMZhTIioKwKEBVVeQkBQLniIgMmZzimn44GkImp0JWFIghDsYYsjnZuC6b1r4LiGEBWRmQFS3NsADksjIAINoqGvcWBI5omOPsqOTIl8C1Y5nRXEXKivBHa6uIsxkFR04MQxAETIpwjIyU/k5qia5f/VnMmiXqA90+gKkAGBRFBWcMkTDX7JKs2Y2WMMdoNv/vvP0BYNgnQeBgDFBVQOAMOUkBZwyMqVBVOGxBOWyF3zTILjUPftvj1mhoTL+cQxQZcpKKiMiRzspgDIDKoCgKRJFDVYGcpIJzLR0OQAU0HXMGgQOyDCiqCs68O3J6HmBKn5PmCB9EWkTkZBWSrODIibMIhQRwDoiCZoulvC0WQxy5nGzRmMA5BIEhk5M1vQkMAoCsrEJRVIiitqZEklWoqmqcI0lqXtMMjAOqAiBvs6NhAZnsWP/Qy5YTRKkEaZv92n79mCBySAogyQoUBRBDDJwxHDs1AggCBAFQFK1OCHm7z6D1g3KyCkVREBI4wiEGSVEhySoY0/o95nYlnZEREjhCIY5sToaiABGRQ1a0umwek+n5VBQVnHOt36VWb2KgGG7lSDQesSktiEaCvb10RkLqzGiFckQUopnG/aWiz2MdOXEWgiBY5rEmAtXQQFXseTKZ3JlIJDoAvAIgB2AAwBsA7gMQSiQS/zl/6svJZHJlIpH4SwAfJZPJ7wFYAeDxRCJxP4BjAG6rRp7rEUHgGBzJYWP/AI4NjqI93oL1PV2It4oAYByLx6K47UsX4ZHtByx/Hxscxc1Xn4+r5n8Kfbbz9PR6e7qwfedb2HfoqCV9QeQ4PpRBn+nea7s78MQLb2IwlUZvTxfapkagKipOpLKW83p7ujAjFoYQ4jg6mHYc+81vBxENh/Dx9pjjWPu0aEmT0IXKigYr/mhtFXHE5X2dG482XGMUjoYc+tU1S5PQ9UG0VcTRwTS273wL1195AR59yt0umW2Y+fiklhD+/LF/NX67a3kHnnvpHdxw1QWGndJ/u2XpHMMWlMNW+E2D7FLz4Lc9/kLXebjs4o859PrGb07g4vNn4JevfogFF30Mjz7lbK/1djYa0SbstvzkdQym0li3ohNP/XPS0k5Pm6a65u9HLvWJNEcUItIi4vTZLE6nMhYt/sXKhZAk1aL5dSs68dKB3xoadusf3n3rAsQmiXjg8V8jHoti9R9cgnRGNtJeOHcmln8hgU3b9jvs9/VXXoBX3vydw+a72XKCKJUgbbNf268fu+/2LrSoIRwfHLXUp7tvXYAfPHcI02JhdC+dY9H33bcuQHxKBCdOW6/ZsOpynDmbw9M/P+yw6+tWdOJne9/Fwf93wmgjTqeyruO8tqkRnBjKWPJprlP29qTaeJVxrfNFBCcaCeH6e54NdM1z370RE9fxQe1opnF/qYhhwXMeayJMQldLA9VywYFkMvlQMpmck0wmL00mk3ckk8kfJZNJnkwmP2f6b2X+3L/ITz4jmUweSiaTVySTybnJZPI/JpPJD6uV53pDAozGGACODY5iY/8AJNuxZddcaHQ2zH8DwLVdsw1R2Y8dGxxFX/8AlnTOcqSfyanGdfqxR7YfwLJrLjSuy+RUZGU4zuvrH0BWBkYziuuxz312JubMnu56LJ0tbVBRqKwIf5z1eF9nM4030HPTr65Zoj7Q7cOSzlnGoApw2iWzDTMflyTV8tujTx3Aks5ZFjul/2a2BeWwFX7TILvUPPhtj39vwadc9do191z09Q/g2q7Zht7d2uRHth/AmeEszgxnDR1v2rbf0U4Pnc245s+tPpHmiEKkswqOnRpxaPF4Xjvm3zZt22/RsP672e4+/MNXcOzUiKHxM8NZS9pLOmcZk8/69bqtfvSpA642382WE0SpBGmb/dp+/diDP9D6J/b69PAPX8Gyay7Eks5ZDn0//MNXIMvOa46eGsHDP3zF1a5v2rYfN/+etY3wGudlcqojn+Y6ZW9Pqo1XGdc6XwTRzDTTuL9UCs1jTQSqpQHa0dJAyMrYBIvOscFRyHlH8vqxWKvo+jcACNz9PHN6sfxXfGv67vfWz9XOUyz5sKZR+rFSRFqorEj0/pAVpazvpJY007M0K/o7KmaXzDbMfJzbQiCY0zLbKf033RaUw1b4TYPsUvPgtz1WVffz9N/9tMnRsKaOaHjsN3s7nZMUSxhqPX9eaZLmCC9kRUE0HHLoxu03u4bNv5vtrq5h8286hWx+sfRJy0Q5CNI2+7X95mOcFa8jfq7R66BXnRG4v7S9+sR6uvb2pNp4lXGt80UQzQyNlakMqvX8VVsBTYwfgTO0x1ssv7XHWyBwZjmWGsm5/g0AsgLPY3p6KdMS+7H0ecFztfO453njOVYKhcqK8Ee530ktaaZnaVb0d1TMLpltmPm4PaCzOS2zndJ/021BOWyF3zTILjUPfttjxtzP03/30yansxLSWcmhY/M5Yshqy/Q8eKVJmiO8EDhHOis5dOP2m13D5t/Nek1ntbWkqZGcI51CNr9Y+qRlohwEaZv92n7zMUX11rCX/t2u0euO1zW6t5BiaRca07m1J9XGqxxrnS+CaGZorExlUK3nnxil2SSEAKzv6TKEofvECtmO7dh9GGu7Oxx/A8CLA+9pvpVdjul+Xnbtf9+RfkRkxnX6sbXdHdix+7BxXURkCAtwnNfb04WwALREuOuxf3v7KN5676TrsWi4NIkWKivCH5M83tekSOOZDTf96pol6gPdPuza/z7uWu5tl8w2zHw8FGKW3+5a3oFd+9+32Cn9N7MtKIet8JsG2aXmwW97/PNXPnTV68ChI+jt6cKLA+8Zendrk9d2d2DK5DCmTA4bOl63otPRTk+dFHHNn1t9Is0RhYiGOdrPaXVosS2vHfNv61Z0WjSs/262u3ffugDt57QaGp8yOWxJe9f+97FuRaer/b5reYerzXez5QRRKkHaZr+2Xz923+1a/8Ren+6+dQF27D6MXfvfd+j77lsXQBCc18w8pxV337rA1a6vW9GJp39ubSO8xnkRkTnyaa5T9vak2niVca3zRRDNTDON+0ul0DzWRKBaGmCq2rQ+UGcDePfkyWEo9qVxDUJbWwzHj1vd8PuNvBwWORQZkBTF8rfAGVoiAkYzWjTxsChAVVXkJC06ckRkyOQU1/TD0RAyORWyokAMcTDGkM3JxnV6MDc9eqgerdwcPTTaKhr3FgSOaJjj7KjkyJfAtWP2AIRuZeJFkIjWfmlri1VjxnI26kS7eiRUvQwbORrumH61ZzFrtpHxWycaQbu6fQDTor8rigrOGCJhrtklWYUgMLSEOUazihH9Xf+QkMmpkFXt/TIGqKq2kiYnKeBM/0112AI/tqJYOfu1N5WwS9UkiA0u0/2q9ZVoNgJq12973BoNjemXM4giR05SEBE50lkZjDFABRRFhSgyqCqQkzXtCwIDB6ACmo7zq+xkWYWiquBMG6yfc84kz/4CGAA1X584Gk5z46HaerXdu+5trheRFhE5WYWU11mIM3AOiIJmi6W8LRZDHLmckteYpmGBa7rN5BRNbwKDACArq1AUQBS1gYwkq1BVFZwzhAQGSdI1zcA4oCoA8jY7GhaQySqafS9gy2tFLXVWCRpZu3b8vpsgbbNf268fE0QOSdE0rygqxBADZwySogIqIAiAkj+u1x/9BeTy14QEhnBIu0aSVa3d0O/DGESRIZ2RERI4QiGObE6GogARkUNWVEiydUym51NRAM6Z1u9SvduTauNWjn7yVW/abTbbAAR7pra2WElBCGtRZtRfaK5xf6kY81j5/oZ5HmsiEFQDpWiXFg40GLKs+b8KAYCsQvY4puQriv1vyCrSI+bzJNMxGVkZnuln05JxTM3JUG3X6eSysikNGTnTsfRIbuyYJCMjya75giwjMzq+yl6orAh/jOTf17n5RnlkpHFLUdev/iwTqC1pGHT7oKN/cM6OmmyKBKQlq43R36XxmzL2b8WUjv6b/dWXw1b4TYPsUvPgtz3OjpraPRmQZE2/WUm2bEMTAChZ7W+9c6bKY3oVAEDRNM0wputiWjOnD9n7fILQ0T/+cwAz822mAiADU39SAqS8LdYxNCaP6V2RrZpV8sGlDe0rgCxZNY18XxT567Kjprpm+p20TJSLIG2zX9uvH9Mnp3n+PzWnabddr1v5BMx1Rke/BhKQH7JZ2g29Tkjy2NhKyrctHICcr2/2MZmeT73OGvkv8NzVhPpKBFF9mmncXyr6PJZeBrkJVgTV0MDEWVNPEARBEARBEARBEARBEARBVBVaAU0QBEEQBEEQBEEQBEEQLsSmtCAaCTZ9ls5ISJ0ZrVCOCKLxoAlogiAIgiAIgiAIgiAIgnAhGgmV5NO6ubyAE8T4oAnoGuAn0IUgcAym0pAYKxrgIiwwZGXVFGhIC1Sk/64FHuIAUxHi1mAUQt7hnjlIoX4vczDBsChAyV8X4hxcAHKSHiRJ8/cnKwpCAofAGbL5YEkA8gE2GGRFMQJcyLLiGTRRCxDHjYCH9kCGRPXQHdEfOTEMQRAaOhhBMz0LMGYHjg2OAAKvm2BMQRAEDlVgUKHZIM1OqJBlBdFICDlJAaBCDAnI5UzBS0WOdFax2AcAmi9opkLgY0Gu9OA9nDPkcrKnbTTbJnseC5XzWCCf4mnVkkYPftgI6HqWZE1zsgIIHJAVGAEzQyEGWVIRFrXjWoCpsbYzk5MhChxCPoiUquYDREEFUxm4AGRzY201AIs+dW3LioqQwMG5NViwHmTY3uaSHiYWbsGiFVm19MkYGHKyguOnR6CGBEPDosjAFBVK/riS15rItTRkptUBLQAhR4jDEdw6FBYsNrwlwjGakR32yVynFEXrwwrQgie59YML2WFnUE6tn1rI7pO9JHTsdYYxFaqHxuS8Vrmg9Q1UlY0FfldU5GRNe0PDaQgCt2hday+0IJ96EGUxxPNjMK1eiSKHLKmmejYW6NNeJ8waBuDIn6yojvpgfpZ679sQBFE+mm2sXArhaAiZnGqUQURkyKalWmeralRDAzQBXWUEgWNwJIeN/QM4NjiK9ngL1vd0Id4qWjow2jl7HecAcFzf29OFX776IRZc9DE8+tQBy+/bd76FfYeOoj3egm/ddhkkScXDP3zFcg4A9NnyM2NqBMeHMujrH0A8FsVtX7oIj2wfS3ttdweeeOFNDKbSWLeiE0/9c9K4j/nYXcs78NxL7+CGqy5AJCzgn158G7csnYPpsTBOprLY6JG+Pe+9PV2YEQvX7L1NRFpbRRwZTFu00dvThXPj0YZrjJrpWQB/dqTeEQSOszkZo2clpDMyXn7jd7iy45PYtG0/5n1mBr646NN46p+T+PLvXwRJymLTtv04NjiKm68+H1fN/5TjXU5qCeH7z76OP7z2s2AMSGdki025+9YF+MFzhzAtFkb30jmW63U7dcvSOR622L2c3Y57pVVLmkEv9Y6u58FTGUPLLx34raFpvdzXrejEkePD+PQnpmIwlXFtV6fFwlj+hYTlOnNbqrevG1ZdjlxOwYP597pw7kyHts3tsd6unk5lHW0u6WHiIIYFnEhlHf2+sMix4fFfIx6LYvUfXIJ0RsZPf/kOrr/yAkvf8i9WLgTnDINnrPrdsOpy5CQFD/7Aqr9psQhe3Pc+nv7Fb9Aeb8F37lqMY6ed7fEvX/3QOGd9Txemx8JIpSUMnnK5j0n3bn1Gux0GtL7zj3a+5Xge7z442UtCw63OeGnMrJu7b12AUIjhO0+87DmWisciiEVDSKUlnDmds4zR1nZ34F9e/hDXds2y/O427poWi+CJF97AvkNHXdsCt3pjzp9Z61OmKA3RtyEIonw021i5FMLRkDH/ZS6DtqmRCTEJXS0NUBDCKiMBRoMOAMcGR7GxfwCSz3PcjvX1D+DartlGh9r8+5LOWca/U2ezRgfGfM7pVNpxr0xONcS37JoLjQ6Tfs4j2w9g2TUX4tjgKDZt22+5j/nYo08dwJLOWXhk+wGkzmaxpHMWNvYPICuPPYdb+va89+WvIarH2YxiaAAYew9nM43X6WymZwH82ZF6RwJw9NQIzgxn8cj2A7i2a/bYJPPvXWjYlRDnxu8AcG3XbNd3KUkqlnTOQups1kjTfM7DP3wFy665EEs6Zzmu1+1UEFvsddwrrVrSDHqpd3Q9m7Vs1jQAo72cM3u6ca5bu7qkc5bjOnNbqrevR0+NGJMJAFy1bT5fb1fd2lzSw8QhK8Ohk439Azh6asTok+k2dEnnLEff8vjgKI6edOr36KkRY/JZ/+2R7Qdw7NQIru2abfwmS8776/1Yc36yMlzriV33bn1Gux3WbaDb8wS1+8TEw63OeGnM3u9Inc0WHEsdPTViaN0+Rntk+wHc/HsXOn53G3cdOzVi/ObWFrjVG3P+9N829g9gMJVuiL4NQRDlo9nGyqVgnv8Cxsogk1NrnLPqUC0N0AroKiMrqvFSdY4NjmrboXyco/9tPyZw999j+a/yABANh1zPiYZDjt9kRTHOjbWKBdO238ftmH6faDh4+uY8EdXD/I509PfQaIajmZ4F8GdH6h1ZUQ3bY7dh+t+xVhGcWW2bl63jDA6bYT/HbJfcjgWxxaECx93SqiXNoJd6R9ezWcteWpUVxbM99qNR/Rx7GkHaatLDxMWrPdTtsVv/zYzZbtt/90pXMC138bq/+ZxC9aRY3bHn3dx39qojQew+MfHw0qybxuznmOuV1/FCbYKf8Z15jOV1ryBjQKlB+jaNQDYno60t5vt8ClhH1IpmGyuXwkQvg2o9/0Qoy7pC4Azt8RbLy22Pt0DgDJDV4ufk/7YfkxX331Om5fLprOR6Tjpr/Zat3Ysb56ZGcgXTtt/H7Zh+n5ykBk7fnCeiepjfkY7xHuTGWo7eTM8C+LMj9Y7AmWF77DZM/zs1ksPUyRHLs3rZOkUFUiM5iCFvO2m2S27HAtliWfU87pZWLWkGvdQ7up7NWvbSqsC5Z3vsR6P6OfY0grTVpIeJi1d7qNtj3Y6aNWc+12y37b97pSsrkaL3N+/oL1RPitUd87/tfWevOhLE7hMTDy/NummsUL3yOl6oTfAzvjOPsbzuFWQMGGqQvk0jEBaFQEHrKGAdUSuabaxcChO9DKr1/DSjV2VCANb3dKE93gIAhr+tkM9z3I719nThxYH3cNfyDsfvu/a/b/w7NimMu29d4DhnWizquFdEZOjN32fH7sNY221Ne213B3bsPoz2uOaL7P9n7/2j5DjuO7FPVXX3/FgsyF1yF6LfMYAl44Y0HSdLaIe6OJJogmISK7SO5nsGSPmIpWP57r1ciCfKsfDDuaN9BkhdLPqBuUT3LMcCFIsE7cfjWYrkF4lgKPHkJ+0KxNmxRI5gikSkmMTix5JY7Oz86K7KH93VU91dPdMzOzs7O6jPe3iYqd/fqk99vt/qnZlS+1HzHtkzg1ML57B/7wzGxxycWjjn/9Yga9mhaz8+9oNBHYPBYSxHQw4ArXUYy20+2RglW4BsOjLssABsmyxi6xYH+/fO4IX5N3Fg3yymJwp4/qWzoa64nIfpAPDC/JvatbQsglML5zA+5oRtqmUefXAXnnvxLE4tnEvUlzrVjRan5ae1tZEYBb4MOySfVS6rnAYQ+svX3rwUltX51VML5xL1VF8q/eu2ySIOK+uq47ZaXvpVnc81fLh24DAkeHJoroxtk8UwJpMaemrhXCK2nJooYNsNSf5umyzi8MNJ/k1PFvHC/JthGrOS/cs4Vh2Pw6DdJ3He62LGuA5LDdTZ063uG1x70O2ZNI7F447xMaftWWrbZDHkevyMtn/vDJ5/6WwiXXfump4shmk6X6DbN+r4ZNqhuTImxvObIrYxMDDoH0btrNwL1OdfQGsOcjbZ4JENBoPiABFiZP+CuQPAG5cuXQXnw2Vjltu1GaMgNkOt7ibKxOvLm45bN3v7tyK3bgUHKCUA8W899oKblBklYMFDXe4BLueRvtQbnx2bBTcw8/Dm5KYrQAkBswg8V8ATAhb1b21uuP5tygAAATDm3wCt3qCs2uHYNDKGnE1Rb4rIDe3Nhv81pgsXNu5vw1NT44NQoB0YEu7Km1Al1zbzbbijZAvQ0gEQAgjR8WbyYeSuvPVdwNcgXycEPE8gn2Nour49tkXRbAZrxwjyNkWtwSP6APi/0wgiwm9LuJ4AD3SJUoJmk6dqY9rt7p3muXVTfOe2NhJZ/E4WDFqDB8RboA+6K/nsegIWo+FcS04zSmAF/tKxCTwOND0BzgUs5vvOetODzSiYRdFoehAi4BUEiCCgDGg0W74aQISfktse98dAKUGj6YFRipxNUA/2UdznDhtf+4WNjBmGUXMl1PhOaij3RCQmIyBoen4sJ0SgHYTAtgkIF+AgEf7a1G/DI/4eoBSwGIVFEfJOcs1yWETDCzmK1bqX0Cd1T3EOP8aEb6cuDm6nwy0tRyRWbqf7WfVyo2PTfmOYudst+rU24Z4R/j7wJVekns/kWYlz+PtHCDgWheACzWB/5B0Gr+FFuO7xFu8YJWi6HLZFwbmv6yTYg54rlH1GwIBQ+9U9oXIYQGJ8HheJ/TA5OYbLl1eGLrbJspbDxt2pqfGuPwE97FrSzZ7q1n5g4+YgI7/WxZ5h4e2onZV7gZO3gmdQIngmRa6JCwgluuVAL9w1f8TcAHgeB0Ew+Z6A7gPtnscxNTmGC7XlRJl4/abnx9MSDAA8hOnyvf+fHzRYQRpXGo6Pp9nwwn548BUtv4znPywKyvGgHwsAuN8VC14jVkYZSsQOHtwwKMfQiNjooTn633oYSlSrTRAANwVOuVrdvAsxSrYArf0jA6bNaI3n8VAQCKJa4tZ4qBmu29IiuEBdfa/ogyzPA/GhwT9wX47aaWPsZWSM7eZZ5mdpayORxe8YrA2SzxQAD+abQ/GPXusbbPJS3ZCjrs8ZC4BwPbiu1/qKWsyXxtdQ5WdT9ceu7/HlPmkoeXGfa/hwbUGN7+IaqvKDApia9LkVPsBSyKLyVz2etGJMjgaQ0B5vNRrj1aqeVp/UPUWDNlXex+PgdjosNVBClm2n+2Z/GEhE9kwAAj3H1LOSLOfvq+gZ7LotY+GDKcl19Twl4xZZj4X9INIO9+K+RkR9QcxfqOOTeyK+HzZLbGNgYNA/jNpZuRc0am5kDhrX2BQMggPXzmfqDQwMDAwMDAwMDAwMDAwMDAwMDAwMBgrzANrAwMDAwMDAwMDAwMDAwMDAwMDAwGBdMLCf4CiVSr8G4GDw9i8rlcpvlUql/xzAHwPYCuBbAP5ZpVJxY/WuB/AlAO8FcAHAr1YqlbcHNW4DAwMDAwMDAwMDAwMDAwMDAwMDA4PeMJBPQJdKpSKApwB8GMB/BuCDpVLpbgB/CuCfVyqVfwj/Z6g+oan++wBerlQqtwL4PIBjgxjzoOFfPkHhEgLBKBjrbmlkfY8QCMYgLJpoR9eH7TAIxoI0Blve5gX/R9jVPCdvRfqx8jZg+fmcMTDHCtprjYMG7XtKn7qxckbBHAvUYW3ngHMR2oCg/Xh51U4wBitvgSvvO83JZsCgxl0utYPsAAAgAElEQVQs2hCM4a2LVyEYQ7Fo972PQdkiuf7WxZUE1w0GB916M0YBi8Ep2C3NsRhg+2uWK9oQVkuL8kUb1JH7moJYTKs9vj5RCIuBOhY86vcty/aDc5tVQwx6QzufCbT44PtgBo/6FwpTxwp5nQt01ecjC8uygJcqh2yHgdh6Hqv+dHGparht0DPivPbjP5/HF5aqgEVDPSWBtkr+WnkLzLECjfY5Qx0WxpDqXojHpywoK2M05lhAMA4Z4wmlb79tSxvjSqT5GMNlgzR04kdW/rTjnnrmsfIWYDG8fWklSPP/McffS7K+7bDI/gBjCY1X31PHbyMsH8RGjNFwX3EW9M9Y6jgvLFXb7rG4zfExZt1fZl8aGAwXCrFzf2Edzv3DjkE8+xhmDML+QX0CmsF/2D0GYAWADf++kkKlUvlOUOY4gN8F8LlY3Y8C+FDw+hkA/2upVLIrlcrIXMnJGMVStYmjx+exuLSK6YkCDs2Vcf312W6F1tV/ZM8MvvLy63jgnlswERAnXuaxT3wAjSqPpB2cK+PGcQeEElx4t47HY3ljBQu/87m/ws//zI346H/53kj+/r0zuH48hy9+9Qf47vfP447btmHPR0p44sRCWObwXBm2TfHY57+TGOsvf+h9yOcY/ujf/S2Wlms4NFfGRNGO3C597u0r+P0/+W6kzy9+9dWw/A3jDi4tN0Kb0sZwfcqcxPscRqTxpd/jLhZtvLVUS3Dgpol8327EHZQttsNwcbmRsOXGcQfNa+12gQ2Ebr0Pz5VhOxRv/ORd/NT0eGSNHtkzg9Ovvo0P3X5zJP3QXBme4PjMie+10h4u45n/6zVFe27BEyfmE1px/biDvffckmivF84Nir8Gw4FOOiL58MzXX8O9H3wfnnr2TCb+SR+45yMlvHzmJ/jw7TdjomiDMoJLVxs4+oUkj5eWa3jsEx9As8lxJDaek1/394HhtkEWpPH6Bz+6iO03XYevvPw6fuUXd+KzXzqNifE8HvrorTh2ssXtf/Ebd6Bac/EHf3o6THv0wV0YH7Pxxa/+ILEXJN8fvOcWFAoWLlyo4tjJM9q2P73v/XBdgc9+6XRkD+Qchj9/4YdhjKvGiVofE4s7DZcNJDppXVYtzMq9g/tmQQjRntnuu3MnbJtEzkFqrHPHbdsS/kNq/jvLDfzmr/wcanUvsocOzZXhOBSP/dF3Us9FOl8SP0fq9gpjFCtND0uX65E+5Tmr3f4yPsbAYLhQKNp4W3Puf89EHqt9OvcPOwbx7GOYMSj7B/KnxkqlsgzgfwLwGoCfAHgTQAPAW0qxtwD8A031n5Llgp/nuAJgah2HO3C4QOiAAWBxaRVHj8/j3ZV6z/WfevYMds9ux9Hj83BTypy/XE2kPX58Hg0PqDdFSD41z3UFFpdWcd+dOxP5x06eweLlKnbPbgcA7J7dHgY4ssyR4/M4f7mqHeuxk2dw5WoD99+1M5wD9fdYXCB8+Kz2qZZveFE708aQNifxPocRgxr3Sp1rObBS719gOChbGh60tphnz4OFbr2PHJ/H+UtV3LLjhsQaPfXsGdxd3pFIP3p8HleuNqJpX5iPaU9Sn+6/ayd2z27XttcL5zarhhj0hk46Ivmwe3Z7+MBNlmvHP+kDnzixgLvLO0IONTyED5/j7UgffkQzHrkPDLcNsiCN1+Xbbgq5KR8A33/XzvBBkyx7YWk1fPgs0558+nQYD8b3gmzzSBBTyvZ0bV+52gj7lmnHTp7B8kojEuNKpPqYWNxpuGwg0UnrsmphVu69c7WeemZ78unTiXOQGuvo/IfU/Pvv2okrVxuJPXQ0iLHanYt0viR+jkybu/OXq4k+j2TYX8bHGBgMF6op5/5qH8/9w45BPPsYZgzK/oF8ArpUKv08gF8HsB3Au/B/euMeAOpHfAkAnXVE8z7zLNxww5auxroRWFxqBQattFU0XY7pqfGe648XbT+dkDBNRd6xtPU8IQAhtHk0WA1Gk+0tLq0i71jIO/77sH9NmbSxqvXl2KeCOWhnZzh2Hh132hjS5iTe50Yijbtp89Dvcb918aqeH1zgpj71MzhbVlK53i9bNhLDwFcV3XI371jwONfmtdOaeJrUgrR9r2pFPC8L5+L5g+LvsGGUbWsXM3TSEcmHXvgn64R8JwSep/fDsp00Hy7zw7564OMocXuzjbcXrCXWTeO1COJAlc86bqfxUMZz7fhOCdbedoY4Uecv+s3la4Fn64FBnNParU0nrcuqhVm51063decgtX4n3xKOrc0Yutln6jlSN4eLS9XUup32V68+Zpj22Xpxd5hsTMN6j3Gj5mC9+h2mNU3j7SDO/cOOa30OBmX/oH6C478CcKpSqSwCQKlUOg7gtwDcpJR5D4C/19T9/4K8n5RKJQvAOIBLWTu+dOkqOM/2UxYbBkYxPVGILPj0RAG2RXHhwnLP9ZerTUxPFAAhwjS1TK3hausxQgBCtHlyKj2ebG96ooBaw0XT9QvJ/nVlVKhjVevLsYdz0MbOcOw0Ou60MaTNSaLPGAbpQFK5mzIP7cbdCxhjen5Q0r9+NtoW0kdbNghTU+OZbBhm7tYaLhjV57XTGhWqFqTte1UruuWcdp4HxN9hQla+9bO/QaJdzNBRRwI+9MI/WUfyHUIk/Fm8nTQfvqx8Ta5nPo4ItwfN13jfg8JaYt00XpMgDlT5rON2Gg9lPNeO71xgzW1niRN1/qKfXN5Inq0HNgt3s6Dj2nTSuqxamJF77XRbdw5S63fyLbal9xmd2ug0ptS9wmhq3Y77qwcfk2WfDRt3exnPsGtJN3rX63psxBysJ782A28Hcu4fclzrc9CL/b1wd1C/9v/XAO4ulUpjpVKJALgXwDcB1Eql0i8EZf4JgL/U1P0agIeC13vgX0g4Uj/CYgE4NFf2HS8Q/g7WdWO5nus/smcGpxbO4dBcGVZKmW2TxUTawbkyHAbkbIKDmjwrCG6ef+lsIn//3hlMTxZxauEcAODUwjkc2DcbKXN4roxtk0XtWPfvncHWLQ6ee/FsOAfqX0gsAL/z63ck+lTLOyxqZ9oY0uYk3ucwYlDjHstRLQfGcv2TjUHZ4jBobTH3EA4WuvU+PFfGthuKeO3NS4k1emTPDF6YfzORfmiujK1bnGjaw+WY9iT16bkXz+LUwjlte71wbrNqiEFv6KQjkg+nFs7hkT0zmfknfeCBfbN4Yf7NkEMOAw49rOex9OGHNeOR+8Bw2yAL0ng9//23Qm5+6uO7MD1RwHMvnvVjPaXs1EQBv/VruyJpjz64K4wH43tBtnk4iClle7q2t25xwr5l2v69MxgfcyIxrkSqj4nFnYbLBhKdtC6rFmbl3vVbcqlntkcf3JU4B6mxjs5/SM1/7sWz2LrFSeyhQ0GM1e5cpPMl8XNk2txtmywm+jycYX8ZH2NgMFwoppz7i3089w87BvHsY5gxKPuJEIP5dHCpVPo0gIfhXz44D+C/B1AC8HkAWwG8AuDhSqVSL5VKvwfg7yuVyr8tlUqT8C8ofB+AdwB8vFKpvJmhyx0A3tgUn4CGfxmDC8Dj/qeeLACTk2OZ/9oi63MOUEoAIgDhO3j1cpZ4H5QRNDzA4xyMUjgM4aVsTt5CvSnCvJxN4DV52E8ux+C6HK4nQCmBzQgoBBqeCMfBGOB5AOcClLY+ch8dKyCEgM0oBAQaTR6OL34RxQ03bMHFd6rwuIBFKShDorxqp0UpLJug0eTgHLAoAYNoOyftLr+YmhqP/yTMemAHOnC323H3imLRxkqdh/2M5Wjff4R/ULbYDvO5LgQYIRGub2Z08QnooeUuAHiEwLEp6g3urxElIAQQHMjnKGoN7u9vSlHIUTRcAdfz97XNCCyKhPZwDrhBP4wSNF0OSgCHETQ80RXn0uZ5UPwdFmzAJ6AHwVsgY8wQ6ojGZwItPoAAEARcCFiMgBDfDzFGkHcoanUOLgQoIUFZwGIErscjHLIdBlcATVeAsSiPVX/qbxbRE7fTMArc3uBPQA+F5mZBnNc5m6De5AABiPJLeC4XsC0KwX2OUUJg2wQi0FrPC/SW+fFWvSmCeJQofG/FpwAgGIEbaHfOpvA8AZfzMMbzuACBX8Zvm8DjPBHjSqT5mPXk8gh+AnrTcLcTsqxNJ63LqoXtuKeez2yLwnVbewgUPp+D3zmsNz0wSuAwAg8I94dFCWyGiMarmu/YFAQETU+Ac9/32BTgnoAHAjc4izk2hdv095lunIwRCCTPkWlzp+7h+DlrLfMeR8ZPqA4Vd6emxnHvp/4ic8Nf+ezHhl5Luv0EdDf2Axs3B1k/Ab0e9gwLbwtFG1Xl3F/M0WvmAkKJQTz7GGZ0a38v3B3YHxorlcpnAHwmlvzXAMqasv9CeX0ZwC+v7+g2Hp7HQRAsiCfQ7WMxWZ8BUCt7mjJqH54HJc1DU6nQqLmRPHnGlv24NT9IsIKOPM/vTx0H96LvZfPxsRK0go52c0ApAQkO6PA8cC9ZPmqnB9fzYzsaG0PanGwGDGrc1WoTBMBNgVOuVvvf06BsaTa8iC3NzbLYI4Z2691wvVZecAYhAOpVL7KnawEP5b4Wrv+Xzbj2QGmLyzwAzYjurY1zm1VDDHqD1BGdz/STeOTiCgb/jyhC1nGBesDz+BcwpD9Wm5QPt2VdlceqP51SdM1w26BbxHnd8FoXsKiHcguAaAR/wA/yXSWOk/Eg9/ybxuN7QYWnvJBa7jXUONALY8hWGiJpOk6m8dZw2SANnbQuqxa245565nFdn+fvkXsrSJflZX3pX9QzTFzj1fe8EY2NEMRGiLQLuCnnLTnOqYnWnu+0VzyPR/Zw/JzVqa7ZlwYGw4PV2Ll/dR3O/cOOQTz7GGYMwv5r4/PkBgYGBgYGBgYGBgYGBgYGBgYGBgYGA4d5AG1gYGBgYGBgYGBgYGBgYGBgYGBgYLAuMA+gDQwMDAwMDAwMDAwMDAwMDAwMDAwM1gXmstl1QPxSBXmJhOcJeNy/aIhSAs45KKXwPA4uAEoICBEQwr/A5fK7qxCM+pfpMQpK/Qv3HJuCe4CQF7twDjtI010oob0MI+hPXg6Ts/3LKVx5cQWlqZdIMEaDyyw4KCX+BUueaLXNBRijYBZBU7n8L35xxiAuhjFYG+QP0b918SoYY5v6EkJ5sYK0ZbNfrDCItRkUpKaI4DI2Twj/IhvqX4TjxS59455I5Yx6mVY+Z6Hpti4vjF8YF794y7YJanVPe6np4lIVYDT1cqK4pqZdctm6MFaAUqq9MLbdHKm6y5DU1LQ2cgXbv8iRc1iMglESXnRkdHf9IDkGIgAQcC5ACAKf7l/cW29yrU+Wl8DlHYZanYc8zTsU9dXOez2urXmHhRyQF801am5knB73OW9bFLW6G7mQTuUso6RjzNHPCxENBoO4JloMuLBUhUf8uNVhBE1l3RmjbS8EjCOuYzajIJSg0fT8WJERMKG/KDrt4mm17bT4Vu4lGTPTGD/jbce5q9pt29S/dNHzL3RjjODKSg1gLLIfOum5iX2HA92uhcphx2bg3L8QmVGKnEPhBX5f+nnbZvC4AKBcxhlcNnj+chXMsYI7CP1YR/K101kJUC/wjJ7bsl6Wq7OZMYql5RpcQnriZr+5rba3tFzz59/sFQODdcEonS17hTwvyTnIGnOPCqT/eOviChhjqefZtcA8gO4zGKNYqjZx9Pg8FpdWMT1RwKG5MjzB8ZkT3wvTfuvXdiGfs7C80sCxk2fC9Ef2zOArL7+OvffcAh6r8+iDu/DC/Dn84vtvxpe/9Tru/eD78NSzZzAxnsdDH7010s7huTJsm+Kxz38nTHvsEx9As8lxRBnbI3tmcPrVt3H3HdvxznI90cb1RTsR4L9TbUba2L93BlMTBayuupH0A/tm8ew3Kvju98+H8/DM118L3+vGc2iujIlYnwYbg2LRxltLNTyurM/BuTJumsj3zRml7Zd+c6BQtPG2xpb3TOQ35UPoQazNoCA15emvvxZqmqqTxbyF3/vj70bsHCtY+J3P/VWCM5QRXFxu4PHj8/j5n7kRv/QLP40nTixE6t447qDZ8GA7LCyratbXvv0G/ubvLuLQXBk3jDu4tNxI5aeOv6qGy75UW9PKP3DPLam81+nup/e9H4zQTHsnV7Cx+E6UL/v3zuCLX30VS8s1o7vrBMmxkxpu7987g+u2OHBshv/9L/429Isqf37wo4so/9xNWNTs9enr820D4jjX7vvwe/Gh229OtDN1XQ6Ci9S9sFxtYO89t0Tyfvuh98N1BZ58+nTb+OLgXBknFZ9veDbciGviHbdtw56PlEINveO2bQku7N87g5zD8Ocv/LCthgF6HXv0wV3YUoxq/OGHy7i+YANAQi/TdCtNW0+/+jY+dPvN2j0Y56fatpqn2j0xnsfD994W4b7cy//H117NxPVBxT0GndHtWqgc1p29Ds6VMbHVwbtXGpEy6pkt7vt/+UPvQz7HkHcs/MfKIn72vTdG9pjuPHd4roxCwcKFC9XEue2G63K48G49ofUyHmlnMyD33Ld74ma/uW32ioHB4DBKZ8teoTsvZYm5RwW6s7HqP/oF8xMcfYYLhI4SABaXVnH0+DyuXG1E0v7gT0/DojQMHGT6U8+ewe7Z7XhcU+fJp0/jvjt34thJv4wMZO6/a2einSPH53H+cjWSdv5yNQz81f7uLu/A4uWqtg1XY1+8jWMnz8B1RSL9iRML2D27PTIP6nvdeI5q+jTYGKzUeShAgL8+jx+fx0q9f0Ff2n7pNweqKbZU+2jLIDGItRkUpKaomga0dPLC0mrCTtcVWs40PITzct+dO8MHJ2pd6T/VsjL/iRMLuO/OnWGbDa89P3X8VTU87qvblW/He53uXrnayLx3ao0kX46dPIP779q5bnvOoMUxHbePnTyDC0urcF0R8Ysqf8q33QQKot3rtUb7vR7n2t3lHdp26k3Rdi/Isah5yyuN8AGcTNP5c2m7fG94NtyI82D37PaIhuq4cOzkGSyvNDpqGKDXsSefTmr8kS/47ej0Mk230rRV8l63B+P8VNtW81S7779rZ4L7ci9n5fqg4h6Dzuh2LVQO685ejx+fh+cmy+j4J7X+2Mkz/nnvchXl225K7DHdee5IEAfpzm31ptBqvYxH2tm8Vm72m9tmrxgYDA6jdLbsFbrzUpaYe1SgOw/ozrNrhXkA3Wd4vPVgRGJxaRV5x0qkUQJt2fGinVqH0WgZAJHX7frMO5a2HKPpef7Xxjrb186WtPdZ+zTYGHicp6xP/0Q4jU/95sAgbBkkRskeyYGsOib1Jp7mcRGZF6mVyXI86Fc/h4xGy7bjZxp/pS3x9ehcXs97Xb1u9DPNDqnHRnfXB3Le23GbEmj95OLSKoQQPe/1OGfa7Yd2e0E3dh330vgYt83wbHgR50F87dvxuJOG+e1nj499PU/XS7Vcu7Z1MbOuLV3b8nXWeDsr1wcV9xh0RrdroZZP44K6j2SZdvyT/Mk7FoRY+xmyk89oZ/Naudlvbpu9YmAwOIzS2bJXXOtzMCj7zQPoPoNRgumJQiRteqKAWsNNpHEBbdnlajO1jsejZQBEXrfrs9ZwteU8np7HYk960uxrZ0va+6x9GmwMGKUp69M/2UjjU785MAhbBolRskdyIKuOSb2JpzFKIvMitTJZjgb96udQfqtTlm3HzzT+Slvi69G5vJ73unrd6GeaHVKPje6uD+S8t+M2F9D6yemJAgghPe/1OGfa7Yd2e0E3dh330vgYt83wbHgR50F87dvxuJOG+e1nj49ZcAdAVt1KK6uLmXVt6dqWr7PG21m5Pqi4x6Azul0LtXwaF9R9JMu045/kT63hgpC1nyE7+Yx2Nq+Vm/3mttkrBgaDwyidLXvFtT4Hg7L/2pjNAcICcGiuHC7e9IT/e1VbtziRtN/6tV1wOcf+vTOR9Ef2zODUwjkc1NR59MFdeP6ls9i/1y/zyB6/7nMvnk20c3iujG2TxUjatskiDsfG9sieGbww/yamJ4vaNuI/Em4BiTb2752BZZFE+oF9szi1cC4yD+p73XgOafo02BiM5aj/u0fK+hycK2Ms1z/ZSNsv/eZAMcWWYh9tGSQGsTaDgtQUVdOAlk5OTRQSdloW0XLGYQjn5fmXzuLAvtlEXYf5/aplZf6BfbN4/qWzYZsOa89PHX9VDZd9IUP5drzX6e7WLU7mvZN3knzZv3cGz714dt32nEGLYzpuy7sTLItE/KLKn/nvvwUOod3reaf9Xo9z7YX5N7Xt5GzSdi/Isah542MOHn1wV8f4Qtou3xueDTfiPDi1cC6ioTou7N87g/Exp6OGAXode/TBpMYffthvR6eXabqVpq2S97o9GOen2raap9r93ItnE9yXezkr1wcV9xh0RrdroXJYd/Y6OFcGs5JldPyTWr9/74x/3pssYv77byX2mO48dziIg3TntpxNtFov45F2Nq+Vm/3mttkrBgaDwyidLXuF7ryUJeYeFejOA7rz7FpBhBjZr7HsAPDGpUtXwQf8VZ34DcAOI/AAeF5wkzfzbxP3b5Qn8Dzh34pMCAgBhAAYAwj8241bdfwbuh2bgnuACG749m9Z7nwjvZqmuyXc5SK4TRmwKAlvU9bZ598A7d8AbjEC4olW29y/FZxZBM2mvJ2ZwGaI3CqeNsZ2F0tMTY3jwoXlPqxSb5iaGh/En913YIO4G4e8DVeuz3rchjuo2+ALRRtVxZZijm7KCwglul2bYeau1BRBAAiACxF+GkfeDN/SU4B7IpUz6u3v+ZyFpsv9S6o0N8HHb4q3bYJa3Yu0KfkpxTnOT5kf19S0W4PD8hygVJZHR97rdJchqalpbchbnT3OYTEKRgnqTa/rPTdoDR4Qb4F10l3JMRABwPfXhJAwDqAQqDe51ifXmxwgQN5hqCl7XXcjt25d4tqad1jIAUYpcjZBo+ZGxulxn/O2RVGru2G5epNHOMto55jDYSQzP9cLGxkzDLPmpiHkgRBgAU89LuB5vu44jKDpAW6wpowR/6uZGTQMSOqYzSgIJWg0PT/2ZARMtGJPlcMWpaDMj4N1fErTYnUvyZhZ2iL5GW87zl3VbtsmENx/TQkBY4BjUdTrPLIfOun5IOKeXrEZuZuGThrQ7VqoHHZsCh6cnRglyDkUXuD3pZ+3bdr6yYggvvF1FBDBeYkCYawj+drprAQAghHtuS0e28TjkXY2M0ZBbBbof/fc7De31fbyOQui6XU6Jw4Vd6emxnHvp/4ic8Nf+ezHNvScmwXd+NVu7Qc2bg6y2LVe9gwLbwdx7h92tM5L6TH3KCMeB6adZyV64a75I+I6wPM4CILJ9QSawZqFaS4gXScP0tU/LBAA3GsJoVrHAsADEsjVZgB4w38t+/SUtnRp8T9kSF7R4B88II1q0vFbQTmuFAzbdgHPjbbX9NLHE08zGA5Uq00QADcFXKxW+79C8f2yXhxYjdmyug62DBKDWJtBQWqKqmng0GgEknoa40yz4YV5bq2plPPCurqy8Dy4XrJNyU+px/FZlvlxTY33pS2vlOm0ejrdlf4jy96prypz4XrwMtYzWBskxyQkT+T6Aek+WdaruzyyB+qr2VYsrq31VR7huxpPRvaC68F1vUi5OGfl2Nv58zSfbzC8iPAA/gcnwkO5or+qDkmeZlnfuI7J92Gs6EbbiXLYA9dodLxsu70EtHgc5We07Th3VbuFGm8DEB6w9boiLlSXM3N9UHGPQWd0uxYqh3kjyueGos2Si7zBk/yTP/UViysIWnzNdFby9Oe2eGwTj0fa2ex5HFOTY7hQW+6Jm/3mttrexHh+6B/OGhhsZozS2bJXyPOSnIOsMfeoQPoPaX/aeXYtuDY+T25gYGBgYGBgYGBgYGBgYGBgYGBgYDBwmAfQBgYGBgYGBgYGBgYGBgYGBgYGBgYG6wLzANrAwMDAwMDAwMDAwMDAwMDAwMDAwGBdYB5ArzMYoxCMwiUEYAzMseASCsEYqGNBWBSwGOy8BWExuJSCMwq7YOPiO1V4lEJYDCxvgzMGl1C/HUYj7VOHQTAGlxAIxuDkrbBfwWhYXh0XgvI8qB9v0wvaEpafj+C1YCzI89tVbdT1pcKOjdNuc61mN+2upY5BOopFG4IxvHXxKgRjKBbtvvcxqDUbhC2DRCFmT2ET2iPXXuoKy1mhPjDHAqyWVsBmIT98fWvl5Qp2UJYGemaBOizULallVqxeO/2RkH3JeXby0asTpKb544uNSWNvmu52gu2wcD44o2COlZiHLPYYrA2qXkl+uW38oc+PwEcHHJV8geVzVKd9kbW1fO7KOCJXsMEcC5xRPyawGN69WvN9M6Wh34YlY45kXAAW7VuOs50tjFHkCnaEc/mi3ZN+G189HGgXk6naJ7kkeSW11spbEY31tTzJJ/k+Xk6uuxxHGGMqPJbjStt70lfo6gA+16jDYId+goAH8biVt+AREsTjrBWrdxnbyn4MpzcX1nrOYI4VarGwWOreoAFHfS1vaejiUhV23g7Pg35bAQ8tX1+pw0AU/yFjHCtvp5751P2HIPbhyntb2T/dxiJrmTsDA4PhxqidlXvBKJyv14JBcMBcQriOYIxiqdrE0ePzWFxaxfREAfv3zuCLX30VS8s17N87g5zD8Ocv/BAP3HMLnvn6a/ju98/jjtu2Yc9HbsETJ/x6/vsSnjixELZzeK6MyXEHl5Yb+OYrP8YHZ/5BJP/gXBnfeuXHeP6bP8L0RAGH5sqYKNrwPP+G+3eqTRyJjWtiPIfxvIVLy43ImD/5wO1wHIo/+8YPce8H34ennj0TGYdtUzz2+e+EaWpfKmyH4eJyA48rbR+cK+PGcSdxu6Zu7g7NlXH99em3DafV0Y3FoDOKRRtvLdUS63XTRL5vN+IOas0GYcsgUSjaeFtjz3sm8ljdJPbItX/m66/h3g++D6dffTvUsYnxPB766K04drKlNY/smSU27SEAACAASURBVMFXXn4d/93Hfg4r79YTtp8M9FPq2Y3X5/Hu1Qb+4E9PY3FpFfd9+L340O03Z9IfCSdv4YKmr6nrcmjU3FDTTgY2qNp4cK6M6evz4c3J7XR3zGZt+W47LKHLn973fjBCI2md7DFYG1S90nH0sU98AM0mj6zxwbkyfvCji7j1p28MfbrK51/+0PvCmEBqH7NpgncH9s3ia99+A+NFGx+5YzuWluth33fctg1777klUj7e/vXjTqKMjEd0eTpbDj9chtX08LuKv28Xa2SZR+OrNw7tYjJCSYKDn973friuwGe/dDrCy2e/UQm195E9Mzj96ttarVU1WvLzgXtuwY3X5XDh3bpWR2W5vffcgrGChd/53F9F9l47X7H3nlswdV0OV6pNUE5Qi2mo1N9v//Xf42ffe0OkjcMPl2Fb2WJbAOBcGE5vMvSiQ3Ef8Ju/8nOo1T18+Vuv4x9/+Gfwh8+8ktgb7yw38PC9t+H5l87i4//NrXBdHjmvHdhXRrHA0GhwLFcbER4emiujWLRw+Z06nnz6dIS7121x8O9feh3L1UZCv9X9pjtD6mKmLLHIWubOwMBguDFqZ+VeMArn67VgUBwwf65cR7hA6JwBYHFpFcdOnsH9d+0MXy+vNLB7djuOHp/H7tntAIDds9vDg2rr/UKknSPH59Hw/PbvLu9I5D8epMv3R4/Pw1XGdUQzrvOXq2Gbat4fPvMKrlz1xykPBuo4zl+uRtLUvlQ0PISEVsepe1aim7ujx+fx7kq9q/lOG4tBZ6zUuXa9Vur9Cy4HtWaDsGWQqKbYU91E9si1l7qi6tj9d+0MD2GAb99Tz57B7tntcF2htV3qp9Qzz0P48BkA7i7vyKw/EvWmvq960/9DmNQ0nTY+fnwetUZrPdrpbie+63T5ytVGIq2TPQZrg6pXOo6ev1xNrPHjx+dRvu2miE9X+azGBFL7dLx74sQC7rtzJ+4u78D5y9VI37tntyfKx9vXlWmXp7PlyBfmsRjz9+1ijSzz2E09g/6iXUym4+CVq43w4bNMe+LEQkR7pZZ30mjJz6OBnqbpqCz3+PF5uK5I7L12vkJq9fnL1cgf62Q5qb937ro50caRL2SPbQHg3ZW64fQmQy86FPcBV676D4x3z24PHz7LtuTeuP+unXjy6dPYPbsdFqWJ89oTJ+bBCMWld1cTPDx6fB5EkPDhs0w/dvIMLiyt4r479fqt7jfdGVIXM2WJRdYydwYGBsONUTsr94JROF+vBYPigPkE9DrC4yJcQInFpVWMBx9lX1xaRd6xkHei6eNFO1Iv/l7W9TjH4tIqGIU2X/02lF9ewGozrrxjhW3q8uQ4dXnJsYkEudLa9jjXlNWPselyEOiRVkc3FoPO6Ga9eu9jMGs2CFsGiVGwR6691DdVx9I0b7xogxK9Do0rXxFaXFpNlEvTyXZz1mmeZX47jbbCttrpbnu+68aRd6xNz4HNBnUNdWuetiZCpMcC8ZjA4wKAvrz06fF+2u2XePu6Mrq8NFt0/j4t1kiD8dXDgXb6Jl+rSONEXHvTtDZeTvKzk47KdBoEf2q5TnU8zpF3rNQ92CkvOTd6jjbdtLk0nB5W9KJDcR8g67Tjofo6LX4RQnTtP/KOBUbb7wE5ziz7MUssopsHtQ3DdwODzYtROFuuFdf6HAzKfvMJ6HUEowTTE4VI2vREAcvBR9inJwqoNVwsV5uRdPleIv5e1mWUYnqiAI9Dm69+C8ovT9qOq9ZwwzZ1eWnjqDXcRJrsKzof+rYZTdIwbYy2lU7ZtDq6sRh0Rjfr1Xsfg1mzQdgySIyCPXLtpa6oOpamNcvVJrjQ692y8tWg6YlColyaTrabs07zLPPbaXTc3ngZX3fb8103jlrD3fQc2GxQ11C35mlrQkh6LBCPCRglqbzzuM/jeD/t9ov8v12ZbmzR+fu0WCMNxlcPB9rpWzeaE9feNK2Nl5O866SjMp0Hv8CmlutUh1GKWsNN3YOd8pJzo+eobaXNpeH0sKIXHYr7ALknsuhru/iFENK1/6g1XHi8/R6Q48yyH7PEIrp5UNswfDcw2LwYhbPlWnGtz8Gg7B/IbJZKpd8olUr/Ufn3bqlU+lEs7UKpVPo/NXX3lUqlt5RyRwYx5n7AAnBorhwu5PSE/ztbz714Nnw9Pubg1MI5HJor49TCOQDAqYVzOLCvVc9/Pxtp5/BcGQ7z239h/s1E/sEgXb4/NFcO/3JhATisGde2yWLYppr3yQdux9Yt/jgf2TOTGMe2yWIkTe1LhcPg/y5qbJy6e7N0c3dorozrxnJdzXfaWAw6YyxHtes1luufbAxqzQZhyyBRTLGnuInskWsvdUXVsedePIv9e6Na88ieGZxaOAfLIlrbpX5KPWMM+K1f2xWWe2H+zcz6I5Gz9X3lbP+QJTVNp40H58rIO631aKe7nfiu0+WtW5xEWid7DNYGVa90HN02WUys8cG5Mua//1bEp6t8VmMCqX063h3YN4vnXzqLF+bfxLbJYqTvUwvnEuXj7evKtMvT2XL44TKmY/6+XayRZR67qWfQX7SLyXQc3LrFwac+vivBS1V7pZZ30mjJz0OBnqbpqCx3cK4MyyKJvdfOV0it3jZZhCe4Nh7fNlnES6d/nGjj8MPZY1sAuG4sZzi9ydCLDsV9wNYtDvbv9fn2yQdu1+6N5148i0cf3IVTC+fgcp44rx3YV4YnOG64rpDg4aG5MgQRePTBXQnuTk0U8PxLev1W95vuDKmLmbLEImuZOwMDg+HGqJ2Ve8EonK/XgkFxgAiRfqnbeqBUKt0G4N8D+EeVSuVikPYeAN8G8F9XKpWzsfL/C4C/qlQqz3TZ1Q4Ab1y6dBWcD9ZGFYxRuPC/rmRRCsYI6k3uf8qJEXhcgMD/9ETD5eBcgBCCnEPhuhyuJ0ApgWVRuE0etEPAIMILBV3Ab8vz+2GUIGf7/cj3FhC5GIIxCg8ELhegFLAYAfGibXIOUEoA4rfBPUAQAQgCLusF7UkbdX2psB2Ghud/xJ9RCoch9cIsde5ku5OTY7hwYTnTfHcaSy+YmhofxJ/3d2AIuAv4P0a/Um/xaCxH+34RwXqvmcQgbBkkCkUbVcWeYo62vSBhGLkr1x4EgCCwLALXFfC4QM6m8LiAywUYISAUEMHXO5lNUW+KUEfyDkW9qeglIwD8/nmgi5QS2DZBsynCOWunPxJO3gr6amlro9b6ZJzUNH98gCf88eYdGl5AqNqbprudYDsMTY7QRpsRMIrIPGSxZ62Ymhpvq8Hr0N+gPlK1Axm4q+qVY1NwD3A5D7ULiPpDhxE0PBH4+yA94DMEQCnQaPKE9kV4xwhsi6JW92DRID7wBJqeAOcCFiNwbIp6g4MLAUp8v01AwChBvemBURqJCyxKQVmrbzlOVYfjtlgALIeh1uAh5wo5itW617V+D0r3B83XWN9Dp7lxtIvJVA5azOeSyzkISKBDgGNTuEEZGS9CIMEn+V6NKyEQrrscB2SMqfAYwtdq7gnt3svZDB4PxkBI4E8Q2sIYhWAEjFE0FT9hM38f1uscduBvPK8VYwPZY9upqXFcvrwyEE4PApuBu1nRTgN60SG1Ts72/+Lb9DgIISCAdm/YNgXnAgL+ea8pz2eMwLEC7nEOi9KIrucd/2wo4J/JPCXGoZSgVncT2h7Xc4tSWDZBo8nBOWBRApsBTQ+psUgW3RyUhqvIMq5h4+7U1Dju/dRfZG74K5/92Ib5rKzoxq92az+wcXOQkV/rYs+w8HbUzsq9oNvz9aihWw70wt2N+GPl5wAckg+fA/zPAP5t/OFzgFkAO0ul0iEAfw3gf6hUKksDGGdf4Hn+bxZb/ht4nnztPxiRK9Z0PRAA8sNrzVWvJYQc8IJ8WdeLtS/bkvmNyHuB+CMJGSSoY4mPmQV5ACBDCjlemecp6Wl9qWg2vMh8NNsUjs5d+3bXUscgHdVqEwTATQEXq9X+z+ig1mwQtgwSqzF7VjehPXLtW+9bWuI1FI0CgCDPQ1JX66teq2xMz4CWXrkxnWynPxKNmhuZ5/jzXalpUNsGwjHF7W03znaQD4VCHxDob1Y9NegPVO7xyJoIrT9sBrzgMX6ovlWnfZJ3FgC4gOt64dpLbtHgH1zguokiLlxYRvwD8B5a/IjGBR64lxxnO1s8AN5qdO/Vql5P+m189XCgXUwW1z6pN0BLh9xA09R4EUCCT/J9vJx8GepogDiP5bh0e88LfiojrY7ncV9vkdRQWY83eMQ3dBvbyn4MpzcX1nrOkNxTPxum2xs8iGcIfC2X9acmWg+8/PiGR3S95rZGpPKTe6rviGt7XM89uJ7iL5TYp5dYRDcPhu8bi/GtBeRz3T3SqdVdLF9Z7VzQ4JrCqJ2Ve8EonK/XgkFwYKAPoEul0t0ACpVK5c+VtJ0A7gTwGynV3gLwBwD+CsBRAP8GwMez9nnDDVt6He5QYGpqfKOHMHS4VuZk2Lg7SvM+SrYAw2fPsHG3Xxi2ed4ojPI8bGbujvK6rAXXwrysN2+vhTlcK8wc9YZBaO4wr82wjs2MqzPWi7u92NjLp3Lza5jL9V6HjVrn9ep3M/J2mMa8UbjW52C97B/0J6D/KYAnY2m/CeB/q1QqdV2FSqVyn3xdKpX+NYDXu+lwGH7GoFds5FdHhxUbPSeDFKJh4u5Gz3s/MUq2ANntuVa52y+MGm96xQb8BMfA+gI2L3cNP/XY4J/gGFhf68lbw63OGLU5GhXuAsO9NsM6ts08rmHjbi/j6Xbue7W51zXu9ic4esEw/wRHL9iMvB1GDRgkrvU5WM/nCwP7Re1SqeQA+DCAL8ey/jGAkyl1riuVSp9Ukgj8n2QzMDAwMDAwMDAwMDAwMDAwMDAwMDAYcgzySsefB/DDSqWyIhNKpdKN8H+S442UOlcB/HapVLojeP/PATy/vsNcGxijsPM2BGMQFoWwGDxKIRiDk7fAHAucUbiEAJafBsbgEgLOKKjDwJi/LJwLiKCslbcggnKCMVCHQTAallX7l3UEo377Vqt95liJOmn1mePXFRaN9G3lrbB9tS3bYZFythP/Rb7k+NqNxSAdg5rHYtHn8lsXr0IwhmLRXpd+BoF8zJb8JrYFGI21YYHmCYvBJT6nqeOvjbBUzbGRK9gh56nDIrqm6qbcG1K3PEoTZfptg2AUXjBWv9/WnozropO3etq7jNGEr7ADP5ClrSz63A6y/lsXV3qqfy1A1WXqsHC9JF9lLCAsfy3l+kH1sRYDc6yATzTgS3Td4vuGM4ZL764mOGHno/soV8imEd34F5VX1GnFG91yO62/XvMMskMXN7a0jOH85RXAYiCOzzdi+XGgGssyx9LqcZb+GKO+nlitPUCVPdCvdU2zsxttzsrHeMyOYM9beQvEYcHc+WmGtxuHRLwQW1edf1e1XZbtVI6p2mgx2AUbLiG4fKUW0Wwrb0U02ynYiqb673X8ifNStukF+9PfuyQ8n/YaBxgYGIw2RuFsuVb4583WHGSNnUcFhRgHCuvAgUH+BMd7AfwkQxpKpdIfA/hypVL5cqlU+lUAnyuVSgUAPwTw0LqPtEcwRlEXAlffWcWz36jg3g++D089ewaLS6uYnijg4FwZlkXwr/74u1hcWsUdt23DA/fcgqPH58My+/fOYGI8h/G8hXNvX8Hv/8l38fM/cyN+6Rd+Gk+cWAjLHdg3i5fP/AQfvv1mTBRteB4HYxRL1WbYXrv2x2yWuK1YrT8xnsdDH70VX/7W6wk7Duybxde+/Qb+5u8u4tBcGRNFG5QRXFxu4HGlr4NzZdw47oSXZ8XHNz1RCOtv1pvCNwKDmsdi0cZbS7XEmt40kd90N+LmizbOa2zZNpFHbZPZAozG2jBGsdL0cOWdJp58+nRox6cfmkU+x/C7n/9ORHO2FG385bffwA/eWMLD994WqaPq5qXlBp75+msJ3WqnfWuxIb4XH9kzg6+8/DoeuOcW3HhdDhferSfW6Vuv/BjPf/NHmfcuYxTvVJs4orTz6IO7MD5mR+YprS3bYR31uR3WWv9agM5/Hjt5JvJa5cjpV9/Gh26/GSdTuPrFr76KpeUaDs6VcfLrr+G73z8fzvvWLTYuXFpN7IF8jiHvWPjiV3+A8aKTiBsOzpUxfX0e9dV0jejGv6i80NmZldtp/QHoKc/EE9kRn//7Pvzejry8ftzBno/cgidOzGs5mzXOlHUf+8QH0KjyRKwq2+vHuuri47333JLQNHWvxfttx1XORZini9nlnt9d/k/w7tVGZJ8cnivjesPbgUOupy5eODRXxg3jDi4tNyLr/dsPvR+uKyLa+9gnPoBmk0f88ycfuB2OQ/GZE9/TauPBuTK+94O38Y35/zfkna/Z7w33lY6jh+bK8ATHZ058r8Wfh8uwLYrHglhAV2//3hn84EeXsOvWbQmfYPy4gYEBMBpny7UiV7Cx+E5yDjrFzqOCQtHG2xoOvGcij9U+cmBgf3avVCp/VqlU9sbS5iuVygc0ZX+jUql8OXj9cqVSub1SqdxaqVQ+VqlU3h3UmLuFC8B1BZ44sYDds9vDYAYAFpdW8fjxeTBCw7Tds9vDwEaWOXbyDM5frqLhAb//J/6D6vvu3BkGDLLcEycWcHd5B44enw9/k8QFIu21a1/3OyZq/fvv2oljJ89o7XjixALuu3MnFpdWw/4bHkKyqvaqMU18fGp9g+wY1Dyu1Ll2TVfqm++QtJpiy+omtAUYjbVxAZy/XA0PcoBvx2e+uIDFy9WE5riuwN3lHbj/rp2JOqpuHj0+r9Wtdtq3Fhvie/GpZ8+E2ltvCu063V3eEb7PsnddIDzcynpPPn06MU9pbWXR53ZYa/1rATr/GX8NtDhyd3kHHm/D1fvv2hnO8+7Z7WHe48fnwT1o98CVqw0sXq5i9+x2bdzw+PF51BrtNaIb/6LyQmdnVm6n9ddrnkF2xOcxCy93z24PH5LF87qJM2Xd85er2lhVttePddXFxzpNU/davN92nHt3pd56iK/Ze3LPXwhsU/OOGN5uCOR66rh+NPBv8fVeXmkktPf85WrCP//hM6/4epyijY8fn8edu26O8M7nTXuOHj0+H7Yr0458YR7nlVhAV+/YyTO4c9fNWp9g/LiBgQEwGmfLtaLW0M9Bp9h5VFBN4UC1zxww3/vqIzwuQIm/WONFO1w8icWlVVDSep9WJu9Y8DgP8xiFtpxM94Ifkve4iJRr337yx+fV+rJuWhvyG4Oyf3W8ajmPtwgbH59a3yA7BjWPWdZ0s2CUbAFGwx6PC+QdK1Wj4mmU+FqYRTe71b612KDrR/aftk7qN66z7N20fnTzpNf2tfFlFPi23tD5z/hrCdV/p+WPB5/yVV/L90Kk8yHvWBgv2qlxQ6c168a/qLxIs6NXbvtxRW95BtkRn8csvMzC2SxxpkSaH1DbW+u6Zo2P43tN7bcd55putpg9zVbD28FDrme6diX9nm79OsUxae0LIcLXOs1uF8e0S2vXn/HjBgYGaTCxvpmDQdnf1QPoUqn0sb72PmJglIALYHqigOVqE9MThUj+9EQBaoyZVqbWcMEoDfM8Dm05mc6Cp9qMkki59u0TxKHWl3XT2pDfFJT9q+NVyzGq/D5ebHxqfYPsGNQ8ZlnTzYJRsgUYDXsYJag13FSNiqdx4WthFt3sVvvWYoOuH9l/2jqp37TOsnfT+tHNk17b18aXUeDbekPnP+OvJVT/nZa/HHzVTX0t3xOSzodaw8VytZkaN3Ras278i8qLNDt65bYfV/SWZ5Ad8XnMwsssnM0SZ0qk+QG1vbWua9b4OL7X1H7bcc62ssXsabYa3g4ecj3TtSvp93Tr1ymOSWufEBK+1ml2uzimXVq7/owfNzAwSIOJ9c0cDMr+bls72tfeRwwWAMsiOLBvFqcWzuGRPTPhIsrfUPEED9NOLZzDoblypMz+vTPYNlmEw4Df+fU7MD1RwPMvncWBfbORcgf2zeKF+TdxaK4c/pC3BUTaa9e+7se/1frPvXgW+/fOaO04sG8Wz790FtMThbB/h8H/jZyYver9FvHxqfUNsmNQ8ziWo9o1HcttPhEupNhS2IS2AKOxNhaAbZNFPPrgrogdn35oFtOTxYTmWBbBC/Nv4rkXzybqqLp5aK6s1a122rcWG+J78ZE9M6H25myiXacX5t8M32fZuxaAw7F2Hn1wV2Ke0trKos/tsNb61wJ0/jP+Gmhx5IX5N3GwDVefe/FsOM+nFs6FeQfnyqAM2j2wdYuD6ckiTi2c08YNB+fKyDvtNaIb/6LyQmdnVm6n9ddrnkF2xOcxCy9PLZzDgX3J2FJyNmucKetumyxqY1XZXj/WVRcf6zRN3Wvxfttx7rqxXJin23tyz08Ftql5hw1vNwRyPXVcPxT4t/h6j485Ce3dNllM+OdPPnC7r8cp2nhwroyXTv84wjufN+05emiuHLYr0w4/XMY2JRbQ1du/dwYvnf6x1icYP25gYACMxtlyrcg7+jnoFDuPCoopHCj2mQNEfgUoC0ql0kkA/w+AlwFclemVSuWVvo6qP9gB4I1Ll66CD/CrbYxRUJuh0eRA8IEGzgUoIcjZBB4Hmp4A5wIWI3AsgkZTwOUClAIWIyCegOdx3HDDFlx8p+p/VT3H0Gz6XztllIAxwPMELCBycQljFC4QlsvZFA1XwPUEKCWwGQE8nnrZiVo/ZzN4XEAQABzwhAAjBLZNUKt7YJRE+rcdhobnf3yfUQqHIXGxRXx88fF3wtTUOC5cWM5cvt+YmhofxMdUdqADd9c6j1lRLNpYqfOwn7Ec3bQXEeSLNlYVWwo5uikvIJTodm2Ghbsq/JvbfV30vJa2ORbFaoOHabZNwQhQa3jwuIBjU3COQNeiuin3BggAQcCFgH8RfKtML0jTHtkf5wClBCACEAj3ZFwXczZBvcm73ruMUXggEV/BADQ8kamtLPrcDmH9wA90W79XDIi3QB9iBlWXHZuCe4DLechXL4gFENDEZv76EUIgRDC3lMCiBI0mB6VAzqaoN0Vk3bgnWvsmaNNiBICIcKKQs9BwfX/PKEXeoZkuUenGv6i8cmwGzgVcj3fN7bT+es2T2MiYYRg1Vwdd3FhvikDLCAR8jhEKNJsCNiNgls9RzgMtor6uxfU4S38WAMoImoGms6C9pmwP/Ylv0uzsRpvTODc1NY7Ll1fCvDBmD/SS+NsTlk38n5Dx/LOBRQkYevdL64XNwt0saKcBiXhB+lcgEk+o/p1REmq75ACAtuXkeUry27EpVmsecg4DFTzU7HyOoemKMPbJORSNJg80lSLnUDQ9nuCP7F/y0mEEDU+Ac4Cx1t4t5Pzzaac4YKPPWmnIMq5h4+7U1Dju/dRfZG74K5/9WNdz320fvfaj9pe1bi9je+6J/xaO3d1fRmp1F8tXVjsXbIOM/FqXuR4W3o7Sub9X5Ao2ao3WHGSNnUcFhaKNqsKBYo62vYCwF+52+0f3O4J/v6GkCQDv7bbjUYUXPNxVV0JKqPTxNPgHF5DfmrIAwAO4EgdQSkA8DguAW3NBYuWI/zLRf6ucQCMIamU9r8PzArW+FwxO2hIGWV6rfbW5ZsNT+vbQ1PQVH5+5+6I3DGoeq9UmCICbAqdcrW7eFavFbKltYluA0Vgbz+NAoGWqttUiWgK4ngcXrXI8EFOdbqbpL1+n6ZH9sWAsYXrwf1wXGxF7s+9dL6bl3AM4sreVRZ/bQdaXfOu2/rUAVZdVjsrX8eNU02v513Bt/C0R8qkR0Xpl3VQeAbhh0l8XlRPNWjNSt76abdG68S8qr3gQM/TC7bT+es0zyA5d3Khq6HSw50VQRriA60ZjWQ96Pc7Sn4dWbGoF7XG09kC/1jXNzm60OSsfIzG7Ajd+DuijfQbdQxsvKGui8+/yTwVxDrQr56na6AJN14MFYHLrGC5cWNbzxgMaq2qs44XvdfxReSl9Cwv6k3u3WUvxJwYGQwTHZj095B2+P5lsPozC2XKtqK9G5yBr7DwqWI1xYHUdONDVA+hKpfLTfR+BgYGBgYGBgYGBgYGBgYGBgYGBgYHBSKKrB9ClUulGAP8EwBa0/rj6M5VK5ePrMDYDAwMDAwMDAwMDAwMDAwMDAwMDA4NNjG5/guPPAKwCuA3ANwB8BP7vQRsYGBgYGBgYGBgYGBgYGBgYGBgYGBhE0O2VhtsrlcpHAXwNwL8B8AsAbun7qIYcuYINwRhcQiAYQ65gB5dpUbiEAIyBOgxekG/lrVZ5i8HK22Fd5vjv4+0JxvDWxRUIxmArVxTbDgN1LHBG4RIKWAyM+cvIGAV1WCRPWBTUscLXgjF41P9fWBQsyFP7Tmtfhc5evw2qLW+wOVEsSi5ehWAMxaK90UPqGaNkCzBa9tgOi2iTlbcgFF1ieRvMsQI9Y2ChRhHAiuqPo+ptoJ+qXglGg/5oJs2SdReXqtqycuyyP1g0Uk4dD3UsEKs3rYz3Y+WthC9y8ul/U47PS7uyBtkQXxMd19Q1th2m9cWwWeiThcUgLAbOfJ7ToI5L/HZpkCaYH2P4/LcCfrbaVOvxIDaIjyU+dol2NrTbS/F4ggV5a4kJ2o3FYO2Ixo1+PCfXVPL07UsrEQ7FuSP5GNc+2b66fjp9zjJGXbxJHQYwfWxLAv57GWJU2b7aDmcUVt4CgrHKNnR9cUZxZaUWlpV9MEYTvkraa3g9/ND5XLmuUltDnlEaOfvRUAuDPRHotuSSlfd58fallVDD5RnR9wc0OO+19qbUV5V7Mv6BHT0bElt/dlsrOvkGfw/4e4IE4zfcNjDYPBils2WvuNbnoPUc8mr4bLDf6PYE+nbw/1kAP1epVL5UKpWuqVXJFWwsvlPD48fnsbi0iumJAg7OlTGx1cHBp/5DmLZ/7wy++NVX8bM/PYEP3X5zpPyBfbP42rffwN/83UX8q3/2j7Cy2sATJ6LtfeuVH+P5b/4ofH/jiKZk6gAAIABJREFUuAMAuFJz8c5yHcdOngnLH364jMktDpZrLpYuR/M+9fFdsCyCP/vGD3HvB9+Hp55t5X3ygdvhOBSfOfG9MO3QXBme4JG0ww+XcX3Bjtw8v1Rt4qhik7R3abmGQ3NlTBRb5Q02J4pFG28tJbl+00R+092IO0q2AKNlj+0wXFxuhLbccds27L3nloRm2jb1b4ZnBFeuNiI6p+rPwbkyTn79NXz3++cxPVHAv/zEB+C6HEe+kN5+mmbptE4tGx/79EQBj+yZwVdefh0P3HMLpq7Ph/5iYjyPhz56a2TcWbVS18+//MQH0GzyyNgOzpUxdV0OjZobqe/kLVx4t57gi66sQTbo1uTQXBmOTfHY57+TWGPKCC4tN/DM119L+GLJmXs/+L7I/7/6kX8IRmhkjR99cBe2FC383h9/N0z79L73o9Hg+MNnXgk5vucjt0Tiiv17ZzAxnsOYzUAZSYxdxhncE6mcBxDJU/eSjt8H9s3i5TM/wYdvv7mnmKDT/jNYGxijWGl6ibjx0FwZ33zlx9h163siPN2/dwbvubGIKzHuyHXedet7Qu3T8eW+D783EQ9L3jUb+ktu0uLN//t7P8Yvvv9mfPlbryf2028/9H64rsCTT5/uGKPK9uP70t9DJTxxYiGy9yyL4F9/8XuRvXv61bfxwZl/ECl7eK6MQsHChQtXE3N743U5XHy3bng9xNDp+4F9s9hStOF6HI/90Xe0mqdyM56ezzH80b/7W1w/7iS4pfJTjSHkeUzXlyx33507USxYWF5pJM6G6tltrWinxwDwTrWJI7F9ms8xFBwLYzYz3DYwGHKM0tmyV1zrc5D2nHP6+jzqq/2zv9s/Sy6WSqX/EcACgF8vlUr3Aij2bTSbALUGDxcFABaXVvH48Xl4LiJpx06ewf137cTd5R2J8k+cWMB9d+7E4tIqPA/hIVFt7+7yjsj7hgc0PGDxcjUMMGT+kS/4+ec1eZ/90mlcudrA7tntYWAt8/7wmVdw5Wojknb0+Hwi7cgX5qE+onCBMACJ2yvbMI80Nj9W6nqur9Q3XxA5SrYAo2VPw0PElt2z27WayQjF0pUaGKEJnVP15/Hj89g9uz3MW7xcDR8+p7Wfplk6rVPLxse+uLSKp549g92z23H0+HzEX9x/187EuLNqpa6fxcvVxNgePz6PelMk6tebQssXXVmDbNCtydHj8zh/uapd44bnc0nniyVn4v9fudpIrPGTT5/GhaXVSNqVq43w4TPgczweVxw7eQbnL1fDsej40PDacz6ep+4lHb+fOLGAu8s7eo4JOu0/g7XBhT5uPBrEoHGeHjt5BlzDHbnOqvbp+KKLhyXv2o1RF2/ed6fPN91+Wl5phA+f1Tq6GFW2H2/H30MLib23vBKNj5969gzuLu9IlD1yfB6uK7RzW28Kw+shh04jnzixANcVOH+pmqp5Kjfj6VeuNnD/XTu13FL5qe4jeR7T9SXLPfn0aVg0GRfFz25rRSffcESzT69cbYR+x8DAYLgxSmfLXnGtz0Hac85ao7/2d/sA+p8CqFcqlf8A4HsAfg/Ap/s6oiGHx3m4KBKLS6vwOE+kjRdtMAptefmNJEra56vte5wj71ip/afl5R0L40U7NS9LmsdbDyo8LrRtjQd/BY+XN9icyMr1zYBRsgUYLXvitqRpFSVA3rFSNVPVn3Hl61JxXUxrX6dZaVony6atg+xDze+m3+Q4kv208wVZ6m9WvgwL0uY0zX/K8mk8kOnq/+18uoqsHM87VmQsyXHytpyP56n9pPUpY6BeYoJO+89gbfC4SOVYWuwqhH5NZPmW9iX5ktZmOx1K40C8PxVpNuliVNl+vJ1uYuY0u9J8Vfr+M7weFqStkYxDgM6aF0+XZ7F2PkB9rfKtk99I51r/ONWNb5B5eccK/Y6BgcFww5wVzBwMyv6uHkBXKpVFAJ8vlUr/KYCDAH6hUqk839cRDTkYpZieKETSpicKYJQm0parTXgc2vLym0hctM9X22eUotZwU/tPy6s1XCxXm6l5WdIYJcocEG1by8FXE+LlDTYnsnJ9M2CUbAFGy564LWlaxQVQa7ipmqnqz7LyNam4Lqa1r9OsNK2TZdPWQfah5nfTb3IcyX7a+YIs9TcrX4YFaXOa5j9l+TQeyHT1/3Y+XUVWjtcabmQsyXHStpyP56n9pPUpY6BeYoJO+89gbWCUpHIsLXYlRL8msnxL+5J8SWuznQ6lcSDen4o0m3Qxqmw/3k43MXOaXWm+Kn3/GV4PC9LWSMYhQGfNi6fLs1g7H6C+VvnWyW+kc61/nOrGN8i8WsMN/Y6BgcFww5wVzBwMyv6uWiuVSh8A8DqArwL4KQA/LpVK/0VfRzTkyDvU/y2UYHHkb6MwC5G0/Xtn8NyLZ/HC/JuJ8gf2zeL5l876C8qAA/uS7b0w/2bkvcMAhwHTk0Xs3zsTKX/4YT9/mybvUx/fha1bHJxaOIdH9kTzPvnA7di6xYmkHZorJ9IOP1yO/Fi4BeBQzCZpr2zDXG+1+TGW03N9LLf5RHiUbAFGyx6HIWLLqYVzWs30BMfE1jw8wRM6p+rPwbkyTi2cC/OmJ4s4/HD79tM0S6d1atn42Kcn/N9lPLVwDofmyhF/8dyLZxPjzqqVun6mJ4uJsR2cKyNnJw96OZto+aIra5ANujU5NFfGtsmido0d5nNJ54slZ+L/b93iJNb40Qd3YWqiEEnbusXBJx+4PcLxeFyxf+8Mtk0Ww7Ho+OCw9pyP56l7ScfvA/tm8cL8mz3HBJ32n8HaYEEfNx4KYtA4T/fvnQHVcEeus6p9Or7o4mHJu3Zj1MWbz7/k8023n8bHHDz64K5MMapsP96Ov4dmE3tvfCwaHz+yZwYvzL+ZKHt4rgzLItq5zdnE8HrIodPIA/tmYVkE224opmqeys14+tYtDp578ayWWyo/1X0kz2O6vmS5Rx/cBZcn46L42W2t6OQbDmv26dYtTuh3DAwMhhujdLbsFdf6HKQ958w7/bWfCJH9azGlUull+D/D8aVKpTJTKpV+CcDvViqV2b6Oqj/YAeCNS5eugvf5qz+5go1aw/9JDEYp8g6F2/Dgwv+KkkUpKAOaTQFKCWyboNn0v6LEGIFtUdQaHhghsBgBoQTNpv/VV0YJ8g712xcCjBA4DOEFLbbD4IHA9Tg4ByxGwIQIL1MRjMD1BLgALEogIMAoBecCAgIQBFwIUEIA4o/V4wKu17Kl6Qlt+yoYowl7G00ORgksYN0um5iaGseFC8vr0nbG/gfxxGYH1om73aJYtLFSb3FzLEc37Y/wj5ItQPf2DDN3bYeh4SHUJtumaLrcv3SQElg2BbiAy32NISBoKhpFaUt/cjZFvSlCfXYYwD0R6hWjBA4jaHgifN9Os6TWgRBAiERZOXap136xVjknb4XjcWwGwQWaXvdaGfYTjNm2CRghEV+Us0nqpYLqODqVbYdBa/CAeAv0wN3WmqRzTV1j22FockAAgGjxnVBAcPgfCRAAiP/eZr4P5xxwg73AGAEg4HkAD2IKSgk8IQDeapMxhPXo/8/euwfJcZx3gr/KrKp+gANyQM6AWIsCRJluitxzeATOyGGvJC5BM+Ks4Gm5DB8halcY6ST7vOEjwtSGhYfWljdEgrZX1ILesCKWWguQLRG0j0fLPMpxNEFT0skhDUTCL1JscUUSS50gDB5DcoB+1uP+qM7qrKzMenT39HT35C8Cga7K1/d9+eXv+7Kmu5IE68Rw/Ygsouwsz+Dju6iDWMavpYJFO/lEpx0NcpV+coIkWbJgPXOGUeZchkje6AV5o0WBlusDBsKc0TC6PkSoEeEiahpwHT/Gfax/fv5k/Kw6gJCXUZZv2haB5wK+Iea2AOmsKRZDknLULsdH9bUtAqcdxB02ruvFx/J9H6UCRbPphTEqfOBGg5za83yY1IBFAn379eu1xjj4blb0ygGymOu3gzlyjWDNWJYR+JkXxH+297OsYN/Fc6EPH6124EumZaDV9uD7wTeLCTHQcjr5jQH4ng+bGnCBcG0WLBK+7oL5noHA1z0fIAZbxz4s0wDx4nu3fm2WFhtcGHC8YO9LaSAfH3f6RZa5HDXfnZmZwu2f/Frmjp/43Adz+2veMXodhx8va9teZRumPgwZ/WtNZBsVv520vXIv2Og26D7n7D6XTDqAsBffzftHyXK1Wn2xUqkAAKrV6tcrlcp9eQcddzTrbRjoGM910awHyTN/z3MBCgAu4LhcmQM4jhsa3u3k3d22QLPuwgCwrUOEbS43Z4k66fyDA7hhXx7gcmVe0K8HLxwD6MjF2nTKRF1k/fNwXS+mb/DZl9bXGE/Uau2IL9Zq4zu7k6QLMFn6tFsB5zFuclw3womu2+VYto3hOcpDl39aAje1Yxzro+1Gr5Msx7iOJaZiXSY7H0wNdHmz1XDCcq/zc9peuDIyTieuOIjGnaTnOLwcaXU1siE6J3Jfc4X6rBzgYnHHH/nKBrr5AdCdd48bgwKh/892/JP1yeqJ7dJkDy49pQ5iGb+W3Ih/B/Lza6EXJMmi0T9ieaOLcE4Zru74FvMh143mrB5XX5xvcf5U/Jwmoyzf9FTrCZ0/8qCbhyflqKx/8G0AOG40P+b15McyAGzetAlna6vxMVwvEqvYFk779ehDFnN5mAD8VvCZ510KwGvxvtPlcuZLTmfNsbwizGE6CY4BhGuD+Y/b6VP8wYDH/c/q+u3+eFeFtNgAdHX23e461NDQGA9M0t6yV2x0G7DnnEx/9mxwkMj7fep2pVKZRiemVNiTaA0NDQ0NDQ0NDQ0NDQ0NDQ0NDQ0NDQ0Beb8B/VkA3wCwrVKpPALgNgC/OnCpNDQ0NDQ0NDQ0NDQ0NDQ0NDTWFVObSygWgkdHMzNT6yyNhobGuCLvA+iPAfhDABaCb0//x2q1+v2BS6WhoaGhoaGhoaGhoaGhoaGhsa4oFsye3n+soaGhwSPvA+jHEHzjeRuA/wrgTJZGlUrl4wB+g7v1DgB/AmATgH8B4FLn/u9Wq9XHhbY/B+CLADYD+CaA/71areY/OWmASDqAr1SgqDeDQ6FMSoKDJdpueMBfo+XC9XyUCiZajhccUmVR+L6PtuNx9TycPncJlFJYloFG040d+GOZBIbR7b9cIKh1xu4eBpT9oBcg+WCiLPYYxcNUNHoHexH/6XMXQSkd6xfxT5IuAFAqW6hx+pQLBPUx1geIH5QXHODaPcSMcY1tEbS4evxBsOwwHnQOplJx2KB4i3Fm97DZrkzNejtyaK1tURgG0Ha84FAqQkCR7YAembwAMuvQC7fLxl9eqQGUaJ5Hfh8SD2kyafBGWc/34XuA15kbPqewqYG2CzieF7axzSBH4A8zPLtSA7EpTGKEa6hcNNFsB3kGJQSlAkG96abKm0cv/gA3A4by0MNB2lFjcMjCK+JhRbFD+5hvIt3/HQSHZxJCYBjB/+ywtsBvAk7kuYlSAsMiaDt+6MvBQbXBS2ZNIDwYkfE+OxSWHRwXPyitu55UfqrSU3aQYq/g7c8OVRQPMswqr0Z/SDpgNY2XxMPp+f1Yl3vj12yfx8djsS+Rt/l9Zhqvi3E/POSeO4y42fYyc2+/eYSGxlqh1XZzfyu70XSw+lZ9jSQaT0zaXrkXTOL+Og9YDGL6px1C2AtyPYCuVqtfAfCVSqVyPYJvQ5+oVCp/C+APq9XqUkK7LyJ4iIxKpXIjgL8A8BkAfwPgfdVq9XTCsH8K4OPVavU7lUrlvwH4BIAv5JF7kKCUYKXWxv1HlrC8UsfsdAl7d8/hy09+H1dM2dh92/U4JClbWW1g/+ICjj31EqbKNn75F9+BB46ewPRUER/5wLtw+NjJsA2r990XzmB2uoR9e+bx9W+/itVaK7X/bz7/Ol58dUXa51VTdmKiYNkU51Zbkf7T2snscWBxAdNlSyfIY45y2cLplUbMH7ZNF8cuGE2SLkAQHH8i0efq6eLYBkm7aOLsm82YTi++cg43XnsVrpyycX61hRdeOYcbrr0qVm96s439D/2/4b177prDE9/6IXbfdn2MwwbFW4wzX1TItG26GPrd9FQRv/qv/zkaTTfCzQcXF3BFyrgyeQ8uLsCyCD7z8HdSdeiF29PG3+g8n9cmlBK8UWvjPiF+l4sWXM/D73/5e9Kc4kO3XR8Z4967d2Jqk4UvP/kibn/vO/HQo11f+vf/Zic2lYKyn37bFbjphqvDOX/PjVtj+YNM3jx6sbqPPPUS/tX7fxqff+T5iA7TUwVssmhu397ovjUsZOWVT3/sPbi8aIZ/BGRzLvpfmv+LY/3WR24CfOD3/yTq+1dMFbC5aKLdckEpQdP3cfGNJh442m27b888TJPgK3/1fez5wA1o1TypTPv2zOOysoUCDY68ka1BmZ8m6Snm6PsXF3DZpmJf9pftBT7ziZ9H2/Fw35fS5dXoD6J/ZuVLINisL78Rz8eYj8j6kvnQVVM2CCWRvsS2n/jgjZFcI0lOQg1p3BfH/ebzr+Pxb7wSaStDv3mEhsZawrZoT9/OXl0jecYRk7ZX7gWTuL/OA1U8m72iONCH0HkPIUSlUiEArgPwMwgeYC8D+KNKpfK7Gbv4AoADAGoA3g7gjyuVyj9UKpXf7fTNj7UdQKlarX6nc+sIgF/JK/Mg4QBhggIAyyt1HD52Enfech12zW8PJ0wsW16p49CRJeya3447br4ODxw9geWVOu685bow4WRtWD12/cDRE7jj5mz937qwQ9lnWn7QchHrP62dzB73H1nCun5FXWMguNT0pP5wqTl+m55J0gUAagp9amOqDwA0275Up4Ubt+H+Dg/d37mW1XMdRO499OjJkDNFDhsUbzHOVMnE+92dt1yHty62Ytx8X4ZxZfLed2QJZy7UMunQC7enjb/ReT6vTRwgfPDF6h8+dhJvrDaweqmlzCnEMR786nNYvlDDrvnt4UMxVvaf/rRbdvPOayJzLssfZPLm0YvV3TW/PXz4zOtw5kKtJ9/e6L41LGTllc/+8XfD+eDnXPS/NP8Xx1q91AofPrN7h4+dxPKFWshNDgDH8cOHz6zeA0dPwCQEu+a348yFmlKmB46egOP4QT+Qr0GZnybpKeboh44sYWW10Zf9ZXn7mQu18OFzmrwa/UH0z6x8CQCNljwfYz4i60vmQy033pfYVsw1kuRUxX1x3FsXdqTqiIT+9LPnwYJ9kzfPPw2NQWDS9sq9YBL313mgimeN1mD1z/UN6Eql8lkAHwXwCoA/AvAr1Wq1XalUNgH4HwB+J6X9rQgeKP95pVK5FsAzAP4dgDcB/N8A/jcAD3NN/hkA/tvRpwG8LY/MV155WZ7qqVhe6Sbm3Xt1THX+YpxWNlW2QEm33lTZSmzDrilJr5tWz/V9bEsIVKfPXcrdTmUPGMaaBcWNEmwH7bt5cfrcRbk/eMl+NIqYJF2A0denF99V6eT7fqgbfy3Wcz0vdo9xochhvfKWWMY4M0kmnuvDcXKOq5K3aJuxe7K+euH2LOOvJc+vF7L6bl6bZJ1Ddj/JX4q2iaKdXCb6pCovEOXNoxerq+q7aJs9+/YgfWvSfFSGXji3F15Jm/M8/l+0TeX4jJuWV2oghtzXiYFYfq2qB8PoyijTV7EOsuboTg+xl7eJbJwk+0wS9w4j102zleifefxblbswH8nqQ67vAym8nYfXXVeel8j2l2JbQJ3viP1lzSMGhVHy+7Xw3V6/ydsLRsmW6wXRBhvh2YXKb0d9bzkMbHQbDEv/vO+AngXwy9Vq9e/5m9Vq9VKlUvlQhva/BuDBTptXANzBCiqVyh8C+AiiD6AJAP7lcwaAXI/gz5+/GHt/XV+gBLPTpcjkzE6XsNr5Wn5a2Wqtjc2bCmG91Vo7sQ27dj2k1k2rRw0DZ8+qf2xCKc3fTmEP+H7iWL1iZmZqTfrNM/6wMHDfzQmlP5BkPxpFTJIuQG/6jLrvqnQyDCPUjb+O6x79QQ/jRimH9cBbMu5hMifJxHO9ZcrrpfKlQt5GK/pdJVVfPXF7hvHXiud5DDtpz+y7eW2ScQ7Z/aScotFy0Hb8xLKrrihFylV5QUzePHp16qr6brScnn17UL61njnDqHNuT7ySMud5/L/RcpTjh9xECTxfvg48HxFeVcnk+QB8P7zO5Kcpeoo5utlLLkGj8SGPfdaae0fed3MgEwcI/pnHv1XxlflIVh+ihgEIuYTYVsw1kuRkOVOW/aXYFsDg84gBIMtcjprvjtKDRxF5522UdekVvA3W0r/GwW8nba/cCza6DYb1fCHXKziq1eqvig+fubKnktpWKhUbwPsB/GXn+n+qVCp3clUMAOLLRX6E4MBDhqsB/DiPzIOGCeDA4kIQpBFMyt7dc3jsmZdx/MSp4D0pkjL2DpXjJ07h8Wdfxr4985idLuGxZ17G3t1zkTasHrvet2cejz+brf+nl15T9pl2UIpNEes/rZ3MHgcWF3L/ZUNj9LCpQKT+sKmQ+809645J0gUAygp9ymOqDwAULEOq09ILp3Ggw0MHOteyetRE5N49d82FnCly2KB4i3GmSibe7x575mVsvsyOcfPBDOPK5D24uICtW8qZdOiF29PG3+g8n9cmJoCDkvh9xVQRU5tsZU4hjnHv3Tsxu6WM4ydO4Z67or707/9Nt+zZ516PzLksf5DJm0cvVvf4iVP4zQ+9O6bD1i3lnnx7o/vWsJCVVz79sfeE88HPueh/af4vjjW1ycZv/dubYn4zu6UccpMJwDQN7NsTbbtvzzwcz8PxE6ewdUtZKVPwrujgcDXVGpT5aZKeYo6+f3EB01P53wHN20SWt2/dUsbBj2aTV6M/iP6ZlS8BoGjL8zHmI7K+ZD5k03hfYlsx10iSUxX3xXGfXnotVUck9NfPIZwaGusJ8XUnADb0604mba/cCyZxf50HqnhWtAerv+H7w/mGZaVSuQnAf65Wq/+ic/2zAJ4E8LMALgJ4AsDRarX6iNDunwD8WrVa/XalUvmvAF6uVqt/kGHIHQBeXYu/rPMnJZvCifX86cQmJSDEQKvtcqcPBycVlwomWk5wMrdtUfi+j7YjnFLMneLdaLqxU5ktk8Awuv3zpy4XLArX8+G4+U4q7uWE42GeYD8C34A2hjDMDqyR7+YFOw2Xze04n4Y7SboA3VN6mT5pp/SOg+/aRRPNdvfUecsy0G57IacwrrEtihZ3cnvRJnBaLhwAng8QwwAMH/ANJYfl5S0V9zDOtGyCdisqU7Pejpxob1sUhgG0HQ+eB5jEAIWfiS9l8gLIrEO/p9ez8WEYgO+vKc/zGJLfAj34bl4fopTAhQHH90EMAyYNVPN8H74HeJ1++JzCpgbaLuB4PggBTGrANoMcwQcAP2hPiRGUE6OzhjyUiyaa7SDPoISgVCCoN91UefPo1fULwIABx+3Kabi9+/agfGudvwE98pybhVeuuqKM8+cvxtrAAOAb8JhvIt3/HQCeBxBiwDCC/z3P7/hNsCYo/NihsYZF0HZ8uC6LDQRtxwX8YFxCjeBdtASAh0j+7HfWAOvLhRFZTyo/VelZsEi4xhiXXnF5uSc/4+1vWwSeCzieF5mLrPIOEuPgu1mRlQPEtcDvt9J4iY/z4n6sy72S646f8vFY7EvkbX6fmcbrYtwP95ed64JlBDFCaJuW7/SaR/SLjN9QHSnfnZmZyvVKjSc+98GeXsHRS5tevgE9LNkmrc24+O2k7ZV7Qd799aShG4Oi+1kVevHdYf4R/VoE32gGAFSr1X+oVCqHAHwbgAXgMfbwuVKpfB3Ab1er1e8B+DCAhyuVymYAzwN4aIgyS+G6Hgx0jOe68Fz22UejxpU5LjyuXrPuhmXtRjv87HV+7ijW29YJtA7Xf9tF2M5vu/C5dvVat39X6LOdMT9ot9yIblnaRe3hQ59HMTmo1doRX6zVxnd2J0kXAKgL+tTHXB8AaDUcjksAp8N3TDPGNe0I5wDNelDDACB+GUfFYYPiLcaZTt2VytSsx7medP7BReZxVfJm1aEXbpeNzzaA4+9t/SOvD7GHAizx8rgGoe+6iOQUbJ5MrqzR6vo7Ou1mrgzmpYWuT7Q434ProlFzM8mbRy9Wl4GXMyt0DrF+yMIrhBjSNgzMb7P4P+/nQPederzfiI/4XNcDBM53XDeUwQXgdmIF+PWCIIbE+kI2P1Xp2RL2AHm5VDZGEB9cTrboOuhlXWnkg7gW+P1WGi81Ba6t19wE7u1es/yN9yGxL5G3I/vMFF4X436zHr1u5dBR1l8/vq+xvmDf/tXQ4DFpe+VeMIn76zxgMYjpz/azg8TQHkBXq9U/A/Bnwr0/QnCYoVj3l7nPfw9gYc0F1NDQ0NDQ0NDQ0NDQ0NDQ0JhQDPOwQw0NDQ0eG+OFJhoaGhoaGhoaGhoaGhoaGhoaGhoaGkOHfgCtoaGhoaGhoaGhoaGhoaGhoaGhoaGxJtAPoHsApQQ+JXAMA3bJgk8pHMOATykKJUtZZhdNaR+gFMRm9Qgsm8KnBMsrNfiUgNJs0xS0645nFk04hoFC2YJvdu8Xy5ZUDn5sds2PbRfNsH+YFNQ2wz4tfQzyRKJcDnz49LmL8ClFmfOdcUNJ0KU0xroAQFHQpzhG+lBKQGwKYpvwKIVDCHyTwhI4k9gUHiUwiybAuKfDbQFPBe2ITUOuEjktK3+q5GR9raw2pH2xOm5HZt8kkXF5Xia2CZi0J9l4/mXxJI+ug7SLRgCVTcX4HskFzG4uUBD83SpZHX82O/4UnSfe12BSGDaFSwh8SrG8cimWg4i5xSDmXMwz+Ngv81GN0UDS+pflgcSmoJ35ZDHG5njYowTUNmEWTXiUhH7I+I/lkjJfFTmyV7kHUc7X43VjMaVX3syTW2eZG1V80RgNiHMociHL18S94vJKDTSSF9Aw7856LfJskj8l8Xcvemo/1NCYLEzSvr9XTNrzgrwYhv56d5ATlBKs1Nq61crXAAAgAElEQVS4/8gSfmnh7bjphqtx6MgSllfqmJ0uYf/iAl585RxqDUdaNnN5AW7bC/tgZXt3z+HLT34fK6sN7F9cwLGnXsJ3XziD2ekSDiwuYLpsJZ56bdkU51ZbkfH27ZlHo9nGpYYdk2PrdBHtphuTQzU2tQjOvtmM9CPKfNWUPdTTkDXWFuWyhdMrjZjvbJsujt2JuKWyhZ9IdLl6ujiWJ9sWyxbOSPTZOl1EY8T1oZTgUttFs+ah1mjj8LGTSv7Zt2ce33/1PN71jivxwNETEW579K+rYb17796JzZssTBVNnF9tRTgtC3+q5BT5UexLVueeu+bwxLd+iA/ddj2uurwQ8ub0VBEf+cC7Ivpmlc0umjH+3b+4gE0lE5/+wt+m9pdFF418UNn0yik74oPvuXErdt92vZRHZfz64ivnsOOfXR7GVjZPABLzhnvumsNz3/8J3vfua5Rxut85l+UZLPYbxJD66MzlBbQaziBNr5ETSesfiPvV73zi5wEAzZYb4V2Rnz+15yYQg0TmPIsf8hyZ5I9pvNVvOT/OG7U27hPknZ4qAAA+8/B3cvGmOK6MA5Lsn1SW1XYaw0OW+ebXDl8u5gVpbfk48fDXXojxbJLPE2oo+TvL3k3nERoak41J2vf3ikl7XpAXw9Jf/+kyJxwgDL437+wm2ACwvFLHoSNLWLhxm7Ks2fYjfbCyw8dO4s5brgvr7ZrfHpbdf2QJadu3lovYeA8cPYGtWy6TylFvelI5VGM3236sH1Fm/ex5snCp6Ul951Jz/BLNmkKX2hjqAgB1hT71MdDHAXDmQg1vrDbCTRcg558Hjp7Awo3bwocg/H2+3oNffQ5nLtTQcuOcloU/VXKm9SWr89CjJ7Frfjvu7/A9m6c7b7kupm9W2WT8e+jIEhzHz9RfFl008kFlU9EHd81vV/KoKn/gYyubp7S84aFHT+LWhR2JcbrfOZflGSz2q3y02fb7GFFjEEha/7Ky5Qs1GDBivCvy81sXW7E5z+KHPEcm+WMab/Vbzo9zn0TeMxdqOHOhlps3xXFlHJBk/6SyrLbTGB6yzDe/dvhyMS9Ia8vHCf6a8WySPyXxdy966jxCQ2OyMEn7/l4xac8L8mJY+usH0Dnhet0Nv+93PzMsr9Th+76yzPW8SB982VTnGw/852675E2c63nK8XqVgx9b1Q8vs+ttjMW5UZDkO+OGSdIFGG99XM9H0TZRtM1M/KPiUrFe0TYT7JL/IZiKH/m+kjhU5F92rxfZVHoRA7F7sv6y6KKRD2qbRudKPe/yOWX+Ho2tfqa8gRJkiNO9z3lyPjG+nDTpSFr/srKibYIYyb7E6snqZPHDLkeq/TGNt/otTxuHxam09mlyJ3F/3rnJajuN4SHrfDP/58vFumlt2bXv+5FrxrPJ/tQfR+s8QkNjsqHzOG2DYemvH0DnBCUGZqdLAADD6H5mmJ0uwTAMZRklJNIHX7ba+Wo7/7nbTnjSEJOLKMfrVQ5+bFU/vMyUaHeaJCT5zrhhknQBxlsfSgw0Wg4aLScT/6i4VKzXaDkJdknmT5WcaX0lcajIv+xeL7Kp9BL3far+suiikQ9qm0bnSj3v8jll/h6NrUamvMH1kCFO9z7nyfnE+HLSpCNp/cvKGi0Hnp/sS6yerE4WP+xypNof03ir3/K0cVicSmufJncS9+edm6y20xgess4383++XKyb1pZdG4YRuWY8m+xP/XG0ziM0NCYbOo/TNhiW/hvDmgOECeDA4gJmp0t49rnXsb/zGUD4npSlF04rywqWEemDle3dPYfHnnk5rHf8xKmw7MDiQurLum2K2Hj79szjzIWLUjlKBSKVQzV2wTJi/Ygy63MIJwubCkTqO5sK40cbZYUu5THUBQBKCn1KY6CPCWDrljKumCpi7+65RP7Zt2ceSy+cxr498zFu4+vde/dObN1Shk3jnJaFP1VypvUlq3PPXXM4fuIUDnT4ns3TY8+8HNM3q2wy/t2/uADTNDL1l0UXjXxQ2VT0weMnTil5VJU/8LGVzVNa3nDPXXN4eum1xDjd75zL8gwW+1U+WrD0w4n1RtL6l5XNbinDhx/jXZGfN19mx+Y8ix/yHJnkj2m81W85P85Bibxbt5SxdUs5N2+K48o4IMn+SWVZbacxPGSZb37t8OViXpDWlo8T/DXj2SR/SuLvXvTUeYSGxmRhkvb9vWLSnhfkxbD0N/if8UwYdgB49fz5i/AG/PMgSgkcBD9HKhdNNFvBz08pISjaBI2WKy0rWEZ4GA/fh0kICAVabQ+UGLCpgZbrA4YB+D5MINMBD5ZN0XIRjmdZBhpNF5tKJhotLziUhRCUCiQ8qIyXgx+bXfNj20UTzXbwMy6TBt+gbrZdUEJgUwzlAMKZmSmcPbu65uMkjD+M3fQOrJHv5kW5bOFS0wv9YVOBjO1BBKWyhRqnS7lAxvpAgWLZQp3Th1/XMoyS7wYnqRsADDhu8MoiQgzYFkGr1dWJUgOO68G2CJy2D8fzYBIC0zLQbnd4ihqgBDBcPzx4iue0rPypkpP1VSyY8Nuu9IA/B4DnAYQYgOEDPsJxeV62LQrP8+G4Xm7ZeP5l8cRte5l1HZRdhs3BQ/JboAfeVdlUjO+2TTq5QOCvBTPIBQolK4jNLP7aJMwD2m0PhCAyT7yvUWrAIIDj+CCGARgAfERyEDG36GctMIh5Bh/7ZT663gcQrmfOMGqcq1r/sjzQBWAQA067mw8WLAOttg/H80EIYFECgwT+5fvo+GHAfyyXlPtqlCOTkMZb/Zbz9VwYoW4mNWC43ffqprUX/SxPbp1lblTxZa0wSr7bL4bBAeIcFiwS4cJSgXTyteheEYYBkxhww7yAYFOBdPLubNcizyb5UxJ/Z7HZIPOrXpBlLkfNd2dmpnD7J7+WueMnPvfBXPV1m/FoMy5+O0n7/l4xac8L8iKv/r34rv7DZQ9wXQ8GAuO16u3wM1wXzbqrLONjPN8HXBeeyz77aLvBXpIF2qyPddstN9Kn0+mzWYvK0ah1e4zK0R2bXfNjtxpOt8xx4XJ9tvUBhBOJWsd3tnV8sVYb34muC7rUx1gXAGgI+jTGSB/X9cDIJfybqge0HZ7DAM8Nyp3OBofnti4XAZ7Qt4rDepGT9TU9VZQmkKwO7cgc3u/8z/Oy1/k5dy+yRfiXiydZdR2kXTQCqGwqxvcWlxfAAdiv+puRHAFod+p56PoTP08RX3MAn31GN1/gcxAxtxjEnIt5Bh/7VT6qsf5IWv+yPJCBjzFsPpm/sgdPsu/FsFxS5qvhuH3KPYhyvh4QjT0MvfBmntw6y9yo4ovGaECcw5YQAxq1KG8ynuYfqLKymlA37Vrk2SR/SuLvXvTUfqihMVmYpH1/r5i05wV5MQz9N8b3yTU0NDQ0NDQ0NDQ0NDQ0NDQ0NDQ0NIYO/QBaQ0NDQ0NDQ0NDQ0NDQ0NDQ0NDQ0NjTaAfQGtoaGhoaGhoaGhoaGhoaGhoaGhoaKwJ9ANoBSybwqcUjmEAJgWxTTiGAZ9SlMtWWOabFHbJCstKXBlMCsq1s7ijhotcPbNowTdpWI+VnT53Eb5JYRa7/dtFEz4lQf+UgtisHemU0dh4vC6iHCKCw8FI2Cel2kVGEcOapzLvix3fH1dMki7A+OqThY8sO+BcjxI4BoFhBlznmyTkY56DzaIZrgHZ2sjDgTz4vlZWG9J1xuoQxRi9jp3FbiLn20X1sQ6lhLiVRaZCKepvhZLc30T7B3JH52Mc4wyT2TcJfErh9iF7lr5YHZcQUNuMrAeYwVrwKIFVtAAzOi/EpvD4/knwf3AtHydtLlT1eL+kBRPgcplSh5NUbVV24P1azI+y+GqWdZtFN7GM2BSgdKz8Nit4PcXckj+siHa4zuPqijwEi4b+ZtgUMLtzy/OBzJ58/y4h8M2u3xLbhG+SSDtRVlUckPGQDCLPspxbtVbZOOffrHNxKZCb2DTCd3wfvNzss2yMpHkR/TRYe2xss7PuJ8tPk5A3rvD7sNjeTthvWQIPiXXF61LKNet7eaUGuxQtE+USr5PivGgHcY2J60C1bxyUjTU0NMYL47q3HCQ2ug2y7vf6gT6EUALLpji32sKhI0tYXqljdrqEvbvn8OUnv4+V1Qb2Ly7g2FMv4bsvnMHsdAn7FxfwvRd/glqjjfe9+5rEdldN2aAmwZmVBg4dWcLP/vRV+OVffAceOHoCyyt1vOfGrdh92/WRPvbtmcfXv/0q/uG/n4uNzfq/4R3TsbH3Ly5g9ooilt9oxO5fNWXHTj6mlGCl1sb9XN0DiwuYLltDPeVYIxnDmqdy2cLplbjvbJsujt2JuJOkCzC++si4VeQjy6Z4q+HgjdUmDh87Gdb71J6b4Dg+vvOPP8Z7594WcmbAkQu4rGyiZFOcX21F1sZnf/0XcGm1nYkDeWRZZ6zON55/PSZTXv7Na7dDv/GLeOvNuC1nLi+g1XAi7UtlCz+R+Mv3XvwJ/nrpf6TKVChZUj1mryiiWe/6m2gzWTz7zCd+Hu22h/vGKM4wvR556iXc/t534qFHT/Yse5a+AIR2nJ4q4lf/9T9Ho+lG1sMnP7wT/9ffvIy7fqmCR/+6GuYEv/OJn0e94eDPj/8g1v89d83hiW/9EB+67frYOEn6qNbCVZcXcPbNJg515PzIB94VkZFx0k9WGrG2V07ZOL/aktqB5TlTZTuSH2VZP3njY1J9cR5E/Ubdb7NCZgM+d/30x96DyzsPvS61XaxcaMbmmc9LmZ/9yq0/A8fx8eBXn4vxgcyeBxcXUCqZOHu2FrnP+vtf3vdOlAsmPPj4vaPfk8rK+1YSD8nmThWfvvn869j5rqtja5X34V+59WfQbEXX6L1378SWywuo1x18VeLnvNyf2nMTWi0Pn3/k+Yg9LIvgMw9/R6kr89M36m3c9yX5/E2KnyYhbR2LKJatcB/Gz/VbFxv4nYe/G/GZtH2azL9k+8QfL6/iD77yfKT+Ly28HTfdcHWs7YuvnMPDX3tB2bcszot2UHEyk+uO918r3TdeNWXntvEk+5aGxkbBuO4tB4mNboOs+71+of90KUHLRWh4AFheqePwsZO485brsLxSx6EjS9g1vz0sO3RkCTfvvAa3LuxIbddygXrTC+vdcfN1YVIDALvmt8f6eODoCdxxs3xs1r9s7ENHltBoedL7sr2bA4SJBat7/5ElxFMcjfXEsObpUlPuO5ea45doTpIuwPjqI+NWkY9aLrB8ofsAgtV762ILn/vKc7h1YUeEMwOOXILj+Gi58bXhOH5mDuSRZZ2xOjKZ8vJvXrt5Cls2236sfU3hLzfvvCaTTCo9Gq2ov4k2k8WzMxdq4cNndm/U4wzTa9f89vABEtCb7Fn64u145y3X4a2Lrdh6+NxXnsOu+e144OiJSE6wfKGG3/+T70n7f+jRk9g1v106TpI+qnrNdndt3XnLdTEZGSfJ2rK1KpOT5TlifpTFV/PGx6T64jyI+o2632aFzAZ87vrZP/5uaI8zEm4W81LmZ6uXWuHDZyDKBzJ73nck4HHxPuvv8LGTeONiE29dbCll5X0riYdkc6eKT7cu7JCuVd6HVy/F1+iDX30OjuPjPoWf83K/dbEVPnzm7XHmQi1RVzYv7OFzUr1JRt51X1fExLfNbgYQ9Zm0fZrMv2T7xOt3XBmrf/POa6RtF27clti3LM6LdlBxMpNLtW9U8aveI2poTDbGdW85SGx0G2Td7/UL/Q1oCVzPCw3PsLxSx1TnL+n8Z3bt+z4oQWo71/PCzwBibabKlrQP9isn2dhTZUs5tkoX1/Nik+96vqKurx1lhDCsecrjO6OOSdIFGF99ssjteh6Kthmrx+6puI4Y8v6JoebGJFtlWWesziD4NwmyfnxfJZ+M2+Vy+L6fSaaseog2k8Uz2dyOepxheqnicx7Zs/TFPgOI5A9iXdYHnxMw+6r6Z/fFcZL0Ua8FLyJn3jWQJmfSulL7ar74mFSffU7Wb3T9NitUNojmroE9VOtXlpeyzwy8DVX2VPE1q1+049aW5dlZ8uq4n8t9NY3fk9Yo0yfJzwG1XUV9VfOSZf7G3U+TkLaO4/XVvAREfSZtn5Y2t0l9q+I4i81JvJrGZ2lyJfm1DHqPqKEx2RjXveUgsdFtMCz99TegJaCEYHa6FLk3O13Cauer9/xndm0YBlwPqe0oIZH+xTartba0D/brJtnYq7W2cmyVLpRI3mVKDEVdI1ZXY/0wrHnK4zujjknSBRhffbLITQlBo+XE6rF7Kq7zfHn/nq/mxmRZ09cZqzMI/k2WJd6PYajkk3G7XA7DMDLJlFUP0WayeCab21GPM0wvVXzOI3uWvng7rtbaSpuxPvicgNVV9c/ui+Mk6aOuRyJy5l0DaXImrSsV8sbHpPriPIyb32aFygbR3DWwR5IvitdiXd6GKnuq+JrVb7QcNFqOtLwrK1GOy7eJ+7ncV9P4PWmNMn2S/BxQ82K6rup1LNabZORf98kxjZ+vtH1a2twm9a2K4yw2J/GqXK90zmJy5eVXvUfU0JhsjOvecpDY6DYYlv5DsWalUvl4pVL5O+7fm5VK5b9UKpVfrVQq/1SpVP6xUql8qVKpxF48ValU9lQqldNc2/vWWl6bInjfSWcCZqeD96k99szLmJ0O3oVy/MSpsGz/4gKefe51PL30Wmo7mwKlAgnrPf7sy9i3Zz5sc/zEqVgf+/bM4/Fn5WOz/mVj719cQNEm0vuycyZMAAeEugcWFzbEX3zGCcOap00Fue9sKowfCU+SLsD46iPjVpGPbArMbilj7+65SL3Nl9n45Id34uml1yKcGXDkAkzTgE3ja8M0jcwcyCPLOmN1ZDLl5d+8diMKWxas+GawrPCXZ597PZNMKj2KdtTfRJvJ4tnWLWUcHLM4w/Q6fuIU7rlrri/Zs/TF2/GxZ17G5svs2Hr45Id34viJU9i3Zz6SE8xuKeO3/u1N0v7vuWsOx0+cko6TpI+qXsHqrq3Hnnk5JiPjJFlbtlZlcrI8R8yPsvhq3viYVF+cB1G/UffbrJDZgM9dP/2x94T22CrhZjEvZX42tcnGvXfvlPKBzJ4HFwMeF++z/vbunsMVlxWw+TJbKSvvW0k8JJs7VXx6euk16VrlfXhqU3yN3nv3TpimgYMKP+fl3nyZjd/80Ltj9ti6pZyoK5uXgx9Vz9+k+GkS8q77kiIm/mj5LQBRn0nbp8n8S7ZPfOm187H6zz73urTt0gunE/uWxXnRDipOZnKp9o0qftV7RA2Nyca47i0HiY1ug6z7vX5hsJ/5DAuVSuVGAH8B4MMAvgJgJ4BVAEcA/F21Wv28UP8PAfxttVp9JOdQOwC8ev78xcgJ3llh2RQtN/gqukkJCDHQarughGBTgeBS04PreaCUoGAR1BoOKCEoFwhqnTKTElBioNlpZ1OEB+cUyxbqnXrFgom248F1PVBCUCqQTpkPSg1YJkGjGfRfsAw020GZSQgIBVptD5QYKFgEzbYfyMWNx+siyiGCUgIHwU+tKDFgAiN1uMTMzBTOnl1dz/GH8af+HUjx3WHNU7lsdXw9GGdTgYztS/gnSRcgvz6j4rtZ+MiyKVwYcFwPngdY1IBBAn83YIAQA67rcxxpwG93OFSyNgg1MnMgD76vYsGE33Zj64zVodSA63bH7YV/kyDrxyBGhPMLliE9mAgIDiKsKeJWFpkKJQuNVtffijaRHkgh2t+mBlpsrjrzASAzfw3Jb4EU32V6wQDgG/A8H4SgJ+7N0her4/mAbRL4ABzXh+f7MIkBH8HPtwsWRdvx4LjdeWm7wbVhGIAPeL4PYhiA4QO+fJy0uVDV4/2yYFO4rg+nk8uUCwT1WlvZVmUHPpcR86Msvppl3WbRTSyzLQLPBRzPyxR3R4Vzs4DXU8wtr7qijPPnL4b1fGoEvugBJjFgWx0e8n1QI+Bq3wNgAKTz2XWDueX5QGZPAGH/vg8Q9u1Kv8OxnbqsnSir6FtJPCSbO5FnWc6tWqtd/g82Z3xcogQw3GBORD+nxAjlZjq4nh8bg7WVzYvop64R2C0Y20Db8UCM3jhqnHyXQbWOVfsGfh8W29sJ+61SwUSL4yGxrnjN7wVl112/6uzdWt2y7h5Qfp0U50U7iGtMXAeqfaPKZuu9R8yyBxw1352ZmcLtn/xa5o6f+NwHc9XXbcajzbj47aTtlXvBRrdB1v0eQy++ux5/uPwCgAMAzgD4d9Vq9S0AqFQq/wjg7ZL68wCuq1QqBwD8PYD/o1qtrqy1kO2WCwMdAzkuPPbZdVGrRctajhuW1YUyl2vX5vZMjVo7rOc0up/humh0+tjWCbQO13/LRaSu57LPPlquFylj40V0EeQQ4Ub68JH/MYnGMDCseap1/JT5Yq02vh4xSboA46tPFj5iD5hI55/vAD4C7gOAzrONDs8CDrcXk60NV+DNJA7kwfc1PVWUJpCsjhcZAz3xbxJU/fD3kp7L1bmYI8atLDI161F/a9bljUT7tyN26XLVuMUZphcDBQAXPcmepS9WhwJwO4d/sPWAzvMwA0C7s/kX54X/roL4hTbZOGlzoarH+6XbDBYinw8ltVXZgc9lxPwoi69mWbdZdBPLvM4CGye/zQpXyB/53DJ8CNypB5fzRRfRvLRzj7VgW1w2tzwfKO3pcv7LPdtiHMtuyWQVfSuNh0SIPNvidOH1EMe5cssmnD27GolL/GM5WR9dud1QN9kYqnkR/TS0SWdstu4nyU+TkDcv5vdhsb2dsN9qN5Lritf1lGvWN3ugKtsDqq7T/n6dxFniOlDtG7P0PWkcqKGhMb57y0Fio9sg636vHwz1AXSlUrkVQKlarf5559apzv0ZAL8BYFHS7DSA/wTgbwHcD+C/IPj2tIaGhoaGhoaGhoaGhoaGhoaGhoaGxghj2N+A/jUAD/I3KpXKTwH4KwD/rVqtPis2qFard3B1fx/AD/MMeOWVl/Uk6KhgZmZqvUUYOWwUm4ya706S3SdJF2D09Bk13x0URs3O64VJtsM4++4kz0s/2Ah2WWu/3Qg27BfaRr1hGJw7ynMzqrJpudIxzvmCxnAxjn47SjKvFza6DdZK/6E9gO4cMPh+cN9yrlQq1wP4fwA8VK1WPydpczmAj3HvhTbQeZVaVgzq3WLrgfV+3/EoYr1tMkwiGiXfXW+7DxKTpAuQXZ+N6ruDwqT5Ta8Yth2GnfyNq+9q/5RjPe0yKZyrfSsdk2ajSfFdYLTnZlRlG2e5Rs13N/oDLI0A4+i3o8gBw8RGt8FaPl8Y5pGOPwvgB9Vq9RIAVCqVKQBPAfi07OFzBxcB/FalUnlP5/o3ADy+5pKCHbRC4BgGiE0BSuEYBnxKUCpb8DvXxDZhWKyMosiV+ZTC4o4TVvUJSkHsbv+WTeFTguWVGsyiGemvUIqODbNbZhdNaf+i/Kx/xzAAk4LYplRejdEF70s+JeEBOINGuePPp89dhE8pymVrTcbRyI9xnZssvlssWyG3eZSCFkzQTl2fEtileDnjrmGtjWHDLppRnW2zw/nyeCOCjx0sluRBEDcoTp+7lDjWpNpfhaz6yurx96gQzwsdH/dNAp9SuISE8dolBNQ24XP1HSf5kL08c5JUP6lM9DG7c+2m5CF5xuhHr17baASglAAdX/RNCt+k8CiFYZugIT8F9rWKVsyfs3KVbFyfku5aWMO5S/OPtLUR+Djp2CVYr5ZNQSkBsSm8SO4drG/ebsQ2O3rGOUL762Ah2lbkJD6+mkUrwrf8fsunJLb3K5Xj8TbCjZ32yyu12B5QjOuy2J20lliszrLW8tQdNLRva2iMHsZ1bzlIFAUbFDeYDYah/zBfwXEtgB9x1x8HsBXAJyuVyic79/6yWq3+dqVS+WLn819WKpX/FcAXKpVKCcAPAHxkrQWllGCl1sb9R5YwPVXERz7wLhw+dhLLK3V84oM34oZrr8IhSdnsdAn7Fxdw7KmX8N0XzoTXV03Z8Fxf2efsdAl7d8/hy09+HyurjbCPqbKNX/7Fd+CBoyewvFLHe27cit23XZ849jeffx0vvroSlon1+D5UY181ZaeeMq+xfuD9k83hgcUFTJetgZ5GXS5bOL3SiPjK/sUFbJsubqjTYEcR4zo3WXy3WLawvNKI1Nm7ew7TmwuAD7z6/72Jn5qdkpZvLlk492ZzzdfGsGEXzZhen9pzE6hBIvdU/F0oWVh+I+4vs1cUE082ZrBsinOrrVh7caxhcdOoIKu+snoHFxdgWQSfefg7ifF857uuxkOPRnOFv/ne6/iXN10Tqy+b+7xzklQfgLLMtKnUx8ScRJUrZRkjzaZpvrbR/HOQoJTgjVob93G2u+euOTzxrR/ijpuvw2VlC//xi98Jy/btmcejf13Fd184I807s+aabM4eeeol3P7ed0bWwqDnLs0/kso9z4/ZZ+/uORQLFEXbRLFA8ZOzlyJr4N67d6JgERzq5PisTcGm+POnf4C7b7s+5Ajtr4OFbC5l+zfZXozntse/8UpkX7i8Uscd778W73v3NcrrpH3YDe+YTq2r2me2W27mWA0kx/X1sL/2bQ2N9cW47i0HiWLZwhmJDbZOF9HYADYYlv5D+3NjtVr9s2q1upu7/ny1Wi1Uq9Wf4/79dqfs49Vq9S87n79VrVbfXa1W31WtVj9YrVbfXGtZHSAMinfecl2YMALAwo3bwkkRy5ZX6jh0ZAm75rdHrltucp/LK3UcPnYSd95yXaSPO26+Lkx4AGDX/PbUsW9d2BEpE+vxfajG1s+eRxu8LwHBHN5/ZCnfu2ky4FLTi/nKoSNLuNTUyeF6Y1znJovv1pterM7hYydx5nwNZy7UcP2OK5XlzbY/lLUxbMj0eutiK3ZPxd+NltxfGq1s/tJyIW0vjjUsbhoVZNVXVu++I0s4c6GWGs/ZAzd2//Cxk7jjZnl92dznnZOk+kllKh8TcxK+jM+Vst1dnDgAACAASURBVIzRj169ttEI4ADhw1UgsN1Dj57ErvntePCrz+HsSi1S9sDRE+H8yvLOrLkmm7Nd89tja2HQc5fmH0nlb15qxuxz+NhJvHWxheULNXguYmvgwa8+hzcuNmNtVi+1sGt+e4Qj1krnjQrZXMr2b7K9GM9tQHRfCAC3LuxIvE7ah2Wpq9pnAtljdd66g4bmYg2N0cO47i0HibrCBvUNYoNh6a9/7yKB6/mh4afKVvgZAHxfXQYEEzXFfVV9eaUO1/MS+xTbsc+UIFKPb6fqg5LkelnGdr2NscjGFbwvMQTzNth36LmepxhH+8d6Y1znJovvqnQr2iaKtplYrrbL+L3Tl4dMr6JtZvaBfv0la/thcdOoIKu+qnpFO/gRWlI8z3NfPvf55iSpfnKZ3EfEnIQvi+dKyWP0o1evbTQCqGzH5pb5slgGqOc/C/+wcdV9DG7u0vwjqbztJMctfu8glsvuJdlV+2v/SPJn8TqJhwHE5lasn7SXSxsrzz4z0Ct7rF/PPFJzsYbG6GFc95aDxEa3wbD01w+gJaDEwOx0CQCwWmuHnwHAMNRlADA7XcIq9xX12ekSKCGJfYrt2GfXQ6Qe307Vh+sl18syNiXaLUYZvC8xBPNmDHgcohhH+8d6Y1znJovvqnRrtBw0Wk5iudoug10bw4ZMr0bLyewD/fpL1vbD4qZRQVZ9VfUareD7XknxPM99+dznm5Ok+sllch8RcxK+LJ4rJY/Rj169ttEIoLIdm1vmy2IZoJ7/LPzDxlX3Mbi5S/OPpHLLTI5b/N5BLJfdS7Kr9tf+keTP4nUSDwOIza1YP2kvlzZWnn1moFf2WL+eeaTmYg2N0cO47i0HiY1ug2HpvzGsmRMmgAOLC5idLuGxZ17G3t1z4WQsvXA6eHempIy9J+X4iVORa5sm9zk7Hbz/67FnXo708fizL2Pfnvmw3vETp1LHfnrptUiZWI/vQzW2PodwtMH7EhDM4YHFhYG/0H1TgcR8Zf/iAjYVNG2sN8Z1brL4bqlAYnX27p7D1ivL2LqljJdeO68sL1jGUNbGsCHTa/Nlduyeir+LttxfinY2f7EppO3FsYbFTaOCrPrK6h1cXMDWLeXUeH7PXfFc4fFn5fVlc593TpLqJ5WpfEzMSfgyPlfKMkY/evXaRiOACeCgYLt77prD8ROncO/dOzEzXY6U7dszH86vLO/MmmuyOTt+4lRsLQx67tL8I6n88k2FmH327p7D5stszG4pg1DE1sC9d+/EFZcVYm2mNtk4fuJUhCPWSueNCtlcyvZvsr0Yz21AdF8IAE8vvZZ4nbQPy1JXtc8EssfqvHUHDc3FGhqjh3HdWw4SJYUNShvEBsPS3/D9if25yw4Ar54/fxFeDz/poZTAQfAzIdsi8FzA8TxQYqBcoKg1PbieB9ui8P3g53eUEJQKBPVOGSUENkV48IOqT5MQEAq02kH/NjXQcn3AMGCZBtptP+yvaBM0Wt2xPc+H4wZlBctAs+3F+hflZ/27ng+TEhBioNV2Y/KOImZmpnD27Op6jj+MP8/vQIrv8r5EiQETWJODO8plC5eaXjjOpgIZ+4MI1tuHBoW8czNOvlssW2i0PDiuD0ICHoQT1HEAFGyKVjtaTnwf7Za7Zmtjvf3GLppoOX5XZ2qAkuD90LJ4I6JQssLYwWJJlgMIGSybouUCru+DGoZyrEHbf0h+C/SYM2TVV1YPQHivYFG4XDwv2gTNtgcfAHzA84N2hBhoOx5sk8DxfLhucH/L5iJWVi71JWOW+kllXR8Lygo2QbPlwfN8WJahzEPyjJFXL3HdDit2dsYeCc7tF8yGlBK4MOD6AQfBAHwPoDTwS9fx4HgdfrAI2o4X8Weef/LkmmzOYADwDXieD0KwJnOX5h+q8pmZKVy4cAkuDDi+D2IYoBQwYIDCh+f68KkBx/XheYBJDRAS9GMguE+JAUqN4KeuPmIcsdb+ymNSfBdQx25xLkVOKlgkjK/Fgom24wWHUQr7LUoMlAo0svcrF0i4T5T5P2sPw4DhI7IH5MeVtU1bS2GszrDWVHWHke/0wsVZ5Bo1352ZmcLtn/xa5o6f+NwHc9XXbcajzbj47STu+/OiWLY6fM74nWyIAwgZ8urfi+/qPzYq4LoeDAQG8jqB2wQA10e9xpc5XJmLRs0Ny+C6aLtZ+nThud3+226QZ7NAy/fXrLvKsVuddmL/ovxtrh4cF55CXo3RBe9LcH2s1bTVam0YALZ1fLFW0w4yKhjXucniuyzQBXUAl6tkAGjVvVg5qzKstTFstBo833d1VsUbEc16OxZL8qDdciP+phprUu2vQlZ9VfXYPVeI52x+WFZHAcADvM5nt8X3B5im+tsJeeckqX5SWdTHgFYnX6EAvBY4/YQ8JMcY/ejVaxuNAOEfCIDAEdF5JszxL9tUtN1ojsvnrnlzTTZnDBQIOLAHHbKOpfKPtLUBdG3gu4CP0FSAG/zslACAE9xnerE147ndeyJHaH8dLMS5FDmpxZU7jWj8bAl1GzUvUl4X9oKi/7P2bJ/H7wFbrpfYNm0tsVidZa3lqTtoaC7W0Bg9jOvecpBoCDZobDAbDEP/jfF9cg0NDQ0NDQ0NDQ0NDQ0NDQ0NDQ0NjaFDP4DW0NDQ0NDQ0NDQ0NDQ0NDQ0NDQ0NBYE+gH0BoaGhoaGhoaGhoaGhoaGhoaGhoaGmsC/QA6Aygl8CmBYxjwKYFdNOFTGlybFGbR6pRRFEqWsp1l0/DaLllhHzApqG2GfdhF9au5+T6JTQEmByWglGSqx8sPk4JwY1vc8ceBvDQiF68PP15W22VpozE6KJcDPz197iJ8SlEuW+mNRhSTpAsw3vqI3ELs4B+1TXiUwCUEvknhmyTCS5QSEJt261BWp8stYt/WGh/pzjgOHVnFcQsc1/smhRXGi/75MA+/5rGLrF8WN5i/JcWpjQxV3PdNAmJHY6/ZsambEMNd0mlnUjgGAcxoLC4IPFAsW5lyhLXU3e34mLg2VXZKy1G6+ZHOIXqBiifYfcazvB/ybVZWG9G8t8PZ3bkh8CiBwXhcaC+bt7y5YVJ9WZnId7Rogtg0koevhT+Ja4DFhaxrIU2mvHboZYyNCtFnimUrcs3HPLFuJM5L2paE66A+wfJKDYYV3YcV+8zlkuZaXMdMjkGtUw0NjfHGOO8tBwXG58wG/LO9jYBh6K93kCmglGCl1sb9R5awvFLHHe+/Fu979zU41LmenS5h3555fP3br+If/vs57F9cwOwVRTgtN9LuPTduxe7brsehI0v4pYW346Ybro70sXf3HL785PexstrA/sUFzFxeSJRleqqIj3zgXTh87GTYx4HFBUx3iCKp3v7FBRx76iV894Uz0rGvmrIBAOdWW6GMvPzieKpTi0XbZWmjMTooly2cXmlE5nz/4gK2TRfH7kTcSdIFGG99LJtGuCXg0AWUSxTn32hEuOqeu+bwxLd+iN23XY+Zywt4q9bGyoWmtM6HbrseV11ewNk3mzG7XDVlK0+B7weM4x556iXc/t534qFHozx79XQRPxHmiY8X/fBhHn6V2VxlF1m/n/31X8AliV1nLi+EhyNqyG23f3EB33z+dfziz/0Umi03FrMf4WJx1hjO2i3csBXXvm06Ni8vvnIO3zz5Y2WOsBbxV6Y7vzb5cZNyGVmuwecoOofIBxVPXDll4/xqS8pdn/nEz6Pd9nCf4Fd83rhvzzy+dfJHuOmGq8O52bt7DsUCxVTZRr3uRNrz85Y3N0yqDyBWdvCjCzBNgt99+DsR3rUsAmIY+PKTL8bW3CD8KWkN3P7ed6auhTRb5LVDljK9jgKIMVK212Mxz/f8xP2R2FbGaSwuPP6NV6T7sK3TxfAw5jxI8hFqEWl+xMvB+4zew2lobCyM895yUCiULCy/EbfB7BVFNOuTb4Nh6a//lJkCBwiDLwDcurAjnBQAWF6p44GjJ3DHzddheaWOQ0eW0Gh5sXa75reH7W7eeU2sj8PHTuLOW7p9NNt+oix33nJduGFjfdx/ZAlOhnqHjixh1/x25dgtF2i5iMjIyy+Ol9V2WdpojA4uNb3YnB86soRLzfFLPCdJF2C89RG5JeDQJVCDxLjqoUdPhtzTbPs4c6GmrHN/p47MLmvw7BlAl+N2zW8PH+Dw49Yk88THi374MA+/ymyusousX8eR21UWpzYyZLY7dGQJty7swOqlljRm87E4awxn7X7uZ7ZK52Xhxm2JOcKwdOfXpqOoK8opyzX4HEXnEPmg4omWq+auMxdq4cNjdk/MGx84egK3LuyIzM3hYyfx1sUWHMePtefnLW9umFRfVnbfl5awfKEW411qECxfqEnX3CCQtAayrIU0mfLaIUuZRgAxRsr2eizmpe2PxLYyTmNxgV2L+7B6j7lc0lyr8iNejn7WqYaGxnhjnPeWg0KjJbdBo7UxbDAs/fUD6BS4nh9OAgBQgsg1EFyzXyUtr9Thel6s3VTZCq9935f2MdX5qzPrI0kWvj++D9fzM9Wb4n5SIRvb9Tyl/OJ4Kog2yNJGY3Qg+gCg9s1RxyTpAoy3PirZk3iR6Va0zdQ6w7QL4zg1P8rlicaL3vgwD7/msYusX2LI4944+NswoZoTSpDou/x1nhietJZ6idn9QKV7d2360rqinGk5i84h8kHNE56Su7L6KsuH+bkp2mYCX/gpMsnnNam+qqxom7F7xAh0k625QSBtDaSthTSZerFDWplGAJFLVXu9LPsjsa2K0/i3WWTZA2bTI8kPkvMRvm5aXxoaGpOHcd5bDgob3QbD0l8/gE4BJQZmp0vhteshcg0E1+zXSLPTJVBCYu1Wa+3w2jAMaR+rnZ83sD6SZOH74/ugxMhUb5X7KYVsbEqIUn5xPBVEG2RpozE6EH0AUPvmqGOSdAHGWx+V7Em8yHRrtJzUOsO0C+M4NT/K5YnGi974MA+/5rGLrF/Pl8e9cfC3YUI1J66HRN/lr/PE8KS11EvM7gcq3btr05DWFeVMy1l0DpEPap4gSu7K6qssH+bnptFyEvjCSJFJPq9J9VVljZYTu+f5gW6yNTcIpK2BtLWQJlMvdkgr0wggcqlqr5dlfyS2VXEa/xaLLHvAbHok+UFyPsLXTetLQ0Nj8jDOe8tBYaPbYFj6bwxr9gETwIHFhXAynl56LXgXSud6djp4t9zjz76M2engPSlFm8TaHT9xKmz37HOvx/rYu3sOjz3T7aNgxQM83+djz7yMvbvnIn0cWFyAmaHe/sUFHD9xSjm2TQGbIiIjL784XlbbZWmjMTrYVCCxOd+/uIBNhfGjjUnSBRhvfURuCTh0Aa7vxbjqnrvmQu4pWAa2bikr6xzo1JHZZa3OIWQcd/zEKdxzV5xny5J54uNFP3yYh19lNlfZRdavacrtKotTGxky2+1fXMDTS69hapMtjdl8LM4aw1m7v/vBGem8LL1wOjFHGJbu/No0FXVFOWW5Bp+j6BwiH1Q8YVM1d23dUsZBiV/xvrpvzzyeXnotMjd7d89h82U2TNOItefnLW9umFRfVnbwowuY3VKO8a7re5jdUpauuUEgaQ1kWQtpMuW1Q5YyjQBijJTt9VjMS9sfiW1lnMbiArsW92GlHnO5pLlW5Ue8HP2sUw0NjfHGOO8tB4WiLbdB0d4YNhiW/obvT+xPaXYAePX8+Yvw+vy5EKUEDoKfI1FioGARNNvBz5koJbBMgkbTASUERZuEL+kW29nUQMsNfg5XLppotoKfcpk0+MZ0s+2CEoKCZaDVcDAzM4WzZ1eVstgWgecCjueBEgMmEDnYRFWPl9+kBIQYaHXGtinCQ6ksm6LlBl/HZ3I1216oDz9eVttlaZMEmU2GiZmZqWE8cdmBAfluvyiXLVxqdud8U4GM7UEEk6QLkF+fUfLdLrcEstPOg1ADBtquB98HCDEA+IBvhLwUnMhuwHH9oI5hAIYP+Ai5ReQtntP6gYp7GMcZhgHfB1zfBzW6MhdKFhodrqeUwDYJ6k1nIHyYh1/z2EXWLw3jBouDxlAOIByS3wID4l1V3IcRfLPA7eQAJjVgmgbabR+e54MQSGO45wOWSYKfULtBO9vsxuJNJRMNjgdKBYJ6003NEdYCocxeZ/0Ka1Nlp7QcpZsf5ddhPXOGUeFcFU+w+zAA+EbEDwGEbYoFEwR+ZP1TGsxzMDceSOeeAcBw/Uh72bzlzQ2T6svKCDUiMca0DPieDwqEefgg1wTzM3ENGAbg+8i8FtJkymuHLGUKfUbCdweBrBwgxsiAS73IHojFPLFu0SbdOC9pWy4Q1LjroL4LGAaIEeQPbB9WKpCeDiBkSJpru2h296+cHGJd0Z8HvV56RZa5HDXfnZmZwu2f/Frmjp/43Adz1ddtxqPNuPjtpO2Ve0F33xbYgH+2txGQV/9efFf/ITMDXNeDgY6xXB8t/tpx4Thup8xFs+4q27VdhNetejvShxvWcxMPzOL79DoVWf9uxnqi/B43dpvrpN1yOfkDuXh9sjzSEW2wRmeBaawRarXAT7d1kr5abXxncJJ0AcZbnyi3AB4nevg3Vm6Pw3jJdT3Alf90h3Uh8lZ7jc3COA7g+BFdmZsC17fDeNE/H+bh1zx2kfXL7jF/W6uDHccdqrgPAB742As4TlBGAcCFNIZTAF4r2q7ldH2tKfBAo+ZmyhHWArzM/GCycfPkKC50DtEPVDzBcxcQ90PWZnqqiLNnV6Wc7XL3fBfgt7RJ3JQ3N0yqL+crROR1Ow28FLn6hWwN8DZOWwtpMuW1Qy9jbFSIMZLnUnFvJtZt1pPb1mvy+uyBqi+07QdJc91qOAly979ONTQ0xhvjvLccFNi+jdmAf7a3ETAM/TfG98k1NDQ0NDQ0NDQ0NDQ0NDQ0NDQ0NDSGDv0AWkNDQ0NDQ0NDQ0NDQ0NDQ0NDQ0NDY02gH0BraGhoaGhoaGhoaGhoaGhoaGhoaGisCYbyDuhKpfJxAL/B3XoHgD8B8BcAHgRQAvBotVr9tKTt2wH8KYBZAFUAH65WqxcHIVfS4YK2ReF5Phw3+fCeYtmKHDSxqUA6L28PDngp2gSX6k7sEMKCReGG/RPlgU55DgLk6/L9R3Xp7TBBjY0JdhjB6XMXQSkd68MI2Ev1mS7jfqjAJM2NCJ7LbIvC93y0XT886IoQA67jw3WDw6/WisPsoomWE3DnmQs1WLYJuN668CUvCyEEFjVgUiNy8BF/sGDSIbhph9N1D9LyQQgBjKAPzwWWV2oAJUqbJ43Lrn0SHL436IMi1xLDPoyJHbjpdg56Cw4v7B5w1Wh5EV9IOghprQ98Ux1wxec1/fhjmpzdQ/QCPrCpAReA4/r4yflLoJSCwlfqEDlYmhBYloFG05WWjYOv5kU/h+CJ6zt6AGBgM8MIDo2VHX5oIDhUlhDApEZ4SCDPPYM4vI8h5qM2QbPldWQMfIgSAkKBVtuL+KbJ3e+VAyglcA0jiGFsTVMCShAcMgr1WpPZetJz9zxrVbRPqUBR53Ik/oBWVWzir/PsCdPmSpSbHVJ4+txFUJNGDrPPu0dLOmC437g1aocQamhorC0meW+ZFezZHh87+jkYdtzAYsrpc0H+vBZ571AeQFer1S8C+CIAVCqVGxE8eP49AN8G8H4ArwN4slKp/M/VavWvhOZ/BOCPqtXqsUql8h8A/AcAn+pXJkoJVmpt3H9kCcsrdcxOl7B/cQHHnnoJb6y28JEPvAuHj50Myw4sLmC6bEUCb7Fs4cxKA4ckfXz3hTPh9Ss/WsHSi2ew+7brcejIEqanirH+9y8uYObyQuQhtON4OLfaivX/zedfx+PfeCUiF6FGWJfvXzWWrA+dVGjwKJctnJb497bp4tgFo0LJwvIbcV1mryiO5UPoSZobEZZNI1z20dtvxINffS7U81N7bgIxSET3teAwu2ji3JvNSIzYu3sO05sL2GTSofKlTJZ7796JqU0Wfvfh70R84KopG57r9xzfZLHxtz5yExzHj8yDzOZi2/fcuDWMe6zdZ3/9F3Cx5uCBo0sxuUf1wZ7MJmsZNykluNR28dYbbXz3n36M9869DQ8cPaHMM/bunkOt6aBsEulDujyyJ9UHoCyjFsHZN5sxTpq5vAC37fWdb6nkfOSpl3D7e9+Jhx6Ntnd9D7939HvhvYOLC7hCoYNoz3175vH1b7+KqbKF9737mphOo+yreZHHP7Ks74MfXYBpkggv3XPXHJ741g/xoduux5VTNs6vtvDIUy/hX73/p/H5R56P8OsVUwV8+ckXw7ng2/Iy9bIm7aIZ8VEm/zGJD+3dPYe/+d7r+Jc3XRPxzb275/DlJ7+PldVGbg6glOCNehvPPvd6bE3v2zOPb538Ed7/7muka01m60nP3ZP2aeJaXa21IvaR2Wv/4gJ+vLyKP/jK88r9Grvm28v2UWkxM6nvO95/bYxXVHqkzTOfL2XJA/L4zLDjnoaGxvpikveWWaF6trd1urghHkInxZRB5r3r8QqOLwA4AOBaAC9Xq9VXq9Wqg+Bbzr/CV6xUKhaA9wH4Pzu3joh1eoUDhEEVAJZX6jh0ZAm75rfjzluuCxMNVnb/kSWI30+uN71wgsQ++Ouf+5mt2DW/Pawr6//QkSU029FvMK2sNqT937qwIyZXy4W0f9VYsj40NHhcUvj3peb4JZ6NllyXRmv8dAEma25EiFzGHnoCgZ5vXWzFdF8LDmu2/ViMOHzsJM6crw2dL2WyPPjV57B8oRbzgZbbX3yTtV291IrNg8zmYls+7rF2juOHD59FuUcVMpusZdx0AJy5UMODX30Oty7sCB9UsbHFPOPwsZP4yflLUnnyyp5UP6ms2falnNRs+wPJt1Ry7prfHj445Nu/dbEVuXdfgg6iPR84egJ33Hwdbl3YIdVplH01L/L4R5b1fd+XlmK89NCjJ7Frfjvu79iOzRt7+MzqHT52EssXapG54Ns6CbJk8RvRR5n8Mh86fOwk7rg57puHj53Enbdc1xMHOADu+9KSdE0/cPQEbl3YoVxrMltPeu6eZ62K9pHZ69CRJVy/40plX/x12p4tLWYm9S3jFZUeafPM50v8WKo8II/PDDvuaWhorC8meW+ZFapne/UNYoOkmDJIDOUb0AyVSuVWAKVqtfrnlUrlQwBOc8WnAbxNaHIVgLc6D6hVdRJx5ZWXSe8vr3QT5O69OqY63zyQlcEwMDMz1RX43MXEPti163mYKlthXf5ztJ6PbZH+L0nrURK9hhH8lE/Wv2osWR+8bqOMcZGzX6h8d1hQ+bfop+OASdIFGH19+vFdnvdk/FW0zUz83C9UNi7a5tD5MlEW4Z7rBz+V7TW+yWJjVpuLbWXzRwz5+K4/ur6ryhfWyg+WV2qhzSmR20vMM1R+mVf2pPrhZ0mZ63pKTqK0d39UgcmpynFkayNJB9GeLEcaZV/l0SvnZvUPGTfksT2r6/p+6rwVbXlbXqZe1qTIo6zfpDw51W+58dK4gMms6je8L/FTlYzjlLur0Os+jV1TErePep/lJfbFrrPso5JiZlLfSfOfd55V+8SkPCCzzxh9tl8jjJK/r/c+TWN8MA5+O+p7y2Fgo9sgKaYMUv+hPoAG8GsI3vkMBN++5r/yawAQ/7wg1oGkTiLOn78ofzciJZidLkWMPDtdwmrn6/WyMvg+zp5d7XZBaWIf7JoSgtVaO6zLf47WMyL9m6a8f/6XT0wuSgxp/6qxZH3wY48qZmam1lXOYQYQpe8OCSr/Fv10HDBJugC96TMuvsvrJuOvRsvJxM/9QmXjRssZOl8mysJhdroEahiA7/ce3ySxMbPNhbay+fN8+fjUkPvusJN2qe8q8oU18wNKQpu7ntxeYp6h9Mu8sifUZ59lZZTI21HSpz8m2Ij1kXVtJOkg2pPlSHl8VcRYcG4G/wjzrgzrW2V7VpcaRuq8tR1f2jbiEz2sSZFHWb9JeXKa37LxMuWmHZlV/bL7Mj9VybhWHDQSvpuyT2PXrhe3j3qfRRL7YtdZ9lFJMTOp76T5zzvPylwwIQ/I4jMzM1N9tV8rZFlnI+G7HEbpwaPG+mEc/HbS9sq9YKPbICmmDPL5wtBewVGpVGwE73v+y86tHwHYxlW5GsCPhWbLAC6vVCq0c71NUqcnmAAOLC4EwRSBcfcvLuD4iVN47JmXsXf3XKTswOJC7Gl9qUCC98hK+uCv/+4HZ3D8xKmwrqz//YsLKFhGpP/pqaK0/6eXXovJZVNI+1eNJetDQ4PHJoV/byqsx5t7+kPRlutStMdPF2Cy5kaEyGX33r0zoufmy+yY7mvBYQXLiMWIvbvnsPXK8tD5UibLvXfvxOyWcswHbNpffJO1ndpkx+ZBZnOxLR/3WDvTNLBvT1w2m2JkIbPJWsZNE8DWLWXce/dOPL30GvbtmU/MM/bunsPVV26SypNX9qT6SWUFy5ByUsEyBpJvqeQ8fuIU7rkr3n7zZXbk3sEEHUR77tszj8effRlPL70m1WmUfTUv8vhHlvV98KMLMV665645HD9xCgc6tmPz9psfeneMX2e3lCNzwbc1E2TJ4jeijzL5ZT60d/ccHn827pt7d8/hsWde7okDTAAHP7ogXdP79szj6aXXlGtNZutJz93zrFXRPjJ77V9cwEuvnVf2xV+n7dnSYmZS3zJeUemRNs98vsSPpcoD8vjM/8/e3UfJcdd3vv/Ur7q6e0Ya2WMjCbMYCYgpY7MJQtY4uQRwsGHPQgzrcImFIVjJQpKzuQffmIe1TC4P2cVyYG3W3ptLDpBEJkDk5DoGvObuIZgYExKQH3STC5iKebCXBGPJ1hiN1NMP9XD/qKru6u7qnmmpe6a79X6dw0Hd9etf/epX39/jtLvWetwDsL6meW25Wr329mZOkzroN6YMkxVFa/MNS9d1L5L0Xz3P+8XkdVnSI5J+SdIPJf13SX/ied5fdnzubkmf8Tzvs67rvlfSszzP+51VmeauPgAAIABJREFUnHK7pB/2++tkv6crr/TE41T6pMz06cMbSkYn0te2UblodGLZ73oqcsmxFTTzb39afGrz5jk9/dNK29ON+z0dOfsk5Gz+7dcy+BOWx8kYfAPaWjnVKduuFWJ3raRPw01jZZKfhluacVStt66lXDQT+QDC1KD3ZpJiN9uXFR1bURipEUQyxpJtWzJGCvxIQRDJGI2sDyuWC6r7kfzk3I5tSUG4Lv1lXlkKtpXEdNy3Z59U3Dm+ZcefomMUBpIf5o9v6WfDUDLGkqw4jzCQIktSFPWs837nTV9HxlKjkX2v9xOW1yhupRVit/O6Rj1u2rZRZFsKQsk28c9speeeKRlV62FbLJw5V9ZTTx3vmdcgZe+Xvt+xYrnQnEN1zmtOJR5XKqcsSZGlMIz7g6JtKZDkB5GiSLItS7ainteQnfvZxshxLFVrQe6xQZ4GPil97krxkZ13rdS+C5KMbcX9dxTJtizFX8Zs9RnZ+2bJSuJYKtiWbEn1IGrrexTl9/En0yZbMZrc+6JRrR4mZZTC5H1jS/VG2BabBWOa73eeb7VzU9s2CixLJtumbUu2kYIg6tvW8up6VH3QuMTuIG21s35mSnayPmv1m8u1oO/YlH09yJpwpXvVWe64LGHz/jsFo2rNP6k1Wna+tNI8YLUxk8bzWo97qy3XCmnGInYz5dHl7/z8qjO+66bXD5Sez0zGZyYlbqdp3X+yWnt7mTn3aVQHzTElmcOtNO89mdhdyz9kPk/xt54lSZ7nVV3X3SPpDkllSV9U8rBB13U/KekLnud9QdJ/kHSb67q/J+l/SnrTsAoUBKEsJZUQRKpnXofJf0KYHutV7dVKI5NHoEolaL32A9X8oJlHI1DzWNCWf9Dzx70b9aAt/3omj85yZdNm8w9zztUrDyCrksT3Ocmkr1KZ3GipLbdfS215cq9Fmq570ynbl7X3X1IUqNln2cl7o7rydPOsIGnzWev7x69sWRRIQXLd2fGhkamIzvEtO/6EyYDTawxIP5vWr9T67astSbz1qvN+501fq63caiv3uOq8rlEXOQhCKWj9Nlm2vqqV7P2LY8GY3vO/QcveL32/Y/Wq3zVf6fW5QeJxpXKm0nhN48motWHR7xrqba8D+UHvY5MQq4MaJD5Wat+B4nhsvpew1OqnO+9bGsdh0Ir1bN/T8c+TKneqPUal+nLQLEv2vGGQF5tB8/2T7QPSzbu2Nu23XveL07y6nnaDtNXO+qlWwrY5UjW7PusxNp3smnCle9VZ7rQsadn85lpx8DVa5zqx3zxg0JhZ63EPwPqa5rXlalU76qB6mtVBOqak1z+Kee+abUB7nvcXkv6i4717JP1cTtq3Zf79mKRLRl0+AAAAAAAAAMBwnR4/aAIAAAAAAAAAWHNsQAMAAAAAAAAARoINaAAAAAAAAADASKzlQwjXmi31fzDPJJj08o/COtfJdsUP0/RHeI6xjN1xK8+pmKZrkVZ9Pdt1msbusEzrdQ1qjethu0Yft9IUxO4kl32U1rFetmtK+lxia2VTVkfbNSWxu1bnOFnjWrYJLtd2TVHsYjpMYtwS39TBqPYXrCiKTrJIY+8XJX1tvQuBqfRcSY+OMH9iF6NC7GISjTpuJWIXo0Gfi0lF7GJSEbuYRMQtJtVAsTvNG9AlSbskPS4pWOeyYLqM+i+UxC5GhdjFJFqLb0ATuxgF+lxMKmIXk4rYxSQibjGp+AY0AAAAAAAAAGD98RBCAAAAAAAAAMBIsAENAAAAAAAAABgJNqABAAAAAAAAACPBBjQAAAAAAAAAYCTYgAYAAAAAAAAAjAQb0AAAAAAAAACAkWADGgAAAAAAAAAwEmxAAwAAAAAAAABGgg1oAAAAAAAAAMBIsAENAAAAAAAAABgJNqABAAAAAAAAACPBBjQAAAAAAAAAYCTYgAYAAAAAAAAAjAQb0AAAAAAAAACAkWADGgAAAAAAAAAwEmxAAwAAAAAAAABGgg1oAAAAAAAAAMBIsAENAAAAAAAAABgJNqABAAAAAAAAACMxzRvQBUnbk/8HJgmxi0lF7GJSEbuYRMQtJhWxi0lF7GISEbcYC9McgM+W9MOnnjquMIzWuywnZX5+VouLlfUuxlhZ7zrZvHnOWoPTjF3srne9D9M0XYu0+us5XWN3WKYtbk7WWtfDGsWtNOGxS3zmW896mZY+l9ha2bTV0bTErjTe92ZcyzbJ5Rq32B3XujwV03hNEvOFrGm9x4M43etglPsL0/wN6IlXKNjrXYSxQ52sj2mq92m6Fmn6rmdcUc8x6mE8cV/yUS+njjpcGXU0vsb53oxr2SjX8EximVcyjdckTe91nQzqgjoY5fWzAQ0AAAAAAAAAGAk2oAEAAAAAAAAAI8EGNAAAAAAAAABgJNiABgAAAAAAAACMBBvQAAAAAAAAAICRKKx3AVzX/X1J/6ukSNIfe553c8fxF0v6pKRNku6T9Nue5/lrXtAhs20jX1IQRio5toIwkh+Eso1RuWhUrYd6/MkTsm1bJcdSvRHJD+Pjtm0pCEMpsuQ4lqq1QOWSrTCU/CBUGEoF29JM0ehELVAUScayJCuSIkszJaO6HykIIwVBJGMs2bZkyVLBSH4YyU/ej0IpSM47WzJarodxmYu2Gn6oIAhl20ZOwahWD1SwLRVso1ojVBiGKthGpaLRci1QGEaaKRVU7/hctearYBsVHaNaPYzPZxsVC0bLNV/GWCrYlqwgUhhGimyjIIxkG0sFSUEQrrquOz/T79g4K804qqZ1lcRMbbkx9PPMzjo6UQv1+JPHZdu2NpSMKpXhn2ctzMw6qmSuZbZktDyh1zLJiuWC6n4kP4xUKsR/A40k+UEUx1e91T/MFo0qtVAbSkYn0veNiV/XWvG/oWQURGq2iaJjK4oiNfxIxkgF28gYS9Wa39bHpp8v2lKjHqxvxWDidPbDMyWjhh+pEUSyrEiSpSiMVC7ZqjVa8es4RkEQdo2Vs8Xu/rU8G5/DDyLZtiXbWAqjSFEUj1sFY8kYq9mvpbFs20aRbclPxs2CMXJsqR7EY3/RMQqDeHw3xiTzA03F+Ih2ebFQdCzVGlFzvuUULDUakcIojqmZolHVj+eljz95QoWCHc8JJdnGku+HiizJtjL52nEsBmH82k5is96I47HUnOPFbaLhh4p7f0thGMmyJNu2ZBSfp94IVHRsWZbU8KNmOykWLNWrE78MwBhZqa/La0O2omaa8qyj5Voo25Ykq20tZoxkjFGj0VrfOE78+vGnTsjYthzbUqRIQSCFUbIuM2p77RSMojAeX9L2ZhtLdT+UUzAKk7WbbSw5jpHvh83Xtm1JihSEUhhGcgpGlqRGppyOWXkexJiwek7RVj1Qcy3fa545yHoujbPsnKPaYx0zSL7p+qhtvZ+Tb5pnOt/ol+cozi+11qXZ+X/eujSt/9XM8wdZ6672/IOmHTf1RqDNm+cG+ky15mvp2PKISoRpsq4b0K7rvkLSKyX9rCRH0ndc173b8zwvk+zTkt7med43XNf9Y0lvl/SxtS/t8Ni20WKloRv2H9T8XFlvfe0LdcuBQzq8uKwt8zPau2dBB770XX3z20/o4gu36k2vPl837D/YPH7N7h0qFW395Zf/SVe+ytUP/+Vpnb/9bD19vNYzny3zM3rHlTt019e+r7e+9gItV319+M8eaMuzXLI1N1vU0Z/WdOe9j+jylz1ft94e55ctx8/+zDP0mpc+Vzfedn/z89ddvaAvfv0HWqrUdeWr3LZj1+9Z0Fcf+pGWKo2cz+3SF7/+Q83NOnr5S87Vvsx1pnn+4/ee1DW7d2jz/Iwe+8kx/ec/+WZb3vOzTs8JULauOz8jqeexcZ5QlWYcHX662lZXe/csaMuZ5aFuQs/OOnp8sfs858yXJ2YATc3MOvpJzrU8c77MJvQaKpYLevKntWbf95u/8iJJUrUWaONMQUsVq+sezZ9RzI3D+x76ke786g+ar88+s6S9/9ffan6urF+//ELd/NkH2/q3MzYW9bl7v5/b1+zds6BnzBXZhMaqdfbDF1+4VVe/9gItLtX0hfu+3xw/X7XwHF10wTO74u07P3hSFzz3GTrw160x+vqkT0r71/Kso8OL1bYx6j9efZHq9VAf/fOH2uL7U3c/rMWlqvbuWdDmM0o6Vmlo8WhrTnDxhVu1+9Xna1+PeUc6P3jTq8+f6PER7Wzb6EQjaIuFK17xvNw+MO1T3/3ml2j7vzpDTy+1zynf82sXaaZcUL0R6sCXvqs3XvYC1epB33lnZ2zel5kL3v7XXts8MzsXlaT/5+uP6rW/+DxVqo22c1y/Z0HPOKPEJjSGot86If1jeGcb2jI/o/fuWdCZs46ckq0nFqu676Ef6dKF5+inx+td8VooWPr9T36zqy/O9uvGMm3jSedaau+eBYVRqD+4rbV2u/aqnfrywcf0Sxed23bO665e0O1/3d4Oz9hY1J998WE9vVTXb/7Ki1StBV3lPLvPPGilekKLU7T15FJ9xXnmIOu58qyjJ3Lmwlvny12b0IPku9r10SB5juL80urXpaut/0HyHGXacVR0bF3+zs8P9Jm7bnq9lkZUHkyXdf0JDs/zvirpl5JvNG9RvCF+Ij3uuu42STOe530jeWu/pDeudTmHzZeaA/gbXnlecwIgSYcXl7Vv/0FdumubJOnSXduaadPjtxw4pKUTdV26a5tuvO1+vfgFW3V4sdI3n8OLy7r19kO6dNc2HT5aaW4+Z/M8drwu349082cf1KW7tjUXBZ3luOKS85qTovTzN952UFdccl6zTNljN+w/qMsWtvf43P264pLzdNnC9mYn3ZlnWj7fj5qbz9m8+y1BsnXd+Zl+x8ZZtR521dW+/QdVrQ93Aniiln+eE7XJm2hWelxLZQKvZZLVGlFb33fseF3HksXaxplS7j0KGsp9/7KF7W2vG/WomW+6+Zwev+XAIR1J+q68vmbf/oNi7xmD6OyHL921TU8cjcfh7Ph5yc5zc+Nt4cJztO+29jH6ho7+dbkWdo1Rx47Xm5vP6Xu3HDikN7zyvGbetUbULEu2fPv6zDvS+cGkj49o50tdsdCrD0z71PO3n63DR7vnlB/+swdkyWrOLZdO1Fecd3bGZnYu2DnPzM5Fjx2v64pLztPTS9Wuc9yQxDgwDCv1dXlt6PDisj6UpFlO5peXLWzXkSSGO/M6sric2xenaY4dr3eNJ53rpX37D+rY8Xrbezd/9kFdcUl3f37jbd3t8Mjisi7dta0598orZ795EGPC6tWD/HlrZ/0Osp5b7rGOWc5ZxwyS72rXR4PkOYrzS6tfl662/gfJc5RpgdPNuv8Eh+d5Ddd1PyjpXZL+UtK/ZA4/S9LjmdePS3r2IPmfffbGUy7jsB1erDQ7pLlZp/nv1vFlzSXfQOp1vFwsqFyM/x2EocrFQt98Ol/3ytNYrXTZNNnXtsn/vG16l9c2rX+f7LG0bJ1pZFk9/zORbF13fqbXOfvlt5Z6xe7jTx7PLXcQRjpniOVeq/OshWm6ljzjEK9Zq4ndzr4pCMMe9yj/fdt0p0vz7dW/9etrgmjlWBi3el4v01wPq50zdPYp2bE1G4NRFOXGW/p+dztoxWFev7XSWJ/m0ZkuW6Z+845JGR8HMWnlPRm94vbwYqUrFvrN4ST1nVNm54jpe51p+s07s31wv746LWevcozj2H06xNkorMU6rd+96bdO2Lx5LrcNZdMEQdiM7V7p0piW8vvffv11r3zS93q15852mK4b28rf8ZnsPKizzlaqp7UyTu2s91z3xKrmmYOsTdY77Xqff7Cyrq7+x6Gs62GUfe44tc9hmLbrGdSorn/dN6AlyfO897uu+weS7lL8ExsfTw4ZxT8Ql7IkDfSno6eeOq4wHLNvSthGW+ZndHhxWUuVRvPfqS3zM1pK/vOMXserdV8NP9KW+RnZxqha9/vmk33tFKyeeYZRqZkumyb7OgiV+/kg7F3e9L/OOpVjYZSfRlGkI0d6/Ecfmbru/Eyvc/bLby07ol6xa9t2brltY/Wuh5OwVudZC9N0LZ02b55b1TWMW+ymfZGkZj+Wf4/y38/+F59pOql/nxmEpebrrvNY/WNhtfU87da6HtZ68rfaOUNnn5IdW7MxaFn54236fucYne2T8vqtlcb6NI/OdNky9Zt3nMr4OI7Ws92OQ58ru3t+2G8OF3+k95wynYf1m0vmzTs7z5E3z0zfr9b9pJylnuUYt7F72saHsYjdIVnx3vRZJxw5spTbhrJp0jlKEPbun9OYlvLnKP366175pO/1as+d7TBdN0rq2XbTeVBuna1UT2tgNe1sHGK355qjY545yNpkvdOu9/mHUtacef56lzU1DnGbdbLlmbZxcJquZ1Cj3F9Y15/gcF33/OQhg/I8ryLprxT/HnTqnyWdk3n9TEk/XrsSjkZB0vV7FrRlfkZ3fOURXbN7RzyIS83fCLrn/sckSffc/1gzbXr8mt07NLehqHvuf0zXXb1L/+8/PaEt87N989kyH//G4z33P6YtZ83qPb92UVeemzYWVShYuvaqnbrn/sf0jitb+WXLcee9j+i6q3e1ff66qxd0572PNMuUPXb9ngV9+eCjPT63S3fe+4i+fPDR+PehcvJMy1coWPq937i4K+9+f0XJ1nXnZ/odG2floumqq717FlQuDrc5byjln2dDaV27jZMy2+NaZifwWiZZybHa+r5NG4vatLGoa3bv0PHlWu49sh3lvv/lg4+2vXaKVjPfa6/a2dW/bU76rry+Zu+eBRXtta8PTK7Ofvie+x/T1rPicTg7ft774I9y4+3gtx/X3qvbx+jrO/rXmZLpGqM2bSzqd9/0kq74vuMrjzTzLjlWsyzZ8u3tM+9I5weTPj6iXUHqioVefWDap3730ae05azuOeV7fu0iRYqac8u5DcUV552dsZmdC3bOM7Nz0U0bi7rz3kd05ly56xzXJzEODMNKfV1eG9oyH/8GdEFxP53G9uYkhjvz2jw/k9sXp2k2bSx2jSed66W9exa0aWOx7b1rr9qpO+/t7s+vu7q7HW6en9E99z/WnHvllbPfPIgxYfWKdv68tbN+B1nPzfRYx8zkrGMGyXe166NB8hzF+aXVr0tXW/+D5DnKtMDpxoqi9ft2sOu6r5H0QUm/qPibzv9d0p94nncgk+Zbkn7L87yvu677cUmPeJ73kVVkv13SD8fyG9Bqf5JwybEVhPHTxtMnxVbroYIokm1ZKjmW6o1Ifth6mnEQhlJkyXEsVWuByiVbYai2p5HPFI1O1AJFkWQsK3nKvaWZklHdjxSE8f+MZcm2JUuWCkbykycpG2MpCtUsx2zJaDl5gnmpGD/BPAgi2Xb8dOZaPVDBtlSwjWqNsFmOUtFouRYoDKWZkq26HzYf7OEUjKq1+HPFzBPSbdtSsRB/zpj4Cc1WEOnMM2f15NOVgZ7A3O+pzYM+0Xnz5rm1WPVs1wqxO8jThU9F6wm+cf1M0hN8O7WeshxfS7+nLE+SAf5CORaxWywXmv1PsRBPxCLFfdeGklGl3upXZotGlVqoDSWjE+n7SRx2xmUQqdkmio6tKIrU8KOk/zAyxlK15rf3sat4OnbqdP9LeGodvgG9VjtN2zXgnKGzH54pGTX8SI0gSn/FQlEYqVyyVWu04tcpGgV+2DVWzha7+9fybHwOP2kTtrEURpGiSAqT+DfGUr0RyjZWM5Zt2yiyrWROIBWMJceW6kHS9hyjMIjHPmPS+YFOaXwcR+v8Deix6HPzYqHoWKo1ouZ8yykYNRqhwihSwcTzx6ofteaUBUvGshRJso0l3w8VWZJtWQqCSEHyOWMsBWHUHZu2pVJmjlcuxXPIVBhGsqx4fpv+p4/1RqCiY8uypIbfmtsWC9bYPYBw2saHcYndYVjNvVmpr8trQ7aiZpryrKPlWijblqQkXaZNGGOp0cisbxyjetL3W5Ylx7YUWVLgR4qiSJaJ+/ogyceYuI1GYTy+pG3BNpbqfiinYBSGyTrRsuQ4Rr4fNl/bdnw707bpFIwsqS0vx7TmQb3qbL3HhFV+A3osYtcp2qoHrTV0r3nmIOu5NM6yc47OBxCeTL6t9VGcttf6qJVnfP/75TmK80vZdWmctte6tFn/q5jnD7LWXe35B007LnGbKc9JPYRwysbBqbqeQY1yf2Fd/3Dped4XXdddkHRIUiDpDs/zDriu+0VJ7/M87wFJb5b0Cdd1N0l6SNKt61fi4QmCUJaSxV7yn1QV4gOqLQeyJJ2T3Pi0v4yPS2EQ/xaJJPlB/L6fTMZN8j/50rIf5H7FvVqJM0zPL0lRkEz4k9cmOVc2zXLyOVuSX22VX77k+4FsSVEoNRpBWzlqfut1o+1zgXw/aOZR94O2PBvpseSaJckYS1YQJu9HWs1zw7J13fmZfsfGWW25kSl3HDOjUKk02mKxUpmUGuq23HEtyxN8LZMs3TiwJQWZB5IYxX1Mtg9YTvqESvb9IP+11OqvwrY+VQqDUKG6+9j0dYNQwEno7IfTsTU77lqS6sthW7z6SX/dOVZW/O5ATBeXaZtIW4yluA0pjN9L+7U0loMglILMnCCQGkG2jbTG8+zANw3jI9rlxUI9O79L5nBpTEVhKxaNpK1JbKWxl279pr+J18wnbL3OxmbeHC+dQ6bSL6ZFQSsGs315tp3Ux2vvGVNgpb4urw1l01ST+WVzrZKmS9pAWztJ1jeWpGcmbStIPmcl/8trS36mTaZtIUiOh8lcKl2v+UmG6eswk382fTav1XwdgzFh9Rr19rV8r3nmIOu5NM465xynmu9yR7691kdpnuk19ctzFOeXWuvSNG2vdWla/6uZ5w+y1l3t+QdNC5xO1v2/nPE87wOSPtDx3msy//4HSQtrWyoAAAAAAAAAwKnih2gAAAAAAAAAACPBBjQAAAAAAAAAYCTYgAYAAAAAAAAAjAQb0AAAAAAAAACAkWADGgAAAAAAAAAwEmxAAwAAAAAAAABGgg1oAAAAAAAAAMBIsAENAAAAAAAAABgJNqABAAAAAAAAACPBBjQAAAAAAAAAYCTYgAYAAAAAAAAAjAQb0AAAAAAAAACAkWADGgAAAAAAAAAwEmxAAwAAAAAAAABGgg1oAAAAAAAAAMBIsAENAAAAAAAAABgJNqABAAAAAAAAACPBBjQAAAAAAAAAYCTYgAYAAAAAAAAAjAQb0AAAAAAAAACAkWADGgAAAAAAAAAwEmxAAwAAAAAAAABGgg1oAAAAAAAAAMBIsAENAAAAAAAAABiJwnoXwHXd90v61eTl3Z7nvSfn+G9IWkze+oTneX+4hkUEAAAAAAAAAJyEdd2Adl33MkmvlrRDUiTpf7iue4XneXdmkl0kabfneX+/HmUEAAAAAAAAAJyc9f4G9OOS3ul5Xl2SXNd9WNJzOtJcJOl613W3SbpP0rs8z6uubTEBAAAAAAAAAIOyoiha7zJIklzXPU/S1yW91PO8R5L3Nkr6C0nXSvqepP2SHvM8772ryHK7pB+OpLA4nVlrcI7tInYxfMQuJtFaxK1E7GL46HMxqYhdTCpiF5No7OL28nd+fqDM77rp9QMWB1Ni4Nhd729AS5Jc171Q0t2S3p1uPkuS53nHJb0mk+4mSX8iaTUb0JKkp546rjAcj032QW3ePKcjR5bWuxhjZb3rZPPmuTU71zjF7nrX+zBN07VIq7+e0zV2h2Xa4uZkrXU9rGXcSpMbu8RnvvWsl2npc4mtlU1bHU1L7ErjfW/GtWyTXK5xi91xrctTMY3XJDFfyDrZ8kxTXExrnK/WKPcXzMkUaJhc132ppHskXed53m0dx57juu5vZN6yJDXWsnwAAAAAAAAAgJOz3g8hPFfS5yRd6XneV3KSLEv6sOu6fyPpUUm/I+nOnHQAAAAAAAAAgDGz3j/B8S5JZUk3u66bvvdHkl4n6X2e5z3guu5vSbpLUlHS30q6aT0KCgAAAAAAAAAYzLpuQHued42ka3IO/VEmzR2S7lizQgEAAAAAAAAAhmLdfwMaAAAAAAAAADCd2IAGAAAAAAAAAIwEG9AAAAAAAAAAgJFgAxoAAAAAAAAAMBJsQAMAAAAAAAAARoINaAAAAAAAAADASLABDQAAAAAAAAAYCTagAQAAAAAAAAAjwQY0AAAAAAAAAGAk2IAGAAAAAAAAAIwEG9AAAAAAAAAAgJFgAxoAAAAAAAAAMBJsQAMAAAAAAAAARoINaAAAAAAAAADASLABDQAAAAAAAAAYicIwM3Nd9xWSzpJkpe95nvdXwzwHAAAAAAAAAGAyDG0D2nXdT0j6t5K+JylK3o4ksQENAAAAAAAAAKehYX4D+lJJL/Q8b2mIeQIAAAAAAAAAJtQwfwP6R2w+AwAAAAAAAABSw/wG9Ndd1z0g6S5Jy+mb/AY0AAAAAAAAAJyehrkB/QvJ/78t8x6/AQ0AAAAAAAAAp6mhbUB7nvdLw8oLAAAAAAAAADD5TnkD2nXd/+p53v/uuu5dir/x3MbzvNed6jkAAAAAAAAAAJNnGN+Avif5//97CHkBAAAAAAAAAKbEKW9Ae553V/LPv+k4FEmqrPR513XfL+lXk5d3e573no7jL5b0SUmbJN0n6bc9z/NPqdBryLaNAlnyw1C2bWQbS41GINsYGVuqN0KVS7YajUhBGMo2RuWiUSOI9MTRikLLqGBbKhSstjS2LYWhZIylepLfhpLRiVqoIAw1Wy7IDyL5QagwlJyCpTCKpMjShpJR1W8dKzlGfhgpCCIVC5Ysy1KtEahUKsj3QwVBqKJjKwwjBWEkYyzZRgoCKQxDOY5RGCi+RmM1g8qXJEtSZCkMQxljqeQY1ZNzG2PJsY0iRao3Wp8NgnDVdetLCsKo7byRbckPIllWXIAwjP9dsC1ZQbTq/CHNzjo6UQv1+JPHZdu2NpSMKpXGUM9RLBdUy8R2ybFUrw6/iZfQmb0WAAAgAElEQVRmHFXrrWspF41qy8O9Fqws225nywXVGnEfY9tGG4pxHyYrUtp2bWO1+jnbqOQYWZbU8Nv7N1lSoxFqplRQELaO2XbcX1lBJGNbqgdq60eDIGrrs7L9CX0FUk7RVj2QSkWjWj0eZ4uOrSiK1PDj8azoGPlJXDqOURTGMVQoGFlWa6x2HCMpStJGyVhoKVQcw/F4HQ+fjmNUT9tI8tkoisfrSPGYHkaRjp2oKrLtZmwX7bjc2XjPlr2QzEdqjUC2sVS0LdWDqGs8pU1Mv9KMo0YyX4wiJXM8S0EYKQwjPXH0hIplR0EQKpIUhVIYxXGrJAZLxbhfrtbDuE+142PGGElxPxuEUTKfNWo0AoWRMv17PAcsOkaNRhyjxph4LIikcslO8m7FclAPFAShnKKtQK3yxuWKi2bsuB1ZQfwfaGbjOS/me8V33nyTtjD90vsex1Ucj7axctc86dojDCOVkg644UcKk/62VDSq1lpjRxhGOvJ0RVHBbs6Bypk06XxHUaRaI5KsSJasZMyI5zaWpIKx2ubQ5aKJ20pmTl1rhHH7y6w9bWOpXLS72r6lSEcWKzJFO17nRa01lC0120wxWftFViTbmGYbz865+rUR2zZtdVYwRrZan8mr+8WlaryunoC2V551tJxZP82UjKo566d0bZK9f73WJmmeadpeeUqt9Vuatt/6bWbWUSWTdrZktJyTNk2XXlOvdIOef5A6WG1ZB8lzkLXuIGvWdN6YnZc16kFuWuB0MsyHEH5d0rMkHVO8+XyGJN913SclvdHzvL/r/IDrupdJerWkHcln/ofruld4nndnJtmnJb3N87xvuK77x5LeLuljQyz3yNi20dOVhj60/6AOLy5ry/yMrtm9Q5+6+2EtLlV1ze4d+s4PntLOF27Vjbfd30zz/rf/vJZONHTzZx/U4cVlXXzhVu1+9fnal8nnuqt36WuH/lkXXfDMZn579yzovod+pNmyo//l556lp5dquuXAoeZn3v2Wi/R3//gvuuzibc1j83NlvfW1L2xL11muvDTXXb1Lt/+1p6eX6l3H3rtnQY5jdNvd39HlL3u+br39UM/ruGb3DpVLtj7+V9/S4lJV1+9Z0Pyss+LkwraNFisN3ZDJ6717FjQzU9CRIxV94b7vt507Pdf8XEkbHHsiJi/rbXbW0eOL1bb7tXfPgs6ZLw9tE7pYLujIT2td59h8Rmmom9ClGUeHn+6+li1nltmEXkPZdvuqhefoogue2bwnt1z7ch07YenAl76b23az/dymjY5+8mSlrd9511t26h/+6Yh+4WfP0U+P19uOXXvVTm3a6ChqSB/8xDe6+tFXXbxNjUbY1levti/C9HOKtp5cquvHh5f0rC1z2rf/4MDjYjaG/9Nv/4KWq0Hb+NU5Fr77LRfpn/7nUV3wvGd09VszZVt/+oVvN9vJz/7MM/Salz63bR5x/Z4FFR2jDyTx3mv8/dTdD+vMuWLXsQ+8/edpE6eB0oyjp0/U2+aLF1+4VVe+ym2Lp7Sv3PnCZ7b1ze+4cofu+tr39aZXn98Wb1vmZ/TON+/U7ExBxzr64+uuXtDtf/1dffPbT3TFYed533HlDj348E/08pec29UONm10VHIK+mml0TZnzpbrdS9/vsolW3OzRS0v+814zmsPveI7b75JW5h+eff9PW+9SL4ftcVadu2Rrqt+81depGotaIv7vXsWdOBL322OD3nrlHQdd+dXf9AW5x//3P+nf/eKn9FH//yhrjGjXCzoU3d/R9/89hO64hXPa2sr/fr9C54737Ye7GxzL9vx7La2eO1VOzW3wdEHP/GN5vj3hfu+rzde9gLV6kH3nGuD03O9ZdtGJxqBFo/WutaPZ846ktRV99m+ZtzbXnnW0RM566et8+W2DeNB1iarzVMabP02M+voJzlpnzlfbtvYXW26Qc8/SB2stgyD5DlIWQdZs6bzxs60z5grsgmN054ZYl5flvTrnufNe553luJvNe+X9MuSPtrjM49LeqfneXXP8xqSHpb0nPSg67rbJM14nveN5K39kt44xDKPlC81J7uS4k2WA4f0hlee1/z3JTvPbQ7waZrDRyvNyY0kXbprW7MDS9PceNv9umxhe1t++/Yf1GUL23XJznN1+Ghrcyb9zEc+/YAuW9jeduwNrzyvK11nufLS3Hjb/bp017bcYx/af1BPHK3o0l3bmhOrXtdxy4FDOna83ryGG/Yf1Gq2HX2pOTHJntf3I91y4FDXudNzPXG0sqr8IZ2ohV33a9/+g/E3VIek1ohyz1FrdP2c/Cmp1vOvpVof3wnsNMq220t2ntt2TzbOlLRv/8GebTfbz4WBuvqd//LpB3XJznN1JEmfPXbzZx/UE09VdPhoJbcffeJopauvXm1fhOlXD6R9+w/q/O1nN2N20HExG8NB0D1+dY6FH/n0A1q48Jzcfsu2TFs7ueKS87rmETck4/BK4+8bXnle7jHaxOmhWg+75ouX7trWFU9pX9nZN996ezzf6oy3w4vLuukzD8q2TE47ifv59HU2DjvPe+vth3TZwvbcdhAG8Ryic86cLVfarnw/aovnvJjvFd95803awvTLu+9LJ+pdsZZde6RjQ/aPLmm6dH6Tjg95c510HZd9HQZxvKabz+mxNLYPJ+stSV1tpV+/37keTI+nba6zLd782Qebc6jsNSyd6L7Wmz/7YN/1li/piZxzfyhpV3l1n+1rxr3tLfdYPy13rJ8GWZusNk9psPVbpUfaSkfa1aYb9PyD1MFqyzBInoOUdZA1azpv7EzL3jMw3G9A/5zneb+evvA87w7Xdfd6nnfIdd1i3gc8z/t2+m/Xdc9TvGn90kySZynepE49LunZgxTq7LM3DpJ8qA4vtibjrfeWNZf8dffw4rKiKOpKUy4W2t6bm3Vy87FNd362kaIo6sojezx7rFfe2XL1SpM9b+excrGgclGruo40bTMvy9LmzXPqZfPmuZ51a6xW2Xqda6X8x8V6xq4kPf7k8dw6DMJI5wyp/tbiHGt5nvUybvHcK3az7baz7wvCsG/bXanfTN/v1feVi93DXV6fmD3W2VeMWz2vl2muh7zYffzJE0l/EZ7SuJgeS8epzuOdY2G/OM+eP50L5OWXGrS8q20Tk2DSynsyTna+8PiTxweec3a+n6bv7GOzc7K8z+S9HuS8URQp6NPnZ8vVWY5e15gX373mm4wPw7EWc92TuTd5971XrGXjazVjQL+5jm3aX3f299ljaZtLx43OttKv3+8391lpTEnzXWkd2Gu8OLxY6TvG9MqzeT1jMg71it3VrjkGWZusd9r1Pv/klfVEftpo/dedo+xzx6FdDtO0Xc+gRnX9w9yALriu+yLP874lSa7rvkiS7bpuWZLT74Ou614o6W5J7/Y875HMofiHElssSQN9ZfGpp44rDIf7bcpVs422zM+0dUBb5me0lPwnHVvmZ2RZVleaat1ve2+p0sjNJwi78wuS3zztzCN7PHusV97ZcvVKkz1v57Fq3VfDj1Z1HWna9LWiSEeOLOVW6ebNc/GxHnUbRq2y9TpXv/xXspYd0brGriTbtnPr0DbWSdffepxjLc+zHpptYhXp1krP2M20286+zzamb9tdqd9M3+/V91Xr3d+ZyesTs8eyfcVq63narXU9rPXkLy920/4jjdGTHRfTY+k4tdJY2C/Os+dP5wL9Yn7Q8q6mTUyC9Wy3Y9HnrsC27YHnnHkxlNfHZudkveKu8/Ug57UsS6ZPn58tVxiVVnWNufHdY745zePDJMTuap30vcm5771iLRvnS5WGnEJ+352N837tLPu6s7/PHkvbXDpudLaVfv3+pg2lvmvFfudL8+13rX3XW7bpO8ak/+7VpldaJ66VXrG72jXHIGuT9U673uefmrJa+evOcYjbrJMtz7SNg9N0PYMa5f7CMH+C4zpJ97qu+1XXdb+m+Cc53inpg5Lu7PUh13VfKukeSdd5nndbx+F/lnRO5vUzJf14iGUeqYKk9+5ZiAdLxR3PNbt36I6vPNL8970P/kjXXb2rLc2Ws2Z17VU7m+/dc/9j8W8XZdJcd/Uuffngo2357d2zoC8ffFT3PvgjbTlrVtfs3tH2mXe/5SJ9+eCjbcfu+MojXek6y5WX5rqrd+me+x/LPfbePQvaetas7rn/Mb3jyh19r+Oa3Tu0aWOxeQ3X71lY1V9FCpKu78jrvXsWVChYumb3jq5zp+faetbsUP/qMs02lEzX/dq7Z0EbSsPrNkqOlXuOkmMN7RySVC7mX0u5OMwuECvJttt7H/xR2z05vlzT3j0LPdtutp8ztrr6nXe9ZafuffBH2pykzx679qqd2nr2rLacNZvbj249a7arr15tX4TpV7SlvXsW9N1Hn2rG7KDjYjaGbbt7/OocC9/9lot08NuP5/ZbQRS2tZM7732kax5xfTIOrzT+3vGVR3KP0SZOD+Wi6Zov3nP/Y13xlPaVnX3zO66M51ud8bZlPv4N6CAKc9pJ3M+nr7Nx2Hned1y5Q18++GhuOzB2PIfonDNny5W2q0LBaovnvJjvFd95803awvTLu+9zG4pdsZZde6Rjw6aNxa64T+c36fiQN9dJ13HZ18aO4/V33/SS3DFjS7LektTVVvr1+53rwfR42uY62+K1V+1szqGy1zC3oftar71qZ9/1VkHS1pxzvzdpV3l1n+1rxr3tzfRYP810rJ8GWZusNk9psPXbbI+0sx1pV5tu0PMPUgerLcMgeQ5S1kHWrOm8sTNt+oBo4HRmRdHw/ursuu5Zkl6u+Oebvu553qLrunOe5+Vun7uue66khyRd6XneV3qk+Zak3/I87+uu635c0iOe531kFcXZLumH6/8tUqNAlvzmk4EtNZKnD6dPIi6XbDU6nmAcP5U4eTKwbalQsJI08ROYbVsKQyVPDw+aT5hNnzg7Wy7IT55sHEaSY1sKo/hp5RtKRlW/daxUMPLDSEEQqViwZFnxE5NLJVu+Hz/RvOgYhWF8/uYT0pPyOU7+E6F9Kf7OemQlTzGWSo5R3Y+SpzhbcmxLkaLmE5lXerJ49q8xeU8ll1pPoraS/4wrDFtPcF7pqcwr2bx5brg7o/m2awxiV8o+xTiu435PBj5ZgzxR+FS0nogcJU//7v1E5EkywF8oxyJ2s+12tlyIn86ePAF+QzHuw5SUNEzuVdzPhbJtSyXHyLLib/z4QaQwiuTYlmRJjUakmZKtIGz1n9knshvbSp5I3epHgyBq67Oy/Um2rzjd/xKeWodvQK9F3EorxG76NPNS0aiW9CNFxyiKIjX8eHwrOkZ+EpeOYykKk/hKxtV0nHOceGHj+2HHWCjV/TAeryVZkeQ4RvVGPA6nn42ieLyOFI/pYRRptmyrmnkyfLrIycZ7tuyFZD5SawSyjaWibakeRF3jab82MQnW+RvQY9HnrqQ04zTnnFGUzPHs1hzPsiwVHaMgCBVJikIpTNJJkqK4XViWmmNsITmWpgmS2Irns0aNRqgw6ujfTXyeRiNszjXjOaRULpk476AVy0E9UBCEcoq2AlkKwiiZa7bKZew4CyuI6yYbz3kx3yu+8+ab0zw+TErsrsap3Jv0vqfrLVnx/c9b86RrjzCM24MUz1PSdVypaJI+urWmkhV/4Tc7L03TpPMdRVH8G7NWJCuJc8uK5y+WpIKx2ubQ5WLSVjJz6lojbZemufaMz2cnbT9UFMXXmN54Y6QgiNt6uoaypWabKTpGYSBFViTbmGYbz865+o0Xtm3a6qxgLNlqfSav7p2CragRrLROHIvYLc86Ws6sn2ZKputhgVJ2bdK6f73WJq0847S98pSy67ewuUfQa/02M+uokkk7WzJdDxZsTxdfU690g55/kDpYbVkHyXOQte4ga9Z03pidl/V6AOG4xG2mPLr8nZ8fKPO7bnr9tI2DU3U9gxrl/sIp/xHRdd23eJ73add1r+049ALXdeV53s19Pv4uSWVJN7uum773R5JeJ+l9nuc9IOnNkj7huu4mxZvVt55qmddSOkgWJMmPfz/ElqRACoP4fb/qx5OI+AOqLced09b0xvuS7yuTJv6sFOeXfq5SCZpp6kkna5L/Ra09HVUqQduxoB42Pxcm/W1BUlDNvJ/8cL+dnDRUnJ8tKay3PqMgUtq1ZqMxveZ6tj6CeHKT99nVCIIwUyeZzwbtX+1P/9gY8sP/A6tUGrIknZPEYho7w1TviP9RPaChttx+LWk7w9rKttt6ck/i/jFQxQ+6+40w08/5Ut1v3bds/6YkTaMath1L+9343N39qBX/U5Ly+xNA8aLBklRfDrrGxTSe/Mz4FiXjoi0pasS/JdZK14quvLEwO143/KAtZrOfTdPYkjZt2KBaZanZjzaCVpr0s9myy4/jPo33RpAf/7SJ6VfLzBclxX2u35rjdS5C0vezP4hXT8bTZryEzaza3/clP4nprv49kBpJfKdzxmYZK0FXLKfSBX1euaKg+3f8Vor5PD3nm5hq6X3PxmMaXr3WHkZSUOueh9T87rEjbVtpm8mmyc53uuZFasV2XWqbQ9ey/Xwyp86+TteeCiLVllvlzF5cWi5L7WuoMHOuMNPuQoVt5V7NnymDIGyrMwVtTT637ufnN0zMhlC1Y/1U7bF+qmXnwZk9gH55pml75Sm11m/ZPYJeljvSLvdIu9xxTb3SDXr+QepgtWUdJM9B1rqDrFnTeWPnvAw43Q3jv2I5L/n/f51zrO+fVzzPu0bSNTmH/iiT5h8kLZx06QAAAAAAAAAA6+KUN6A9z3t/8s+feJ6391TzAwAAAAAAAABMh2E+geuXh5gXAAAAAAAAAGDCDfNBsj9wXfdLkv5W0vH0zRV+AxoAAAAAAAAAMKWGuQF9NPn/50pyFD9serIelw4AAAAAAAAAGJphbkDfIOlzkv6V4ofiPil+lgMAAAAAAAAATlvD/A3o/ybpw57nnel53hmS/rOkPxxi/gAAAAAAAACACTLMDeitnufdlr7wPO9PJW0eYv4AAAAAAAAAgAkyzA3oguu6Z6UvXNd9hqRoiPkDAAAAAAAAACbIMH8D+r9J+obrurcr3njeLemjQ8wfAAAAAAAAADBBhvYNaM/zPi7ptyUVJc1K+g+e531sWPkDAAAAAAAAACbLML8BLc/zviLpK8PMEwAAAAAAAAAwmYb5G9AAAAAAAAAAADSxAQ0AAAAAAAAAGAk2oAEAAAAAAAAAI8EGNAAAAAAAAABgJNiABgAAAAAAAACMBBvQAAAAAAAAAICRYAMaAAAAAAAAADASbEADAAAAAAAAAEaCDWgAAAAAAAAAwEiwAQ0AAAAAAAAAGAk2oAEAAAAAAAAAI8EGNAAAAAAAAABgJArrXQAAAAAAAAAAwHDMbZpRuTTYtm+9EYyoNGOyAe267iZJfyfplz3Pe7Tj2Psl/YakxeStT3ie94drW0IAAAAAAAAAGH/lUkGXv/PzA33mrpteP6LSjMEGtOu6F0v6hKQX9EhykaTdnuf9/dqVCgAAAAAAAABwqsbhN6DfLul3JP24x/GLJF3vuu4/uq77f7quW167ogEAAAAAAAAATta6fwPa87y3SZLrul3HXNfdKOmQpHdL+p6k/ZL+D0nvXW3+Z5+9cRjFXDebN8+tdxHGzulSJ+MWu9NU79N0LdL4Xc+4xe6wjFs9r5dprodJjt1pvi+n4nSol1HH7elQh6eKOjo5a9HnjvO9GdeyUa6VrTZ2x6nMwzKN1yRN73VljbLPnbb6m7brGdSorn/dN6D78TzvuKTXpK9d171J0p9ogA3op546rjCMRlC60du8eU5HjiytdzHGynrXyVp2ROMUu+td78M0Tdcirf56TtfYHZZpi5uTtdb1sNaTv0mNXeIz33rWy7T0ucTWyqatjqYldqXxvjfjWrZJLte4xe641uWpmMZrkpgvZJ1seaYpLqYpzkd5P08m73H4CY6eXNd9juu6v5F5y5LUWK/yAAAAAAAAAABWb6y/AS1pWdKHXdf9G0mPKv6t6DvXtUQAAAAAAAAApkq9EQz87d5qzdfSseURlWh6jOUGtOu6X5T0Ps/zHnBd97ck3SWpKOlvJd20roUDAAAAAAAAMFWKjq3L3/n5gT5z102v13T8aMdojc0GtOd52zP/fk3m33dIumM9ygQAAAAAAAAAOHlj/RvQAAAAAAAAAIDJxQY0AAAAAAAAAGAk2IAGAAAAAAAAAIwEG9AAAAAAAAAAgJFgAxoAAAAAAAAAMBJsQAMAAAAAAAAARoINaAAAAAAAAADASLABDQAAAAAAAAAYCTagAQAAAAAAAAAjwQY0AAAAAAAAAGAk2IAGAAAAAAAAAIwEG9AAAAAAAAAAgJFgAxoAAAAAAAAAMBJsQAMAAAAAAAAARoINaAAAAAAAAADASLABDQAAAAAAAAAYCTagAQAAAAAAAAAjwQY0AAAAAAAAAGAk2IAGAAAAAAAAAIwEG9AAAAAAAAAAgJFgAxoAAAAAAAAAMBJsQAMAAAAAAAAARoINaAAAAAAAAADASLABDQAAAAAAAAAYicJ6F0CSXNfdJOnvJP2y53mPdhx7saRPStok6T5Jv+15nj+M89q2kS8pCCPZxlLJMao1IgVhqA0zBVXroYIgVKFgZCxL9UYg2xhtKBmdqIUKwlC2bTRTNFquB1JkyRjJKRjV6q3jBdtSFElBECkII5UcoyCM5IeRCsaSMZbCKIrThJFsy1KpZHRksSLfMjLGkmNbiiRFSTo/iGSMVLCNLCvOO0quKwwjOQWjMDlHqZCcLwhVsI3KRaPlpPxFx1YURWr4rWup1MIkD0tWet3J56rJ5wq2kW0s1ZI6KdrxueuBJCuSJUt+GMlYlmxbsmSpYKRIluqNVt2UCpbq1fh2OkVb9UDxMWPkOJaqtUC2sVSQFAThMG47BjA76+hELdTjTx6XbdvaUDKqVBrrXayTMk3XIk3X9di2UWRbCoKkD7QtbSganUj6YNsYWUaKQklWJKdgq+G3jpWKRgVLrX7ZGMlIVqS2fkpGUijNlIwatUB20VatHsYJo7gfNpaa/U151pElqZLWc8HWhmJ7Paf3Ic0jiOI+PL0f6fG0v7VtS/WO8cH3wxX7uNlZR5V6KD8IZYyRY1sqFay2a87GQOf4tqFkt41bJceoUvVlm7hvry23rqmzL7ZtqVwwq4q3YrnQHEdtYzRTao03tjEqOZZkWa0xMnkvHQcmSWcdpxMqX5LjGDX8SGEyDsqKZJtkXA7CtjHVMpKScd1xLFmRVPcjGWPJtq3mNwXqfihjrDixLDkFS2EYn//xJ4+rULA1m2k3JcdWEEaKFMdmFEWykrJIVlcMVOut8b1QMKrW/FZc1UIFUZScXzJJewzDVp4F05prZK8vzHwuiiIVkrF/OYnngh3Pc/LmWNm5WBpP1STuOmOtYMd1WHKM6n5cjsNHKyoUC1KQ5GEbBbLkh3FdFmxLjt269rQ/ievFUrloN48VHVtRGKkRhFMzL8nGcNExCkPF989Ysu04MJ9eqkoFW34QqlwqtPpe26hYiOe/xXTOGcRzR2PF/a4xRk7BkizJb0QKo3h+KsX3Kkz6+3j+3crXsqQomdRGUaRy0Y6PJ3Nky7SOS/G8O4ri89tGyVgS5zVbjOMhSs5pxaEYn9tYitQxptiZ9pAZY+p+KN+P5DhGUXKtaRt9eqkqy7HV8OO5uWMbRYrbrG3F128l5fTDuOBBcv0FO14HRIoUhZKfXGO5aCRLqmX6/YIdp5MkI6strkPLUiOI69M2lhp+2BzPJHX1VZMQu/3WaZ3rhM5js0m/lY5Z8VgUtI2J9aQPy8ZFkKzNZotG1aQfiaL43gVhnM5YlvwgVBiqdU8iS6WiURBE8ZzDNs224AfxPfPb7rl0+GhFpRlHQRCv19KYtEw8LygV2+MyXU9GURwFUdiaMxULRnU/UMmxW20pHYPrrTziNWrQvJ4wzPTrkgpGWlyqyrcsFYxRwYnXbena0sqshzvHbts2km3UCMLkOo0cIzXqwdoGzhhY7Rw9O0fsnMeNW9pB1h3jUtbV5Dkz67T1FbMlo+U1vi7gdLLuG9Cu614s6ROSXtAjyaclvc3zvG+4rvvHkt4u6WOnel7bNlqsNHTD/oM6vLisLfMz2rtnQQe+9F0tXLBVz3v2vPZljl2ze4c+dffD+vevu0DHnELbsb17FrRpg6OPf+4f9e9fd6GePl7vOh5Gof7gtgc0P1fWW1/7Qt1y4FDz+LVX7VShYOnDn3pAhxeXdfGFW7X71ed3nf/MuZIqy74+8ukH2t4/Y2NJ9xx8TDtf+EzdevuhtnN0nu+KVzxPL3/Judq3/2BuWdI6+Oa3n2i77gueO9/8XGedLC5VtXfPgjbMFPTHn/+W/t0rfkYf/fOH2tKVS7bOnCvp2FJD+25rr5vNZ5QUhZGeXGqvt+uu3qUvfv2H+sfvPanr9yxoftY51duOAczOOnp8sdoVy+fMlyduAJ2ma5Gm63ps2+hEI9Di0VqzL3r76y/UBc97Rtv1vePKHbrra9/Xm//tC+X7Dd3Y0Y9smCno9z72d13pX/fy5zf7qfS93a8+X1vmyzq8WNWBL31Xl7/s+br19lY/eP2eBW2dLyuQdKRPPaf3IS+PNF16n+bnynrH7herVg904233t/VzXzv0z3rFS87V/KyTuykwO+voJ4vVtvHqP159kWzLdI1h58yXVasFbePbgd//N13xku1f9+5Z0JYzy6otN+QU7a6++Nbfffmq4q1YLujIT2vNdHlj2Qfe/vOqN8Kucm8+ozRRm9B5c4gPvP3n1WiE+tYPnuyK3/e89SL5fqSbP/tg23UfO15VueTops88mDsmp+OnJH38r77VFcczZVvv+6O/1+HFZb37zS/Rs7bMtY3vX7jv+12xmf38A9/5iSrVRtf4nsbHUqXedQ/zrqXX9WXnE+l5f/VVLyQix2cAACAASURBVOiK3c75xHd+8KTKRbtrLrY3aZthGLXFWlrmh3/4VFfdX7N7h+bnSporF3R0qa4PraIdPfCdn2i2XGjmlXdv0nnJJGzk5cnG8PxcWb9++YVt9++a3Tu0eX5GleW6bth/UD/7M8/Qa1763K7+68GHn9AFzzu7K27T+3ntVTs1N+vog5/8hubnyvrNX3mRqrWgmT6vn0hj5fKXPV8PPvyTrvjMHk9juegYffmbj+llO57dLOPFF27VW197geqNUJVqI7c9vOfXdqlcMvr9T36zZ3nSWH56qZ5bT2dsLOrPvvhw29y5XLLb2uyDD/9Ely48R7VG0Hb9af6FgqX/lJShV195ze4dmi07CqM45maKhTiuj9f1oT/Nb0/v3bMgxzH6wCe+MVGx22+dltZz2k/NzTpdMdKZdu+eBf348JI+8pmH9OH/7aUKNpVVbwT66fF6blxcv2dBQcfa7Qv3fV9vvOwFqtXb79/vvukl+txXv9fskz/6mUM6c66o3/yVf61qLWjGX+c9v++hH+myi7fpxHJD/+XT7TH1nR88pZ0v3NrW3t73tp9XpdrQHV95pKu81129oLM2FXXk6WrfMbhXm82OC2m9XXzhVl35KretDJ19dTp2t+Zyla5+8uy54mm1Cb3aOfogc/n1Trve5x9VWWeS+XVn2mfOl7s2oUd1XcDpZhx+guPtkn5H0o87D7iuu03SjOd530je2i/pjcM4qS81JzWSdHhxWfv2H9Slu7bpxS/Y2uww0mO3HDikN7zyPJ19xmzXsX37D0qRpUt3bZNlmdzjx47XdXhxWW945XnNgTk9fvNnH9TSiXrzvUt3bcs9vyWrufmcff/IYkWXLWxvTkSy5+g832UL25t555UlrYPO685+rvNY+jnfj3Tprm3NzedsumPH6woDNTefs+erNSLVA3Xlf+Nt9+uKS+L8b9h/UJOzNTEdTtTC3Fg+URvfBUsv03Qt0nRdjy/picyCRZIWLjyn6/puvf2QLt21TQVjmpvP6bG0/8lLn+2n0vf27T+oalKHl+7a1uw708/esP+gKrVQtRXq+USfPNJ02f7WktVcyKXpbrztfl22sL1vH3eiFnaNV8eO13PHsBO1sGt8y4uXbP+6b/9BVevxNeX1xauNt1ojakuXN5Y9cbSSW+5aI9IkyZtDPHG0og/tP5gbv0sn6s1Nq/S9ffsP6tlbNummzzzYc0xOx89jx+u5cWxbppn+/O1nd43vebGZ/fwlO8/NHd/T+Mi7h3nX0uv6svOJ9Lx5sds5n1i48Jzcudi+/Qe1XAu7Yi0tc17d33LgkJ44WlE9UHPzOT3Wqx1dsvPctrzy7s2kz0uyMfyGV57Xdf9uOXBIvh8101xxyXm5/dclO8/Njdv0fsb5VprnOXa83pY+L8bSWLn19kO58Zk93vz80XgunC3jpbu26fDRip5eqvZsDx/+s/t1ZHG5b3nSWO5VT0cWl7vmzp1t9rKF7TqSxFze3Dvblnv1lbccOKSnl6rNPqEZ13/auz19aP9BPXG0MnGx22+dlr5O+6m8GOlMu2//QZ2//WxJ0tlnzMr3Ix1J6qrXPKBz7Xbprm1aOtF9/z765w+19clveGXcd4aB2uKvs3yXLWzX4aOV5uZzeuyWA4d0yc5zu9rbkcU4bV55b7ztoIKO8fv/b+/cw+woqkX/6957ZpIJCSSSIFzR6FGWqAcJj6AowlFQQB4qKCAoURH4lAtXQCXgAzi+Xyi+xYN41KPeQ0BRAYEgik8CyFuWXCURNArmKq9JZmbv3eePqt7Tu6d7z57Hfq/f9+XL7K7q6lVVq1atqq6uytLlvDabbEtxub1sz6dNkiFtq+O+O8uXi8uxj+aegcZ99On48u2O2+7nN0vWkZy4Iy3Ml2H0G21fAa2qJwCISFbwDsDGxO+NwFOmk/6TnrRV5vXYGa69tpmFwwOUK5Vph5UrFRYODxBFUWb4vEFX1AuHB+qG14sTBuTeWwgnwpL3p9PKi5fOZ/p38r6suLF89fKXVzZl/zliVlghnPg7/j5u6dKF9AN5utsqNv798dz62r7L6qCX8gKdn5/p6O5D/xhh3mCxJj95tmLh8ECuHfRfkE6Kn7ZT8bXYlufZrHKlUv17cpgr57ge6qWRtLd5slftaxBk2res+k6XWVK2QiGoCcvrt5L2dSJPT0yKm9/v1epbWs6scqkndzfpbpYPEectS3/z812rI/X8g3mDE9fiuFFiH4KstOr18/H9ef17IWy8DvPyl+VPxH/nxY3lKtf1GbLD6vlg5Urj9RKX61R1k9dmW81M/IWkDjfid+bpST17Hf+d9IHjazFT6Wg9/zP5f9oXTj4vfU86rUZ88Hq6O2+wWG2fWdfifMTPaaQPy9PNpKxApl7nlX8yvNN1t944Lfm7ZpwwRdy4Xy9X3PYkcRlPpRdZvkTWs+L2EMdz28fk13msE422q6nkTffVefHqtdlkPhtpC3HfneXLVeNEc9e/d4LOxuTpbqM++nR8+XbHbffze1XWdtDM+YVOap/topfKoFl5afsE9BT4nRGrBMC0Xh1t2vQ4lUrGyqpCyLLF82uMw7LF83lsZJxli4enHVYIQx4bGWfbbeZnhm8Zc2sNXBr54fXiVCJy7y1XhqphyfvTaZUr5IYl85n+vWjBUN24sXz18hcEQU7ZBdW/02Hx14HLFs+vbvj38MOP0S5aaVRydbdFFAqF3PpqZx3MhF7KC8wsPx2ru4WQLWOlmvzk2YrHRsbZeqtsW5R+3ITNrrVT8bVCGNb8zrLp8d955RzXQ700kvY2T/bYLhNFmfWXVd/pMkvKRhTVhCXlSD83L0/JuHn3p/UtfW9WudSTOyvvrXbkGtbdDB8izluW/ubnu1ZH6vkH46Woei2OGwQTs1ZZadXr5+P7y5UoVz8arcO8/GX5EwPF/PYd/x0Ebu/UfJ8hOyzPdmwZc3s6Nyp3XK5T1U1em4UOtrkxhal1L+l3Jv3HZJx69jr+O+kDp+t/Kh3Ne25ax9O+cPJ56Xtm4oMn85N1f9w+s67F+Yif00gflqebSVmBTL3OK/9keMfrbp1xWvJ3sh+bKm7crxfCkFK5XC3jqfQiqWv17FfcHuLnbrvN/Lp1HutEo+1qKnnTfXVevHptNllujbSFat+d4ctV4wRz42svXbpwynQ6QXcb9dGn48u3O267n9+rssZ0gt4mmak83TimzqOXyqCZeZlJ2p2wBUc9HgS2T/x+MhlbdcyEInD2qpXOCcMZhdWrVrJ23QZu+/3f3J6YibDTjl7BmuvvY9MjI5PCVq9aCUHE2nUbiKJKZviirQZZtng+a66/j9OOXlETfvrrd2fhgsHqtbXrNmQ+PyLincftMen60sXDXHfTek49asWkZ6Sfd91N66tpZ8kSl0E638n70mHxfcViwNp1G3jHMbtNirdoq0HCAqw+fnLZDA0EDBaYlP5Zx+/J5Te49M9etbLj35b0GguGwkxdXjDU6WZjMr2UF+it/BSB7ZYM19iim+7eOCl/px61grXrNlCqVDgrw44U/aAwHT9pp+Jrq1etZJ4vw7XrNlRtZ3zv2atWMjwUMjRFOS+ok0YcL2lvIyLOOn7PSXbuupvW17VxC4bCSf3Voq0GM/uwBUPhpP4tS1+S9nX1qpXuwCvItMWN6tvQQFATL6sv227JcKbcQwOp5X8dTpYPsd2SYc5ZtTJTfxcuGOT01+8+Kd8PPvQoZxy7e26fHPefi7YazNTjclSpxr93/aZJ/XuWbibvv+GWBzL791g/suowKy95+Uv6E/Fzs3Q37U/cdPfGTF9s9aqVzB8KJ+laLHNW2Z929Aq2WzLMYAHOabAd3XDLAzVpZdVNt/slSR1ec/19k+rvtKNXUCwG1TiX33Bfpv264ZYHMvU2rk+X7nD1OYu2GqyJn6Vjsa6cetSKTP1MhlfvX+J84aSMa9dtYNmSYbZZOC+3PbzrDXuydPH8uvLEupxXTksXz5/kO6fb7HU3rWep17ks3zvZlvNspTsPZl7VJlT1+k357emcVSvZbslw1+luvXFa/Du2U1k6ko67etVK7l2/CYBNj4xQLAYs9WWV5wekx25r121g4YLJ9feOY3arsclrrne2MyxQo39p+a67aT3Llgxz5nGTdeqGWx6Y1N6WLnZxs+Q96/iVFFL9d5Yu57XZZFuKy23tug2TZEjb6rjvzvLl4nKMD6rvFxr1mabjy7c7bruf3yxZh3PiDrcwX4bRbwTJzzfbiYisB/ZT1fWp63cBJ6nqL0TkK8B9qvrxBpJcDtxf7y1PvdOVkyevF4shYZB9QnuhEDJ/0J0CTuRONXYnDLsT4wv+lPUocidel6OIoaI/Kd6ftByGAZUocnH86ddDQyFjY5XqKdsDhcCdeOzjlSoRYeDSDgKXdhQAEdWTiiv+GdXnld3Jy/MGQzaPVqqnnkdRxHjJfbY9f9CdGl2JIgYKgT/tuELB37fF31f0p2yP+jKJnYuxMhBEBAT+tGh3QniAO1k5wqUXfyY+VJw4QXlgsMBYfHJ56nTr+MTuRt5+N5OlSxe2YpZkOVPobquYOME3PjW8e0/w7aW8wPTz08m6WyiERIWAcnniRPcFgyFPjFWqNjEMoVIBAmdj3cnw3nYPhhQDt+daOXLxCSGI3Oqw0VKleo0KzB8KGR8tUxgsMDpWcd/WRFCJIsKAqr2ZNzxAgNsjLilX+rCTJ0Yr1bTj58f1kTwFe3CgQKEQMDY2kV6x4E67j5+Zx/DwACOpPmGoGOSesJ3u3xYMFWr6raGBkJEtJQphyLzBkNHNE3masMXu3kIB5hXDhvRtcF6x2o8WwpD5Q3F/434PDQQQBK6PrPa9Qe4BhC3SW5iB7qbLOJ7QKQEDAyHjpYiK76sJnC5WfF+c7FPDkGq/PlAMCaKIsbK7r1AIqisFxkoVwsR3+gMDIZVyRLkSUfb9+3DcbsoRQwOu748AIuc/BEHgGgbBJB3YMubqqVgIKRZDtoyWJvTKt634+WEYEFVc/gKfv2IYVH2NZP4q8X0BRJWIYiFkqBiw2etzsRh4H6tS1a1YV5O+WKxPW7zeTejahK9VKlcYGggZK3k5/HXKPo1CSJnA+VAhFAsBA4WJvBfCkKHBuFwC5g0WqmGDAwWiSsR4uVLjl+TRyTY3JqnDgwMhlQrVcisUnGEshM7elsoR84YKE7a3EDBYdP7vYMLnHCi4+hwdd/o6UHRtvjTufcui0+hSOaISJf3vCZsQBP6jtwCiCs5GjU/0B0E4EU7k+oQomqjzsm8XBd8mxkquHZTKUbybGxX/rCiAgUJYk69qe0j0MWO+DAaKIZHPa9xGi6HTvfFSrc8+luh7Ai9nyddT2ee/GAaEfpwQp1sInAwEMJqw+0WfLvgVRAm9rgQB417+QhgwXqpU+zNgkq3qBt2tN05LjxPSYcOx3fL3ur6oXNMnjlWo2uTAf3Mb68XwYMiWUkSpXCHydefGNRAGQVV/i2FQtbFDgyHlcsSoHzcNFEPCwOkdeJ33Y6jQ62l8T8nb00IYEITOLxgarG1v8Xgyipw+R7HPEbq2OFaqTLSlciLfYxNpuDFquZqfSiVh1/FjtdDZ/2IYUhxwtrlSgYFiPC4sV/vzZN9dKIRQCBlP5HMgZM4OIGxwBXRH6G6jPnrSR0z7cZ0Wdzrjjs6Rdeo05w8P1NiK4aFw0gGEzc5Xp+htQh4OPeP700r8B588vCNX/86UXiqDZuZlJrrbkS/AReRK4H2qejNwLHCRiCwCbgUunKvnlMsVNzEKUI4YS/weHRmfCBsvU6nGKzMyUp4IK5XZUnK/AaIyjI0nwitQ9n1zfK3sD3uKw2MXsHoPMDZSnuhoy1BO9d1xvErieixDAagknlHzvBJVeYupeHFYiN/7pOL2P4nDRkvJfEM5USbj5VoZYjniMomAsXQ+S5D8KnB8LJF+uUypPFE3fXZ+Rccw4tvB9l4XR0a6tyZ6KS/QW/kplytQrrUNI0l7g7MjsX0pJcPKMLa5zBi1NjQ2GuWMa1t8WZU3VzJtVlyS8URXTTmXass5rof4pmL1erkm3NnbEhVq81kuud9T1V7stMZ5LpdhZIwam5nUgXT/NjJSqem3xkrl6n2jm2ufXmuLXT8z4q9NpW9jW0o1Mm0ZqbXrY4m+olp/Xaq66TKOsxEAJR+WXPgV9/XpPjVK6E40Vpnod31YnG4hmQiuHcTPS+pn2teIdTztIaZ1IKkfpURYXNfJ59dMgsXlkZO/5H0BUClX2DyWiDtOwseixseq8cW8PlXlr9E11yYCYKw84dcsXVI7YVFOhMW6PUpKRzeXq3U6urlS034n7u0NvySpwxXfEJNlA/CkpcM8/PBjFIHSlkqN/Rr3epL0JaNK0j+c8F9jXYjjxr4mFaeLSb85jh//P7a5tj+I+4uYZFtI29jNCZud1NdYLwOgNF7r345m9DHV/CX9Zlwb3ca3v3Se033PeEqWOP+VhC8c35O0y+k6SSTp/i7X+vIVJvdnWbaq06k3TkuPE9Jhm0dq+6zavsj1iTFuMt/9HetF3NfHOlMZm/AXKtTWX3w9qSeUJvQy+ZzQh1WAZYkJ1aStjPzf6fZWTr2nTbaZca+zYyndrcm3lyluh2n7HAHjwNKlwzy85bFqGVfHheOJcWGiP49xvlylJp/du8xjdjTqo4+k+rh6vny7405n3NEpsjaS5uZUvja3IV+G0U90zAS0qi5P/H1w4u/bgZXtkMkwDMMwDMMwDMMwDMMwDMOYObYRjWEYhmEYhmEYhmEYhmEYhtEUbALaMAzDMAzDMAzDMAzDMAzDaAo2AW0YhmEYhmEYhmEYhmEYhmE0hY7ZA7oJFICaE+O7kW6Xvxm0uUyWAw/iDhRvFh2pu50mz2zopbxAw/lZTp/q7lzRq/maLi0uh+U0X2+hB3S3m2VvJm0sl+X0iM013ZqaHiuj5fSI7rbqGTOlU2XrYrmW02G626llORt6MU9g/sJs6VW9mA69VAbNml8IoiiaOlZ38mLgxnYLYfQkTwfWNzF9012jWZjuGt1Is/UWTHeN5mA21+hWTHeNbsV01+hGTG+NbmVautvLE9BDwJ7ARqDcZlmM3qLZbyhNd41mYbprdCOtWAFtums0A7O5Rrdiumt0K6a7Rjdiemt0K7YC2jAMwzAMwzAMwzAMwzAMw2g/dgihYRiGYRiGYRiGYRiGYRiG0RRsAtowDMMwDMMwDMMwDMMwDMNoCjYBbRiGYRiGYRiGYRiGYRiGYTQFm4A2DMMwDMMwDMMwDMMwDMMwmoJNQBuGYRiGYRiGYRiGYRiGYRhNwSagDcMwDMMwDMMwDMMwDMMwjKZgE9CGYRiGYRiGYRiGYRiGYRhGU7AJaMMwDMMwDMMwDMMwDMMwDKMpFNstgOEQkfcDr/M/f6Sq7xKR/YFPAfOB76rqe9omYBsRkU8A26rqKhHZFfgqsAj4GXCyqpbaKmAfkKyDdssyG0TkUOD9wALgGlU9rc0izRgROQ5Y7X9epapntlOebmc6Nrgf7FAjdldEngp8E1gGKHCsqj7eNqF7HBFZBPwSOERV1/ezfsZYu50Zc1Fu/dL+Z2MLRWQb4FvAM4CHgdep6l/bkpE+oVE72SGyfQ14MfCEj3Keql7eYpk6dvyZI1vbyywLETkBOCVx6enAN4Dv0QFlOVPq5GsBHVgPjZI1hjIfwZG2U20Wp+Vk2Z12ytMOROR84EggAv5DVT81l+nbCugOwHf0LwdWALsCu4vIMcDFwOHAzsCeInJQ+6RsDyLyMuD4xKVvAqeo6k5AALy1LYL1ERl10JWIyDOALwGvAnYBduvWNiUiw8CFwL7A84F9vB0xZsAMbHBP26Fp2N0vAF9Q1WcDNwPvbamgfYSI7AX8HNjJ/55Pn+pnjLXbmTGH5dbz7X8ObOEHgBtVdWfgIuAzLRG8T5mmnWyrbJ49gJeo6q7+X6snnzt2/Jkj26tpc5nloapfjWUCjgUeAj5KB5TlbMjJ17l0aD00Qp0xVN/7CDl2qm+oY3f6BhHZF3gpbq5kD+B/i4jM5TNsAroz2AicoapjqjoO/A7X8O9T1fv927dvAq9tp5CtRkSWAB8EPuR/Pw2Yr6q/9lEuoc/KpNWk66DLeTVu9cGDvp0dBfymzTLNlALOfi8ABvy/zW2VqLtp2Ab3uh1q1O6KyADwEuDS5PWWCttfvBV4O/AX/3slfaifKazdzoxZl1s/tP85soWvxK2ABvg2cJCPbzSHhuxkJ8jmJ8GeClwsIneIyHki0upxeSePP7NkeyrtL7NG+CJwNu7Lh04oy7kiztcI3VEPeWSNocYxHwEm29B+I8/u9A2q+lPg37zNWobbMeOJ+ndNj24yFj2Lqt4dGzwReRZu2X8F1whiNgJPaYN47eTLwDnAP/zvHbAyaTXpOuhmngkUROQKEbkNeBtdmi9VfQy3wupe4EFgPe5zKWMGTNMG97odatTubgs8mvg8sdfKoaNQ1RNU9cbEpbx66XX9rGLtdmbMUbn1Q/ufC1tYvceHPwosba7Y/cs07GTLyZDtycD1wJuBFwD7AG9psUwdO/7Mke1q2lxmU+FXUM5X1f+mg/RvtqTy1XbdnQ05Y6gxeqSuZkOGneorcuzOle2VqvWo6riInAfcA6wF/jyX6dsEdAchIs8FrgXeCfwRt+9KTIBzCvoCv+fUA6q6NnE5pI/LpNXk1EE3UwT2xzlJLwT2oku3FhGRXXCO39NwDm4ZsD2gZ0mDNrhn7dA07W76OvRIOXQJjdZLz+hnHv3ebmfKLMutp9v/HNrCIHW97/WuxXRsu1fVP6rqq1V1o6qOAJ8FDm6HLJ08/kzKpo6OKLM6nITb8xk6WP9mQDVfnaS7MyFnDPVyeqeujFmSsjv3tVuedqCq78e9MN+ROd6OxiagOwQReRHuDcNZqvp13Bu57RNRnkx/fQ5xFPByv1L1fOAw4AT6u0xazaQ6EJEL2izTbPgrcJ2qPqyqm4HLcZ9ndiOvANaq6kOqOor7VGy/tkrU5UzDBveybZ6O3X0I2FpECv769vROOXQD/aifk7B2OzPmoNx6vf3PlS38s4+HiBSBhcCmpktvxHRsuxeRfxWRIxKXAtw2AK2Wo2PHn2nZOqXM8hCRQdy+wlf4Sx1TlrMhna9Or4cGyBtDdX1dGbMnwyb2FSLybH8gJ/4F02W4/aDnDJuA7gBEZEfcKbmvV9Xv+Mu/cUHyTO/Uvh64ql0ythpVPUBVn+cPPXgfcIWqvgnY4g0DwBvoozJpNTl18I52yzULfgi8QkS28W3qIOCWNss0U24H9heRBSISAIcC69osU9cyHRusqhvoUTs0Hbvr90a7ETdRA/BGeqQcuoS+08801m5nxlyUW6+3/zm0hVf63/jwG318ozV08lgqAD4tIov9vuAn4hZGtIxOHn/myNb2MpuCXYDfq2q8X2pHlOUckM5Xp9fDVGSNoX6K+Qh9T47d6TeeAVwkIkP+5dPhuIMp54ziXCZmzJgzgXnAp2TikMkvAauANT7sSiYOOOlnjsU1ikXArbhTbA1jSlT1NyLyMZwRHcB9WvO19ko1M1T1GhFZgZtAHwduAj7SXqm6muna4H6zQ3n5fRvwdRF5D/An4Jg2ydd3qOoWEVlFf+untduZMVfl1o/tf7pl8V7gEhG5G/inv99oEVPYybaiqneIyIeBX+B80jWq+u0Wi9HJ48882dpdZvV4Bm7VM9DZ+jdN0vnqBN2dMXXGUJdjPkK/k2l3VPVL7ROptajqlSKyEvgtbnuaNXM9GR9EUXrbMsMwDMMwDMMwDMMwDMMwDMOYPbYFh2EYhmEYhmEYhmEYhmEYhtEUbALaMAzDMAzDMAzDMAzDMAzDaAo2AW0YhmEYhmEYhmEYhmEYhmE0BZuANgzDMAzDMAzDMAzDMAzDMJqCTUAbhmEYhmEYhmEYhmEYhmEYTcEmoA3D6BhEZD8Rucv/fb6IvHGa958pIpc0RTij5xCRI0XkhpnoWiKNVSLyw7mWrdmIyPtE5PB2y2HMjKStNAwjHxG5UkSeM0WcS0TkzJwws5VG0xGR9SKyxxymd4KIvM3/fbKInDVXaRuGYfQajYwFReQwEbmwVTL1KsV2C2AYhpGFqr6v3TIY/UGf6tpLgXvaLYRhGEYzUdWDZ5mE2UqjG3kxcBeAqn6pzbIYhmF0NI2MBVX1CuCKFojT09gEdA8jIvsBHwb+BAjwBPAR4FT/ew3wfeCjwAbg2cBmYJWq/k5ElgJfA/4F2AT8FbhLVc9taUaMrkdE3gycAZSBv+P07S+qeo4PPw44AvhM4p5LcPr2CRHZgtPdlwPbAx9T1S+KyABwIXAA8BDwN+ARf//WPr1/BQaAtcA7VbUkIqM43X8+cCxwKPBqYAyn66tUdWPTCsRoGyJyPq7ONwH3+WuXMKFr55GhCyJSwungQcAC4GxVvSyV9guAjwFDOD29VlXf4sMOAT6A+/LoCeBkVb1dRPbG2eAFuPZxnqr+UERW4dpECDwNeBC4CDgF2An4lKp+0qf9FuBtPu4m4BRVvdfn61FcG9gRuAN4I3A8sAfwcREpq+rlc1C0RuvZSkS+g+u75wFvxdXx54FdgQi4CqerDds9EdkZZzufBBSAC1X1Yu9T5PkLW2c9F/gE8JiqvldEtgf+ArxUVX/i7f6hqnrUFDq8BOeH/FBV392UkjQ6GhG5DThDVdeKyDE433Sxqm4Wka8CdwLLgX1xOvtb4FRVfVRE1gNHqurNfhXoW4DHgJ8Br1LV5f4xe4vIL4HtcBN3rwdWYbay7xGRELgAeAGwEAiAE4Dbgc8CLwJKwPeAc3D9edb1AZwNnaSnqecdCrwHGARGgDNV9Vcici7wQmAH/+wzgC/jdPbJONv8Ov/cw4ADRGQzsBTYVlVPEZHnAp/D2fcI+KSq/qe37x8E/gg8z8t6kqr+Yi7KNex9CgAACnJJREFU0OgdROQi4KGMMdzFZOvtdmToqao+5O3zb4BdcL6K2VhjzhGRE3HzX2XcXMEpOB+16l/i+34/FjwYZ6vLwG3A/riXevvh/IlDROQG4Fc4e/tU4DrgRFWttC5n3YltwdH77Al8RFV3xU1ErAZeCewGvB3nxOwBfFZVd8E59d/w914I3K2qOwOvBfZusexGDyAiz8cZ8QO9jl2Bm5x7k4jEL8FOBOqt0BgC/q6qewNHAheIyDzchMVOwHNwk9BPTdxzAXCLqu4OrAC2BU73YYPAD1RVcB3R/wH2VNU9gGuAvWaXa6MT8Z9RH4GbJNsb2DoVviP5ulAARrw+vQ642L+kS3Ia8D5V3Qunk4eJyO7e+f4m8CbfBj4OfEREFuNs7htUdTfgcOCLIhLr8T7AyTjHfEfgaOBlwMHAB0QkFJF9cRPK+6jqCtwEeNKB3x04ENgZN0HzWlX9PHAz7oWMOfvdy1OAC3z//mXgXFy/vQn30mEP3GRzvLXAlHbP2+RLgbO8ru8LnOlfrkB9fyHruZfhXtqA08O/4mw1uAmSNQ3o8LCqPtcmn/uatB79A9hHRAKcPdwaN9G3u6o+H/ei4yPJBETkFbgJ5T1xdnFh6hn/CzfI3AnXtl5jttLw7IUbL71QVZ8DfB04Czgf9/JvZ5xf8SKczcy7fhZT6+mzgA8BB3t7eCJwmYgs8FGeBqxQ1eNwPsGvVPWFwDNwk35v8Lp6Ba5/+Hwi7aK/Htvwg4APicgLE/n8pH/u17wchpHm80wew11Nvt5m6mkivbtUdWezsUYzEJGXAu8C/s3b3f/CvRQMyPAvReRJON/2OO9f/wTnH2TxL7hJ6die7tusfPQSNgHd+9yvqr/1f/8B+Imqjqnq33ET0kuA21X1Rh/nYmCFb3wHA18B8KtBL22t6EaP8DLgx6r6AICqflpVDwTuB17pV9vtgJsAqcf3/f+34iakF+AGi//ldfoJ4FuJ+IcAJ/mVU7cAK3GTIzGxzv8Zt5LkVhH5BHCbqn5vZlk1Opz9gctU9TFVLeHsXZKpdOFzAKp6B27F3UtS9x8PbCMiZwNfAOYDW+EGn3fFtlhVL1PVg3ArmbYHvuf19ErciqRdfHrrVPUB/zb9fuAa//cfcIPbYdwLxWcCv/RpfAxYLCJLfBpXq+qoqo57mePrRvfzB1X9jf/7NmAZzgH+nKpGqjqKe7F3UOKeqezeTjiH+mKvTz/F6fEKf1+ev5D33J8DT/EvYQ7EfQVwgIgM4hz1K5lah38+65Iyup3LgYP8hPM+wKdwLzJegLOHh+Be4P3W69CrcC8BkxwM/Leq/lNVI9wkSpLvqeqIqpZxK6CXNS03Rlehqr/Crew8ydvLI3F9+/7Af6hq2fuh+6rqDXWuN6KnB+D8grU+zreACs5GAvza+y+o6mdwdvN0nM/xPC9XHjsB8+Kvt1T1L7ivYQ/04RtU9Tb/962Yv2Bk4HUkPYYrkqO3DejpjRhG8zgQ+K6qPgygqpcwMaGc5V++BLhHVW/38b+OmzPL4geqWvFfsfw/zGY2hG3B0fuMpn6PZ8QpJf4O/P9lfz1IhJXnUC6jfyjhJtUAEJH5uBUcnwfeDPwe+IqqRiJSL53NAIl4sW4mdTSpywXcas/f+eduk5QDeNynV/Er8PbADRouEJGrVfVd08yn0R3k6UsjupCMHzLZJv4MtwXC1cD/xa0mCpjcBgLcy5AC8Du/YjoO2wF4GLdFQiP2uwB8I3577z8V3gG3QhB8u/FEqfwb3U1SH+K6Dam1cyHuU+qYunYPt+rjEb/qAwA/efwIbrIvz1/IfK5/zg9xk3974VY9rcZ9VfVLVX1cRKbS4ccbLhGjJ1HVO/1Li8NwWyf9APguTh8vxW0tdJqqXgUgIlvhXtIlmcqnzWpPhoGIvBK3LdEncYsh7gWOY3LfviNudWfe9QJT62kBWKuqR6Xu/wtuy6THE9c/iltccTFuld4A9fW2QK2dhto+wvwFo1FqxnA4PcrU2wb01Pp4o5kUcFvNJQlwepile2lfAdzLlCzMZs4AWwFtAOwqIvGKuxNxg8J/Aj/C7ZUXf47waiY7LoYxFT8B9vf7fwKchFvhdiluVd2RTF6J2ihXAW8UkXl+S46jEmE/Bt4hIoGIDOE+OzwlnYDfIuQu3ETgh3Fbd+w5Q3mMzuYq4LUiso2f5Ep+AtiILrzRx9sNtwfuTxP3buPjvtuvLnoKbsVSAbe/3c5+70VwK6C+CfwaeJaIvMSnsStuciXvU68sfgwck2hfJ+P2O5+KErUTk0Zv8GPglITdOxG4Nh2pjq4rsNnv6RgPIO/CbVkA+f5Cvedehvv88U5VHQOux51PsSYh80x02OgvLsdtV3CNqt6L23bjWJx+xfo36G37RTgdS/Ij4Ahx+5WD828b8WnNVhoH4Fa6fRG3JcurcH37dcDxfjusIZxfu2+d643o6Vrg5SLybAC/F+kduC9R0rwC+LSqfgN3DsoBXi7I1tt7gXEReY1PewfctmST+gjDmIL0GK6e3tbTU8NoNlcDR8fbJorIm3BbxpVy4v8C2Cn2dUXkCCC9iM2YBTYBbYDbk/GDInInzqmKJ2XeATzbX1+DOzRgpD0iGt2Kqt4JvBO4WkRux30Kc7KfiLgUN4Hx9xkm/2XcYOAu3GTg/YmwU3HbdNyJc4LuxE18p+W7Hbda9WYRuRn3Rv/0dDyj+1HVK3GO8s24SeFHUuFT6cKLRORWn8ZRqvqPxL3/xA0kbxWRu3B7Pf4C9/nh33ATJV/3nyaeDhztPwc7AnfA1e241advUNX108jTNbg91q8VkTtwB2e9xn9iXo8rgA+LyPGNPsvoCk7FbR1wp/+nuIOlasjTdW+XDwdO8Pp0DfBenTiIKs9fqPfc63ArmuNJjh/jDnv5gZdlpjps9BeX4178xXp0LbDRb+/178B63KFu9+BWIZ2RvFlVr8dN+P3K6/zWNObTmq00vgTs5+3erbhtX54OnIdbWXc7Tveu9C+g8643oqf34F7gfcf7Bf8OHKaqWSv1zgc+4e3mFbjPyeOtOq4CThaR1Ym0x3F2+zR/z3XA+ar6k5kXjdGPpMdwU+htPT01jKaiqtfiFllcLyJ347ZLPIScVc2q+v+BY4D/9GO+V+Amq20ObI4Iosj8+35G3KnHn1PV52WEvQ34rboTbIdwezS9P/50zDBmg7iDKX4GvF1Vf91ueQyjHiISAUtn8bLEMLqaev6CYXQ6IrIHsLeqXuh/nw7slfxk3DAMw5gaG8MZvYqILMLt+X+uqo74r15/BOxgCyPmBtsD2qjHPcBn/f6Mg7jDW2zy2Zg14k6j/zbwBXNcDMMwDMNoMr8H3i0iJ+I+pf0TbsWeYRiG0SA2hjN6GVV9VETGgHUiMo47H+J1Nvk8d9gKaMMwDMMwDMMwDMMwDMMwDKMp2B7QhmEYhmEYhmEYhmEYhmEYRlOwCWjDMAzDMAzDMAzDMAzDMAyjKdgEtGEYhmEYhmEYhmEYhmEYhtEUbALaMAzDMAzDMAzDMAzDMAzDaAo2AW0YhmEYhmEYhmEYhmEYhmE0hf8BIVjZ9tXJxZMAAAAASUVORK5CYII=\n", "text/plain": [ "
" ] }, "metadata": {}, "output_type": "display_data" } ], "source": [ "sns.pairplot(auto_df);" ] }, { "cell_type": "markdown", "metadata": {}, "source": [ "#### **(b)** Compute the matrix of correlations between the variables using the function cor(). You will need to exclude the name variable, which is qualitative." ] }, { "cell_type": "code", "execution_count": 14, "metadata": {}, "outputs": [ { "data": { "image/png": "iVBORw0KGgoAAAANSUhEUgAAApsAAAJICAYAAAAuOC/lAAAABHNCSVQICAgIfAhkiAAAAAlwSFlzAAALEgAACxIB0t1+/AAAADl0RVh0U29mdHdhcmUAbWF0cGxvdGxpYiB2ZXJzaW9uIDIuMi4yLCBodHRwOi8vbWF0cGxvdGxpYi5vcmcvhp/UCwAAIABJREFUeJzs3XeYZGWZ9/FvzzCuBCXIIEkcMNxIDoOACzogiKBy6YIgQTAgIiLLC4YlOirJjCImDKAyKGIiiywMiAkQSTLcyyqgBB0RRkkyM3S9f5zTUtT2JKynTnfV98NVV/c5dU7VXd3A3PM7z/OcoVarhSRJklTChKYLkCRJUv+y2ZQkSVIxNpuSJEkqxmZTkiRJxdhsSpIkqRibTUmSJBVjsylJkqRibDYlSZJUjM2mJEmSirHZlCRJUjE2m5IkSSrGZlOSJEnFLNV0Af1i3v2/bzVdQy/N/exRTZfQiKEVntV0CT3XmvNQ0yX03IS1Vm+6hJ479vj7mi6h5/7j8blNl9Bzc4cHM2Oa9ufvDjVdQy/6hEkrr9P45xzNYP5bJ0mSpJ4w2ZQkSSpt+ImmK2iMyaYkSZKKMdmUJEkqrTXcdAWNMdmUJElSMSabkiRJpQ2bbEqSJEldZ7IpSZJUWMsxm5IkSVL3mWxKkiSV5phNSZIkqftMNiVJkkpzzKYkSZLUfSabkiRJpXlvdEmSJKn7TDYlSZJKc8ymJEmS1H0mm5IkSaW5zqYkSZLUfSabkiRJhXlvdEmSJKkAk01JkqTSBnjMps2mJElSaV5GlyRJkrrPZFOSJKk0b1cpSZIkdZ/JpiRJUmmO2ZQkSZK6r6+SzYiYBhwNzAXWBs4DHgZeDwwBuwA3A98HXgY8BOyTmXfW554KzAd+AayXmdN6+wkkSVJfGuClj/ox2dwSOAiYChwC/CUzpwI3AW8CJgO/yMyNgG8Dn42IScA3qRrPTYF5jVQuSZLUZ/qx2bwlM/+YmY8C9wP/Xe+/C1gR+AfwjXrfmcD2wIbA7My8qd7/tR7WK0mS+l1ruPxjjOrHZnNux/b8ju3hzGzV30+on3+C/vxZSJIkNWoQG6xlIuJ19fdvBS4GZgErRsSG9f69gdZoJ0uSJC2x4eHyjzFqEJtNgDdGxE3ATsBhmTkX2Bf4RkT8Gnge8FiTBUqSJPWDvpqNnpkzgWlt21Pavp8OEBEfzMz92s+LiAnArsA2mflIRBwOrFG+YkmSNAhaLe8gNNAycxh4ALg2Im4AXg6c2GxVkiRJ419fJZuLIzOHFrD/ZODkHpcjSZIGwRieLV6ayaYkSZKKGbhkU5IkqefG8Gzx0kw2JUmSVIzJpiRJUmmO2ZQkSZK6z2RTkiSptGHX2ZQkSZK6zmRTkiSptAEes2mzKUmSVJpLH0mSJEndZ7IpSZJU2gBfRjfZlCRJUjEmm5IkSaU5ZlOSJEnqPpNNSZKk0kw2JUmSpO4z2ZQkSSqs1fJ2lZIkSVLXmWxKkiSVNsbGbEbE3sAxwCTglMw8reP5nYGP1ps3A+/MzIcjYgXgLGAd4C/AHpn5p4W9l8mmJEnSAImINYATgG2ATYADI2K9tudXAM4E3pSZGwE3AifWTx8P/DQzXwKcDnxmUe9nsylJklRaa7j8Y/HtAFyemQ9k5iPAucDubc+/CLgrM2+tty8AXl9//xqqZBPgbGDniJi0sDfzMrokSVIfqBPJFUZ5ak5mzmnbXh24r237PuClbdu3A8+LiI0z80ZgD2DVznMzc35E/B2YDNy7oLpMNiVJkkobHi7/gMOAO0Z5HNZRzQSg1bY9BPwzGq0b0/2AL0fEtVSN5Ny2Y1nQuaMx2eySuZ89qukSeuoZh5646IP60PCf72i6hJ4bvu4nTZfQe2tMabqCnttg3uymS+i5kyY93nQJPfe+uUs3XYLKOgU4Y5T9czq27wa2bdtelbZkMiImAndn5pb19hbA7+qn76mPvzsilgKeBfx1YUXZbEqSJJW2ZGMqn5Y6kexsLEdzGTA9IiYDjwC7AQe2Pd8CLo2ILama0MOB79TPXUSVep4I7Ek1WWjewt7My+iSJEkDJDPvAY4GrgBuAGZk5jURcVFETM3MYeCdwCVAAg8CH69PPxbYKiJ+CxwMvHtR72eyKUmSVNoYW2czM2cAMzr27dL2/YXAhaOc9wCw65K8l8mmJEmSijHZlCRJKq0HYzbHKptNSZKk0sbYZfRe8jK6JEmSijHZlCRJKs1kU5IkSeo+k01JkqTSBniCkMmmJEmSijHZlCRJKs0xm5IkSVL3mWxKkiSV5phNSZIkqftMNiVJkkpzzKYkSZLUfSabkiRJpTlmU5IkSeo+k01JkqTSHLMpSZIkdZ/JpiRJUmkmm5IkSVL3mWxKkiSV1mo1XUFjTDYlSZJUjMmmJElSaY7ZHJsiYlpEzKy//0pETF2Cc6dHxPRStUmSJC224eHyjzFq3CSbmXlA0zVIkiRpyTTSbEbEEHAy8AZgPvAl4AhgSmYOR8Q04APAR9vOmQlMrzePAh4FXgLcDOydmXMj4n3AgcD9wIPANfW5rwY+DEwC7gDekZl/jYg7gV8BmwA7Al8AVq3f40OZeV7XP7wkSRo83q6y53YH/h3YEHgp8FbgMWBa/fx+wBkLOf9lwCFUzeZawE71Jfa3AZsCOwBrAkTEZKrGdqfM3BT4MW1NLHBxZgawHXBnZm4OvB3Y9l/9kJIkSYOuqcvorwDOyczHgceBTSJif+DNEfFL4JXAwcBWCzj/lsy8GyAiZgErAQFclJkP1/u/C0wEtqRqSK+ICOp9D7S91q/qrz8HToyINYALgY906bNKkqRBN4bHVJbWVLI5D/jnglMRMQU4l+pS9u5UTeM/FnJ++3MtYKjt64j59deJwNWZuUlmbgJsAezWdtxjAJl5O7AucBZVqnlNRIzpCVSSJEljXVPN1FXAbhExKSKWAS4B1gAuBk5k4ZfQF+S/gddFxPIR8Uyq8aBQJZdbR8SL6+1jgU90nhwRh1CN0/wuVaq6CvDsp1GHJEnSU7Va5R9jVCPNZmb+APgZcD1wLfCZzPwf4NvA3zPzVws7fwGveQNwSv16VwJ31fv/RDWW85yIuBnYjGoyUqdvAFEf81PgfZk5Z0nrkCRJ0pOGWmOkE46IicAJwOzM/FTT9SypR45709j4QfbIMw49sekSGjH85zuaLqHnhq/7SdMl9N4aU5quoOe+s//VTZfQc9+bOHh5wvvmLt10CY14+Z++O7Too8p67OvvL94nLP3WjzX+OUczltbZvI5qyaJdmy5EkiRJ3TFmms16WSJJkqT+42x0SZIkqfvGTLIpSZLUt7yDkCRJktR9JpuSJEmFtYYHatGapzDZlCRJUjEmm5IkSaU5G12SJEnqPpNNSZKk0pyNLkmSJHWfyaYkSVJpAzwb3WZTkiSpNCcISZIkSd1nsilJklSayaYkSZLUfSabkiRJpbUGd4KQyaYkSZKKMdmUJEkqzTGbkiRJUveZbEqSJJU2wIu6m2xKkiSpGJNNSZKk0lqO2ZQkSZK6zmRTkiSptAEes2mz2SVDKzyr6RJ6avjPdzRdQiMmPHftpkvoueE1pjRdQs8NrbR60yX03OT5TzRdQs8tt9Tg/RG48rKPNl2CBtDg/ZcmSZLUYy3X2ZQkSZK6z2RTkiSptAEes2myKUmSpGJMNiVJkkpznU1JkiSp+0w2JUmSSnPMpiRJktR9JpuSJEmlDfA6mzabkiRJpXkZXZIkSeo+k01JkqTSXPpIkiRJ6j6TTUmSpNIcsylJkiR1n8mmJElSYa0BXvrIZFOSJEnFmGxKkiSV5phNSZIkqftMNiVJkkoz2ZQkSZK6z2RTkiSpNO8gJEmSJHWfyaYkSVJpY2zMZkTsDRwDTAJOyczT2p7bBDij7fDJwIOZuUFE7A+cDPy5fu7CzDx6Ye9lsylJkjRAImIN4ARgc+Bx4OcRcUVm3gqQmTcAm9THLgNcAxxUnz4VODwzz17c97PZlCRJKqzVg2QzIlYAVhjlqTmZOadtewfg8sx8oD7vXGB34MOjnHskcGVmXl1vbwG8KCKOAm4E3pOZDy6sriUesxkRZ0TEURFx0dM4d0pE3Lmk5/VaRKwdEV9tug5JkqQlcBhwxyiPwzqOWx24r237PmDNzheLiOWBA4EPdRz7EWAj4I/A5xZV1NNNNu/NzF2e5rnjwfOBFzRdhCRJ6hO9GbN5Ck8dazliTsf2BKC9oCFgtOny+wI/zMzZIzsy8w0j30fEx4DfLaqoRTabETEEfBJ4LXAvMBGYGRF3ZuaUeoDp+4EnqLrnfYGtgGOBecDaVNf6D+h43Q2AU4HlgFWAkzLzixGxEvBVYF2qcQSHZ+blEfFqqnh3Uv0+78jMv9ZJ6dnAjsB8qm77COBFwBGZeU5EPBf4EvA8qh/mkZl5WURMB9aoj30+8JXMPAH4LLBORJyWme9e1M9IkiSpafWl8s7GcjR3A9u2ba9K1eN1ej1w4shGnXS+LTM/Xe8aouq9FmpxLqPvBmwKrA+8EXhhx/PHA6/KzM2pmsB16/0vA/6z3n4m0Nm0HQAcn5lbANsBH6/3fwT438x8CfBm4ISImEw182mnzNwU+DHw0bbX+lNmTgVmAf8FvIqq6T2yfv4zwNfqGncFvhQRz6qf26g+fkvgv+rxDocC19loSpKkrhgeLv9YfJcBr4yIyfUEoN2AS9oPqMPGzYFftO1+GHh/RGxZbx8C/GBRb7Y4zeY04PuZOS8z/wJ0jtU8H/hZHaV+r57BBHBVVlrAN4HtO847AnhmRBxJ1bAuV+9/RX08mXlzZm5N1QiuBVwRETfUH+5Fba91cf31LqpBrPPr71es9+8AfLg+92KqdHTkMvkVmTm3jogfAJZfjJ+JJEnS4htulX8spsy8BzgauAK4AZiRmddExEURMbU+bDIwNzP/0XbeE8AewBciYhZVM/r+Rb3f4ozZbFHFpCOeEpdm5n/Wk2leA3yrvjR9d8dxEzrPA84BHqRqVr8N7FXvn0fbOIKIWJfq0v3Vmblrve+ZPNmcAsxdUH21icD2bbOuVgNmU8XD/2g7rvOzSpIk9Z3MnAHM6Ni3S9v3s6kur3ee91NgsyV5r8VJNi8D9oiIf4uIFYFXjzwREUtFxO3A/Zl5EvANqkvuANtExBoRMQHYjyfTxxE7Asdl5o+AnevXmwhcRd141o3mJVRjPreOiBfX5x4LfGIJPuflwMH1a64H3AIss5Dj5+OyUJIkqVvGULLZa4tsNutmcCZVg3YecGvbc/OB44CfRMR1VBODRsZS3kvVfN4K3AN8peOlpwNXR8StVINU76SaTPRBqvWbbgTOAt6cmfcBbwPOiYibqTrqI5bgc74H2CoibgK+A+ybmQ8t5PhZwAoR8c0leA9JkiR1GGq1ut8JR8Q0YHpmTuv6i49Rj37qHWP3rxQFTNzpTU2X0IgJz1276RJ6bv4NlzZdQs8NrbR60yX03H/vvNg3A+kb33rmY02X0HPHLjWv6RIasd7vLmx8iNzf37lT8T7h2V/6ceOfczRLvKi7JEmStLiKjEvMzJlUs9glSZI0hsdUlmayKUmSpGKccS1JklSayaYkSZLUfSabkiRJhbVMNiVJkqTuM9mUJEkqzWRTkiRJ6j6TTUmSpNKGmy6gOSabkiRJKsZkU5IkqTBno0uSJEkFmGxKkiSVZrIpSZIkdZ/JpiRJUmkDPBvdZlOSJKkwJwhJkiRJBZhsSpIklTbAl9FNNiVJklSMyaYkSVJhjtmUJEmSCjDZlCRJKs0xm5IkSVL3mWxKkiQV1hrgZNNms0tacx5quoSeGr7uJ02X0IjhNaY0XULPLbXJq5ouoefmz5zRdAk9t+Vm9zZdQs9dNmu1pkvouR/MX7bpEhqxXtMFDDibTUmSpNIGONl0zKYkSZKKMdmUJEkqbJDHbJpsSpIkqRiTTUmSpNJMNiVJkqTuM9mUJEkqzDGbkiRJUgEmm5IkSYWZbEqSJEkFmGxKkiQVNsjJps2mJElSaa2hpitojJfRJUmSVIzJpiRJUmGDfBndZFOSJEnFmGxKkiQV1hp2zKYkSZLUdSabkiRJhTlmU5IkSSrAZFOSJKmwlutsSpIkSd1nsilJklSYYzYlSZKkAkw2JUmSCnOdTUmSJKkAk01JkqTCWq2mK2jOv5RsRsS0iJjZpVokSZLUZ0w2JUmSChvkMZvdaDYnR8RFwAuABN4I7A0cAbSAXwOHZObDEfEX4DpgNeB1wLeAZYFh4NDM/GVEbAF8GlgGuB94Z2beUSeoNwAvB54JHJaZl0bEc4GvAmsB84Gj6ve8ITPXAIiIe4DDM/M7EXFkfdwXgNOADYCJwEcz8+yIeAuwP7AycH5mHtWFn5EkSdJA6sYEobWAdwMvAVYFDgKOBl6RmRsCjwAfrI9dmaqp2wR4G3BBZk4FjgO2iYhnAF8B9s7MzYBPAqe3vdez6/17A2fWx58KXJ6ZGwG7A1+rP9cfI2KDiFiXqql+Rf0arwYuAI4Bfp2Zm1M1sEdHxDr1MWsCm9poSpKkbmgNDxV/jFXdaDZvzMw7MnMYmAWsQJUI/rV+/svAK9uO/1X99TLgvRExA3gO8DngxVQJ6XkRcQPwUWCdtnNPB8jMG4D7gI2A7amSTTLz9/XrbwlcVL/v9sBngG0jYnnguZk5C9gBOKh+n6uoEtb16/e5PjPn/6s/GEmSpEHXjWazvSlrAQ92PD9E2+X6zHys/vozYD3gx8CewPlUl7N/n5mb1Onn5sA2C3ivCfV252cYeb8LqRrK7YDvA09QJaI/ro+bCOzb9l5bAZfUzz22OB9ckiRpcbRa5R9jVal1NneNiJXq798BXNF5QER8jKrZOxM4BNgMuA1YKSK2rQ97GzCj7bQ31edOBVYEbgYuB95e718H+HfgF8D1VEnpizPztrqGY6guoVOf9676vNWAm6iGBEiSJKlLSjSbfwdOAq6MiNuoLqsfM8pxpwK715exfwDsl5mPU00w+mRE3EQ1UeftbeesExHXU12a3zMznwAOBbaPiJuBHwIHZOZ9mdkCrqa6tA9Vc/ls4Mp6+0PA0hFxS/3c+zPzd935EUiSJD1pkMdsDrXGcu7app6NPj0zZzZcyqgeOe5N4+MH2SUTXjCl6RKascaUpivouaU2eVXTJfTc/JkzFn1Qn3nkqz9puoSeO2HWak2X0HMrtyY2XUIjjr7rrMY7sd9tsFPxPuEFt/y48c85Gm9XKUmSpGLGzaLumTmt6RokSZKejtZw0xU0x2RTkiRJxYybZFOSJGm8Gm6NyeGUPWGyKUmSpGJMNiVJkgprmWxKkiRJ3WeyKUmSVNhYXnS9NJNNSZIkFWOyKUmSVNg4uWFjESabkiRJKsZkU5IkqTDHbEqSJEkFmGxKkiQV5h2EJEmSpAJMNiVJkgob5DsI2WxKkiQNmIjYGzgGmASckpmndTwfwJeAFYE/AW/KzAcjYi3gW8AqQAL7ZObDC3svL6NLkiQV1mqVfyyuiFgDOAHYBtgEODAi1mt7fgg4Dzg5MzcGfgP8V/3054HPZ+a6wHXAsYt6P5tNSZKkwbIDcHlmPpCZjwDnAru3Pb8Z8EhmXlJvnwicFhGTgJfXxwOcAbxxUW/mZXRJkqTCejEbPSJWAFYY5ak5mTmnbXt14L627fuAl7ZtvxD4U0R8FdgUmAW8B1gZ+Htmzm87b81F1WWyKUmSVFirNVT8ARwG3DHK47COciYA7Rfeh4Dhtu2lgGnAFzJzM+D3wKdGOY+O80ZlsylJktQfTgHWHuVxSsdxdwOrtW2vCtzbtv0n4PbMvK7ePpsq+ZwNLB8RE+v9q3WcNyovo0uSJBW2JBN4nq76UvmcRR4IlwHTI2Iy8AiwG3Bg2/M/ByZHxMaZeSPwOuDXmTkvIn4K7AnMAPYDLl7Um5lsSpIkDZDMvAc4GrgCuAGYkZnXRMRFETE1Mx8D3gCcHhG/BbYHjqhPP5hq9vqtwLZUyyctlMmmJElSYWPtdpWZOYMqnWzft0vb97/iqZOGRvbfRTWec7HZbHbJhLVWb7qE3lpjStMVNGJopQH7PQPzZ85Y9EF9ZqlpezddQs+d8N5bmi6h566ZN7vpEnruU63RJipLZdlsSpIkFTbIt6t0zKYkSZKKMdmUJEkqbKyN2ewlk01JkiQVY7IpSZJUWA+W2RyzTDYlSZJUjMmmJElSYY7ZlCRJkgow2ZQkSSrMdTYlSZKkAkw2JUmSChtuuoAGmWxKkiSpGJNNSZKkwlo4ZlOSJEnqOpNNSZKkwoYH+BZCNpuSJEmFDXsZXZIkSeo+k01JkqTCnCAkSZIkFWCyKUmSVJiLukuSJEkFmGxKkiQV5phNSZIkqQCTTUmSpMIcsylJkiQVYLIpSZJUmMlmn4mI1SPiokUcMz0ipo+yf+2I+Gqx4iRJkgZIXyabmXkvsMvTPP35wAu6WI4kSRpwgzwbfcw3mxFxM7BHZs6KiBnA3zLzXRGxNXAM8FNgD2Ai8GPgA1QN48zMnBIRawJnASsCNwOvyMw165d/aUT8HFgD+HpmTgc+C6wTEadl5rt790klSZL6z3i4jH4h8Mr6+w2BbervXw1cAGwObAFsStU07tNx/meA72TmRsC59TEjngtsV7/G+yLiWcChwHU2mpIkqVuGh8o/xqrx0GxeBLwyItYDfgs8ERGrADtTNZlbAr8GrgemAut3nL8j8E2AzPwBMKftuYsz8/HMvB+4H1ip5AeRJEkaNGP+Mjrwc+AMYAdgJvBnYHdgEvA34JTM/BRARKwAzAdWbjv/CRbcVM9v+74FAzygQpIkFTM8wC3GmE82M3M+cA3V5e2ZwOXA0VSJ5+XAmyNiuYhYCvghVSPa7jJgb4CI2BlYYRFvOZ/x0YRLkiSNeWO+2axdCCybmbcBV1KNtbwgM88Hvgf8CrgFuAE4s+Pc/wR2i4jfAHvy1Mvoo5kFrBAR3+xi/ZIkaYC1evAYq8ZFgpeZ3+TJcZdzaKs7M48Hju845U5gSv397sChmXlrRGxGNcmIeuZ5+3tMadvcoGvFS5IkDbBx0Wz+i24Hzo6IYeAfwDsarkeSJA2YQb6DUN83m5l5MXBx03VIkqTBNTzkBCFJkiSp6/o+2ZQkSWraWJ7AU5rJpiRJkoox2ZQkSSpskCcImWxKkiSpGJNNSZKkwoYHdzK6yaYkSZLKMdmUJEkqbJjBjTZNNiVJklSMyaYkSVJhrrMpSZIkFWCyKUmSVJiz0SVJkqQCTDYlSZIK8w5CkiRJUgEmm5IkSYU5G12SJEkqwGRTkiSpMGejS5IkSQWYbEqSJBXmbHRJkiSpAJNNSZKkwgY52bTZlCRJKqw1wBOEbDa75Njj72u6hJ7aYN7spktoxOT5TzRdQs9tudm9TZfQcye895amS+i5j113YtMl9NzWG+7fdAk996qHbmy6hEb8rekCBpzNpiRJUmGDfBndCUKSJEkqxmRTkiSpMJNNSZIkqQCTTUmSpMJaTRfQIJNNSZIkFWOyKUmSVNjwAK+zabIpSZKkYkw2JUmSCnM2uiRJklSAyaYkSVJhJpuSJElSASabkiRJhbnOpiRJklSAyaYkSVJhrrMpSZIkFWCyKUmSVJiz0SVJkqQCTDYlSZIKG+TZ6DabkiRJhQ2PsXYzIvYGjgEmAadk5mkLOO41wOcyc+16+xXA94E/1of8JjPfurD3stmUJEkaIBGxBnACsDnwOPDziLgiM2/tOO65wCeA9rn0U4FPZOZJi/t+jtmUJEkqbLgHjyWwA3B5Zj6QmY8A5wK7j3LcV4APdezbAnhVRNwUEedFxPMW9WYmm5IkSX0gIlYAVhjlqTmZOadte3Xgvrbt+4CXdrzWocD1wC87Xws4JzO/HxEHAd8G/n1hdZlsSpIkFdbqwQM4DLhjlMdhHeVM4KlzloZoC0cjYgNgN+AjnZ8jMw/KzO/X338RWD8ill/YZzfZlCRJ6g+nAGeMsn9Ox/bdwLZt26sC97ZtvxFYDbgOeAawekT8FHgFcCRwcmY+0Xb8/IUVZbMpSZJUWC8Wda8vlXc2lqO5DJgeEZOBR6hSzAPbXueDwAcBImIKMDMzt6233wDcDpwTEfsBv6rHfS6Ql9ElSZIGSGbeAxwNXAHcAMzIzGsi4qKImLqI0/cHDouI3wJvBQ5Y1PuNuWQzIlqZ+S/frj4iXgrslpkfiIhdgamZedy/XqEkSdKSGf6XO5vuyswZwIyOfbuMctydwJS27d8CL1uS9xpzzWYXrQc8FyAzzwPOa7YcSZKkwfO0ms2IWAr4ArABVUN3E7AXcFD9eAI4v04Vnw98HVgFeBQ4IDNvqq/zH0Z1Kf/XwLsz8x9t77EccFr9HhOBj2bm2RHxFqoId2XgfKqu/FRgufo9TqKahv9hYLmIOBq4B5iWmW+JiK2AzwDPBO4H3pmZ/xsRM4FrqAbMTgbek5kXP52fjyRJUruxdgehXnq6YzZfBszNzK2BF1Kt6XQocDDVOk0bAZtHxObA54HvZeYGwHTgmIhYH3gH8LLM3ASYDby34z2OAX6dmZsDLweOjoh16ufWBDbNzKOoxgocn5lbANsBH68HyB4HnJeZJ4y8YEQ8g6oRPSQzNwa+CJzd9p7PqD/T/wOOf5o/G0mSJNWeVrKZmVdFxF8j4t3AusCLqAaZnp+Zf6sP2wH+eQ/NverzLgIuiohD6nN+GRFQTau/vuNtdgCWiYi31dvLAuvX31+fmSPT7I8AXh0RRwIbUiWcC/Ji4MHMvLau57sR8eW29aEuqb/eAqy0eD8NSZKkhRvcXPPpX0bfleoy9WeoLpGvTDXV/tltx6xOddl8Xtu+IeAlVJfFz8nMQ+v9y41Sy0Rg38y8vj7mucADwD7AY23HnQM8SHVJ/dvUje0CjJbkDtXvBTByGb/FU+8DKkmSpKfh6V5G34GqWfw6VZO5HVWzuEtELFeP6Tyb6mbtVwFvajvvy8BM4A0RsUrdgH6B/7u6/eXAuwAiYjWqcaFrjVLLjsBxmfkjYOf6+IlUC4x2NrAJPCcitqiP2wO4KzMfeDo/BEmSpMUxxu6N3lNPt9k8HdgrIm4Gvgv8DFgR+BzwC+BG4KrMvAw4BNgtIm6gupn7gZmdPNGsAAAdgElEQVR5Y/395cBvqZLFkzve40PA0hFxS33c+zPzd6PUMh24OiJupZrccyewNtVkn60i4p+vm5mPA3sCn6tf95B6W5IkSQUMtVqDPIqge947Za+B+kFuMG/iog/qQ5PnP7Hog/rMlpvdu+iD+swJs1ZruoSe+9h1JzZdQs9tveH+TZfQc7c/NHj/PQP87eHfNT407gM96BM+eufZjX/O0XgHIUmSJBXTz4u6S5IkjQkDdfmzg8mmJEmSijHZlCRJKmwszxYvzWZTkiSpMG9XKUmSJBVgsilJklTY4OaaJpuSJEkqyGRTkiSpsEGeIGSyKUmSpGJMNiVJkgprDfCoTZNNSZIkFWOyKUmSVJhjNiVJkqQCTDYlSZIK8w5CkiRJUgEmm5IkSYUNbq5psilJkqSCTDYlSZIKc8ymJEmSVIDJpiRJUmGusylJkiQVYLIpSZJUmPdGlyRJkgow2eyS/3h8btMl9NxJkx5vuoSeW26pwftP5rJZqzVdQs9dM2920yX03NYb7t90CT33i5vPbLqERiy9+rZNlzCQBnnM5uD9yamuGMRGU5LGOxvN5ngZXZIkSSrAZFOSJKmwQb6MbrIpSZKkYkw2JUmSChtuOWZTkiRJ6jqTTUmSpMIGN9c02ZQkSVJBJpuSJEmFDQ9wtmmyKUmSpGJMNiVJkgrzDkKSJElSASabkiRJhXkHIUmSJKkAk01JkqTCnI0uSZIkFWCyKUmSVJiz0SVJkqQCTDYlSZIKcza6JEmSVIDJpiRJUmGtlmM2JUmSpK4z2ZQkSSpskNfZtNmUJEkqzAlCkiRJUgEmm5IkSYW5qLskSZJUgMmmJElSYYM8QchkU5IkScX0VbIZEd8ErsrM0+vtmcAHgOOB5wCPAu/JzN9ExAbAqcBywCrASZn5xYiYDmwFrAWcmplf6PkHkSRJfcVF3fvH14A3A0TE84HJwKeA92fmZsCBwLfrYw8Ajs/MLYDtgI+3vc4zM3M9G01JkqR/TV8lm8BMYPWImELVdJ4DHA18PSJGjlkuIp4DHAG8OiKOBDakSjhH/KpXBUuSpP43yOts9lWzmZmtiDgT2AvYE3gNcERmbjJyTESsCTwAnAs8CJxPlXbu1fZSj/WsaEmSpD7Wb5fRAc4ADgL+kJl3AbdHxL4AEbEjcFV93I7AcZn5I2Dn+vmJvS9XkiT1u1YP/hmr+q7ZzMw/An+gajoB9gEOiIibgJOAPTOzBUwHro6IW4FtgTuBtXtdryRJUj/rq8voETEErAasCvwIIDNvA6Z1HpuZn6KaPNRperkKJUnSIHKdzf6xG3AjcGRmPt50MZIkSYOur5LNzDyXauKPJEnSmOE6m5IkSVIBfZVsSpIkjUWO2ZQkSZIKMNmUJEkqbCyvg1mayaYkSZKKMdmUJEkqbHiAZ6PbbEqSJBU2uK2ml9ElSZJUkMmmJElSYS59JEmSJBVgsilJklTYWEs2I2Jv4BhgEnBKZp7W8fwbgA8BE4FrgQMzc25ErAV8C1gFSGCfzHx4Ye9lsilJkjRAImIN4ARgG2AT4MCIWK/t+WWBzwE7Zub6wDOBt9RPfx74fGauC1wHHLuo9zPZlCRJKqzVg6WPImIFYIVRnpqTmXPatncALs/MB+rzzgV2Bz4MkJmPRMSUzJwXEctQpZgPRsQk4OXA6+vXOQO4EvjAwuoy2ZQkSeoPhwF3jPI4rOO41YH72rbvA9ZsP6BuNHcG/gisDFxaf/17Zs5f0HmjMdmUJEkqrEdjNk+hShs7zenYnsBTl/4cAoY7T8rMi4HnRMSJwBeA9/F/lwz9P+d1stmUJEnqA/Wl8s7GcjR3A9u2ba8K3DuyERErAVMz89J611nAd4DZwPIRMTEznwBWaz9vQbyMLkmSVFirB/8sgcuAV0bE5HpM5m7AJW3PDwHfqmeeA7wRuDoz5wE/Bfas9+8HXLyoN7PZlCRJGiCZeQ9wNHAFcAMwIzOviYiLImJqZv4VOBC4ICJuBIInJwEdTDV7/VaqdPSYRb2fl9ElSZIK68Vs9CWRmTOAGR37dmn7/ofAD0c57y5g2pK8l8mmJEmSijHZlCRJKmys3UGol0w2JUmSVIzJpiRJUmFjbcxmL9lsdsnc4cEKid83d+mmS2jEyss+2nQJPfeD+cs2XULPfao12t3e+turHrqx6RJ6bunVt130QX3msXt/2nQJGkA2m5IkSYU5ZlOSJEkqwGRTkiSpsCW8w09fsdmUJEkqbHiAJwh5GV2SJEnFmGxKkiQVNsiX0U02JUmSVIzJpiRJUmGO2ZQkSZIKMNmUJEkqzDGbkiRJUgEmm5IkSYU5ZlOSJEkqwGRTkiSpMMdsSpIkSQWYbEqSJBXmmE1JkiSpAJNNSZKkwhyzKUmSJBVgsilJklRYqzXcdAmNMdmUJElSMSabkiRJhQ07ZlOSJEnqPpNNSZKkwloDvM6mzaYkSVJhXkaXJEmSCujbZjMido2IDy/imK9ExNRe1SRJkgZTq9Uq/hir+vYyemaeB5y3iGMO6FE5kiRJA2ncNpsRcRSwL/AEcCnweeBC4H7gMeAsYFpmviUipgGnAvOBXwDrZea0iJgJTK9f8ijgUeAlwM3A3pk5t1efR5Ik9a/hMZw8ljYuL6NHxM7ArsBUYFPghcCrgQD2zcwd246dBHwT2CczNwXmLeBlXwYcQtVsrgXsVOwDSJIkDYhx2WwCrwTOzsxHM3M+8LV63+zMvLPj2A3r/TfV219bwGvekpl3Z+YwMAtYqUDdkiRpALV68M9YNV6bzc66h6iGBDw2yrFPjHL8aP7R9n2rfk1JkiT9C8Zrs3k5sFdELB0RSwFvBa5YwLGzgBUjYsN6e28Yw+2/JEnqO4M8G31cNpuZeQFwAXAd8FvgD8D5Czh2LtVEom9ExK+B5zF6AipJkqQuG7ez0TPzeOD4jt1T2p4/AzgjIiZQTSbaJjMfiYjDgTXqY6a1nTut7dy3lKhZkiQNJu8g1MfqCT8PANdGxA3Ay4ETm61KkiRpMIzbZHNJZObJwMlN1yFJkgbTWB5TWVrfJ5uSJElqzkAkm5IkSU3yDkKSJElSASabkiRJhTlmU5IkSSrAZFOSJKkw19mUJEmSCjDZlCRJKswxm5IkSVIBJpuSJEmFDfI6mzabkiRJhbWcICRJkiR1n8mmJElSYYN8Gd1kU5IkScWYbEqSJBXm0keSJElSASabkiRJhTkbXZIkSSrAZFOSJKkwx2xKkiRJBZhsSpIkFWayKUmSJBVgsilJklTY4OaaMDTIsa4kSZLK8jK6JEmSirHZlCRJUjE2m5IkSSrGZlOSJEnF2GxKkiSpGJtNSZIkFWOzKUmSpGJsNiVJklSMzaYkSZKKsdmUJElSMTabGpci4tlN1yBJiysi3tl0DVJTvDf6OBQRVwDtv7gW8BgwCzgxMx9spLCCIuK1wLbAR4BrgcnAezPzjCbrKikiTs3M93TsOzMz92+qptIi4vjMPKbpOnppEH/PgygibsnMDZquo9ciYilgJ2AlYGhkf2Z+o7Gi1HNLNV2AnpZbgXnA1+rtvYE1gXuBrwL/0VBdJX0QOAB4E3AN8G7gSuCMBmsqIiK+AqwDTI2I9duemgQs30xVPfO6iDg2M/v+b8GD/HuOiJ2AE4AVqRqQIaCVmes0WlhZf4yIy4FfUYUDAGTmh5srqSdmAM+nCkNG/rtuATabA8Rmc3zaKjM3b9u+KSKuzcx9I2K/xqoqLDNvjIjpwLcy8+GImNR0TYUcD0wBPgN8qG3/fKr/YfezvwK3RcT1PPUP5Lc1V1Ixg/x7PhU4HLiFp16l6We/bPt+aIFH9Z+NMnPdpotQs2w2x6dJEbF+Zv4WoE5FJkbE0sAzmi2tmD9HxKnAVGDfiPgk8IeGayoiM+8E7gQ2rsemLs+TfzgtBzzQTGU9cWbTBfRKx+95WZ56mbHff8/3Z+YFTRfRS5nZ/hcKImIIWLuhcnppVkSslpn3NV2ImmOzOT4dClwcEX8GJgIrAG8GptO/lybeBWwHfCYzH4mI31N93r4VEUcCR1KlfSNaVJde+1JmnhkRU4D1gR8Dz8vMO5qtqqyIOA54P/CXtt19/XsGfhoRnwIuAf4xsjMzr2qupLIi4kDgE8CybbvvAF7YTEU9swyQEXELT/1db99cSeo1m81xKDNnRsQ6wIZUl9xuy8x5EfHzPh7rdmlmvmRkIzNPa7KYHjkAeEFm/mWRR/aJiNgTOAZYGngZ8IuIeG9mfqvZyop6K/D8zPzrIo/sHy+tv27atq8F9HMDciSwMdXwiaOBXYB/b7Si3jix6QLUPJvNcSgi1gI+C7ySaqLQRRHx//q8KbkxIt5MNTmofSxfX15Kr/2B/r6UOpoPUDWZV2Xm7IjYFLgM6Odm817gb00X0UuZuR1ARDwLmJiZcxouqRdmZ+YdEXEzsGFmfj4iDm66qFIiYrPMvJ7BGZOrhbDZHJ/OAr5Ddel8AvA2qrFuuzRZVGFb1o92/X6p8Xbg6nqpq/bLT/08e/WJzHwoIgDIzPsiYrjhmoqoL58DzKFKcC+mulIB9Pfvub4y823gBcBQRNwF7JGZtzdbWVGPRMR2wE3A6yPiWqoEv18dBBzIUye/jej3FFsdbDbHp2dn5ufatj8dEW9pqpheyMxBGEjf6Z76AYMze/W3EXEI1SS4TYCDgRsarqmUkd/pNaPs63dfAj6WmecCRMQewOnAtCaLKuw9VENjjgDeDiTVkm59KTMPrL9u13Qtap6Luo9DEfEF4Gcj49gi4jXAGzPzLY0WVlBErAh8jCoJ2Z1qoP3h/X75rZ6l/AKqJWKWzsxHGi6pqPrzHgPsQDX57XLgQ5n5UKOFqasi4jeZuWnHvpsyc6OmauqViFixH2+8sSD12qLt+v4mJPq/TDbHp/8ADoyILwHDVLP9qNfYbGXmxCaLK+R04FKqiQUPA/dRDSd4TZNFlRQR2wNfpmq6tgZuiYi9M/PSZisr6iDgjMw8sulCeiUi/gisTnU5HarVJeYAvwfekZn9mOw+3jamj4jYHHi04ZqKqpP6bwPLRMRWwFVUQweub7ay4mYxeDchUQebzfFpQ6o76azYvrNzHbc+s3Zmfjki3pWZc4GjI+LGposq7CRgG+DizPxTRLwcOJuq6e5Xk4AvRcQqVEsfXQBcmZnzF37auHYlcG5m/hAgInYG9qCaBHga/Tlj+TDgexHxANXQgZWo/p/Wzz4LvAGYkZn3RsS7gC/y5Mz8fjWQNyHRU01ougA9LRdSLaHRrt/HQ8yPiOWpP2dEvIgq1e1nEzLzTyMbmXlrk8X0QmaenJnTgK2oxrSdyVPXGe1HG4w0mgCZeTHVXVd+Q59OIMnMXwIvBvYD9gdeXO/rZ8tk5j/vDJWZPwH+rcF6emVS++1YB+QmJOpgsjlOZebbm66hxz4IzATWiogfUl1W7sdbGLa7OyJeC7QiYgWq+8H381JPRMQbgVcA2wJPAOdQjdvsZ3Mi4p1UyztNAPYBHoiIdemzQCAipmfm9Ij4Oh1/QY6Ifr0t6YgHImJjnvwL8z4MxtJmg3gTEnWw2RyffhgRB1D9Idy+VErfNiKZeUlEXEe1/NFE4J2Z+eeGyyrtnVT3zX4e1fi9/6ZaSqSffZrq/0unAN/PzP9puJ5e2Ifq9/wxqgb7J1SJ3+7AfzVYVwm/rr/OHOW5fr86cwzwOWD9iJhDtbTZvs2WVF7HTUieAGYNwE1I1MHZ6ONQRJwEHALc37a7lZl9t+Zk21qEo+rntQgHVVSLbG5PtQzOi4FbM3OfRotSV0XEkZl5Use+EzPzqKZqKq1eL3cy1RrJZ2TmHxsuqaiFpdhAv6fY6mCyOT69DlglMx9b5JHj38i6gy+lmsH4Xao09w3AnQ3V1BMRsTvVLe46J4L13V8qOkykmii0dP3oy1nKEXFBZr42Iu5g9D+M++73HBEnA6sAu9bjrkcsRTVOt2+bzczcrr77237AhfVC9mcCP8rMec1WV8TCUmwNGJvN8elOqgak75vNkRn2EfEzYOvMfLTePgW4osnaeuCTVGOb7mq6kF6JiLupxqVeBEzv82Vh3lF/ndZkET32PWA9qlvtXtm2fz7wkUYq6qHM/ENEfIPq8x5ENZ7xhIj4r8z8QbPVdVdmnl9/u3dm7tRoMWqczeb49Azg1oi4BZg7sjMz+/n2X5N5avoziWq5lH72v8DVmdnvs+7bbUI1KealVJPB7s7M2Q3XVERm3ld/vSsi9gbWB04Ads/Mvpw4kZnXAtdGxA8z85/3g4+IIaCv7xIWEW+nSjVXo0o0t8nMuyNideA3QF81m22Wjojn9fuwAS2czeb4dELTBTTgdOC6iLiIqhl5LdWkin72SeCKiLiSAblnNrAZ8HXgl1S/5y9FxNsz84JmyyqnvrS8JrA58FHgrRGxcWYe0WxlRe0ZEZ8Alm3bdyfV3bL61SuAD2bmzPad9ZqbBzdTUk+sAtwZEbNpuxrXj8NEtGA2m+NQZl656KP6S2Z+vL7t2TSqhHOPzOz3Rd2PAW6jmsE5KPfMPpEq8bkDoJ7F+n2qxd371U5UTfb1mfn3iNgRuInqHtr96kiqtYKPB44GdqE/F6//p8xc4ALmmfm9XtbSY7sAr6YaOrEU1Soq/fzfs0Zhs6lxISKWAlYFZlM1XhvX6U9fXm6sTRrAGZuTRhpNgMz8fUT01VqToxgZJjEyTOTf6P8bFszOzDsi4mZgw8z8fJ+ne4PsXcALqa5YDAFvpRoycViTRam3bDY1XswAnk91n92RP5Rb9PeiwD+JiEOAS3jq2Ny+XU8V+ENEHEZ1z2SAA+j/CVLnUC2Hs1L92fej+ve9nz0SEdtRJbivj4hr6dO7JYlXAZuOjD2PiAuBm5stSb1ms6nxYiPgJQO2CPBe9df2y6ktoJ/HOr0dOJXq0uoQ1SW3fl/Ifluqy4oPUS3gf1w/j1GtvYfqd/3e+uttVHeUUf9ZimpC5+Nt2080V46aYLOp8WIW1WX0+5oupFcys69n544mM2fXt/HbGJgH3DwAf8E4nmpM2wZU/09+NCJmZ+Y1zZZV1F6ZeXj9/W6NVqLSzqKa6Hh2vb0X/Z/cq4N3ENK4EBE/prof+i3AP0b29/NyTxGxItUtDF9AdevCTwCHZ+acRgsrqJ4ccyZwL0/eR3mPesmcvhYRk6l+z0dT3bThGQ2XVExE3AhsMgB/kRAQESMThCYAl2fmhQ2XpB4z2dR4cWLTBTTgdOBSqjUnH6ZKdc8CXtNkUYV9Gth5ZKWBiJgKfBGY2mhVBUXEacA2VJcWrwQO5qkLnvejvwK3RcT1PHU5nEGbEDcQMvMSqrHnGlD9PstT41xEbFZ/21rAo5+tnZlfBoYzc25mHk21HmM/e7x9SavMvI7+X/ZpRarPmFTDRW5rX/C8T51J9RfIS6ga65HH/2/v7kP9rOswjr+nznyo7GGUulJz2uWqLdcwScMs7VGLQjIbKszSEk2KjB6XEwM9jNLEx5xWNqcJii4fsGTiIs0Nm+7B7RrWVApRIivbwrlx+uN7nzyIDUbnPt/7d5/rBYP9ztjhOpyzs8/53N/v5xMRPZTOZnTdmZS1fue/wp8NA719jA5slbQXTVHd7JLu+0icZZIWUrq6W4GTKAOhjwKwvaxmuDbYngMgaTrlUeMdkva0PbVusvbY/rmkAyhbk+4B3jp65FVE9EvObEZ0VHPO6UJgP+C3lDOrp/X5vJOk7e27H+7jGV1JohSZx1IuRi0H7rS9qGqwFkn6HGVpwe7AEZQRSOf2+WOOmMhSbEanNcXH//wi7WPxMZqkKcDhlMsyD9l+pnKkGGOSVlFGH90NPGC792NhmrOaHwCW2Z4laR/gXtvvrBwtIlqQx+jRdfNrB6ilGXr9A9tHNt2vByWdbPuB2tnaIml/YCFwAGX+5GJKN/eJirFaZXtm7QwVbLP9fPmyBttPS+r7EZGICSvFZnTayB54SXdR1p3dbnvL9v9Wb/yQsk0G25b0CeAXwGFVU7XramABMAQ8A9xI2RJ1VM1QMebWNtuxJks6lHID/5HKmSKiJbmNHoNiiDL4eoOkyyX1ueAasZvtNSMvbK+nbOLosym2fw1ge9j2NcBrK2eKsXcWMJUy9ug64J+UgjMieiidzRgITYfzfkm7UwZf3yrpH5RHrlfafmG772AwrZc0ROlmDlM2b2yoG6l1/5b0Fl66gf9+XlpzFz1hexPw7eZXRPRcis0YGJKOBk4BPkK5THET8GFgCfDResla8wXKKsMbKasbl1HGQPXZ1yiXZaZJegR4A/DZupFirDTnMl/pwt8kyrSBncc5UkSMg9xGj4Eg6UlgM2UQ9C3AvrYfl7QzsML2e7b7DnpA0iTKoPc/1c7SJkmTgbdTbuCvs/1i5UgREfF/SLEZA0HSOcDcZkzK/pTNIxc3G3Z6SdKXKJdl9hz15idsT6sUqXWS3ktZ3XgZpcM5CzilWXcXPSFpV+BcQMBXgK8CF02gy38RE0ouCMWgOAM4EsD2k8Bsyn9SffYtypDvm4BplI/391UTte9SYC3lXO5myuf5gqqJog2XA6+mfH63AgdTLgpFRA+l2IxBMRkY3fXYQv93oz/brPBbDcywfQWl+OyznWzfAxwH3GL7KXK2vI9m2/4O8KLtzZQRX4dWzhQRLck38RgUtwFLJd1MKTJPAG6vG6l1m5rB7quAT0taQVnv12ebJX2dsvP+7Ob4xPOVM8XYG24epY/8wDiF/v/wGDFhpbMZA8H2NymPWEV5pHyp7Xl1U7XuHOBTlPOpbwRMOcvYZydTzqieYPs5yizGOXUjRQsuAe4F9pZ0CfAwcHHdSBHRllwQiojOkPSHiTBZIEDSO4APUqYO3Gd7deVIEdGSFJsRHSNpI9t5pGj7wHGMM66ataQXAst7Oqg/AEkzgO/aPknSdMqa0tNtu3K0iGhBzmxGdM/RtQNUdBhwP4Ckkbdl2Hf/XAPMB7C9TtIFwLWUsVcR0TPpbEZ0VDPc/CzKZZmtwF3AtbbzjzYGmqTVtme87G0rbc+qlSki2pPOZkR3LaTcPr+GcpnvVOBdlAHYvSRpD+A84BjK96elwLxml3b0x7OSvgwsal5/HnimYp6IaFGKzYjuOtz2ISMvJP0KWFMxz3i4jDLM/TTKvuzTgauAU2qGijE3F7iCsiFrC7AM+GLVRBHRmhSbEd21UdJBth9vXr8Z+EvNQONgtu3Rg+vPlvRYtTTRCttPSZpne6WkvSif9z/XzhUR7ciczYjumgw8KuluSXcA64CpkpZKWlo5W1t2kvS6kRfN77dWzBMtkHQRMNS83AP4vqT59RJFRJvS2YzorpfvBF9QJcX4+hGwvDkyMAn4JGUUUvTL8TSrV20/LelYYCXNDfWI6JcUmxHd9SBwiO1VkuYAs4Ah23+tnKtNNwCvAfYCnqNsjUpns392oVx++1fzevTqyojomRSbEd21iHJuczfgfOB64GeUrlBf3QzsQzkyMFJ8DFM+9uiPq4GHmw42wMfp/yrWiAkrxWZEd73N9omShoCFtockragdqmWHjL6BH711JeVM8quAv1MGuu9TNVFEtCYXhCK6axdJU4DPAHdK2pvy6LHP/ihpv9ohonWLKd3MM4D3Ad8ApldNFBGtSWczorsWAA8BS2yvkbQBmFc5Uysk3Ud5XP4mYLWkRxl1VtP2h2pli1bMBA4GfgxcB3wP+GXVRBHRmhSbER1lezGlAzRiuu1ttfK0bH7tADGunrU9LGk9MNP29ZJ2rR0qItqR3egRHSPpDtvHS9rIK9zQtX1ghVgRY0bST4AXKGc3b6B0NefYnlk1WES0Ip3NiO75naRTSbcv+utM4Ajbj0k6DzgGmFM5U0S0JJ3NiI6R9NPmtwcCBwF3AduAjwFrbR9XK1tERMSOSmczomNsz4X/Xpp598gQd0mvB26rmS0iImJHZfRRRHftC/xt1OtNZBZhREQMmHQ2I7rrTuA3km6l7Ak/kYyHiYiIAZMzmxEdJukE4GjKrfR7bS+pmygiImLHpNiMiIiIiNbkzGZEREREtCbFZkRERES0JsVmRERERLQmxWZEREREtCbFZkRERES05j8ONba9n6AADgAAAABJRU5ErkJggg==\n", "text/plain": [ "
" ] }, "metadata": {}, "output_type": "display_data" } ], "source": [ "corr_matrix = auto_df.corr().abs()\n", "f, ax = plt.subplots(figsize=(12, 9))\n", "sns.heatmap(corr_matrix, vmax=1, square=True)\n", "plt.xticks(rotation=90)\n", "plt.yticks(rotation=0);" ] }, { "cell_type": "markdown", "metadata": {}, "source": [ "#### (c) Perform a multiple linear regression with mpg as the response and all other variables except name as the predictors. Use the summary() function to print the results. Comment on the output. For instance:\n", "\n", "1. Is there a relationship between the predictors and the response?\n", "2. Which predictors appear to have a statistically significant relationship to the response?\n", "3. What does the coefficient for the year variable suggest?" ] }, { "cell_type": "code", "execution_count": 15, "metadata": {}, "outputs": [ { "name": "stdout", "output_type": "stream", "text": [ " OLS Regression Results \n", "==============================================================================\n", "Dep. Variable: mpg R-squared: 0.824\n", "Model: OLS Adj. R-squared: 0.821\n", "Method: Least Squares F-statistic: 224.5\n", "Date: Tue, 11 Sep 2018 Prob (F-statistic): 1.79e-139\n", "Time: 09:26:33 Log-Likelihood: -1020.5\n", "No. Observations: 392 AIC: 2059.\n", "Df Residuals: 383 BIC: 2095.\n", "Df Model: 8 \n", "Covariance Type: nonrobust \n", "==================================================================================\n", " coef std err t P>|t| [0.025 0.975]\n", "----------------------------------------------------------------------------------\n", "Intercept -17.9546 4.677 -3.839 0.000 -27.150 -8.759\n", "C(origin)[T.2] 2.6300 0.566 4.643 0.000 1.516 3.744\n", "C(origin)[T.3] 2.8532 0.553 5.162 0.000 1.766 3.940\n", "cylinders -0.4897 0.321 -1.524 0.128 -1.121 0.142\n", "displacement 0.0240 0.008 3.133 0.002 0.009 0.039\n", "horsepower -0.0182 0.014 -1.326 0.185 -0.045 0.009\n", "weight -0.0067 0.001 -10.243 0.000 -0.008 -0.005\n", "acceleration 0.0791 0.098 0.805 0.421 -0.114 0.272\n", "year 0.7770 0.052 15.005 0.000 0.675 0.879\n", "==============================================================================\n", "Omnibus: 23.395 Durbin-Watson: 1.291\n", "Prob(Omnibus): 0.000 Jarque-Bera (JB): 34.452\n", "Skew: 0.444 Prob(JB): 3.30e-08\n", "Kurtosis: 4.150 Cond. No. 8.70e+04\n", "==============================================================================\n", "\n", "Warnings:\n", "[1] Standard Errors assume that the covariance matrix of the errors is correctly specified.\n", "[2] The condition number is large, 8.7e+04. This might indicate that there are\n", "strong multicollinearity or other numerical problems.\n" ] } ], "source": [ "f = 'mpg ~ cylinders + displacement + horsepower + weight + acceleration + year + C(origin)'\n", "y, X = patsy.dmatrices(f, auto_df, return_type='dataframe')\n", "\n", "model = smf.OLS(y, X).fit()\n", "print(model.summary())" ] }, { "cell_type": "code", "execution_count": 16, "metadata": {}, "outputs": [ { "data": { "text/plain": [ "year 2.332943e-40\n", "weight 6.375633e-22\n", "C(origin)[T.3] 3.933208e-07\n", "C(origin)[T.2] 4.720373e-06\n", "Intercept 1.445124e-04\n", "displacement 1.862685e-03\n", "dtype: float64" ] }, "execution_count": 16, "metadata": {}, "output_type": "execute_result" } ], "source": [ "# The following predictors have P-values < 0.05 which suggests we can reject \n", "# the null hypothesis that they have no relationship with the response:\n", "model.pvalues[model.pvalues < 0.05].sort_values()" ] }, { "cell_type": "markdown", "metadata": {}, "source": [ "The coefficient for the `year` variable suggests that there is a positive relationship between `year` and `mpg` – the response variable increases as year increases. Specifically, this suggessts that cars manufactured more recently tend to be more fuel efficient and that all else being equal efficiency increases by 0.75 mpg/year. \n" ] }, { "cell_type": "markdown", "metadata": {}, "source": [ "#### (d) Use the plot() function to produce diagnostic plots of the linear regression fit. Comment on any problems you see with the fit. Do the residual plots suggest any unusually large outliers? Does the leverage plot identify any observations with unusually high leverage?" ] }, { "cell_type": "code", "execution_count": 17, "metadata": {}, "outputs": [ { "data": { "image/png": "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\n", "text/plain": [ "
" ] }, "metadata": {}, "output_type": "display_data" } ], "source": [ "# Make predictions\n", "y_pred = np.array(model.predict(X))\n", "\n", "# Analyse residuals\n", "lm_plot(lm_stats(X, y, y_pred))" ] }, { "cell_type": "markdown", "metadata": {}, "source": [ "- **Linearity:** The u-shaped pattern in the top-left residual plot suggests that there a non-linear relationship that our model is unable to account for. The bottom left Scale-Location plot suggests that a square-root transform reduces the non-linearity of this relationship allowing our model to achieve an improved fit.\n", "- **Heteroscedasticity:** A conical pattern is apparent in the top-left plot suggesting that variance of residuals increases with response. The bottom left plot suggests that a square-root transform helps to mitigate this effect.\n", "- **Outliers:** The top-left plot suggests that there are five potential outliers towards the high end of our predicted responses. The bottom-right plot suggests that these outliers have low leverage, and so probably don't have a overly dispraportionate effect on our model.\n", "- **Leverage:** The bottom right plot reveals one point with particularly high leverage, although with a value around -1.2 this point isn't an outlier in terms of studentised residuals.\n", "- **Normality:** The top-right plot shows a normal distribution except for high value residuals. This suggests that the model under-estimates `mpg` for a larger number of observations than we might expect assuming a normal distribution.\n", "\n" ] }, { "cell_type": "markdown", "metadata": {}, "source": [ "#### (e) Use the * and : symbols to fit linear regression models with interaction effects. Do any interactions appear to be statistically significant?" ] }, { "cell_type": "code", "execution_count": 18, "metadata": {}, "outputs": [ { "name": "stdout", "output_type": "stream", "text": [ " OLS Regression Results \n", "==============================================================================\n", "Dep. Variable: np.log1p(mpg) R-squared: 0.905\n", "Model: OLS Adj. R-squared: 0.899\n", "Method: Least Squares F-statistic: 152.3\n", "Date: Tue, 11 Sep 2018 Prob (F-statistic): 2.31e-172\n", "Time: 09:26:34 Log-Likelihood: 346.65\n", "No. Observations: 392 AIC: -645.3\n", "Df Residuals: 368 BIC: -550.0\n", "Df Model: 23 \n", "Covariance Type: nonrobust \n", "=============================================================================================\n", " coef std err t P>|t| [0.025 0.975]\n", "---------------------------------------------------------------------------------------------\n", "Intercept 3.7093 1.873 1.981 0.048 0.027 7.392\n", "C(origin)[T.2] 0.0367 0.020 1.799 0.073 -0.003 0.077\n", "C(origin)[T.3] 0.0333 0.019 1.720 0.086 -0.005 0.071\n", "cylinders 0.3466 0.310 1.119 0.264 -0.262 0.955\n", "displacement -0.0124 0.007 -1.826 0.069 -0.026 0.001\n", "horsepower 0.0088 0.012 0.721 0.472 -0.015 0.033\n", "weight -0.0001 0.001 -0.225 0.822 -0.001 0.001\n", "acceleration -0.2061 0.080 -2.591 0.010 -0.363 -0.050\n", "year 0.0111 0.022 0.507 0.613 -0.032 0.054\n", "cylinders:displacement -0.0005 0.000 -2.604 0.010 -0.001 -0.000\n", "cylinders:horsepower 0.0006 0.001 0.633 0.527 -0.001 0.002\n", "cylinders:weight 1.701e-05 3.11e-05 0.546 0.585 -4.42e-05 7.82e-05\n", "cylinders:acceleration 0.0098 0.006 1.555 0.121 -0.003 0.022\n", "cylinders:year -0.0068 0.004 -1.897 0.059 -0.014 0.000\n", "displacement:horsepower 2.538e-06 1.08e-05 0.235 0.814 -1.87e-05 2.38e-05\n", "displacement:weight 9.392e-07 5.18e-07 1.813 0.071 -7.97e-08 1.96e-06\n", "displacement:acceleration -0.0002 0.000 -1.876 0.062 -0.000 1.09e-05\n", "displacement:year 0.0002 8.32e-05 2.377 0.018 3.42e-05 0.000\n", "horsepower:weight -1.528e-06 1.07e-06 -1.431 0.153 -3.63e-06 5.71e-07\n", "horsepower:acceleration -0.0002 0.000 -1.158 0.248 -0.000 0.000\n", "horsepower:year -0.0001 0.000 -0.723 0.470 -0.000 0.000\n", "weight:acceleration 5.38e-06 8.74e-06 0.615 0.539 -1.18e-05 2.26e-05\n", "weight:year -3.245e-06 8.01e-06 -0.405 0.686 -1.9e-05 1.25e-05\n", "acceleration:year 0.0024 0.001 2.553 0.011 0.001 0.004\n", "==============================================================================\n", "Omnibus: 12.734 Durbin-Watson: 1.605\n", "Prob(Omnibus): 0.002 Jarque-Bera (JB): 25.323\n", "Skew: -0.092 Prob(JB): 3.17e-06\n", "Kurtosis: 4.232 Cond. No. 3.46e+08\n", "==============================================================================\n", "\n", "Warnings:\n", "[1] Standard Errors assume that the covariance matrix of the errors is correctly specified.\n", "[2] The condition number is large, 3.46e+08. This might indicate that there are\n", "strong multicollinearity or other numerical problems.\n" ] } ], "source": [ "# Let's fit a model with all possible interaction effects\n", "\n", "f = 'np.log1p(mpg) ~ cylinders + displacement + horsepower + weight + acceleration + year + C(origin) \\\n", " + cylinders*displacement \\\n", " + cylinders*horsepower \\\n", " + cylinders*weight \\\n", " + cylinders*acceleration \\\n", " + cylinders*year \\\n", " + displacement*horsepower \\\n", " + displacement*weight \\\n", " + displacement*acceleration \\\n", " + displacement*year \\\n", " + horsepower*weight \\\n", " + horsepower*acceleration \\\n", " + horsepower*year \\\n", " + weight*weight \\\n", " + weight*acceleration \\\n", " + weight*year \\\n", " + acceleration*year \\\n", " '\n", "\n", "y, X = patsy.dmatrices(f, auto_df, return_type='dataframe')\n", "model = smf.OLS(y, X).fit()\n", "print(model.summary())" ] }, { "cell_type": "code", "execution_count": 19, "metadata": {}, "outputs": [ { "data": { "text/plain": [ "cylinders:displacement 0.009575\n", "acceleration 0.009944\n", "acceleration:year 0.011091\n", "displacement:year 0.017965\n", "Intercept 0.048352\n", "dtype: float64" ] }, "execution_count": 19, "metadata": {}, "output_type": "execute_result" } ], "source": [ "# If we take p < 0.05 as our significance threshold\n", "# which features appear as significant in this model?\n", "model.pvalues[model.pvalues < 0.05].sort_values()" ] }, { "cell_type": "markdown", "metadata": {}, "source": [ "Only the above features seem to be statistically significant in this model.\n", "\n", "Notice the warning (2) that suggests the might be multicollinearity in this model. We can observe in the correlation plot above that there is strong collinearity between mpg, cylinders, displacement, horespower, and weight. This means further investigation is required to discern which, if any, of these features have a *causal* relationship with our response `mpg`. " ] }, { "cell_type": "markdown", "metadata": {}, "source": [ "#### (f) Try a few different transformations of the variables, such as log(X), √X, X2. Comment on your findings.\n", "\n", "Let's start by taking log of the response variable only." ] }, { "cell_type": "code", "execution_count": 20, "metadata": {}, "outputs": [ { "data": { "image/png": "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\n", "text/plain": [ "
" ] }, "metadata": {}, "output_type": "display_data" } ], "source": [ "# Note: taking log of response variable to normalise distribution\n", "f = 'np.log(mpg) ~ cylinders + displacement + horsepower + weight + acceleration + year + C(origin)'\n", "y, X = patsy.dmatrices(f, auto_df, return_type='dataframe')\n", "\n", "# Fit model\n", "model = smf.OLS(y, X).fit()\n", "\n", "# Make predictions\n", "y_pred = np.array(model.predict(X))\n", "\n", "# Analyse residuals\n", "lm_plot(lm_stats(X, y, y_pred))" ] }, { "cell_type": "markdown", "metadata": {}, "source": [ "**Comment:** Taking the log of the response variable dramatically improves aspects of our diagnostic plots.\n", "\n", "- there is less pattern in the left-hand residual plots suggesting the log transform of `mpg` as decreased the non-linearity of its relationship to features and so improved our models ability to fit the date.\n", "- there is a reduction in the conic shape of the bottom left plot suggesting a reduction in homoscedasticity\n", "- the top-right plot suggests the model is now inclined to overstimate instead of underestimate\n", "- there are no-longer any outliers exceeding a studentised residual value of three. there are now two outliers <-3\n", "\n", "\n", "**Question:** What if we take logs of all our predictor variables also?" ] }, { "cell_type": "code", "execution_count": 21, "metadata": {}, "outputs": [ { "data": { "image/png": "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\n", "text/plain": [ "
" ] }, "metadata": {}, "output_type": "display_data" } ], "source": [ "f = 'np.log1p(mpg) \\\n", " ~ np.log1p(cylinders) \\\n", " + np.log1p(displacement) \\\n", " + np.log1p(horsepower) \\\n", " + np.log1p(weight) \\\n", " + np.log1p(acceleration) \\\n", " + np.log1p(year) \\\n", " + C(origin) \\\n", " + np.log1p(cylinders*displacement) \\\n", " + np.log1p(cylinders*horsepower) \\\n", " + np.log1p(cylinders*weight) \\\n", " + np.log1p(cylinders*acceleration) \\\n", " + np.log1p(cylinders*year) \\\n", " + np.log1p(displacement*horsepower) \\\n", " + np.log1p(displacement*weight) \\\n", " + np.log1p(displacement*acceleration) \\\n", " + np.log1p(displacement*year) \\\n", " + np.log1p(horsepower*weight) \\\n", " + np.log1p(horsepower*acceleration) \\\n", " + np.log1p(horsepower*year) \\\n", " + np.log1p(weight*weight) \\\n", " + np.log1p(weight*acceleration) \\\n", " + np.log1p(weight*year) \\\n", " + np.log1p(acceleration*year) \\\n", " '\n", "\n", "y, X = patsy.dmatrices(f, auto_df, return_type='dataframe')\n", "model = smf.OLS(y, X).fit()\n", "#print(model.summary())\n", "\n", "# Make predictions\n", "y_pred = np.array(model.predict(X))\n", "\n", "# Analyse residuals\n", "lm_plot(lm_stats(X, y, y_pred))\n", "\n" ] }, { "cell_type": "code", "execution_count": 22, "metadata": {}, "outputs": [ { "data": { "text/plain": [ "np.log1p(cylinders * displacement) 5.450938e-07\n", "np.log1p(year) 3.554276e-06\n", "Intercept 4.103286e-06\n", "np.log1p(horsepower * acceleration) 4.088496e-04\n", "np.log1p(cylinders * acceleration) 5.328883e-03\n", "np.log1p(horsepower * weight) 1.089279e-02\n", "np.log1p(horsepower * year) 2.345207e-02\n", "np.log1p(displacement) 3.208041e-02\n", "np.log1p(cylinders * horsepower) 4.141974e-02\n", "dtype: float64" ] }, "execution_count": 22, "metadata": {}, "output_type": "execute_result" } ], "source": [ "# If we take p < 0.05 as our significance threshold\n", "# which features appear as significant in this model?\n", "model.pvalues[model.pvalues < 0.05].sort_values()" ] }, { "cell_type": "markdown", "metadata": {}, "source": [ "**Comment:** Comparing the above diagnostics grid with the preceding grid suggests that taking log of predictors has further reduced the non-linearities in our dataset. \n", "\n", "The number of observations that appear as outliers has reduced from two to one. Heteroscedasticity has also been reduced. The distribtuion of studentised residuals is more normal.\n", "\n", "There is notably increased variance in the leverage of our observations.\n", "\n", "**Question:** what if we only use the features from this model with p-values < 0.05?\n" ] }, { "cell_type": "code", "execution_count": 23, "metadata": { "scrolled": true }, "outputs": [ { "name": "stdout", "output_type": "stream", "text": [ " OLS Regression Results \n", "==============================================================================\n", "Dep. Variable: np.log(mpg) R-squared: 0.892\n", "Model: OLS Adj. R-squared: 0.889\n", "Method: Least Squares F-statistic: 394.3\n", "Date: Tue, 11 Sep 2018 Prob (F-statistic): 1.11e-179\n", "Time: 09:26:35 Log-Likelihood: 302.85\n", "No. Observations: 392 AIC: -587.7\n", "Df Residuals: 383 BIC: -552.0\n", "Df Model: 8 \n", "Covariance Type: nonrobust \n", "=======================================================================================================\n", " coef std err t P>|t| [0.025 0.975]\n", "-------------------------------------------------------------------------------------------------------\n", "Intercept -3.1388 1.506 -2.084 0.038 -6.100 -0.177\n", "np.log1p(displacement) -3.7868 11.515 -0.329 0.742 -26.428 18.855\n", "np.log1p(year) 64.6712 19.481 3.320 0.001 26.369 102.973\n", "np.log1p(cylinders * displacement) 3.6912 11.443 0.323 0.747 -18.807 26.189\n", "np.log1p(cylinders * horsepower) 29.1632 11.349 2.570 0.011 6.849 51.478\n", "np.log1p(cylinders * acceleration) -33.2810 11.141 -2.987 0.003 -55.187 -11.375\n", "np.log1p(horsepower * weight) -0.4950 0.084 -5.925 0.000 -0.659 -0.331\n", "np.log1p(horsepower * acceleration) 32.6861 11.002 2.971 0.003 11.055 54.317\n", "np.log1p(horsepower * year) -61.5613 19.215 -3.204 0.001 -99.342 -23.780\n", "==============================================================================\n", "Omnibus: 11.748 Durbin-Watson: 1.485\n", "Prob(Omnibus): 0.003 Jarque-Bera (JB): 18.234\n", "Skew: 0.205 Prob(JB): 0.000110\n", "Kurtosis: 3.974 Cond. No. 1.18e+05\n", "==============================================================================\n", "\n", "Warnings:\n", "[1] Standard Errors assume that the covariance matrix of the errors is correctly specified.\n", "[2] The condition number is large, 1.18e+05. This might indicate that there are\n", "strong multicollinearity or other numerical problems.\n" ] }, { "data": { "image/png": "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\n", "text/plain": [ "
" ] }, "metadata": {}, "output_type": "display_data" } ], "source": [ "# Let's use just the most significant features from the set of all log features\n", "significant_features = model.pvalues[model.pvalues < 0.05].drop('Intercept').index\n", "f = 'np.log(mpg) ~ ' + ' + '.join(list(significant_features))\n", "\n", "y, X = patsy.dmatrices(f, auto_df, return_type='dataframe')\n", "\n", "# Fit model\n", "model = smf.OLS(y, X).fit()\n", "\n", "# Make predictions\n", "y_pred = np.array(model.predict(X))\n", "\n", "# If we take p < 0.05 as our significance threshold\n", "# which features appear as significant in this model?\n", "print(model.summary())\n", "\n", "# Analyse residuals\n", "lm_plot(lm_stats(X, y, y_pred))" ] }, { "cell_type": "markdown", "metadata": {}, "source": [ "The top-left plot suggests that our model is now fitting the data with even less loss. There is no notable heteroscedasticity, and the distribution of the residuals appears normal.\n", "\n", "The variance in leverage of observations is reduced and there are three potential outliers.\n", "\n", "In the top-left residual plot there is a repeating linear pattern for `Fitted values < 2.8`. This pattern is not visible in the plot below.\n", "\n", "**Question:** Would the taking square root of features provide improved results?\n", "\n", "I tried the above, taking square root of log(features)\n" ] }, { "cell_type": "markdown", "metadata": {}, "source": [ "### 10. This question should be answered using the Carseats data set.\n", "\n", "#### (a) Fit a multiple regression model to predict Sales using Price, Urban, and US." ] }, { "cell_type": "code", "execution_count": 52, "metadata": {}, "outputs": [ { "data": { "text/html": [ "
\n", "\n", "\n", " \n", " \n", " \n", " \n", " \n", " \n", " \n", " \n", " \n", " \n", " \n", " \n", " \n", " \n", " \n", " \n", " \n", " \n", " \n", " \n", " \n", " \n", " \n", " \n", " \n", " \n", " \n", " \n", " \n", " \n", " \n", " \n", " \n", " \n", " \n", " \n", " \n", " \n", " \n", " \n", " \n", " \n", " \n", " \n", " \n", " \n", " \n", " \n", " \n", " \n", " \n", " \n", " \n", " \n", " \n", " \n", " \n", " \n", " \n", " \n", " \n", " \n", " \n", " \n", " \n", " \n", " \n", " \n", " \n", " \n", " \n", " \n", " \n", " \n", " \n", " \n", " \n", " \n", " \n", " \n", " \n", " \n", " \n", " \n", " \n", " \n", " \n", " \n", "
SalesCompPriceIncomeAdvertisingPopulationPriceAgeEducationShelveLocUrbanUS
09.50138.073.011.0276.0120.042.017.0BadYesYes
111.22111.048.016.0260.083.065.010.0GoodYesYes
210.06113.035.010.0269.080.059.012.0MediumYesYes
37.40117.0100.04.0466.097.055.014.0MediumYesYes
44.15141.064.03.0340.0128.038.013.0BadYesNo
\n", "
" ], "text/plain": [ " Sales CompPrice Income Advertising Population Price Age Education \\\n", "0 9.50 138.0 73.0 11.0 276.0 120.0 42.0 17.0 \n", "1 11.22 111.0 48.0 16.0 260.0 83.0 65.0 10.0 \n", "2 10.06 113.0 35.0 10.0 269.0 80.0 59.0 12.0 \n", "3 7.40 117.0 100.0 4.0 466.0 97.0 55.0 14.0 \n", "4 4.15 141.0 64.0 3.0 340.0 128.0 38.0 13.0 \n", "\n", " ShelveLoc Urban US \n", "0 Bad Yes Yes \n", "1 Good Yes Yes \n", "2 Medium Yes Yes \n", "3 Medium Yes Yes \n", "4 Bad Yes No " ] }, "execution_count": 52, "metadata": {}, "output_type": "execute_result" } ], "source": [ "# Load data\n", "carseats = pd.read_csv('./data/Carseats.csv').drop('Unnamed: 0', axis=1)\n", "\n", "# No missing values found\n", "\n", "# Pre-processing\n", "# Convert quantitive datatypes to numerics\n", "datatypes = {'quant': ['Sales', 'CompPrice', 'Income', 'Advertising', 'Population', 'Price', 'Age', 'Education'],\n", " 'qual': ['ShelveLoc', 'Urban', 'US']}\n", "# Use floats for all quantitive values\n", "quants = carseats[datatypes['quant']].astype(np.float_)\n", "carseats_df = pd.concat([quants, carseats[datatypes['qual']]], axis=1)\n", "\n", "carseats_df.head()" ] }, { "cell_type": "code", "execution_count": 63, "metadata": {}, "outputs": [ { "name": "stdout", "output_type": "stream", "text": [ " OLS Regression Results \n", "==============================================================================\n", "Dep. Variable: Sales R-squared: 0.239\n", "Model: OLS Adj. R-squared: 0.234\n", "Method: Least Squares F-statistic: 41.52\n", "Date: Tue, 11 Sep 2018 Prob (F-statistic): 2.39e-23\n", "Time: 11:03:28 Log-Likelihood: -927.66\n", "No. Observations: 400 AIC: 1863.\n", "Df Residuals: 396 BIC: 1879.\n", "Df Model: 3 \n", "Covariance Type: nonrobust \n", "===================================================================================\n", " coef std err t P>|t| [0.025 0.975]\n", "-----------------------------------------------------------------------------------\n", "Intercept 13.0435 0.651 20.036 0.000 11.764 14.323\n", "C(Urban)[T.Yes] -0.0219 0.272 -0.081 0.936 -0.556 0.512\n", "C(US)[T.Yes] 1.2006 0.259 4.635 0.000 0.691 1.710\n", "Price -0.0545 0.005 -10.389 0.000 -0.065 -0.044\n", "==============================================================================\n", "Omnibus: 0.676 Durbin-Watson: 1.912\n", "Prob(Omnibus): 0.713 Jarque-Bera (JB): 0.758\n", "Skew: 0.093 Prob(JB): 0.684\n", "Kurtosis: 2.897 Cond. No. 628.\n", "==============================================================================\n", "\n", "Warnings:\n", "[1] Standard Errors assume that the covariance matrix of the errors is correctly specified.\n" ] } ], "source": [ "# Feature engineering\n", "f = 'Sales ~ Price + C(Urban) + C(US)'\n", "y, X = patsy.dmatrices(f, carseats_df, return_type='dataframe')\n", "\n", "# Fit model\n", "model = smf.OLS(y, X).fit()\n", "print(model.summary())\n", "\n", "# Make predictions\n", "y_pred = np.array(model.predict(X))\n" ] }, { "cell_type": "markdown", "metadata": {}, "source": [ "### (b) Provide an interpretation of each coefficient in the model. Be careful—some of the variables in the model are qualitative!\n", "\n", "Our model suggests that:\n", "\n", "- Sales decrease with increase in Price. For each unit increase in Price, Sales decrease by `0.0545 * 1000 = 54.5 dollars` on average.\n", "- There seems to be a statistically significant relationship between Sales and whether the store is in the U.S. or not. We expect carseats sold in the US to achieve a salprice 1,200 dollars higher on average.\n", "- There does not seem to be significant relationship between Sales and wheter the store is in an urban or rural relationship\n", "\n", "### (c) Write out the model in equation form, being careful to handle the qualitative variables properly.\n", "\n", "$\\hat{y} = 13.0435 + (-0.0219 \\times Urban) + (1.2006 \\times US) + (-0.0545 \\times Price) $\n", "\n", "Where Urban and US are encoded as dummy variables:\n", "\n", "- Urban: Yes => 1\n", "- Urban:No => 0\n", "- US: Yes => 1\n", "- US: No => 0\n", "\n", "### (d) For which of the predictors can you reject the null hypothesis H0 :βj =0?\n", "\n", "US, Price\n", "\n", "### (e) On the basis of your response to the previous question, fit a smaller model that only uses the predictors for which there is evidence of association with the outcome." ] }, { "cell_type": "code", "execution_count": 65, "metadata": {}, "outputs": [ { "name": "stdout", "output_type": "stream", "text": [ " OLS Regression Results \n", "==============================================================================\n", "Dep. Variable: Sales R-squared: 0.239\n", "Model: OLS Adj. R-squared: 0.235\n", "Method: Least Squares F-statistic: 62.43\n", "Date: Tue, 11 Sep 2018 Prob (F-statistic): 2.66e-24\n", "Time: 11:03:45 Log-Likelihood: -927.66\n", "No. Observations: 400 AIC: 1861.\n", "Df Residuals: 397 BIC: 1873.\n", "Df Model: 2 \n", "Covariance Type: nonrobust \n", "================================================================================\n", " coef std err t P>|t| [0.025 0.975]\n", "--------------------------------------------------------------------------------\n", "Intercept 13.0308 0.631 20.652 0.000 11.790 14.271\n", "C(US)[T.Yes] 1.1996 0.258 4.641 0.000 0.692 1.708\n", "Price -0.0545 0.005 -10.416 0.000 -0.065 -0.044\n", "==============================================================================\n", "Omnibus: 0.666 Durbin-Watson: 1.912\n", "Prob(Omnibus): 0.717 Jarque-Bera (JB): 0.749\n", "Skew: 0.092 Prob(JB): 0.688\n", "Kurtosis: 2.895 Cond. No. 607.\n", "==============================================================================\n", "\n", "Warnings:\n", "[1] Standard Errors assume that the covariance matrix of the errors is correctly specified.\n" ] } ], "source": [ "# Feature engineering\n", "f = 'Sales ~ Price + C(US)'\n", "y, X = patsy.dmatrices(f, carseats_df, return_type='dataframe')\n", "\n", "# Fit model\n", "model = smf.OLS(y, X).fit()\n", "print(model.summary())\n", "\n", "# Make predictions\n", "y_pred = np.array(model.predict(X))" ] }, { "cell_type": "markdown", "metadata": {}, "source": [ "### (f) How well do the models in (a) and (e) fit the data?" ] }, { "cell_type": "code", "execution_count": 66, "metadata": {}, "outputs": [ { "data": { "image/png": "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\n", "text/plain": [ "
" ] }, "metadata": {}, "output_type": "display_data" } ], "source": [ "# Analyse residuals\n", "lm_plot(lm_stats(X, y, y_pred))" ] }, { "cell_type": "markdown", "metadata": {}, "source": [ "**Comment:** The diagnostic plots above illustrate the fit of our model. The top-left plot shows only subtle discernible pattern suggesting our linear model has achieved a reasonable fit to the data. The distribution of residuals appears normal and there is no heteroscedasticity. \n", "\n", "### (g) Using the model from (e), obtain 95% confidence intervals for the coefficient(s).\n" ] }, { "cell_type": "code", "execution_count": 76, "metadata": {}, "outputs": [ { "data": { "text/html": [ "
\n", "\n", "\n", " \n", " \n", " \n", " \n", " \n", " \n", " \n", " \n", " \n", " \n", " \n", " \n", " \n", " \n", " \n", " \n", " \n", " \n", " \n", " \n", " \n", " \n", " \n", " \n", "
min.max.
Intercept11.7903214.271265
C(US)[T.Yes]0.691521.707766
Price-0.06476-0.044195
\n", "
" ], "text/plain": [ " min. max.\n", "Intercept 11.79032 14.271265\n", "C(US)[T.Yes] 0.69152 1.707766\n", "Price -0.06476 -0.044195" ] }, "execution_count": 76, "metadata": {}, "output_type": "execute_result" } ], "source": [ "# Extract 95% confidence intervals\n", "conf_inter_95 = model.conf_int(alpha=0.05)\n", "conf_inter_95.rename(index=str, columns={0: \"min.\", 1: \"max.\",})" ] }, { "cell_type": "markdown", "metadata": {}, "source": [ "### (h) Is there evidence of outliers or high leverage observations in the model from (e)?\n", "\n", "No observations exceed the studentised residual thresholds for outliers of +/-3, although a couple come close.\n", "\n", "There is one observation that stands out as particularly high leverage in the bottom-right plot above, but this observation is not an outlier." ] }, { "cell_type": "markdown", "metadata": {}, "source": [ "## 11. In this problem we will investigate the t-statistic for the null hypothesis H0 : β = 0 in simple linear regression without an intercept. To begin, we generate a predictor x and a response y as follows." ] }, { "cell_type": "code", "execution_count": 123, "metadata": {}, "outputs": [ { "data": { "image/png": "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\n", "text/plain": [ "
" ] }, "metadata": {}, "output_type": "display_data" } ], "source": [ "np.random.seed(1)\n", "x = np.random.normal(size=100)\n", "y = 2*x + np.random.normal(size=100)\n", "df = pd.DataFrame({'x': x, 'y': y}) \n", "sns.scatterplot(x=x, y=y, color='b');" ] }, { "cell_type": "markdown", "metadata": {}, "source": [ "#### (a) Perform a simple linear regression of y onto x, without an intercept. Report the coefficient estimate βˆ, the standard error of this coefficient estimate, and the t-statistic and p-value associated with the null hypothesis H0 : β = 0. Comment on these results. (You can perform regression without an intercept using the command lm(y∼x+0).)" ] }, { "cell_type": "code", "execution_count": 118, "metadata": {}, "outputs": [ { "name": "stdout", "output_type": "stream", "text": [ " OLS Regression Results \n", "==============================================================================\n", "Dep. Variable: y R-squared: 0.798\n", "Model: OLS Adj. R-squared: 0.796\n", "Method: Least Squares F-statistic: 391.7\n", "Date: Tue, 11 Sep 2018 Prob (F-statistic): 3.46e-36\n", "Time: 15:19:39 Log-Likelihood: -135.67\n", "No. Observations: 100 AIC: 273.3\n", "Df Residuals: 99 BIC: 275.9\n", "Df Model: 1 \n", "Covariance Type: nonrobust \n", "==============================================================================\n", " coef std err t P>|t| [0.025 0.975]\n", "------------------------------------------------------------------------------\n", "x 2.1067 0.106 19.792 0.000 1.896 2.318\n", "==============================================================================\n", "Omnibus: 0.880 Durbin-Watson: 2.106\n", "Prob(Omnibus): 0.644 Jarque-Bera (JB): 0.554\n", "Skew: -0.172 Prob(JB): 0.758\n", "Kurtosis: 3.119 Cond. No. 1.00\n", "==============================================================================\n", "\n", "Warnings:\n", "[1] Standard Errors assume that the covariance matrix of the errors is correctly specified.\n" ] } ], "source": [ "# Linear regression without intercept\n", "f = 'y ~ x + 0'\n", "y, X = patsy.dmatrices(f, df, return_type='dataframe')\n", "\n", "# Fit model\n", "model = smf.OLS(y, X).fit()\n", "print(model.summary(alpha=0.05))\n" ] }, { "cell_type": "markdown", "metadata": {}, "source": [ "Our model estimates the coefficient of x as 2.1067 which is a slight overestimate of the known true value of 2.0, however the 95% confidence bounds do include the true value of x. \n", "\n", "Our model suggests that there is a significant relationship between x and y.\n", "\n", "#### (b) Now perform a simple linear regression of x onto y without an intercept, and report the coefficient estimate, its standard error, and the corresponding t-statistic and p-values associated with the null hypothesis H0 : β = 0. Comment on these results." ] }, { "cell_type": "code", "execution_count": 119, "metadata": {}, "outputs": [ { "name": "stdout", "output_type": "stream", "text": [ " OLS Regression Results \n", "==============================================================================\n", "Dep. Variable: x R-squared: 0.798\n", "Model: OLS Adj. R-squared: 0.796\n", "Method: Least Squares F-statistic: 391.7\n", "Date: Tue, 11 Sep 2018 Prob (F-statistic): 3.46e-36\n", "Time: 15:24:06 Log-Likelihood: -49.891\n", "No. Observations: 100 AIC: 101.8\n", "Df Residuals: 99 BIC: 104.4\n", "Df Model: 1 \n", "Covariance Type: nonrobust \n", "==============================================================================\n", " coef std err t P>|t| [0.025 0.975]\n", "------------------------------------------------------------------------------\n", "y 0.3789 0.019 19.792 0.000 0.341 0.417\n", "==============================================================================\n", "Omnibus: 0.476 Durbin-Watson: 2.166\n", "Prob(Omnibus): 0.788 Jarque-Bera (JB): 0.631\n", "Skew: 0.115 Prob(JB): 0.729\n", "Kurtosis: 2.685 Cond. No. 1.00\n", "==============================================================================\n", "\n", "Warnings:\n", "[1] Standard Errors assume that the covariance matrix of the errors is correctly specified.\n" ] } ], "source": [ "# Linear regression without intercept\n", "f = 'x ~ y + 0'\n", "y, X = patsy.dmatrices(f, df, return_type='dataframe')\n", "\n", "# Fit model\n", "model = smf.OLS(y, X).fit()\n", "print(model.summary(alpha=0.05))" ] }, { "cell_type": "markdown", "metadata": {}, "source": [ "We know that y increases by 0.5 for every unit increase in x. Our model understimates this coefficient predicting 0.3789.\n", "\n", "The standard error is now almost 10x lower at 0.019, the t-statistic is the same, and the p-value still suggests we should reject the null hyptothesis that there is no relationship between y and x.\n", "\n", "\n", "### (c) What is the relationship between the results obtained in (a) and (b)?\n", "\n", "from a) \n", "$\\hat{y} = βx + ε$\n", "\n", "rearranging for b)\n", "\n", "$\\hat{x} = \\frac{y - ε}{β}$\n", "\n", "we know that $β = 2$\n", "\n", "The standard error is lower for b), but because the coeffient is proportionally closer to 0 the t-statistic is unaffected.\n" ] }, { "cell_type": "markdown", "metadata": {}, "source": [ "(d) see workings\n", "\n", "### (e) Using the results from (d), argue that the t-statistic for the re- gression of y onto x is the same as the t-statistic for the regression of x onto y.\n", "\n", "In the equation given for t, y and x are associative – switching their order does not effect the result.\n", "\n", "### (f) In R, show that when regression is performed with an intercept, the t-statistic for H0 : β1 = 0 is the same for the regression of y onto x as it is for the regression of x onto y." ] }, { "cell_type": "code", "execution_count": 133, "metadata": {}, "outputs": [ { "name": "stdout", "output_type": "stream", "text": [ "Intercept 1.564347\n", "x 19.782585\n", "dtype: float64\n", "Intercept -1.089559\n", "y 19.782585\n", "dtype: float64\n" ] } ], "source": [ "# Fit model\n", "model_a = smf.ols(formula='y ~ x', data=df).fit()\n", "model_b = smf.ols(formula='x ~ y', data=df).fit()\n", "\n", "print(model_a.tvalues)\n", "print(model_b.tvalues)" ] }, { "cell_type": "markdown", "metadata": {}, "source": [ "## 12. This problem involves simple linear regression without an intercept.\n", "\n", "### (a) Recall that the coefficient estimate βˆ for the linear regression of Y onto X without an intercept is given by (3.38). Under what circumstance is the coefficient estimate for the regression of X onto Y the same as the coefficient estimate for the regression of Y onto X?" ] }, { "cell_type": "markdown", "metadata": {}, "source": [ "when \n", "\n", "$\\sum_{i=1}^{n} x_i = \\sum_{i=1}^{n} y_i$\n", "\n", "in which case $\\hat{β} = 1$\n", "\n", "### (b) Generate an example in R with n = 100 observations in which the coefficient estimate for the regression of X onto Y is different from the coefficient estimate for the regression of Y onto X." ] }, { "cell_type": "code", "execution_count": 139, "metadata": {}, "outputs": [ { "data": { "image/png": "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\n", "text/plain": [ "
" ] }, "metadata": {}, "output_type": "display_data" } ], "source": [ "np.random.seed(2)\n", "x = np.random.normal(size=100)\n", "y = 1.2*x\n", "df = pd.DataFrame({'x': x, 'y': y})\n", "sns.scatterplot(x=x, y=y);" ] }, { "cell_type": "code", "execution_count": 142, "metadata": {}, "outputs": [ { "name": "stdout", "output_type": "stream", "text": [ "Intercept -1.387779e-16\n", "x 1.200000e+00\n", "dtype: float64\n", "Intercept -2.775558e-17\n", "y 8.333333e-01\n", "dtype: float64\n" ] } ], "source": [ "model_a = smf.ols(formula='y ~ x', data=df).fit()\n", "model_b = smf.ols(formula='x ~ y', data=df).fit()\n", "\n", "print(model_a.params)\n", "print(model_b.params)" ] }, { "cell_type": "markdown", "metadata": {}, "source": [ "### (c) Generate an example in R with n = 100 observations in which the coefficient estimate for the regression of X onto Y is the same as the coefficient estimate for the regression of Y onto X.\n" ] }, { "cell_type": "code", "execution_count": 150, "metadata": {}, "outputs": [ { "name": "stdout", "output_type": "stream", "text": [ "Intercept -1.179612e-16\n", "x 1.000000e+00\n", "dtype: float64\n", "Intercept -1.179612e-16\n", "y 1.000000e+00\n", "dtype: float64\n" ] }, { "data": { "image/png": "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\n", "text/plain": [ "
" ] }, "metadata": {}, "output_type": "display_data" } ], "source": [ "x = np.random.normal(size=100)\n", "y = x\n", "df = pd.DataFrame({'x': x, 'y': y})\n", "sns.scatterplot(x=x, y=y);\n", "\n", "model_a = smf.ols(formula='y ~ x', data=df).fit()\n", "model_b = smf.ols(formula='x ~ y', data=df).fit()\n", "\n", "print(model_a.params)\n", "print(model_b.params)" ] }, { "cell_type": "markdown", "metadata": {}, "source": [ "## 13. In this exercise you will create some simulated data and will fit simple linear regression models to it. Make sure to use set.seed(1) prior to starting part (a) to ensure consistent results.\n", "\n", "### (a) Using the rnorm() function, create a vector, x, containing 100 observations drawn from a N(0,1) distribution. This represents a feature, X." ] }, { "cell_type": "code", "execution_count": 221, "metadata": {}, "outputs": [], "source": [ "np.random.seed(1)\n", "\n", "mu, sigma = 0, 1\n", "x = np.random.normal(mu, sigma, 100)" ] }, { "cell_type": "markdown", "metadata": {}, "source": [ "### (b) Using the rnorm() function, create a vector, eps, containing 100 observations drawn from a N(0,0.25) distribution i.e. a normal distribution with mean zero and variance 0.25." ] }, { "cell_type": "code", "execution_count": 222, "metadata": {}, "outputs": [], "source": [ "mu, sigma = 0, 0.25\n", "eps = np.random.normal(mu, sigma, 100)" ] }, { "cell_type": "markdown", "metadata": {}, "source": [ "### (c) Using x and eps, generate a vector y according to the model\n", "\n", "Y =−1+0.5X+ε. (3.39) " ] }, { "cell_type": "code", "execution_count": 223, "metadata": {}, "outputs": [], "source": [ "y = -1 + 0.5*x + eps" ] }, { "cell_type": "markdown", "metadata": {}, "source": [ "### What is the length of the vector y? What are the values of β0 and β1 in this linear model?\n", "\n", "\n", "β0 = -1 \n", "β1 = 0.5" ] }, { "cell_type": "code", "execution_count": 224, "metadata": {}, "outputs": [ { "name": "stdout", "output_type": "stream", "text": [ "y length: 10.660986158906175\n" ] } ], "source": [ "print('y length: ' + str(np.linalg.norm(y)))" ] }, { "cell_type": "markdown", "metadata": {}, "source": [ "### (d) Create a scatterplot displaying the relationship between x and y. Comment on what you observe." ] }, { "cell_type": "code", "execution_count": 225, "metadata": {}, "outputs": [ { "data": { "image/png": "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\n", "text/plain": [ "
" ] }, "metadata": {}, "output_type": "display_data" } ], "source": [ "ax = sns.scatterplot(x=x, y=y)\n", "plt.xlabel('x')\n", "plt.ylabel('y');" ] }, { "cell_type": "markdown", "metadata": {}, "source": [ "The above plot suggests a linear relationship between x and y with an intercept of roughly -1 and gradient of 0.5\n", "\n", "### (e) Fit a least squares linear model to predict y using x. Comment on the model obtained. How do βˆ0 and βˆ1 compare to β0 and β1?" ] }, { "cell_type": "code", "execution_count": 226, "metadata": {}, "outputs": [ { "name": "stdout", "output_type": "stream", "text": [ " OLS Regression Results \n", "==============================================================================\n", "Dep. Variable: y R-squared: 0.800\n", "Model: OLS Adj. R-squared: 0.798\n", "Method: Least Squares F-statistic: 391.4\n", "Date: Tue, 11 Sep 2018 Prob (F-statistic): 5.39e-36\n", "Time: 20:34:37 Log-Likelihood: 4.1908\n", "No. Observations: 100 AIC: -4.382\n", "Df Residuals: 98 BIC: 0.8288\n", "Df Model: 1 \n", "Covariance Type: nonrobust \n", "==============================================================================\n", " coef std err t P>|t| [0.025 0.975]\n", "------------------------------------------------------------------------------\n", "Intercept -0.9632 0.023 -40.999 0.000 -1.010 -0.917\n", "x 0.5239 0.026 19.783 0.000 0.471 0.576\n", "==============================================================================\n", "Omnibus: 0.898 Durbin-Watson: 2.157\n", "Prob(Omnibus): 0.638 Jarque-Bera (JB): 0.561\n", "Skew: -0.172 Prob(JB): 0.755\n", "Kurtosis: 3.127 Cond. No. 1.15\n", "==============================================================================\n", "\n", "Warnings:\n", "[1] Standard Errors assume that the covariance matrix of the errors is correctly specified.\n" ] } ], "source": [ "model = smf.ols(formula='y ~ x', data=pd.DataFrame({'x':x, 'y':y})).fit()\n", "print(model.summary())" ] }, { "cell_type": "markdown", "metadata": {}, "source": [ "$\\hat{β_0} = -0.9632$ \n", "$\\hat{β_1} = 0.5239$\n", "\n", "The model estimates parameters close to the true values. The intercept is overestimated and the gradient is overestimated.\n", "\n", "### (f) Display the least squares line on the scatterplot obtained in (d). Draw the population regression line on the plot, in a different color. Use the legend() command to create an appropriate legend." ] }, { "cell_type": "code", "execution_count": 232, "metadata": {}, "outputs": [ { "data": { "image/png": "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\n", "text/plain": [ "
" ] }, "metadata": {}, "output_type": "display_data" } ], "source": [ "y_pred = model.predict()\n", "y_act = -1+(0.5*x)\n", "\n", "plt.figure(figsize=(8,8))\n", "ax = sns.scatterplot(x=x, y=y)\n", "plt.xlabel('x')\n", "plt.ylabel('y')\n", "ax.plot(x, model.predict(), color='r')\n", "ax.plot(x, -1+(0.5*x), color='y')\n", "ax.legend(['Least squares', 'Population regression']);" ] }, { "cell_type": "markdown", "metadata": {}, "source": [ "### (g) Now fit a polynomial regression model that predicts y using x and x2. Is there evidence that the quadratic term improves the model fit? Explain your answer." ] }, { "cell_type": "code", "execution_count": 235, "metadata": {}, "outputs": [ { "name": "stdout", "output_type": "stream", "text": [ " OLS Regression Results \n", "==============================================================================\n", "Dep. Variable: y R-squared: 0.800\n", "Model: OLS Adj. R-squared: 0.796\n", "Method: Least Squares F-statistic: 193.8\n", "Date: Wed, 12 Sep 2018 Prob (F-statistic): 1.32e-34\n", "Time: 12:08:50 Log-Likelihood: 4.2077\n", "No. Observations: 100 AIC: -2.415\n", "Df Residuals: 97 BIC: 5.400\n", "Df Model: 2 \n", "Covariance Type: nonrobust \n", "================================================================================\n", " coef std err t P>|t| [0.025 0.975]\n", "--------------------------------------------------------------------------------\n", "Intercept -0.9663 0.029 -33.486 0.000 -1.024 -0.909\n", "x 0.5234 0.027 19.582 0.000 0.470 0.576\n", "np.square(x) 0.0039 0.021 0.181 0.856 -0.038 0.046\n", "==============================================================================\n", "Omnibus: 0.893 Durbin-Watson: 2.152\n", "Prob(Omnibus): 0.640 Jarque-Bera (JB): 0.552\n", "Skew: -0.170 Prob(JB): 0.759\n", "Kurtosis: 3.132 Cond. No. 2.10\n", "==============================================================================\n", "\n", "Warnings:\n", "[1] Standard Errors assume that the covariance matrix of the errors is correctly specified.\n" ] }, { "data": { "image/png": "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\n", "text/plain": [ "
" ] }, "metadata": {}, "output_type": "display_data" } ], "source": [ "f = 'y ~ x + np.square(x)'\n", "model = smf.ols(formula=f, data=pd.DataFrame({'x':x, 'y':y})).fit()\n", "print(model.summary())\n", "\n", "# predict\n", "y_pred = model.predict()\n", "y_act = -1+(0.5*x)\n", "\n", "# plot\n", "plt.figure(figsize=(8,8))\n", "ax = sns.scatterplot(x=x, y=y)\n", "plt.xlabel('x')\n", "plt.ylabel('y')\n", "ax.plot(x, model.predict(), color='r')\n", "ax.plot(x, -1+(0.5*x), color='y')\n", "ax.legend(['Least squares quadratic', 'Population regression']);" ] }, { "cell_type": "markdown", "metadata": {}, "source": [ "The model selects a small coefficient for the quadratic variable and so the fit of our model is only slightly affected. The p-value > 0.05 for this variable suggests that it is not statistically significant in representing our data. The R-squared value is identical for both models suggesting that the quadratic model hasn't reduced the error of our model in fitting the data.\n", "\n", "Overall the quadratic term does not seem to have improved our model.\n", "\n", "### (h) Repeat (a)–(f) after modifying the data generation process in such a way that there is less noise in the data. The model (3.39) should remain the same. You can do this by decreasing the variance of the normal distribution used to generate the error term ε in (b). Describe your results." ] }, { "cell_type": "code", "execution_count": 237, "metadata": { "scrolled": true }, "outputs": [ { "name": "stdout", "output_type": "stream", "text": [ "y length: 10.617729160380613\n", " OLS Regression Results \n", "==============================================================================\n", "Dep. Variable: y R-squared: 0.989\n", "Model: OLS Adj. R-squared: 0.989\n", "Method: Least Squares F-statistic: 9084.\n", "Date: Wed, 12 Sep 2018 Prob (F-statistic): 1.97e-98\n", "Time: 12:35:35 Log-Likelihood: 165.13\n", "No. Observations: 100 AIC: -326.3\n", "Df Residuals: 98 BIC: -321.1\n", "Df Model: 1 \n", "Covariance Type: nonrobust \n", "==============================================================================\n", " coef std err t P>|t| [0.025 0.975]\n", "------------------------------------------------------------------------------\n", "Intercept -0.9926 0.005 -211.252 0.000 -1.002 -0.983\n", "x 0.5048 0.005 95.309 0.000 0.494 0.515\n", "==============================================================================\n", "Omnibus: 0.898 Durbin-Watson: 2.157\n", "Prob(Omnibus): 0.638 Jarque-Bera (JB): 0.561\n", "Skew: -0.172 Prob(JB): 0.755\n", "Kurtosis: 3.127 Cond. No. 1.15\n", "==============================================================================\n", "\n", "Warnings:\n", "[1] Standard Errors assume that the covariance matrix of the errors is correctly specified.\n" ] }, { "data": { "image/png": "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\n", "text/plain": [ "
" ] }, "metadata": {}, "output_type": "display_data" } ], "source": [ "np.random.seed(1)\n", "\n", "# generate x\n", "mu, sigma = 0, 1\n", "x = np.random.normal(mu, sigma, 100)\n", "\n", "# generate epsilon\n", "mu, sigma = 0, 0.05\n", "eps = np.random.normal(mu, sigma, 100)\n", "\n", "# generate y\n", "y = -1 + 0.5*x + eps\n", "print('y length: ' + str(np.linalg.norm(y)))\n", "\n", "# Fit model\n", "model = smf.ols(formula='y ~ x', data=pd.DataFrame({'x':x, 'y':y})).fit()\n", "print(model.summary())\n", "\n", "# Make predictions\n", "y_pred = model.predict()\n", "y_act = -1+(0.5*x)\n", "\n", "# Plot results\n", "plt.figure(figsize=(8,8))\n", "ax = sns.scatterplot(x=x, y=y)\n", "plt.xlabel('x')\n", "plt.ylabel('y')\n", "ax.plot(x, model.predict(), color='r')\n", "ax.plot(x, -1+(0.5*x), color='y')\n", "ax.legend(['Least squares', 'Population regression']);" ] }, { "cell_type": "markdown", "metadata": {}, "source": [ "A dataset with reduced variance results in reduced error in our model. The value of R-squared has increased from 0.8 to 0.989 suggesting a much closer approaximation to the true value of f(x) - this can be seen in reduced dispartiy between the least squares line and the population regression line in th eplot above.\n", "\n", "### (i) Repeat (a)–(f) after modifying the data generation process in such a way that there is more noise in the data. The model (3.39) should remain the same. You can do this by increasing the variance of the normal distribution used to generate the error term ε in (b). Describe your results." ] }, { "cell_type": "code", "execution_count": 238, "metadata": {}, "outputs": [ { "name": "stdout", "output_type": "stream", "text": [ "y length: 11.173260452749581\n", " OLS Regression Results \n", "==============================================================================\n", "Dep. Variable: y R-squared: 0.522\n", "Model: OLS Adj. R-squared: 0.517\n", "Method: Least Squares F-statistic: 107.0\n", "Date: Wed, 12 Sep 2018 Prob (F-statistic): 2.20e-17\n", "Time: 12:39:10 Log-Likelihood: -65.124\n", "No. Observations: 100 AIC: 134.2\n", "Df Residuals: 98 BIC: 139.5\n", "Df Model: 1 \n", "Covariance Type: nonrobust \n", "==============================================================================\n", " coef std err t P>|t| [0.025 0.975]\n", "------------------------------------------------------------------------------\n", "Intercept -0.9265 0.047 -19.717 0.000 -1.020 -0.833\n", "x 0.5477 0.053 10.342 0.000 0.443 0.653\n", "==============================================================================\n", "Omnibus: 0.898 Durbin-Watson: 2.157\n", "Prob(Omnibus): 0.638 Jarque-Bera (JB): 0.561\n", "Skew: -0.172 Prob(JB): 0.755\n", "Kurtosis: 3.127 Cond. No. 1.15\n", "==============================================================================\n", "\n", "Warnings:\n", "[1] Standard Errors assume that the covariance matrix of the errors is correctly specified.\n" ] }, { "data": { "image/png": "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\n", "text/plain": [ "
" ] }, "metadata": {}, "output_type": "display_data" } ], "source": [ "np.random.seed(1)\n", "\n", "# generate x\n", "mu, sigma = 0, 1\n", "x = np.random.normal(mu, sigma, 100)\n", "\n", "# generate epsilon\n", "mu, sigma = 0, 0.5\n", "eps = np.random.normal(mu, sigma, 100)\n", "\n", "# generate y\n", "y = -1 + 0.5*x + eps\n", "print('y length: ' + str(np.linalg.norm(y)))\n", "\n", "# Fit model\n", "model = smf.ols(formula='y ~ x', data=pd.DataFrame({'x':x, 'y':y})).fit()\n", "print(model.summary())\n", "\n", "# Make predictions\n", "y_pred = model.predict()\n", "y_act = -1+(0.5*x)\n", "\n", "# Plot results\n", "plt.figure(figsize=(8,8))\n", "ax = sns.scatterplot(x=x, y=y)\n", "plt.xlabel('x')\n", "plt.ylabel('y')\n", "ax.plot(x, model.predict(), color='r')\n", "ax.plot(x, -1+(0.5*x), color='y')\n", "ax.legend(['Least squares', 'Population regression']);" ] }, { "cell_type": "markdown", "metadata": {}, "source": [ "Increasing the variance in the dataset has increased the error in our model. This is reflected by a reduction in R-squared and an increased disparity between the least squares line and the true population regression line.\n", "\n", "### (j) What are the confidence intervals for β0 and β1 based on the original data set, the noisier data set, and the less noisy data set? Comment on your results.\n", "\n", "**Less Noisy: Epsilon = 0.05**\n", "```\n", " [0.025 0.975]\n", "-------------------------------\n", "Intercept -1.002 -0.983\n", "x 0.494 0.515\n", "```\n", "\n", "**Noisier: Epsilon = 0.5**\n", "```\n", " [0.025 0.975]\n", "------------------------------\n", "Intercept -1.020 -0.833\n", "x 0.443 0.653\n", "\n", "```\n", "\n", "Increasing the noise in varaince in our dataset appears to have increased the confidence interval in our model. Increased variance increases uncertainty." ] }, { "cell_type": "markdown", "metadata": {}, "source": [ "## 14. This problem focuses on the collinearity problem.\n", "\n", "#### (a) Perform the following commands in R:\n", "\n", "```\n", "set.seed(1)\n", "x1=runif(100)\n", "x2=0.5*x1+rnorm(100)/10\n", "y=2+2*x1+0.3*x2+rnorm(100)\n", "```\n", "\n", "The last line corresponds to creating a linear model in which y is a function of x1 and x2. Write out the form of the linear model. What are the regression coefficients?" ] }, { "cell_type": "code", "execution_count": 296, "metadata": {}, "outputs": [], "source": [ "np.random.seed(1)\n", "x1 = np.random.uniform(size=100)\n", "x2 = 0.5*x1 + np.random.randn(100)/10\n", "y = 2 + 2*x1 + 0.3*x2 + np.random.randn(100)" ] }, { "cell_type": "markdown", "metadata": {}, "source": [ "Form of the linear model is:\n", "\n", "$\\hat{y} = β_0 + β_1 x_1 + β_2 x_2 + ɛ$\n", "\n", "Where the coefficients are:\n", "\n", "$β_0 = 2$ \n", "$β_1 = 2$ \n", "$β_2 = 0.3$ \n", "\n", "### (b) What is the correlation between x1 and x2? Create a scatterplot displaying the relationship between the variables." ] }, { "cell_type": "code", "execution_count": 297, "metadata": {}, "outputs": [ { "name": "stdout", "output_type": "stream", "text": [ "Correlation coefficient: 0.8390596222844917\n" ] }, { "data": { "image/png": "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\n", "text/plain": [ "
" ] }, "metadata": {}, "output_type": "display_data" } ], "source": [ "df = pd.DataFrame({'x1':x1, 'x2':x2, 'y':y})\n", "sns.scatterplot(x='x1', y='x2', data=df);\n", "\n", "print('Correlation coefficient: ' + str(np.corrcoef(x1, x2)[0][1]))" ] }, { "cell_type": "markdown", "metadata": {}, "source": [ "There is correlation between x1 and x2 with coefficient 0.839. The plot suggests a linear relationship with a gradient of around 0.5 and high variance.\n", "\n", "### (c) Using this data, fit a least squares regression to predict y using x1 and x2. Describe the results obtained. What are βˆ0, βˆ1, and βˆ2? How do these relate to the true β0, β1, and β2? Can you reject the null hypothesis H0 : β1 = 0? How about the null hypothesis H0 : β2 = 0?" ] }, { "cell_type": "code", "execution_count": 302, "metadata": {}, "outputs": [ { "data": { "image/png": "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\n", "text/plain": [ "
" ] }, "metadata": {}, "output_type": "display_data" }, { "name": "stdout", "output_type": "stream", "text": [ " OLS Regression Results \n", "==============================================================================\n", "Dep. Variable: y R-squared: 0.261\n", "Model: OLS Adj. R-squared: 0.245\n", "Method: Least Squares F-statistic: 17.09\n", "Date: Wed, 12 Sep 2018 Prob (F-statistic): 4.40e-07\n", "Time: 16:15:57 Log-Likelihood: -142.34\n", "No. Observations: 100 AIC: 290.7\n", "Df Residuals: 97 BIC: 298.5\n", "Df Model: 2 \n", "Covariance Type: nonrobust \n", "==============================================================================\n", " coef std err t P>|t| [0.025 0.975]\n", "------------------------------------------------------------------------------\n", "Intercept 2.1893 0.199 11.021 0.000 1.795 2.584\n", "x1 0.7046 0.637 1.107 0.271 -0.559 1.968\n", "x2 2.5024 1.140 2.194 0.031 0.239 4.766\n", "==============================================================================\n", "Omnibus: 0.607 Durbin-Watson: 2.111\n", "Prob(Omnibus): 0.738 Jarque-Bera (JB): 0.577\n", "Skew: -0.179 Prob(JB): 0.749\n", "Kurtosis: 2.900 Cond. No. 14.3\n", "==============================================================================\n", "\n", "Warnings:\n", "[1] Standard Errors assume that the covariance matrix of the errors is correctly specified.\n" ] } ], "source": [ "df = pd.DataFrame({'x1':x1, 'x2':x2, 'y':y})\n", "\n", "# Plot\n", "fig = plt.figure()\n", "ax = fig.add_subplot(111, projection='3d')\n", "ax.scatter(df['x1'], df['x2'], df['y'])\n", "ax.view_init(30, 185)\n", "plt.xlabel('x1')\n", "plt.ylabel('x2')\n", "plt.show()\n", "\n", "\n", "# Fit model\n", "f = 'y ~ x1 + x2'\n", "model = smf.ols(formula=f, data=df).fit()\n", "print(model.summary())\n" ] }, { "cell_type": "markdown", "metadata": {}, "source": [ "$β_0 = 2$ \n", "$β_1 = 2$ \n", "$β_2 = 0.3$ \n", "\n", "$\\hat{β_0} = 2.1893$ \n", "$\\hat{β_1} = 0.7046$ \n", "$\\hat{β_2} = 2.5024$ \n", "\n", "- The intercept $\\hat{β_0}$ is estimated fairly accurately.\n", "- $\\hat{β_1}$ and $\\hat{β_2}$ are not well estimated. It's almost as though our model has got these the wrong way around.\n", "- we **cannot** reject the null hypothesis for $\\hat{β_1}$\n", "- we **can** reject the null hypothesis for $\\hat{β_2}$\n", "\n", "### (d) Now fit a least squares regression to predict y using only x1. Comment on your results. Can you reject the null hypothesis H0 :β1 =0?\n" ] }, { "cell_type": "code", "execution_count": 336, "metadata": {}, "outputs": [ { "name": "stdout", "output_type": "stream", "text": [ " OLS Regression Results \n", "==============================================================================\n", "Dep. Variable: y R-squared: 0.224\n", "Model: OLS Adj. R-squared: 0.216\n", "Method: Least Squares F-statistic: 28.26\n", "Date: Wed, 12 Sep 2018 Prob (F-statistic): 6.68e-07\n", "Time: 18:06:55 Log-Likelihood: -144.76\n", "No. Observations: 100 AIC: 293.5\n", "Df Residuals: 98 BIC: 298.7\n", "Df Model: 1 \n", "Covariance Type: nonrobust \n", "==============================================================================\n", " coef std err t P>|t| [0.025 0.975]\n", "------------------------------------------------------------------------------\n", "Intercept 2.2486 0.201 11.209 0.000 1.850 2.647\n", "x1 1.8770 0.353 5.316 0.000 1.176 2.578\n", "==============================================================================\n", "Omnibus: 0.292 Durbin-Watson: 2.123\n", "Prob(Omnibus): 0.864 Jarque-Bera (JB): 0.468\n", "Skew: -0.003 Prob(JB): 0.791\n", "Kurtosis: 2.665 Cond. No. 4.26\n", "==============================================================================\n", "\n", "Warnings:\n", "[1] Standard Errors assume that the covariance matrix of the errors is correctly specified.\n" ] }, { "data": { "image/png": "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\n", "text/plain": [ "
" ] }, "metadata": {}, "output_type": "display_data" } ], "source": [ "# Fit model\n", "f = 'y ~ x1'\n", "model = smf.ols(formula=f, data=df).fit()\n", "print(model.summary())\n", "\n", "df = pd.DataFrame({'x1':x1, 'x2':x2, 'y':y})\n", "ax = sns.scatterplot(x='x1', y='y', data=df);\n", "plt.xlabel('x1')\n", "plt.ylabel('y')\n", "ax.plot(df['x1'].sort_values(), model.predict(df['x1'].sort_values()), color='r')\n", "ax.plot(x, 2+(2*x), color='y')\n", "ax.legend(['Least squares', 'Population regression']);\n", "\n" ] }, { "cell_type": "markdown", "metadata": {}, "source": [ "$β_0 = 2$ \n", "$β_1 = 2$ \n", "\n", "$\\hat{β_0} = 2.2486$ \n", "$\\hat{β_1} = 1.8770$ \n", "\n", "- Using only x1 our model makes a much improved prediction of the x1 coefficient $\\hat{β_1}$. \n", "- The p-value now suggests we can reject the null hypothesis H0 :β1 =0 \n", "\n", "### (e) Now fit a least squares regression to predict y using only x2. Comment on your results. Can you reject the null hypothesis H0 :β1 =0?" ] }, { "cell_type": "code", "execution_count": 337, "metadata": {}, "outputs": [ { "name": "stdout", "output_type": "stream", "text": [ " OLS Regression Results \n", "==============================================================================\n", "Dep. Variable: y R-squared: 0.251\n", "Model: OLS Adj. R-squared: 0.244\n", "Method: Least Squares F-statistic: 32.87\n", "Date: Wed, 12 Sep 2018 Prob (F-statistic): 1.09e-07\n", "Time: 18:07:54 Log-Likelihood: -142.97\n", "No. Observations: 100 AIC: 289.9\n", "Df Residuals: 98 BIC: 295.1\n", "Df Model: 1 \n", "Covariance Type: nonrobust \n", "==============================================================================\n", " coef std err t P>|t| [0.025 0.975]\n", "------------------------------------------------------------------------------\n", "Intercept 2.2655 0.187 12.145 0.000 1.895 2.636\n", "x2 3.5613 0.621 5.733 0.000 2.329 4.794\n", "==============================================================================\n", "Omnibus: 0.857 Durbin-Watson: 2.117\n", "Prob(Omnibus): 0.651 Jarque-Bera (JB): 0.746\n", "Skew: -0.210 Prob(JB): 0.689\n", "Kurtosis: 2.946 Cond. No. 6.48\n", "==============================================================================\n", "\n", "Warnings:\n", "[1] Standard Errors assume that the covariance matrix of the errors is correctly specified.\n" ] }, { "data": { "image/png": "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\n", "text/plain": [ "
" ] }, "metadata": {}, "output_type": "display_data" } ], "source": [ "# Fit model\n", "f = 'y ~ x2'\n", "model = smf.ols(formula=f, data=df).fit()\n", "print(model.summary())\n", "\n", "\n", "df = pd.DataFrame({'x1':x1, 'x2':x2, 'y':y})\n", "ax = sns.scatterplot(x='x2', y='y', data=df);\n", "plt.xlabel('x2')\n", "plt.ylabel('y')\n", "ax.plot(df['x2'].sort_values(), model.predict(df['x2'].sort_values()), color='r')\n", "ax.plot(x, 2+(0.3*x), color='y')\n", "ax.legend(['Least squares', 'Population regression']);\n" ] }, { "cell_type": "markdown", "metadata": {}, "source": [ "$β_0 = 2$ \n", "$β_2 = 0.3$ \n", "\n", "$\\hat{β_0} = 2.2655$ \n", "$\\hat{β_2} = 3.5613$ \n", "\n", "Our model overestimates the coefficient. The p-value suggests we can reject the null hypothesis H0 :β2 =0. \n", "\n", "### (f) Do the results obtained in (c)–(e) contradict each other? Explain your answer.\n", "\n", "#### Ground truth\n", "\n", "$β_0 = 2$ \n", "$β_1 = 2$ \n", "$β_2 = 0.3$ \n", "\n", "#### c) y ~ x1 + x2 \n", "```\n", " coef std err t P>|t| [0.025 0.975]\n", "------------------------------------------------------------------------------\n", "Intercept 2.1893 0.199 11.021 0.000 1.795 2.584\n", "x1 0.7046 0.637 1.107 0.271 -0.559 1.968\n", "x2 2.5024 1.140 2.194 0.031 0.239 4.766\n", "```\n", "\n", "#### d) y ~ x1\n", "```\n", " coef std err t P>|t| [0.025 0.975]\n", "------------------------------------------------------------------------------\n", "Intercept 2.2486 0.201 11.209 0.000 1.850 2.647\n", "x1 1.8770 0.353 5.316 0.000 1.176 2.578\n", "```\n", "\n", "#### d) y ~ x2\n", "```\n", " coef std err t P>|t| [0.025 0.975]\n", "------------------------------------------------------------------------------\n", "Intercept 2.2655 0.187 12.145 0.000 1.895 2.636\n", "x2 3.5613 0.621 5.733 0.000 2.329 4.794\n", "```\n", "\n", "$β_1$\n", "- result c) suggests probability < 0.271 that the null hypothesis $H0 :β_1 =0$ is true\n", "- result d) suggests probability < 0.000 that the null hypothesis $H0 :β_1 =0$ is true\n", "\n", "$β_2$\n", "- result c) suggests probability < 0.031 that the null hypothesis $H0 :β_2 =0$ is true\n", "- result e) suggests probability < 0.000 that the null hypothesis $H0 :β_2 =0$ is true\n", "\n", "Yes. We get contradictory results for the p-value associated with $β_1$, which in results c woul cause us to accept the null hypothesis and in d) we would reject it.\n", "\n", "This contradiction can be explained by the collinearity of x1 and x2 which causes an increased standard error for all $β_j$ when our model incorporates both features, which in turn increases the p-values and causes us to incorrectly accept the null hypothesis $H0 :β_1 =0$ when our model uses both features.\n", "\n", "**Revision Notes:** \n", "- Why is coefficient for x2 estimated at ~4, not 0.3?\n", "\n", "### (g) Now suppose we obtain one additional observation, which was unfortunately mismeasured.\n", "\n", "```\n", "x1=c(x1, 0.1) \n", "x2=c(x2, 0.8)\n", "y=c(y,6)\n", "```\n", "Re-fit the linear models from (c) to (e) using this new data. What effect does this new observation have on the each of the models? In each model, is this observation an outlier? A high-leverage point? Both? Explain your answers.\n" ] }, { "cell_type": "code", "execution_count": 338, "metadata": {}, "outputs": [], "source": [ "df2 = df.append({'x1': 0.1, 'x2': 0.8, 'y': 6}, ignore_index=True)\n", "\n", "\n", "# Fit models\n", "model_c = smf.ols(formula='y ~ x1 + x2', data=df2).fit()\n", "model_d = smf.ols(formula='y ~ x1', data=df2).fit()\n", "model_e = smf.ols(formula='y ~ x2', data=df2).fit()" ] }, { "cell_type": "code", "execution_count": 341, "metadata": {}, "outputs": [ { "data": { "text/html": [ "\n", "\n", "\n", " \n", "\n", "\n", " \n", "\n", "\n", " \n", "\n", "\n", " \n", "\n", "\n", " \n", "\n", "\n", " \n", "\n", "\n", " \n", "\n", "\n", " \n", "\n", "\n", " \n", "\n", "
OLS Regression Results
Dep. Variable: y R-squared: 0.289
Model: OLS Adj. R-squared: 0.275
Method: Least Squares F-statistic: 19.92
Date: Wed, 12 Sep 2018 Prob (F-statistic): 5.51e-08
Time: 18:12:29 Log-Likelihood: -144.15
No. Observations: 101 AIC: 294.3
Df Residuals: 98 BIC: 302.1
Df Model: 2
Covariance Type: nonrobust
\n", "\n", "\n", " \n", "\n", "\n", " \n", "\n", "\n", " \n", "\n", "\n", " \n", "\n", "
coef std err t P>|t| [0.025 0.975]
Intercept 2.1996 0.199 11.041 0.000 1.804 2.595
x1 0.2200 0.520 0.423 0.673 -0.812 1.252
x2 3.4392 0.892 3.857 0.000 1.670 5.209
\n", "\n", "\n", " \n", "\n", "\n", " \n", "\n", "\n", " \n", "\n", "\n", " \n", "\n", "
Omnibus: 1.246 Durbin-Watson: 2.089
Prob(Omnibus): 0.536 Jarque-Bera (JB): 1.094
Skew: -0.254 Prob(JB): 0.579
Kurtosis: 2.956 Cond. No. 11.1


Warnings:
[1] Standard Errors assume that the covariance matrix of the errors is correctly specified." ], "text/plain": [ "\n", "\"\"\"\n", " OLS Regression Results \n", "==============================================================================\n", "Dep. Variable: y R-squared: 0.289\n", "Model: OLS Adj. R-squared: 0.275\n", "Method: Least Squares F-statistic: 19.92\n", "Date: Wed, 12 Sep 2018 Prob (F-statistic): 5.51e-08\n", "Time: 18:12:29 Log-Likelihood: -144.15\n", "No. Observations: 101 AIC: 294.3\n", "Df Residuals: 98 BIC: 302.1\n", "Df Model: 2 \n", "Covariance Type: nonrobust \n", "==============================================================================\n", " coef std err t P>|t| [0.025 0.975]\n", "------------------------------------------------------------------------------\n", "Intercept 2.1996 0.199 11.041 0.000 1.804 2.595\n", "x1 0.2200 0.520 0.423 0.673 -0.812 1.252\n", "x2 3.4392 0.892 3.857 0.000 1.670 5.209\n", "==============================================================================\n", "Omnibus: 1.246 Durbin-Watson: 2.089\n", "Prob(Omnibus): 0.536 Jarque-Bera (JB): 1.094\n", "Skew: -0.254 Prob(JB): 0.579\n", "Kurtosis: 2.956 Cond. No. 11.1\n", "==============================================================================\n", "\n", "Warnings:\n", "[1] Standard Errors assume that the covariance matrix of the errors is correctly specified.\n", "\"\"\"" ] }, "execution_count": 341, "metadata": {}, "output_type": "execute_result" } ], "source": [ "model_c.summary()" ] }, { "cell_type": "code", "execution_count": 342, "metadata": {}, "outputs": [ { "data": { "text/html": [ "\n", "\n", "\n", " \n", "\n", "\n", " \n", "\n", "\n", " \n", "\n", "\n", " \n", "\n", "\n", " \n", "\n", "\n", " \n", "\n", "\n", " \n", "\n", "\n", " \n", "\n", "\n", " \n", "\n", "
OLS Regression Results
Dep. Variable: y R-squared: 0.181
Model: OLS Adj. R-squared: 0.173
Method: Least Squares F-statistic: 21.89
Date: Wed, 12 Sep 2018 Prob (F-statistic): 9.15e-06
Time: 18:12:34 Log-Likelihood: -151.28
No. Observations: 101 AIC: 306.6
Df Residuals: 99 BIC: 311.8
Df Model: 1
Covariance Type: nonrobust
\n", "\n", "\n", " \n", "\n", "\n", " \n", "\n", "\n", " \n", "\n", "
coef std err t P>|t| [0.025 0.975]
Intercept 2.3583 0.208 11.331 0.000 1.945 2.771
x1 1.7225 0.368 4.679 0.000 0.992 2.453
\n", "\n", "\n", " \n", "\n", "\n", " \n", "\n", "\n", " \n", "\n", "\n", " \n", "\n", "
Omnibus: 1.904 Durbin-Watson: 1.975
Prob(Omnibus): 0.386 Jarque-Bera (JB): 1.388
Skew: 0.263 Prob(JB): 0.500
Kurtosis: 3.232 Cond. No. 4.23


Warnings:
[1] Standard Errors assume that the covariance matrix of the errors is correctly specified." ], "text/plain": [ "\n", "\"\"\"\n", " OLS Regression Results \n", "==============================================================================\n", "Dep. Variable: y R-squared: 0.181\n", "Model: OLS Adj. R-squared: 0.173\n", "Method: Least Squares F-statistic: 21.89\n", "Date: Wed, 12 Sep 2018 Prob (F-statistic): 9.15e-06\n", "Time: 18:12:34 Log-Likelihood: -151.28\n", "No. Observations: 101 AIC: 306.6\n", "Df Residuals: 99 BIC: 311.8\n", "Df Model: 1 \n", "Covariance Type: nonrobust \n", "==============================================================================\n", " coef std err t P>|t| [0.025 0.975]\n", "------------------------------------------------------------------------------\n", "Intercept 2.3583 0.208 11.331 0.000 1.945 2.771\n", "x1 1.7225 0.368 4.679 0.000 0.992 2.453\n", "==============================================================================\n", "Omnibus: 1.904 Durbin-Watson: 1.975\n", "Prob(Omnibus): 0.386 Jarque-Bera (JB): 1.388\n", "Skew: 0.263 Prob(JB): 0.500\n", "Kurtosis: 3.232 Cond. No. 4.23\n", "==============================================================================\n", "\n", "Warnings:\n", "[1] Standard Errors assume that the covariance matrix of the errors is correctly specified.\n", "\"\"\"" ] }, "execution_count": 342, "metadata": {}, "output_type": "execute_result" } ], "source": [ "model_d.summary()" ] }, { "cell_type": "code", "execution_count": 343, "metadata": {}, "outputs": [ { "data": { "text/html": [ "\n", "\n", "\n", " \n", "\n", "\n", " \n", "\n", "\n", " \n", "\n", "\n", " \n", "\n", "\n", " \n", "\n", "\n", " \n", "\n", "\n", " \n", "\n", "\n", " \n", "\n", "\n", " \n", "\n", "
OLS Regression Results
Dep. Variable: y R-squared: 0.288
Model: OLS Adj. R-squared: 0.281
Method: Least Squares F-statistic: 39.99
Date: Wed, 12 Sep 2018 Prob (F-statistic): 7.41e-09
Time: 18:12:48 Log-Likelihood: -144.24
No. Observations: 101 AIC: 292.5
Df Residuals: 99 BIC: 297.7
Df Model: 1
Covariance Type: nonrobust
\n", "\n", "\n", " \n", "\n", "\n", " \n", "\n", "\n", " \n", "\n", "
coef std err t P>|t| [0.025 0.975]
Intercept 2.2331 0.182 12.271 0.000 1.872 2.594
x2 3.7216 0.589 6.324 0.000 2.554 4.889
\n", "\n", "\n", " \n", "\n", "\n", " \n", "\n", "\n", " \n", "\n", "\n", " \n", "\n", "
Omnibus: 1.271 Durbin-Watson: 2.101
Prob(Omnibus): 0.530 Jarque-Bera (JB): 1.094
Skew: -0.255 Prob(JB): 0.579
Kurtosis: 2.973 Cond. No. 6.20


Warnings:
[1] Standard Errors assume that the covariance matrix of the errors is correctly specified." ], "text/plain": [ "\n", "\"\"\"\n", " OLS Regression Results \n", "==============================================================================\n", "Dep. Variable: y R-squared: 0.288\n", "Model: OLS Adj. R-squared: 0.281\n", "Method: Least Squares F-statistic: 39.99\n", "Date: Wed, 12 Sep 2018 Prob (F-statistic): 7.41e-09\n", "Time: 18:12:48 Log-Likelihood: -144.24\n", "No. Observations: 101 AIC: 292.5\n", "Df Residuals: 99 BIC: 297.7\n", "Df Model: 1 \n", "Covariance Type: nonrobust \n", "==============================================================================\n", " coef std err t P>|t| [0.025 0.975]\n", "------------------------------------------------------------------------------\n", "Intercept 2.2331 0.182 12.271 0.000 1.872 2.594\n", "x2 3.7216 0.589 6.324 0.000 2.554 4.889\n", "==============================================================================\n", "Omnibus: 1.271 Durbin-Watson: 2.101\n", "Prob(Omnibus): 0.530 Jarque-Bera (JB): 1.094\n", "Skew: -0.255 Prob(JB): 0.579\n", "Kurtosis: 2.973 Cond. No. 6.20\n", "==============================================================================\n", "\n", "Warnings:\n", "[1] Standard Errors assume that the covariance matrix of the errors is correctly specified.\n", "\"\"\"" ] }, "execution_count": 343, "metadata": {}, "output_type": "execute_result" } ], "source": [ "model_e.summary()" ] }, { "cell_type": "markdown", "metadata": {}, "source": [ "**Comments:** \n", "\n", "- The addition of this anomaly has increased model c's p-value for $\\hat{β_1}$, suggesting even more strongly the false acceptance of the null hpyothesis.\n", "- When considered seperately the estimates for $\\hat{β_1}$ and $\\hat{β_2}$ are worsened by introducing this anomaly.\n" ] }, { "cell_type": "code", "execution_count": 359, "metadata": {}, "outputs": [ { "data": { "image/png": "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\n", "text/plain": [ "
" ] }, "metadata": {}, "output_type": "display_data" } ], "source": [ "# Model C diagnostics\n", "xc = pd.concat([df2['x1'], df2['x2']], axis=1)\n", "y_pred_c = model_c.predict(xc)\n", "lm_plot(lm_stats(sm.add_constant(xc), df2['y'], y_pred_d))" ] }, { "cell_type": "code", "execution_count": 357, "metadata": {}, "outputs": [ { "data": { "image/png": "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\n", "text/plain": [ "
" ] }, "metadata": {}, "output_type": "display_data" } ], "source": [ "# Model D diagnostics\n", "y_pred_d = model_d.predict(df2['x1'])\n", "lm_plot(lm_stats(sm.add_constant(df2['x1']), df2['y'], y_pred_d))" ] }, { "cell_type": "code", "execution_count": 358, "metadata": {}, "outputs": [ { "data": { "image/png": "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\n", "text/plain": [ "
" ] }, "metadata": {}, "output_type": "display_data" } ], "source": [ "# Model E diagnostics\n", "y_pred_e = model_e.predict(df2['x2'])\n", "lm_plot(lm_stats(sm.add_constant(df2['x2']), df2['y'], y_pred_e))" ] }, { "cell_type": "markdown", "metadata": {}, "source": [ "- model_c : the anomaly is almost an outlier, it is very high leverage\n", "- model_d : the anomaly is an outlier but not particularly high leverage\n", "- model_e : the anomaly is not an outlier but is fairly high leverage\n", "\n", "The anomaly has a detrimental effect on model_C, as a high levarage outlier. It has a less detrimental effect on model d and e where it is either low leverage or not an outlier.\n", "\n", "This suggests that models with collinear features are less robust in context of anomalous observations." ] }, { "cell_type": "markdown", "metadata": {}, "source": [ "## 15. This problem involves the Boston data set, which we saw in the lab for this chapter. We will now try to predict per capita crime rate using the other variables in this data set. In other words, per capita crime rate is the response, and the other variables are the predictors.\n", "\n", "### (a) For each predictor, fit a simple linear regression model to predict the response. Describe your results. In which of the models is there a statistically significant association between the predictor and the response? Create some plots to back up your assertions." ] }, { "cell_type": "code", "execution_count": 505, "metadata": {}, "outputs": [ { "data": { "text/html": [ "
\n", "\n", "\n", " \n", " \n", " \n", " \n", " \n", " \n", " \n", " \n", " \n", " \n", " \n", " \n", " \n", " \n", " \n", " \n", " \n", " \n", " \n", " \n", " \n", " \n", " \n", " \n", " \n", " \n", " \n", " \n", " \n", " \n", " \n", "
coefstd_errtP>|t|[0.0250.975]
Intercept4.42920.417110.61980.03.60985.2486
ZN-0.07350.0161-4.56960.0-0.1051-0.0419
\n", "
" ], "text/plain": [ " coef std_err t P>|t| [0.025 0.975]\n", "Intercept 4.4292 0.4171 10.6198 0.0 3.6098 5.2486\n", "ZN -0.0735 0.0161 -4.5696 0.0 -0.1051 -0.0419" ] }, "metadata": {}, "output_type": "display_data" }, { "data": { "text/html": [ "
\n", "\n", "\n", " \n", " \n", " \n", " \n", " \n", " \n", " \n", " \n", " \n", " \n", " \n", " \n", " \n", " \n", " \n", " \n", " \n", " \n", " \n", " \n", " \n", " \n", " \n", " \n", " \n", " \n", " \n", " \n", " \n", " \n", " \n", "
coefstd_errtP>|t|[0.0250.975]
Intercept-2.05090.6675-3.07230.0022-3.3624-0.7394
INDUS0.50680.05109.92870.00000.40660.6071
\n", "
" ], "text/plain": [ " coef std_err t P>|t| [0.025 0.975]\n", "Intercept -2.0509 0.6675 -3.0723 0.0022 -3.3624 -0.7394\n", "INDUS 0.5068 0.0510 9.9287 0.0000 0.4066 0.6071" ] }, "metadata": {}, "output_type": "display_data" }, { "data": { "text/html": [ "
\n", "\n", "\n", " \n", " \n", " \n", " \n", " \n", " \n", " \n", " \n", " \n", " \n", " \n", " \n", " \n", " \n", " \n", " \n", " \n", " \n", " \n", " \n", " \n", " \n", " \n", " \n", " \n", " \n", " \n", " \n", " \n", " \n", " \n", "
coefstd_errtP>|t|[0.0250.975]
Intercept3.72320.39599.40430.00002.94544.501
CHAS-1.87151.5053-1.24330.2143-4.82901.086
\n", "
" ], "text/plain": [ " coef std_err t P>|t| [0.025 0.975]\n", "Intercept 3.7232 0.3959 9.4043 0.0000 2.9454 4.501\n", "CHAS -1.8715 1.5053 -1.2433 0.2143 -4.8290 1.086" ] }, "metadata": {}, "output_type": "display_data" }, { "data": { "text/html": [ "
\n", "\n", "\n", " \n", " \n", " \n", " \n", " \n", " \n", " \n", " \n", " \n", " \n", " \n", " \n", " \n", " \n", " \n", " \n", " \n", " \n", " \n", " \n", " \n", " \n", " \n", " \n", " \n", " \n", " \n", " \n", " \n", " \n", " \n", "
coefstd_errtP>|t|[0.0250.975]
Intercept-13.58811.7015-7.98580.0-16.9310-10.2451
NOX30.97533.002810.31550.025.075736.8748
\n", "
" ], "text/plain": [ " coef std_err t P>|t| [0.025 0.975]\n", "Intercept -13.5881 1.7015 -7.9858 0.0 -16.9310 -10.2451\n", "NOX 30.9753 3.0028 10.3155 0.0 25.0757 36.8748" ] }, "metadata": {}, "output_type": "display_data" }, { "data": { "text/html": [ "
\n", "\n", "\n", " \n", " \n", " \n", " \n", " \n", " \n", " \n", " \n", " \n", " \n", " \n", " \n", " \n", " \n", " \n", " \n", " \n", " \n", " \n", " \n", " \n", " \n", " \n", " \n", " \n", " \n", " \n", " \n", " \n", " \n", " \n", "
coefstd_errtP>|t|[0.0250.975]
Intercept20.5063.36216.09920.013.900627.1114
RM-2.6910.5317-5.06160.0-3.7356-1.6465
\n", "
" ], "text/plain": [ " coef std_err t P>|t| [0.025 0.975]\n", "Intercept 20.506 3.3621 6.0992 0.0 13.9006 27.1114\n", "RM -2.691 0.5317 -5.0616 0.0 -3.7356 -1.6465" ] }, "metadata": {}, "output_type": "display_data" }, { "data": { "text/html": [ "
\n", "\n", "\n", " \n", " \n", " \n", " \n", " \n", " \n", " \n", " \n", " \n", " \n", " \n", " \n", " \n", " \n", " \n", " \n", " \n", " \n", " \n", " \n", " \n", " \n", " \n", " \n", " \n", " \n", " \n", " \n", " \n", " \n", " \n", "
coefstd_errtP>|t|[0.0250.975]
Intercept-3.75270.9442-3.97450.0001-5.6078-1.8977
AGE0.10710.01278.40940.00000.08210.1322
\n", "
" ], "text/plain": [ " coef std_err t P>|t| [0.025 0.975]\n", "Intercept -3.7527 0.9442 -3.9745 0.0001 -5.6078 -1.8977\n", "AGE 0.1071 0.0127 8.4094 0.0000 0.0821 0.1322" ] }, "metadata": {}, "output_type": "display_data" }, { "data": { "text/html": [ "
\n", "\n", "\n", " \n", " \n", " \n", " \n", " \n", " \n", " \n", " \n", " \n", " \n", " \n", " \n", " \n", " \n", " \n", " \n", " \n", " \n", " \n", " \n", " \n", " \n", " \n", " \n", " \n", " \n", " \n", " \n", " \n", " \n", " \n", "
coefstd_errtP>|t|[0.0250.975]
Intercept9.44890.730612.93370.08.013510.8842
DIS-1.54280.1684-9.16340.0-1.8736-1.2120
\n", "
" ], "text/plain": [ " coef std_err t P>|t| [0.025 0.975]\n", "Intercept 9.4489 0.7306 12.9337 0.0 8.0135 10.8842\n", "DIS -1.5428 0.1684 -9.1634 0.0 -1.8736 -1.2120" ] }, "metadata": {}, "output_type": "display_data" }, { "data": { "text/html": [ "
\n", "\n", "\n", " \n", " \n", " \n", " \n", " \n", " \n", " \n", " \n", " \n", " \n", " \n", " \n", " \n", " \n", " \n", " \n", " \n", " \n", " \n", " \n", " \n", " \n", " \n", " \n", " \n", " \n", " \n", " \n", " \n", " \n", " \n", "
coefstd_errtP>|t|[0.0250.975]
Intercept-2.27090.4448-5.10530.0-3.1448-1.3970
RAD0.61410.034417.83480.00.54650.6818
\n", "
" ], "text/plain": [ " coef std_err t P>|t| [0.025 0.975]\n", "Intercept -2.2709 0.4448 -5.1053 0.0 -3.1448 -1.3970\n", "RAD 0.6141 0.0344 17.8348 0.0 0.5465 0.6818" ] }, "metadata": {}, "output_type": "display_data" }, { "data": { "text/html": [ "
\n", "\n", "\n", " \n", " \n", " \n", " \n", " \n", " \n", " \n", " \n", " \n", " \n", " \n", " \n", " \n", " \n", " \n", " \n", " \n", " \n", " \n", " \n", " \n", " \n", " \n", " \n", " \n", " \n", " \n", " \n", " \n", " \n", " \n", "
coefstd_errtP>|t|[0.0250.975]
Intercept-8.47480.8177-10.36480.0-10.0812-6.8683
TAX0.02960.001915.96610.00.02590.0332
\n", "
" ], "text/plain": [ " coef std_err t P>|t| [0.025 0.975]\n", "Intercept -8.4748 0.8177 -10.3648 0.0 -10.0812 -6.8683\n", "TAX 0.0296 0.0019 15.9661 0.0 0.0259 0.0332" ] }, "metadata": {}, "output_type": "display_data" }, { "data": { "text/html": [ "
\n", "\n", "\n", " \n", " \n", " \n", " \n", " \n", " \n", " \n", " \n", " \n", " \n", " \n", " \n", " \n", " \n", " \n", " \n", " \n", " \n", " \n", " \n", " \n", " \n", " \n", " \n", " \n", " \n", " \n", " \n", " \n", " \n", " \n", "
coefstd_errtP>|t|[0.0250.975]
Intercept-17.53073.1472-5.57020.0-23.7139-11.3474
PTRATIO1.14460.16946.75800.00.81191.4774
\n", "
" ], "text/plain": [ " coef std_err t P>|t| [0.025 0.975]\n", "Intercept -17.5307 3.1472 -5.5702 0.0 -23.7139 -11.3474\n", "PTRATIO 1.1446 0.1694 6.7580 0.0 0.8119 1.4774" ] }, "metadata": {}, "output_type": "display_data" }, { "data": { "text/html": [ "
\n", "\n", "\n", " \n", " \n", " \n", " \n", " \n", " \n", " \n", " \n", " \n", " \n", " \n", " \n", " \n", " \n", " \n", " \n", " \n", " \n", " \n", " \n", " \n", " \n", " \n", " \n", " \n", " \n", " \n", " \n", " \n", " \n", " \n", "
coefstd_errtP>|t|[0.0250.975]
Intercept16.26801.430011.37610.013.458519.0775
B-0.03550.0039-9.14820.0-0.0432-0.0279
\n", "
" ], "text/plain": [ " coef std_err t P>|t| [0.025 0.975]\n", "Intercept 16.2680 1.4300 11.3761 0.0 13.4585 19.0775\n", "B -0.0355 0.0039 -9.1482 0.0 -0.0432 -0.0279" ] }, "metadata": {}, "output_type": "display_data" }, { "data": { "text/html": [ "
\n", "\n", "\n", " \n", " \n", " \n", " \n", " \n", " \n", " \n", " \n", " \n", " \n", " \n", " \n", " \n", " \n", " \n", " \n", " \n", " \n", " \n", " \n", " \n", " \n", " \n", " \n", " \n", " \n", " \n", " \n", " \n", " \n", " \n", "
coefstd_errtP>|t|[0.0250.975]
Intercept-3.29460.6947-4.74240.0-4.6596-1.9297
LSTAT0.54440.047811.38270.00.45040.6384
\n", "
" ], "text/plain": [ " coef std_err t P>|t| [0.025 0.975]\n", "Intercept -3.2946 0.6947 -4.7424 0.0 -4.6596 -1.9297\n", "LSTAT 0.5444 0.0478 11.3827 0.0 0.4504 0.6384" ] }, "metadata": {}, "output_type": "display_data" } ], "source": [ "boston = datasets.load_boston()\n", "boston_df = pd.DataFrame(boston.data, columns=boston.feature_names)\n", "\n", "# Check for missing values\n", "assert boston_df.isnull().sum().sum() == 0\n", "\n", "models_a = [smf.ols(formula='CRIM ~ {}'.format(f), data=boston_df).fit()\n", " for f in boston_df.columns[1:]]\n", "\n", "for model in models_a:\n", " dp = 4\n", " display(pd.DataFrame({'coef': np.around(model.params, dp),\n", " 'std_err': np.around(model.bse, dp),\n", " 't': np.around(model.params / model.bse, dp),\n", " 'P>|t|': np.around(model.pvalues, dp),\n", " '[0.025': np.around(model.conf_int(alpha=0.05)[0], dp),\n", " '0.975]': np.around(model.conf_int(alpha=0.05)[1], dp)}))" ] }, { "cell_type": "code", "execution_count": 506, "metadata": {}, "outputs": [ { "data": { "text/html": [ "

p < 0.05

" ], "text/plain": [ "" ] }, "metadata": {}, "output_type": "display_data" }, { "name": "stdout", "output_type": "stream", "text": [ "ZN\n", "INDUS\n", "NOX\n", "RM\n", "AGE\n", "DIS\n", "RAD\n", "TAX\n", "PTRATIO\n", "B\n", "LSTAT\n" ] }, { "data": { "text/html": [ "

p > 0.05

" ], "text/plain": [ "" ] }, "metadata": {}, "output_type": "display_data" }, { "name": "stdout", "output_type": "stream", "text": [ "CHAS\n" ] } ], "source": [ "# In which of the models is there a statistically significant association between the predictor and the response?\n", "\n", "display(HTML('

p < 0.05

'))\n", "for model in models_a:\n", " if model.pvalues[1] < 0.05:\n", " print(model.params[1:].index[0])\n", "\n", "display(HTML('

p > 0.05

'))\n", "for model in models_a:\n", " if model.pvalues[1] > 0.05:\n", " print(model.params[1:].index[0])\n" ] }, { "cell_type": "code", "execution_count": 490, "metadata": {}, "outputs": [ { "data": { "image/png": "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\n", "text/plain": [ "
" ] }, "metadata": {}, "output_type": "display_data" }, { "data": { "image/png": "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\n", "text/plain": [ "
" ] }, "metadata": {}, "output_type": "display_data" }, { "data": { "image/png": "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\n", "text/plain": [ "
" ] }, "metadata": {}, "output_type": "display_data" } ], "source": [ "#Create some plots to back up your assertions.\n", "def plot_grid(df, response, cols):\n", " \"\"\" Plot all predictor variables against response in grid \"\"\"\n", " variables = df.columns.drop(response)\n", " for i in range(0, len(variables), cols):\n", " g = sns.PairGrid(df, y_vars=[response], x_vars=variables[i:i+cols], height=4)\n", " g.map(sns.regplot);\n", " return\n", "\n", "plot_grid(boston_df, 'CRIM', 4)" ] }, { "cell_type": "markdown", "metadata": {}, "source": [ "### (b) Fit a multiple regression model to predict the response using all of the predictors. Describe your results. For which predictors can we reject the null hypothesis H0 : βj = 0?" ] }, { "cell_type": "code", "execution_count": 504, "metadata": {}, "outputs": [ { "data": { "text/html": [ "\n", "\n", "\n", " \n", "\n", "\n", " \n", "\n", "\n", " \n", "\n", "\n", " \n", "\n", "\n", " \n", "\n", "\n", " \n", "\n", "\n", " \n", "\n", "\n", " \n", "\n", "\n", " \n", "\n", "
OLS Regression Results
Dep. Variable: CRIM R-squared: 0.436
Model: OLS Adj. R-squared: 0.422
Method: Least Squares F-statistic: 31.77
Date: Thu, 13 Sep 2018 Prob (F-statistic): 6.16e-54
Time: 11:27:26 Log-Likelihood: -1661.2
No. Observations: 506 AIC: 3348.
Df Residuals: 493 BIC: 3403.
Df Model: 12
Covariance Type: nonrobust
\n", "\n", "\n", " \n", "\n", "\n", " \n", "\n", "\n", " \n", "\n", "\n", " \n", "\n", "\n", " \n", "\n", "\n", " \n", "\n", "\n", " \n", "\n", "\n", " \n", "\n", "\n", " \n", "\n", "\n", " \n", "\n", "\n", " \n", "\n", "\n", " \n", "\n", "\n", " \n", "\n", "\n", " \n", "\n", "
coef std err t P>|t| [0.025 0.975]
Intercept 10.3701 7.012 1.479 0.140 -3.408 24.148
ZN 0.0365 0.019 1.936 0.053 -0.001 0.073
INDUS -0.0672 0.085 -0.794 0.428 -0.233 0.099
CHAS -1.3049 1.185 -1.101 0.271 -3.633 1.023
NOX -7.2552 5.250 -1.382 0.168 -17.570 3.060
RM -0.3851 0.575 -0.670 0.503 -1.515 0.745
AGE 0.0019 0.018 0.105 0.917 -0.034 0.038
DIS -0.7163 0.273 -2.626 0.009 -1.252 -0.180
RAD 0.5395 0.088 6.128 0.000 0.366 0.712
TAX -0.0013 0.005 -0.254 0.799 -0.011 0.009
PTRATIO -0.0907 0.180 -0.504 0.615 -0.445 0.263
B -0.0089 0.004 -2.428 0.016 -0.016 -0.002
LSTAT 0.2309 0.069 3.346 0.001 0.095 0.366
\n", "\n", "\n", " \n", "\n", "\n", " \n", "\n", "\n", " \n", "\n", "\n", " \n", "\n", "
Omnibus: 680.813 Durbin-Watson: 1.507
Prob(Omnibus): 0.000 Jarque-Bera (JB): 94712.935
Skew: 6.846 Prob(JB): 0.00
Kurtosis: 68.611 Cond. No. 1.51e+04


Warnings:
[1] Standard Errors assume that the covariance matrix of the errors is correctly specified.
[2] The condition number is large, 1.51e+04. This might indicate that there are
strong multicollinearity or other numerical problems." ], "text/plain": [ "\n", "\"\"\"\n", " OLS Regression Results \n", "==============================================================================\n", "Dep. Variable: CRIM R-squared: 0.436\n", "Model: OLS Adj. R-squared: 0.422\n", "Method: Least Squares F-statistic: 31.77\n", "Date: Thu, 13 Sep 2018 Prob (F-statistic): 6.16e-54\n", "Time: 11:27:26 Log-Likelihood: -1661.2\n", "No. Observations: 506 AIC: 3348.\n", "Df Residuals: 493 BIC: 3403.\n", "Df Model: 12 \n", "Covariance Type: nonrobust \n", "==============================================================================\n", " coef std err t P>|t| [0.025 0.975]\n", "------------------------------------------------------------------------------\n", "Intercept 10.3701 7.012 1.479 0.140 -3.408 24.148\n", "ZN 0.0365 0.019 1.936 0.053 -0.001 0.073\n", "INDUS -0.0672 0.085 -0.794 0.428 -0.233 0.099\n", "CHAS -1.3049 1.185 -1.101 0.271 -3.633 1.023\n", "NOX -7.2552 5.250 -1.382 0.168 -17.570 3.060\n", "RM -0.3851 0.575 -0.670 0.503 -1.515 0.745\n", "AGE 0.0019 0.018 0.105 0.917 -0.034 0.038\n", "DIS -0.7163 0.273 -2.626 0.009 -1.252 -0.180\n", "RAD 0.5395 0.088 6.128 0.000 0.366 0.712\n", "TAX -0.0013 0.005 -0.254 0.799 -0.011 0.009\n", "PTRATIO -0.0907 0.180 -0.504 0.615 -0.445 0.263\n", "B -0.0089 0.004 -2.428 0.016 -0.016 -0.002\n", "LSTAT 0.2309 0.069 3.346 0.001 0.095 0.366\n", "==============================================================================\n", "Omnibus: 680.813 Durbin-Watson: 1.507\n", "Prob(Omnibus): 0.000 Jarque-Bera (JB): 94712.935\n", "Skew: 6.846 Prob(JB): 0.00\n", "Kurtosis: 68.611 Cond. No. 1.51e+04\n", "==============================================================================\n", "\n", "Warnings:\n", "[1] Standard Errors assume that the covariance matrix of the errors is correctly specified.\n", "[2] The condition number is large, 1.51e+04. This might indicate that there are\n", "strong multicollinearity or other numerical problems.\n", "\"\"\"" ] }, "execution_count": 504, "metadata": {}, "output_type": "execute_result" } ], "source": [ "response = 'CRIM'\n", "predictors = boston_df.columns.drop(response)\n", "f = '{} ~ {}'.format(response, '+'.join(predictors))\n", "\n", "model_b = smf.ols(formula=f, data=boston_df).fit()\n", "model_b.summary()" ] }, { "cell_type": "code", "execution_count": 508, "metadata": {}, "outputs": [ { "data": { "text/html": [ "

p < 0.05

" ], "text/plain": [ "" ] }, "metadata": {}, "output_type": "display_data" }, { "data": { "text/plain": [ "DIS 8.903325e-03\n", "RAD 1.825980e-09\n", "B 1.554695e-02\n", "LSTAT 8.837421e-04\n", "dtype: float64" ] }, "execution_count": 508, "metadata": {}, "output_type": "execute_result" } ], "source": [ "# For which predictors can we reject the null hypothesis H0 : βj = 0?¶\n", "\n", "display(HTML('

p < 0.05

'))\n", "model_b.pvalues[model_b.pvalues < 0.05]" ] }, { "cell_type": "markdown", "metadata": {}, "source": [ "### (c) How do your results from (a) compare to your results from (b)? Create a plot displaying the univariate regression coefficients from (a) on the x-axis, and the multiple regression coefficients from (b) on the y-axis. That is, each predictor is displayed as a single point in the plot. Its coefficient in a simple linear regression model is shown on the x-axis, and its coefficient estimate in the multiple linear regression model is shown on the y-axis.\n", "\n", "Multivariate regression found 4 of 12 predictors to be significnat where univariate regression found 11 of 12 significant. Multivariate regression seems to find significanlty less predictors to be significant.\n" ] }, { "cell_type": "code", "execution_count": 531, "metadata": {}, "outputs": [ { "data": { "text/html": [ "
\n", "\n", "\n", " \n", " \n", " \n", " \n", " \n", " \n", " \n", " \n", " \n", " \n", " \n", " \n", " \n", " \n", " \n", " \n", " \n", " \n", " \n", " \n", " \n", " \n", " \n", " \n", " \n", " \n", " \n", " \n", " \n", " \n", " \n", " \n", " \n", " \n", " \n", " \n", " \n", " \n", " \n", " \n", " \n", " \n", " \n", " \n", " \n", " \n", " \n", " \n", " \n", " \n", " \n", " \n", " \n", " \n", " \n", " \n", " \n", " \n", " \n", " \n", " \n", " \n", " \n", " \n", " \n", " \n", " \n", " \n", " \n", "
Univariate_coefMultivariate_coef
ZN-0.0735210.036455
INDUS0.506847-0.067166
CHAS-1.871545-1.304904
NOX30.975259-7.255168
RM-2.691045-0.385127
AGE0.1071310.001902
DIS-1.542831-0.716287
RAD0.6141370.539464
TAX0.029563-0.001318
PTRATIO1.144613-0.090721
B-0.035535-0.008917
LSTAT0.5444060.230881
\n", "
" ], "text/plain": [ " Univariate_coef Multivariate_coef\n", "ZN -0.073521 0.036455\n", "INDUS 0.506847 -0.067166\n", "CHAS -1.871545 -1.304904\n", "NOX 30.975259 -7.255168\n", "RM -2.691045 -0.385127\n", "AGE 0.107131 0.001902\n", "DIS -1.542831 -0.716287\n", "RAD 0.614137 0.539464\n", "TAX 0.029563 -0.001318\n", "PTRATIO 1.144613 -0.090721\n", "B -0.035535 -0.008917\n", "LSTAT 0.544406 0.230881" ] }, "metadata": {}, "output_type": "display_data" }, { "data": { "image/png": "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\n", "text/plain": [ "
" ] }, "metadata": {}, "output_type": "display_data" } ], "source": [ "# Get coefficients\n", "univariate_params = pd.concat([m.params[1:] for m in models_a])\n", "multivariate_params = model_b.params[1:]\n", "df = pd.DataFrame({'Univariate_coef': univariate_params, \n", " 'Multivariate_coef': multivariate_params,})\n", "display(df)\n", "\n", "plt.figure(figsize=(20,10))\n", "ax = sns.scatterplot(x='Univariate_coef', y='Multivariate_coef', data=df);\n", " " ] }, { "cell_type": "markdown", "metadata": {}, "source": [ "### (d) Is there evidence of non-linear association between any of the predictors and the response? To answer this question, for each predictor X, fit a model of the form\n", "\n", "$Y = β_0 + β_1 X + β_2 X^2 + β_3 X^3 + ε$" ] }, { "cell_type": "code", "execution_count": 542, "metadata": {}, "outputs": [ { "data": { "text/html": [ "
\n", "\n", "\n", " \n", " \n", " \n", " \n", " \n", " \n", " \n", " \n", " \n", " \n", " \n", " \n", " \n", " \n", " \n", " \n", " \n", " \n", " \n", " \n", " \n", " \n", " \n", " \n", " \n", " \n", " \n", " \n", " \n", " \n", " \n", " \n", " \n", " \n", " \n", " \n", " \n", " \n", " \n", " \n", " \n", " \n", " \n", " \n", " \n", " \n", " \n", " \n", " \n", "
coefstd_errtP>|t|[0.0250.975]
Intercept4.81930.432911.13310.00003.96885.6698
ZN-0.33030.1098-3.00830.0028-0.5459-0.1146
np.power(ZN, 2)0.00640.00391.66960.0956-0.00110.0140
np.power(ZN, 3)-0.00000.0000-1.19610.2322-0.00010.0000
\n", "
" ], "text/plain": [ " coef std_err t P>|t| [0.025 0.975]\n", "Intercept 4.8193 0.4329 11.1331 0.0000 3.9688 5.6698\n", "ZN -0.3303 0.1098 -3.0083 0.0028 -0.5459 -0.1146\n", "np.power(ZN, 2) 0.0064 0.0039 1.6696 0.0956 -0.0011 0.0140\n", "np.power(ZN, 3) -0.0000 0.0000 -1.1961 0.2322 -0.0001 0.0000" ] }, "metadata": {}, "output_type": "display_data" }, { "data": { "text/html": [ "
\n", "\n", "\n", " \n", " \n", " \n", " \n", " \n", " \n", " \n", " \n", " \n", " \n", " \n", " \n", " \n", " \n", " \n", " \n", " \n", " \n", " \n", " \n", " \n", " \n", " \n", " \n", " \n", " \n", " \n", " \n", " \n", " \n", " \n", " \n", " \n", " \n", " \n", " \n", " \n", " \n", " \n", " \n", " \n", " \n", " \n", " \n", " \n", " \n", " \n", " \n", " \n", "
coefstd_errtP>|t|[0.0250.975]
Intercept3.64101.57602.31030.02130.54466.7374
INDUS-1.95330.4826-4.04740.0001-2.9015-1.0051
np.power(INDUS, 2)0.25040.03946.36050.00000.17310.3278
np.power(INDUS, 3)-0.00690.0010-7.23920.0000-0.0088-0.0051
\n", "
" ], "text/plain": [ " coef std_err t P>|t| [0.025 0.975]\n", "Intercept 3.6410 1.5760 2.3103 0.0213 0.5446 6.7374\n", "INDUS -1.9533 0.4826 -4.0474 0.0001 -2.9015 -1.0051\n", "np.power(INDUS, 2) 0.2504 0.0394 6.3605 0.0000 0.1731 0.3278\n", "np.power(INDUS, 3) -0.0069 0.0010 -7.2392 0.0000 -0.0088 -0.0051" ] }, "metadata": {}, "output_type": "display_data" }, { "data": { "text/html": [ "
\n", "\n", "\n", " \n", " \n", " \n", " \n", " \n", " \n", " \n", " \n", " \n", " \n", " \n", " \n", " \n", " \n", " \n", " \n", " \n", " \n", " \n", " \n", " \n", " \n", " \n", " \n", " \n", " \n", " \n", " \n", " \n", " \n", " \n", " \n", " \n", " \n", " \n", " \n", " \n", " \n", " \n", " \n", " \n", " \n", " \n", " \n", " \n", " \n", " \n", " \n", " \n", "
coefstd_errtP>|t|[0.0250.975]
Intercept3.72320.39599.40430.00002.94544.501
CHAS-0.62380.5018-1.24330.2143-1.60970.362
np.power(CHAS, 2)-0.62380.5018-1.24330.2143-1.60970.362
np.power(CHAS, 3)-0.62380.5018-1.24330.2143-1.60970.362
\n", "
" ], "text/plain": [ " coef std_err t P>|t| [0.025 0.975]\n", "Intercept 3.7232 0.3959 9.4043 0.0000 2.9454 4.501\n", "CHAS -0.6238 0.5018 -1.2433 0.2143 -1.6097 0.362\n", "np.power(CHAS, 2) -0.6238 0.5018 -1.2433 0.2143 -1.6097 0.362\n", "np.power(CHAS, 3) -0.6238 0.5018 -1.2433 0.2143 -1.6097 0.362" ] }, "metadata": {}, "output_type": "display_data" }, { "data": { "text/html": [ "
\n", "\n", "\n", " \n", " \n", " \n", " \n", " \n", " \n", " \n", " \n", " \n", " \n", " \n", " \n", " \n", " \n", " \n", " \n", " \n", " \n", " \n", " \n", " \n", " \n", " \n", " \n", " \n", " \n", " \n", " \n", " \n", " \n", " \n", " \n", " \n", " \n", " \n", " \n", " \n", " \n", " \n", " \n", " \n", " \n", " \n", " \n", " \n", " \n", " \n", " \n", " \n", "
coefstd_errtP>|t|[0.0250.975]
Intercept230.142133.73446.82220.0163.8641296.4201
NOX-1264.1021170.8598-7.39850.0-1599.7905-928.4136
np.power(NOX, 2)2223.2265280.65877.92150.01671.81602774.6370
np.power(NOX, 3)-1232.3894149.6867-8.23310.0-1526.4790-938.2999
\n", "
" ], "text/plain": [ " coef std_err t P>|t| [0.025 0.975]\n", "Intercept 230.1421 33.7344 6.8222 0.0 163.8641 296.4201\n", "NOX -1264.1021 170.8598 -7.3985 0.0 -1599.7905 -928.4136\n", "np.power(NOX, 2) 2223.2265 280.6587 7.9215 0.0 1671.8160 2774.6370\n", "np.power(NOX, 3) -1232.3894 149.6867 -8.2331 0.0 -1526.4790 -938.2999" ] }, "metadata": {}, "output_type": "display_data" }, { "data": { "text/html": [ "
\n", "\n", "\n", " \n", " \n", " \n", " \n", " \n", " \n", " \n", " \n", " \n", " \n", " \n", " \n", " \n", " \n", " \n", " \n", " \n", " \n", " \n", " \n", " \n", " \n", " \n", " \n", " \n", " \n", " \n", " \n", " \n", " \n", " \n", " \n", " \n", " \n", " \n", " \n", " \n", " \n", " \n", " \n", " \n", " \n", " \n", " \n", " \n", " \n", " \n", " \n", " \n", "
coefstd_errtP>|t|[0.0250.975]
Intercept111.900264.46001.73600.0832-14.7444238.5447
RM-38.704031.2837-1.23720.2166-100.167122.7591
np.power(RM, 2)4.46555.00540.89210.3727-5.368614.2997
np.power(RM, 3)-0.16940.2635-0.64280.5207-0.68710.3483
\n", "
" ], "text/plain": [ " coef std_err t P>|t| [0.025 0.975]\n", "Intercept 111.9002 64.4600 1.7360 0.0832 -14.7444 238.5447\n", "RM -38.7040 31.2837 -1.2372 0.2166 -100.1671 22.7591\n", "np.power(RM, 2) 4.4655 5.0054 0.8921 0.3727 -5.3686 14.2997\n", "np.power(RM, 3) -0.1694 0.2635 -0.6428 0.5207 -0.6871 0.3483" ] }, "metadata": {}, "output_type": "display_data" }, { "data": { "text/html": [ "
\n", "\n", "\n", " \n", " \n", " \n", " \n", " \n", " \n", " \n", " \n", " \n", " \n", " \n", " \n", " \n", " \n", " \n", " \n", " \n", " \n", " \n", " \n", " \n", " \n", " \n", " \n", " \n", " \n", " \n", " \n", " \n", " \n", " \n", " \n", " \n", " \n", " \n", " \n", " \n", " \n", " \n", " \n", " \n", " \n", " \n", " \n", " \n", " \n", " \n", " \n", " \n", "
coefstd_errtP>|t|[0.0250.975]
Intercept-2.55922.7706-0.92370.3561-8.00272.8843
AGE0.27430.18651.47070.1420-0.09210.6406
np.power(AGE, 2)-0.00720.0036-1.98740.0474-0.0144-0.0001
np.power(AGE, 3)0.00010.00002.71850.00680.00000.0001
\n", "
" ], "text/plain": [ " coef std_err t P>|t| [0.025 0.975]\n", "Intercept -2.5592 2.7706 -0.9237 0.3561 -8.0027 2.8843\n", "AGE 0.2743 0.1865 1.4707 0.1420 -0.0921 0.6406\n", "np.power(AGE, 2) -0.0072 0.0036 -1.9874 0.0474 -0.0144 -0.0001\n", "np.power(AGE, 3) 0.0001 0.0000 2.7185 0.0068 0.0000 0.0001" ] }, "metadata": {}, "output_type": "display_data" }, { "data": { "text/html": [ "
\n", "\n", "\n", " \n", " \n", " \n", " \n", " \n", " \n", " \n", " \n", " \n", " \n", " \n", " \n", " \n", " \n", " \n", " \n", " \n", " \n", " \n", " \n", " \n", " \n", " \n", " \n", " \n", " \n", " \n", " \n", " \n", " \n", " \n", " \n", " \n", " \n", " \n", " \n", " \n", " \n", " \n", " \n", " \n", " \n", " \n", " \n", " \n", " \n", " \n", " \n", " \n", "
coefstd_errtP>|t|[0.0250.975]
Intercept29.94962.447912.23500.025.140334.7589
DIS-15.51721.7374-8.93140.0-18.9306-12.1038
np.power(DIS, 2)2.44790.34677.06050.01.76673.1291
np.power(DIS, 3)-0.11850.0204-5.80200.0-0.1586-0.0783
\n", "
" ], "text/plain": [ " coef std_err t P>|t| [0.025 0.975]\n", "Intercept 29.9496 2.4479 12.2350 0.0 25.1403 34.7589\n", "DIS -15.5172 1.7374 -8.9314 0.0 -18.9306 -12.1038\n", "np.power(DIS, 2) 2.4479 0.3467 7.0605 0.0 1.7667 3.1291\n", "np.power(DIS, 3) -0.1185 0.0204 -5.8020 0.0 -0.1586 -0.0783" ] }, "metadata": {}, "output_type": "display_data" }, { "data": { "text/html": [ "
\n", "\n", "\n", " \n", " \n", " \n", " \n", " \n", " \n", " \n", " \n", " \n", " \n", " \n", " \n", " \n", " \n", " \n", " \n", " \n", " \n", " \n", " \n", " \n", " \n", " \n", " \n", " \n", " \n", " \n", " \n", " \n", " \n", " \n", " \n", " \n", " \n", " \n", " \n", " \n", " \n", " \n", " \n", " \n", " \n", " \n", " \n", " \n", " \n", " \n", " \n", " \n", "
coefstd_errtP>|t|[0.0250.975]
Intercept-0.60502.0565-0.29420.7687-4.64553.4355
RAD0.51221.04690.48930.6249-1.54462.5690
np.power(RAD, 2)-0.07500.1490-0.50360.6148-0.36780.2177
np.power(RAD, 3)0.00320.00460.69860.4851-0.00580.0122
\n", "
" ], "text/plain": [ " coef std_err t P>|t| [0.025 0.975]\n", "Intercept -0.6050 2.0565 -0.2942 0.7687 -4.6455 3.4355\n", "RAD 0.5122 1.0469 0.4893 0.6249 -1.5446 2.5690\n", "np.power(RAD, 2) -0.0750 0.1490 -0.5036 0.6148 -0.3678 0.2177\n", "np.power(RAD, 3) 0.0032 0.0046 0.6986 0.4851 -0.0058 0.0122" ] }, "metadata": {}, "output_type": "display_data" }, { "data": { "text/html": [ "
\n", "\n", "\n", " \n", " \n", " \n", " \n", " \n", " \n", " \n", " \n", " \n", " \n", " \n", " \n", " \n", " \n", " \n", " \n", " \n", " \n", " \n", " \n", " \n", " \n", " \n", " \n", " \n", " \n", " \n", " \n", " \n", " \n", " \n", " \n", " \n", " \n", " \n", " \n", " \n", " \n", " \n", " \n", " \n", " \n", " \n", " \n", " \n", " \n", " \n", " \n", " \n", "
coefstd_errtP>|t|[0.0250.975]
Intercept19.070511.82711.61240.1075-4.166242.3072
TAX-0.15240.0959-1.58890.1127-0.34090.0360
np.power(TAX, 2)0.00040.00021.47560.1407-0.00010.0008
np.power(TAX, 3)-0.00000.0000-1.15780.2475-0.00000.0000
\n", "
" ], "text/plain": [ " coef std_err t P>|t| [0.025 0.975]\n", "Intercept 19.0705 11.8271 1.6124 0.1075 -4.1662 42.3072\n", "TAX -0.1524 0.0959 -1.5889 0.1127 -0.3409 0.0360\n", "np.power(TAX, 2) 0.0004 0.0002 1.4756 0.1407 -0.0001 0.0008\n", "np.power(TAX, 3) -0.0000 0.0000 -1.1578 0.2475 -0.0000 0.0000" ] }, "metadata": {}, "output_type": "display_data" }, { "data": { "text/html": [ "
\n", "\n", "\n", " \n", " \n", " \n", " \n", " \n", " \n", " \n", " \n", " \n", " \n", " \n", " \n", " \n", " \n", " \n", " \n", " \n", " \n", " \n", " \n", " \n", " \n", " \n", " \n", " \n", " \n", " \n", " \n", " \n", " \n", " \n", " \n", " \n", " \n", " \n", " \n", " \n", " \n", " \n", " \n", " \n", " \n", " \n", " \n", " \n", " \n", " \n", " \n", " \n", "
coefstd_errtP>|t|[0.0250.975]
Intercept474.0255156.82293.02270.0026165.9154782.1355
PTRATIO-81.808927.6489-2.95890.0032-136.1306-27.4872
np.power(PTRATIO, 2)4.60391.60862.86210.00441.44357.7644
np.power(PTRATIO, 3)-0.08420.0309-2.72400.0067-0.1449-0.0235
\n", "
" ], "text/plain": [ " coef std_err t P>|t| [0.025 0.975]\n", "Intercept 474.0255 156.8229 3.0227 0.0026 165.9154 782.1355\n", "PTRATIO -81.8089 27.6489 -2.9589 0.0032 -136.1306 -27.4872\n", "np.power(PTRATIO, 2) 4.6039 1.6086 2.8621 0.0044 1.4435 7.7644\n", "np.power(PTRATIO, 3) -0.0842 0.0309 -2.7240 0.0067 -0.1449 -0.0235" ] }, "metadata": {}, "output_type": "display_data" }, { "data": { "text/html": [ "
\n", "\n", "\n", " \n", " \n", " \n", " \n", " \n", " \n", " \n", " \n", " \n", " \n", " \n", " \n", " \n", " \n", " \n", " \n", " \n", " \n", " \n", " \n", " \n", " \n", " \n", " \n", " \n", " \n", " \n", " \n", " \n", " \n", " \n", " \n", " \n", " \n", " \n", " \n", " \n", " \n", " \n", " \n", " \n", " \n", " \n", " \n", " \n", " \n", " \n", " \n", " \n", "
coefstd_errtP>|t|[0.0250.975]
Intercept17.98982.31167.78250.000013.448322.5314
B-0.08450.0565-1.49660.1351-0.19550.0264
np.power(B, 2)0.00020.00030.76020.4475-0.00040.0008
np.power(B, 3)-0.00000.0000-0.66140.5087-0.00000.0000
\n", "
" ], "text/plain": [ " coef std_err t P>|t| [0.025 0.975]\n", "Intercept 17.9898 2.3116 7.7825 0.0000 13.4483 22.5314\n", "B -0.0845 0.0565 -1.4966 0.1351 -0.1955 0.0264\n", "np.power(B, 2) 0.0002 0.0003 0.7602 0.4475 -0.0004 0.0008\n", "np.power(B, 3) -0.0000 0.0000 -0.6614 0.5087 -0.0000 0.0000" ] }, "metadata": {}, "output_type": "display_data" }, { "data": { "text/html": [ "
\n", "\n", "\n", " \n", " \n", " \n", " \n", " \n", " \n", " \n", " \n", " \n", " \n", " \n", " \n", " \n", " \n", " \n", " \n", " \n", " \n", " \n", " \n", " \n", " \n", " \n", " \n", " \n", " \n", " \n", " \n", " \n", " \n", " \n", " \n", " \n", " \n", " \n", " \n", " \n", " \n", " \n", " \n", " \n", " \n", " \n", " \n", " \n", " \n", " \n", " \n", " \n", "
coefstd_errtP>|t|[0.0250.975]
Intercept1.08362.03210.53330.5941-2.90885.0761
LSTAT-0.41330.4657-0.88740.3753-1.32820.5017
np.power(LSTAT, 2)0.05300.03021.75840.0793-0.00620.1123
np.power(LSTAT, 3)-0.00080.0006-1.42310.1553-0.00190.0003
\n", "
" ], "text/plain": [ " coef std_err t P>|t| [0.025 0.975]\n", "Intercept 1.0836 2.0321 0.5333 0.5941 -2.9088 5.0761\n", "LSTAT -0.4133 0.4657 -0.8874 0.3753 -1.3282 0.5017\n", "np.power(LSTAT, 2) 0.0530 0.0302 1.7584 0.0793 -0.0062 0.1123\n", "np.power(LSTAT, 3) -0.0008 0.0006 -1.4231 0.1553 -0.0019 0.0003" ] }, "metadata": {}, "output_type": "display_data" } ], "source": [ "models_d = [smf.ols(formula='CRIM ~ {0} + np.power({0}, 2) + np.power({0}, 3)'.format(f), data=boston_df).fit()\n", " for f in boston_df.columns[1:]]\n", "\n", "for model in models_d:\n", " dp = 4\n", " display(pd.DataFrame({'coef': np.around(model.params, dp),\n", " 'std_err': np.around(model.bse, dp),\n", " 't': np.around(model.params / model.bse, dp),\n", " 'P>|t|': np.around(model.pvalues, dp),\n", " '[0.025': np.around(model.conf_int(alpha=0.05)[0], dp),\n", " '0.975]': np.around(model.conf_int(alpha=0.05)[1], dp)}))" ] }, { "cell_type": "code", "execution_count": 561, "metadata": {}, "outputs": [ { "data": { "text/html": [ "

Features with p < 0.05

" ], "text/plain": [ "" ] }, "metadata": {}, "output_type": "display_data" }, { "data": { "text/html": [ "
\n", "\n", "\n", " \n", " \n", " \n", " \n", " \n", " \n", " \n", " \n", " \n", " \n", " \n", " \n", " \n", " \n", " \n", " \n", " \n", " \n", " \n", " \n", " \n", " \n", " \n", " \n", " \n", " \n", " \n", " \n", " \n", " \n", " \n", " \n", " \n", " \n", " \n", " \n", " \n", " \n", " \n", " \n", " \n", " \n", " \n", " \n", " \n", " \n", " \n", " \n", " \n", " \n", " \n", " \n", " \n", " \n", " \n", " \n", " \n", " \n", " \n", " \n", " \n", " \n", " \n", " \n", " \n", " \n", " \n", " \n", "
P>|t|
ZN2.759026e-03
INDUS5.996850e-05
np.power(INDUS, 2)4.530067e-10
np.power(INDUS, 3)1.704441e-12
NOX5.832574e-13
np.power(NOX, 2)1.522887e-14
np.power(NOX, 3)1.587778e-15
np.power(AGE, 2)4.742487e-02
np.power(AGE, 3)6.784650e-03
DIS7.973990e-18
np.power(DIS, 2)5.552306e-12
np.power(DIS, 3)1.161292e-08
PTRATIO3.233644e-03
np.power(PTRATIO, 2)4.384601e-03
np.power(PTRATIO, 3)6.674970e-03
\n", "
" ], "text/plain": [ " P>|t|\n", "ZN 2.759026e-03\n", "INDUS 5.996850e-05\n", "np.power(INDUS, 2) 4.530067e-10\n", "np.power(INDUS, 3) 1.704441e-12\n", "NOX 5.832574e-13\n", "np.power(NOX, 2) 1.522887e-14\n", "np.power(NOX, 3) 1.587778e-15\n", "np.power(AGE, 2) 4.742487e-02\n", "np.power(AGE, 3) 6.784650e-03\n", "DIS 7.973990e-18\n", "np.power(DIS, 2) 5.552306e-12\n", "np.power(DIS, 3) 1.161292e-08\n", "PTRATIO 3.233644e-03\n", "np.power(PTRATIO, 2) 4.384601e-03\n", "np.power(PTRATIO, 3) 6.674970e-03" ] }, "metadata": {}, "output_type": "display_data" } ], "source": [ "# In which of the models is there a statistically significant association between the predictor and the response?\n", "\n", "display(HTML('

Features with p < 0.05

'))\n", "\n", "sig = pd.concat([model.pvalues[model.pvalues < 0.05] for model in models_d])\n", "\n", "display(pd.DataFrame({'P>|t|':sig.drop('Intercept')}))\n" ] }, { "cell_type": "markdown", "metadata": {}, "source": [ "- There is evidence of a non-linear association between the repsonse and INDUS, NOX, AGE, DIS, PTRATIO\n", "- There is evidence of a linear association between the response and ZN" ] }, { "cell_type": "code", "execution_count": null, "metadata": {}, "outputs": [], "source": [] } ], "metadata": { "kernelspec": { "display_name": "Python 3", "language": "python", "name": "python3" }, "language_info": { "codemirror_mode": { "name": "ipython", "version": 3 }, "file_extension": ".py", "mimetype": "text/x-python", "name": "python", "nbconvert_exporter": "python", "pygments_lexer": "ipython3", "version": "3.6.5" } }, "nbformat": 4, "nbformat_minor": 2 }